Вы находитесь на странице: 1из 1220

JEE ADVANCED

2019

Comprehensive
Physics
JEE ADVANCED
2019

Comprehensive
Physics

McGraw Hill Education (India) Private Limited


CHENNAI

McGraw Hill Education Offices


Chennai New York St Louis San Francisco Auckland Bogotá Caracas
Kuala Lumpur Lisbon London Madrid Mexico City Milan Montreal
San 
Juan Santiago  Singapore Sydney Tokyo Toronto
McGraw Hill Education (India) Private Limited
Published by McGraw Hill Education (India) Private Limited,
444/1, Sri Ekambara Naicker Industrial Estate, Alapakkam, Porur,Chennai - 600 116, Tamil Nadu, India

Comprehensive Physics—JEE Advanced

Copyright © 2018 by McGraw Hill Education (India) Private Limited

No Part of this publication may be reproduced or distributed in any form or by any means, electronic, mechanical, photocopying,
recording, or otherwise or stored in a database or retrieval system without the prior written permission of the publishers. The
program listings (if any) may be entered, stored and executed in a computer system, but they may not be reproduced for publication.

This edition can be exported from India only by the publishers.


McGraw Hill Education (India) Private Limited

1 2 3 4 5 6 7 8 9 D102542 22 21 20 19 18

ISBN (13) : 978-93-87572-57-7


ISBN (10) : 93-87572-57-9

Information contained in this work has been obtained McGraw Hill Education (India), from sources believed to be reliable.
However, neither, McGraw Hill nor its authors guarantee the accuracy or completeness of any information published herein,
and neither McGraw Hill Education (India) nor its authors shall be responsible for any errors, omissions, or damages arising
out of use of this information. This work is published with the understanding that McGraw Hill Education (India) and its
authors are supplying information but are not attempting to render engineering or other professional services. If such services
are required, the assistance of an appropriate professional should be sought.

at Printers, 312 EPIP, HSIDC, Kundli, Sonepat, Haryana


Cover Design: Neeraj Dayal

Visit us at: www.mheducation.co.in


A Word to the Reader
Comprehensive Physics—JEE Advanced is highly useful for aspirants appearing for JEE Advanced. It is as per
the syllabus of JEE Advanced.
Divided into 29 core chapters, it covers the entire gamut of the subject through a combination of conceptual
theory supported by solved problems and practice exercises. Each chapter opens with a review of the basic concepts,
formulae, laws and definition that is linked to the concepts and problems of that chapter. There are plenty of fully
solved questions of all types as per the latest pattern and syllabus.
Questions in each chapter have been classified as follows:
Section I: Multiple Choice Questions with Only One correct choice.
Section II : Multiple Choice Questions with One or More correct choices.
Section III : Multiple Choice Questions based on passage.
Section IV : Matching of Column type questions.
Section V: Assertion-Reason type questions.

Key Features

New Feature

THE PUBLISHERS
Syllabus
General: Units and dimensions, dimensional analysis; least count, significant figures; Methods of measurement and
error analysis for physical quantities pertaining to the following experiments: Experiments based on using Vernier

specific resistance of the material of a wire using meter bridge and post office box.
Mechanics: -
tion; Relative velocity.

Law of gravitation; Gravitational potential and field; Acceleration due to gravity; Motion of planets and satellites
in circular orbits; Escape velocity.
Rigid body, moment of inertia, parallel and perpendicular axes theorems, moment of inertia of uniform bodies
with simple geometrical shapes; Angular momentum; Torque; Conservation of angular momentum; Dynamics of
rigid bodies with fixed axis of rotation; Rolling without slipping of rings, cylinders and spheres; Equilibrium of
rigid bodies; Collision of point masses with rigid bodies.
Linear and angular simple harmonic motions.

theorem and its applications.

(in sound).
Thermal physics: Thermal expansion of solids, liquids and gases; Calorimetry, latent heat; Heat conduction in one

heats (Cv and Cp for monoatomic and diatomic gases); Isothermal and adiabatic processes, bulk modulus of gases;

Electricity and magnetism:


of point charges and of electrical dipoles in a uniform electrostatic field; Electric field lines; Flux of electric field;

charged infinite plane sheet and uniformly charged thin spherical shell.

in a capacitor.

applications; Heating effect of current.


viii Syllabus

circular coil and inside a long straight solenoid; Force on a moving charge and on a current-carrying wire in a
uniform magnetic field.
Magnetic moment of a current loop; Effect of a uniform magnetic field on a current loop; Moving coil galva-
nometer, voltmeter, ammeter and their conversions.

d.c. and a.c. sources.


Optics:

Magnification.

Modern physics: Atomic nucleus; Alpha, beta and gamma radiations; Law of radioactive decay; Decay constant;

these processes.
Contents
A Word to the Reader v
Syllabus vii

1. Units and Dimensions 1.1–1.24


2. Motion in One Dimension 2.1–2.36
3. Vectors 3.1–3.13
4. Motion in Two Dimensions 4.1–4.34
5. Laws of Motion and Friction 5.1–5.61
6. Work, Energy and Power 6.1–6.39
7. Conservation of Linear Momentum and Collisions 7.1–7.38
8. Rigid Body Rotation 8.1–8.68
9. Gravitation 9.1–9.36
10. Elasticity 10.1–10.24
11. Hydrostatics (Fluid Pressure and Buoyancy) 11.1–11.32
12. Hydrodynamics (Bernoulli’s Theorem and Viscosity) 12.1–12.25
13. Simple Harmonic Motion 13.1–13.47
14. Waves and Doppler’s Effect 14.1–14.48
15. Thermal Expansion 15.1–15.14
16. Measurement of Heat 16.1–16.11
17. Thermodynamics (Isothermal and Adiabatic Processes) 17.1–17.41
18. Kinetic Theory of Gases 18.1–18.16
19. Transmission of Heat 19.1–19.23
20. Electrostatic Field and Potential 20.1–20.50
21. Capacitance and Capacitors 21.1–21.35
22. Electric Current and D.C. Circuits 22.1–22.50
23. Heating Effect of Current 23.1–23.23
24. Magnetic Effect of Current and Magnetism 24.1–24.52
25. Electromagnetic Induction and A.C. Circuits 25.1–25.54
26. Ray Optics and Optical Instruments 26.1–26.52
27. Wave Optics 27.1–27.34
28. Atomic Physics 28.1–28.43
29. Nuclear Physics 29.1–29.27
x Contents

Model Test Paper I MTPI.1–MTPI.12


Model Test Paper II MTPII.1–MTPII.12
IIT-JEE 2012 Paper–I IJI.1–IJI.8
IIT-JEE 2012 Paper–II IJII.1–IJII.10
JEE Advanced 2013 Paper–I JAI.1–JAI.8
JEE Advanced 2013 Paper–II JAII.1–JAII.9
JEE Advanced 2014: Paper–I JAI.1–JAI.11
JEE Advanced 2014: Paper–II JAII.1–JAII.10
JEE Advanced 2015: Paper–I (Model Solutions) JAI.1–JAI.11
JEE Advanced 2015: Paper–II (Model Solutions) JAII.1–JAII.12
   JEE Advanced 2016: Paper-I (Model Solutions) P-I.1–P-I.15
   JEE Advanced 2016: Paper-II (Model Solutions) P-II.1–P-II.19
   JEE Advanced 2017: Paper-I P-I.1–P-I.9
   JEE Advanced 2017: Paper-II P-II.1–P-II.11
1
Chapter
Units and Dimensions

REVIEW OF BASIC CONCEPTS Physical Quantity Name of the Unit Symbol


Mass kilogram kg
1.1 THE SI SYSTEM OF UNITS
Time second s
The internationally accepted standard units of the Electric current ampere A
fundamental physical quantities are given in Table 1.1. Temperature kelvin K
Luminous intensity candela cd
Table 1.1 Fundamental SI Units
Amount of substance mole mol
Physical Quantity Name of the Unit Symbol Angle in a plane radian rad
Length metre m Solid angle steradian sr

Table 1.2 Dimensional Formulae of some Physical Quantities

Physical Quantity Dimensional Formula Physical Quantity Dimensional Formula


0 2 0
Area ML T Heat energy ML2T–2
0 3 0
Volume ML T Entropy ML2T–2K–1
Density ML–3T 0 M0L2T –2K–1
Velocity M0LT –1 Latent heat M0L2T–2
0 –2
Acceleration M LT ML2T–2K–1 mol–1
Momentum MLT–1 Thermal conductivity MLT–3K–1
2 –1
Angular momentum ML T Wien’s constant M0LT0K
Force MLT–2 Stefan’s constant ML0T–3K–4
2 –2
Energy, work ML T Boltzmann’s constant ML2T–2K–1
Power ML2T–3 Molar gas constant ML2T–2K–1 mol–1
2 –2
Torque, couple ML T Electric charge TA
–1
Impulse MLT Electric current A
0 0 –1
Frequency MLT Electric potential ML2T–3 A–1
0 0 –1
Angular frequency MLT MLT–3 A–1
Angular acceleration M0L0T–2 Capacitance M –1L–2T4A2
–1 –2
Pressure ML T Inductance ML2T–2A–2
Elastic modulii ML–1T–2 Resistance ML2T–3A–2
–1 –2
Stress ML T ML2 T–2A–1
(Contd.)
1.2 Comprehensive Physics—JEE Advanced

Physical Quantity Dimensional Formula Physical Quantity Dimensional Formula


2 0
Moment of inertia ML T ML0T–2A–1

Surface tension ML0T–2 Permeability MLT –2A–2


Viscosity ML–1T –1 Permittivity M–1L–3T4A2
–1 2 –2
Gravitational constant M LT Planck’s constant ML2T –1

PRINCIPLE OF HOMOGENEITY OF Dimensions of at = dimensions of x


1.2 [ x] [L]
DIMENSIONS or [a] = [LT 1 ] [M 0 LT 1 ]
[t ] [T]
Consider a simple equation,
A + B = C. Dimensions of bt2 = dimensions of x
If this is an equation of physics, i.e. if A, B and C are x L
or b= 2 = [LT–2] = [M0 LT–2]
physical quantities, then this equation says that one physi- t T2
cal quantity A, when added to another physical quantity B,
gives a third physical quantity C. This equation will have
no meaning in physics if the nature (i.e. the dimensions) 1.2
of the quantities on the left-hand side of the equation is The pressure P, volume V and temperature T of a gas
not the same as the nature of the quantity on the right-hand are related as
A is a length, B must also be a length a
and the result of addition of A and B P V b = cT
V2
In other words, the dimensions of both sides of a physical
where a, b, and c are constants. Find the dimensions
equation must be identical. This is called the principle of
a
homogeneity of dimensions. of .
b
1.3 USES OF DIMENSIONAL ANALYSIS
SOLUTION
Dimensional equations provide a very simple method of
deriving relations between physical quantities involved in a
Dimensions of = dimensions of P.
any physical phenomenon. The analysis of any phenom- V2
enon carried out by using the method of dimensions is Dimensions of a = dimensions of PV2
called dimensional analysis. This analysis is based on the Also dimensions of b = dimensions of V
a [ PV 2 ]
There are four important uses of dimensional Dimensions of =
b [V ]
equations:
1. Checking the correctness of an equation. = [PV] = [ML–1 T–2] [L3]
2. Derivation of the relationship between the physical = [ML2 T – 2]
quantities involved in any phenomenon.
NOTE
3. Finding the dimensions of constants or variables in
an equation. 1. Trigonometric function (sin, cos, tan, cot etc) are di-
4. Conversion of units from one system to another. mensionless. The arguments of these functions are also
dimensionless
1.1 -
nents are also dimensionless
The distance x travelled by a body varies with time t as
x = at + bt2, where a and b are constants. 1.3
Find the dimensions of a and b. When a plane wave travels in a medium, the displace-
ment y of a particle located at x at time t is given by
SOLUTION
y = a sin(bt + cx)
The dimensions of each term on the right hand of the
where a, b and c are constants. Find the dimensions
given equation must be the same as those of the left b
hand side. Hence of .
c
Units and Dimensions 1.3

this equation must be the same. Equating the powers


SOLUTION
of M, L and T, we have
Terms bt and cx must be dimensionless. Hence 1
a = 0, b + c = 0 and – 2c = 1, which give b = and
1 2
[b] = [T 1] 1
[t ] c= .
2
1 1 Hence t = k m0 l1/2 g(– 1/2)
and [c] = [L 1 ]
[ x] [ L]
l
b t= k
= [LT –1] = M0 LT –1] g
c
Note that the dimensions of a are the same as those Thus t is independent of the mass of the bob
of y. and is directly proportional to l and inversely

1.4 proportional to g.
The dimensional method can also be used to convert a
2 physical quantity from one system to another. The method
a
P= e ax is based on the fact that the magnitude of a physical
b quantity X remains the same in every system of its
P is pressure, x is a distance and a and b are constants. measurement, i.e.
Find the dimensional formula for b. X = n1 u1 = n2 u2 (1)
SOLUTION where u1 and u2 are the two units of measurement of
quantity X and n1 and n2 are their respective numerical
a2 values. Suppose M1, L1 and T1 are the fundamental units
= [P]
b of mass, length and time in one system of measurement
and M2, L2 and T2 in the second system of measurement.
Also ax is dimensionless. Hence [a] = [L – 1]. Let a, b and c be the dimensions of mass, length and time
[a 2 ] [L 2 ] of quantity X, the units of measurement u1 and u2 will be
[b] = =
[ P] [ML 1 T 2 ] u1 = M1a Lb1 T1c
= [M–1 L–1 T2]
and u2 = M a2 Lb2 T2c
The principle of homogeneity of dimensions can also
Using these in Eq. (1), we have
on other physical quantities. n1 M1a Lb1 T1c = n2 M a2 Lb2 T2c
1.5 a b c
M1 L1 T1
The time period (t) of a simple pendulum may depend n2 = n1 (2)
M2 L2 T2
upon m the mass of the bob, l the length of the string
and g the acceleration due to gravity. Find the depen- Knowing (M1, L1, T1), (M2, L2, T2), (a, b and c) and the
dence of t on m, l and g. value of n1 in system 1, we can calculate the value of n2 in
system 2 from Eq. (2).
SOLUTION
Let t m al bg c 1.6
or t = k m al bg c, Dyne is the unit of force in the c.g.s. system and new-
ton is the unit of force in the SI system. Convert 1
where k is a dimensionless constant.
dyne into newton.
Writing the dimensions of each quantity, we have
2 c
[T] = [M ]a [L]b [LT ] SOLUTION
The dimensional formula of force is
or [M 0 L0 T] = [M a Lb c
T 2c
] [F] = [M1 L1 T–2]
According to the principle of homogeneity of dimen-
Therefore, a = 1, b = 1 and c = –2
sions, the dimensions of all the terms on either side of
1.4 Comprehensive Physics—JEE Advanced

System 1 System 2 SOLUTION


(Given System) (Required System)
[MLT–2] = 10 N (1)
n1 = 1 dyne n2 = ? (number of newtons)
M1 = 1g M2 = 1 kg [ML2 T–2] = 100 J (2)
L1 = 1 cm L2 = 1 m
[LT–1] = 5 ms–1 (3)
T1 = 1 s T2 = 1 s
a b c
Dividing Eq. (2) by Eq. (1), we get
M1 L1 T1 100 J 100 Nm
n2 = n1 L= = = 10 m
M2 L2 T2 10 N 10 N
1 1 2
1g 1 cm 1s Using L = 10 m in Eq. (3), we get T = 2 s. Using
=1 T = 2 s and L = 10 m in either Eq. (1) or Eq. (2), we
1 kg 1m 1s
get M = 4 kg.
1 1
1g 1 cm
=1 1 1.4 LIMITATIONS OF DIMENSIONAL ANALYSIS
103 g 102 cm
=1 10–5 Though the dimensional method is a simple and a very

Hence there are 1 10–5 newtons in 1 dyne, i.e. 1 quantity on other quantities of a given system, it has its
dyne = 10–5 N. own limitations, some of which are listed as follows:
1. In more complicated situations, it is often not easy
1.7
Convert 72 kmh–1 into ms–1 by using the method of will depend. In such cases, one has to make a guess
dimensions. which may or may not work.
2. This method gives no information about the dimen-
sionless constant which has to be determined either
SOLUTION -
Given System Required System vation.
n1 = 72 units n2 = ? 3. This method is used only if a physical quantity var-
ies as the product of other physical quantities. It
L1 = 1 km L2 = 1 m
fails if a physical quantity depends on the sum or
T1 = 1 h T2 = 1 s difference of two quantities. Try, for instance, to
1 2
1 1 obtain the relation S = ut + at using the method
M1 L1 2
n2 = n1
M2 L2 of dimensions.
4. This method will not work if a quantity depends
1
1000 m 60 60 s on another quantity as sin or cos of an angle, i.e. if
= 72
1m 1s the dependence is by a trigonometric function. The
method works only if the dependence is by power
72 1000 functions only.
= = 20
60 60 5. This method does not give a complete information
–1
Hence 72 km h = 20 ms –1 in cases where a physical quantity depends on more
than three quantities, because by equating the pow-
NOTE ers of M, L and T, we can obtain only three equa-
Sometimes it is more convenient to use units rather than

72 km 72 1000 m 1.5 SIGNIFICANT FIGURES


72 km h–1 = = = 20 ms–1
1h 60 60 s
indicates the degree of precision of that measurement.
1.8 The degree of precision is determined by the least count
If the units of force, energy and velocity are 10 N, of the measuring instrument. Suppose a length measured
100 J and 5 ms–1 by a metre scale (of least count = 0.1 cm) is 1.5 cm, then
time.
Units and Dimensions 1.5

with a vernier callipers (of least count = 0.01 cm) the

0.001 cm) the same length may be 1.536 cm which has 1.9

It must be clearly understood that we cannot increase


the accuracy of a measurement of changing the unit. For

39.4 kg. It is understood that the measuring instrument has SOLUTION

g or 39400000 mg, we cannot change the accuracy of M


measurement. Hence 39400 g or 39400000 mg still have

only the magnitude of measurement.

M
2. We use the following rule to determine the number

and division of various physical quantities.


The least accurate quantity determines the accuracy of Do not worry about the number of digits after the
the sum or product. The result must be rounded off to the
decimal place. Round off the result so that it has the
appropriate digit.
accurate quantity.

1.10
A man runs 100.2 m in 10.3 s. Find his average speed
(preceding) digit to be retained is left unchanged.

4 which is less than 5. SOLUTION


2. If the digit to be dropped is more than 5, the preced-
100.2 m –1
- Average speed (v) =
10.3s

3. If the digit to be dropped happens to be 5, then the time 10.3 s has only three. Hence the value of the
(a) the preceding digit to be retained is increased
by 1 if it odd, or The correct result is v = 9.73 ms–1
(b) the preceding digit is retained unchanged if it
is even. LEAST COUNTS OF SOME MEASURING
1.6 INSTRUMENTS
result of the indicated rounding-off is therefore, 5.3.
1. Least count of metre scale = 1 mm = 0.1 cm
2. Vernier constant (or least count) of vernier callipers
off to 3.4. = value of 1 main scale division – value of 1 vernier
scale division = 1 M.S.D. – 1 V.S.D
Let the value of 1 M.S.D = a unit
1. For addition and subtraction, we use the following If n vernier scale divisions coincide with m main
rule. scale divisions, then value of
Find the sum or difference of the given measured m
1.V.S.D = of 1 M.S.D
n
that it has the same number of digits after the deci- ma
mal place as in the least accurate quantity (i.e., the = unit
n
1.6 Comprehensive Physics—JEE Advanced

ma m u x y z
Least count = a – 1 a unit =
n n u x y z
3. Least count of a micrometer screw is found by the The proportional or relative error in u is u/u. The
formula values of x, y and z may be positive or negative
Least count = and in some uses the terms on the right hand side
Pitch of screw may counteract each other. This effect cannot be
Total number of divisions on circular scale relied upon and it is necessary to consider the worst
case which is the case when all errors add up giving
where pitch = lateral distance moved in one com-
an error u given by the equation:
plete rotation of the screw.
u x y z
=
ORDER OF ACCURACY: PROPORTIONATE u ma x y z
1.7 ERROR
u, multiply the proportional errors in each factor
The order of accuracy of the result of measurements is (x, y and z) by the numerical value of the power
determined by the least counts of the measuring instruments to which each factor is raised and then add all the
used to make those measurements. Suppose a length x is terms so obtained.
measured with a metre scale, then the error in x is x,
where x = least count of metre scale = 0.1 cm. If the same proportional error in the result of u. When the pro-
length is measured with vernier callipers of least count portional error of a quantity is multiplied by 100,
0.01 cm, then x = 0.01 cm. we get the percentage error of that quantity.
x
.
x x 1.11
100.
x ) of a
1. Suppose a quantity is given by rectangular metal block, a student makes the follow-
a=x+y ing measurements.
Then a = x + y Mass of block (m) = 39.3 g
a x y Length of block (x) = 5.12 cm
= Breadth of block (y) = 2.56 cm
a (x y)
Thickness of block (z) = 0.37 cm
2. If a = x – y -
mum error is The uncertainty in the measurement of m is ± 0.1 g
a= x+ y and in the measurement of x, y and z is ± 0.01 cm.
We take the worst case in which errors add up. Find the value of (in g cm – 3) up to appropriate
a x y
= of .
a ( x y)
3. Suppose we de- SOLUTION
termine the value of a physical quantity u by mea-
suring three quantities x, y and z whose true values m 39.3
= =
are related to u by the equation xyz 5.12 2.56 0.37
u = x y z– –3

m x y z
quantities x, y and z be respectively ± x, ± y and = + + +
ma m x y z
± z so that the error in u by using these observed
quantities is ± u. The numerical values of x, y 0.1 0.01 0.01 0.01
=
and z are given by the least count of the instru- 39.3 5.12 2.56 0.37
ments used to measure them. = 0.0353
–3
Taking logarithm of both sides we have = 0.0353 = 0.0353
log u = log x + log y – log z Round off error
Partial differentiation of the above equation gives = 0.3 gcm – 3.
Units and Dimensions 1.7

–3
Hence is not accurate to the fourth of quantities A, B, C and D are 1%, 2%, 3% and 4%
decimal place. In fact, it is accurate only up to the respectively. Find the percentage error in the mea-
much be surement of X.

is written as SOLUTION
–3
X A B 1 C D
= 2 + + +3
X A B 3 C D
1.12
1
The time period of a simple pendulum is given by =2 1% + 2% + 3% + 3 4%
3
T = 2 L/ g . The measured value of L is 20.0 cm
= 17%
using a scale of least count 1 mm and time t for 100
oscillations is found to be 90 s using a watch of least
NOTE
count 1 s. Find the value of g (in m s–2) up to appro-

the value of g. 2. The quantity which is raised to the highest power con-
-
SOLUTION ured to a high degree of accuracy.

t
If t is the time for n oscillations, then T = . Given
n 1. For a simple pendulum T = 2
L g
T=2 . Squaring, we get
g
T 1
4 2 Ln 2 =
g= (1) T 2
t2
2. For a sphere of radius r,
Putting L = 0.200 m, n = 100 and t = 90 s, A 2 r
Surface area A = 4 r2 =
4 (3.14) 2 0.2 (100) 2 A r
g=
(90) 2 4 3 V 3 r
Volume V = r
= 9.74 m s–2 3 V r
From Eq. (1), the relative error in g is 3. Acceleration due to gravity g =
GM
g L 2 t R
=
g L t g 2 R M
=
Note that there is no error in counting the number (n) g R M
of oscillations. Thus 4. For resistances connected in series
g 0.1 cm 2 1s
= Rs R1 R2
g 20.0 cm 90s Rs = R1 + R2
Rs R1 R2
= 0.005 + 0.022 = 0.027
5. For resistances connected in parallel
g = 0.027 9.74 = 0.26 m s–2
Rounding off g 1 1 1 Rp R1 R2
= –
g = 0.3 m s–2. Hence the value of g must be rounded Rp R1 R2 R 2p R12 R22
off as g = 9.7 m s–2. Hence Rp R1 R2
g = (9.7 ± 0.3) m s–2 =
RP2 R12 R22

1.13 6. Kinetic energy K and linear momentum p are related


as
In the measurement of a physical quantity X =
p2 K 2 p
A2 B K =
, the percentage errors in the measurements 2m K p
C1/3 D3
1.8 Comprehensive Physics—JEE Advanced

Multiple Choice Questions with Only One Choice Correct

1. The pressure P is related to distance x, Boltzmann (c) 1.7 cm3 (d) 17.3 cm3
constant k and temperature as 9. The quantities L/R and RC (where L, C and R
a –ax/k stand for inductance, capacitance and resistance
P= e
b respectively) have the same dimensions as those of
The dimensional formula of b is (a) velocity (b) acceleration
(a) [M –1 L–1T–1] (b) [MLT 2] (c) time (d) force
(c) [M 0 L2T 0] (d) [M0 L0 T 0] 10. Which one of the following has the dimensions of
2. The magnitude of induced emf e in a conductor of ML–1T –2?
length L B is given by (a) torque (b) surface tension
(c) viscosity (d) stress
1
e= ( BL2) 11. The dimensions of angular momentum are
a (a) MLT –1 (b) ML2T–1
The dimensional formula of a is (c) ML –1T (d) ML0T –2
(a) [M 0 L0T] (b) [ML2T –2] 12. According to the quantum theory, the energy E of a
(c) [M 2 LT –1] (d) M 0 L0 T 0] photon of frequency is given by
3. Two resistors R1 = [3.0 ± 0.1] and R2 = (6.0 ± E=h
0.3) are connected in parallel. The resistance of where h is Planck’s constant. What is the dimen-
the combination is sional formula for h?
(a) (2.0 ± 0.4) (a) M L2 T –2 (b) M L2 T –1
(c) (2.0 ± 0.2) (d) (2.00 ± 0.04) 2
(c) M L T (d) M L2 T2
4. If the resistances in Q. 3 above were connected in
- 13. The dimensions of Planck’s constant are the same
tance of the combination will be as those of
(a) 1.1% (b) 2.2% (a) energy
(c) 3.3% (d) 4.4% (b) power
5. Which of the following pairs of physical quantities (c) angular frequency
do not have the same dimensions? (d) angular momentum
(a) Pressure and Young’s modulus 14. The volume V of water passing any point of a
(b) Emf and electric potential uniform tube during t seconds is related to the
(c) Heat and work cross-sectional area A of the tube and velocity u of
water by the relation
6. Which of the following pairs of physical quantities V A u t
have different dimensions? which one of the following will be true?
(a) Impulse and linear momentum (a) = = (b) =
(b) Planck’s constant and angular momentum
(c) = (d)
(c) Moment of inertia and moment of force
(d) Torque and energy 15. The frequency n of vibrations of uniform string of
7. A = A0 e–a/kT, k is Boltzmann length l and stretched with a force F is given by
constant and T is the absolute temperature. The di- n=
p F
mensions of a are the same as those of 2l m
(a) energy (b) time where p is the number of segments of the vibrating
(c) acceleration (d) velocity string and m is a constant of the string. What are the
8. A cube has a side of 1.2 cm. The volume of the cube dimensions of m?
(a) M L–1 T–1 (b) M L–3 T0
3
(b) 1.73 cm3 (c) M L T–2 0
(d) M L–1 T0
Units and Dimensions 1.9

16. If velocity (V), acceleration (A) and force (F) are 29. The dimensions of capacitance are
taken as fundamental quantities instead of mass (a) M–1 L–2 TA2 (b) M–1 L–2 T2 A2
–1 –2 3 2
(M), length (L) and time (T), the dimensions of (c) M L T A (d) M–1 L–2 T4 A2
Young’s modulus would be 30. If velocity (V), force (F) and energy (E) are taken
(a) FA2V –2 (b) FA2V–3 as fundamental units, then dimensional formula for
(c) FA2V–4 (d) FA2V –5 mass will be
17. The dimensions of permittivity ( 0) of vacuum are (a) V –2F 0 E (b) V 0 FE 2
(a) M–1 L–3 T4 A2 (b) ML–3 T2 A2 (c) VF –2E 0 (d) V–2 F 0E
(c) M–1 L3 T4 A2 (d) ML3 T2 A2 31. Frequency (n) of a tuning fork depends upon length
(l) of its prongs, density ( ) and Young’s modulus
18. What are the dimensions of permeability ( 0) of (Y) of its material. Then frequency and Young’s
vacuum? modulus will be related as
(a) MLT–2 A2 (b) MLT–2 A–2 (a) n Y (b) n Y
(c) ML–1 T–2 A2 (d) ML–1 T–2 A–2 1 1
(c) n (d) n
Y Y
1 2
32. The dimensions of 0 E ( 0 = permittivity of free
19. The dimensions of 1/ 0 0 are the same as those 2
of space and E
(a) velocity (b) acceleration (a) MLT –1 (b) ML2T –2
(c) force (d) energy (c) ML T –1 –2
(d) ML2T–1
20. IIT, 2000
(a) MLT –2 K–1 (b) ML2 T–2 K–1
33. Of the following quantities, which one has dimen-
(c) M0L2T–2 K–1 (d) M0LT–2 K–1 sions different from the remaining three
21. What are the dimensions of latent heat?
(a) Energy per unit volume
(a) ML2 T –2 (b) ML–2 T–2
0 –2 (b) Force per unit area
(c) M LT (d) M 0 L2T –2
(c) Product of voltage and charge per unit volume
22. What are the dimensions of Boltzmann’s con-
(d) Angular momentum
stant?
34. If the time period t of a drop of liquid of density d,
(a) MLT –2 K–1 (b) ML2T–2 K–1 radius r, vibrating under surface tension s is given
0 –2 –1
(c) M LT K (d) M0L2T–2 K–1
by the formula t = d a r b s c and if a = 1, c = – 1,
23. The dimensions of potential difference are then b is
(a) ML2T – 3 A – 1 (b) MLT–2 A–1 (a) 1 (b) 2
(c) ML2T – 2 A (d) MLT–2 A (c) 3 (d) 4
24. What are the dimensions of electrical resistance? 35. In the measurement of a physical quantity X =
(a) ML2T –2 A2 (b) ML2 T–3 A–2 A2 B
. The percentage errors introduced in the
(c) ML2 T –3 A2 (d) ML2 T–2 A–2 C 1/ 3 D 3
25. measurements of the quantities A, B, C and D are
(a) MLT –3 A–1 (b) MLT –2 A–1 2%, 2%, 4% and 5% respectively. Then the mini-
(c) MLT A –1 –1
(d) MLT0 A–1 mum amount of percentage of error in the mea-
26. surement of X is contributed by:
(a) ML0 T –1 A–1 (b) M0L T –1 A–1 (a) A (b) B
–2 –1
(c) MLT A (d) ML 0 T–2 A–1 (c) C (d) D
27. 36. Which of the following has the dimensions
(a) ML2 T –2 A–1 (b) ML2 T–2 A–2 ML–1 T–1?
–2 –2 –1
(c) ML T A (d) ML–2 T–2 A–2
(c) Bulk modulus (c) Angular momentum
28. The dimensions of self inductance are 37. Pressure gradient dp/dx is the rate of change of
(a) ML2 T–2 A–1 (b) ML2 T–2 A–2 pressure with distance. What are the dimensions of
(c) ML–2 T–2 A–1 (d) ML–2 T–2 A–2 dp/dx?
1.10 Comprehensive Physics—JEE Advanced

(a) ML–1 T–1 (b) ML–2 T–2 a, b


(c) ML T –1 –2
(d) ML–2 T–1 and c respectively are
38. If E, M, J and G respectively denote energy, mass, 5 1 1 1 5 1
(a) , , (b) , ,
angular momentum and gravitational constant, then 6 2 3 2 6 3
EJ 2 1 1 5
has the dimensions of (c) , , (d) 1, 1, 1
M 5G 2 3 2 6
(a) length (b) angle
(c) mass (d) time 46. In a system of units in which the unit of mass is
IIT, 1990 a kg, unit of length is b metre and the unit of time
39. If e, 0, h and c respectively represent electronic is c second, the magnitude of a calorie is
charge, permittivity of free space, Planck’s con- 4.2c 4.2c 2
(a) (b)
e2 ab 2 ab 2
stant and speed of light, then has the dimen-
0 hc abc 4.2
sions of (c) (d)
(a) current (b) pressure 4.2 abc
(c) angular momentum (d) angle 47. The error in the measurement of the radius of a
40. If L, R, C and V respectively represent inductance, sphere is 1%. The error in the measurement of the
resistance, capacitance and potential difference, volume is
then the dimensions of
L
are the same as those (a) 1% (b) 3%
of RCV
1 48. If the error in the measurement of the volume of a
(a) current (b)
current sphere is 6%, then the error in the measurement of
1 its surface area will be
(c) charge (d) (a) 2% (b) 3%
charge
41. If E and B respe (c) 4% (d) 7.5%
E 49. The moment of inertia of a body rotating about a
has the 2
B in the SI system. What is
dimensions of the value of the moment of inertia in a system of
(a) displacement (b) velocity units in which the unit of length is 5 cm and the unit
(c) acceleration (d) angle of mass is 10 g?
42. If C and V respectively represent the capacitance of (a) 2.4 × 103 (b) 2.4 × 105
a capacitor and the potential difference between its (c) 6.0 × 103 (d) 6.0 × 105
plates, then the dimensions of CV2 are V
(a) ML2T–2 (b) ML3T–2 A–1 50. A quantity X is given by 0L where 0 is the
t
(c) ML2T–1A–1 (d) M0L0T0 permittivity of free space, L is a length, V is a
43. If h and e respectively represent Planck’s constant potential difference and t is a time interval. The
h dimensional formula for X is the same as that of
and electronic charge, then the dimensions of (a) resistance (b) charge
e
are the same as those of (c) voltage (d) current
IIT, 2001
51. ) of a liquid by the
44. If energy E, velocity V and time T are chosen as
the fundamental units, the dimensional formula for the formula
surface tension will be R4 P
(a) E V2T–2 (b) E V–1T–2 l Q
(c) E V–2T–2 (d) E2V–1T–2 where R = radius of the capillary tube,
45. l = length of the tube,
oscillates with a period proportional to Pa db Ec P = pressure difference between its ends,
where P is the static pressure, d is the density of and
Units and Dimensions 1.11

Q
Which quantity must be measured most accurately? -
dent also measures the diameter of the wire to be
(a) R (b) l
0.4 mm with a uncertainty of ± 0.01 mm. Take g
(c) P (d) Q 2
52. The mass m of the heaviest stone that can be from the reading is
(a) (2.0 ± 0.3) 1011 N/m2
v, the speed of water, density (d) of water and the (b) (2.0 ± 0.2) 1011 N/m2
acceleration due to gravity (g). Then m is propor- (c) (2.0 ± 0.1) 1011 N/m2
tional to (d) (2.0 ± 0.05) 1011 N/m2
(a) v2 (b) v4 IIT, 2007
6
(c) v (d) v
53. The speed (v) of ripples depends upon their wave- 59. In a vernier callipers, one main scale division is
length ( ), density ( ) and surface tension ( ) of x cm and n divisions of the vernier scale coincide
water. Then v is proportional to with (n – 1) divisions of the main scale. The least
count (in cm) of the callipers is
(a) (b)
n 1 nx
1 1 (a) x (b)
(c) (d) n (n 1)
x x
54. The period of revolution (T) of a planet moving (c) (d)
n (n 1)
round the sun in a circular orbit depends upon the
radius (r) of the orbit, mass (M) of the sun and the IIT, 2007
gravitation constant (G). Then T is proportional to 60.
(a) r1/2 (b) r measuring the acceleration due to gravity (g) using a
(c) r3/2 (d) r2 simple pendulum. They use different lengths of the
pendulum and/or record time for different number of
55. If the velocity of light (c), gravitational constant oscillations. The observations are shown in the table.
(G) and planck’s constant (h) are chosen as fun- Least count for length = 0.1 cm
damental units, the dimensions of time in the new Least count for time = 0.1 s
system will be
(a) c –5/2G2h–1/2 (b) c–3/2G–2h2 Student Length of the Number of Total time for Time
2 –2 1/2 –5/2 1/2 1/2 pendulum oscillations (n) period
(c) c G h (d) c G h (cm) (n) oscillations (s) (s)
56. The amplitude of a damped oscillator of mass m
I 64.0 16.0
varies with time t as
II 64.0 4 64.0 16.0
A = A0 e( at m )
The dimensions of a are III 20.0 4 36.0 9.0
(a) ML0T–1 (b) M0LT–1 If EI, EII and EIII are the percentage errors in g. i.e.
–1
(c) MLT (d) ML–1T
g
57. A student measures the value of g with the help of 100 for student I, II and III, respectively,
a simple pendulum using the formula g
4 2
L (a) E1 = 0 (b) EI is minimum
g= 2 (c) EI = EII (d) EII is minimum
T
The errors in the measurements of L and T are L
and T respectively. In which of the following cas- 61. The density of a solid ball is to be determined in an
es is the error in the value of g the minimum?
(a) L = 0.5 cm, T = 0.5 s with a screw gauge, whose pitch is 0.5 mm and
(b) L = 0.2 cm, T = 0.2 s there are 50 divisions on the circular scale. The
(c) L = 0.1 cm, T = 1.0 s reading on the main scale is 20 divisions. If the
(d) L = 0.1 cm, T = 0.1 s measured mass of the ball has a relative error of
58. 2%, the relative percentage error in the density is
(a) 0.9% (b) 2.4%
long, by Searle’s method. In a particular reading, (c) 3.1% (d) 4.2%
IIT, 2011
1.12 Comprehensive Physics—JEE Advanced

ANSWERS
1. (c) 2. (a) 3. (b) 4. (d) 5. (d) 6. (c)
7. (a) 8. (c) 9. (c) 10. (d) 11. (b) 12. (b)
13. (d) 14. (b) 15. (d) 16. (c) 17. (a) 18. (b)
19. (a) 20. (c) 21. (d) 22. (b) 23. (a) 24. (b)
25. (a) 26. (d) 27. (a) 28. (b) 29. (d) 30. (d)
31. (a) 32. (c) 33. (d) 34. (c) 35. (c) 36. (b)
37. (b) 38. (b) 39. (d) 40. (b) 41. (b) 42. (a)
43. (c) 44. (c) 45. (a) 46. (b) 47. (b) 48. (c)
49. (b) 50. (d) 51. (a) 52. (c) 53. (d) 54. (c)
55. (d) 56. (a) 57. (d) 58. (b) 59. (c) 60. (b)
61. (c)

SOLUTIONS 5. The correct choice is (d).


6. The correct choice is (c).
1. 7. Since kT has dimensions of energy, the correct
1
k JK K choice is (a).
[a] = =
x m 8. V = L3 = 1.2 cm 1.2 cm 1.2 cm
3
–1
=Jm
= [M L2T–2] [L–1] L = 1.2 cm,
= M L T –2
[a] Hence the correct choice is (c).
Also [P] =
[b] 9. L/ R is the time constant of an L-R circuit and CR
[a] ML1 T 2 is the time constant of a C-R circuit. The dimension
[b] = = 1 2
= [M 0 L2T 0] of the time constant is the same as that of time.
[ P] ML T
Hence the correct choice is (c).
So the correct choice is (c). 10. ML–1T –2 are the dimensions of force per unit area.
BL2 [M 0 L0 T 0 ] [ML0 T 2 A 1 ] [L2 ] Out of the four choices, stress is the only quantity
2. a = =
e [ML2 T 3 A 1 ] that is force per unit area. Hence the correct choice
is (d).
= [M0 L0T1]
11. The angular momentum L of a particle with respect
1 1 1 to point whose position vector is r is given by
3. =
R R1 R2 L=r p
R1 R2 3.0 6.0 where p is the linear momentum of the moving
R= = = 2.0 particle.
R1 R2 9.0
Dimensions of L = dimension of r dimensions
R
=
R1 R2 of p
R 2
R12 R22 = L MLT –1 = ML2T–1
Thus the correct choice is (b).
0.1 0.3
= 2
= 0.019 dimension of E ML2 T 2
(3) ( 6) 2 12. Dimensions of h = =
dimension of T 1
R = 0.019 R2 = 0.019 (2)2 = 0.076
= ML2 T –1
Hence the correct choice is (b). Thus the correct choice is (b).
4. R = R1 + R2 = 3.0 + 6.0 = 9.0 13. The correct choice is (d).
R = R1 + R2 = 0.1 + 0.3 = 0.4 14. The dimensions of the two sides of proportionality
R
100 are
R
L 3 = L 2 (LT –1 ) T = L2 + T –
0.4 Equating the powers of dimensions on both sides,
= 100
9.0 we have
= 4.4% 2 + =3
So the correct choice is (d). – =0
Units and Dimensions 1.13

1 L MLT 2
L1
which give = and =
(3 – ), i.e. = . Dimensions of =
2 0
A2
Thus the correct choice is (b). = MLT A – 2 –2
15. Squaring both sides of the given relation, we get Therefore, the correct choice is (b).
p2 F p2 F 19. Dimensions of
n2 = 2 or m =
4l m 4 l 2 n2 1 1
dimensions of m 1
2
dimensions of F
0 0
MLT A2 M 1L 3 T 4 A 2 2
=
dimensions of l 2 dimensions of n 2 1
= = LT – 1
( p is a dimensionless number) 1
2 2 2
2 L T
MLT –1 0
= = ML T which are the dimensions of velocity. Hence the
1 2
L2 T correct choice is (a).
Hence the correct choice is (d). 20. The heat energy content H of a body of mass m at
16. Dimensions of Young’s modulus Y are ML–1 T–2. temperature is given by H = ms
The dimension of V, A and F in terms of M, L and where s
T are H
(V) = LT –1, (A) = LT –2 s=
m
and (F) = MLT –2 Dimensions of s
Let (Y ) = (V a A b F c ) dimensions of heat energy
Putting dimensions of Y, V, A and F. We have dimension of mass dimension of temperature
(ML–1 T –2 ) = (LT –1) a ( LT – 2 ) b ML2 T 2
= = M 0 L 2T – 2 K – 1
(MLT – 2) c M K
or M1 L – 1 T – 2 = M c La + b + c T – a – 2b – 2c Thus the correct choice is (c).
Equating powers of M, L and T we have 21. Latent heat L is the amount of heat energy H re-
quired to change the state of a unit mass without
c = 1, a + b + c = – 1
producing any change in temperature. Thus
and – a – 2b – 2c = – 2
H
which give a = – 4, b = 2 and c = 1. L=
2 –4
Thus (Y) = (FA V ) m
Thus the correct choice is (c). ML2 T 2
Dimensions of L =
17. According to Coulomb’s law of electrostatics, force M
F between two charges q1 and q2 a distance r apart = L T = M 0 L 2T – 2
2 –2

in vacuum, is given by Thus the correct choice is (d).


1 q1q2 22. According to the law of equipartition of energy,
F= the energy per degree of freedom of a gas atom or
4 0 r2 molecule at a temperature kelvin is given by
1 q1q2 1 2E
or 0 = E= k or k =
4 F r2 2
where k is the Boltzmann’s constant.
Q2 dimensions of E
Dimensions of 0 = Dimensions of k =
MLT 2 L2 dimension of
–1 –3 2 2
=M L T Q
Q ML2 T 2
=M L T A A –1 –3 4 2 ML2 T 2 K 1
T K
The correct choice is (a). 23. The potential difference V between two points is
18. The force per unit length between two long wires the amount of work done in moving a unit charge
carrying currents I1 and I2 a distance r apart in vac- from one point to the other.
uum, is given by
II 4 rf work done W
f = 0 1 2 or 0 = Thus, V=
4 r I1 I 2 charge moved q
1.14 Comprehensive Physics—JEE Advanced

ML2 T 2 ML2 T 3 A 1
Dimensions of V = = ML2 T –2 Q –1 = = ML2 T –2 A–2
Q A/T
= ML2 T –3 A –1 ( Q = AT) Thus the correct choice is (b).
Hence the correct choice is (a). 29. When a capacitor of capacitance C is charged to a
24. From Ohm’s law, resistance R is given by potential difference V, the charge Q on the capaci-
potential difference tor plates is given by
R= Q
current Q = CV or C =
V
ML2 T 3 A 1
Dimensions of R = dimensions of Q
A Dimensions of C =
2 – 3 –2 dimensions of V
= ML T A AT
Thus the correct choice is (b). =
25. Force F q in an electric ML T 3 A 1
2

E is given by = M –1 L –2 T 4 A2
F Hence the correct choice is (d).
F = qE or E =
q 30. Let (M) = V a F b E c
dimensions of F MLT 2 Putting the dimensions of V, F and E, we have
Dimensions of E =
dimensions of Q AT (M) = (LT –1 )a (MLT –2 )b (ML2T –2 )c
= MLT–3A–1. or M 1 = M b +c La+b+2c T –a–2b–2c
26. The force F q moving Equating the powers of dimensions, we have
with speed v perpendicular to the direction of a
B is given by b+c=1
F a + b + 2c = 0
F = qvB or B =
qv – a – 2b – 2c = 0
MLT 2 which give a = – 2, b = 0 and c = 1. Therefore
Dimensions of B = = ML0 T –1Q–1 (M) = (V –2 F 0 E).
Q LT 1
Thus the correct choice is (d).
= ML0T –2A–1 ( Q = AT)
Hence the correct choice is (d) 31. Let n l a b Yc
27. linked with a circuit of area A Putting dimensions of all the quantities, we have
B is given by (T –1) La (ML–3 ) b (ML–1 T –2)c
= BA cos Equating powers of M, L and T on both sides, we get
where
b + c = 0, a – 3b – c = 0 and – 2c = – 1
vectors.
1 1
Dimensions of = dimensions of BA which give a = – 1, b = – and c = . Thus
( cos is dimensionless) 2 2
= ML0 T –2 A–1 L2 n l –1 1/2 Y 1/2
= ML2 T–2 A–1 Hence the correct choice is (a).
Thus the correct choice is (a). 32. We know that
28. The self inductance L of a coil in which the current q1 q2 F
dI F= and E =
varies at a rate is given by 4 0 r 2
q
dt dI
e=–L q1 q2
dt 0 =
where e is the e.m.f. induced in the coil. Now, 4 F r2
the dimensions of e.m.f. are the same as those of 1 q1 q2 F2
potential difference, namely, ML2 T –3 A–1. Hence 0 E2 =
2 F r2 q2
e
Now, L=– q1 q2 F
dI = 2
dt q r2
Dimensions of L
1 F
dimensions of e Dimensions of 0 E 2 = dimensions of =
= 2 r2
dimensions of I / dimensions of t
Units and Dimensions 1.15

2 41. The force on a particle of charge q moving with


M LT
= ML–1T –2 a velocity in E and B
L2
= q (E + B)
Hence the correct choice is (c).
Hence the dimensions of E are the same as those of
33. Energy per unit volume, force per unit area and
vB.
product of voltage and charge density all have 42. Energy stored in a capacitor of capacitance C hav-
dimensions of ML2T–2, but the dimensions of ing a potential difference V between its plates is
angular momentum are ML2T–1. Hence the correct given by
choice is (d). 1
U = CV2
34. Given t = da/2 rb/2 sc/2. Substituting dimensions, we 2
have Hence, the dimensions of CV 2 = dimensions of en-
(T) = (ML–3)a/2 (L)b/2 (MT–2)c/2 ergy. Hence the correct choice is (a).
= M(a + c)/2 · L(–3a/2 + b/2) T–c h (ML2 T 1 )
3 b 43. Dimensions of = ML2 T–2 A–1
Equating powers of L, we have, a = 0. e AT
Given a = 1. 2 2 Dimensions of B = MT–2 A–1
3 b B area
= 0 or b = 3, which is choice(c).
2 2 44. Let surface tension = Ea Vb Tc. Using the dimen-
A2 B sions of , E, V and T and equating powers of M,
35. Given X = 1/ 3 3 a, b and c. The correct
C D
Taking logarithm of both sides, we have choice is (c).
1 45. The correct choice is (a).
log X = 2 log A + log B – log C – 3 log D 46. Let n1 be the magnitude (i.e. numerical value) of
3
Partially differentiating, we have a physical quantity when the fundamental units
x A B 1 C D are (M1, L1, T1) and n2 the magnitude of the same
= 2 3 physical quantity when the fundamental units are
x A B 3 C D
(M2, L2, T2), then, it is obvious that
A
Percentage error in A = 2 = 2 2%
A n1 (M1x L1y T1z ) = n2 (M 2x Ly2 T2z ) (i)
= 4% where x, y and z are the dimensions of the physi-
B cal quantity in mass, length and time respective-
Percentage error in B = = 2%
B ly. Now, we know that 1 calorie = 4.2 joule = 4.2
1 C 1 kg m2s–2
Percentage error in C = = 4%
3 C 3 n1 = 4.2, x = 1, y = 2 and z = – 2. Hence, in the
4 second system of units in which M2 = a kg,
= % L2 = b m and T2 = c s, we have from (i)
3
D
Percentage error in D = 3 = 3 5% M1
x
L1
y
T1
z
D n2 = n1
= 15% M2 L2 T2
x y z
1 kg 1m 1s
contributed by C. Hence the correct choice is (c). = n1
a kg bm cs
36. The correct choice is (b)
1 2 2
37. The correct choice is (b) 1 1 1
= 4.2
38. Dimensions of J and G are ML2T–1 and M–1L3 T–2 a b c
respectively.
4.2c 2
39. Dimensions of o and h are M–1L–3 T4 A2 and =
ML2T–1 respectively. ab 2
4 3
40. RC has the dimensions of time (T). V has the 47. V = r . Taking logarithm of both sides, we
dimensions of emf which has the dimensions of 3
dI have
L . log V = log 4 + log + 3 log r – log 3
dt
1.16 Comprehensive Physics—JEE Advanced

Differentiating, we get 55. The correct choice is (d).


V r 56.
=3 = 3 × 1% = 3% at/m is dimensionless. Therefore,
V r
dimension of m M
48.
V
=3
r
or 6% = 3
r
or
r
= 2%. Dimension of a = =
V r r r dimension of t T
Now surface area s = 4 r2 or log s = log 4 + 2 log r = M L0 T–1
s r 57. The proportionate error in the measurement of g
=2 = 2 × 2% = 4%.
s r is
49. The dimensions of moment of inertia are (ML2). g
=
L
+ 2
T
We have g L T
n1(u1) = n2(u2) Hence g will be minimum if L and T are
or n1(M1 L21 ) = n2 (M 2 L22 ) minimum. Thus the correct choice is (d).
2 FL 4MgL
n1 (M1L22 ) M1 L1 58. Y= = (1)
n2 = 2
= n1 Al d 2l
(M 2 L 2 ) M2 L2
where
Given n1 = 6.0, M1 = 1 kg, L1 = 1 m, M2 = 10 g and M g –2
L2 = 5 cm. Therefore,
2 L l 10–3 m
1 kg 1m l = ± 0.05 mm, d = 0.4 mm = 0.4 10–3 m
n2 = 6.0 × ×
10 g 5 cm d = ± 0.01 mm
2 Substituting the values of M, g, L, d and l in Eq. (1)
1000 g 100 cm
= 6.0 × × we get
10 g 5 cm Y = 2.0 1011 Nm–2
= 6.0 × 100 × (20)2 = 2.4 × 105 From Eq. (1) the proportionate uncertainty in Y is
50. The capacitance of a parallel plate capacitor is giv- given by
en by C = 0A/d. Hence the dimensions of 0L are Y M g L 2 d l
the same as those of capacitance. =
Y M g L d l
V
Dimension of 0L Since the values of M, g and L M = 0,
t
g = 0 and L = 0. Hence
dimension of C dimensions of V
= Y 2 d l 2 0.01 mm 0.05 mm
time = =
Y d l 0.4 mm . mm
dimension of Q
= ( Q = CV) = 0.05 + 0.0625
time
Hence the correct choice is (d). = 0.1125
Y = 0.1125 Y = 0.1125 2.0 1011
51. The correct choice is (a). = 0.225 1011 Nm–2
is given by the Since the value of Y
relation decimal place, the value of Y must be rounded
R l P Q
= 4 Y = 0.2 1011
R l P Q –2
Nm
It is clear that the error in the measurement of R is Y + Y = (2.0 0.2) 1011 Nm–2
Hence the correct choice is (b).
of R4 in the formula. Hence the radius (R) of the 59. Vernier constant = value of 1 M.S.D – value of
capillary tube must be measured most accurately. 1 V.S.D.
Thus the quantity which is raised to the highest Now n V.S.D = (n – 1) M.S.D = (n – 1) x cm
power needs the most accurate measurement. n 1
1 V.S.D = x cm
52. Take m v a d b g c and show that a = 6. n
1 n 1
53. Take v a b c
and show that a = –. V.C. = x cm – x cm =
x
cm
2 n n
3
54. Take T r a Mb G c and show that a = – . Hence the correct choice is (c).
2
Units and Dimensions 1.17

2 Thus the percentage error is minimum for student I.


l 4 l
60. T = 2 g= Therefore, 61. Least count of screw gauge
g T2
pitch
=
g l 2 T number of divisions on circular scale
=
g l T 0.5 mm
g = = 0.01 mm
For student I, EI = 100 50
g diameter of ball = 2.5 mm + 20 0.01 mm
0.1 2 0.1 = 2.7 mm
= 100 M
64.0 .0 Density =
5 4 3
= % r
16 3
is
0.1 2 0.1
For student II, EII = 100 M 3 r
64 64.0 =
M r
15
= % 0.01
32 = 2% + 3 100
0.1 2 0.1 2.7
For student III, EIII = 100 = 2% + 1.1% = 3.1%
20.0 36.0
19
= %

II

Multiple Choice Questions with One or More Choices Correct


1. Which of the following are not a unit of time? 5. In Q.4, the dimensions of b are the same as those of
(a) parsec (b) light year (a) wave velocity (b) wave frequency
(c) micron (d) second (c) particle frequency (d) wavelength
2. Choose the pair of physical quantities which have b
identical dimensions. 6. In Q.4, the dimensions of are the same as those
c
(a) Impulse and linear momentum of
(b) Planck’s constant and angular momentum (a) wave velocity (b) angular frequency
(c) Moment of inertia and moment of force (c) particle velocity (d) wave frequency
(d) Young’s modulus and pressure 7. The Van der Waal equation for n moles of a real
3. The dimensions of energy per unit volume are the gas is
same as those of a
P (V – b) = nRT
(a) work (b) stress V2
(c) pressure (d) modulus of elasticity where P is the pressure, V is the volume, T is the
4. When a wave traverses a medium, the displacement absolute temperature, R is the molar gas constant
of a particle located at x at time t is given by and a, b are Van der Waal constants. The dimen-
y = a sin (bt – cx) sions of
where a, b and c are constants of the wave. Which (a) a are the same as those of PV2
of the following are dimensionless quantities? (b) b are the same as those of V
(a) y/a (b) bt a
(c) are the same as those of RT
b V
(c) cx (d)
c (d) bP are the same as those of RT.
1.18 Comprehensive Physics—JEE Advanced

8. In Q.7, which of the following have the same


dimensions as those of PV? 0.3)
a
(a) nRT (b)
V 0.2)
ab (c) The minimum resistance obtainable is (2.0 ±
(c) Pb (d)
V2 0.3)
9. In Q.7, the dimensions of nRT are the same as (d) The minimum resistance obtainable is (2.0 ±
those of 0.2)
(a) pressure (b) energy 14. A physical quantity P is given by
(c) work (d) force a 3b 2
P=
10. Which of the following are dimensionless? d c
(a) Boltzmann constant (b) Planck’s constant The percentage errors in the measurements of a, b,
(c) Poisson’s ratio (d) relative density c, and d are 1%, 3%, 4%, and 3% respectvely.
11. For a body in uniformly accelerated motion, the P is 14%
distance x of the body from a reference point at time P is 10%
t is given by
x = at + bt2 + c measurement of b.
where a, b and c are constants of motion. (d) The minimum error is contributed by the
(a) The dimensions of c are the same as those measurement of c.
of x, at and bt2. 15. When a plane wave travels in a meduim, the
diplacement y of a particle located at x at time t is
(b) The dimensional formula of b is [M0 LT–2].
given by
a
(c) is dimensionless. y = a sin (bt – cx)
b
(d) The acceleration of the body is 2b. where a, b, and c are constants.
12. The side of a cube is L = (1.2 ± 0.1) cm. The (a) The unit a is the same as that of y.
volume of the cube is (b) The SI unit of b is Hz.
3 (c) The dimensional formule of c is [M0L–1T0]
(b) (1.73 ± 0.02) cm3
b
(c) (1.7 ± 0.4) cm3 (d) (1.7 ± 0.3) cm3 (d) The dimensions of are the same those of
c
13. Two resistances R1 = (3.0 ± 0.1) and R2 = (6.0 ± velocity.
0.2) are to be joined together.

ANSWERS AND SOLUTIONS


1. Choices (a), (b) and (c) are units of length and cx are also dimensionless. Hence the correct
2. The dimensions of moment of inertia are ML2T 0 choice are (a), (b) and (c).
and of moment of force are ML2T–2. All other pairs 5. Since bt is dimensionless, the dimensions of b =
in (a), (b) and (d) have identical dimensions. dimensions of 1/t = T–1, which are the dimensions
3. Dimensions of energy per unit volume are = of angular frequency as well as wave frequency.
dimensions of energy / dimensions of volume = Hence the correct choices are (b) and (d).
ML2T –2 / L3 = ML–1T –2. Stress, pressure and modu- 6. Dimensions of bt = dimensions of cx. Therefore
lus of elasticity all have the dimensions of ML–1T–2. b x
Dimensions of = dimensions of = LT –1.
The dimensions of work are ML2T–2. Hence the c t
correct choices are (b), (c) and (d). Hence the correct choices are (a) and (c).
4. Since the sine function is dimensionless, sin (bt 7.
– cx) is dimensionless. Therefore, y and a must a ab
PV – Pb + = n RT
have the same dimensions, i.e. y/a is dimension- V V2
less. Since the argument of a sine function (or any From the principle of homogeneity, it follows that
trigonometric function) must be dimensionless, bt all the four choices are correct.
Units and Dimensions 1.19

8. The correct choices are (a), (b), (c) and (d). Rs ± Rs = (9.0 ± 0.3)
9. The dimensions of nRT = dimensions of PV Thus choice (a) is correct and choice (b) is wrong.
= ML–1 T –2 L3 = ML2 T–2 The minimum value is obtained when the resis-
which are dimensions of energy as well as work. tances are joined in parallel.
Hence the correct choices are (b) and (c). 1 1 1
10. The correct choices are (c) and (d). =
Rp R1 R2
11. From the principle of homogeneity of dimensions,
the dimensions of c must be the same as those of R1 R2 3.0 6.0
Rp = = = 2.0
x at and bt2. Therefore, choice (a) is correct. Also R1 R2 3.0 6.0
dimension of bt2 = dimension of x. Hence [b]= RR
Now Rp = 1 2 ( R 1 + R 2 = R s)
[LT–2]. Hence choice (b) is also correct. Velocity Rs
of the body is Rp R1 R2 Rs
dx d =
v= = [at + bt2 + c] = a + 2bt Rp R1 R2 Rs
dt dt
and acceleration is 0.1 0.2 0.3
=
dv d 3.0 6.0 9.0
= (a + 2 bt) = 2b, which is choice (d)
dt dt = 0.033 + 0.033 + 0.033
a
choice (c) is wrong since has dimension of = 0.099 0.1
b Rp = 0.1 Rp = 0.1 2 = 0.2
time [T]
12. Volume of cube (V = L3) = 1.2 cm 1.2 cm Minimum value is (Rp ± Rp) = (2.0 ± 0.2) .
3
. Now L = (1.2 ± 0.1) cm has Hence choice (c) is wrong and choice (d) is is
V must correct.
1
14. log P = 3 log a + 2 log b – log d – log c
value of V = 1.7 cm3. Now 2
V = L3 P 3 a 2 b d 1 c
=
V 3 L 3 0.1 P ma a b d 2 c
= = = 0.25 1
V L 1.2 = 3 1% + 2 3% + 3% + %
V = 0.25 V = 0.25 1.7 cm3 = 0.425 cm3 2
The error in V is in = 3% + 6% + 3% + 2% = 14%
the value of V should be rounded off as V = Hence the correct choices are (a), (c) and (d).
0.4 cm3. Thus the correct result is V ± V = (1.7 ± 15. The value of any trignomatric function is a dimen-
0.4) cm3, which is choice (c). sionless number. Hence choice (a) is correct. The
13. - argument of a trignometric function is also dimen-
sionless. Hence (bt – cx) is dimensionless. Hence b
value is has dimension [T–1] the same as that of frequency
Rs= R1+ R2= 3.0 + 6.0 = 9.0 and c has dimension of [L–1]. Thus choices (b), (c)
Error in Rs= Rs= R1+ R2= 0.2 + 0.1 = 0.3 and (d) are all correct.

III

Multiple Choice Questions Based on Passage


physical quantities of a given system. This method has its
Passage-I own limitations. In a complicated situation, it is often not
easy to guess the factors on which a physical quantity will
The dimensional method is a very convenient way of depend. Secondly, this method gives no information about
the dimensionless proportionality constant. Thirdly, this
1.20 Comprehensive Physics—JEE Advanced

method is used only if a physical quantity depends on the


product of other physical quantities. Fourthly, this method least count of the measuring instrument used. The number
will not work if a physical quantity depends only on
degree of precision of that measurement. The importance
Finally, this method does not give complete information
in cases where a physical quantity depends on more than calculation cannot increase the precision of a physical
three quantities in problems in mechanics.
1. The dimensional method cannot be used to obtain in the sum or product of a group of numbers cannot be
denpendence of greater than the number that has the least number of
(a) the height to which a liquid rises in a capil-
lary tube on the angle of contact weakest link.
(b) speed of sound in an elastic medium on the
3.
modulus of elactricity.
0.0123 kg. What is the total mass supported by the
(c) height to which a body, projected upwards
-
with a certain velocity, will rise on time t.
(d) the decrease in energy of a damped oscillator ures?
on time t.
(c) 0.0124 kg (d) 0.012 kg
2. In dimensional method, the dimensionless propor-
tionality constant is to be determined 4. The radius of a uniform wire is r = 0.021 cm. The
value is given to be 3.142. What is the area of
(b) by a detailed mathematical derivation -
(c) by using the principle of dimensional homo-
geneity. (a) 0.0014 cm2 (b) 0.00139 cm2
(d) by equating the powers of M, L and T. 2 2

5. A man runs 100.5 m in 10.3 s. Find his average


Passage-II
In the study of physics, we often have to measure the (a) 9.71 ms–1 –1
–1 –1
physical quantities. The numerical value of a measured

ANSWERS AND SOLUTIONS


1. The correct choices are (a), (c) and (d). The the sum must be rounded off to the fourth decimal
height of a liquid in a capillary tube depends on place. Therefore the correct choice is (c).
cos , where is the angle of contact. The height 4. A = r2 = 3.142 (0.021)2 2
. Now,
1
S to which a body rises is given by S = ut + at2,
2 -
1
which is a sum of two terms ut and at2. The A = 0.0014 cm2, which is choice (a).
2 100.5 m
- –1
5. Average speed =
tially with time. 10.3 s
2. The correct choices are (a) and (b).
3. the time has only three. Hence the result must

- 9.71 ms–1. Thus the correct choice is (a).


curate only upto the fourth decimal place. Hence
Units and Dimensions 1.21

IV

Matrix Match Type

1. Match the physical quantities in column I with their SI units in column II


Column I Column II
(a) Stefan’s constant (p) JK–1 mol–1
(b) Universal gas constant (q) Fm–1
(c) Electrical permittivity (r) Hm–1
(d) Magnetic permeability (s) Wm–2 K–4
ANSWER
1. (a) (s) (b) (p)
(c) (q) (d) (r)
2.
Column I Column II

(b) 0.034 (q) 4


(c) 0.002504 (r) 2
(d) 1.25 1.07 (s) 5
ANSWER
2. (a) (s) (b) (r)
(c) (q) (d) (p)
3. Match the quantities in column I with their order of magnitude given in column II
Column I Column II
(a) 2.6 104 (p) 105
(b) 3.9 104 (q) 10–23
10–24 (r) 10–24
(d) 4.2 10–24 (s) 104
ANSWER
3. x = 4.3 103, take its loga-
rithm to the base 10. Log x = 3.633 and round if off as log x = 4 . So the order of magnitnde of x is 104.
(a) (s) (b) (p)
(c) (r) (d) (q)

4.
(Q) given in column II.
Column I Column II
(a) Angular momentum (p) M L–2 T–2
(b) Latent heat (q) M L2 Q–2
(c) Torque (r) M L2 T–1
(d) Capacitance (s) M L3 T–1 Q–2
1.22 Comprehensive Physics—JEE Advanced

(e) Inductance (t) M–1 L–2 T2 Q2


(f) Resistivity (u) M0 L2 T–1

ANSWER
4. (a) (r) (b) (u)
(c) (p) (d) (t)
(e) (q) (f) (s)

SOLUTION
(a) Angular momentum L = r p = r (mv)
[L] = [L M L T–1] = [M L2 T–1]
Q heat energy [M L2 T 2 ]
(b) Latent heat = = = [M0 L2 T–2]
m mass [M]
(c) Torque = r F
[ ]= [L M L T–2] = [M L2 T–2]
Q Q Q2
(d) Q = CV C=
V W /Q W
Q2
[C] = = [M–1 L–2 T2 Q2]
[M L2 T 2 ]
LdI W LQ
(e) |V | =
dt Q T2
WT 2 [M L2 T 2 ] [T 2 ]
[L] = = [M L2 Q–2]
Q2 Q2
RA VA WA [M L2 T 2 L2 ]
(f) = = M L3 T–1 Q–2
L IL QIL Q
Q L
T

5. Column I gives three physical quantities. Select the appropriate units given in Column II.
Column I Column II
(a) Capacitance (p) Ohm second
(b) Inductance (q) (coulomb)2 (joule)–1
(r) coulomb (volt)–1
(c) Magnetic induction (s) newton (ampere metre)–1
or
–1

IIT, 1990
SOLUTION
unit of Q
5. (a) From Q = CV, unit of C = = coulomb (volt)–1
unit of V
Q2
From U = , unit of C = (coulomb)2 (volt)–1
2C
Units and Dimensions 1.23

dI
(b) From |e| = L , unit of L = volt second (ampere)–1 = ohm second
dt
(c) From F = BIL, unit of B = newton (ampere metre)–1
Hence (a) (q), (r)
(b) (p), (t)
(c) (s)
6. Some physical quantities are given in Column I and some possible SI units in which these quantities may be

indicate your answer by darkening appropriate bubbles in the 4


Column I Column II
(a) GMeMs (p) (volt) (coulomb) (metre)
G – universal gravitational constant,
Me – mass of the earth,
Ms – mass of the Sun
3RT
(b) (q) (kilogram) (metre)3 (second)–2
M
R – universal gas constant,
T – absolute temperature,
M – molar mass
F2
(c) (r) (metre)2 (second)–2
q2 B2
F – force, q – charge,
B
GM e
(d) (s) (farad) (volt)2 (kg)–1
Re
G – universal gravitational constant
Me – mass of the earth,
Re – radius of the earth
IIT, 2007
SOLUTION
Gm1m2
6. (a) F = . Therefore the SI unit of G is Nm2 kg–2.
r
SI unit of GMeMs = (Nm2kg–2) kg2 = Nm2 = kg ms–2 m2 = kg m3 s–2.
3RT SI unit of PV SI unit of work
(b) SI unit of = = = Nm kg–1 = kg ms–2 m kg–1 = m2 s–2
M SI unit of M kg
F F2
(c) From F = q v B = SI unit of v. Hence SI unit of 2 2 = (ms–1)2 = m2 s–2
qB q B
GM e Nm 2 kg 2 kg kgms 2
m 2 kg 2 kg
(d) SI unit of = = = m2 s–2
Re m m
(p) Since volt coulomb = work, SI unit of (volt) (coulomb) (metre) = SI unit of work metre = Nm m
= Nm2 = kg ms–2 m2 = kg m3s–2
coulomb
(s) Since farad = and coulomb volt = work, the SI unit of (farad)(volt)2 (kg–1) = (coulomb) (volt)
volt
kg–1 = SI unit of work kg–1 = kg ms–2 m kg–1 = m2s–2
1.24 Comprehensive Physics—JEE Advanced

Hence the correct choices are as follows (c) (r), (s) (d) (r), (s)
(a) (p), (q) (b) (r), (s)

Assertion-Reason Type Questions


In the following questions, Statement-1(Assertion) is Statement-2
followed by Statement-2 (Reason). Each question has The smaller the least count the greater is the num-
the following four options out of which only one choice
is correct. 2. Statement-1
(a) Statement-1 is true, Statement-2 is true and State- The dimensional method cannot be used to obtain
the dependence of the work done by a force on
(b) Statement-1 is true, Statement-2 is true but State- the angle between force and displacement x.
ment-2 is not -
ment-1. Statement-2
(c) Statement-1 is true, Statement-2 is false. All trignometric functions are dimensionless.
(d) Statement-1 is false, Statement-2 is true. 3. Statement-1
1. Statement-1 The mass of an object is 13.2 kg. In this measure-
The order of accuracy of measurement depends on
the least count of the measuring instrument. Statement-2

SOLUTIONS
1. The correct choice is (b). 3. The correct choice is (c). The degree of accuracy
2. Work done is W = F x cos . since cos is di-
mensionless, the dependence of W on cannot be measurement cannot be increased by changing the
determined by the dimensional method. Hence, the unit.
correct choice is (a)
2
Chapter
Motion in One Dimension

REVIEW OF BASIC CONCEPTS

2.1 SCALAR AND VECTOR QUANTITIES


2.1
A scalar quantity has only magnitude but no direction, such
A particle moves from A to B along a circle of radius
as distance, speed, mass, area, volume, time, work, energy,
R -
placement in terms of R and
A vector quantity has both magnitude and direction,
such as displacement, velocity, acceleration, force,

POSITION VECTOR AND DISPLACEMENT


2.2 VECTOR
The position vector of a particle describes its instantaneous
position with respect to the origin of the chosen frame of

and is denoted by vector r Fig. 2.2


For one-dimensional motion (say along x-axis), r = x i ,
where x is the distance of the particle from origin O SOLUTION
For two-dimensional motion (say in the x–y plane), Path length = arc AB = R
r = xi y j , where (x, y) are the x and y coordinates of

For three-dimensional motion, r = x i yj zk


Displacement vector
r1 is the position vector of a particle at time t1, and r2 at
time t2, then the displacement
vector is given by
s = r2 – r1
Vector s is the resultant of Fig. 2.3
vectors r2 and – r1
The displacement vector
s = AB = AC + CB
= 2 AC
particle after a given interval Fig. 2.1 s = 2R sin ( /2)
2.2 Comprehensive Physics—JEE Advanced

INSTANTANEOUS VELOCITY AND v = u + at


2.3 AVERAGE VELOCITY 1 2
x = x0 + ut + at
2
The rate of change of displacement with time at a given
1 2
instant is called instantaneous velocity and is given by or s = ut + at
2
dx
v= and v2 – u2 = 2 a(x – x0)
dt
or v2 – u2 = 2a s
total displacement
vav =
time interval in a straight horizontal direction, we will consider
only the magnitudes of u, v, a and s and take care of
2.2 their direction by assigning positive or negative sign
a will mean accelera-
The position of a particle moving along x-axis is
tion – a
given by x = 2t t2 + t , where x is in metre and t
-
(a) Find the velocity of the particle at t
(b) Find the average speed of the particle in the time Quantities directed vertically upwards are taken to
interval from t = 2 s to t be positive and those directed vertically downwards

SOLUTION to gravity is directed downward for a body moving


(a) x = 2t t2 + t vertically up or falling vertically down, we take a
–2
dx =–g
v= = 2 – 6t t2 nth second is given by
dt
sn = displacement in n seconds – displacement in
v at (t = 2 s) = 2 – 6 (2)2 = 2 ms–1
(n – 1) seconds
(b) Position at t = 2 s is 1 1
= un + a (n) 2 u (n 1) a (n 1) 2
x1 = 2 (2)2 + (2) = 0 2 2
Position at t a
2
sn = u + (2n 1)
x2 = 2 2
x2 – x1 Applications

1
from A with initial velocity u and reaches B with a
Average velocity = 12 ms velocity v, then the velocity midway between A and
2
B is
2.4 INSTANTANEOUS ACCELERATION u 2 v2
v =
The rate of change of velocity with time at a given instant 2
is called instantaneous acceleration and is given by (ii) A body starting from rest has an acceleration a for
dv a time t1 and comes to rest under a retardation b for
a= a time t2 s1 and s2 are the distances travelled in
dt
t1 and t2,
EQUATIONS OF ONE DIMENSIONAL s1 b t1
2.5 MOTION WITH CONSTANT ACCELERATION (a)
s2 a t2
Let x0 be the position of a particle at time t = 0 and let u be 1 ab
(b) Total distance travelled (s1 + s2) = T2,
its velocity at t a for 2 a b
time t where T = t1 + t2
x and acquires a velocity v ab
The particle suffers a displacement s = x – x0 in time t (c) Maximum velocity attained is vmax = T
a b
The equations of motion of the particle are
2.3

(d) Average velocity over the whole trip is (f) the speed with which the body hits the ground is
v
vav = max v= 2 g a h u
g a
2 g a
(iii) At time t = 0 a body is thrown vertically upwards
with a velocity u At time t = T, another body is NOTE
thrown vertically upwards with the same velocity Note that t1 is less than t2
u
T u Some useful tips
t=
2 g
(iv) A body is dropped from rest and at the same time …… are in the ratio 12 : 22 2
another body is thrown downward with a velocity
u from the same point, then acceleration, the distance covered by it in the lst,
(a) the acceleration of each body is g,
(b) their relative velocity is always u,
x
(c) their separation will be x after a time t = air resistance is nelected, the speed with which it
u
(v) From the top of a building, body A is thrown up- u, the
wards with a certain speed, body B is thrown down- maximum height attained is proportional to u2 and
wards with the same speed and body C is dropped the time of ascent is proportional to u
t1, t2 and t are (v) For a freely falling body,
their respective times of reach the ground, then (a) velocity time
t = t1 t2 (b) distance fallen (time)2
(vi) A body of mass m is dropped from a height h on a (c) velocity distance fallen
x
(a) the average retardation in sand is given by
2.6 GRAPHICAL REPRESENTATION
gh
a=
x Displacement – time (x – t) graphs (Fig. 2.4)
because loss in PE (mgh) = work done against
the resistive force of sand (max
(b) total average force exerted by sand is a > 0)
a < 0)
F = mg + ma = m(g + a)
-
(vii) A body is thrown vertically upward with a velocity ating
u
a constant acceleration (or retardation) a (< g)
(a) the net acceleration during upward motion
= g + a,
(b) the net acceleration during downward motion
= g – a,
(c) the maximum height attained is
u2
h=
2 g a
(d) the time taken to reach the maximum height is
2h u Fig. 2.4
t1 =
g a g a
NOTE
(e) the time of descent is
The slope of x – t graph gives velocity for uniform mo-
2h u
t2 = 1/ 2
g a g2 a2
2.4 Comprehensive Physics—JEE Advanced

v – t) graphs for uniformly acceler-


2.3
The displacement x (in metres) of a body varies with
time t (in seconds) as
2 2
x = t 16t 2

How long does the body take to come to rest?


O t O t O t
a o u o a o u o a o SOLUTION

dx
v= = t 16
dt
O O O
t t t 0= t 16 t = 12 s
–u
u o a o u o a o u o a o
2.4

vertically upwards with a velocity of 10 ms –1


O
t
(a) After how long will the ball hit the ground?
–u (b) After how long will the ball pass through the point
c
u o a o will it hit the ground? Take g = 10 ms–2

Fig. 2.5 SOLUTION


(a) s u = + 10 ms –1, a = – 10 ms–2
NOTE
1 2
Acceleration = slope of (v – t) graph Now s = ut + at
2
v – t) graph
1
t+ (– 10)t2
a–t 2
t2 – 2t
(t + 2) (t
t t is not

(b) s = 0, u = + 10 ms –1 and a = – 10 ms –2
0 = 10t t2 t=2s
Fig. 2.6 –1
(c) v = u + at = 10 – 10
a – t), (v – t), (x – t The negative sign indicates the velocity v is di-

2.5

-
elled by the body in 20 s, (b) the displacement of the
body in 20 s and (c) the average velocity in the time
Fig. 2.7 interval t = 0 to t
2.5

29a 29 2
S = u m
2 2

2.7 RELATIVE VELOCITY IN ONE DIMENSION


A and B are moving in a straight line with
velocities vA and vB respectively, the relative velocity of
A with respect to B
vAB = vA – vB
The relative velocity of B with respect to A will be
Fig. 2.8
vBA = vB – vA

SOLUTION 2.7
(a A police van moving on a highway with a speed of 10
ms –1
–1
–1
Speed , with what speed will
(ms–1) 6

SOLUTION
5 10 15 20 ( ) vV = 10 ms–1
–1
car is vC
–1
Fig. 2.9 respect to van is vBV vB be the veloc-
vBV = vB – vV vB = vBV + vV =
= area of OAB + –1

area of BCD a speed


–1
1 1 vBC = vB – vC
= 6 10 6 10 60 m
2 2
(b OAB – area of 2.8
BCD From the top of a tower 60 m tall, a body is thrown
vertically down with a
velocity of 10 ms –1
displacement same time, another body is
(c) Average velocity = 0
time thrown vertically upward
from the ground with a
2.6 velocity of 20 ms –1
how long will the two bod-
A body moving in a straight line covers a distance of
ies meet? (b) At what height
above the ground do they
meet? Take g = 10 ms –2
SOLUTION Fig. 2.10
a
Sn = u 2n 1 SOLUTION
2
(a C and let t be the
9a
S = u (1)
2
For body 1: s = – h1 , u1 = – 10 ms –1, a = – 10 ms –2
a
S = u (2) 1
2 – h1 = – 10t + 10 t 2
2
a = 2 ms –2 and u=
–1 h1 t (t + 2) (1)
2.6 Comprehensive Physics—JEE Advanced

For body 2: s = + h2, u2 = + 20 ms –1, a = – 10 ms –2 v t


h2 = 20t t2 t (– t dv = a dt
0
Adding (1) and (2), h1 + h2 t or 60 = u

t t and obtain the expression for v(t


dx
(b) Using t h2 x, use v = which gives
= 20 m dt
dx = v(t) dt
Alternative method v and integrate both
sides
–1
u12 = u1 – u2 x t
dx = v dt
x0 0
s12 = – h1 – (h2) = – (h1 + h2) = – 60 m
a12 = a1 – a2 = – g – (– g 2.10
Using
A particle starts from rest at x
1 2 time t –2
which varies with time as shown
s = ut at t
2 -
t ticle and (b) its displacement in time interval from t
h2 = 20t t 2 = 20 (2)2 = 20 m = 0 to t
–2
5ms
2.9
–1
The driver of train A
sights another train B moving on the same track at a
speed of 10 ms –1 -
ately applies brakes and achieves a uniform retarda- 6s
tion of 2 ms –2
Fig. 2.11
minimum distance between trains A and B when the
driver of A sights B? SOLUTION
2
SOLUTION (a) m = ms
6
A B is, c –2
y = mx + c, the
–1
uAB = uA – uB acceleration a varies with time as
A B is a= t
6
aAB = aA – aB = – 2 – 0 = – 2 ms –2
dv
= t
A dt 6
B vAB v t
of A B is sAB which is found by using the relation dv = t dt
v2 – u2 = 2 as which gives 0 0
6
0 – (20)2 = 2 (– 2) sAB sAB = 100 m v= t2 t (1)
12
SOLVING PROBLEMS INVOLVING NON-
2.8 UNIFORM ACCELERATION zero at t v will be maximum at t
–1
Using t vmax
(a) Finding velocity and displacement if the depen- dx
dence of acceleration on time is given. (b) From v = , we have dx = v dt
dt
dv
Use a = which gives dv = a dt x t 2
dt
given expression for a in terms of t and integrate dx = v dt = t2 t dt
0 0 0
12
2.7

2 dv dv
x=
t t a= dt
12 2 dt a
0

2 2 v t
= = m dv
12 2 9 = dt
a
(c) Finding velocity and displacement if the depen- u 0

dence of acceleration on displacement is given Hence we obtain the expression for v(t
dv dx d v dv dx
Use a(x) = = =v v= dx = vdt
dt dt dx dx dt
dx
v
dt x t
or v dv = a(x) dx dx = v dt
x0 0
a(x) in terms
of x Hence we get an expression for x
v x
v dv = a x dx 2.12
v x0 t = 0) moving with a veloc-
Hence we get an expression for v(x) in terms ity u -
of x ates it at a rate a = k v where v is the instanta-
dx
v(x) = neous velocity and k
dt
dx
dt and integrate
v x
x t SOLUTION
dx
= dt
v (x) dv dv dv
x0 0 (a) a = k v = = – k dt
dt dt v
where v

2.11 v
dv
t
= – k dt
A particle is moving along the x-axis with an accel- v 0
eration a = 2x where a is in ms –2 and x
u
1/ 2 1/ 2
the particle starts from rest at x 2 v u = – kt
when it reaches the position x 2 u
Putting v = 0, we get t =
k
SOLUTION

dv dv dv dv
a =v 2x = v v dv = 2x dx a=
= v
dx dx dt dx
v dv v dv
k v = v dx =
v d v = 2 x dx dx k
0 1
0
v2 x2
x
2 1
= 2 dx = v dv
2 2 1 0
k u
–1
which gives v 2u /
x=
(c) Finding velocity and displacement if the depen- k
dence of acceleration on velocity is given
2.8 Comprehensive Physics—JEE Advanced

Multiple Choice Questions with Only One Choice Correct


1.
k k
last second of its fall, the ball covers a distance (a) (b)
g = 10 ms–2, 2 2
the height of the tower is k2
(c) (d) 2 k2
2
7. The velocity of a particle at time t (in second)
2. A ball is thrown vertically upwards from the foot of
is related to its displacement x (in metre) as
v= x
(a) 1 ms–1 (b) 2 ms–1
–1 –1
Take g = 10 ms–2
8. A car, starting from rest, has a constant accelera-
–2
(c) 100 m (d) 120 m for some time and then has a con-
3. Two cars travelling on a straight road cross a ki- stant retardation of 2 ms–2 for some time and
lometer stone A at the same time with velocities
20 ms–1 and 10 ms–1 with constant accelerations of The maximum velocity of car during its motion is
1 ms–2 and 2 ms–2 (a) 12 ms–1 –1

kilometer stone B at the same instant, the distance –1


(d) 21 ms–1
between A and B is 9. A freely falling body, falling from a tower of height
h covers a distance h/2 in the last second of its mo-
(c) 1000 m (d) 1200 m The height of the tower is (take g = 10 ms–2)
4. The acceleration a of a body moving with initial nearly
velocity u changes with distance x as a = k2 x ,
where k -
elled by the body when its velocity becomes 2u is 10. A
/ / ms–1 towards a bigger ball B
u u collision with ball B, ball A retraces the path and
(a) (b) 2k –1
2k
A during the
/ /
u u time interval 0 to 6 s?
(c) (d) (a) zero (b) 2 ms–1
2k 2k
–1 –1
5. A particle is moving along the x - 11. –1
neous velocity when it is at a distance x from the A monkey is running northwards on the roof of the
origin is v = p q x 2 , where p and q are positive –1

of the monkey as observed by a person standing on


instant is the ground?
–1
2q x in the southward direction
(a) zero (b) –1
( p q) in the northward direction
–1
2px in the southward direction
(c) (d) – q x –1
in the northword direction
( p q)
12. A police van moving on a highway with a speed
–1
6. The velocity of a particle moving along the x-axis -
is given by v = k x where k ing away in the same direction with a speed of
–1
The acceleration of the particle is
2.9

–1
, with what speed will the bullet hit the 19. (x-t)

(a) 120 ms–1 –1


–1 –1
the velocity-time (v-t) graph of the motion of the
–1
13. Car A on a
B and C, each moving with
–1
in opposite directions on the
other lane are approaching car A -
stant when the distance AB = distance AC = 1 km,
the driver of car B decides to overtake A before C

car B so as to avoid an accident?


(a) 1 ms–2 (b) 2 ms–2
–2 –2

14. The driver of a train A Fig. 2.12


ms–1 sights another train B moving on the same
track towards his train at a speed of 10 ms–1
immediately applies brakes and achieves a uniform
–2

what must be the minimum distance between the


trains?
(a) 100 m (b) 200 m

15.
g = 10 ms–2, the

16. A body, starting from rest, moves in a straight line


with a constant acceleration a for a time interval t
during which it travels a distance s1
move with the same acceleration for the next time
interval t during which it travels a distance s2
relation between s1 and s2 is
(a) s2 = s1 (b) s2 = 2s1 Fig. 2.13
(c) s2 s1 (d) s2 s1 20. g
17. v1 is the velocity of the body at the end = 10 ms–2, what is the ratio of the distances trav-
v2 that at the end of the
second time interval, the relation between v1 and motion?
v2 is (a) 1 : 9 (b) 2 : 9
(a) v2 = v1 (b) v2 = 2v1
(c) v2 v1 (d) v2 v1 21.
–1
g = 10 ms–2, what is the ratio
18. A body dropped from the top of a tower hits the
of the distances travelled by the bullet during the

(a) 9 : 1 (b) 9 : 2
the tower?
(a) 1 s (b) 2 s 22. A body moving in a straight line with constant
(c) 2 2 s (d) s acceleration of 10 ms–2
2.10 Comprehensive Physics—JEE Advanced

th
v
th (c) (d) v
in the 6 second? 2 2
29. The displacement of a body from a reference point,
is given by
23. A body, moving in a straight line with an initial x = 2t
–1
and a constant acceleration, cov- where x is in metres and t
rd
that the body is
distance will it cover in the next 2 seconds? (a) at rest (b) accelerated
(c) decelerated (d) in uniform motion
(c) 90 m (d) 100 m 30.
24. A body, moving in a straight line, with an initial (a) 2 ms–1 –1
–1
velocity u and a constant acceleration a, covers a (c) 6 ms (d) 12 ms–1
th
second and a distance 31.
of 60 m in the 6th u and a (a) 2 ms–2 –2
–2 –2
respectively are (c) 6 ms
(a) 10 ms–1 –2
(b) 10 ms–1, 10 ms–2 32. A car, starting from rest, at a constant acceleration
–1 –2 –1
, 10 ms–2 covers a distance s1 in a time interval t -
25. A car, starting from rest, has a constant accelera- tance of s2 in the next time interval t at the same ac-
tion a1 for a time interval t1 during which it cov-
ers a distance s1 t2, the (a) s2 = s1 (b) s2 = 2s1
car has a constant retardation a2 and comes to rest (c) s2 s1 (d) s2 s1
after covering a distance s2 in time t2 33. A car moving at a speed v is stopped in a certain
following relations is correct? distance when the brakes produce a deceleration
a1 s2 t1 a nv, what must be
a1 s1 t1
(a) (b) the deceleration of the car to stop it in the same
a2 s2 t2 a2 s1 t2 distance and in the same time?
a1 s1 t2 a1 s2 t2 (a) na (b) na
(c) (d)
a2 s1 t1 2
a2 s2 t1 (c) n a (d) n a
26. 34. Two balls are dropped from the same point after an

1 1 1
(a) a1t1 = a2 t2 (b) (a1 t1 a2 t2 ) g = 10 ms–2
2 2 2
1
(c) a1 a2 t1 t2 (d) zero 35.
–1
h1 during
27. s, h2
the maximum speed attained by it will be g = 10 ms–2, h1
1 1 and h2 will be related as
aa 2 aa 2
(a) 2s 1 2 (b) 2s 1 2 (a) h1 h2 (b) h1 = 2h2
a1 a2 a1 a2 h2
1 1
(c) h1 = h2 (d) h1 =
s a1a2 2 s a1a2 2
(c) (d) 36. A ball is thrown vertically downward with a veloc-
2 a1 a2 2 a1 a2 ity u
28. A car, starting from rest, is accelerated at a constant u
rate until it attains a speed v reach the ground is given by
a constant rate u 2u
(a) (b)
g g
v u u
(a) zero (b) (c) (d)
2 g g
2.11

37.
u2 2u 2
(a) (b)
g g
u2 u2
(c) (d)
g g
38. -
–1
and leaves it at a
–1

train?
–2 –2
–2 –2

39.
platform?

(c) 10 s (d) 12 s
40. The motion of a body is given by the equation
dV t
V(t)
dt
where V(t) is the speed (in ms–1) at time t (in sec-
t = 0, the magnitude
of the initial acceleration is
–2
(b) 6 ms–2
–2
(c) 9 ms (d) zero
41.
(a) V(t) = (1 – e t )
(b) V(t) = 2 (1 – e t)
t
2
(c) V(t) = 1 e 2

2t
(d) V(t) = 1 e
2
42. -
tion is half the initial value is
(a) 1 ms–1 (b) 2 ms–1
–1 –1

43. Two stones are thrown up simultaneously with


initial speeds of u1 and u2 (u2 > u1

variation of x = (x2 – x1), the relative position Fig. 2.14

t = 10 s? Assume that the stones do not rebound 44. A body starts from rest at time t = 0 and undergoes
2.12 Comprehensive Physics—JEE Advanced

45.
+3 time t
–1 –1
Acc (ms 2)

0 –1 –1
1 2 3 4
3
Time (s) 46.
from t = 0 to t
Fig. 2.15 (a) 6 m (b) 9 m

the velocity-time (v–t) graph of the motion of the 47.


body from t = 0 s to t represents the displacement-time (x – t) graph of
the motion of the body from t = 0 s to t

Fig. 2.17

48. A body, moving in a straight line, covers half the


distance with a speed V, the remaining part of the
distance was covered with a speed V for half the
time and with a speed V for the other half of the

2V V V V V V
(a) (b)
2V V V 2V V V
2V V V V
(c) (d)
V V V V V V
49. A particle moving in a straight line covers half the

the distance is covered in two equal time inter-


-

Fig. 2.16 motion is


2.13

motion and air resistance, its velocity v varies with


the height h

50. v – t) graphs

(a) (i), (ii) and (iv) only


(b) (i), (ii) and (iii) only
(c) (ii) and (iv) only
(d) all

Fig. 2.19

54. A car, starting from rest, accelerates at a constant


–2

constant rate of 10 ms–2


Fig. 2.18
attained by the car?
–1
51. A stone dropped from a building of height h reaches (b) 10 ms–1
–1 –1
the ground after t - (c) 20 ms
ing if two stones are thrown (one upwards and 55.
the other downwards) with the same velocity u car is 6 s?
and they reach the ground after t1 and t2 seconds
respectively, then the time interval t is (c) 100 m (d) 120 m
t1 t2 56. The distance x covered by a body moving in a
(a) t = t1 – t2 (b) t = straight line in time t is given by the relation
2
2x2 x=t
(c) t = t1 t2 (d) t = t12 t22 v is the velocity of the body at a certain instant
52. x) of a particle is related to time (t) of time, its acceleration will be
as x = at + bt2 – ct (a) – v (b) – 2v
where a, b and c v v
velocity of the particle when its acceleration is zero 57. The distance x covered by a body moving in a
is given by straight line in time t is given by
2
b2 x2 = t2 + 2t
(a) a + b (b) a + The acceleration of the body will vary as
c 2c
1 1
b2 b2 (a) (b) 2
(c) a + (d) a + x x
c c
1 1
(c) (d)
x x
53. A ball is dropped vertically from a height h above
58. A body is thrown vertically up with a velocity u
vertically to a height h passes three points A, B and C
2.14 Comprehensive Physics—JEE Advanced

u u u
with velocities , and average velocity of the particle is
AB 2 –1
is –1
BC
–1
20
(a) (b) 2 (d) zero
10
(c) (d) 1 65. A body of mass m1 -
cally upwards with an initial veloc- Fig. 2.20
59. A body is thrown vertically up with a velocity u
ity u reaches a maximum height h
passes a point at a height h above the ground at time
Another body of mass m2 -
t1 while going up and at time t2
-
Then the relation between u, t1 and t2 is
zontal and with speed u
2u 2u travelled along the incline is
(a) t1 + t2 = (b) t2 – t1 =
g g (a) 2h (b) h
u u h
(c) t1 + t2 = (d) t2 – t1 = (c) (d) h
g g 2
60. t1, t2 and h is 66. The displacement x of a particle moving in one
dimension is related to time t by the equation
2h h
(a) t1t2 = (b) t1t2 =
g g t= x
where x is in metres and t -
2h h
(c) (t1 + t2)2 = (d) (t1 + t2)2 = ment of the particle when its velocity is zero is
g g
61. A body dropped from a height H above the ground 67. t = 0) moving with a
strikes an inclined plane at a height h above the velocity u
result of which it decelerates at a rate
a=–k v
the maximum time to reach the ground if where v is the instantaneous velocity and k is a
(a) h =
H
(b) h =
H T taken by the particle
2 2 to come to rest is given by
H H 2 u 2u
(c) h = (d) h = (a) T = (b) T =
2 2 k k
62. A body of density enters a tank of water of density 2u / 2u 2
after falling through a height h (c) T = (d) T =
k k
depth to which it sinks in water is
68.
–2
h h t which varies with time as shown
(a) (b)
( ) ( )
will be
h h
(c) (d) ms–1 ms–1
ms–1 ms–1
63. A body, falling freely under gravity, covers half the

for n seconds, then the value of n is

(c) 2 – 2 (d) 2 + 2
64. A to point B,
Fig. 2.21
2.15

69. v) of 71. a – t)
a body with position (x) from the origin O
v – t)
variation of the acceleration (a) with position (x)? of the motion of the body? Assume that x = 0 and
v = 0 at t
v
v

Fig. 2.22

Fig. 2.25

v v
(ms–1) (ms–1)

0 0
2 4 ( ) 2 4 ( )

(a) (b)

Fig. 2.23
v v
(ms–1) (ms–1)
70. The velocity (v) of a body moving along the postive
x-direction varies with displacement (x) from the
0 0
origin as v = k x , where k 2 4 ( ) 2 4 ( )
the graphas show
the displacement-time (x – t) graph of the motion? (c) (d)

Fig. 2.26

72. -
val t = 2s to t
(a) 2 m (b) – 2 m

73. A particle of mass m moving with initial velocity u


enters a medium at time t
a resistive force F = kv wher k is a constant of the
medium and v
velocity of the particle varies with time t as
kt kt
(a) v = u + (b) v = u –
m m
kt/m
(c) v = u e (d) v = u ekt/m
74. x of the particle varies with
Fig. 2.24 time t as (assume that x = 0 at t = 0)
2.16 Comprehensive Physics—JEE Advanced

mut u
(a) x = 75. at
k time t given by 2
mu 1 2 m m
(b) x = t t (a) t = ln(2) (b) t = ln(2)
k 2 k 2k
mu kt/m m m
(c) x = 1 e (c) t = (d)
k k 2k
mu kt/m
(d) x = 1 e
2k

ANSWERS

1. (c) 2. (a) 3. (a) 4. (b) 5. (d) 6. (c)


7. (b) 8. (c) 9. (a) 10. (b) 11. (c) 12. (a)
13. (a) 14. (b) 15. (c) 16. (c) 17. (b) 18. (c)
19. (d) 20. (a) 21. (a) 22. (b) 23. (c) 24
25. (d) 26. (a) 27. (a) 28. (d) 29. (b) 30. (d)
31. (d) 32. (c) 33. (c) 34. (c) 35. (c) 36. (b)
37. (d) 38. (c) 39. (c) 40. (b) 41. (b) 42. (a)
43. (a) 44. (d) 45. (c) 46. (c) 47. (b) 48. (a)
49. (d) 50. (c) 51. (c) 52. (c) 53. (a) 54. (c)
55. (a) 56. (d) 57. (c) 58. (a) 59. (a) 60. (a)
61. (c) 62. (a) 63. (d) 64. (b) 65. (a) 66. (a)
67. (a) 68. (b) 69. (d) 70. (c) 71. (c) 72. (d)
73. (c) 74. (c) 75. (a)

SOLUTIONS
1. Let h be the height of the tower and t be the time 1 2
h = ut + gt
2
1 2 1
h= gt (1) 2+ (– 10) (2)2
2 2
= 60 m
1
and h = g (t – 1)2
2
9h 16 h
Now h = h – =

16 h 1
= g(t – 1)2 (2)
2
t
Fig. 2.27
1 2
h= 10 3. Let s be the distance between A and B
2
2. Let h = AB be the height of the tower and P be the 1 1
s = 20t + 1 t2 = 20t + t2 (1)
2 2
by the ball to go from B to P
time taken to go from A to P - For the second car,
fore, time taken by the ball to go from A to B is 1
s = 10t + (2) t2 = 10t + t2 (2)
t 2
u Equating (1) and (2), we get t
–1
0 = u – 10 u value of t in either (1) or (2) gives s
2.17

dv d v dx dv Hence h is given by
a= = = v 2
dt dx dt dx 1 1 1 h
h= gn2 = g 1
v d v = a dx = k2 x dx 2 2 g
2u 1 h 2 2h 1 h2
v d v = k2 x dx = g 1 = 2h g
g2 g g
u
u
/ g = 10 ms–2, we get
x= 2
h – 60h + 100 = 0
2k
The positive root of this quadratic gives h
dv d v dx dv
5. a = = = v
dt dx dt dx 20
10. The time taken by ball A to reach ball B =
dv d
= p – qx2)1/2 A covers a
dx dx
distance of 20 m upto ball B
1 –1
×2s
= (p – qx2)–1/2 (– 2 q x)
2
qx
= v = (p – qx2)1/2 12 m
v Average velocity = = 2 ms–1 which is
Hence a = – qx 6s
6. The acceleration is
dv d v dx dv 11.
a= = = v
dt dx dt dx –1
-
Given v= k x –1

d Therefore, the velocity of the monkey as observed


a= k x (k x ) –1
dx
The negative sign indicates that the direction of this
k k2
= k x =
2 x 2 –1
12. = 10 ms–1
7. Given v2 x v2 = u2 + 2ax,
2
we have u u = 2 ms–1
moving, the net speed of the bullet = speed of the
8. Let t1
The velocity at the end of t1 is –1
v = u + at1 t1 t1 –1 –1
–1
The time during which the car decelerates is and the
–1
t2 = (t – t1) where t is the total time taken for the car
t2, the initial velocity is the bullet will hit the car with a speed which is
v t1 the relative speed of the bullet with respect to the
–1
t1 – 2t2
t1 – 2 (t – t1)
t1 – 2t 13. Let us suppose that cars A and B are moving in the
positive x C is moving in the
2 2 –1
t1 = t = negative x vA
–1 –1 –1
= + 10 ms , vB and vC
The car attains the maximum velocity at the end of –1 –1
B
t1 –1
with respect to A is vBA = vB – vA
–1
vmax t1 The relative velocity of C with respect to A is vCA
–1
h 1 1 h = vC – vA t = 0, the
9. g (2n – 1) so that n = 1 distance between A and B = distance between A and
2 2 2 g
C C will cover a distance
2.18 Comprehensive Physics—JEE Advanced

AC A at a time t given for the next time interval t during which the body
by
AC 1000 m 1 2 1 2
t= = s2 = ut + at = at2 + at = at2 ( u = at)
vCA ms 1 2 2 2
Car B will overtake car A C does s2 s1
and avoid an accident, if it acquires a minimum 17. Here v1 = 0 + at = at and v2 = v1 + at = at + at =
acceleration a such that it covers a distance s = AB 2 at v2 = 2v1
= 1000 m in time t
–1
speed u = vBA 1 2
tion 18. Let h h= gt =
1 2 2
1 2 g g
s = ut + at 2
2
1 h
2 The time t taken to fall through g is given by
×a 2
2 h 1 2 1 2
= gt g= gt or t2 t= 2 2
which gives a = 1 ms–2 2 2 2
14. The relative speed of train A with respect to train B
–1
19. -
is given by ment x increases linearly with time t
2
=2 s velocity v is a positive constant between t = 0 and
which gives s t t t
15. v) is zero
between t t t
t = 20 s, displacement (x) decreases linearly with
(u) of the bullet is ( time (t v) is constant but
–1
u = gt = 10 negative between t t
maximum height (h) attained by the bullet is h
1 2 1 2 20.
= gt = 10 total time taken by the stone to hit the ground is
2 2
time taken by the bullet to return to the point of given by
1 2 2h 2
- h= gt or t =
2 g 10
1 2 1
tance of s1 = gt = 10 (2)2 h1
2 2
given by
1 g
h1 = g(1)2 =
t seconds, where t is the time taken by the bullet to 2 2
rise to a height h -
of t is given by tance h given by
1 1 1
h = ut – gt 2 t– 10 t2 h= g 2 g
2 2 2
or t2 – 10 t + 21 = 0 or (t t h2 through which the stone falls in the
which gives t

16. - Now h1/h2


t is
1 2 1 2 21. The maximum height h attained by the bullet is
s1 = 0 +at = at given by
2 2
The velocity of the body at the end of this time in- u2
v2 – u2 = – 2gh or h = ( v = 0)
terval is v = 0 + at = at 2g
2.19

or h = - t1 is given by
2 10
en by the stone to attain this height is given 2a1s1 = v2 – u2 = a12 t12 ( v = a 1t1 and u = 0)
u 1
by t = or s1 = a 1t 12 (i)
g 10 2
(t = 1 s), the stone covers a distance h1 given by The distance covered in the next time interval t 2 is
given by
1 2 1
h1 = ut – gt 1– 10 (1)2 – 2a2 s2 = 0 – a21 t21 ( v = 0 and u = a1t1 now)
2 2
s), the stone cov- 2 2
t 1 a12 2 1 a2 t2
ers a height h given by or s2 = t1 = ( a 1t 1 = a 2t 2)
2 a2 2 a2
1 2
1
h 10 = 120 m
2 or s2 = a 2 t22 (ii)
2
s1 a1 t12 a2 a12 t12
h2 From (i) and (ii) we get =
h1/h2 s2 a2 t22 a1 a22 t22
a2
22. The distance covered in the nth second is given by = ( a 1t 1 = a 2t 2)
a1
1
sn = u + a n s2 a1 t2
2 Thus we have
s1 a2 t1
1
s =u+a =u+ a total distance s1 s2
2 2 26. Average speed =
total time t1 t2
–1
u+ 10 or u 1 1
2 As shown above, s1 + s2 = a1 t12 + a 2 t22
1 2 2
s6 6 which gives
2 1
s6 = a1t1 (t1 + t2) ( a 1t 1 = a 2t 2)
2
1 1
23. Now, sn = u + a n a t t t
2 2 1 1 1 2
Average speed =
t1 t2
1
a which gives a = 10 ms–2 1 1
2 = a 1t 1 = a 2t 2
2 2
1
s
2
27. The maximum speed v attained by the car = speed
1 it attains at the end of time interval t1 during which
and s
2
s =s +s v = a 1t 1 = a 2t 2
1 v2
Now s1 = a1 t 12 = ( v = a 1t 1)
a 11a 2 2a1
24. u+ and 60 = u +
2 2
–1 1 v2
These equations give u and a = 10 ms–2 and s2 = a 2 t22 = ( v = a 2t 2)
2 2a2
25. - v2 1 1
t1 is v = 0 + s = s1 + s2 =
2 a1 a2
a 1t 1 = a 1t 1
1/ 2
time interval t2 a1 a2
have, from v = u + at, or v = 2s
a1 a2
0 = a1 t1 – a2 t2 ( u = a 1 t 1)
2.20 Comprehensive Physics—JEE Advanced

28. The distance s1 covered by the car during the time a* = (nv)2/2s = n2v2 a * = n 2a
it is accelerated is given by 2 s1 = v2, which gives
s1 = v2/2 s2 covered during the time
34. t1
the car is decelerated is, similarly given by s2 =
1 1
v2/2 falls a distance h1 = gt 12 = 10 2
2 2
v2 1 1 The second ball falls for t2
s = s1 + s2 = (i)
2 1 1
t1 is the time of acceleration and t2 that of decel- it falls a distance h2 = gt 22 = 10 2
=
2 2
eration, then v = t1 = t2 or t1 = v/ and t2 = v/ h1 – h2
Therefore, the total time taken is
1 1 35. The total time taken by the bullet to reach the high-
t = t1 + t2 = v (ii)
est point (where its velocity becomes zero) is given
From (i) and (ii), the average speed of the car is by 0 = u – gt or t = u/g
given by
total distance s v 1 2 1
h1 = ut – gt 1– 10 (1)2
total time t 2 2 2

–1
29. 2 seconds is v = u – gt
x = 12t t2 + 9
x changes with time t, the body
1 2
seconds is h2 10
by 2
h1 = h2
dx
v= t 36. From v = u + gt u = u + gt or 2u = gt or
dt t = 2u/g
v changes with time t, the body 37. From 2gh = v2 – u2, we have 2gh u)2 – u2 u2
is not in uniform motion; it is accelerated because or h 2
u /g
v increases with t 38. The total distance covered in order to cross the plat-
30. v t v form is s = length of train + length of platform =
= u + at u = 12 ms–1
–1
speed is u v
31. Now v t v = u + at, we ms–1 v2 – u2 = 2as – 2
–2 2 –2
a a is = 2a a which is
given by
–2
dv d 39. a
a= 12 t
dt dt the relation, v = u + at t or t
–2
or a
dV t
32. 40. Given V(t) (i)
t is v = at dt
distance covered during this time interval is s1 = t = 0, V(0) = 0 (given), the acceleration at
1 2 t = 0 is
at v = at is the initial velocity for the
2 dV t
next time interval t - a(0) = –2
elled in the next time interval t is dt t 0

1 2
s2 = a t2 + at = at2 41. V(t) with respect to t given in each
2 2
-
Thus s2 s1
pression for dV(t)/dt
33. The distance over which the car can be stopped is
dV t d d
given by 2 as = v2 or a = v2/2s v becomes nv, the = 21 e t
= 2 (1 – e t
) = 6e t

value a* of a to stop the car in the same distance is dt dt dt


2.21

= 6 – 6 (1 – e t
V (t) time interval = 6 ms–1
V(t) = 2 (1 – e t) (ii) time interval from t = 2 s to t
42. v = u + at t
–1
at time t * given Therefore, velocity at t s after t = 2 s)

and velocity at t
2 s after t
V(t *) or V(t*) = 1
velocity time graph from t = 2 s to t -
or 2(1 – e t*
) = 1 or e t*

t * ln e–1 t* The graph is again linear but its slope is negative


* BC in
or t
Putting this value of t
45. As shown above, the velocity of the body in time
V at t e ) interval t = 2 s to t v t
= 2(1 – e –1 At t v
–1

46. The distance covered = area under the velocity time


43.
graph from t = 0 to t ABC in
stones are given by
1 2
x 1 = u 1t – gt ( g is negative) =2 area of triangle ABD
2
1
1 2 =2 BD AD = BD AD = 6 ms–1 2 s = 12 m
and x2 = u2 t – gt 2
2
47. x–t)
stones:
x = x2 – x1 = (u2 – u1)t
x2 – x1) varies linearly with t, the graph is a t = 0 s and t
straight line upto t
At t t = 0 s to t = 2 s, the slope of the graph must increase
x1 = 0; so we have for time with t AB
between 6
1 2 t = 2 s to t
x = x 2 – x 1 = u 2t – gt
2
Thus (x2 – x1) versus t graph is not linear; it is a
BC of graph (c), the slope is increasing
Now at t = 10 s, x = 0, therefore
1
0 = 10u2 – g (10)2 48. Let the total distance be 2 s and let t1 and t2 be
2
–1
or u2 g
2
Hence x t t
t1 = s/V
t2
44. t = 0, u distance s1 covered in time with speed V is s1
–2 2 t2
t = 0 to t = 2 s, acceleration a =V
t2
and the distance s2 covered in time with
t = 0 to t = 2 s), 2 2
the velocity of the body is v = 0 + at or v t t2
–1
speed V is s 2 = V , so that
at t = 1s, v and at t = 2 s, v = 6 ms–1 2
from t = 0 to t = 2 s, the velocity–time graph is
t2 t2
AB of s = s1 + s2 = V V
2 2
For the next time interval from t = 2 s to t t2
acceleration a –2 = (V + V ) , which gives
2
initial velocity u
2.22 Comprehensive Physics—JEE Advanced

2s 1 2
t2 = h = – ut1 + gt1 (ii)
V V 2
s 2s For the stone thrown downwards with velocity u,
Total time taken = t + t2 =
V V V we have
1 2
s 2V V V h = ut 2 + gt (iii)
2
V (V V ) Notice that the displacement of the stone in the
total distance three case is the same, equal to h
Hence, average speed= and (iii) we have
total time
1 2 1 2 1 2 1 2
2s V (V V ) 2V V V gt = – ut1 + g t 1 ut1 = g t 1 – gt
= 2 2 2 2
s 2V V V 2V V V
1 2 1 2 1 2 1 2
and gt = ut2 + g t2 ut2 = gt – g t2
49. Let the total distance be S 2 2 2 2
S is
2 t1 t12 t2
t1 =
S /2
=
S t2 = t 2 t22
6
Let t2 be the time taken to cover a distance S1 with which gives t = t1 t2
t that to cover distance S 2 with
dx
52. Velocity is v = = a + 2bt ct2 (i)
Then S1 t2 and S 2 t dt
S d2 x
Now S1 + S2 = and t2 = t Acceleration is a = = 2b – 6ct
2
d t2
S
Therefore = S1 + S 2 t2 or Acceleration is zero at time t given by 0 = 2b – 6ct
2 b
S or t = t
t2 = c
S S S b b2 b2
Total time taken t = t1 + t2 = v = a + 2b c a
6 2 c 9c 2 c
total distance S
Average speed = = 53. The velocity at a height h is given by v 2 = u2 + 2gh
total time S/ For downward motion, u = 0 and the value of g
is negative and h becomes more and more nega-
v 2 increases with h
vector is directed downwards, v becomes more and
50. Motion corresponding to graph (i) cannot be re- v2 h, the graph of v versus
alised because if we draw a line parallel to the h is
v-axis, the body will have two different veloci- For upward motion, v 2 = u2 + 2gh g is
ties at a given value of t directed downwards and h -
Graph (iii) is also not possible because at some ly, v2 decreases with h
values of t, the graph is parallel to the v-axis which velocity vector is positive, v becomes less and less
- v with h is para-
responding to graphs (ii) and (iv) are possible in v becomes less and less positive, graph

51. For the stone dropped with zero initial velocity, we 54. Let t1 be the time during which the car has an accel-
–2
have eration a1 and t2 be the time during which
1 the car has a deceleration a2 = – 10 ms–2
h= gt 2 (i)
2 car starts from rest, the maximum speed attained
For the stone thrown upwards with velocity u, we by the car v = 0 + a1t1 = a1t1 t2
have this v
2.23

Therefore, 0 = v – a2t2 or v = a2 t2 a1 t1 = 1 2
a2 t2 or t1/t2 = a2/a1 t1 = 2t2 59. h = ut – gt
2
t1 + t2 = 6, we get t1 t2
2u 2h
55. The distance moved in t1 or t2 – t+ =0
g g
1
s1 = 0 × t1 + a1 t21 The roots of this quadratic equation are t1 and t2
2
The distance moved in t2 = 2s is 2u
The sum of the roots is t1 + t2 =
1 g
s2 = vt2 – a t2
2 2 2 2h
56. x2 x = t with respect to t we 60. Product of roots is t1 t2 =
g
have
dx dx 61. The time t1 taken by the body to strike the inclined
x =1 (i) plane is given by
dx dt dt
Now =v xv v x v 2( H h)
dt t1 =
g
t, we
have The time t2 taken by the body to reach the ground
2 after striking the plane is
dx d2x d2x 2h
x =0 t2 =
dt dt 2 dt 2 g
v2 xa a=0 2( H h) 2h
Total time t = t1 + t2 = + g
v2 g
or a= (ii)
x dt
2
Time t will be maximum if
d x dh
where a = x v
dt 2 d 2( H h ) 2h
a v = 0
dh g g
57. Given, x2 = t2 + 2t 2 1 1
1/ 2 1/ 2
or (H h) h = 0
dx dx g 2 2
2x = 2t + 2 or x =t+1 (i)
dt dt 1 1 H
or = or H – h = h or h =
H h h 2
2 2
dx d x 62. u= 2gh
x 2
dt dt g( )
2
Effective retardation in water is geff =
dx
or x2 x a = x2 (ii) v
dt
Find the maximum depth x from the relation
d2x v2– u2 = 2geff x
a= 2
we have dt 63. h is the total distance travelled in n seconds,
(t + 1)2 + x a = x2 then
1
or t2 + 2t + 1 + x a = x2 (iii) h= gn2
2
x2 = t2 + 2t n – 1)th second, the
h 1
a = 2/x distance fallen is h = = gn2
2
2 2 1
u u u2 u2 u2 1
58. – 2g(AB) = = =– (i) Hence g(n – 1)2 = h = gn2
2
2 2 or n2 n+2=0
u u u2 u2 u2
– 2g(BC) = = – =– (ii) The positive root of this equation gives the required
16 9 value of n
2.24 Comprehensive Physics—JEE Advanced

not displacement AB
64. Average velocity = = becomes zero at t
time t
particle will be maximum at t t=6s
65. For the body of mass m1, we have
u2
h= vmax = – (6)2 6
2g 12
–1
For the body of mass m2, if S is the maximum
distance travelled along the incline then
v2 – u2 = 2aS v0
69. The slope of the given v versus x graph is m = –
Now, when S is maximum, v a = – g x0
g and intercept is c = + v0 v varies with x as
sin = – g
2 v0
v=– x + v0 (1)
66 t= x x0
where v0 and x0 -
x = t2 – 6t + 9 (i)
tiating with respect to time t, we have
dx d 2
velocity v = = (t – 6t + 9) = 2 t – 6 (ii) dv v0 dx
dt dt =–
dt x0 dt
Find t v t
v0
or a=– v (2)
dv x0
67. Given a = – k v or =–k v
dt
v–1/2 dv = – k dt
v1/2 = – kt + c v0 v0
given initial condition (v = u at t = 0), we get a=– x v0
x0 x0
c= 2 u
2
2 v1 2 u1 2 = – kt
v0 v02
or a= x x–
Now, use t = T and v 0 x0
Thus the graph of a versus x is a straight line
68. The slope of the line is m = – ms–2 per second 2
6 v0
–2
and its intercept is c y = mx + c, having a positive slope = and negative
x0
the acceleration a (in ms–2) as a function of time t
is given by v02
intercept = –
a=– t x0
6
dv 70. Given v = k x v 2 = k 2x
or =– t
dt 6 have
dv dx dx
= k2 = k 2v
t
or v= t dt 2v v
0 6 dt dt dt

dv k2
or v=– t2 t+k (1) =
12 dt 2
where k k2
particle starts from rest, v = 0 at t this in dv = dt
2
(1) we get k
k 2t
v=– t 2
t (2) v=
12 2
2.25

dx k 2t
= F kv
dt 2 73. Acceleration a =
m m
k2 dv
=
k
v
dx = tdt
2 dt m
dv k
k2 2 = dt
x= t v m

Thus x t2 v
dv k
t
= dt
71. a – t graph is m = ms–2 u
v m0
2 2
–2 kt
c |ln|vu =
line is m
v kt
a= t ln =
2 u m
kt/m
dv
= t v = ue (1)
dt 2

dv = t dt dx kt/m
2 74. = ue
dt
kt/m
dx = u e dt
v t
dv = t dt
0 0
2 x t
kt/m
dx = u e dt
2
t t 0 0
v= t (t )
kt/m t
e
Thus v = 0 at t v–t x= u
k /m 0

2 mu
dx t x= (1 e kt/m
)
72. = t
dt k
t2
dx = t dt u
75. Putting v =
2
x u kt/m
2 = ue
dx = t dt t dt 2
0 2 2
1
2 = e kt/m 2 = ekt/m
t t 2
x=
2
2 2 or ln(2) =
kt
t=
m
ln(2) ,
m k
1 2 2
=
2
2.26 Comprehensive Physics—JEE Advanced

II

Multiple Choice Questions with One or More Choices Correct


1. At time t
–1 –1
t
5. Two bodies of masses m1 and m2 are dropped from
heights h1 and h2
is neglected, which of the following statements will ground after time t1 and t2 and strike the ground
be true? with speeds v1 and v2
(a) The two bullets will be at the same height correct relation from the following:
above the ground at t t1 h1 t1 m2 h1
(b) The two bullets will reach back their starting (a) (b)
t2 h2 t2 m1h2
(c) The two bullets will have the same speed at v1
v1 h1 m2 h1
t (c) (d) v
(d) The two bullets will attain the same maxi- v2 h2 2 m1h2
6. v-t) graphs shown in
2. The ratios of the distances covered by a freely
motion of a particle?
nth seconds of its motion
(a) form an arithmetic progression
(b) form the series corresponding to the squares
n natural numbers

(d) form a series corresponding to the differ-


ences of the squares of the successive natural
numbers
3. The displacement x of a particle varies with time
a
according to the relation x = 1 e
bt
b
(a) At t = 1/b, the displacement of the particle
a/b)
(b) The velocity and acceleration of the particle
at t = 0 are a and – ab respectively
Fig. 2.28
(c) The particle cannot reach a point at a distance
x from its starting position if x > a/b 7. Two balls of different masses are dropped from the
(d) The particle will come back to its starting
point as t is neglected and the value of g remains constant,
4. which of the following statements are true?
(a) The heavier ball reaches the ground before
of the following statements are correct? Take
g = 10 ms–2
(a) The net displacement of the bullet in 10 s is time
zero
(c) The heavier ball hits the ground with a higher
(b) The total distance travelled by the bullet in
speed
(c) The rate of change of velocity with time is
speed
constant throughout the motion of the bullet
2.27

8. A body is dropped from the top of a tower of height (a) the stone passes the point from where it was
h h
g = 10 ms–2, how long does it take to (b) the speed with which it passes the point of
reach the ground? –1

(a) (2 + )s (b) (2 – )s
6 6 )s
(d) the stone hits the water with a speed of
9. t = 0) located at x = 0 –1
moves along the positive x-direction under the
13. A balloon is rising vertically upwards at a velocity
of 10 ms–1
with x as
v=k x opens his parachute and decelerates at a constant
–2
where k g = 10 ms–2
(a) the displacement x varies with time t as x =
k 2t 2

k 2t –1
(b) the velocity v varies with time t as v =
2 (c) He hits the ground with a speed of 10
k2
(c) the acceleration of the particle is ms–1
2
(d) the distance s travelled by the particle in time
k 2T 2 14.
T is s =
a constant reterdation a which varies with instanta-
neous velocity v as
10. The motion of a body is given by a = – kv
dv where k
dt v of the particle is u at time t = 0, then
where v is the velocity (in ms–1) at time t (in sec- (a) the velocity at time t is given by v = u – at
t (b) the velocity decreases exponentially with
(a) the velocity of the body when its acceleration
is zero is 2 ms–1 u 1
(c) the velocity will decrease to in time
(b) the initial acceleration of the body is 6 ms–2 2 k
(c) the velocity of body when the acceleration is (d) the total distance covered by the particle
half the initial value is 1 ms–1 before coming to rest is u/k
15. A body moves from point A to point B with a
velocity v1 and returns to point A with a velocity
v2
11. A stone falls freely from rest and the total distance 1
covered by it in the last second of its motion is (a) the average speed is (v1 v2 )
2
g = 10 ms–2, 2 v1v2
(b) the average speed is
(v1 v2 )

(c) the stone hits the ground with a speed of


–1 1
(d) the average velocity is (v1 v2 )
(d) the acceleration of the stone during the last 2
16.
of a tower of height h with a speed u reaches the
12. - ground after time t1 -
–1 cally downwards from the top of the tower with
g = 10 ms–2, the same speed, it reaches the ground after time t2
2.28 Comprehensive Physics—JEE Advanced

Then where k
1 2 1 of the particle is u at time t = 0, then
(a) h = g t1 t22 (b) h = g t1t2
2 2 (a) the velocity at time t is given by v = u – at
(b) the velocity decreases exponentially with
1 1
(c) u = g(t – t ) (d) u = g(t + t2)
2 1 2 2 1 u 1
(c) the velocity will decrease to in time
17. 2 k
a constant ratardation a which varies with instanta- (d) the total distance covered by the particle
neous velocity v as before coming to rest is u/k
a = – kv

ANSWERS AND SOLUTIONS

1. The two bullets will attain the same height at time placement x is maximum when t xmax =
1
t=n n– n2 n a
1 e
a
2 b b
1 2
n which gives n
2 4. -
t tion, its net displacement is zero, which is choice
or t
is 2 10 = 20 s while that of the second bullet is u) of the bullet is
(
- u = gt = 10 –1
directed upwards which
t h) attained by
ms–1 1 2 1 2
–1 the bullet is h = gt = 10
2 2
- Therefore, the total distance travelled by the bullet

For heights h << radius of the earth, the magnitude


2. The distance covered in the nth second (with u = 0) of g
is given by
1 1
sn = g n2 – (n – 1)2 g(2n – 1)
2 2 5.
1 1 2 1 2
s1 = g, s2 = g, s = g h1 = 0 t1 + gt = gt
2 2 2 2 1 2 1
1 1
and h2 = 0 t2 + gt 22 = gt 2
3. Velocity of the particle is given by 2 2 2
Therefore
dx d a t1 h1
v= 1 e bt ae bt
dt dt b t2 h2
Acceleration of the particle is given by
Also, we have v1 = 0 + gt1 = gt1 and v2 = gt 2
dv d
= ae bt abe bt v1 t1 h1
dt dt Therefore v
At t = 1/b, the displacement of the particle is 2 t2 h2

a 1 a 1 2a 1 1
x= 1 e 1 e 6. t
b b b
t = 0, the values v particle can have two different velocities at the
and respectively are v = ae–0 = a and = – abe–0
= – ab -
2.29

7. (b) At t v = 0 at
speed with which a ball reaches the ground are in- t v -
tion (a0) = 6 ms–2
8. The total time t taken by the body to reach the a0 –2
(c) Putting a =
1 2 1 2
ground is given by h1 = gt = 10 t2 = t2 v v = 1 ms–1
2 2
The body moves for (t – 1) second before the begin-
(d) is wrong because acceleration a change with
an instant (t – 1) second is v = g(t – 1) = 10 (t time because v
This is the initial velocity for the motion in the last 11. Let h1
-
ond is
1 1 2 9
h2 = 10 (t – 1) 1 + 10 (1)2 = 10(t – h1 = – g h1 = g
2 2 2
= 10t Let t
h1 (t – 1) is g(t – 1) and the distance covered in the
h2 =
last one second is
t2 1
10t – h2 = – g(t – 1) 1 – g (1)2
2
or t2 – 6t g g
h2 = g(t – 1) + = gt –
The two roots of this quadratic equation are 2 2
t= 9 g
Given h1= h2 g = gt – t
2 2
9. The height H from which the stone falls is
dx 1 1
Given v = k x or = kx1/2 or – H = – gt2 = – 10 2
dt 2 2
x–1/2 dx = k dt x1/2 = kt + c H
where c The speed with which the stone hits the ground is
x = 0 at t c – v = – gt = –10 –1
v –1

dx k t 2 2
k 2t
Use v = and x = v=
dt 2 (d) is wrong because for bodies not too far away
2 2 from the earth, the acceleration due to gravity is
dv d k t k –2
Acceleration a = = = = constant
dt dt 2 2
12. Let us assign a positive sign to quantities directed
dx k 2t k2 in the upward directon and a negative sign to those
= or dx = tdt
dt 2 2
s
k2
T
S
u = +10
dx = tdt
ms–1
2 O
O
g = – 10 ms–2
1
k 2T 2 values in the relation s = ut + gt2, we get
which gives s = 2
1
t + (–10)t2 = 10t t2
dv 2
10. Given a = v (1)
dt
or t2 – 2t t + 2)(t
(a) Velocity v0 when acceleration a = 0 is obt-
a = 0, we get which gives t
v0 = 0 or v0 = 2 ms–1 of t
2.30 Comprehensive Physics—JEE Advanced

The total time the parachutist takes (after his exit


coming back to the pointfrom where it was pro-
s

1
0 = 10t + (–10)t2 14.
dv
= – kv
2 dt
v t
which gives t t = 0 is not possible dv
= kdt
u
v 0
v
– v = u + gt log e = – kt
–1
u
= 10 + (–10) v = u e– kt (1)
–1
v
u
v= in time t given by
13. 2
city of the balloon and has an upward velocity of u
= ue– kt
10 ms–1 u = +10 ms–1 g = –10 ms–2 (act- 2
t
given by 1
= e– kt 2 = ekt loge(2) = kt
1 2 2
s = ut + gt log e (2) 0
2
t= =
1 2
k k
= 10 (–10)
2

dx
= u e– kt
dt
x
the velocity of the balloon is 10 ms–1
t
dx = ue kt
dt
x0 0
u – kt
x – x0 = – e
v = u + gt k
–1
= 10 + (–10) x = x0 –
u – kt
e
(directed downwards) k
At t u = –20 ms–1 and u
Now x = 0 at t = 0 which gives x0 =
to hit the ground, his displacement s k
–2
a (directed
u
x= (1 – e– kt) (2)
by k
1 2 1 v = 0 at t =
s = ut + gt t + t2
2 2 distance travelled by the particle before coming to
or t2 t + 12 = 0 or (t – 6)(t – 2) = 0 t = , which
which gives t t = 6 s, then the velo- gives x = u/k
city with which he hits the ground is v = u + at =
6 = 10 ms–1 v is 15. Let s = distance between points A and B t1 is the
time taken to go from A to B and t2 the time taken
correct answer is t = 2 s, in which case, the velocity to return from B to A, then
with which he hits the ground is
s s
v 2 = –10 ms–1 t1 + t2 =
v1 v2
2.31

Total distance travelled


Average speed = u
Total time taken v= in time t given by
2
2v1v2 u
s s = ue– kt
= = 2
s s (v1 v2 )
v1 v2 1
= e– kt 2 = ekt loge(2) = kt
2
-
log e (2) 0
t= =
k k
16. The net displacement in both cases is –h (vertically)

velocity is +u - dx
wards, the initial velocity is –u = u e– kt
dt
(since a = – g in both cases)
x t
1 dx = kt
– h = ut1 – gt12 (1) ue dt
2 x0 0

u – kt
1 2 x – x0 = – e
and – h = ut2 – gt2 (2) k
2
1 u – kt
Equations (1) and (2) give u = g (t1 – t2) and x = x0 – e
1 2 k
h = g t1t2 u
2 Now x = 0 at t = 0 which gives x0 =
k
dv
17. = – kv u
dt
v
x= (1 – e– kt) (2)
dv
t k
= kdt
u
v 0
v = 0 at t =
distance travelled by the particle before coming
v t= ,
log e = – kt
u which gives x = u/k
v = u e– kt (1)

III

Multiple Choice Question Based on Passage


Questions 1 to 4 are based on the following passage
Passage I
total distance travelled
v=
total time taken
by the total length of the path travelled by the body in a

net displacement
v =
time taken
2.32 Comprehensive Physics—JEE Advanced

The direction of the velocity vector is the same as that of r


(c) r 1 + (d)

2.
1. A cyclist starts from
km h–1of the cyclist is
centre O of a circular
track of radius r = 1 km,
reaches edge P of the
track and then cycles 3.
along the circumfer- approximately
ence and stops at point
Q –1
the displacement of the Fig. 2.29
4. ) of the cyclist
cyclist is is

(a) r 1 + (b) r
6
SOLUTION
1. Net displacement is OQ = shortest distance be- 2 r
tween the starting point O and end point Q = 1 km =r+ =r 1 = 1 km 1 km
6
=r
displacement 1km
2. Average velocity = = 1 = 6 km –1 distance km
time h 4. Average speed = = 1
6 time h
6
–1

3. OP + PQ (along the arc)

Question 5 to 7 are based on the following passage. 6. The average velocity of ball A during the time
Passage II interval 0 to 9 s is
–1
(a) along positive x
A is rolled along the positive x-direction with a –1
–1 along negative x
towards a bigger ball B
collision with ball B, ball A retraces the path and reaches (c) ms–1 along negative x
–1
9
5. The average velocity of ball A during the time (d) zero
7. The average velocity of ball A during the time in-
ms–1 along positive x terval 0 to 6 s is
–1
along negative x (a) 2 ms–1 along positive x
(b) 2 ms–1 along negative x
(c) ms–1 along negative x
9 (c) ms–1 along positive x

(d) ms–1 along positive x


(d) ms–1 along negative x
SOLUTIONS
5. Time taken by ball A to reach ball B is Therefore, the velocity of ball A
–1
20 along positive x- direction, which is choice
t1 =
- 6. Time taken by ball A to retrace its path and reach
ment of ball A = 20 m along positive x the starting point after collision with ball B is
2.33

–1
20
t2 = x-
Net displacement of A
Therefore, the net displacement in time interval 0 to along positive x-
of A during 0 and 6 s is
7. A covers a 12 –1
distance of 20 m along positive x-direction up to 6s = 2 ms along positive x-direction

Questions 8 and 9 are based on the following passage


Passage III
t = 0) moving with a
velocity u (c) 2 k u (d) k u
which it decelerates at a rate 9. The distance covered by the particle before coming
a= k v to rest is
/
where v is the instantaneous velocity and k is a positive u 2u /
(a) (b)
k k
8. The particle comes to rest in a time /
u / 2u
2 u u (c) (d)
(a) (b) 2k k
k k

SOLUTION
dv 1/ 2 kt
8. Given a = – kv1/2 or = – kv1/2 v1/ 2 = u
dt 2
1
Thus v 2
dv = – k dt
k 2t 2
v = u – ktu1/2 +
v 1/ 2
d v = – k dt ds
v=
dt
or 2v1/ 2 = – kt + c (i)
where c ds 1/ 2 k 2t 2
Therefore, = u kt u
t = 0, v = u u1/2 = c dt
this value of c in (i), t = 0 to t = , we have
we have 2(v1/ 2 u1/ 2 ) = – kt (ii) ku 1 / 2t 2 k 2t
Let be the time taken by the particle to come to s = ut 2 12 0
v = 0 at t =
1 1/ 2 2 1 2
2u1 / 2 or s= u ku k (iv)
2(0 u 1/ 2
) =–k or = (iii) 2 12
k t from (iii) in (iv), we get
2u / u / u / 2u /
9. s covered in this time, we use s= – + or s=
k 2k 12k k
2.34 Comprehensive Physics—JEE Advanced

IV

Matching

1. x–t

Fig. 2.30

Column I Column II

ANSWER
1. (a) (q) (c) (p)
(b) (s) (d) (r)
Explanation: The slope of x – t AB BC, the slope is
CD, the slope is zero and in DE
2. The displacement x of a particle moving along the x-direction varies with time t
x = a + bt – ct2
where a, b and c
Column I Column II
b c)
(b) Acceleration when velocity = 0 (q) –2c
a
b2
a+
c
SOLUTION
x = a + bt – ct2
x at (t = 0) = a
dx
Velocity v = = b – 2ct
dt
b
v = 0 at t =
2c
2.35

dv
Acceleration = = –2c (which is constant)
dt
2
b b b b2
Value of x at t =a+b –c =a+
2c 2c 2c c
Hence the solution is
(a) (s) (c) (r)
(b) (q) (d) (p)

Assertion-Reason Type Questions

- Statement-2
A body moving along a curve with a constant speed
following four options out of which only ONE choice is
3. Statement-1
- A wooden ball and a steel ball of the same mass,
released from the same height in air, do not reach
-
Statement-2
-
The apparent weight of a body in a medium depends
on the density of the body relative to that of the

4. Statement-1
1. Statement-1
body is a straight line parallel to the time axis, then
A body moving in a straight line may have non-zero
Statement-2
Statement-2 Velocity is equal to the rate of change of displace-

the instant when it reaches the highest point is zero 5. Statement-1


–2

2. Statement-1 a curve, then the body is either uniformly acceler-


A body moving in a straight line with a constant
Statement-2
The slope of the velocity-time graph gives the

SOLUTIONS
1.
2. geff = g 1
moving along a curve continuously changes where = density of the medium and = density

a body moving along a curve with a constant speed 4.


graph is parallel to the time axis, then, rate of
3.
due to gravity in a medium is given by 5.
2.36 Comprehensive Physics—JEE Advanced

VI

Integer Answer Type

1. The displacement x of a particle moving in one 2. The motion of a body is given by


dimension, under the action of a constant force, is d v(t )
related to time t by the equation v(t)
dt
t= x
where v(t) is the velocity (in ms–1) of the body at
where x is in metres and t - time t t
placement (in metre) of the particle when its veloc- –1
its velocity (in ms ) when the acceleration is half

SOLUTIONS
2. The acceleration of the body at time t is
1. Given x =t
x = t2 – 6t + 9 (i) d v(t )
a(t) = v(t) (1)
dt
The instantaneous velocity of the particle is
Putting t
dx d 2
v= (t – 6t + 9) = 2t – 6 a v(0)
dt dt
Now, v will be zero at time t given by 2t – 6 = 0 or t = 0, v
t t Hence
obtained from relation (i) by putting t a(0) = 6 ms–2
2
gives x (at t –6 a(0) 6 –2
net displacement of the particle is zero when its a(t) = , we have from
2 2

v v = 1 ms–1
3
Chapter
Vectors

REVIEW OF BASIC CONCEPTS If the two vectors are represented in magnitude and
direction by the two adjacent sides of a parallelogram
3.1 EQUAL VECTORS drawn from a point, then their resultant is represented
in magnitude and direction by the diagonal of the
Two vectors A and B of the same physical quantity are
parallelogram passing through that point.
equal, if and only if they have the same magnitude and
the same direction. We can test the equality by shifting B
parallel to itself until its tail coincides with the tail of A.
If the tips of the two vectors also coincide, the vectors are
equal. (Remember, the vector shifted parallel to itself is
equal to the original vector since the magnitude and the
direction of the vector do not change when it is shifted
parallel to itself).
Fig. 3.1
3.2 UNIT VECTOR
Figure 3.1 show two vectors A and B of magnitudes
A vector having a unit modulus is called unit vector. If A
A and B inclined at an angle . The magnitude R of the
is a vector, then unit vector A along the direction of A resultant vector R is given by

A A R= A2 B2 2 AB cos
A =
|A| A The angle which the resultant vector R subtends with
vector A is given by
Conventionally, unit vectors along x, y and z directions
B sin
are denoted by i , j and k respectively. tan =
A B cos
3.3 NULL VECTOR B sin
or sin =
If two vectors A and B are equal, their difference (A – B) R
zero or null vector and is represented In vector notation the resultant vector is written as R =
by the symbol 0. It has zero magnitude A + B.
direction. A vector which is not null is called a proper
vector. Thus, if Special Cases
A= B (i) When the two vectors are in the same direction, i.e.
then A – B= 0
= 0°, then R = A2 B 2 2 AB cos 0 = A + B.
3.4 ADDITION OF VECTORS Therefore, the magnitude of the resultant is equal to
the sum of the magnitudes of the two vectors. Also
vectors is known as the parallelogram law of vector tan = 0 or = 0, i.e. the direction of the resultant
addition and may be stated as follows. is the same as that of either vector.
3.2 Comprehensive Physics—JEE Advanced

(ii) When the two vectors are in opposite directions, If the two vectors are represented in magnitude and
2 2 direction by the two sides of a triangle taken in the same
i.e., = 180°, then R = A B 2 AB cos180
= A – B. Also = 0 order, then their resultant is represented in magnitude
and direction by the third side of the triangle taken in the
(iii) When the two vectors are at right angles to each opposite order.
other, i.e. = 90°, then
R A2 B 2 2 AB cos 90 = A2 B 2 . Also 3.6 SUBTRACTION OF VECTORS

tan = .
B Suppose we wish to subtract a vector B from a vector A.
A Since
Note: Rmax = A + B and Rmin = A – B A – B = A + (– B)
the subtraction of vector B from vector A is equivalent
3.1 to the addition of vector – B to vector A. Hence the
The following pairs of forces act on a particle at an A – B) is as follows:
angle which can have any value
(a) 2 N and 3 N (b) 3 N and 3 N
(c) 2 N and 6 N and (d) 3 N and 8 N
The resultant of which pair cannot have a magnitude
of 4 N?

SOLUTION
For pair (a) : Rmin = A – B = 3 – 2 = 1 N and Fig. 3.2
Rmax = A + B = 3 + 2 = 5 N Choose a convenient scale and draw the vectors A and
For pair (b) : Rmin = 0, Rmax = 6 N B as shown in Fig. 3.2 (a). If B is to be subtracted from A,
For pair (c) : Rmin = 4 N, Rmax = 8 N draw the vector negative of B, i.e. draw the vector – B [see
For pair (d) : Rmin = 5 N, Rmax = 11 N Fig. 3.2 (b)]. Now shift the vector – B parallel to itself so
that the tail of– B is at the head of A. Vector C is the sum
Hence the correct answer is (d). of vectors A and – B, i.e. [see Fig. 3.2 (c)]
3.2 C = A + (– B) = A – B
The magnitude of the resultant of two vectors of the
same magnitude is equal to the magnitude of either RESOLUTION OF A VECTOR INTO
3.7 RECTANGULAR COMPONENTS
vector. Find the angle between the two vectors.
Consider a vector A in the x–y plane making an angle
SOLUTION
with the x-axis. The x and y components of A are Ax and
Given A = B and R = A or B. Ay. The magnitudes of Ax and Ay are (Fig. 3.3)
R= A2 B2 2 AB cos

A= A2 A2 2 A2 cos

= 2 A2 1 cos
1
A2 = 2A2(1 + cos ) cos =
2
= 120°

Fig. 3.3
3.5 TRIANGLE LAW OF VECTOR ADDITION
Ax = A cos , along x-direction
The parallelogram law of vector addition yields the triangle
and Ay = A sin , along y-direction
law of vector addition. In Fig. 3.1, vector QP = vector OS
= B. In triangle OQP, vector OP = R. Hence, the triangle
Thus Ax = Ax i = (A cos ) i
law of vector addition may be stated as follows:
Vectors 3.3

NOTE
and Ay = Ay j = (A sin ) j
The magnitude of a component along + x direction or +
From parallelogram law y direction is taken to be positive while the magnitude of
a component along – x direction or – y direction is taken
A = Ax + Ay = (A cos ) i + (A sin ) j to be negative.
The magnitude of A in terms of the magnitudes of its
rectangular components is ADDITION OF VECTORS USING
3.8 COMPONENTS
A= Ax2 Ay2
Two or more vectors are added by using components as
Ay
Also tan = follows:
Ax (a) Resolve each vector into its rectangular compo-
3.3 nents.
(b) Add the magnitudes of all the x-components taking
Resolve A into x and y components. The magnitude into account their signs.
of A is 4 units.
Rx = sum of x-components of all the vectors
(c) Similarly Ry = sum of y-components of all the
vectors
(d) Magnitude of resultant is
R= Rx2 Ry2
(e) The angle which the resultant subtends with the
x-axis is given by
Ry
Fig. 3.4 tan =
Rx

SOLUTION 3.4
Find the resultant of two vectors A = 4 units and B =
3 units shown in Fig. 3.6 by using
(a) parallelogram law and
(b) components method.

Fig. 3.5

In Fig. 3.5(a):
1
Ax = – A cos 60° = – 4 = – 2 units
2 Fig. 3.6
3
Ay = + A sin 60° = + 4 = + 2 3 units SOLUTION
2
In Fig. 3.5(b): (a) Angle between the two vectors is (Fig. 3.7)
3 = 90°
Ax = + A cos 30° = + 4 = 2 3 units
2 R= A2 B2 2 AB cos
1
Ay = – A sin 30° = – 4 = – 2 units = 42 32 2 4 3 cos 90
2
= 5 units
3.4 Comprehensive Physics—JEE Advanced

3
= 2 3 along + y direction
2
2 2
3 3 3
R2 = Rx2 Ry2 2 3 2
(180° – )
2 2
= 25
R = 5 units
60° 30° The angle subtended by the resultant with the
x-axis is given by
Fig. 3.7
3 3
2
R B Ry 2 4.6
= tan = 2.347
sin 180 sin Rx 3 1.96
2 3
B sin 3 sin 90 3 2
sin =
R 5 5 giving 67° above the x-axis. Therefore, the an-
gle which the resultant subtends with A is 67° – 30°
3
= sin 1 37° = 37°.
5
(b) The x and y components of A and B are [see Fig. NOTE
3.8] The components method is more useful if more than two
vectors have to be added.

3.9 SCALAR OR DOT PRODUCT


The scalar (or dot) product of two vectors A and B is
A and B and
the cosine of the angle between them, i.e.
A B = AB cos
The scalar product is represented by putting a dot
between the two vectors. The scalar product of two vectors
Fig. 3.8
is a scalar quantity.
3 Properties of the Scalar Product
Ax = A cos 30° = 4
2 (i) The scalar product is commutative, i.e. A B = B A
= 2 3 units along + x direction (ii) A A = A2 or A = A A . This is true because in
this case = 0°.
1
Ay = A sin 30° = 4 (iii) If two vectors A and B are perpendicular to each
2 other, then = 90° and A B = AB cos 90° = 0.
= 2 units along + y direction Note that A B can be zero when neither A nor B
1 is zero.
and Bx = B cos 60° = – 3
2 (iv) For unit vectors i , j and k along the three rect-
= – 3/2 units along – x direction angular axes x, y and z, we have
and By = B sin 60°
i i = j j = k k =1
3
= 3 units along + y direction
2 and i j = j k = k i =0
Rx = Ax + Bx
(v) A B = (Ax i + Ay j + Az k ) (Bx i + By j
= 2 3 3/ 2 along + x direction
+ Bz k )
Ry = Ay + By = Ax Bx + Ay By + Az Bz
Vectors 3.5

Some Examples of Scalar Product


i j = – j i = k,
(i) Work done is W = F s where F is the force vector
and s is the displacement vector. j k = – k j = i
(ii) Power consumed is P = F v where F is the force
vector and v is the velocity vector. k i = – i k = j,
(iii) Electric current is I = J A where J is the current
density vector and A is the area vector. i i = j j = k k =0
= B A where B is the magnetic
induction vector and A is the area vector.
i j k
3.10 VECTOR OR CROSS PRODUCT (vii) (A B) = Ax Ay Az
Bx By Bz
If the smaller angle between two vectors A and B is ,
then the vector or cross product of vectors A and B is
= i (AyBz – AzBy) + j (AzBx – AxBz)

A B = (AB sin ) n + k (AxBy – AyBx)


where A and B are the magnitudes of vectors A and B and
Some Examples of Vector Product
n is a unit vector perpendicular to the plane containing A
and B. The vector product of two vectors A and B is equal (i) Torque = r F, where r is the position vector and
to a vector C, i.e. F is the force vector.
A B =C (ii) Linear velocity = r where is the angular fre-
The magnitude of vector C is given by quency vector and r is the position vector.
C = AB sin (iii) Angular momentum = r p where r is the position
vector and p is the linear momentum vector.
The direction of C is perpendicular to the plane formed
by A and B and is given by the right hand screw rule. 3.5
Properties of a Vector Product Find the angle between vectors A = i + j and
(i) Vector product is anticommutative, i.e. B= i – j.
(A B) = – (B A)
(ii) A A = 0, i.e. the vector product of a vector SOLUTION
by itself is zero. This is because, in this case,
A B = AB cos
= 0, and hence sin = 0.
Therefore A A = AA sin = 0 Magnitude of A is A = 12 12 = 2
Hence, the condition for two vectors to be parallel
( = 0°) or antiparallel ( = 180°) is that their vector Magnitude of B is B = 12 12 = 2
product should be zero.
If A B = 0, it means either (i) A is zero or, (ii) B = 0 (i j) ( i j) = 2 2 cos
or (iii) the angle between them is 0° or 180°.
(iii) The distributive law holds for both scalar and vec- i i j j = 2 cos ( i j j i 0)
tor products, i.e.
1 – 1 = 2 cos gives cos = 0 or = 90°
A (B + C) = A B + A C
A (B + C) = A B + A C
3.6
(iv) ( A) B = A ( B) = (A B); a real number. Two vectors A and B are inclined at an angle . Using
(v) |A B|2 = |A|2 |B|2 – (A B)2.
of the resultant of vectors A and B is given by
(vi) i , j and k are the three mutually perpendicular
unit vectors at the origin O and along OX, OY and R= A2 B2 2 AB cos
OZ respectively; the right-hand rule gives:
3.6 Comprehensive Physics—JEE Advanced

SOLUTION SOLUTION
R = A + B. Therefore,
(A B) = (2 i 3 j) (5 i 4 j)
R R = (A + B) (A + B)
=A A+A B+B A+B B = 10 i i 8i j 15 j i 12 j j
R2 = A2 + 2 A B + B2
( i i j j 0 and j i k)
= A2 + B2 + 2 AB cos
= 0 8 k 15 k 0
R= A2 B2 2 AB cos
= 23 k
3.7
Thus the magnitude of (A B) is 23 units and its
Given A = 2 i 3 j and B = 5 i 4 j . Find the mag- direction is along + z axis.
nitude and direction of (A B).

Multiple Choice Questions with Only One Choice Correct


1. The unit vector parallel to the resultant of vectors 5. The magnitude of the resultant of two equal vectors
is equal to the magnitude of either vector. The angle
A= i 2j 3 k and B = 2 i j k is
between the two vectors is
1 1 (a) 60° (b) 90°
(a) (3 i j 2k) (b) (2 i j 2k)
14 3 (c) 120° (d) 150°
1 1 6. Given P = A + B and Q = A – B. If the magnitudes
(c) (3 i 2j 2 k ) (d) (3 i j 2k) of P and Q are equal, the angle between vectors
17 14
A and B is
2. The value of n so that vectors A = 4 i nj k and (a) zero (b) 45°
(c) 90° (d) 180°
B = 2i 3j 2 k are perpendicular to each other
is 7. A vector C when added to vectors A = 3 i 5j k
(a) – 1 (b) – 2 and B = 2 i 3 j 4 k gives a unit vector along the
(c) – 3 (d) – 4 y-axis as their resultant. Then C is
3. The angle between vectors A = i 5 j and
B = 2 i 10 j is (a) 5i 3j 3k (b) 5 i 3j 3k
(a) 0° (b) 90°
(c) 5i 2j 3k (d) 5 i 2j 3k
(c) 120° (d) 150°
4. The angle between vectors A and B is 60°. The ratio 8. If the sum A + B of two vectors A and B equals the
A B difference A – B between them, then
is (a) A is a null vector
|A B|
(b) B is a null vector
1
(a) (b) 2 (c) both A and B are null vectors
2 (d) neither A nor B is a null vector
1 9. Given A B 0 and A C = 0. What is the angle
(c) (d) 3
3 between B and C?
Vectors 3.7

(a) 45° (b) 90°


(c) 3 (d) 2
(c) 135° (d) 180°
10. The resultant of two vectors A and B subtends an 19. In Q.18, the angle subtended by vector i + j with
angle of 45° with either of them. The magnitude of the x-axis is
the resultant is (a) 30° (b) 45°
(a) zero (b) 2 A (c) 60° (d) 75°
(c) A (d) 2 A 20. If i , j and k are unit vectors along x, y and
11. A and B are two vectors in a plane at an angle of 60° z-axes respectively, the angle between the vector
with each other. C is another vector perpendicular
to the plane containing vectors A and B. Which of i + j + k and vector i is given by
the following relations is possible? 1 1
(a) = cos–1 (b) = sin –1
(a) A + B = C (b) A + C = B 3 3
(c) A B = C (d) A C = B 3 3
12. Vector C is the sum of two vectors A and B and (c) = cos –1 (d) = sin–1
2 2
vector D is the cross product of vectors A and B.
What is the angle between vectors C and D? 21. Given that 0.2 i + 0.6 j + a k is a unit vector.
(a) zero (b) 60° What is the value of a?
(c) 90° (d) 180° (a) 0.3 (b) 0.4
13. The resultant of two vectors of magnitudes 3 units (c) 0.6 (d) 0.8
and 4 units is 1 unit. What is the value of their dot
product? 22. Given A = i + j and B = i + k . What is the value
(a) – 12 units (b) – 7 units of the scalar product of A and B?
(c) – 1 unit (d) zero (a) 1 (b) 2
14. The resultant of two vectors of magnitudes 3 units (c) 3 (d) 2
and 4 units is 1 unit. What is the magnitude of their 23. The cross product of vectors A and B in Q. 22 is
cross product?
(a) i + j + k (b) i – j + k
(a) 12 units (b) 7 units
(c) 1 unit (d) zero (c) i + j – k (d) i – j – k
15. Three vectors A, B and C are related as A + C = B. 24. Given A = 2 i + 3 j and B = i + j . The component
If vector C is perpendicular to vector A and the of vector A along vector B is
magnitude of C is equal to the magnitude of A, 1 3
what will be the angle between vectors A and B? (a) i j (b) i j
2 2
(a) 45° (b) 90°
5 7
(c) 135° (d) 180° (c) i j (d) i j
2 2
16. The magnitude of the resultant of (A + B) and
(A – B) is 25. In Q.24, above, what is the component of vector A
perpendicular to vector B and in the same plane as B?
(a) 2A (b) 2B
1 3
(a) j i (b) j i
(c) A2 B2 (d) A2 B2 2 2
17. In Q.16, what is the angle between the resultant 5 7
vector and vector A? (c) j i (d) j i
2 2
A
(a) zero (b) cos–1 26. The magnitudes of vectors A, B and C are respec-
B
tively 12, 5 and 13 units and A + B = C. The angle
(c) cos–1
B
(d) cos–1
A B between A and B is
A A B (a) zero (b)
18. If i and j are unit vectors along x-axis and y-axis (c) (d)
2 4
respectively, the magnitude of vector i + j will be 27. The sum of two forces acting at a point is 16 N. If
(a) 1 (b) 2 the resultant force is 8 N and its direction is perpen-
3.8 Comprehensive Physics—JEE Advanced

dicular to the smaller force, then the forces are


(a) 6 N and 10 N (b) 8 N and 8 N F = (4 i + j + 6 k )
(c) 4 N and 12 N (d) 2 N and 14 N newton. The work done by the force is
28. Vector A of magnitude 4 units is directed along the (a) 60 J (b) 59 J
positive x-axis. Another vector B of magnitude 3 (c) 58 J (d) 57 J
units lies in the x-y plane and is directed along 30º 33. A vector A is along the positive z-axis and its vector
with the positive x-axis is as shown in Fig. 3.9. product with another vector B is zero, then vector B
The A B is could be
(a) 6 units (b) 6 2 units (a) i + j (b) 4 i
(c) 6 3 units (d) 12 units
y
(c) j + k (d) 7k
34. A body, initially at rest, is acted upon by four forces
B
^
F1 = i + k , F2 = 2 j + 3 k , F3 = 3 i and F4 = 3 j 4 i .
j
In which plane will the body move?
30° (a) x – y plane (b) x – z plane
x
O x^ A (c) y – z plane (b) none of these
Fig. 3.9
35. A is a vector which when added to the resultant of

29. In Q. 28, the magnitude of A B is vectors (2 i 3 j + 4 k ) and ( i + 5 j + 2 k ) yields a


unit vector along the y-axis. Then vector A is
(a) 6 units (b) 6 2 units
(c) 6 3 units (d) 12 units (a) 3i j 6k (b) 3 i + j 6k

30. The angle between vectors A = i + 2 j + 3 k and (c) 3 i j + 6k (d) 3 i + j 6k


B= i 2 j + 3 k is given by 36. The angle between two vectors A and B is . Vector
R is the resultant of the two vectors. If R makes an
3 2
(a) cos = (b) cos = angle with A, then
7 7 2
1 B
(c) cos = (d) zero (a) A = 2B (b) A =
7 2
31. Two vectors C = A + B and D = A – B are perpen- (c) A = B (d) AB = 1
dicular to each other. Then
(a) A is parallel to B 37. What is the torque of a force F = (2 i 3 j + 4k)
(b) A is perpendicular to B newton acting at a point r = (3 i + 2 j + 3 k ) metre
(c) B is a null vector
about the origin?
(d) A and B have equal magnitudes.
(a) 6 i 6 j + 12 k (b) 17 i 6 j 13 k
32. A body moves from a position r1 = (2 i 3j 4k)

metre to a position r2 = (3 i 4 j + 5 k ) metre under (c) 6 i + 6 j 12 k (d) 17 i + 6 j + 13 k

ANSWERS

1. (d) 2. (b) 3. (a) 4. (c) 5. (c) 6. (c)


7. (a) 8. (b) 9. (b) 10. (b) 11. (c) 12. (c)
13. (a) 14. (d) 15. (a) 16. (a) 17. (a) 18. (b)
19. (b) 20. (a) 21. (c) 22. (a) 23. (d) 24. (c)
25. (a) 26. (c) 27. (a) 28. (c) 29. (a) 30. (a)
31. (d) 32. (d) 33. (d) 34. (c) 35. (a) 36. (c)
37. (b)
Vectors 3.9

SOLUTIONS
1. The resultant of A and B is 10. As shown in Fig. 3.10, the angle between vectors
A and B is 90°. Also A = B. Therefore, the magni-
R = A + B = (i 2j 3 k ) + (2 i j k) tude of the resultant is given by
R2 = A2 + B2 + 2 AB cos
= (3 i j 2k) = A2 + A2 + 2A2 cos 90° = 2A2
or R = 2 A.
Magnitude of R is R = 32 12 ( 2)2 = 14
R 1
R = = (3 i j 2 k )
R 14
So the correct choice is (d).
2. A and B will be perpendicular to each other if A B
= 0, i.e.

(4 i nj k ) (2 i 3j 2k) = 0
Fig. 3.10
8 + 3n – 2 = 0 n=–2
Hence the correct choice is (b).
3. B = 2 ( i 5 j ) = 2 A. Hence the magnitude of B is 11. Since C is perpendicular to both A and B, the sum
twice that of A and the direction of B is the same of any two cannot yield the third vector. Hence
as that of A. So the correct choice is (a). choices (a) and (b) are not possible. Choice (d) is
A B AB cos also not possible because B is not perpendicular to
4. = = cot . A. Choice (c) is possible.
| A B | AB sin
12. Vector C lies in the plane containing vectors A and
= cot 60° ( = 60°)
B, and vector D is perpendicular to both A and B.
1 Hence D must be perpendicular to C. Hence the
=
3 correct choice is (c).
2 2 2
5. R = A + B + 2AB cos . It is given that R = A = B. 13. Let be the angle between the two vectors. The
Thus, we have resultant is given by
A2 = A2 + A2 + 2A2 cos R2 = A2 + B2 + 2AB cos
1 Putting the values of R, A and B we get
cos = – = 120° (1)2 = (3)2 + (4)2 + 2 3 4 cos
2
6. If is the angle between A and B, the magnitudes or cos = – 1 or = 180°
of P and Q are given by Now A · B = AB cos = 3 4 cos 180°
= – 12
P = A2 B2 2 AB cos Hence the correct choice is (a).
14. The magnitude of A B = AB sin = 3 4
and Q = A2 B2 2 AB cos sin 180° = 0. Hence the correct choice is (d).
Given P = Q. If follows that cos = 0 or = 90°. 15. Since A + C = B, vector B is the resultant of vectors
A and C. Using the triangle law of vector addition
7. j = C + A + B = C + (3 i 5 j k ) (2 i 3 j 4 k ) (see Fig. 3.11), we have = 45° ( A = C)

= C +5i 2 j 3k

C= 5i 3 j 3k
8. Given A + B = A – B which gives B = – B. This
is possible only if B is a null vector. Hence the
correct choice is (b).
9. Since A · B = 0, it follows that A is perpendicular
to B. Also A C = 0. Therefore A is parallel to C.
Fig. 3.11
Hence B is perpendicular to C. Therefore, the cor-
rect choice is (b). Thus the correct choice is (a).
3.10 Comprehensive Physics—JEE Advanced

16. The resultant R of vectors (A + B) and (A – B) is


R = (A + B) + (A – B) = 2A (A · B) = 2 i 3j i j
The magnitude of the resultant = 2A. Hence the
correct choice is (a)
=2 i i 2i j 3j i 3 j j
17. Since R = 2A, R is parallel to A. Hence the correct
choice is (a). =2+0+0+3=5

18. Let i and j represent the magnitudes of vectors i B i j


Also B =
B
and j respectively. Since i and j are unit vectors, i j

i = 1 and j = 1. Therefore, the magnitude of vector


i j 1
= i j
2
1 1 2 2
i + j = i 2+ j 2 12 12 2.
Thus the correct choice is (b). 5
Thus the answer is i j which is choice (c).
19. The angle subtended by vector i + j with the 2
x-axis is given by
i 1 25. Since i j i j 0 , vector i j is per-
tan = =1
1
j
pendicular to vector i j . Let i j = C. Now
or = 45° which is choice (b).

i j k .i (A · C) = 2 i + 3 j i j
i i j i k i
20. cos = 1/ 2
2 2 2
1 1 1 1/2 1 The required component is
i j k i
C i j
(A · C) = 2i 3j i j
=1 0 0 1 C
i j
3 3

( j i k i 0 and i i 1) 1 1
= i j j i
Hence the correct choice is (a). 2 2
21. Here (0.2)2 + (0.6)2 + a2 = 1 or a2 = 1 – 0.04 – 0.36,
i – j = 1 1 = 2 and (2 i 3 j) ( i j)
= 0.6 or a = 0.6 . So the correct choice is (c).
= –1
Thus the correct choice is (a).
22. A · B = i j i k i i i k j i jk
26. Since A + B = C, vector C is the resultant of vec-
=1+0+0+0=1 tors A and B. If the angle between A and B is , the
Thus the correct choice is (a). magnitude of the resultant is given by
C 2 = A2 + B2 + 2AB cos
23. A B= i j i k
or (13)2 = (12)2 + (5)2 + 2 12 5 cos
which gives cos = 0 or = /2. Hence the correct
= i i i k j i j k
choice is (c).
27. F is the resultant of F1 and
=0 j k + i which is choice (d). F2 and F1 is the smaller force. Now
24. The component of vector A along vector B = (A · B) B F 22 = F 21 + F 2 = F 21 + (8)2 (i)
B
where B = where B is the magnitude of vector and F1 + F2 = 16 or F2 = 16 – F1 (ii)
B
B. Now
Vectors 3.11

31. Since C and D are at right angles to each other,


C D = 0 or (A + B) (A – B) = 0
or A2 – A B + B A – B2 = 0
or A = B ( A B = B A)
32. Displacement r = r2 – r1

= (3 i 4 j + 5 k ) – (2 i 3j 4k )

= i j 9k metre
Fig. 3.12
Work done W = F r
Using (ii) in (i) we have (16 – F1)2 = F 21 + 64,
which gives F1 = 6 N. = (4 i + j + 6 k ) (i j + 9 k ) newton metre
Therefore F2 = 16 – 6 = 10 N. Hence the correct
choice is (a). = (4 – 1 + 54) = 57 newton metre or 57 J.
33. The vector product of two non-zero vectors is zero
28. Let i and j be the unit vectors along positive x and if they are in the same direction. Hence, vector B
y-axes respectively. Then must be parallel to vector A, i.e. along ± z-axis.
A = 4 i and B = 3 cos 30º i + 3 sin 30º j 34. Resultant force = F1 + F2 + F3 + F4

3 3 3 = ( i + k ) + (2 j + 3 k ) + 3 i + (3 j – 4 i )
= i+ j
2 2
= 5 j + 4k
3 3 3
A B=4i i+ j =6 3
2 2 35. Given A + (2 i – 3 j + 4 k ) + ( i + 5 j + 2 k ) = 1 j
36. The angle which the resultant R makes with A is
( i i = 1; i j = 0) given by
B sin
tan
3 3 3 A B cos
29. A B=4i i+ j = 0 + 6k
2 2
given . Hence
2
( i i = 0; i j =k )
B sin
tan =
= 6k 2 A B cos
Hence the correct choice is (a).
30. Angle between vectors A and B is given by sin 2 B sin cos
A B 2 2 2
or =
cos = A B cos
AB cos
2
A B = ( i + 2 j + 3k ) ( i – 2 j + 3k ) which gives A = B.
= (1)2 + 2 – 2 + (3)2
=1–4+9=6 i j k
A = {(1)2 + (2)2 + (3)2]1/2 37. Torque = r F= 3 2 3
2 3 4
= (1 + 4 + 9)1/2 = 14
1/ 2
B = (1)2 ( 2) 2 (3) 2 14
= i [8 – (– 9)] – j (12 – 6) + k (– 9 – 4)
6 6 3
cos = = 17 i – 6 j – 13 k
14 14 14 7
3.12 Comprehensive Physics—JEE Advanced

II

Multiple Choice Questions with One or More Choices Correct

1. Two vectors of the same physical quantity are un- (c) 1 cm, 5 cm (d) 1 cm, 7 cm
equal if 5. The dot product of two vectors A and B is zero if
(a) they have the same magnitude and the same (a) A is a null vector and B a proper vector
direction (b) A is a proper vector and B is a null vector
(b) they have different magnitudes but the same (c) A and B are both null vectors
direction (d) A and B are proper vectors perpendicular to
(c) they have the same magnitude but different each other.
directions 6. The cross product of two vectors A and B is zero if
(d) they have different magnitudes and different (a) A is a null vector and B is a proper vector
directions. (b) A is a proper vector and B is a null vector
2. Given A = – B. This means that vectors A and B (c) A and B are both null vectors
(a) have equal magnitudes (d) A and B are proper vectors parallel to each
other.
(b) have unequal magnitudes
7. If A B = C, which of the following statements is/
(c) are in opposite directions
are correct?
(d) are in the same direction. (a) C is perpendicular to A
3. Which of the following is a null vector? (b) C is perpendicular to B
(a) Velocity vector of a body moving in a circle (c) C is perpendicular to both A and B
with a uniform speed (d) C is parallel to both A and B
(b) Velocity vector of a body moving in a straight 8. Which of the following vector identities is/are
line with a uniform speed false?
(c) Position vector of the origin of a rectangular (a) A B = B A (b) A B = – B A
coordinate system (c) A B = B A (d) A B = – B A
(d) Displacement vector of a stationary object 9. A and B are two perpendicular vectors in a plane.
4. The magnitudes of four pairs of displacement C is another vector perpendicular to the plane con-
vectors are given. Which pairs of displacement vec- taining vectors A and B. Which of the following
tors cannot be added to give a resultant vector of relations is/are possible?
magnitude 4 cm? (a) A + B = C (b) A + C = B
(a) 1 cm, 1 cm (b) 1 cm, 3 cm (c) A B = C (d) A C = B

ANSWERS AND SOLUTIONS

1. The correct choices are (b), (c) and (d). or Rmax = A + B


2. Given A = – B, i.e. A + B = 0. Two vectors add When = 180°, R is minimum given by
up to zero only if they have equal magnitudes and R2min = A2 + B2 – 2AB
opposite directions. Hence the correct choices are or Rmin = A – B
(a) and (c).
Thus, the magnitude of resultant will lie between
3. The correct choices are (c) and (d)
A – B and A + B. Hence the correct choices are (a)
4. The magnitude R of the resultant of two vectors A
and (d).
and B depends upon the magnitudes of A and B and
5. A · B = AB cos = 0 if A = 0 or B = 0 or = 90°.
the angle between them and is given by
Hence all the four choices are correct.
R2 = A2 + B2 + 2AB cos 6. A B = AB sin = 0 if A = 0 or B = 0 or = 0°.
When = 0, R is maximum given by Hence all the four choices are correct.
R2max = A2 + B2 + 2AB 7. The correct choices are (a), (b) and (c).
Vectors 3.13

8. The scalar product is commulative, i.e. A · B = B · A. choices (a) and (b) are not possible. Since A is
Vector product is anti–commutative, i.e. A B perpendicular to B, the three vectors are mutu-
= – B A. Hence choices (b) and (c) are false. ally perpendicular. Hence choices (c) and (d) are
9. Since C is perpendicular to both A and B, the sum possible.
of any two cannot yield the third vector. Hence

III

Matching
1. Match the following.
Column I Column II
(a) A · B = B · A. (p) False
(b) A B = B A (q) A and B are perpendicular to each other
(c) A · B = 0 (r) A and B are parallel to each other
(d) A B = 0 (s) True
ANSWER
1. (a) (s) (b) (p)
(c) (q) (d) (r)
4
Chapter
Motion in Two Dimensions

REVIEW OF BASIC CONCEPTS Since y x2, the trajectory of the body is para-
bolic.
4.1 PROJECTILE MOTION
t f)
Projectile is the name given to a body which, after having Putting y = h and t = tf in Eq. (4.2), we get
been given an initial velocity, is allowed to move under the 2h
. tf =
g
R)
Putting t = tf and x = R in Eq. (4.1), we get
2h
R = u tf = u
g
Magnitude of resultant velocity at time t is
2 1/ 2
v= u g2 t2
Fig. 4.1 The angle which the resultant velocity vector
subtends with the vertical is given by
(i) A body projected horizontally with a velocity u from
a height h. [Fig. 4.1] Horizontal and vertical dis- u
tan =
tances covered in time t are gt
x = ut (4.1) (ii) A body projected from the ground with a velocity u
1 2 at an angle with the horizontal. (Fig. 4.2)
y = gt (4.2) The horizontal and vertical distances covered in
2
time t are
Differentiating Eqs. (4.1) and (4.2) w.r.t time t, we
get the horizontal and vertical velocities. x = (u cos )t (4.5)
dx 1 2
vx = u (4.3) and y = (u sin )t – gt (4.6)
dt 2
Horizontal and vertical components of the velocity
dy at time t are
vy = gt (4.4)
dt vx = u cos (4.7)
Equation to trajectory and uy = u sin – gt (4.8)
Eliminating t from Eqs. (4.1) and (4.2), we get Magnitude of resultant velocity at time t is
g 1/ 2
y= 2
x2 v = v 2x v 2y
2u
4.2 Comprehensive Physics—JEE Advanced

The angle subtended by the resultant velocity 1/ 2


vector with the horizontal is given by u sin u 2 sin 2 2h
which gives T =
vy g g2 g
tan =
vx The horizontal range is R = (u cos )T

(i) The horizontal range is the same for angles and


(90° – ).
(ii) The horizontal range is maximum for = 45°.
Rmax = u2/g
R
(iii) When horizontal range is maximum, hmax = max
4
1 2
(iv) At the point of projection, KE = mu , PE = 0.
2
1 2
Fig. 4.2 Total energy E = mu .
2
Equation of trajectory 1
Eliminating t from Eqs. (4.5) and (4.6), we get (v) At the highest point, KE = mu 2 cos2
2
gx 2 1 1
y = (tan )x – and PE = total energy – KE = mu 2 mu 2 cos 2
2u 2 cos 2 2 2
tf ) 1 2 2
= mu sin .
Putting y = 0 and t = tf in Eq. (4.6), we get 2
2u sin R and hmax from the equation of trajectory
tf =
g y = ax – bx2
h ) where a and b are constants, refer to Fig. 4.4.
Put t = tf /2 and y = hmax in Eq. (4.6), we get
u 2 sin 2
hmax =
2g
H R)
Putting t = tf and x = R in Eq. (4.5) we get
u 2 sin 2
R=
g
(iii) A body projected from a height h with a velocity u
at an angle with the horizontal. (Fig. 4.3) Fig. 4.4

(a) At O and B, y = 0. Putting y = 0 in the above


equation, we have 0 = ax – bx2 x = 0,
a
x = a/b. Therefore R .
b
R a
(b) At A, y = hmax and x = . Using these
2 2b
a2
values in y = ax – bx2, we get hmax = .
4b
(vii) If A and B are two points at the same horizontal
Fig. 4.3 level on a trajectory at a height h from the ground,
If T (see Fig. 4.5), then
1 1
h = uy T g T 2 = u sin T gT2
2 2
Motion in Two Dimensions 4.3

vx = u cos ( – )
and vy = u sin ( – )
The x and y components of acceleration due to
gravity are – g sin and – g cos respectively, as
shown in Fig. 4.6. Let Tf
inclined plane. Since the net vertical displacement
in time Tf is zero (i.e., h = 0), we have
1
Fig. 4.5
0 = vy T f g cos T f2
2
2u sin 1
(a) tf = t1 t2 or 0 = vy g cos T f
g 2
1 1
(b) h = gt1 t2 or 0 = u sin ( – ) g cos T f
2 2
(c) Average velocity during time interval (t2 – t1) 2u sin
or Tf = (iii)
is vav = u cos g cos
( during this interval, the vertical displace-
ment is zero)
(viii) Velocity and Direction of Motion of Projectile at
any Height. Let P be a point on the trajectory of a
projectile at a height h and let v be the velocity of
the projectile at that height. If is the angle which
the velocity vector makes with the horizontal, then
the horizontal and vertical components of the
velocity are given by vx = ux = constant
or v cos = u cos (i)
and v 2y = u 2y – 2gh
Fig. 4.6
or (v sin )2 = (u sin )2 = – 2gh (ii)
Squaring Eq. (i) and then adding to Eq. (ii), we During this time, the horizontal component of ve-
get locity u cos remains constant. Hence, horizontal
v2 = v 2x v 2y u2 2 gh distance OQ is
OQ = (u cos )Tf
or v = (u2– 2gh)1/2
Range of the projectile on the inclined plane
This gives the speed of the projectile at height h. is
The direction of the velocity vector (i.e., direction
OQ u cos Tf
of motion) is obtained by taking the square root of R = OP = (iv)
Eq. (ii) and then dividing by Eq. (i). We get cos cos
1/ 2 Using Eq. (iii) in Eq. (iv), we get
sin u 2 sin 2 2 gh
tan = = 2 u 2 sin cos
cos u cos R= 2
g cos
(ix) Time of Flight and Range of a Projectile on an
Inclined Plane
Consider an inclined plane OAB making an angle 4.1
with the horizontal (Fig. 4.6). Let a body be pro- A body is projected horizontally with a velocity of
jected with a velocity u at an angle with the hori- 10 ms–1 from the top of building 20 m high. Find
zontal. Let us choose the x-axis along the plane OA (a) horizontal distance from the bottom of the build-
and y-axis perpendicular to the plane OA. Let the ing at which the body will strike the ground.
body hit the inclined plane at point P so that R =
(b) the magnitude and direction of the velocity of the
OP is the range on the inclined plane. The x and y
body 1 s after it is projected. Take g = 10 ms–2.
components of the velocity of the projectile are
4.4 Comprehensive Physics—JEE Advanced

Refer to Fig. 4.7. The horizontal and vertical compo-


SOLUTION
nents of initial velocity are
(a) Refer to Fig. 4.1 on page 4.1. Given h = – 20 m,
(a) ux = 20 cos 30° = 10 3 ms–1
u = 10 ms–1, and g = – 10 ms–2. Time taken by
the body to go from A to B is uy = 20 sin 30° = 10 ms–1 (vertically upwards)
2h 2 20 Horizontal acceleration ax = 0 and vertical accel-
t= =2s eration ay = – 10 ms–2 (vertically downwards).
g 10 Vertical displacement S = – H = – 15 m.
Horizontal distance OB = R = ut = 10 2 = 20 m
Since the vertical and horizontal motions are
(b) Refer to Fig. 4.1 again. Horizontal velocity at t independent of each other, we have, for vertical
= 1 s is motion.
vx = u = 10 ms–1. Vertical velocity at t = 1 s is (since 1
initial vertical component of velocity uy = 0) S = u yt + a yt 2
2
vy = uy + at = 0 – 10 1 vy = – 10 ms–1
1
Magnitude of resultant velocity is –15 = 10t + (– 10)t2
2
v= v 2x v 2y 102 ( 10) 2 –15 = 10t – 5t2
= 200 10 2 ms 1 t2 – 2t – 3 = 0
The angle which the resultant velocity vector t = – 1 s or 3 s.
subtends with the vertical is given by Since t = –1 s is not possible, the ball with strike
v 10 ground at point C after 3 seconds.
tan = x =1 = 45°
v y 10 (b) Horizontal range R = OC = uxt = 10 3 3
= 30 3 m
4.2
(c) Horizontal velocity component at C is
A ball is thrown with a velocity of 20 ms–1 at an angle
of 30° above the horizontal from the top of a building vx = ux = 10 3 ms–1
15 m high. Find (take g = 10 ms–2) Vertical velocity component at C is
(a) the time after which the ball hits the ground. vy = uy + ayt = 10 – 10 3 = – 20 ms–1
(b) the distance from the bottom of the building at The negative sign shows that the ball is moving
which it hits the ground. downwards.
(c) the velocity with which the ball hits the ground. Resultant velocity v = v 2x v 2y
(d) the maximum height attained by the ball above
the ground. = (10 3 ) 2 (20)2 10 7 ms 1

SOLUTION vx 10 3 3
tan =
vy 20 2

1 3
= tan with the vertical.
2
(d) Maximum height attained above the ground is
hmax = h + H

u 2 sin 2
= +H
2g
v
v
v (20) 2 sin 2 (30 )
= + 15
2 10
= 5 + 15 = 20 m
Fig. 4.7
Motion in Two Dimensions 4.5

4.3 v 2y = 0 vy = 0
A stone thrown from the ground at an angle of 45° Hence point P is at the highest point on the trajec-
above the horizontal strikes a vertical wall at a point tory where the velocity is only horizontal. Thus
10 m above the ground. If the wall is at a distance of 20 the stone strikes the wall at P with a velocity of
g = 10 ms–2) 20
10 2 ms–1 in the horizontal direction.
(a) the speed with which the stone is projected, 2
(b) the magnitude and direction of the velocity of
the stone when it strikes the wall. 4.4
A ball projected with a velocity of 10 ms–1 at an
SOLUTION angle of 30° with the horizontal just clears two verti-
cal poles, each of height 1.0 m. Find the separation
between the poles. Take g = 10 ms–2.

SOLUTION
Refer to Fig. 4.9. Let us calculate the two values of t
at which the ball passes just above P and R. For each
pole h = 1.0 m
Fig. 4.8
(a) Let P be the point on the wall where the stone
strikes it. Taking the point of projection O as the
origin, the coordinates of P are (20 m, 10 m)
[Fig. 4.8]
x = (u cos )t (1)
1 2 Fig. 4.9
y = (u sin )t – gt (2)
2
Putting x = 20 m, y = 10 m, g = 10 ms–2 and y-component of velocity is uy = u sin = 10 sin 30°
= 45° in Eqs. (1) and (2), we have = 5 ms–1.
1
20 = (u cos 45°)t =
u
t h = uyt + gt 2
2
2
1
20 2 1.0 = 5t + ( 10)t 2
t= (3) 2
u 5t2 – 5t + 1.0 = 0
1
and 10 = (u sin 45°)t – 10 t2
2 The two roots of this quadratic equations are t1 = 0.72 s
ut and t2 = 2.76 s. Therefore
10 = – 5 t2 (4) OQ = uxt1 = 10 cos 30° 0.72 = 6.2 m
2
Using (3) in (4) and OS = uxt2 = 10 cos 30° 2.76 = 23.9 m
2
u 20 2 20 2 QS = 23.9 – 6.2 = 17.7 m
10 = 5
2 u u
4000 4.5
10 = 20 – 2
u2 = 400
u A projectile has the same range R = 40 m for two
u = 20 ms –1 angles of projection. If T1 and T2 are the times of
T1 T2. Given g = 10 ms–2.
20
(b) At point P, vx = 20 cos 45° = ms–1
2 SOLUTION
20 –1 For a given speed u of projection, a projectile has the
uy = u sin = 20 sin 45° = ms
2 same range for angles of projection and (90° – ).
400
v 2y = u 2y – 2g 10 v 2y = – 2 10 10 Therefore
2
4.6 Comprehensive Physics—JEE Advanced

2u sin vBA = vB – vA = vB + (– vA )
T1 =
g Thus the magnitude and direction of vector vBA can be
2u sin (90 ) 2u cos vB and –vA
and T2 = which is vector OC as shown in Fig. 4.10.
g g
Magnitude of vector vBA is given by
4u 2 sin cos
T 1T 2 = 1/ 2
g2 vBA = v 2A vB2 2v A vB cos
2u 2 sin (2 ) 2R 1/ 2
= 2 = v 2A vB2 2v A vB cos ( = 180° – )
g g
2 40 The angle which the resultant vector OC subtends
= = 8 s2
10 with vector OD is given by
OC CD
4.6 =
sin sin
A ball is thrown from a point with a speed u at an
angle with the horizontal. From the same point and CD sin vB sin
sin =
at the same instant, a person starts running with a OC vBA
constant speed u/2 to catch the ball. Will he be able to
catch the ball? If yes, what should be the value of ? (i) If vector vA and vB are in the same direction, = 0°,
SOLUTION then vBA = v 2A vB2 2v A vB = vB – vA.
The person will catch the ball if the horizontal rane (ii) If vector vA and vB are in opposite direction, =
180°, then vBA = vB + vA.
i.e. if
u
R= tf (i) To cross the river of width d along the shortest path
2
2 which is PQ, the boat must move along PR making
u sin (2 ) u 2u sin
= an angle (90° + ) with the direction of the stream
g 2 g such that the direction of the resultant velocity v is
1 along PQ. Angle is given by (see Fig. 4.11)
cos = = 60°
2

4.2 RELATIVE VELOCITY IN TWO DIMENSIONS


The relative velocity of a body B with respect to body A

vBA = vB – vA
If vectors vA and vB are inclined to each other at an angle
as shown in Fig. 4.10, the relative veliocity vBA is found
as follows.
Fig. 4.11

v
sin =
vb

Also v= vb2 v2
The time taken to cross the river along the shortest
path is given by
d d
t=
Fig. 4.10 v vb2 v2
Motion in Two Dimensions 4.7

(b) To cross the river in the shortest time, the boat vm


should move along PQ. The shortest time is given or = tan–1
by vr
d Thus, the man must hold the umbrella at an angle
t= with the vertical towards north.
vb
At this time, the boat will reach the point R on the 4.3 UNIFORM CIRCULAR MOTION
opposite bank of the river at a distance x from the
point Q (Fig. 4.12). From the Figure, we have (i) For a body moving in a horizontal circle
The centripetal acceleration of a body of mass m
moving in a circle of radius R with a constant speed
v (or angular speed ) is
2
ac = v= 2
R= v
R
Centripetal force is
2 mv 2
fc = mac = m R=
R
Fig. 4.12
(ii) For a body moving in a vertical circle
x = d tan , The minimum speed to complete the circle when
vw the body is at the top of the circle is v = Rg . The
but tan = Therefore, minimum speed to complete the circle when the
vb
body is at the bottom of the circle is v = 5 Rg .
vw
x= d NOTE
vb
(ii) Holding an Umbrella to Project from Rain Let vr The magnitude of velocity (v) and the magnitude of
be the velocity of the rain falling vertically down- acceleration (v2/R) for a body in uniform circular mo-
ward and vm the velocity of a man walking from tion are constant but the direction of velocity v (which
is along the tangent and the direction of acceleration
north to south direction (Fig. 4.13). In order to pro-
tect himself from rain, he must hold his umbrella c (which is towards the centre keep on changing with
time.
in the direction of the resultant velocity v, which is
given by 4.4 NON–UNIFORM CIRCULAR MOTION
v= vr2 vm2
If the speed of the body revolving in a circle changes
with time, it is said to be in non-
v
uniform circular motion. In this
case, the acceleration of the body
is the resultant of two components
(Fig. 4.14).
(i) Radial (or centripetal) accel-
eration directed towards the
centre O and has a magni-
tude ac = v2/R, where v is the Fig. 4.14
instantaneous speed and R is
the radius of the circle.
Fig. 4.13
(ii) Tangential component directed along the tangent
This is the speed with which the rain strikes the which causes the change in the magnitude of veloc-
umbrella. If is the angle subtended by the resul- ity. Its magnitude is given by
tant velocity v with the vertical, then from triangle dv
ORM , we have at =
dt
RM vm The net acceleration of the body is
tan =
OR vr a= ac2 a t2
4.8 Comprehensive Physics—JEE Advanced

and it makes an angle with the tangent given by


Net acceleration a = at2 ac2
ac
tan =
at = k2 (4 kn)2
2 2
= k 1 16 n
4.7
A particle moves along a circle with a velocity
4.8
v = kt, where k is a constant. Find the net accelera-
tion of the particle at the instant when it has covered A particle moves along a circle of radius R. Its speed
nth fraction of the circle after the beginning of v varies with distance x covered along the circle as
motion. v = k x , where k is a constant. If R = 1.0 m and
x
SOLUTION vector and the tangential acceleration vector.
Let v be the speed of the particle at the instant when
it has covered nth fraction of the circle of radius R. SOLUTION
Centripetal acceleration at that instant is Tangential acceleration is
v2 dv dv d x vd v
ac = (1) at =
R dt dt d x dx
Tangential acceleration at that instant is at dx = v dv
dv dv d x vd v Integrating
at = x v
dt dt d x dx
at dx = v dv
vdv = at dx (2) 0 0
Let x be the distance covered. Then x = (2 R)n. v2
Integrating Eq. (2) we have at x =
2
v x
v 2 (k x )2 k2
v d v = at dx at =
0 0 2x 2x 2
Centripetal acceleration is
v2
= at x = at (2 R)n v2 k 2 x
2 ac =
R R
v= 2at (2 R)n (3) Angle between net acceleration and tangential
Using (3) in (1), we get acceleration is given by
2at (2 R)n ac k 2 x/R x
ac = = 4 at n tan = 2
R at k /2 2R
dv d Putting x = 0.5 m and R = 1.0 m. We get
Now at = (kt ) = k
dt dt 2 0.5
tan = =1 = 45°.
ac = 4 kn 1.0

Multiple Choice Questions with Only One Choice Correct


1. The maximum height attained by a projectile is (a) 30° (b) 45°
3 /4 times its horizontal range. The angle of 3
(c) 60° (d) tan–1
projection of the projectile with the horizontal is 2
Motion in Two Dimensions 4.9

2. A body is projected with a speed u at an angle maximum height, the ratio of the initial kinetic en-
with the horizontal. The speed of the body when ergy of P to that of Q is
it is at the highest point on its trajectory is 2 / 5
3 3
times its speed at half the maximum height. The (a) (b)
value of is 4 2
(a) 30° (b) 45° 1 1
(c) (d)
3 2 2
(c) 60° (d) tan–1
2 8. With what minimum speed must a body be pro-
3. A body of mass m is projected from the ground with jected from the origin in the x-y plane so that it can
linear momentum p such that it has the maximum pass through a point whose x and y coordinates are
horizontal range. The minimum kinetic energy of 30 m and 40 m respectively? Take g = 10 ms–2.
(a) 10 ms–1 (b) 20 ms–1
p2 (c) 30 ms–1 (d) 40 ms–1
(a) zero (b) 9. It is possible to project a particle with a given
m
velocity in two possible ways so as to make it pass
p2 p2 through a point P at a distant r from the point of
(c) (d)
2m 4m projection. The product of the times taken to reach
this point in the two possible ways is then propor-
4. Two bodies are projected simultaneously from the tional to
same point on the ground with speeds 10 ms–1 and
(a) 1/r (b) r
10/ 3 ms–1 at angles 30° and 60° respectively with 1
the horizontal. The separation between them when (c) r3 (d)
r2
they hit the ground is (take g = 10 ms–2)
10. A projectile has a maximum range of 200 m. What
10 is the maximum height attained by it?
(a) 10 3 m (b) m
3 (a) 25 m (b) 50 m
1 1 (c) 75 m (d) 100 m
(c) 5 3 m (d) 5 3 m
3 3 11. A body thrown along a frictionless inclined plane
of angle of inclination 30° covers a distance of
5. A projectile is given an initial velocity = (2 i j) 40 m along the plane. If the body is projected with
ms–1. The cartesian equation of its trajectory is the same speed at angle of 30° with the ground, it
(take g = 10 ms–2) will have a range of (take g = 10 ms–2)
(a) y = 2x – 5x2 (b) 2y = 2x – 5x2 (a) 20 m (b) 20 2 m
2
(c) 4y = 2x – 5x (d) 4y = x – 5x2 (c) 20 3 m (d) 40 m
6. A body is projected from the ground with a speed u 12. Which of the following remains constant during the
at an angle with the horizontal. The magnitude of
the average velocity of the body between the point (a) kinetic energy
of projection and the highest point of its trajectory (b) momentum
is (c) vertical component of velocity
u
(a) (sin + cos ) (d) horizontal component of velocity
2 13. A body is projected with kinetic energy K at an
u angle of 60° with the horizontal. Its kinetic energy
(b) (1 + 2 cos2 )1/2
2 at the highest point of its trajectory will be
u (a) 2 K (b) K
(c) (1 + 3 cos2 )1/2 (c) K/2 (d) K/4
2
14. A body, projected with a certain kinetic energy, has
u a horizontal range R. The kinetic energy will be
(d)
2 minimum at a position of the projectile when its
7. A body P is projected vertically upwards. Another horizontal range is
body Q of the same mass is projected at an angle (a) R (b) 3R/4
of 60° with the horizontal. If both attain the same (c) R/2 (d) R/4
4.10 Comprehensive Physics—JEE Advanced

15. Four projectiles are projected with the same speed cally upwards and ball B at 30º to the vertical. They
at angles 20°, 35°, 60° and 75° with the horizon- reach the ground simultaneously. The velocities of
tal. The range will be the longest for the projectile projection of A and B are in the ratio
whose angle of projection is (a) 3 :1 (b) 1 : 3
(a) 20° (b) 35°
(c) 60° (d) 75° (c) 3 :2 (d) 2 : 3
16. A player throws a ball which reaches the other 23. A body is projected with a velocity v = (3 i + 4 j )
player in 4 seconds. If the height of each player is ms–1. The maximum height attained by the body is
1.8 m, what is the maximum height attained by the (take g = 10 ms–2)
ball above the ground?
(a) 0.8 m (b) 8 m
(a) 19.4 m (b) 20.4 m
(c) 21.4 m (d) 22.4 m (c) 80 m (d) 800 m
17. The maximum height attained by a projectile is 24.
increased by 1% by changing the angle of projec- (a) 0.8 s (b) 1.0 s
tion, without changing the speed of projection. The (c) 4.0 s (d) 8.0 s
25. A body is projected with a velocity at an angle
(a) 20% (b) 15% with the horizontal. The velocity of the body will
(c) 10% (d) 5% become perpendicular to the velocity of projection
18. A projectile has a range R T. If the after a time t given by
range is doubled (by increasing the speed of projec- 2u sin u sin
tion, without changing the angle of projection), the (a) (b)
g g
T 2u u
(a) (b) 2T (c) (d)
2 g sin g sin
(c)
T
(d) 2 T 26. A body is projected at an angle with the hori-
2 zontal. When it is at the highest point, the ratio of
19. A projectile has the same range R when the maxi- the potential and kinetic energies of the body is
mum height attained by it is either h1 or h2.Then R, (a) tan (b) tan2
h1 and h2 will be related as (c) cot (d) cot2
(a) R = h1h2 (b) R = 2 h1h2 27. At time t = 0 a body is projected horizontally from
a certain height with a velocity u. The radius of
(c) R = 3 h1h2 (d) R = 4 h1h2 curvature of its trajectory at time t is
20. A ball is projected vertically upwards with a cer- 1/ 2
tain initial speed. Another ball of the same mass is u2 gt 3/ 2
u2 g 2t 2
(a) 1 (b) 1
projected at an angle of 60° with the vertical with g u g u2
the same initial speed. At the highest point, the
1/ 2 3/ 2
ratio of their potential energies will be u2 g 2t 2 u2 g 2t 2
(a) 4 : 1 (b) 3 : 2 (c) 1 (d) 1
g u2 g u2
(c) 2 : 3 (d) 2 : 1
IIT, 1989 28. A body is projected from the bottom of an inclined
21. A body of mass m1, projected vertically upwards plane which has an inclination of 10° with the
with an initial velocity u reaches a maximum height horizontal. At what angle from the horizontal
h. Another body of mass m2 is projected along an should the body be projected so that its range on
inclined plane making an angle of 30° with the hor- the inclined plane is maximum for a given velocity
izontal and with speed u. The maximum distance of projection?
travelled along the incline is (a) 45° (b) 50°
(a) 2h (b) h (c) 55° (d) 60°
h h 29. From the top of a tower, two balls are thrown
(c) (d) horizontally with velocities u1 and u2 in opposite
2 4
directions. If their velocities are perpendicular to
22. Two balls A and B are projected from the same each other just before they strike the ground, the
location simultaneously. Ball A is projected verti- height of the tower is
Motion in Two Dimensions 4.11

2 2 36. A plumb line is hanging from the ceiling of a train.


u1 u2 u1 u2
(a) (b) If the train moves along a horizontal track with a
2g 2g uniform acceleration a, the plumb line gets inclined
u12 u22 u1u2 to the vertical at a angle
(c) (d) a g
2g 2g (a) tan–1 (b) tan–1
30. A body is projected at time t = 0 with a velocity u at an g a
angle with the horizontal. It hits the ground at time a g
(c) sin–1 (d) cos–1
t = tf where tf g a
velocity of the body during the time interval t = 0 37. A body moves along a circular track of radius
to t = tf is 20 cm. It starts from one end of a diameter, moves
(a) u cos (b) u sin along the circular track and reaches the other end
u u of the diameter is 5 seconds. What is the angular
(c) (1 cos 2 )1 / 2 (d) (1 sin 2 )1 / 2
2 2 speed of the body?
31. A body is projected at time t = 0 with a velocity u (a) rad s–1 (b) rad s–1
at an angle with the horizontal. The horizontal 2 3
and vertical components of its velocity will become (c) rad s–1 (d) rad s–1
equal at time t 0, 4 5
38. A cyclist is moving with a speed of 6 ms–1. As the
(a) if 0 (b) if
4 4 2 approaches a circular turn on the road of radius
120 m, he applies brakes and reduces his speed at a
(c) if 0 (d) for no value of constant rate of 0.4 ms–2. The magnitude of the net
6
32. A boy whirls a stone in a horizontal circle 2 m acceleration of the cyclist on the circular turn is
above the ground by means of a string 1.25 m long. (a) 0.5 ms–2 (b) 1.0 ms–2
- (c) 2.0 ms–2 (d) 4.0 ms–2
tally, striking the ground 10 m away. What is the 39. A car is travelling at a velocity of 10 km/h on a
magnitude of the centripetal acceleration during straight road. The driver of the car throws a par-
circular motion? Take g = 10 ms–2. cel with a velocity of 10 2 km/h when the car is
(a) 100 ms–2 (b) 200 ms–2 passing by a man standing on the side of the road.
–2
(c) 300 ms (d) 400 ms–2 If the parcel is to reach the man, the direction of
33. A body is moving in a circle with a uniform speed throw makes the following angle with the direction
v. What is the magnitude of the change in velocity of the car,
when the radius vector describes an angle ? (a) 135° (b) 45°
2 1/2
(a) zero (b) v (1 + cos ) 1
(c) tan–1 2 (d) tan–1 2
(c) 2v cos (d) 2v sin
2 2
34. A particle is acted upon by a force of constant 40. Rain is falling vertically with a speed of 4 ms–1.
magnitude which is always perpendicular to the After some time, wind starts blowing with a speed
velocity of the particle. The motion of the particle of 3 ms–1 in the north to south direction. In order
takes place in a plane. It follow that to protect himself from rain, a man standing on
(a) its velocity is constant the ground should hold his umbrella at an angle
(b) its acceleration is constant given by
(c) its kinetic energy is constant 3
(a) = tan–1 with the vertical towards south
(d) it moves in a straight line. 4
35. A particle of mass M is moving in a horizontal
3
circle or radius R with uniform speed V. When it (b) = tan–1 with the vertical towards north
moves from one point to a diametrically opposite 4
point, its 3
(c) = cot–1 with the vertical towards south
(a) kinetic energy changes by MV2/4 4
(b) momentum does not change
3
(c) momentum changes by 2 MV (d) = cot–1 with the vertical towards north
(d) kinetic energy changes by MV2 4
4.12 Comprehensive Physics—JEE Advanced

41. In Q.40 above, with what speed does the rain strike 46. A stone tied to a string of length L is whirled in a
the umbrella? vertical circle with the other end of the string at the
(a) 3 ms–1 (b) 4 ms–1 centre. At a certain instant of time, the stone is at its
(c) 5 ms–1 (d) 6 ms–1 lowest position and has a speed u. The magnitude
42. A swimmer can swim in still water with a speed of of the change in its velocity as it reaches a position
5 ms–1. While crossing a river his average speed is where the string is horizontal is
3 ms–1. If he crosses the river in the shortest (a) u2 2 gL (b) 2gL

(a) 2 ms–1 (b) 4 ms–1 (c) u2 gL (d) 2(u 2 gL)


(c) 6 ms–1 (d) 8 ms–1
43. IIT, 1998
–1
speed of 2 ms . A person in a boat at a point P on 47. A projectile is projected with a velocity u at an
the bank of the river wants to cross the river by the angle , which of
shortest path to reach a point Q directly opposite on the graphs shown in Fig. 4.15 shows the variation
the other bank. If he can row his boat with a speed of range R versus u?
of 4 ms–1 in still water, he show row his boat at an
angle of
(a) 30° upstream with the line PQ
(b) 30° downstream with the line PQ
(c) tan–1(0.5) upstream with the line PQ
(d) tan–1(2) downstream with the line PQ.
44. In Q.43 above, the time taken by him to cross the
river by the shortest path is
(a) 3 s (b) 3 3 s
(c) 6 3 s (d) 18 3 s
45. A body moving in a circular path with a constant
speed has a
(a) constant velocity
(b) constant momentum
(c) constant kinetic energy
(d) constant acceleration Fig. 4.15
IIT, 1992 IIT, 2008

ANSWERS

1. (c) 2. (c) 3. (d) 4. (d) 5. (c) 6. (c)


7. (a) 8. (c) 9. (b) 10. (b) 11. (c) 12. (d)
13. (d) 14. (c) 15. (b) 16. (c) 17. (d) 18. (b)
19. (d) 20. (a) 21. (a) 22. (c) 23. (a) 24. (a)
25. (d) 26. (b) 27. (d) 28. (c) 29. (d) 30. (a)
31. (b) 32. (b) 33. (d) 34. (c) 35. (c) 36. (a)
37. (d) 38. (a) 39. (a) 40. (b) 41. (c) 42. (b)
43. (a) 44. (c) 45. (c) 46. (d) 47. (d)

SOLUTIONS
u 2 sin 2 u 2 sin (2 ) 3
1. hmax = and R = Given hmax = R. Hence.
2g g 4

2 u 2 sin cos u 2 sin 2 3 2 u 2 sin cos


= =
g 2g 4 g
Motion in Two Dimensions 4.13

tan = 3 = 60° 5. Given u cos = 2 ms–1 and u sin = 1 ms–1.


–1 1
2. Maximum height attained is These equations give u = 5 ms , tan = and
2 2 2
2 2 2 2
u sin u sin cos = .
H= gH = u 5
2g 2
The equation of the trajectory is
Speed at maximum height is ux = u cos
Speed v at H/2 is given by g x2
y = x tan –
H 2 u cos 2
2
v2 = u2 – 2g
2 1 10 x 2
= u2 – gH = x 2
2 2
2 ( 5 )2
u 2 sin 2 5
= u2 –
2
x 5x2
2 2 =
It is given that u x2 = v , i.e. 2 4
5 2
4y = 2x – 5x , which is choice (c).
2 2 u 2 sin 2 6. Average velocity is
u2 cos2 = u
5 2 net displacement
vav =
time
2 2 u 2 sin 2
u2(1 – sin2 ) = u R 2
5 2 H2
2
= (1)
3 T /2
sin = = 60°
2
u 2 sin 2
3. For maximum horizontal range, = 45°. Also where H = maximum height =
2g
p = mu which gives u = p/m.
The kinetic energy is minimum when the body is at u 2 sin (2 )
R = horizontal range =
the highest point of its trajectory. At this point the g
u 2u sin
velocity is ux = u cos = u cos 45° = . T
2 g
2
1 u (mu ) 2 p2 Substituting in Eq. (1), we get
Minimum K.E. = m = = .
2 2 4m 4m u
vav = (1 + 3 cos2 )1/2
4. The vertical components of velocity of the two 2
bodies are 10 sin 30° = 5 ms–1 and (10 / 3 ) sin 60° The correct choice is (c).
= 5 ms–1. Since their vertical velocity components 7. For body P, u 12 = 2 gh
u22 sin 2 (60 ) 3 u22
the separation between them when they hit the For body Q, h=
ground is 2g 8g
x = difference in their horizontal ranges 8 gh
u 22 =
= R1 – R2 3
1
(10) 2 sin 60 (10 / 3 ) 2 sin 120 mu12 u2 3
= – K.E. of P 2
= = 12
10 10 K.E. of Q 1 u2 4
mu22
5 2
= 5 3
3 So the correct choice is (a).
8. In projectile motion, the equation of the trajectory is
= 5 1
3 m g x2
3 y = x tan –
So the correct choice is (d). 2 u 2 cos 2
4.14 Comprehensive Physics—JEE Advanced

g x2 projected at angle = 30° with this speed will have


= x tan – (1 + tan2 ) a range of
2u2
Substituting the given values, we have u 2 sin 2 20 20 sin 60
R= 20 3 m
2 g 10
10 (30)
40 = 30 tan – (1 + tan2 ) Hence the correct choice is (c).
2u2
12. Since the velocity of the projectile changes continu-
or 900 tan2 – (6u2 tan ) + (900 + 8u2) = 0 ously, both kinetic energy and momentum undergo
The value of tan will be real if a change with time. Only the vertical component
(6u2)2 4 900 (900 + 8u2) of velocity changes due to gravity; the horizontal
or u4 100 (900 + 8u2) component always remains constant. Hence the
correct choice is (d).
or u4 – 800u2 90000
13. At the highest point, the velocity has only the hori-
or (u2 – 400)2 – 160000 90000
zontal component vx = v cos = v cos 60° = v/2.
or (u2 – 400)2 250000 1
or u2 – 400 500 u2 = 900 Now kinetic energy mv2 is proportional to v2.
2
u = 30 ms–1 Since the velocity is reduced to half, the kinetic
energy becomes one–fourth, i.e. K/4. Hence the
9. The range of a projectile is r = 2v 02 cos sin /g,
correct choice is (d).
the two possible angles of projection are and
(90° – 14. Kinetic energy is minimum when the projectile is
these two angles are at the highest point of its trajectory. At the highest
point, its range = half the horizontal range. Hence
2v0 sin
t1 = the correct choice is (c).
g
15. Range R = v20 sin 2 /g. For the same v0, R sin 2 .
2v0 sin (90 ) 2v0 cos Since sin 2 is the largest for = 35°, the correct
and t2 =
g g choice is (b).
16. t = 2v0 sin /g. Since t = 4 s, we
4v02 sin cos 2r
so that t1t2 = . have v0 sin = 2g.
2
g g
v02 sin 2 4g 2
Thus t1t2 r. Hence the correct choice is (b). Now hmax = = 2g
2g 2g
10. For maximum range = 45°. Hence Rmax = v20/g
= 2 9.8 = 19.6 m
and hmax = v02/4g. Thus hmax = Rmax/4 = 200/4
= 50 m. Hence the correct choice is (b). Height above the ground = 19.6 + 1.8 = 21.4 m
Hence the correct choice is (c).
11. Let u be the initial speed with which the body
is thrown along the inclined plane. As shown in v 2 sin 2
17. Given h = 0 . Differentiating partially we
Fig. 4.16, the effective deceleration is given by 2g
a = g sin v02
g get ( v0 = constant) h = 2 sin cos . Thus
= g sin 30° = = 5 ms –2 2g
2
h 2cos
= 0.01 (given).
h sin
cos
Therefore, = 0.005. We also have T =
sin
2 v0 sin 2 v0 cos
which gives T= . Thus
g g
T cos cos
Fig. 4.16 . But = 0.005. Therefore,
T sin sin
The body stops after covering a distance, say, s
along the plane, which is given by – 2as = 0 – u2 T
= 0.05 or T = 0.005 T.
or u = 2 a s 2 5 40 = 20 ms –1. A projectile T
Motion in Two Dimensions 4.15

Hence T increases by 0.5%. Thus the correct choice 22. The correct choice is (c). Use
is (d).
2u A 2uB sin 60
2 v02 sin cos 4 v02 sin 2 tA = and tB =
18. Now R = and T 2
= . g g
g2 g2
From these two equations we have T2 = 2R tan or 23. The correct choice is (a). The magnitude of
velocity is
T R . Hence the correct choice is (b).
19. The range of a projectile is the same for two angles v = (3)2 (4) 2 = 5 ms–1
of projection and 90° – . For these two angles of The angle subtended by the velocity vector with the
projection, the maximum heights are horizontal (x-axis) is given by
v02 sin 2 4 4
tan = which gives sin =
h1 = 3 5
2g
Now,
v02 sin 2 90 v02 cos 2 4 2
and h2 = (5) 2
2g 2g v 2 sin 2 5
hmax = = 0.8 m
v04 sin 2 cos 2 2g 2 10
h 1h 2 = .
4g 2 4
2 5
2v sin 5 = 0.8 s,
4 v04 sin 2 cos 2
24. tf =
Also R2 = 2
. g 10
g
which is choice (a).
Which give R2 = 16 h1h2 or R = 4 h 1 h 2 . Hence
25. Velocity of projection is v0 = (u cos ) i +
the correct choice is (d).
20. (u sin ) j . At time t, the velocity of the body is
u2 v = (u cos ) i + (u sin – gt) j
h1 =
2g
The dot product of v0 and v is
where u is the initial speed of projection. The maxi-
v0 v = u2 cos2 + u sin (u sin – gt)
mum height attained by the second ball is
or v0 v = u2 – (u sin )gt (i)
( = 90° – 60° = 30°)
Since v is perpendicular to v0, v0 v = 0. Using this
u 2 sin 2 (30 ) u 2 in (i), we have
h2 =
2g 8g u
Now, PE of ball 1 at height h1 = mgh1 and that of 0 = u2 – (u sin )gt or t =
g sin
ball 2 at height h2 = mgh2. Therefore, the ratio of
h u2 8 g Hence the correct choice is (d).
potential energies = 1 = 4. Hence the 26. The correct choice is (b).
h2 2 g u 2
correct choice is (a). u 2 sin 2 mu 2 sin 2
PE = mghmax = mg
21. For the body of mass m1, we have 2g 2
u2 KE =
1
m(u cos )2.
h=
2g 2
27. The horizontal and vertical distances travelled in
For the body of mass m2, if S is the maximum dis- time t are
tance travelled along the incline then
x = ut
v2 – u2 = 2aS
Now, when S is maximum, v = 0. Also a = – g sin 1 2 gx 2
and y= gt =
g 2 2u 2
= – g sin 30° = – . Hence
2 dy gx gt
g = 2 ( x = ut)
0 – u2 = 2 S dx u u
2
d2y g
u2 and 2
=
or S = = 2h, which is choice (a). dx u2
g
4.16 Comprehensive Physics—JEE Advanced

The radius of curvature of the trajectory at time t is 2u sin


given by and tf =
g
2 3/ 2 The correct choice is (a).
dy
1 3/ 2 31. Horizontal component of velocity at a time t is
dx u2 g 2t 2
R= 1 vx = u cos
d2y g u2
vertical component of velocity at that time t is
dx 2
vy = u sin – gt
Hence the correct choice is (d).
They will become equal at time t = t* if
28. Refer to Fig. 4.6 on page 4.3. The range along the
u cos = u sin – gt*
inclined plane is given by
which gives t* = u(sin – cos )
2v02 sin( ) cos
R= Now t* must be positive. Hence sin > cos or
g cos 2
> /4. Hence the correct choice is (b).
v02 32. Given, h = 2 m, R = 1.25 m and horizontal distance
= [sin(2 – ) – sin ]
g cos 2 s = 10 m. When the string breaks, the stone is pro-
jected in the horizontal direction, which means that
R is maximum when sin(2 – ) = 1 or 2 – = 90° there is no initial vertical velocity. From s = ut +
or 1
gt 2, we have ( u = 0),
1 1 2
= (90° + ) = (90° + 10°) = 55°.
2 2 1 2
h = gt (i)
Hence the correct choice is (c). 2
29. Let h be the height of the tower. Let u1 be along The horizontal distance travelled in time t is
positive x-direction u2 along negative x-direction. s = vt (ii)
The two balls hit after at time t given by where v is the velocity of the stone in the horizon-
1 2 2h tal direction which is the same as its velocity in
h= gt t=
2 g circular motion.
At time t, the respective vertical velocities (along Eliminating t from (i) and (ii) we get
the negative y directions) are gt each. Hence the g s2
velocities of the two balls at time t are v2 =
2h
– v1 = u1 i – (gt) j v1 = – u1 i + (gt) j Now, centripetal acceleration is
v2 g s2 10 100
and – v2 = – u2 i – (gt) j v2 = u2 i + (gt) j ac = = 200 ms –2
R 2hR 2 2 1.25
Since v1 and v2 are perpendicular to each other, Thus, the correct choice is (b).
v1 v2 = 0 33. Refer to Fig. 4.17. AB and CD represent the two
[– u1 i + (gt) j ] [u2 i + (gt) j ] = 0 velocity vectors. The change in velocity v = v2 – v1.
which can written as v = v2 + (– v1). Thus, to
– u 1u 2 + g 2t 2 = 0 v, we reverse the direction of vector AB as
2h
g 2t 2 = u 1 u 2 g2 = u 1u 2
g v2 and – v1 by triangle or parallelogram law. This
is shown in Fig. (c).
u1u2
h= . Hence the correct choice is (d). The magnitude of vector v is given by
2g
v = [(v1)2 + v22 + 2(v1)(v2)cos(180° – )]1/2
Displacement R
30. Average velocity = = [2v2(1 – cos )]1/2 ( v1 = v2 = v)
time tf
= 2 v sin
2u 2 sin cos 2
where R = is the horizontal range
g Hence the correct choice is (d).
Motion in Two Dimensions 4.17

Fig. 4.19

37. If r is the radius of the track, then distance moved


in 5s = r = 20 cm. Therefore, speed along the
20
circle (v) = = 4 cms –1. Now, angular speed
5
v 4
= rad s –1. Hence the correct choice
r 20 5
is (d).
2 38. Referring to Fig. 4.20, the cyclist is moving on a
straight road from A to B with a velocity v = 6 ms–1.
As he approaches the circular turn, he decelerates
– 1
at rate at, represented by vector BD. The magnitude
of deceleration is at = 0.4 ms –2. At point B, two
Fig. 4.17 accelerations t and c, the centripetal acceleration
34. Since the force is always perpendicular to the directed towards the centre C act on the cyclist.
velocity (i.e. the direction of motion) of the v 2 (6) 2
particle, no work is done by the force on the Now ac = = 0.3 ms –2. Using the law of
R 120
particle. Hence the kinetic energy of the particle parallelogram of vector addition, vector BE gives
remains constant. The particle will move in a circle the resultant acceleration a whose magnitude is
in a plane. Thus the correct choice is (c).
( DE = c)
35. As shown in Fig. 4.18, at diametrically opposite
points A and B, the magnitude of the velocity is a = (a t2 + a c2)1/2 = {(0.4)2 + (0.3)2}1/2 = 0.5 ms –2
same (= V) but the directions of the velocity are
Hence the correct choice is (a).
opposite. Hence the change in momentum is MV –
(– MV) = 2 MV. Thus the correct choice is (c).

Fig. 4.20
Fig. 4.18
39. In Fig. 4.21 vc represents the velocity of the car
36. When the train is at rest or moving with a uniform and vP that of the parcel. M is the position of the
velocity, the plumb line hangs vertically along OB man. From parallelogram law, the direction of the
(Fig. 4.19). If the train moves with an acceleration resultant velocity vr must be along the direction
along which the man is standing. It follows from
a, the plumb line gets inclined along OC, the direc-
is given by
tion of the resultant of accelerations a and g. It is
= a/g. Hence the vc 10 1
sin = or = 45º
correct choice is (a). vp 10 2 2
4.18 Comprehensive Physics—JEE Advanced

Fig. 4.23

Fig. 4.21 43. Refer to Fig. 4.24. Let PR be the direction along
Hence the correct choice is (a). which he should row his boat. The boat is acted
40. Velocity of rain (vr) = 4 ms –1 vertically down- upon by two velocities – boat velocity (vb) and wa-
wards. Velocity of wind (vw ) = 3 ms–1 from north ter velocity (vw). The angle should be such that
to south direction. A rain drop is acted upon by that the resultant velocity (v) is along PQ, i.e.
two velocities vr and vw as shown in Fig. 4.22. vw 2
From the triangle law, the resultant velocity of sin = = 0.5
vb 4
the rain drop is v = OW. In order to protect him-
self from rain, he must hold his umbrella at an = 30° (upstream).
angle with the vertical (towards north) given
by

RW vw 3
tan =
OR vr 4
Thus the correct choice is (b).

Fig. 4.24

Hence the correct choice is (a).


44. Time taken to cross the river by the shortest path
PQ is
Fig. 4.22
PQ PQ 36
t= 6 3s
41. The magnitude v of the resultant velocity gives v vb2 vw2 (4) 2
(2)2
the speed with which the rain strikes the umbrella,
which is given by Hence the correct choice is (c).
v= [v2r + v2w]1/2 = [16 + 9] 1/2
= 5 ms –1 45. Since the direction of the velocity changes from
point to point on the circle, choices (a), (b) and (d)
Hence the correct choice is (c). are incorrect.
42. In order to cross the river in the shortest time, 46. From energy conservation, [see Fig. 4.25]
the resultant velocity v of the swimmer must be 1 2 1
perpendicular to the velocity vw of water, as mu = mv 2 + mgL
2 2
vs2 = v2 + v 2w or v 2w = v 2s – v2 v= u2 2 gL
= 25 – 9 = 16
–1
or vw = 4 ms which is choice (b). v= v2 ( u )2 2(u 2 gL)
Motion in Two Dimensions 4.19

v 47. ,R u2. So the correct graph is (d).


O
B

v v
L

–u
u
A

Fig. 4.25

II

Multiple Choice Questions with One or More Choices Correct


1. vertical, the speed of the bob is v. The radial
different angles of projection, say, and , that give acceleration of the bob at this instant is given by
it the same range. Then, and are such that v2 v2
(a) cosec = sec (a) (b)
(b) tan ( + ) r r
(c) sin2 – cos2 = sin2 – cos2 v2
(d) cot = cos sec (c) (d) r 2
r
2. A ball is projected upwards at a certain angle with 6. A body is moving in a circle of radius r with a
the horizontal. Which of the following statements uniform speed v, angular frequency , time period
are correct? At the highest point T and frequency . The centripetal acceleration is
(a) the velocity of the projectile is zero given by
(b) the acceleration of the projectile is zero
(c) the velocity of the projectile is along the v2 2
(a) (b) 4 r
horizontal direction. r
2
(d) the acceleration of the projectile is vertically 4 r
(c) v (d)
downwards. T 2

3. Choose the correct statements from the following. 7. Which of the following statements are true about a
The range of a projectile depends upon body moving in a circle with a uniform speed?
(a) the angle of projection (a) The speed of the body is constant but its
(b) the acceleration due to gravity velocity is changing
(c) the magnitude of the velocity of projection
(b) The acceleration is directed towards the
(d) the mass of the projectile
centre
4. A particle is acted upon by a force of constant mag-
(c) The velocity and acceleration vector are per-
nitude which is always perpendicular to the veloc-
pendicular to each other.
ity of the particle. The motion of the particle takes
place in a plane. It follows that: (d) Elastic, frictional, gravitation and magnetic
(a) its velocity is constant forces can cause a uniform circular motion.
(b) its acceleration is constant 8.
(c) its kinetic energy is constant from a place in the enemy country at a distance x
(d) it moves in a circular path
5. A simple pendulum of length r and bob mass m half-way point. Then
(a) the velocity with which the missile was pro-
swings in a vertical circle with angular frequen-
cy . When the string makes an angle with the jected is gx
4.20 Comprehensive Physics—JEE Advanced

(d) the initial speed of the projectile is


x
(b) you have a warning time of (a2 + c2)1/2.
2g
12. A projectle thrown at an angle of 30º with the
(c) the speed of the missile when it was detected horizontal has a range R1 and attains a maximum
is gx height h1. Another projectile, thrown with the same
2 speed, at an angle of 30º with the vertical has a
(d) the maximum height attained by the missible range R2 and attains a maximum height h2. Then
x
is . (a) R2 = 2R1 (b) R2 = R1
4
(c) h2 = 2h1 (d) h2 = 3h1
9.
v. An armyman with an anti-aircraft gun on the 13. The maximum height attained by a projectile is
ground sights the enemy plane when it is directly increased by 1% by increasing its speed of projec-
u. tion without changing the angle of projection. Then
Then the percentage increase in the
(a) the angle with the vertical at which the gun (a) horizontal range will be 2%
(b) horizontal range will be 1%
v
cos–1
u
(b) the angle with the vertical at which the gun 14. The speed of projection of a projectile is increased
by 1% without changing the angle of projection.
v Then, the percentage increase in the
sin–1 .
u (a) horizontal range will be 1%.
(c) the maximum height at which the enemy (b) maximum height attained will be 2%
u2 v2
.
2g
(d) the maximum height at which the enemy 15. A body is projected at time t = 0 from a certain
(u v) 2 point on a planet’s surface with a certain veloc-
. ity at a certain angle with the planet’s surface
2g (assumed horizontal). The horizontal and vertical
10. From the top of a tower of height 40 m, a ball is displacements x and y (in meters) respectively vary
projected upwards with a speed of 20 ms–1 at an with time t (in seconds) as
angle of elevation of 30º. The total time taken by
the ball to hit the ground is T and the time taken to x = 10 3 t
come back to the same elevation) is t. The horizon-
y = 10 t – t2
tal distance covered by the ball is x. If g = 10 ms–2,
then (a) The acceleration due to gravity on the surface
of the planet is 10 ms–2.
T T
(a) =2 (b) = 2 (b) The maximum height attained by the body is
t t 25 m.
(c) x = 40 2 m (d) x = 40 3 m
11. The horizontal distance x and the vertical height y (d) The horizontal range is 100 m.
of a projectile at time t are given by 16. For a particle moving in a circle with a constant
speed,
x = at and y = bt2 + ct
(a) the velocity vector is always along the tan-
where, a, b and c are constants. Then gent to the circle.
(a) the speed of the projectile 1 second after it is (b) the acceleration vector points towards the
a2 + b2 + c2)1/2 centre of the circle.
(b) the angle with the horizontal at which the (c) the velocity and acceleration vectors are per-
–1 c
pendicular to each other.
a (d) the velocity and acceleration vectors are par-
(c) the acceleration due to gravity is –2b. allel to each other.
Motion in Two Dimensions 4.21

17. A particle is acted upon by a constant force which linear speeds of points A, B and C respectively at
is always perpendicular to its velocity. The motion that instant, then
of the particle takes place in a plane. It follows (a) vA = vB = vC (b) vA > vB > vC
that vB
3R 3
(a) its speed is constant (c) vA = 0, vB = (d)
2 vC 4
(b) its acceleration is constant
(c) its kinetic energy is constant
(d) its momentum is constant.
18. A stone of mass 250 g is tied to the end of a string
of length 1.0 m. It is whirled in a horizontal circle
with a frequency of 30 rev./min.
(a) The tension in the string changes as the stone
moves in the circle.
(b) The tension in the string is constant equal to Fig. 4.27
2
newton. IIT, 2004
4 21. The trajectory of a projectile in a vertical plane
(c) The speed of the stone is ms–1. is y = ax – bx2, where a and b are constants and
(d) The maximum speed with which the stone x and y are respectively the horizontal and vertical
can be whirled is 20 ms–1. distances of the projectile from the point of projec-
19. A uniform disc of radius R is rotating about its tion.
axis with angular speed . It is gently placed on a a
(a) The horizontal range of the projectile is .
horizontal surface which is perfectly frictionless 2b
(Fig. 4.26). If vA, vB and vC are the linear speeds of (b) The maximum height attained by the projec-
points A, B and C respectively, then a2
tile is .
4b
2a
, where g is the
bg
acceleration due to gravity.
(d) The angle of projection from the horizontal is
= tan–1 (a – 2bx).
IIT, 1990
22. The coordinates of a particle moving in a plane are
given by
x = a cos (pt)
Fig. 4.26
and y = b sin (pt)
(a) vA = vB = vC (b) vA = vB > vC
where a, b and p are positive constants and b < a.
(c) vA = vC > vB (d) vA < vB < vC
Then
20. A uniform disc of radius R is rolling (without (a) the path of the particle is an ellipse
slipping) on a horizontal surface with an angular (b) the velocity and acceleration of the particle
speed as shown in Fig. 4.27. O is the centre of are perpendicular to each other at t = /2p.
the disc, points A and C are located on its rim and (c) the acceleration of the particle is always
R directed towards a focus.
point B is at a distance from O. During rolling,
2 (d) the distance travelled by the particle in time
the points A, B and C lie on the vertical diameter interval t = 0 to t = /2p is a.
at a certain instant of time. If vA, vB and vC are the IIT, 1999
4.22 Comprehensive Physics—JEE Advanced

ANSWERS AND SOLUTIONS

1. For the same range + = 90º or = 90º – . 9. Let G be the position of the gun and E that of the
Choices (a), (b) and (d) satisfy this relation between v, when
and but choice (c) does not. u in a direction with
2. The correct choices are (c) and (d). the horizontal (Fig. 4.28). The horizontal compo-
nent of u is
u 2 sin 2
3. Range R = , is independent of the mass
g vx u cos
of the projectile. Hence choices (a), (b) and (c) v
are correct.
4. The magnitudes of velocity and acceleration remain
constant but their directions are changing continu-
ously. In uniform circular motion the force is radial
(centripetal) and is always perpendicular to the ve- v
locity which is tangential. Thus, choice (c) and (d)
are correct.
5. The radial component of acceleration is ar =
centripetal acceleration
v
v2
= =r 2 ( v=r ) Fig. 4.28
r
Hence, the correct choices are (c) and (d). Let the shell hit the plane at point P and let t be
2 r the time taken for the shell to hit the plane.
6. Since v = r = 2 r= , all the four choices are It is clear that the shell will hit the plane, if the
T horizontal distance EP travelled by the plane in
correct.
time t = the distance travelled by the shell in the
7. All the four choices are correct. horizontal direction in the same time, i.e. v t = vx
8. For maximum range = 45° for which Rmax = v20/g v
t or v = v x = u cos or cos = .
Hence v0 = g Rmax gx which is choice (a). u
The warni To avoid being hit, the plane should have a
minimum altitude = maximum height attained by
the shell which is
warning time is
u 2 sin 2 u 2 (1 cos 2 )
hmax =
tf v0 sin gx sin 45 x 2g 2g
t=
2 g g 2g
v2
u2 1
Hence choice (b) is also correct. u2 (u 2 v2 )
=
At half-way point, the missile is at its maximum 2g 2g
height. Therefore, the vertical component of Hence the correct choices are (a) and (c).
velocity is zero at this point. Hence the velocity is
2u sin 2 20 sin 30
given only by the horizontal component which is vx 10. t = =2s
g 10
gx
= v0 cos = gx cos 45° = , which is choice Initial downward velocity = u sin = 20 sin 30° =
2
10 ms–1. The time taken to fall through a height of
(c). The maximum height attained is 40 m is given by
1
v02 sin 2 gx sin 2 45 x 40 = 10t1 + 10 t21 which gives t1 = 2 s. Hence,
hmax = 2
2g 2g 4
the total time taken to hit the ground is T = 2 + 2 =
Thus, all the four choices are correct. 4 s. Therefore T/t = 2. Also, the horizontal distance
Motion in Two Dimensions 4.23

travelled in 4 s = (u cos ) T = 20 cos 30° 4 R 2 v0


= 40 3 m. = =2 0.005 = 0.01 = 1 %
R v0
Hence the correct choices are (a) and (c).
2v0 sin
11. The horizontal component of velocity is T=
g
dx d
vx = at = a (i) T v0
dt dt = = 0.005 = 0.5 %
T v0
The vertical component of velocity is
Hence the correct choices are (b) and (c).
dy d R
vy = b t 2 ct = 2 bt + c (ii) 14. The correct choices are (a) and (d). Use =
dt dt R
The value of vy at t = 1 s is (2b + c). Therefore, the 2 v0 h 2 v0 T v0
, and .
magnitude of velocity at t = 1 s is v0 h v0 T v0
v = (v 2x + v 2y) 1/2 = [a2 + (2b + c)2] 1/2 15. The horizontal and vertical displacements are given
by
If a projectile is projected with an initial velocity x = (v0 cos )t (1)
v0 at an angle with the horizontal, the horizontal 1 2
and y = (v0 sin )t – gt (2)
and vertical components of its velocity at time t are 2
given by Given x = 10 3 t (3)
vx = v0 cos (iii) and y = 10 t – t2 (4)
and vy = v0 sin – gt (iv) Comparing (3) with (1) and (4) with (2), we have
Comparing (iii) and (iv) with (i) and (ii) above we v0 cos = 10 3 (5)
have v0 cos = a and v0 sin = c. Dividing, we get, v0 sin = 10 (6)
tan = c/a.
1
Compairing (iv) with (ii) we have g = – 2b and g =1 g = 2 m s–2
2
We have seen above that v0 cos = a and v0 sin
= c. Squaring and adding we get: v 20 = a2 + c2 or v0 Equations (5) and (6) give v0 = 20 ms–1 and
= 30°.
= (a2 + c2) 1/2.
Hence the correct choice are (b), (c) and (d). v02 sin 2 (20)2 sin 2 30
hmax = = 25 m
12. The range is the same for and (90° – ). Hence R1 2g 2 2
= R2 for = 30° or 60 °. 2v0 sin 2 20 sin 30
v 2 sin 2 tf = = 10 s
Since hmax = 0 and v0 is the same, we have g 2
2g
v02 sin 2 (20)2 sin 60
2 R= 100 3 m
h1 sin 30 1 h g 2
or h1 = 2 .
h2 sin 2 60 3 3 Hence the correct choices are (b) and (c).
Thus the correct choice are (b) and (d). 16. The correct choices are (a), (b) and (c).
v02 sin 2 17. Since the force is always perpendicular to the velocity
13. We know that h = . The increase h in h (i.e. the direction of motion) of the particle, no work
2g
is done by the force on a particle. The particle will
when v0 changes by v0 can be obtained by partially
move in a circle in a plane. Its speed and hence
differenting this expression. Thus
its kinetic energy will remain constant. Since the
2 v0 v0 sin 2 h 2 v0 direction of the velocity is along the tangent to the
h= ,
2g h v0 circle, it will keep changing with time. Hence, the
momentum will not remain constant. Since velocity
v0 1 h 1 is changing with time, the acceleration (which is
= 0.01 = 0.005
v0 2 h 2 perpendicular to velocity) will also keep changing.
Hence the correct choices are (a) and (c).
v02 sin 2 18. Since the stone is whirled in a horizontal plane, the
Now range R =
g weight mg of the stone (which acts vertically down-
4.24 Comprehensive Physics—JEE Advanced

wards) is perpendicular to the plane of the circular dy dx dx


motion and, therefore, has no component along this = a 2bx
dt dt dt
plane. Hence the tension in the string is constant. vy = avx – 2bx vx = (a – 2bx)vx (2)
Given m = 250 g = 0.25 kg, R = 1.0 m, frequency ( )
30 At the maximum height vy = 0. Using this
= 30 rev./min = = 0.5 Hz. Angular frequency in Eq. (2), we get (a – 2bx) = 0 or x = a/2b.
60
( ) = 2 = 2 0.5 = rad s –1. The necessary cen- Putting this value of x in Eq. (1), we have
(since y = hmax at this value of x)
tripetal force is provided by the tension in the string. 2
a a a2
Therefore, hmax = a b
2
2b 2b 4b
m v2 m R 2
T= =m R (c) The time t to reach the maximum height is
R R given by
2
2 1
= 0.25 1.0 = N hmax = gt 2
4 2
Speed of the stone is v = R = 1.0 = ms–1.
2hmax 2a 2 a
The maximum speed is given by t=
g 4bg 2bg
m vmax
2
= 100
R
2
100 R 100 1.0 tf = 2t = a
or vmax = bg
m 0.25
dy d
= 20 ms –1 (d) tan = (ax – bx2) = a – 2bx
dx dx
Hence the correct choices are (b), (c) and (d) Hence the correct choices are (b) and (d).
19. Since the surface is perfectly frictionless, the disc 22. Given x = a cos (pt) (1)
will not roll on the surface; it will simply keep on y = b sin (pt) (2)
rotating at point A where it is placed. Now, linear
x2 y2
speed = distance from centre angular speed. From (1) and (2) = cos2 (pt) + sin2 (pt) = 1
2 2
Therefore, a b
R Hence the path of the particle is an ellipse.
vA = R, vB = and vC = R
2 Let the position vector of the particle at time t be
Hence the only correct choice is (c).
20. The disc is rolling about the point O. Thus the axis r = xi yj
of rotation passes through the point A and is perpen- dr dx dy
dicular to the plane of the disc. From the relation v Velocity v = i j
dt dt dt
= r where r is the distance of the point on the rim
about the axis of rotation, we have v = – a p sin (pt) i + bp cos (pt) j
3R
vA = 0, vB = (AB) = At t = /2p, v = – ap sin i + bp cos j
2 2 2
and vC = (AC) = 2R
vB 3R 1 3 = – ap i (3)
Hence .
vc 2 2R 4 2
Acceleration a = – ap cos (pt) i – bp sin (pt) j 2

Hence the correct choices are (c) and (d).


21. Given y = ax – bx2 (1) At t = /2p, a = – bp2 j
(a) The value of y is zero at x = 0 and x = R At t = /2p, v a = abp3 ( i j ) = 0.
(horizontal range). Putting y = 0 and x = R
in Eq. (1), we get R = a/b. Hence v a .
(b) Differentiating Eq. (1) with respect to time t, It is easy to see that choices (c) and (d) are incor-
we have rect.
Motion in Two Dimensions 4.25

III

Multiple Choice Questions Based on Passage

Two objects are projected from the same point with the (a) sin (b) cos
same speed u at angles of projection and with the (c) tan (d) cot
horizontal respectively. They strike the ground at the same 3. The ration h1/h2 is equal to
point at a distance R from the point of projection. The
(a) sin2 (b) cos2
respective maximum heights attained by the objects are h1
and h2 and t1 and t2 (c) tan2 (d) cot2
1. R, h1 and h2 are related as 4. The sum (h1 + h2) is equal to

(a) R = h1h2 (b) R = 2h1h2 u2 u2


(a) sin2 (b) cos2
g g
(c) R = 2 2h1h2 (d) R = 4 2h1h2
u2 u2
t (c) (d)
2. The ratio 1 is equal to g 2g
t2

SOLUTIONS
Since the horizontal range in the same + = 90°. 1. From Eqs. (1), (2) and (3), we have
Therefore = 90° – and we have
u2 2 gh1 2 gh2
u2 R= 2 4 h1h2
h1 = sin2 (1) g u2 u2
2g
Hence the correct choice is (d)
u2 u2 2. From Eqs. (4) and (5) it follows that the correct
h2 = sin2 = cos2 (2) choice is (c).
2g 2g
3. From Eqs. (1), (2), (4) and (5), we have
u2 h1
R= 2 sin cos (3) = tan2
2g h2
2u sin Hence the correct choice is (c).
t1 = (4)
g 4.
2u sin 2u u2
t2 = cos (5) h1 + h2 = , which is choice (d).
g g 2g

The position vector r with respect to the origin of a particle 6. The magnitude of the initial velocity of the particle
varies with time t as is
r = (at) i + (bt – ct2) j (a) a2 c2 (b) b2 c2
where a, b and c are constants.
5. The trajectory of the particle is a (c) a 2 b 2 (d) (a + b – 2c)
(a) straight line (b) circle 7. The angle with the horizontal along which the
(c) parabola (d) none of these particle is projected is given by
4.26 Comprehensive Physics—JEE Advanced

b a 10. The maximum height to which the particle rises is


(a) sin = (b) cos =
c c 2b 2 b2
(a) (b)
b 2c c 2c
(c) tan = (d) tan =
a a 2
b2 b2 4b 2
8. (c) (d)
4c c
b ab 11. The horizontal range of the particle is
(a) (b)
c c ab ac
(a) (b)
c b c b
(c) (d)
a ac bc
9. The acceleration due to gravity at that place is (c) (d) abc
a
(a) 2 a (b) 2 b
(c) 2 c (d) none of these
SOLUTIONS
1 2
5. Comparing Eq. r = (at) i + (bt – ct2) j with Eq. r = and gt
y = (u sin )t – (6)
x i + y j , we get 2
x = at (1) Comparing Eqs. (5) and (6) with Eqs. (3) and (4)
and y = bt – ct2 (2) we have u cos = a and u sin = b which give
tan = b/a, which is choice (c).
b
Eliminating t from (1) and (2) we get y = x– 8. When t = tf, y = 0. Putting y = 0 and t = tf in Eq. (2)
a
we get 0 = tf (b – ctf ) gives tf = 0 and tf = b/c. But tf =
c
2
x2 which is the equation of a parabola. Hence 0 is not possible. Hence the correct choice is (a).
a 9. Comparing Eq. (6) with Eq. (2), we get g = 2c,
the correct choice is (c). which is choice is (c).
6. From Eqs. (1) and (2), we have 1 b
10. Now y = hmax when t = tf = . Putting y = hmax
dx 2 2c
vx = =a (3)
dt and t = b/2c in Eq. (2) we get
2
vy =
dy
= b – 2ct (4) b b b2
hmax = b c
dt 2c 2c 4c
Putting t = 0 in Eqs. (3) and (4), the initial values of
vx and vy are a and b respectively. The initial speed 11. R = (u cos )tf (7)
of the particle is b
Now u = a2 b 2 , tan = which gives
2 2
u = a b , which is choice (c). a
a b
7. If the particle is projected with an initial velocity u cos = 2 2 and tf = . Putting these values
a b c
at an angle with the horizontal, then the horizon-
tal displacement x and vertical displacement y at ab
in Eq. (7), we get R = . Hence the correct choice
time t are is (a). c
x = (u cos )t (5)

An object of mass m is whirled with a constant speed v in


a vertical circle with centre O and radius R. T1, T2, T3 and
T4 are the tensions in the string when the object is at A (top
of the circle), B, C (the lowermost point of the circle) and
D respectively (Fig. 4.29)

Fig. 4.29
Motion in Two Dimensions 4.27

12. Tensions T1, T2 and T3 are related as


Rg
(a) T1 = T2 = T3 (b) T1 < T2 < T3 (a) (b) Rg
2
(c) T1 > T2 > T3 (d) T1 = T3 < T2
13. Tension T4 is given by (c) 2Rg (d) 2 Rg

mv 2 Here g is the acceleration due to gravity.


(a) T4 = + mg cos
R 15. The minimum speed the object must have at the
lowest point C to complete the circle is
mv 2
(b) T4 = – mg cos
R (a) Rg (b) 2Rg
mv 2
(c) T4 = + mg sin (c) 2 2Rg (d) 5 Rg
R
mv 2 16. At the minimum speed of Q.15, the tension in the
(d) T4 = – mg sin string is
R
14. The minimum speed the object must have at the (a) 4 mg (b) 5 mg
highest point A to complete the circle is (c) 6 mg (d) mg

SOLUTIONS
12. Refer to Fig. 4.30
mv 2
T3 = + mg
R
Hence the correct choice is (b).
13. At point D, the force towards the centre is
F4 = T4 – mg cos
mv 2
= T4 – mg cos
R
mv 2
T4 = + mg cos , which is choice (a).
R
14. In order to keep a body of mass m in a circular path,
Fig. 4.30
the centripetal force, at the highest point A, must at
When the object is at A, since the weight mg acts least be equal to the weight of the body. Thus
vertically downwards, the force towards centre O is.
mv 2A
F1 = T1 + mg = mg or vA = Rg
R
mv 2 gives the minimum speed the body must have at the
= T1 + mg
R highest point so that it can complete the circle.
Hence the correct choice is (b).
mv 2
T1 = – mg 15. The minimum speed vC of the body must have at
R
At point B, weight mg has no component along BO. the lowest point C is given by v2C = v2A + 2 2 Rg
Hence the force towards O is where we have used v2 = u2 + 2gh, with h = 2 R.
Thus
mv 2
F2 = T2 = T2 v2C = Rg + 4 Rg = 5 Rg
R
At point C, the weight mg acts in opposite direction or vC = 5 Rg , which is choice (d).
to tension T3. Thus at C, the force towards centre O 16. The tension at this point is given by
is
F3 = T3 – mg vC2
T3 = m g = m(5 g + g) = 6 mg
mv 2 R
= T3 – mg Hence the correct choice is (c).
R
4.28 Comprehensive Physics—JEE Advanced

The kinetic energy of a particle moving along a circle of 2a 1 / 2 a 1/ 2


radius R depends on distance (s) as K = as2 where a is a (c) s (d) s
m 2m
constant.
19. The tangential force acting on the particle is
17. The centripetal force is given by
(a) mas (b) 2mas
as 2 as 2
(a) (b) (b) as (d) 2as
2R R
20. The net force acting on the particle is
2as 2 4as 2 1/ 2
(c) (d) s s2
R R (a) 2as 1 (b) as 1
18. The speed of the particle around the circle is R R2
1/ 2
a 1/ 2
a 1/ 2
s2
(a) 2s (b) s (c) 2as 1 (d) zero
m m R2

SOLUTIONS
1 1/ 2
17. Given KE = mv2 = as2. Therefore, the centripetal ds 2a
But =v = s . Therefore,
2 dt m
force is
1 2 2 a 1/ 2 2 a 1/ 2 2 a s
2 mv at = s
mv 2 2 2as 2 m m m
fc = ,
R R R Tangential force is ft = mat
which is choice (c). 2a s
=m = 2as, which is choice (d).
18. The speed v of the particle around the circle is m
given by 20. Net force acting on the particle is
1/ 2 f = (f 2c + f 2t )1/2
1 2a
mv2 = as2 or v=s 2 1/ 2
2 m 2 a s2 2
= (2 a s )
Hence the correct choice is (c). R
19. The tangential acceleration is 1/ 2
1/ 2 1/ 2
s2
dv d 2a 2a ds = 2 as 1
at = s R2
dt dt m m dt
Thus the correct choice is (c).

A conical pendulum consists of a string of length L 22. The angle of inclination of the string with the verti-
at one end carrying a body of mass m at the other end. The cal is given by
mass is revolved in a circle in the horizontal plane about-a g g
(a) cos = 2
(b) sin = 2
The angular frequency of revolution of the body is . The L L
2 2
string makes an angle with the vertical axis. L L
(c) cos = (d) sin =
21. The tension in the string is g g
m 2 L 2 23. The linear speed of the body is
(a) (b)
L m (a) L (b) L sin
(c) m 2L (d) m L2 (c) L cos (d) L tan
Motion in Two Dimensions 4.29

SOLUTIONS
21. Let T be tension in the string. Figure 4.31 shows Thus
the forces acting on the system. Tension T can be T cos = mg (1)
resolved into two mutually perpendicular compo-
nents. The horizontal component T sin provides m v2
and T sin = = m 2r
the centripetal force for circular motion and the ver- r
tical component T cos balances the weight mg. But r = L sin . Therefore,
2 2
T sin =m L sin or T = m L (2)
Hence the correct choice is (c).
mg
22. From (1), we have cos = (3)
T
Using (2) in (3), we get
mg g
cos = 2 2
, which is choice (a).
m L L
23. Linear velocity is v = r = L sin , which is choice
(b).
Fig. 4.31

IV

Matching
1. For a body projected at time t = 0 horizontally with velocity u from a height h.
Column I Column II
2h
(a) Horizontal displacement (p)
g
2h
(b) Vertical displacement (q) u
g
(c) Time taken to hit the ground (r) is proportional to t
(d) Horizontal range (d) is proportional to t2

ANSWER
(a) (r) (b) (s)
(c) (p) (d) (q)

2. A body is projected from the ground with velocity u such that its range is maximum.
Column I Column II
2u
(a) Maximum height attained (p)
g
u
(b) Horizontal range (q)
g 2
u2
g
u2
(d) Time to reach maximum height (s)
4g
4.30 Comprehensive Physics—JEE Advanced

ANSWER
For maximum range = 45°.
(a) (s) (b) (r)
(c) (p) (d) (q)

3. A body is projected with velocity u at angle = 30° with the horizontal.


Column I Column II
13u
(a) Velocity at maximum height (p)
4
2 7
(b) Velocity at half the maximum height (q) u
3
3u
(c) Average velocity between the point of (r)
projection and highest point 2
2 7
(d) Velocity at t = u
3 8
ANSWER

u 2 sin 2 30 u2
Maximum height h =
2g 8g
3u
(a) At maximum height, vx = ucos and vy = 0. Therefore, v = ucos = ucos 30° = .
2
h
(b) For half the maximum height, h = .
2
vy2 = (u sin )2 – 2 gh = u2 sin2 – gh
2 2
u u u2 u2
= u2 sin2 30° – g
8g 4 8 8
3u
vx = u cos 30° =
2
3u 2 u2 7
v= v 2x v 2y u
4 8 8
net displcemrnt OA
(c) Average velocity = = (see Fig. 4.32)
time taken 1
tf
2
R2
OA = h2
4
u2 3u 2
Now R= sin 60° =
g 2g
1/ 2
u4 3u 4 13u 2
OA =
64 g 2 16 g 2 8g
1 u sin u
Now tf = Fig. 4.32
2 g 2g
Motion in Two Dimensions 4.31

OA 13u 2 2 g 13u
Average velocity =
1 8g u 4
t
2 f
2
(d) At t = t f,
3
2 2 u
vy = u sin –g tf = u sin 30º – g
3 3 g
u 2u u
=
2 3 6
3u
vx = u cos = u cos 60° =
2
1/ 2
3u 2 u2 2 7
v= v 2x v 2y u
4 36 3
Thus the answer is as follows:
(a) (r) (b) (s)
(c) (p) (d) (q)

4. Match objects in circular motion listed in column I with the sources that provide the necessary centripetal force
listed in column II.
Column I Column II
(a) A boy whirling a stone tied to a string in a circle (p) Frictional force
(b) The moon revolving around the earth (q) Muscular force
(c) The electron revolving around the proton (r) Gravitational force
in a hydrogen atom
(d) A car negotiating a curved road (s) Electrostatic force

ANSWER
(a) (q) (b) (r)
(c) (s) (d) (p)

Assertion-Reason Type Questions


In the following questions, Statement-1 (Assertion) is (c) Statement-1 is true, Statement-2 is false.
followed by Statement-2 (Reason). Each questions has (d) Statement-1 is false, Statement-2 is true.
the following four choices out of which only one choice
is correct.
A body is projected horizontally with a velocity u
(a) Statement-1 is true, Statement-2 is true and State- from the top of a building of height h. It hits the
ment-2 is the correct explanation for Statement-1.
ground after a time t = 2h / g .
(b) Statement-1 is true, Statement-2 is true but
Statement-2 is not the correct explanation for The vertical and horizontal motions can be treated
Statement-1. independently.
4.32 Comprehensive Physics—JEE Advanced

A body is projected from the ground with kinetic The velocity vector at a point is always along the
energy K at an angle of 60° with the horizontal. If tangent to the trajectory at that point.
air resistance is neglected, its kinetic energy when
it is at the highest point of its trajectory will be In a uniform circular motion, the centripetal force
K/4. is always perpendicular to the velocity vector.

At the highest point of the trajectory, the directions Then the force does no work on the body and its
of the velocity and acceleration of the body are per- kinetic energy remains constant.
pendular to each other.
In a non-uniform circular motion, the particle has
One end of a string of length R is tied to stone of two acceleration-one along the tangent to the circle
mass m and the other end to a small pivot on a and the other towards the centre of the circle.
frictionless vertical board. The stone is whirled in
a vertical circle with the pivot as the centre. The In a non-uniform circular motion, the magnitude
minimum speed the stone must have, when it is at and the direction of the velocity vector both change
the topmost point on the circle, so that the string with time.
does not slack is gR .
In a non-uniform circular motion, the acceleration
At the topmost point on the circle, the centripetal of the particle is equal to sum of the tangential
force is provided partly by tension in the string and acceleration and the centripetal acceleration.
partly by the weight of the stone.
The two accelerations are perpendicular to each other.
The maximum range on an inclined plane when
a body is projected upwards from the base of the In a uniform circular motion, the kinetic energy of
plane is less than that when it is projected down- the body remains constant.
wards from the top of the same plane with the same
speed. The momentum of the body does not change with
time.
The maximum range along an inclined plane is in-
dependent of the angle of inclination of the plane.
In a uniform circular motion, the acceleration is
always directed towards the centre of the circle.
In projectile motion, the velocity of the body at a point
on it trajectory is equal to the slope at that point. Otherwise the speed of the body moving along the
circle will change with time.
SOLUTIONS
1. The correct choice is (a). The time taken by the body At the highest point of the trajectory, the velocity
to hit the ground is the same as if it was dropped of the body is horizontal (parallel to the ground)
from that height and fell freely under gravity. but its acceleration is g directed vertically down-
2. The correct choice is (b). If m is the mass of the wards.
body and u its velocity of projection, the initial 3. The correct choice is (a). When the stone is at the
kinetic energy is topmost point A on the circle, the centripetal force
1 is provided by (mg + T) as shown in Fig. 4.33.
K = mu2
2
At the highest point, the horizontal velocity is mv 2
Thus = mg + T
(u cos 60°) and vertical velocity is zero. Hence the R
kinetic energy at the highest point is When the stone is at A, the string will not slack if
1
m (u cos 60°)2 =
1 1
mu2 =
K mv 2
K = tension T = 0, which gives = mg v= Rg
2 4 2 4 R
Motion in Two Dimensions 4.33

u2
R max =
g (1 sin )
Thus R max > Rmax. Since the range R depends on
angle , Statement is false. Hence the correct
choice is (c).
. 5. The correct choice is (d). At the highest point on
the trajectory, the slope is zero but velocity is
Fig. 4.33
u cos .
4. The range along the inclined plane when a body 6. The correct choice is (a).
is projected with velocity u at an angle with the 7. The correct choice is (a).
horizontal is given by
8. The correct choice is (d). The acceleration of the
2u 2 sin( ) cos particle is given by
R = 2
g cos
a= ac2 at2
2
u where ac = centripetal acceleration and at = tangen-
= [sin (2 – ) – sin ]
g cos 2 tial acceleration.
where is the angle of inclination of the plane. 9. The correct choice is (c). The speed of the body
Range R will be maximum of sin(2 – ) = 1 or remains constant but the momentum changes with
2 – = 90° in which case time because the direction of the velocity vector
changes with time.
u2 u2
Rmax = 2
[1 – sin ] = 10. The correct choices is (a). If the acceleration vector
g cos g (1 sin )
is directed towards the centre of the circle, it will
If the body is projected downwards from the top of have a component along the tangent, as a result the
the same inclined plane, the maximum range will be speed of the body will change and the motion no
longer remains uniform.

VI

Integer Answer Type


1. A body falling freely from a given height H hits an v
inclined plane in its path at a height h. As a result of
this impact, the direction of the velocity of the body
becomes horizontal. For what value of H/h, will the
body take the maximum time to reach the ground?
IIT, 1986
2. On a frictionless horizontal surface, assumed to be
the x-y plane, a small trolley A is moving along a
straight line parallel to the y-axis with a constant Fig. 4.34
velocity vT = ( 3 1) ms–1 as shown Fig. 4.34. At 3. A train is moving along a straight line with a con-
a particular instant when OA makes an angle of 45° stant acceleration ‘a’. A boy standing in the train
with the x-axis, a ball is thrown from origin O. Its throws a ball forward with a speed of 10 m/s, at an
velocity makes an angle of 60° with the x-axis and angle of 60° to the horizontal. The boy has to move
its velocity is such that it hits the trolley. Find the forward by 1.15 m inside the train to catch the ball
magnitude of the veolcity of the ball with respect back at the initial height. The acceleration of the
to the horizontal surface. train, in m/s2, is
IIT, 2002 IIT, 2011
4.34 Comprehensive Physics—JEE Advanced

SOLUTIONS
1. A body falling freely from point P at a height H
H d 2t
hits the inclined plane at point Q at a height h. As It is easy to check that when = 2, is nega-
a result, the velocity becomes horizontal and the tive. h dh 2
2. Refer to Fig. 4.36. It follows that v B is the resultant
the ground at point R. The motion of the body from of v and vT . In triangle OAB
P to R via Q is shown in Fig. 4.35.
vB vT
P =
sin135 sin15
vT sin (90 45 )
H–h vB =
sin (60 45 )
Q
( 3 1) cos 45
H =
Q sin 60 cos 45 cos 60 sin 45
( 3 1) 1 / 2
h = = 2 ms–1
3 / 2 2 1/ 2 2

Fig. 4.35 v

If t is the time taken by the body to fall from P to


Q through a height (H – h), then we have
1/2
1 2 2( H h) v
(H – h) = gt or t =
2 g v
The time t taken by the body to fall from Q to R is
given by ( the initial velocity at O is zero)
1/2
1 2 2h
h= gt or t =
2 g
The total time taken by the body to reach the ground Fig. 4.36
is 3. u = 10 ms ,–1
= 60°
1/2 1/2
2( H h ) 2h
t =t + t = (1) 2u sin 2 10 sin 60
g g t= 3s
2 g 10
dt d t
Time t will be maximum if = 0 and is Let v be the velocity of the train. The horizontal
dh dh 2
negative. Therefore, differenting Eq. (1) w.r.t. h and velocity of ball at the instant it is thrown = (v + ux)
dt = (v + u cos ). Therefore, the horizontal range of
setting = 0, we have the ball with respect to the ground is
dh
1/2 1/2 R = (v + ucos )t, where t = 3s
dt d 2( H h) 2h
=0 = It is clear that
dh dh g g
Distance travelled by ball in time t + 1.15 = R
2 1 1/2 1 1/2 1
= (H h) ( 1) h i.e. vt + at 2 + 1.15 = (v + u cos )t
g 2 2 2
1 1 1 1 2
= at + 1.15 = (u cos )t
g (H 1/2
h) h1/2 2
1
1 1 H a ( 3 )2 + 1.15 = (10 cos 60°) 3
which gives = =2 2
1/2 1/2
(H h) h h a = 5 ms–2
5
Chapter
Laws of Motion and Friction

REVIEW OF BASIC CONCEPTS SOLUTION


5.1 NEWTON’S FIRST LAW OF MOTION
-
every body
continues in its state of rest or of uniform motion in a 5 –1
straight line unless it is compelled to change that state by v=
2
an external unbalanced force.” –2
Using u v t = 0.5 s in v = u + at
–2
5.2 NEWTON’S SECOND LAW OF MOTION we get a
Force F = ma = 1.5 5 = 7.5 N.
the rate
of change of linear momentum of a body is directly
proportional to the applied force and the change takes 5.2
place in the direction in which the force acts.
Linear Momentum

p = mv SOLUTION
F1 = F2 = 120
dp dv
= m ( m F= F12 F22 2 F1 F2 cos
dt dt
= ma 1
= 102 102 2 10 10
dv 2
a=
dt = 10 N
NOTE F 10 –2
a= =
p–t m 5

(F – t 5.3

5.1
v1 (2 i 3j k) 1
to v 2 ( 3i 2j 3k) 1
5.2 Comprehensive Physics—JEE Advanced

3 1/ 2
SOLUTION 2 10 1
v= 3
1
10 x
[ k = 10–3 2
m = 10–3
or p = mv2 – mv1
1/ 2
1
v= 2 1
= m[ ( 3 i 2j 3 k ) (2 i 3j k x
1/ 2
1
= m ( 5 i j 4k) x v= 2 1
0.5
–1
p = 2 ( 5)2 ( 1)2 (4)2 = 2
1/ 2
= 2 42 12 96 s 1 –1 1
v 2 1 x=
x
p 12.96
F = = 4.32 ~ 4.3 N
t 3 5.3 NEWTON’S THIRD LAW OF MOTION
whenever one
EXAMPLE 5.4
body exerts a force on a second body, the second body

x action there is an equal and opposite reaction


k
F=
x2
LAW OF CONSERVATION OF LINEAR
k = 10–3 2
x 5.4 MOMENTUM
v
x law of conservation of linear momentum
–1
. when no net external force acts on a system
consisting of several particles, the total linear momentum
SOLUTION of the system is conserved, the total linear momentum
dv d v dx dv being the vector sum of the linear momentum of each
F = ma = m =m = mv
dt dx dt dx particle in the system’.
k Recoil of a Gun
F=
x2
k dv k
2
= mv v dv dx
x dx mx 2

k mb mg
2
v dv = x dx vb vg
m
v2 k mb vb + mg vg
= c (i)
2 mx
c v=0
x c=
k
. mb vb + mg vg = 0
m
mb v b
or vg = –
v2 k k mg
=
2 mx m
1/ 2
2k 1
v= 1
m x
5.3

mb vb = v2 v2 2v 2 cos
vg =
mg
= 2v 2 (1 cos ) 2v cos
5.5 IMPULSE 2

A B
t
5.6 CONTACT FORCES

(1) Normal Reaction

Impulse of a force is the product of the average force


and the time for which the force acts and it is equal to
the change in momentum of the body during that time.
–1 normal reaction
or N s.
I = Fav t = p (R).
m
R = mg
5.5
[Fig.
m v R = mg cos

SOLUTION
v1 v2

= mv2 – mv1 = m(v2 – v1)


Fig. 5.2
= m[v2 + (– v1
m v

v = v2 + (– v1 v2 v1

(2) Tension
T

Fig. 5.1

v is [ v1 5.7 FRICTION
v2 = v
v= v12 v22 2v1v2 cos
5.4 Comprehensive Physics—JEE Advanced

–1
or = ( )

5.8 BANKING OF ROUND TRACKS

limiting friction

limiting static friction (fs


s
fs
s =
R
R F

mv 2
F=
kinetic or sliding friction (fk R
( k m v
fk R
k = F
R
k s.
F N; N
Angle of Friction = mg
v v = ( Rg)1/2

f BANKING OF CURVES
R
-

f
= Fig. 5.3
R
–1
( )
Angle of Repose

m
Fig. 5.5 Car on a banked curved
R road

N1 N2
N = N1 + N2
-
N cos mg
N sin

Fig. 5.4
mv 2
N sin = – F cos
f ) = mg sin R
R) = mg cos N cos = mg + F sin
f F= N
= =
R
5.5

F 5.10 MOTION IN A VERTICAL CIRCLE


m
v2 Rg n
= v2 = Rg v O
Rg v2 1 n R A
T1. Since
v R mg
centre O
R .
R

( v = (Rg )1/2

v2
=
Rg

A CYCLIST NEGOTIATING A CURVED


5.9 LEVEL ROAD Fig. 5.7

mv 2
F = T1 + mg =
R
mv 2
or T1 = – mg (i)
N R
B OB mg
N = mg OB
m string is T2 B

F = N = mg mv 2
F = T2 = (ii)
R
At C mg
T3
C
mv 2
F = T3 – mg =
R
mv 2
or T3 = + mg
R

C
Fig. 5.6 mv 2
2 T3 mg
mv /R R
F
A
mv 2
> mg
R T 1.
R m
A
v= gR
5.6 Comprehensive Physics—JEE Advanced

m v 2A SOLUTION
= mg or v 2A = Rg
R

vC
C

vC2 = v 2A + 2 2 Rg
v2 = u2 + 2gh h = 2R Fig. 5.8
vC2 = Rg + 4 Rg = 5 Rg M
dx is m = dx
L
or vC = 5 Rg L
2
m x
o
vC2 2 L
T1 = m g = m (5g + g) = 6 mg M
R = xdx
L o

1 2
5.6 = ML
2
m
L k= 5.8
–1
l
m
r

SOLUTION
–2
30 –1 g .
=2 =2 = .
60
F = kx x SOLUTION

r = L + x.
2
mr =F
2
m(L + x) = kx
2
mL
x= 2
k m
2
0.02 0.5 3.14
= 2
2 0.02 3.14

r= L + x
Tension F = kx = 2 0.05 0.1 N Fig. 5.9

5.7 OQ = r PQ = l
M O = T sin
AB L A
mv 2
i.e. = T sin (i)
B. r
5.7

mg = T cos (ii)
–2
g .
mg 0.2 10
T= = =4N
cos cos 60 SOLUTION
OP OC
v2 OC 1
= = = 60°.
rg OP 2
r = CP = OP sin = 5 sin 60°
v= rg n = 0.2 10 t 60
5 3
–1 =
2

5.9
m

SOLUTION
Fig. 5.11
2
mg m r
N
2
r
g C N sin
2
2 mr = N sin (i)
2 r
mg = N cos (ii)
g
30 mg 0.5 10
N= = 10 N
60 cos cos 60
–2 2
r g . m R sin = N sin
2 2 2 2
4 v r 4 3.14 0.5 0.3 N 10 1
= = 0.3 = 2 ds
g 9.8 mR 0.5 5
2
T= = second = 3.14 s
5.10
m -
R SOLVING PROBLEMS IN MECHANICS BY
OC 5.11 FREE BODY DIAGRAM METHOD
O

Fig. 5.10
5.8 Comprehensive Physics—JEE Advanced

F – R = m 1a (i)

free body diagram R = m 2a (ii)

(1) Two masses tied to a string going over a frictionless


pulley m1 m2 (m1 F = (m1 + m2)a
m2 F
a=
(m1 m2 )
m2 is
m2 F
F2 = m2 a =
(m1 m2 )
(3) Three masses in contact
m1 m2 m3
F
m1
a a m2
m3

Fig. 5.12

m1 is m1g – T.
m1
m 1g – T = m 1a (i)
m2 is (T – m2g).
m2 is
T – m 2g = m 2a (ii)

Fig. 5.14
m1 m2
a= g R m2
m1 m2 m1 m2
R m3
2m1m2
T= g m2 m3
m1 m2

(2) Two masses in contact F – R = m 1a (i)


m1 m2 R – R = m 2a (ii)
F m1 R = m 3a (iii)
a. To
a m2
F
a=
(m1 m2 m3 )
m2 is F2 = R

F 2 = R = R + m 2a

Fig. 5.13 F 2 = m 3a + m 2a
5.9

= (m2 + m3)a F m1
a a
(m2 m3 ) F
F2 = m2 m3
(m1 m2 m3 ) T
m3 is m1 m2 T
m3 F m2 m 3.
F 3 = R = m 3a =
(m1 m2 m3 )

(4) Two masses connected with a string


m1 m2

F m2
a a
m1
T

Fig. 5.17

Fig. 5.15 T = m 3a (i)


T – T = m 2a (ii)
T = m 1a (i) F – T = m 1a (iii)
F – T = m 2a (ii)
F
a=
F (m1 m2 m3 )
a=
m1 m2 m1 m2 is T

m1 F (m2 m3 ) F
T = m 1a = T = (m2 + m3)a =
m1 m2 (m1 m2 m3 )
m2 m3 is T

m3 F
T = m3 a
(m1 m2 m3 )
Fig. 5.16 (6) Two masses connected by a string and suspended
F m1 from a support m1 m2

m2
F
a= F T m1 m2
m1 m2 tension T m1

m2 F
T= T = T + m 1g (i)
m1 m2
T = F + m 2g (ii)
(5) Three masses connected by strings
m1 m2 m3
T = F + (m1 + m2)g
5.10 Comprehensive Physics—JEE Advanced

m1m2 g
T = m 1a =
(m1 m2 )

m 2 < m 1.
m1

f = R = m 1g

Fig. 5.20

m1
Fig. 5.18 T – f = m 1a T – m 1g = m 1a

(7) Two blocks connected by a string passing over a (8) Two blocks connected by a string passing over a

table m1 plane T

m2
a. (a) Mass m1 moving up along the incline with accelera-
tion a

(a)

F.B.D. of 1 F.B.D. of 2

1 2

2
(b)

Fig. 5.19
Fig. 5.21
m1 m2 m1 m2 [see Fig.
T = m 1a (i)
m 2g – T = m 2a (ii) T – mg sin = m 1a (i)
m 2g – T = m 2a (ii)
m2 g (m2 m1 sin ) g
a= a=
(m1 m2 ) (m1 m2 )
5.11

T = m2 (g – a)
m1
f = R = m1g cos
m1

T – m1g sin – f = m 1a
T – m1g sin – m1g cos = m 1a
(b) Mass m1 moves down the incline with acceleration a
m1g sin – T = m1a T – m 2g
= m 2a
m1 sin m2 g
a=
m1 m2
T = m2 (g + a)
(9) Two blocks connected by a string passing over a

plane m1
1 Fig. 5.23
m2
2 T f m1 m2
m1 m2 = R = m 1g
T – m1g sin 1 = m 1a
m2 m1
m2g sin 2 – T = m 2a f m1
T f = m 1a m 1g g=a (i)
m2 sin 2 m1 sin 1 g a
a= R m2
m1 m2
m2
T = m1(a + g sin 1) = m2 (g sin 2 – a)
R + m 2g = R (ii)
F – f = m 2a
F – R = m 2a (iii)
F – m 1g = m 2a

F = (m1 + m2) g
F F m1 m2
Fig. 5.22
F < F m1 m2
(10) One block placed on top of the another
m1 m2 Case 2: The maximum force that can be applied on the
upper block so that it does not slip on the lower block.
. [Fig. 5.24(a)]
Case 1: The maximum force that can be applied on the F F m1
lower block so that the upper block does not slip [Fig. m2
m1 m2
F F m1 does f = R = m 1g
m2 m1 on m2
5.12 Comprehensive Physics—JEE Advanced

-
–2
g .

SOLUTION

Fig. 5.24
R m2
a
m1
F = R = m 1a R = m 1g
or F – m 1g = m 1a (i)
m2
f = m 2a 1m 1g = m 2a
m1 g
a= (ii)
m2

(m1 m2 )m1 g
F =
m2
F < F

F F Fig. 5.26

5.11 Fcos = ma
m F = 10 N F cos 10 cos 60 2
a= 5
m 1

Fcos – f = ma
Fcos – mg = ma
F cos mg
a=
m
10 cos 60 0.2 1 10
=
1
Fig. 5.25 –2
5.13

F sin + R = mg or
SOLUTION
Fsin = mg – R. Since F sin < mg

5.12

m1 m2

AB CD
Fig. 5.27. String AB

CD

Fig. 5.29
P AB
Q R cos = mg
CD CQ
–2 R sin = ma
g . Fig. 5.27
a= g
SOLUTION g
=g
CD is m = 0.5 3
Since string AB AB 5.14
m
P = m1 + m + m2

P = 5.25 10 = 52.5 N a
Q QD
+ m2
= 0.5
Q = 3.4 10 = 34 N
Fig. 5.30
a.
5.13
SOLUTION
m

-
a

Fig. 5.31

T sin
Fig. 5.28 direction is
5.14 Comprehensive Physics—JEE Advanced

T sin = ma (i)

T cos = mg (ii)

a
=
g
a= g = 9.8
2
= 9.8 3 17
Fig. 5.33
5.15
m1 m2
T2 = 2T1 (iii)
- a1 = 2a2

m1
m2 g –2
. m2 a1
m2g – 2T1 =
2
T1
2m2 g 2 2 10 –2
a1 = =
m2 4m1 2 4 1.5
a1 5 –2
a2 = =
2 2

T1 = m1a1 = 1.5 5 = 7.5 N


T2 = 2T1 = 2 7.5 = 15 N

5.16
Fig. 5.32 m1 m2
m1 m2
SOLUTION
a1 a2 m1 m2
x1 x2 m1 m2 is
m1 m2 t m2
m1 F -
x1 m2 m1 m 2.
x1
x2 = or x1 = 2x2
2

d 2 x1 d 2 x2
= 2 a1 = 2a2 (i)
dt 2 dt 2
m1
m2 T1 m1
Fig. 5.34
T2 m 2.
m1 SOLUTION
T 1 = m 1a 1 (i) m1 m2 is f = R1 = m1g.
m2 T
m 2g – T 2 = m 2a 2 (ii) m1 m2
5.15

R = mg = 50
R 1 = m 1g (i)
R1 – mg = ma
F – f – T= 0 (ii)
R1 = m(g + a) = 50 (9.8 + 2.2) = 600 N
(c) mg – R2 = ma
T – f= 0 T=f (iii) R2 = m(g – a) = 50 (9.8 – 2.2) = 380 N
R 2 = R 1 + m 2g NOTE
Using (iii) in (ii)
F – f – f= 0 F = 2f = 2 m1g a
m1 m2 F
F
F = 2 m 1g 5.18
=2 0.5 0.1 10 = 1 N AB M
L A B F1 = 4 N
F2
P x A.

Fig. 5.37

SOLUTION
Since F1 F2
Fig. 5.35
a T P we con-
AP PB
5.17
m Mx
AP is m1 =
L
M
a –1 PB is m2 = (L x)
–2 L
.

SOLUTION AP PB.
R
-

Fig. 5.38

AP F1 – T = m1a (i)
PB T – F2 = m2a (ii)

Mx
F1 T m L x
= 1
T F2 m2 M L x
L x
Fig. 5.36 L
5.16 Comprehensive Physics—JEE Advanced

F1 L x F2 x 5.20
T=
L m
4 1 0.2 3 0.2
=
1
= 3.8 N F
-
5.19
m
a –2 F

g –2
. Fig. 5.41

M
SOLUTION
- R
f

–2
g . Fig. 5.39

SOLUTION
A
T
A
A – a)

Fig. 5.42

f = mg
F= R
mg
f R or mg F or F

mg 2 10
Fmin = = = 80 N
Fig. 5.40 0.25

mg – T = m(A – a) (i)
T – Mg = MA (ii) 5.21
m
M
mg – Mg = m (A – a) + MA
F M
m g a Mg
A=
M m
30 10 5 15 10
=
15 30
–2

–2
A–a
T = M(g + A) = 15 (10 + 6.7)
250 N Fig. 5.43
5.17

-
F m M v
–2
g .

SOLUTION
Fig. 5.45
a
SOLUTION
P Q is f = mg
P Q.
f mg
P is aP = g
m m
f mg
Q is aQ =
M M
Q P is
mg
aQP = aQ – aP = g
M
Fig. 5.44 m
= g 1
M
m F – R = ma (i)
1
f = mg (ii) = 0.3 10 1
6
M R = Ma (iii) –2

Mg + f = R
a 5.23
MF m = 500 g
R=
M m
f R –2
a
MF
or mg
M m
mg M m SOLUTION
or F
M g =g+a
mg M m
F =
M
2 10 10 2
= = 60 N mg sin = mg cos
0.4 10
0.58
5.22
5.24
P m Q
m
M
M
P
–1
v -
P Q is -
Q P.
5.18 Comprehensive Physics—JEE Advanced

1 cos 45 sin 45
= 2
0.2
2 1 cos 45

5.25

Fig. 5.46
1 2
a
SOLUTION 2
-
Fx -
Fy SOLUTION
A
Fx = (mg cos ) sin -
Fy = (mg cos ) cos -
Mg a is (A cos –a
Fig. 5.47

m (A cos – a) = Ma
F = mg cos sin
M m a
A=
m cos
N = Mg + mg cos2
2 1 1/ 2
=
F m cos sin 1 cos 45
=
N M m cos 2 –2

Multiple Choice Questions with Only One Choice Correct


1. m 2u
(c) (d) u cos
cos
g
m

3 mg 5 mg
2 3
7 mg 9 mg
(c) (d)
6 7
2. P
Q
u B C
M
u
u cos
cos Fig. 5.48
5.19

3. P m 7. m
M Q - F
P -
v
P Q Q
P is mg mg
mg
g (c) F
M
8. m = 10–2
m m x
(c) g 1 (d) g 1
M M k
F= –
2 x2
k = 10–2 2

x v
Fig. 5.49 x
–1 –1
4. –1 –1

a
9.

a
=
g
a a
(c) = 1 (d) = 1
g g
5. m 2 1
(c) = (d) =
m n n
- 10. A B
-

1
(c) 0.3 (d)
2
A B
(c) A B
(d) A B
11.

Fig. 5.50

6.
a 12. N m -
v –1
n

2 a 2 2 a N mv
nNmv
(c) 3 2 a (d) 4 2 a n
5.20 Comprehensive Physics—JEE Advanced

nN m nN v r -
(c) (d)
v m
13.

r v r v2
(c) r2 v (d) r2 v2
( 18. m
(c) 10 N (d) 20 N v
14. A

B mv cos ( mv sin ( )

(c) 2 mv cos (d) 2 mv sin


2 2
19. x–t

A
B.

B
A.
15.
h

2h 2h Fig. 5.52
g g
(c) 8 s (d) 16 s
1 2h 1 2h
(c) · (d) · 20.
sin g cos g
16. M (c) 0.8 Ns (d) 1.6 Ns
21. M v
s

M g sin Mg cos
(c) 2 Mg cos (d) 2 Mg sin v2 v2
M g M g
2s 2s

2v 2 2v 2
(c) M 2 g (d) M 2 g
s s
22. m
u
Fig. 5.51
17. -
v
5.21

u2 u2
2 g sin 2 g cos
u2 u2
(c) (d)
4 g sin 4 g cos
23.
–2
–2
g

Fig. 5.55

27. n

Fig. 5.53

(c) 60 N (d) 600 N


k = 1/(1 – n2 k = 1 – 1/n2
24.
(c) k = 1 / 1 n2 (d) k = 1 1 / n2
28.
–2 –2
–2 –2 x (x) = kx
k x
25.
x0
k
–2 k
. At
cot
–2 (c) (d) k cot
g . k
29.

= 0.5
x x

x
2
x x=

2
(c) x = 2 cot (d) x =
cot
Fig. 5.54 30.
in l
26. m1 m2
-
m1
m2 g l2 1 g (l2 – 1)
g g
(c) (d) 2
–2 2 l 1
g . l 1
5.22 Comprehensive Physics—JEE Advanced

31. t2 sin t2
1
R (c) (d) =1
t1 sin 2 t1
h
10 = 0.95) 36.
R R
)
5 10 g –2
)
R R
(c) (d)
20 30
32. m (c) 40 N (d) 60 N
M 37. M m

m a.
M M
ma Ma
M m M m F M
ma Ma –2
(c) (d) g .
M m
33. m M

M
F m is a
M

Fig. 5.57

(c) 24 N (d) 96 N
Fig. 5.56 38. m1 m2
-
F ma F ma
M M m1
F am
(c) (d) m1 m2
M M
m
34.
m1 m1 + m
g

g 2g
3 3
(c) g
35.
1
t1

2 Fig. 5.58
t 2. two
t1 t2 is
1/ 2
t2 sin 1 t2 sin 2 1 39.
2
t1 sin 2 t1 sin 2 -
5.23

tion 1/ 2 1/ 2
a a
(c) t 1 (d) t 1
is to it. g g
45.
–2
g )
–2 –2
–2 –2 F
40.
x 1.
2F
F
x2 - 3
3F 5F
x1/x2 is (c) (d)
5 6
2 46. F m
(c) 3 (d) 2 a
41. F1 = F2 = F3 = F

a
g
g
2
g g
(c) (d)
2 3
42. m

F1
-
Fig. 5.59
F2
2 1 a 2 1 a
1/ 2 3 F1 F2 is
(c) 2 a (d) a
F2 = F1 F2 = 2F1 47. l1
(c) F2 = 3F1 (d) F2 = 4F1 l2
43.

l1 + l2
1
(l1 + l2)
2
(c) 3l2 – 2l1
44. (d) 3l1 – 2l2
h t
48. -
a
h A B
Fig A
a B
t t
g
Fig. 5.60
5.24 Comprehensive Physics—JEE Advanced

10 2 2 10

53.
49. M
strings A B C W
R -
tension in string B is

(c) 0.75 (d) 0.85


54.
–1 –1
50 .

(c) 300 (d) 480


55. M

Fig. 5.61
–2
100 g )
100
(c) 100 2 g newton (d) g newton (c) 60° (d) 90°
2 56.
50. -
i
F = 600 – 2 105 t j
F t –5

(2 i 3j) 10–5 (2 i 3j) 105


(c) (3 j 2i ) 105 (d) (2 i 3j) 10–5
(c) 0.9 Ns (d) 1.8 Ns 57.
51. –1

L
A
A

58.

1 -
(c)
–2 –2
g )

52. F = 6 i – 8 j + 10 k (c) 6 (d) 8


–2
59.
5.25

=3
(d) cot (c) sec =3 (d) cosec =3
60.
64.

(c) 45° (d) 60°


descending

descending

61. m1 m2
1 2
t 1 2
t0
m1 1 m2 2) (m1 1 m2 2 is Fig. 5.64
m1 + m2)gt0 65.
1 m M
(c) 2(m1 + m2)gt0 (d) (m1 + m2)gt0
2

62. F 2 Mg
2 mg

g –2
) (c) (M m) 2 m2 g
(See Fig. 5.62) (d) (M m) 2 M2 g
(c) 12 N (d) 15 N

Fig. 5.62
63.
Fig. 5.65

66. m
L M

Fig. 5.63
5.26 Comprehensive Physics—JEE Advanced

mL ML
M m
72. m
mL mL f
(c) (d)
M m M m
67. m u

m f
g
m
f f
(c) g + (d) g –
u 2 sin 2 3u 2 sin 2 m m
g 2g 73.

2u 2 sin 2 3u 2 sin 2 g –2
(c) (d)
g g
68. a -
(c) 500 N (d) 600 N
74.
m1 m2
v
av2 cos av2 cos
v1 v2
(c) 2 av2 sin (d) av2 sin
x1 x2
69. x1/x2 is
t seconds to m2
m1
m2 m1
t - m1 m2
(c) (d)
m2 m1
0.5 0.75
(c) 0.5 (d) 0.75 1
70.
v
Fig. 5.66

v/n - 75. M -
n tion a m

1 Ma mg Ma mg
= 1 M m M m
n2
1 ma Mg ma Mg
= 1 2
cot (c) (d)
n M m M m
1 1/ 2 76. -
(c) = 1 2
n
1/ 2
1
(d) = 1 cot
n2 n
71.
–1
n

g
–2
) (c) 3 (d) 4
5.27

77. m
F is N = F

F (d) N
= cot
(c) = sin (d) = cos
78. F
mg mg
1 1
2
mg mg
(c) (d)
2 2
1 1
79. T
Fig. 5.68

82. –1

g g
–2
4 2 R g
3g R is
(c) (d) g
4
80. P m
Q 83. A
P
k –1
–2
g .
A
P Q
P Q is 84.
–1 –2
g
kA
mg
2
(c) kA –1 –1
(0.25)
–1 –1
(c) (0.35) (d) (0.45)
85.
R -

gR
gR
2
(c) 2gR (d) 2 gR
Fig. 5.67 86. -
–2
g
81. m
F
–1 –1
2
–1 –1
f = mg (c) 10 3
5.28 Comprehensive Physics—JEE Advanced

87. A B m m 90.
-
3
-
m
m
g g
g
g
2
g
(c) g
2
g g
(d) Fig. 5.69
2 2
.
88. x–y -

91.
p (t) = A[ i cos (kt) – j sin (kt
y = kx2 (y m on
A k

(c) 45° (d) 90° x a

89. m
a y
a a
a’ gk 2 gk
P
2a a
(c) (d)
F gk 4 gk

92. m
x L -
F a F x
2m a 2
x2 2m a 2
x2
F x F a2 x2
(c) (d)
2m a 2m x

Fig. 5.71

(c) 27 (d) 36
Fig. 5.70
5.29

ANSWERS

1. (c) 2. 3. (d) 4. 5. 6. (c)


7. 8. 9. 10. (d) 11. 12.
13. (c) 14. (c) 15. (c) 16. (d) 17. (d) 18. (c)
19. 20. (c) 21. 22. (c) 23. (c) 24.
25. 26. 27. 28. 29. 30. (c)
31. (c) 32. (c) 33. 34. 35. (c) 36. (c)
37. (d) 38. (c) 39. 40. (c) 41. (d) 42. (c)
43. 44. (c) 45. (d) 46. 47. (c) 48. (c)
49. 50. (c) 51. 52. 53. 54. (d)
55. (c) 56. 57. (d) 58. 59. (c) 60.
61. (c) 62. 63. 64. (c) 65. (d) 66. (c)
67. 68. 69. (d) 70. 71. 72. (d)
73. (c) 74. 75. 76. (c) 77. 78. (d)
79. 80. 81. (d) 82. 83. 84. (d)
85. 86. 87. (c) 88. (d) 89. 90.
91. 92. (d)

SOLUTIONS
1. T a - f mg
Q is aQ = =
M M
3g
a . Q P = aP + aQ
2
m
= g 1
M
4. g = (g – a
T – mg = ma (1)
mg m 3g
T– = a (2)
2 2 2 m (g – a) sin = m(g – a) cos
T= =
7 mg
5.
6
F = mg cos sin
2. AB = AC = r OB = OC = x OA = y
AOB N = 2 mg + mg cos2
r2 = x2 + y2 Now F = N
t F mg cos sin
= =
dr dy N 2 mg mg cos 2
2r = 0 + 2y
dt dt cos sin
=
dy r dr 2 cos 2
=
dt y dt For
dr dy 6. A
Now = u y = r cos
dt dt
M -
u tion a is (A cos – a
.
cos
N
3. P Q is f = mg
P Q. N sin = (2 m) a = m (A cos – a)
f mg 3ma
P is aP = = = g A=
m m m cos
5.30 Comprehensive Physics—JEE Advanced

3a 2aA sA = v2A a B s B = v 2B
= = 3 2 a
cos 45
7. mA aA = mB aB
sA mB 70 7
. Since
f = mg f) = N sB mA 50 5
= F sA + sB sA sB
dv d v dx dv
8. F = ma = m =m = mv
dt dx dt dx
11.
k
F=–
2 x2
k dv k
– 2
= mv v dv = – x–2 dx
2x dx 2m

k 12.
v dv = – x–2 dx
2m
v2 k
= +C (1)
2 2 mx n Nm n Nmv n
C -
v x nNmv
we get C = – k/2m .
v2 k 1 k 1 1/ 2 13.
= 1 v= 1
2 2m x m x
k = 10–2 2
m = 10–2 x=
v –1
.
9.
g sin . 14. m
AB = L AC = h
- -

g sin – g cos g cos


g sin mg cos
R
mg sin .
g sin = – (g sin – g cos )
or cos = 2 sin is
a = g sin

10. mA mB A B
aA aB

vA v
mA aA = mB aB or mA = mB B
t t
or mA vA = mBvB or mA vA = m2B v2B
2 2
(i)
Fig. 5.72
vA vB t
sA sB A
B
v2 – u2 = 2 as
5.31

or v2 – 0 = 2aL 2 0
v= –1

or v= 2aL 2 g sin L 2 gh 2 0 s
At t
h = L sin –1
v = u + at or v = 0 + at.

= mv – (– mv) = 2 mv
v 2 gh 1 2h –1
t= =2
a g sin sin g –1
= 0.8 Ns
15.
16. Mg 21.
Mg sin - v s
Mg sin . To R = Mg
- a v
- s v2 – u2 = 2as u
nent Mg sin v2 = 2 as or a = v2/2s
Mg sin = 2 Mg sin
F1
17. A) = r2
v - M v2
=
Av = r2v 2s
r 2v
F2 = Mg
v2
F1 + F2 = M g
2s
= r2 v2
22. -

mg sin mg
F = mg sin -
r 2 v 2.

r 2 v 2. a = – g sin – g sin
= – 2g sin
18.

19. t t = 2s is

x = 0 to x
t = 0 to 2s. Between t = 2s t

t
Fig. 5.73
x t s
t v2 – u2 = 2as
0 – u2 = 2 (– 2g sin ) s or
u2
20. Between t t s=
is v x–t t 4 g sin
t
5.32 Comprehensive Physics—JEE Advanced

23. t
–2
a a= s
–2
mg u
= 0. Now t
1 2
a s = ut + at
2
–2
s u a=a
F = ma = 60 1.0 = 60 N t= 10 s.
t =
n f 10 s is ( u = 0)
24. s =
nor n mg 1 2
Since - s = ut + at
s 2
f= s mg = 1
0.2 60 10 = 120 N =0+ 3
2
26. m2 = 6
- m2 T = 60 N
ac -
mac nent F1 = m1g sin m1g. Now F1 = m1g
f sin = 5 10 sin 30° = 25 N. Since F1
T T
f 120 –2
mac = f or ac = F
m 60 m1 f
m1
25. - m1
–2
a
is F = ma = 10 3 = 30 N
mg -
R -
F = 30 N

F f
Fig. 5.74
f f
= T – m1g sin 30° – f = 0
R mg
or f = mg = 0.2 10 10 or f = T – m1g sin 30°
= 20 N = 60 – 5 10 sin 30°
= 60 – 25 = 35 N
F
f 27. -
-
F =F–f=
30 – 20 = 10 N g
a1 = g sin 45° =
- 2
a2 = (g sin 45° – k g cos 45°)
F 10 –2 g
a = = (1– k)
m 10 2
5.33

31. A
t22 a1 1 1
= n2 or k =1– .
t12 a2 1 k n2 h. Now OA = OC = R
mg sin
28. F = mg
x is cos
mg sin mg cos mg sin
= mg cos =
OB
OAB OB = x AB
2 AB
= R2 x2 .

Fig. 5.75

f (x) = mg sin – mg cos


= mg (sin – cos )
= mg (sin – kx cos )
f (x x = x0
sin – kx0 cos = 0
n
x0 =
k
Fig. 5.77
29.
g sin – g cos = g (sin – cos ) = g (sin – x
=
0.5x cos R 2
x2
x
x x x 1
or 3= ( = )
R2 x2 3
30. AB = 1 AC = l 3R
2 x = = 0.95 R h =
BC = l 1 = AB/BC = 1/ l 2 1 . A 10
a R – x = R – 0.95 R = 0.05 R
a cos
g sin g
32.
m is F ma.
a cos = g sin M m.
m
g
or a=g = F on M. on m2 =
l2 1 F = ma M is
F ma
a =
M M

33. m is f = ma m
M f = ma.

M = F – f = F – ma
F ma
M =
M
Fig. 5.76
5.34 Comprehensive Physics—JEE Advanced

34. a m
mg – T = ma a=
ss m
or T = mg – ma
2 1mg
T= mg
= = 1g (ii)
m
3 Using (ii) in (i) we get
2
mg = mg – ma F= 1 (M + m)g + 2 (M + m)g
3
a=g =( 1 + 2) (M + m) g
= (0.5 + 0.7) (5 + 3) 10
35. h
h = 96 N
s1 =
sin
h 1
38 .
s2 =
sin 2
m m1
a1 = g sin 1 a2 = g sin 2 -
(m1 + m
-
v12 = 2a1 s1 v22 = 2a2 s2 m 2g
v 1 = a 1t 1 v2 = a2t2 or a12 t11 (m1 + m)g m1 m
2 2 m2 6
= 2a1 s1 a2 t2 = 2a2 s2 (m1 + m)g = m2g or m = – m1 = –4
0.4
t22 a1 s2
=
t12 a2 s1 39. m s k
g sin 1 h sin 1
= F= mg = 0.7 5 10 = 35 N.
g sin 2 sin 2 h s

sin 2 1
= 2
sin 2 f
= F – mk mg = 35 – 0.5 5 10 = 35 – 25
36. m l
= 10 N
tension T x
mg f 10 –2
l–x (l – x) or a=
l m 5
6 10
T= 3 1 = 40 N 40. u v
3
1 = 30° is a1 = g sin 1
37. m 2 = 60° is a2 = g sin 2 v2 – u2
M= 1 mg 1
= 2ax u2 = 2a1x1 = 2a2x2
x1 a
= 2
M m x2 a1
2 (M + m)g
g sin 2 sin 60
2 M = 3
F g sin 1 sin 30
M
41.
a a a cos
M + m)a g
g sin a cos =
F = (M + m)a + 2 (M + m)g (i)
g sin or a = g
m is 1mg - g
Now a = g .
3
5.35

F
a=
M m

m
T= M a
2
m
Fig. 5.78
M F
2
=
42. F1 M m

M 5F
F1 = mg sin – mg cos (i) m= T=
2 6
F2

F2 = mg sin + mg cos (ii) F


46. a= F1
m
F2 + F1 = 2 mg sin F2 F
F2 – F1 = 2 mg cos F = (F12 + F22)1/2 = 2F( F1 = F2 = F)
F F 3.
F2 F1 n t 30
= =2 F – F3
F2 F1 1/ 2 3 = 2 F–F
or F2 = 3 F1
= ( 2 – 1) F
43. m
l F
2 – 1) = ( 2 – 1)a.
L m
L–l
l = ml L– l)
= m (L – l mlg

m(L – l)

mlg = m (L – l) g
L 0.5 120
or l=
1 0.5 1

Fig. 5.79
1 2
44. h= gt
2
a g = 47. F
1 x is
g+a t h= gt2 F = kx
2
k l0
1 1 2
gt2= gt or (g + a)t 2 = gt2 F = k(l – l0)
2 2 l
1/ 2 F
a
or t = t 1 2 = k(l1 – l0) (i)
g
k(l2 – l0) (ii)
45. M m
5.36 Comprehensive Physics—JEE Advanced

3
3 l2 l0 t 3 10 s
= (600 t 105 t 2
0
2 l1 l0
= 600 3 10–3 – 10 (3 10–3)2
5
Which gives l0 = 3l1 – 2l2. Using this value of l0 in
= 1.8 – 0.9 = 0.9 Ns
1
either (i) or (ii) we get k = . Hence the correct choice is (c).
l2 l1
51. The radius of the circular motion of the bead is
When a stretching force of 5 N is applied, let l3 be r = L. The linear acceleration of the bead is a =
the length of the spring. Then r = L. If m is the mass of the bead, then
5 = k(l3 – l0) Force acting on the bead = ma = m L
Substituting the values of l0 and k, and solving we get Reaction force acting on the bead is R = m L
l3 = 3l2 – 2l1 The bead starts slipping when frictional force be-
Hence the correct choice is (c). tween the bead and the rod becomes equal to cen-
48. If the springs A and B are massless or their mass is trifugal force acting on the bead, i.e.
negligible compared to the mass with which they m v2
are loaded, the tension is the same everywhere on R=
r
the spring. Hence each spring balance will read or m L = mr 2 = mL 2 ( v=r )
4 kg. Thus the correct choice is (c). or = 2 = ( t)2 ( = t)
49. Let T be the tension in string C and T in string B.
The y-component T cos balance with the weight or = ( t)2 or t =
Mg and the x-component T sin balances with ten-
sion T . Thus (see Fig. 5.80) 52. The magnitude of the force is
T = T sin
F = F F = [(6 i – 8 j + 10 k ) (6 i – 8 j + 10 k )]1/2
and Mg = T cos
Dividing the two we get = {(6)2 + (8)2 + (10)2]1/2 = (200)1/2
T = Mg tan = 10 2 N
= 100 g tan 45°
F 10 2 N
= 100 g newton Mass = = = 10 2 kg. Hence the
a 1 ms 2
Hence the correct choice is (a).
correct choice is (b).
53. Refer to Fig. 5.81. Since the block moves with a
constant velocity, no net force acts on it. Therefore,
the horizontal component F cos of force F must
balance with the frictional force, i.e. fr = F cos .

Fig. 5.80

50. Now F = 600 – 2 105 t. F will be zero at time t


given by
600 – 2 105 t = 0 or t = 3 10–3 s
t Fig. 5.81
Therefore, impulse = F dt Also fr = (mg – F sin )
0
= (f – F sin )
t
= (600 – 2 105 t)dt (f – F sin ) = F cos
0 or (200 – 100 sin 30°) = 100 cos 30°
Laws of Motion and Friction 5.37

1
or 200 100 = 100 0.866 (2 i + 3 j) kgms 1
2 = 5
= 86.6 10 s
86.6
or = = 0.58, = (2i + 3 j) 105 newton
150
which is choice (b). Hence the correct choice is (b).
54. Change of momentum of one bullet = m (v – u) 57. Magnitude of recoil momentum of the gun = forward
= 0.03 {50 – (–30)} momentum of the bullet = mbvb = (40 10–3 kg)
1200 ms–1 = 48 kg ms–1
= 2.4 kg ms–1
n bullets per
Average force = rate of change of momentum
of 200 bullets
gun is
= 200 2.4 = 480 N, which is choice (d).
F =n
55. Let the body leave the surface at point B as shown
change in momentum
in Fig. 5.82. When the body is between points A and =n
B, we have time
M v2 48 kg ms 1
n
Mg cos – N = =
r 1s
= 48 n newton
Given F = 144 N. Thus 144 = 48 n which gives
n = 3. Hence the correct choice is (d).
58. Force
F = ( s – k) mg
= (0.8 – 0.6) 4 10 = 8 N
F 8N
Acceleration = = 2ms–2, which is
choice (a). m 4 kg
Fig. 5.82 59. No net force acts on the block as it moves at a
When the body leaves the surface at point B, the constant velocity. Therefore, downward force =
normal reaction N becomes zero. Thus upward force or mg sin = mg cos or = tan ,
which is choice (c).
M v2
Mg cos = 60. When a cylinder rolls up or down an inclined
r
plane, its angular acceleration is always directed
v2 (5)2 down the plane. Hence the frictional force acts up
or cos = =
rg 5 10 the inclined plane when the cylinder rolls up or
1 down the plane. Thus, the correct choice is (b).
= or = 60°
2 61. Linear momentum of the system at time t = 0
Hence the correct choice is (c). is p1 (m1 v1 m2 v2 ) and at time t = 2t0 it is
56. Mass of each piece (m) = 1 kg. Initial momentum p2 (m1 v1 m2 v2 ) . Change in linear momen-
= 0. Final momentum = p1 + p2 + p3. From the
tum in time 2t0 = p2 p1 . The rate of change
principle of conservation of momentum, we have
of linear momentum is p2 p1 /2t0. From New-
p1 + p2 + p3 = 0
ton’s second law of motion, the rate of change of
or p3 = – (p1 + p2)
= – (mv1 + mv2) momentum equals the force acting on the two
= – m (v1 + v2) particles, which is (m1 g + m2 g ). Hence
( p2 p1 )
= –1 kg (2 i 3 j) ms–1 = (m1 + m2) g
2t 0
= (2 i 3 j) kg ms–1 or (m1 v1 + m2 v2 ) – (m1 v1 + m2 v2 )
p = (m1 + m2) g (2t0)
Force F= 3
t Hence the correct choice is (c).
5.38 Comprehensive Physics—JEE Advanced

62. The horizontal component of F parallel to the sur- mv – (M + m)V = 0


face is F sin F is given V m
by or =
v M m
F sin = mg Since the distance moved is proportional to speed,
or F sin 60° = 0.5 3 10 the displacement L of the plank is given by
3 L V m
or F = 0.5 3 10 =
2 L v M m
which gives F = 10 N. Hence the correct choice is mL
(b). or L =
M m
63. As shown in Fig. 5.83, the insect will crawl without Hence the correct choice is (c).
slipping if the value of is not greater than that 67. At the highest point of trajectory, the projectile has
given by the condition: force of friction f = mg sin . only a horizontal velocity which is u cos . After
Now f = N, where N is the normal reaction. Thus
N = mg sin horizontal velocity. If u is the horizontal velocity
or mg cos = mg sin or of the other fragment, the law of conservation of
1 momentum gives
cot = = 3, which is choice (a).
(2m) u cos = m 0 + mu
which gives u = 2u cos
Now, the time taken to reach the highest point (as
well as the time taken to fall down from this point)
u sin
is . Therefore, the horizontal distance trav-
g
elled by the other fragment is
u sin u sin
Fig. 5.83 u cos + 2 u cos
g g
64. Let T be the tension in the string. When the system
u 2 sin 2 u 2 sin 2 3u 2 sin 2
is in equilibrium, then for the two equal masses m, =
we have 2g g 2g
T = mg (1) Hence the correct choice is (b).
and for the mass 2 m, we have 68. The mass of water stream striking against the wall
in 1 second = av . Hence, the change in its momen-
2T cos = 2 mg (2)
1 tum per second is (av )v – (– av )v = 2a v2. The
Dividing (2) by (1), we get cos = or = 45°, normal component of the rate of change of momen-
which is choice (c). 2 tum and, therefore, force is 2a v2 cos . Hence the
65. The force F on the pulley by the clamp is given by correct choice is (a).
the resultant of two forces: tension T = Mg acting 69. The acceleration of the block sliding down the
horizontally and a force (m + M)g acting vertically smooth inclined plane is a1 = g sin and down
downwards. Thus the rough inclined plane is a2 = g sin – g cos .
F= ( Mg )2 {(m M ) g}2 Given t1 = t and t2 = 2t. If the length of the inclined
plane is s, we have
= [M2 + (m + M)2]1/2 g 1 1
which is choice (d). s = a1t12 = a2t 22
2 2
66. Before the boy starts walking on the plank, both
or a1t 21 = a2t 22
the boy and the plank are at rest. Therefore, the
total momentum of the boy–plank system is zero. or g sin t2 = (g sin – g cos ) (2t)2
If the boy walks with a speed v on the plank and
or sin = 4 (sin – cos )
as a result if the speed of the plank in the oppo-
site direction is V, then the total momentum of the 3 3
which gives = tan = ( = 45°)
system is mv – (M + m)V. From the principle of 4 4
conservation of momentum, we have Hence the correct choice is (d).
Laws of Motion and Friction 5.39

70. We use the relation v2 – u2 = 2as. Since u = 0, we 75. The forces acting on the balloon are its weight act-
have v2 = 2as. ing downwards and upthrust F acting upwards.
Now v 21 = 2a1s or v2 = 2g sin s ( v1= v) Thus
v 2
F – Mg = Ma (i)
and v22 = 2 a2s or 2 = 2(g sin – g cos ) s
n When mass m is removed, we have
Dividing, we get or n2 (sin – cos ) = sin F – (M – m) g = (M – m)a (ii)
1
which gives = 1 tan , which is choice (a). where a is the new acceleration. Eliminating F
n2 from (i) and (ii) and simplifying we get
71. The force of friction between the block and the belt
Ma mg
is f = mg, where m is the mass of the object. This a =
force produces an acceleration of the block which M m
is given by which is choice (a).
force mg 76. Let m be the mass per unit length of the rope. Let
a= = g x t. The
mass m weight of this part is
The block will slide on the belt without slipping
F1 = mgx
until its speed (v) becomes equal to the speed of the
Now, if a small part dx dt,
belt. Since u = 0, we have
v2 = 2 as
F2 = rate of change of momentum
v2 v2 52 mdx v
or s = = 2.5 m = = mv2
2a 2 g 2 0.5 10 dt
Hence the correct choice is (b). dx
Now = v, where v is the velocity of that part
72. Normal reaction R = f. Therefore, force of fric- dt
tion = R = f. The net downward force F = mg of the rope at that instant. But v2 = 2gx. Hence
F mg f F2 = mv2 = m (2gx) = 2mgx. Total force F = F1 +
– f. Hence, the acceleration a = = F2 = mgx + 2mgx = 3mgx = 3F1
m m
f Hence the correct choice is (c).
g – . Hence the correct choice is (d).
m 77. Refer to Fig. 5.84. Vertical component of F is
73. As the boy is climbing the pole at a constant speed F sin and the horizontal component is F cos .
(no acceleration), the force of friction must be just
mg
balanced by his weight, i.e. R = mg or R = =
40 10
= 500 N. Hence the correct choice is (c).
0.8
74. From the principle of conservation of momentum,
we have
v m2
m1v1 = m2v2 or 1 (i)
v2 m1
Fig. 5.84
opposite force F on each block. Let a1 and a2 be
the accelerations of blocks m1 and m2 respectively. Thus
Then R + F sin = mg
a m1 or R = mg – F sin
F = m1a1 = m2a2 or 2 (ii)
a1 m2 Frictional force R = (mg – F sin ). Also
Also v21 = 2a1x1 and v22 = 2a2x2, which give (mg – F sin ) = F cos
2
x1 v12 a2 m2 m1 m2 mg
= . = or F= (i)
x2 v22 a1 m1 m2 m1 sin cos
[Use Eqs. (i) and (ii)] F
Hence the correct choice is (b). mum, i.e. if
5.40 Comprehensive Physics—JEE Advanced

d v2 10 10
( sin + cos ) = 0 or Rmin = = = 12.5 m
d g 0.8 10
or cos – sin = 0
This is the minimum radius the curve must have for
or = tan , which is choice (a). the car to negotiate it without sliding at a speed of
1
78. Now tan = . Therefore, cos = and 10 ms–1. Hence the correct choices is (b).
2
1 83. Speed of train (v) = 36 km h–1 = 10 ms–1
sin =
2 Radius of the curve (R) = 200 m
1
Distance between rails (x) = 1.5 m
Using these in Eq. (i) above and simplifying, we
Let the outer rails be raised by a height h with
get
respect to the inner rails so that the angle of bank-
mg ing is (Fig. 5.85).
F=
2
1 h v2
Then tan = =
Hence the correct choice is (d). x Rg
79. Force required to accelerate the body of mass m is
F = ( s – k) mg = (0.75 – 0.5) mg = 0.25 mg xv 2 1.5 (10)2
or h= =
F 0.25 mg Rg 200 10
Acceleration = = = 0.25 g, which is = 0.075 m = 7.5 cm
m m
choice (a). Thus, the correct choice is (a).
80. Let a be the acceleration at a time t of the blocks

acceleration is
F = (m + m) a = 2 ma
F =2ma (1)

displacement, i.e. Fig. 5.85


F = kA (2)
84. Now v = 54 km h–1 = 15 ms–1, R = 50 m. The re-
Equating (1) and (2), we get quired angle of banking is given by
kA v2 15 15
a = tan = = = 0.45
2m Rg 50 10
force of friction = ma Thus, the correct choice is (d).
kA kA 85. The horizontal velocity v must be such that the
=m
2m 2 centripetal force equals the weigth of the body, i.e.
Hence the correct choice is (b). mv 2
= mg or v = gR , which is choice (b).
81. Since the block is held stationary, it is in trans- R
lational as well as rotational equilibrium. Hence 86. The motor cyclist can leave the ground only at the
no net force and no net torque acts on the block. highest point on the bridge. At this point, the cen-
No net force will act on the block if f = mg and tripetal force is mv2/R. He will not leave the ground
N = F. No net torque will act on the block, if torque if the centripetal force equals the weight mg. Thus
by frictional force f about centre O = counter torque
mv 2
by normal reaction N about centre O. Hence choice = mg or v = gR = 10 10 = 10 ms–1.
(d) is false. R
Hence, the correct choice is (a).
82. Speed of car (v) = 36 km h–1 = 10 ms–1 -
87. When the system is in equilibrium, the spring force
mum centripetal force that friction can provide is
= 3 mg. When the string is cut, the net force on
mv 2 block A = 3 mg – 2 mg = mg. Hence the acceleration
f = mg =
R of this block at this instant is
Laws of Motion and Friction 5.41

force on block A mg g
a= = =
mass of block A 2m 2
When the string is cut, the block B falls freely with
an acceleration equal to g. Hence the correct choice
is (c).
dp
88. Force F =
dt
d
= [A{ i cos (kt) – j sin (kt)]
dt Fig. 5.87

= Ak [– i sin (kt) – j cos (kt)] The block will topple if the torque due to normal
reaction N about O
Now F p = Ak[– i sin (kt) – j cos kt] mg sin about O, i.e.
N OA = mg sin OB
A[ i cos (kt) – j sin (kt)]
mg cos 5 cm = mg sin 15 cm
= A2k[– sin (kt) cos (kt) + cos (kt) sin kt] 2
2
tan = 34°.
=0( i i = j j = 1 and i j = 0) 3
Hence the angle between F and p is 90° which is Since for toppling is less than for sliding, the
choice (d). correct choice is (b).
91.
89. Refer to Fig. 5.86. Let f be the force producing
the acceleration of each mass. It follows from the

F = T sin + T sin = 2 T sin


F
T= (1)
2sin
Also T cos = mf (2)
Using (1) and (2), we get
F cos F Fx
f= = =
2m sin 2m tan 2m a 2 x2
Hence the correct choice is (b). Fig. 5.88

For the bead to stay at rest, (see Fig. 5.88)


N cos = mg
N sin = ma
a
which give tan = . Now
g
dy d
tan = slope of the curve = = (kx2) = 2 kx
dx dx
a a
2kx = x=
g 2 gk
Fig. 5.86 92. Radius of the circular path is BC = r = L sin , where
90. The block will just begin to slide if the downward L = AB is the length of the string. The vertical com-
force mg sin just overcomes the frictional force, ponent T cos of tension T balances with the weight
i.e. if mg sin = N = mg cos tan = = mg and the horizontal component T sin provides
the necessary centripetal force for circular motion.
3 = 60° (see Fig. 5.87) Hence
5.42 Comprehensive Physics—JEE Advanced

2 2
T sin = mr = m(L sin )
2
T = mL
2
T = mL
2
324 = 0.5 0.5
= 36 rad s–1

Fig. 5.89

II

Multiple Choice Questions with One or More Choices Correct


1. Which of the following statements are true? No (c) can go up to a speed of 100 ms–1 in 10 s.
net force acts on (d) after acquiring a speed of 50 ms–1, can come
(a) a drop of rain falling vertically with a con- to rest, with the engine shut off and brakes
stant speed not applied, in a time of 10 s.
4.
(c) a car moving with a constant velocity on a speed of 15 ms–1 gushes out of a tube of cross
rough road sectional area 1 cm2, strikes against a kachha ver-
(d) a body moving in a circular path at constant
speed.
2. In which of the following situations would a force N on it. The impact of the water stream on the
of 9.8 N act on a stone of mass 1 kg? Neglect air wall will
(a) Just after it is dropped from the window of (
stationary train to the water stream is only about 1500 N.
(b) Just after it is dropped from the window (b) will break it.
of a train running at a constant speed of
36 km h–1. 105 pascal on
(c) Just after it is dropped from the window of a the wall.
train accelerating at 1 ms–2.
5. A block is released from the top of a smooth
inclined plane. Another block of the same mass is
which is accelerating at 1 ms–2.
allowed to fall freely from the top of the inclined
3. plane. Both blocks reach the bottom of the plane
a car and the ground is 0.5. The car starts from (a) in equal time
rest and moves along a perfectly horizontal road. If (b) with equal speed
g = 10 ms–2, the car (c) with equal momentum
(d) with equal kinetic energy
5 ms–2 without slipping.
6. A body of mass 200 g is moving with a velocity of
(b) can attain a speed of 20 ms–1 in a minimum
5 ms–1 along the posotive x- direction. At time t = 0
distance of 40 m.
when the body is at x = 0, a constant forse of 0.4 N
Laws of Motion and Friction 5.43

directed along the negative x-direction is applied to


the body for 10s. (c) T = W2 F2 (d) F = W tan
(a) At time t = 2.5 s, the body will be at x =1.25 m
(b) At time t = 2.5 s, the speed of the body will
be zero.
(c) At time t = 30 s, the body will return to x = 0.
(d) At time t = 30 s, the speed of the body will
be 15 ms–1
7. A train starts from rest with a constant acceleration
a = 2 ms–2. After 5 seconds, a stone is dropped
from the window of the train. If g = 10 ms–2.
(a) the magnitude of the valocity of the stone
0.2 second after it is dropped is 2 ms–1.
(b) the angle between the resultant velocity Fig. 5.90
vector of the stone and the horizontal 0.2 11. Two blocks of masses m1 and m2 are connected
second after it is dropped is = tan–1 (0.2). by a string of negligible mass which passes over a
(c) the acceleration of the stone after it is dropped
is a = 2 ms–2. plane as shown in Fig. 5.91.
(d) the acceleration of the stone after it is dropped m1 and
is g = 10 ms–2. the plane is .
8. A man of mass m
(a) If m1 = m2, the mass m1
His weight when the lift is
up the inclined plane when the angle of in-
(a) stationary is mg clination is , then = tan .
(b) moving up with a uniform speed of 2 ms–1 (b) If m1 = m2, the mass m1
is 5 mg. up the inclined plane when the angle of in-
(c) moving up with a uniform acceleraiton a clination is , then = sec – tan .
(< g) is m (g + a). (c) If m1 = 2 m2, the mass m1
(d) moving down with a uniform acceleration a down the plane if = 2 tan .
(< g) is m (g – a). (d) If m1 = 2 m2, the mass m1
9. Two identical blocks, each of mass m, connected 1
down the plane if = tan – sec .
by a light string, are placed on a rough horizontal 2
surface. When a force F is applied on a block in
the horizontal direction, each block moves with an
acceleration a. Assuming that the frictional forces
m1
on the two blocks are equal,
(a) the tension in the string will be F.
m2
(b) the tension in the string will be F/2.
(c) the frictional force on each block will be
(F – ma). q

(d) the frictional force on each block will be Fig. 5.91


F
ma . 12. Two blocks A and B are connected to each other
2
by a string and a spring of force constant k, as
10. A body of weight W is suspended from a rigid sup- shown in Fig. 5.92. The string passes over a fric-
port P by means of a massless string as shown in tionless pulley as shown. The block B slides over
Fig. 5.90. A horizontal force F is applied at point the horizontal top surface of a stationary block C
O of the rope. The system is in equilibrium when and the block A slides along the vertical side of C
the string makes an angle with the vertical. If the
tension in the string is T, of friction between the surfaces of the blocks is .
(a) F = T sin (b) W = T sin If the mass of block A is m,
5.44 Comprehensive Physics—JEE Advanced

(a) the mass of block B is m. (b) the tension in the rope at t = 6 s will be
(b) the mass of block B is m/ . 10,000 N.
(c) the energy stored in the spring is m2g2/2k. (c) the tension in the rope at t = 11 s will be
(d) the energy stored in the spring is m2g2/k. 8,000 N.
B (d) the height upto which the lift takes the
T
passengers is 40 m.
15. Two blocks of masses m1 = 2 m
T and m2 = m are connected by a
light string passing over a fric-
k
tion less pulley as shown in Fig.
C
5.95. When they are released,
g
A (a) the acceleration of m1 is
3
vertically downwards.
Fig. 5.92 (b) the acceleration of m2 is
2g
13. Two blocks A and B of equal masses m each are vertically upwards.
3
connected to each other by a string passing over
a frictionless pulley as shown in Fig. 5.93. The (c) the tension in the string is
A and the 4g Fig. 5.95
.
surface below is 0.5. When the system is released, 3
(a) the acceleration of the blocks is 3g/4. (d) the distance through which m2 rises in t sec-
(b) the acceleration of the blocks is g/4. gt 2
(c) the tension in the string is 3mg/4. onds after the blocks are released is .
6
(d) the tension in the string is mg/4.
16. Two blocks of masses m1 and m2 connected by a

frictionless surface as shown in Fig. 5.96. A force


F is applied to m2 in the horizontal directon as
shown.
F
(a) the acceleration of each block is .
(m1 m2 )
m1 F
(b) the tension in the string is .
Fig. 5.93 (m1 m2 )
14. A lift is going up. The variation of the speed of the lift (c) the tension in the string is F.
with time is shown in Fig. 5.94. The total mass of m2 F
the lift and passengers is 1000 kg. If g = 10 ms–2, (d) the force on mass m1 is .
(m1 m2 )

Fig. 5.96
17. Ten coins, each of mass m, are placed on top of each
other on a horizontal table.
(a) The force on the 7th coin (counted from the
bottom) due to all the coins above it is 3mg
vertically downwards.
Fig. 5.94
(b) The force on the 7th coin by the 8th coin
(a) the tension in the rope of the lift at t = 1 s (both counted from the bottom) is 3mg
will be 12,000 N. vertically downwards.
Laws of Motion and Friction 5.45

(c) The force on the 7th coin by the 8th coin l


(both counted from the bottom) is 2mg (c) If = 0.25, = 20%
vertically upwards. L
(d) The reaction force of the 6th coin from the l
(d) If = 0.25, = 25%
bottom on the 7th coin from the bottom is L
4mg vertically upwards. 21. A block starts sliding from the top of an inclined
18. Two blocks of masses m1 and m2 are placed in plane of inclination
contact on a horizontal frictionless surface as between the block and the plane varies as =
shown in Fig. 5.97. A force F is applied to mass kx where x is the distance moved down the plane
m1 as shown. and k is a positive constant.
(a) The block has a uniform acceleration along
(a) The acceleration of mass m2 is F/m2
the plane.
m2 F (b) The acceleration of the block increases for
m2 is .
(m1 m2 ) tan
x< .
m2 is F. k
F
(d) The acceleration of mass m1 is . tan
(m1 m2 ) x= .
k tan
(d) The block starts decelerating for x > .
k
22. A block of mass m is lying at x = 0 on a smooth
Fig. 5.97
horizontal surface. A variable force F = kx is
applied to it as shown in Fig. 5.99 where k is a
19. Two blocks of masses m1 and m2 (m2 < m1) are constant. Then
placed on an inclined plane of inclination and (a) the block will move on the surface with a
joined by a string as shown in Fig. 5.98. uniform acceleration.
(b) the block will move on the surface with a
variable acceleration.
(c) the block will lose contact with the surface
mg
after travelling a distance x0 = .
k sin
(d) the block will always remain in contact with
the surface.
Fig. 5.98

and the plane is and the blocks are released,


(a) the acceleration of the blocks is g(sin –
cos ).
(b) the acceleration of the blocks is zero. Fig. 5.99

(c) the tension in the string is zero. 23. A pendulum of length L and bob mass M is oscillat-
(d) the tension in the string is (m1 + m2)g ing in a plane about a vertical line between angular
(sin – cos ). limits – and + . For an angular displacement | |
20. A uniform chain of length L is placed on a rough < , the tension in the string is T and the velocity
- of the bob is V. Then
tween the chain and the table is (a) T cos = Mg
length of the chain that can hang from the edge MV 2
(b) T – Mg cos =
of the table is l. Then L
L (c) The magnitude of the tangential acceleration
(a) l =
(1 ) of the bob is |at| = g sin
L (d) T = Mg cos
(b) l =
(1 ) IIT, 1986
5.46 Comprehensive Physics—JEE Advanced

24. When a bicycle is in motion, the force of friction (c) in the backward direction on both the front
and the rear wheels.
that it acts (d) in the forward direction on both the front and
(a) in the backward direction on the front wheel and the rear wheels.
in the forward direction on the rear wheel. IIT, 2007
(b) in the forward direction on the front wheel and
in the backward direction on the rear wheel.

ANSWERS AND SOLUTIONS


1. Choice (a) is true. Since the direction of motion given by v2 – u2 = 2as or (20)2 – 0 = 2 5 s or
as well as the speed of the drop of rain are con- s = 40 m. Further, since the frictional retardation is
stant, its velocity is constant and hence no net force 5 ms–2, the time needed to come to rest from a speed
acts on the drop. The downward gravitational force of 50 ms–1 is t = v/a = 50/5 = 10 s. Hence, choice
(i.e. weight) of the drop is completely cancelled (a), (b) and (d) are correct.
by the upward buoyant force on it due to air and 4. The rate of change of momentum of water stream
the viscous force due to its motion. Choice (b) is = av2 – 0 = av2 . This is the force F
also true. The weight of the cork is balanced by the water stream on the wall.
upward buoyant force due to water. Choice (c) is F = av2 = (1 10–2) (15)2 1000 = 2250 N
also true. The force due to the car’s acceleration
is balanced by the backward force of friction bet- The pressure on the wall is
ween its tyres and the rough road. Choice (d) is F 2250
P= = 2.25 105 pascal
false. Although the speed of the body is constant, a 1 10 2
its velocity is changing all the time because the Hence, the correct choices are (b), (c) and (d).
direction of motion keeps changing. Hence, the
motion of the body is being accelerated. A force 5. Since the acceleration along the inclined plane
(called centripetal force) must act on the body to (g sin ) is less than g, the blocks take different
produce the acceleration. times to reach the bottom. The speed of each block
Thus, choices (a), (b) and (c) are correct. on reaching the bottom is v = 2g h , where h is
2. In choice (a), a force F = mg = 1 9.8 = 9.8 N acts the height of the inclined plane. Thus choice (b) is
on the stone vertically downwards. In choice (b), correct. The directions of the velocity are differ-
the velocity of the train is constant. Hence, there ent, hence their momenta are not the same. Their
is no acceleration (and therefore, force) along the 1
kinetic energy mv2 is the same. Hence the cor-
direction of motion. When the stone is dropped, 2
the only force acting on it is F = mg = 9.8 N verti- rect choices are (b) and (d).
cally downwards. In choice (c), before the stone 6. Given u = + 5 ms–1 along positive x-direction
is dropped, a force = ma = 1 kg 1 ms–2 = 1 N F = – 0.4 N along negative x-direction
acts on it in the horizontal direction. But, after it is
m = 200 g = 0.2 kg
dropped, this force ceases to act because the stone
is no longer located in the accelerating system (i.e. F 0.4
The acceleration a = = = –2 ms–2. The
train). Hence, in this case also, the net force on the m 0.2
stone is 9.8 N vertically downwards. In choice (d), negative sign shows that the motion is retarded.
the weight of the stone is balanced by the normal The position of the body at time t is given by
1
x = x0 + ut + at2
accelerated in the forward direction along with the 2
train. Hence, the force acting on the stone = ma = 1 At t = 0, the body is at x = 0. Therefore, x0 = 0.
kg 1 ms–2 = 1 N in the direction of motion of the Hence
train. Thus, the correct choices are (a), (b) and (c). 1
3. - x = ut + at2
2
ping is given by Since the force acts during the time interval from
ma = mg or a = g = 0.5 10 = 5 ms–2 t = 0 to t = 10 s, the motion is accelerated only
- between t = 0 and t = 10 s. The position of the
tance s covered to acquire a speed v = 20 ms–1 is body t = 2.5 s is given by
Laws of Motion and Friction 5.47

1 After the stone is dropped, the horizontal velocity


x=5 2.5 + (– 2) (2.5)2 = 1.25 m vx remains unchanged because the acceleration
2
The velocity of the body at t = 2.5 s is is zero along the horizontal direction. The only
v = u + at = 5 + (– 2) (2.5) = 5 – 5 = 0 acceleration of the stone is the acceleration due
to gravity.
t = 0 to
Hence the correct choices are (b) and (d).
t = 10 s) the motion is accelerated. During this
8. mg) on the
time a = – 2 ms–2. Putting u = 5 ms–1, a = – 2 ms–2
1 on him an upward reaction for (R) which it mea-
and t = 10 s in equation x = ut + at2. We have
2 sures. When the lift is stationary, R = mg.
1
x1 = 5 10 + (– 2) (10)2 = – 50 m (i) When the lift is moving up or down with a
2 uniform speed, it has no acceleration of its own.
The velocity of the body at t = 10 s is Hence, the reading of the machine will still be mg.
v = u + at = 5 + (– 2) 10 = – 15 ms–1 When the machine is moving downwards with an
During the remaining 20 seconds, i.e. from acceleration a, a force F = ma acts downwards.
t = 10 s to t = 30 s, the acceleration a = 0, because But the reaction R = mg acts upwards. Hence,
the force ceases to act after t = 10 s. The velocity the effecting reading will be
of the body remains constant at – 15 ms–1 during Reff = F – R = mg – ma = m(g – a)
the last 20 seconds. The distance covered by the
In case (c), R and mg both act in the same direc-
body during the last 20 seconds is
tion (upwards). Hence, the effective reading will
x2 = – 15 20 = – 300 m be
Position of the body at t = 30 s is Reff = mg + ma = m(g + a)
x = x1 + x2 = – 50 – 300 Hence the correct choices are (a), (c) and (d).
= – 350 m 9. Refer to Sec. 5.11 and Fig. 5.15 on page 5.9
The magnitude of the velocity (i.e. speed) of the Equations of motion of m1 and m2 are
body at t = 30 s is 15 ms–1. m 1a = T – f (i)
Hence the correct choices are (a), (b) and (c).
and m 2a = F – T – f (ii)
7. Given u = 0, a = 2 ms–2. Since the stone is located
in the train, the acceleration of the stone is a = Subtracting the two equations, we have
2 ms–2. At time t = 5 s, the velocity of the stone (m1 – m2)a = 2T – F
is v = u + at = 0 + 2 5 = 10 ms–1. Before the Since m1 = m2, we get, we get 0 = 2T – F
stone is dropped, its motion is accelerated with
F
the train. But, the moment it is dropped, its ac- or T=
celeration due to the motion of the train ceases. 2
Therefore, after the stone is dropped, it has the Putting T = 10 N in Eq. (i) above, we have
following two motions: f = T – m 1a
(a) a uniform motion with velocity 10 ms–1 paral- F
= – ma
lel to the ground, i.e. 2
vx = 10 ms–1 (the horizontal velocity) Hence the correct choices are (b) and (d).
(b) an accelerated motion vertically downwards 10. The mass is in equilibrium at point O under the
due to gravity. In time t = 0.2 s, the vertical action of the concurrent forces F, T and W = mg.
velocity of the stone is vy = 0 + gt = 10 0.2 Therefore, as shown in Fig. 5.100, the horizontal
= 2 ms–2. component T sin of tension T must balance with
The resultant velocity of stone at t = 0.2 s is force F and the vertical component T cos must
balance with weight W = mg.
v= v 2x v 2y = (10) 2 ( 2) 2 = 104
Thus F = T sin (i)
= 2 26 ms–1
and W = T cos (ii)
The angle, which the resultant velocity vector,
Squaring Eqs. (i) and (ii) and adding we get
makes with the horizontal is given by
T 2 = W 2 + F 2.
vy 2
tan = = = 0.2 Dividing (i) by (ii) we get F = W tan .
vx 10 Hence the correct choices are (a), (c) and (d).
5.48 Comprehensive Physics—JEE Advanced

and T – mg = ma (ii)
Adding (i) and (ii) we get
1 1 0 .5 g
a= g= g=
2 2 4
Using a = g/4 in Eq. (i) gives T = 3mg/4
Hence the correct choices are (b) and (c).
14. Between t = 0 and t = 2 s, the acceleration of the lift
is
1
AD 4 ms
Fig. 5.100 a= = 2 ms–2
OD 2s
11. The block m1 will just begin to move up the Since the lift is accelerating upward, the tension in
plane if the downward force m2g due to mass m2 the rope at t = 1 s (between t = 0 and t = 2 s) is
trying to pull the mass m1 up the plane just equals T = m (g + a) = 1000 (10 + 2) = 12,000 N
the force (m1g sin + m1g cos ) trying to push
the mass m1 down the plane, i.e. when Between t = 2 s and t = 10 s, the speed of the lift is
constant. Hence a = 0 and T = mg = 1000 10
m2g = m1g(sin + cos ) = 10,000 N
Now, it is given that m1 = m2 = m. Between t = 10 s and t = 12 s, the lift is decelerating.
Therefore, we have Its deceleration is given by
1 = sin + cos
BE 4 ms 1
= sec – tan a= = 2 ms–2
EC 2s
The block m1 will just begin to move down the Tension = m (g – a) = 1000 (10 – 2)
plane if the downward force (m1g sin – m1g = 8,000 N
cos ) on m1 just equals the upwards force m2g The height to which the lift rises = area of OABC
acting on m1 due to m2, i.e. if = 40 m. Hence all the four choices are correct.
m2g = m1g(sin – cos ) 15. The correct choices are (a), (c) and (d). The accel-
m1 1 eration of each mass is the same and is given by
or =
m2 sin cos (m1 m2 ) g
a=
If m1 = 2m2, then we have (m1 m2 )
1 2m1 m2 g
2 = The tension in the string is T =
sin cos (m1 m2 )
1 Since the masses start from rest, the distance moved
= tan –sec by m1 in time t is
2
Hence the correct choices are (b) and (d). 1 2 1 2 1 2
s = ut + at = 0 + at = at
12. Since the blocks slide at the same uniform speed, 2 2 2
no net force acts on them. If M is the mass of 16. The correct choices are (a), (b) and (d). The accel-
block B, then the tension in the string is T = Mg. eration of each mass is given by
Also T = mg. Equating the two, we get M = m
m F
or M = a=
(m1 m2 )
F mg
x= = . Therefore, The tension in the string is T = m1 a. The force on
k k mass m1 is
potential energy stored in the spring is
m2 F
2 2 2 F1 =
1 1 mg m g (m1 m2 )
PE = kx2 = k =
2 2 k 2k 17. The 7 coin from the bottom has 3 coins above it.
th

Hence the correct choices are (b) and (c). Hence, the force on the 7th coin = weight of 3 coins
13. If the acceleration of the blocks is a, then we = 3 mg, vertically downwards.
have Since the 8th coin has 2 coins above, it supports
mg – T = ma (i) the weight of two coins. Hence the force on the 7th
Laws of Motion and Friction 5.49

coin by the 8th coin = weight of 8th coin + weight Solving Eqs. (i) and (ii), we get T = 0 and
of two coins above it = weight of three coins = a = g(sin – cos )
3 mg vertically downwards.
Hence the correct choices are (a) and (c).
From Newton’s third law, the reaction force
th 20. Let M be the mass of the chain and L its length.
coin on the 7th coin is equal and
th If a length l hangs over the edge of the table, the
coin
th th Ml
on the 6 force pulling the chain down is g. The force
th th
coin on the 6 coin = weight of 7 coin + weight L
of 3 coins above it = weight of 4 coins = 4 mg of friction between the rest of the chain of length
vertically downwards. Hence, the reaction of the 6th M (L l)
(L – l) and the table is g.
coin on the 7th coin = 4 mg vertically upwards. L
For equilibrium, the two forces must be equal,
Hence the correct choices are (a), (b) and (d).
i.e.
18. The correct choices are (b) and (d). The accelera- Ml M (L l)
tion of both the masses is the same and is given g= g
L L
by
net force F or l= (L – l)
a= =
total mass (m1 m2 ) L
or l=
Force on mass m2 = m2 a. 1
19. Figures 5.101 (a) and 5.101 (b) show the free
body diagram of the two blocks. l 0.25 1
= = = or 20%.
L 1 1 0.25 5
Hence the correct choices are (a) and (c).
21. The acceleration of the block is given by
a = g(sin – cos )
= g(sin – kx cos )
Acceleration a is not uniform; it varies with x. For
sin > kx cos , a is positive, i.e. a is positive
for x < tan /k. For x > tan /k, a is negative.
For x = tan /k, a
after which it begins to decrease as the block is
decelerated. Hence the correct choices are (b), (c)
and (d).
22. The horizontal and vertical components of F are
F cos and F sin (see Fig. 5.102).

Fig. 5.101

T is the tension in the string and f1 and f2 are the


frictional forces. It follows from the diagrams that
N1 = m1g cos and f1 = m1g cos
N2 = m2g cos and f2 = m2g cos
If a is the acceleration of the blocks down the
plane, the equations of motion are Fig. 5.102
m1a = m1g sin – T – f1 N + F sin = mg
= m1g sin – T – m1g cos (i) N = mg – F sin
and m2a = m2g sin + T – f2 = mg – kx sin
= m2g sin + T – m2g cos (ii) where N is the normal reaction.
5.50 Comprehensive Physics—JEE Advanced

The block will lose contact with surface at x = x0 It follows from Fig. 5.103, that
for which N = 0. Putting N = 0 and x = x0, we MV 2
mg = T – Mg cos
have 0 = mg – kx0 sin x0 = L
k sin
The acceleration a of block along the horizontal Tangential force ft = Mg sin
surface is given by Tangential acceleration at = g sin .
So the correct choices are (b) and (c).
ma = F cos = kx cos 24.
k x cos the force of friction acts in the opposite direction.
a=
m But when a body itself applies a force in order to
which depends upon x. Hence the correct choices move, the force of friction acts in the direction of
are (b) and (c). -
23. plied to the rear wheel and as a result the front wheel
moves by itself. So, while pedalling, the choice (a)
is correct. When pedalling is stopped, the choice (c)
is correct as long as bicycle remains in motion.

Fig. 5.103

III

Multiple Choice Questions Based On Passage


Question 1 to 3 are based on the following passage
Passage I of reference; it holds only for inertial frames of reference.
An inertial reference frame is a frame which moves with
Reference Frames
a constant velocity.
1. A reference frame attached to the earth
reference. Rest and motion are relative; there is nothing
like absolute rest or absolute motion. The position or state (b) cannot be an inertial frame because the earth
of motion of a body may appear different from different is revolving round the sun
(c) is an inertial frame because Newton’s laws
everything else in a moving train are at rest in a reference of motion are applicable in this frame
frame situated in the train but they are in motion in a (d) cannot be an inertial frame because the earth
reference frame situated on the platform. Similarly, a stone
dropped by a passenger from the window of a railway 2. Which of the following observers are inertial?
carriage in uniform motion appears to him to fall vertically (a) A child revolving in a merry-go-round
downwards but to a person outside the carriage, it appears (b) A driver in a car moving with a constant
to follow a parabolic path. velocity
Laws of Motion and Friction 5.51

(c) A pilot in an aircraft which is taking off (c) A reference frame moving at a constant velo-
(d) A passenger in a train which is slowing down city with respect to an inertial frame is also
to a stop. an inertial frame.
3. Choose the correct statements from the following. (d) Newton’s laws of motion hold for both iner-
(a) An inertial frame is non-accelerating. tial and non-inertia frames.
(b) An inertial frame is non-rotating.

SOLUTION
1. The velocity of the earth changes with time (due 2. The observers in (a), (c) amd (d) are all accelerat-
to a change in its direction) as it revolves round ing. Hence, they are non-inertial. Only the driver in
the sun. Therefore, a frame attached to the earth is (b) is inertial since his motion is not accelerated.
accelerated. Accelerated frames and rotating frames 3. The correct choices are (a), (b) and (c).
of reference are not inertial frames. Hence the cor-
rect choices are (b) and (d).

Question 4 to 6 are based on the following passage 5. If t1 and t2 are the respective times taken by them
Passage II to stop, then t1/t2 is
Two bodies A and B of masses m and 2 m respectively are (a) 1 (b) 2
moving with equal linear momenta. They are subjected to 1 1
(c) (d)
the same retarding force. 2 4
4. If x1 and x2 are the respective distances moved 6. If a1 and a2 are their respective decelerations,
by them before stopping, then x1/x2 is then a1/a2 is
1 (a) 1 (b) 2
(a) (b) 1
2 1
(c) 2 (d)
(c) 2 (d) 2 2
SOLUTION
(mu ) 2 p 2 1
4. 2 ax = u2 2 max = mu2 2 Fx = = Since p and F are constants, x . Hence the
m m correct choice is (d). m
where F = ma is the retarding force p = mu is u mu p
5. 0 = u + at t=– =– =– . Hence
linear momentum. Thus a ma F
t1 = t2. Thus the correct choice is (a).
p2
x= 6. a1 = F/m and a2 = F/2m. Hence the correct choice
2 Fm
is (b).

Questions 7 to 9 are based on the following passage 8. If t1 and t2 are the respective times taken by them
Passage III to stop, then t1/t2 is
Two bodies of masses m and 2 m respectively are moving 1 1
(a) (b)
with equal kinetic energies. They are subjected to the 2 2
same retarding force.
7. If x1 and x2 are the respective distances moved (c) 2 (d) 2
by them before stopping, then x1/x2 is 9. If a1 and a2 are their respective decelerations,
then a1/a2 is
(a) 2 (b) 2 (a) 4 (b) 2
1 1
(c) (d) 1 (c) (d) 1
2 2

SOLUTION
1 Since K and F are constants, the correct choice
7. 2ax = u2 2 max = mu2 Fx = mu2. If K
2 is (d).
is the kinetic energy, then Fx = K x = K/F.
5.52 Comprehensive Physics—JEE Advanced

u mu p 2mK
8. 0 = u + at t=– =– =– . Now Hence t = – . Thus t m . Hence the
a ma F F
1 2 correct choice is (a).
p = mu p2 = m2u2 = (2m) mu = 2mK.
2 F 1
9. a = , i.e. a Hence the correct choice is (b).
m m

Questions 10 to 13 are based on the following passage 12. I imparted to the block
Passage IV is given by
A block of masses m is initially at rest on a frictionless a3 a3
horizontal surface. A time-dependent force F = at – bt2 (a) (b)
3b 2 6b 2
acts on the body, where a and b are positive constants.
10. a3 a3
t1 given by (c) (d)
9b 2 12b 2
a 2a
(a) (b) 13. v attained by the block is
b b
a a a3 a3
(c) (d) (a) (b)
2b 2b 4mb 2 8mb 2
11. F is given by
a3 a3
a2 a2 (c) (d)
(a) (b) 12mb 2 16mb 2
2b 4b
2a 2 4a 2
(c) (d)
b b
SOLUTION
12.
dF d 2F
10. = 0 and t1

< 0. dt dt 2 I = Fdt
dF d 0
Now = (at – bt2) = a – 2bt t1
dt dt
dF = (at bt 2 )dt
Putting = 0 and t = t1, we get 0
dt
a at12 bt 3
0 = a – 2 bt1 t1 = = – 1
2b 2 3
d 2F d
Also = (a – 2 bt) = – 2 b, which is a a 2 b a 3 a3
dt 2
dt = – =
2 2b 3 2b 12b 2
negative.
Hence the correct choice is (d).
Hence the correct choice is (c). 13. Now impulse = change in momentum
a a 2
a2 = mv – 0 = mv
11. F = at1 – bt21 = a –b = . I a3
2b 2b 4b v = ma = , which is choice (c).
Hence the correct choice is (b). m 12mb 2

Questions 14 to 17 are based on the following passage


a b
Passage V (c) 2 (d) 2
b a
A body of mass m is initially at rest. A periodic force F = 15.
a cos(bt + c) is applied to it, where a, b and c are constants. a b
(a) (b)
14. The time period T of the force is mb mc
1 2 c b c
(a) (b) (c) (d)
b b ma ma
Laws of Motion and Friction 5.53

16. The smallest value of t after t = 0 when the 17. The distance travelled by the body from time t =
velocity of the body becomes zero is given by 0 to t = t1 is given by
a a 2a
(a) t1 = (b) t1 = (a) cos c (b) sin c
2
a c mb mb 2
c
(c) t1 = (d) t1 = a2 2a 2
b b (c) cos c (d) sin c
mb mb

SOLUTION
14. F will repeat itself at values of t given by 16. From Eq (i) it follows that v = 0 at values of t
cos(bt + c) = + 1, i.e. given by sin(bt + c) = 0 or (bt + c) = 0, , 2 , ...
bt + c = 0, 2 , 4 , c c 2 c
or t = – , , . Therefore,
c 2 c 4 c b b b
t=– , , ,
b b b c c
2 t1 = – = , which is choice (d).
The smallest time interval is T = . Hence the b b b
correct choice is (b). b
dp dx
15. From Newton’s second law of motion, F = 17. Now v = dx = v dt. Therefore, the distance
dt dt
dv moved between t = 0 and t = t1 is
=m
dt t1 t
dv a 1
Thus m = a cos(bt + c) x= vdt = sin (bt + c)dt
dt 0
mb 0

a a
dv = cos(bt + c)dt =– cos(bt1 + c)
m mb 2
t
a a a
v= cos (bt + c)dt = sin(bt + c) (i) =– cos b c
m0 mb mb 2 b
bt + c) = 1, a a cos c
a =– cos( + c) =
v = . mb 2
mb 2
mb
Hence the correct choice is (a). Hence the correct choice is (a).

Questions 18 to 20 are based on the following passage (c) a1 > a2 = a3 (d) a1 = a2 = a3


Passage VI 19. The tension in the string between masses m2 and
Three masses m1 = m, m2 = 2 m and m3 is
m3 = 3 m are hung on a string passing (a) mg (b) 3 mg
over a frictionless pulley as shown in 5mg
Fig. 5.104. The mass of the string is (c) 4 mg (d)
3
negligible. The system is then released.
18. If a1, a2 and a3 are the ac- 20. The tension in the string between masses m1 and
celerations of masses m1, m2 m2 is
and m3 respectively, then 2mg
(a) 4 mg (b)
(a) a1 < a2 < a3 3
5mg
(b) a1 > a2 > a3 (c) (d) 2 mg
Fig. 5.104 3
SOLUTION
18. When the masses are released, mass m1 moves (m2 m3 m1 ) g (2m 3m m) g 2g
a= = =
upward and masses m2 and m3 move downward (m1 m2 m3 ) (m 2m 3m) 3
with a common acceleration given by The correct choice is (d).
5.54 Comprehensive Physics—JEE Advanced

19. The let T be the tension in the string between m1 m 3 g – T = m 3a


and m2 and T be the tension in the string between
2g
m2 and m3 [see Fig. 5.105 (a)]. Figure 5.105 (b) T = m3(g – a) = 3 m g = mg
shows the free-body diagram of mass m3. 3
Hence the correct choice is (a).
20. Figure 5.105(c) shows the free-body diagram of
mass m1.
T – m 1g = m 1a
2g 5mg
T = m1(g + a) = m g =
3 3
Hence the correct choice is (c).

Fig. 5.105

Questions 21 to 23 are based on the following passage F F


(c) (d)
Passage VII 5m 6m
Three blocks of masses m1 = m, m2 = 2m and m3 = 3m 22. The force on mass m2 is
connected by two strings are placed on a horizontal 5F
frictionless surface as shown in Fig. 5.106. A horizontal (a) (b) 2 F
6
force F is applied to mass m1 as shown.
2F
(c) (d) F
3
23. The force on mass m3 is
F
Fig. 5.106 (a) F (b)
2
21. The common acceleration of the blocks is F F
F F (c) (d)
(a) (b) 3 6
m 3m

SOLUTION
21. If F2 and F3 are the forces on masses m2 and m3 Adding Eqs. (1), (2) and (3) we get
respectively, then the free-body diagrams of m1,
F F F
m2 and m3 are as shown in Fig. 5.107 where a a= = =
is the common acceleration of the system. (m1 m2 m3 ) (m 2m 3m) 6m
Hence the correct choice is (d).
22. Adding Eqs. (2) and (3) we have
F 5F
F2 = (m2 + m3)a = (2m + 3m) = ,
6m 6
Fig. 5.107 which is choice (a).
23. From Eq. (3), we have
F – F 2 = m 1a (1) F F
F3 = 3ma = 3m =
F2 – F 3 = m 2a (2) 6m 2
F 3 = m 3a (3) Hence the correct choice is (b).
Laws of Motion and Friction 5.55

Questions 24 to 26 are based on the following passage g


(c) (d) g
Passage VIII 2
Two blocks of masses m1 = m and m2 = 2 m are connected
25. If the system is released the tension in the string is
by a light string passing over a frictionless pulley. The
3mg
mass m1 is placed on a smooth inclined plane of inclination (a) mg (b)
= 30° and mass m2 hangs vertically as shown in Fig. 5.108. 2
2mg
(c) 2mg (d)
3
26. If the inclined plane was rough, it was found that
when the system was released, block m1 remained
at rest. The frictional force between block m1 and
the inclined plane is
3mg
(a) (b) 3 mg
2
Fig. 5.108
4mg 2mg
24. If the system is released, the blocks move with (c) (d)
3 2
an acceleration equal to
g g
(a) (b)
4 3
SOLUTION
24. Since the inclined plane is smooth and m2 > m1, (m2 m1 sin ) g (2m m sin 30 ) g g
block m1 will up the plane and block m2 will a= = =
m1 m2 m 2m 2
move vertically with a common acceleration a.
If T is the tension in the string, the free-body Hence the correct choice is (c).
diagrams of masses m1 and m2 are as shown in 25. From Eqs. (1) and (2), we get
Fig. 5.109
g
T = m2(g – a) = 2m g = mg
2
Hence the correct choice is (a).
26. Since the blocks remain at rest, the equations of
motions of blocks m1 and m2 are (here f is the
frictional force on m1)
T – m1g sin –f=0
and T = m 2g
Fig. 5.109
These equations give
f = m2g – m1g sin
The equations of motion of the blocks are
=2m g – m g sin 30°
T – m1g sin = m 1a (1)
mg 3mg
and m 2g – T = m 2a (2) = 2 mg – = ,
2 2
Equations (1) and (2) give which is choice (b).

Questions 27 to 29 are based on the following passage A and B. Wire A has negligible mass and wire B has a
Passage IX mass m3 = m, as shown in Fig. 5.110. The whole system
of blocks, wires and the support have an upward
Two blocks of masses m1 = 3 m and m2 = 2 m are suspended acceleration a.
5.56 Comprehensive Physics—JEE Advanced

27. The tension at the mid-point C of wire B is


1 3
(a) m(g + a) (b) m(g – a)
2 2
3 5
(c) m(g + a) (d) m(g + a)
2 2
28. The tension at point O of wire B is
(a) 3m(g + a) (b) 3m(g – a)
(c) 2m(g + a) (d) 2m(g – a)
29. The tension at the mid-point D of wire A is
(a) 2m(g + a) (b) 4m(g – a)
(c) 6m(g + a) (d) 8m(g – a)
Fig. 5.110

SOLUTION
27. Refer to Fig. 5.111. Let T 28. Let T1 be the tension in wire A. Since this wire
be the tension at the mid- has negligible mass, the tension is the same
point C of wire B. Then (= T1) at every point on this wire. Let T2 be the
m3 tension at point O of wire B. Then, we have for
T– m2 g wire A.
2
T 1 – T 2 – m 1g = m 1a (1)
m3
= m2 a where T2 is given by
2
T2 – (m2 + m3)g = (m2 + m3)a
m3
T = m2 (g + a)
2 T2 = (m2 + m3) (g + a)
m = (2m + m) (g + a) = 3m(g + a)
= 2m (g + a)
2 Hence the correct choice is (a).
Fig. 5.111
5 29. Putting T2 = 3m(g + a) in Eq. (1), we get
= m(g + a),
2 T1 = 6 m(g + a).
which is choice (d). Hence the correct choice is (c).

IV

Matching
1. Three blocks of masses m1 = 3 m, m2 = 2 m and m3 = m are placed in
contact on a horizontal frictionless surface as shown in Fig. 5.112. A
horizontal force F is applied to m1 as shown. Match items in column I
with those in column II
Column I Column II
Fig. 5.112
(a) Force acting on m2 if F = 12 N (p) 1 N
(b) Force acting on m2 if F = 6 N (q) 3 N
(c) Force acting on m3 if F = 12 N (r) 2 N
(d) Forcer acting on m3 if F = 6 N (s) 6 N
Laws of Motion and Friction 5.57

SOLUTION
The contact forces acting on m2 and m3 respectively are
(m2 m3 ) F
F2 =
(m1 m2 m3 )
m3 F
and F3 =
(m1 m2 m3 )
Hence the correct matching is as follows
(a) (s) (b) (q)
(c) (r) (d) (p)
2. Two blocks of masses M = 5 kg and m = 3 kg are placed on a horizontal surface as shown in Fig. 5.113.
1 = 0.5 and that between the blocks M and the horizontal
surface is 2 = 0.7. Taking g = 10 ms–2, match items in column I with those in column II
Column I Column II
(a) Frictional force between the blocks (p) 5 ms–2
(b) Acceleration of the upper block (q) 96 N
F
applied to M so that the two blocks
move together without slipping. (r) 4 ms–2
(d) The common acceleration of the
Fig. 5.113
blocks if F = 32 N. (s) 15 N

SOLUTION
(a) Force of frictional between blocks = 1mg = 0.5 3 10 = 15 N.
(b) Acceleration of the upper block of mass m is a = 1mg/m = 1g = 0.5 10 = 5 ms–2.
(c) The force of friction between block M (with block m placed on top of it) and the horizontal surface =
(M + m) 2g = (5 + 3) 0.7 10 = 56 N
a = 5 ms–2. The
force due to this acceleration = (M + m)a = (5 + 3) 5 = 40 N
F = 56 + 40 = 96 N
(d) If F = 32 N, the common acceleration of the blocks is
F 32
a = = = 4 ms–2
(M m) (5 3)
Hence the correct matching is as follows:
(a) (s) (b) (p)
(c) (q) (d) (r)

Assertion-Reason Type Questions


In the following questions, Statement-1 (Assertion) is (a) Statement-1 is true, Statement-2 is true and
followed by Statement-2 (Reason). Each question has four -
choices out of which only one choice is correct ment-1.
5.58 Comprehensive Physics—JEE Advanced

(b) Statement-1 is true, Statement-2 is true but The speed of the third (lighter) fragment will be 2
2 v.
Statement-1. Statement-2
(c) Statement-1 is true; Statement-2 is false. The momentum of a system of particles is conserved
(d) Statement-1 is false; Statement-2 is true.
1. Statement-1 5. Statement-1
A block is pulled along a horizontal frictionless F such that the block
surface by a thick rope. The tension in the rope will shown in Fig. 5.115 does not move is mg/cos ,
not always be the same at all points on it. where
Statement-2 block and the horizontal surface.
The tension in the rope depends on the acceleration
of the block-rope system and the mass of the rope.
2. Statement-1
A truck moving on a horizontal surface with a
uniform speed u is carrying sand. If a mass m
of the sand ‘leaks’ from the truck in a time t,
the force needed to keep the truck moving at its Fig. 5.115
uniform speed is u m/ t.
Statement-2 Statement-2
Force = rate of change of momentum. normal
reaction.
3. Statement-1
6. Statement-1
Two blocks of masses m and M are placed on a
horizontal surface as shown in Fig. 5.114. The A ball of mass m is moving towards a batsman at a
speed v
1
and that between the block M and the horizontal it by an angle without changing its speed. The
surface is 2 impulse imparted to the ball is zero.
applied to block M so that the two blocks move Statement-2
without slipping is F = ( 1 + 2) (M + m)g. Impulse = change in momentum
7. Statement-1
A cloth covers a table. Some dishes are kept on it.
The cloth can be pulled out without dislodging the
dishes from the table.
Statement-2
For every action there is an equal and opposite
reaction.
Fig. 5.114
IIT, 2007
Statement-2 8. Statement-1
When a ball dropped from a certain height hits the
acceleration.
4. Statement-1 momentum.
A shell of mass m Statement-2
three fragments having masses in the ratio 2:2:1.
reaction forces, being equal and opposite, cancel
mutually perpendicular directions with speed v. each other.

ANSWERS/SOLUTIONS
1. The correct choice is (a).
2. the vertically upward direction. This perpendicu-
the leaking sand on the truck = rate of change of lar force can do no work on the truck. Since the
momentum = u m/ t. The sand falling vertically truck moves with a uniform velocity, the force
Laws of Motion and Friction 5.59

3. The correct choice is (c). The force of friction From the principle of conservation of momentum,
between block m and block M = 1mg, where the momentum of the third (lighter) fragment of
1 m
blocks. Now, the force of friction between block mass must be 2 p but opposite in direction.
5
M (with block m on top of it) and the horizontal Thus, if V is the speed of the lighter fragment,
surface = 2(M + m)g, where 2 we have
of friction between block M and surface. The mV 2mv
F applied to block M must be = 2p = 2
enough to overcome this force of friction and 5 5
the force due to acceleration of the system. If or V = 2 2v
the acceleration of the system is a then this force 5. The correct choice is (a). The component of F
= (M + m)a. Thus parallel to the horizontal surface is F cos . F
F = (M + m)a + m2(M + m)g (i) F cos just overcomes
Now, since the force on block m is 1mg, its ac- the frictional force f = mg. Thus
celeration a is
mg
force on mass m mg F cos = mg F =
a= = 1 = 1g (ii) cos
mass m m
Using (ii) in (i) we get 6. The correct choice is (d). Refer to the solution
of Q.27 of section I.
F = 1(M + m)g + 2(M + m)g
7.
= ( 1 + 2) (M + m)g
motion also called the law of inertia. The dishes
4. The correct choice is (a). The mass of two frag- are not dislodged even when the cloth is suddenly
2m pulled because the dishes have the inertia of rest.
ments of equal masses = each. The mass
5 Statement-2 is Newton’s third law of motion, it
m
of the lighter fragment = . The momenta of choice is (b).
5
8. The assertion is true but the reason is not correct
2mv
heavier fragments are p = . The resultant of because action and reaction forces do not act on
momenta p and p is 5 the same body and hence do not cancel each
other. Hence the correct choice is (c).
p = (p2 + p2)1/2 = 2p

VI

Integer Answer Type


1. The magnitude of force f (in newton) acting on a 2. A block of mass 0.2 kg is held against a wall by
body varies with time t (in millisecond) as shown in applying a horizontal force of 5 N on the block.
Fig. 5.116. Find the magnitude of the total impulse
(in Ns) of the force on the body from t = 4 ms to is 0.5. Find the magnitude (in newton) of the fric-
t = 16 ms. tional force acting on the block. Take g = 10 ms–2.
IIT, 1994 IIT, 1995
3. Block A of mass m and block B of mass 2m are

massless string and a frictionless pulley as shown

block A and the wedge is 2/3 and that between


block B and the wedge is 1/3. If the blocks are
A
(in ms–2).
Fig. 5.116 IIT, 1997
5.60 Comprehensive Physics—JEE Advanced

table. The upper end of the string is now released.


At any time during the falling of the string, the
total force on the surface of the table is n times the
weight of the part of the string lying on the surface.
Find the value of n.
IIT, 1989
5. A uniform rope of mass M and length L is pulled
by a constant force of 10 N. Find the tension (in
newton) in the rope at a point at a distance L/5 from
Fig. 5.117 the end where the force is applied.
4. A piece of uniform string hangs vertically so that IIT, 1978
its free end just touches the horizontal surface of a

SOLUTIONS
1. Impulse from t = 4 ms to t = 16 ms = area under the
F – t graph = area of EBCD
= area of trapezium EBCF + area of CDF
1
= (200 + 800) N (2 10–3 s)
2
1
+ 800 N (10 10–3 s)
2
= 1 + 4 = 5 Ns
2. Normal reaction R = 5 N. Fig. 5.119
At equilibrium, the force of friction = weight of the
The equations of motion of blocks A and B are
block (see Fig. 5.118)
T – mg sin 45° – A mg cos 45° = ma, where
A = 2/3 and 2 g sin 45°– B 2 mg cos 45° – T
= 2 ma, where B = 1/3.
Adding these equations and solving we get
g
a= –
9 2
Case (b): If we assume that block A moves down
7g
and block B moves up, we would get a = – .
9 2
Fig. 5.118
Thus in both cases, the acceleration has a negative
= mg = 0.2 10 = 2 N value which implies that the blocks will deceler-
3. Case (a): Let us assume that block A moves up the ate. This is not possible because the blocks start
plane and block B moves down the plane. The free from rest. Hence when the blocks are released, they
body diagrams of the blocks are as follows (See move with zero acceleration. Thus acceleration of
Fig. 5.119) block A = 0.
4. Let x be the length of the string lying on the surface
of the table at an instant of time t. If an additional
length dx of the string falls on the surface in time
dt, the velocity v of this element when it strikes the
surface is given by ( u = 0)
v2 = u2 + 2gx = 0 + 2gx
or v2 = 2gx (1)
Laws of Motion and Friction 5.61

The total force on the surface is For part AP the tension is towards the positive
F = rate of change of momentum of element of x-direction and for part BP the tension is towards
length dx + weight of a length x of the string lying the negative x-direction. If a is the acceleration pro-
on the table. duced in the rope by the constant force F, then for
If m is the mass per unit length of the string, then part AP,
d dx T = (mass of AP) a or T = mxa (1)
F= (mdxv) + mxg = mv
dt dt For part BP, we have
+ mxg = mv2 + mxg (2)
F – T = (mass of BP) a = m(L – x)a (2)
dx T
v From (1), we have a =
dt mx
Using (1) in (2) we get Using this in (2), we get
F = 2 mgx + mgx = 3 mgx T ( L x)T
F – T = m(L – x)
But mx = M, the mass of the string lying on the mx x
table. Hence (L x) L Fx
F = 3 Mg or F= T 1 T or T
x x L
Thus n = 3.
L 4L
M At x= L – ,
5. Mass per unit length of the rope is m = . Let us 5 5
L
P at a distance x from the end F 4L 4F 4 10
T= 8N
x = 0. Let T be the tension in the rope at point P. L 5 5 5
A P B
x
x= x=L
x L–x

Fig. 5.120
6
Chapter
Work, Energy and Power

REVIEW OF BASIC CONCEPTS force F in moving the body from a position r1 to a position
r2 is given by
6.1 WORK r2
W= F dr = area under the (F – r) graph
1. Work done by a Force r1

(a) Work done by a constant force


6.1
When a constant force F acting on a body produces a
displacement S, then the work done by the force is given by makes an angle of 60° with the horizontal. Find the
W = F S = FS cos work done if a force of 150 N is applied to drag the
where is the angle between the force vector F and the box through a distance of 10 m.
displacement vector S [see Fig. 6.1]. F and S are the
magnitudes of F and S respectively. SOLUTION
W = FS cos
= 150 10 cos 60° = 750 J

6.2
A horizontal force F pulls a 20 kg box at a constant
Fig. 6.1

(i) If is acute, cos is positive. Hence work is posi- the work done by force F in moving the box through
tive for acute . In this case the force increases the a distance of 2 m.
speed of the body.
(ii) If = 90°, W = 0, i.e. if the force is perpendicular SOLUTION
to displacement work done by the force is zero. Since the box is moved at a constant velocity, the
(iii) If is obtuse, W is negative. In this case the force applied force F just overcomes the frictional force
decreases the speed of the body. f, i.e.
(iv) If = 0, i.e. force F is in the same direction as F = f = mg
displacement S, then W = FS
(v) If =180°, force F is opposite to S (example fric- Work done W = FS cos = mgS cos 0°
tional force), W = – FS. Work done by frictional and = 0.25 20 9.8 2 = 98 J
viscous force is always negative.
6.3
(b) Work done by a variable force
A block of mass m = 5 kg slides down from the top
Suppose a force F is not constant but depends on the of an inclined plane of inclination = 30° with the
position vector r of the body, then the work done by the
6.2 Comprehensive Physics—JEE Advanced

the block and the plane is 0.25. The length of the plane = 2 (2.2 + 9.8) = 24 N
is 2 m. Find the work done by the (a) gravitational Work done by tension is ( T
force, (b) frictional force and (c) normal reaction if and S are in the same direction)
the block slides to the the bottom of the plane.
W1 = TS cos 0°
SOLUTION = 24 4 1 = 96 J
Refer to Fig. 6.2. (b) Since the gravitational force mg
and displacement S are in op-
posite directions, work done by
gravity is
W2 = mgS cos 180° Fig. 6.3
= – 2 9.8 4 = – 78.4 J
(c) Net work done W = W1 + W2 = 96 – 78.4 = 17.6 J

6.5
A block of mass m = 2 kg in suspended by a light
Fig. 6.2
string from the ceiling of a lift. The lift starts mov-
Displacement S = AB = 2 m from A to B. ing down with an acceleration a = 1.8 ms –2. Find the
(a) Angle between F and S is (90° – ) = 90° – 30°
= 60° 5 seconds.
Work done by gravitational force is SOLUTION
W1 = FS cos 60° = mgS cos 60° Tension T = m(g – a) = 2 (9.8 – 1.8) = 16 N
1 Distance moved in t = 5 s is
= 5 9.8 2 = 49 J
2 1 1
S = at 2 = 1.8 5 2
(b) Work done by frictional force is 2 2
W2 = f S cos 180° = 22.5 m
= – f S = – RS Since the tension and displacement are in opposite
= – mg (cos )S directions, the work done by tension is
= – 0.25 5 9.8 cos 30° 2 W = TS cos180°
= – 21.2 J = – TS = – 16 22.5 = – 360 J
(c) Since the normal reaction R is perpendicular to
displacement S, work done by normal reaction 6.6
is
W3 = RS cos 90° = 0 A constant force F = (2 i 3 j) newton displaces

a body from position r1 = (4 i 5 j) metre to


6.4
A block of mass m = 2 kg is raised vertically upwards r2 = ( i 3 j) metre. Find the work done by the force.
by means of a massless string through a distance of
SOLUTION
S = 4 m with a constant acceleration a = 2.2 ms–2.
Find the work done by (a) tension and (b) gravity. Displacement S = r2 – r1

= (i 3 j) (4 i 5 j) 3i 8j
SOLUTION
W = F S = (2 i 3 j) ( 3 i 8 j)
(a) From the free body diagram (Fig. 6.3)
T – mg = ma = – 6 + 24 = 18 J

T = m(a + g) [ i i = j j 1 and i j = 0]
Work, Energy and Power 6.3

Displacement S = 2 m down the inclined plane.


6.7
(a) Work done by gravity = mg sin AB = (mg sin ) S
A body of mass m = 0.5kg travels in a straight line
with a velocity v = 5x3/2 where v is in ms–1 and x is = 5 9.8 sin 30° 2
in metre. Find the work done in displacing the body = 49 J
from x = 0 to x = 2 m. (b) Work done by friction = f S cos 180°
=–fS
SOLUTION
= – RS
dv d
Acceleration a= = 5 x3 / 2 = – mg cos S
dt dt
= – 0.3 5 9.8 cos 30° 2
3 1/ 2 dx
= 5 x = – 14.7 3 = – 25.5 J
2 dt
(c) Work done by normal reaction = RS cos 90° = 0
15 1/ 2 dx
= x 5 x3 / 2 v ( R S)
2 dt
75 2 6.2 ENERGY
= x
2
x 2 2
So,
Work done W = Fdx = madx
energy is measured in the same units as work, namely,
x 0 0
joule. Like work, energy is a scalar quantity.
2
75 Energy can exist in various forms, such as heat energy,
= 0.5 x 2 dx
2 electrical energy, sound energy, light energy, chemical
0
energy, nuclear energy, mechanical energy, etc. Mechanical
2
75 x3 energy is of two types, and
= 0.5
2 3 Kinetic Energy: Energy due to Motion A moving
0
object can do work on another object when it strikes it. In
0.5 75 other words, an object in motion has the ability to do work
= 8 0
2 3
= 50 J
An initially motionless body can move and acquire a
6.8 velocity only if a force acts on it. The work done by the
A block of mass 5 kg slides down from the top of force in causing the body to move measures the kinetic
an inclined plane of angle of inclination 30°. The energy (written as KE) of the moving body, i.e.
KE = W
the plane is 0.3. The length of the plane is 2 m. Find The kinetic energy of a body of mass m, moving with a
work done by (a) by gravity, (b) frictional force and velocity v is given by
(c) normal reaction. 1
KE = mv2
2
SOLUTION This relation holds even if the force is variable, i.e. if
the force varies both in magnitude and direction.
Work-Energy Principle Suppose a body of mass m
moves with an initial velocity . A force F acts on it, as
v. The work
done by the force is given by
W= Fdx ma dx
v
dv 1
=m dx = m(v2 – 2
)
dt 2
1 1
= mv2 – 2
Fig. 6.4 2 2
6.4 Comprehensive Physics—JEE Advanced

the reaction of the spring and is called the


Thus, which is proportional to the displacement x and acts in a
This is the direction opposite to the displacement, i.e.
work-energy principle. F – x or F = –
Thus, when a force does work on a body, its kinetic where is the force constant of the spring. The negative
energy increases; the increase in kinetic energy being equal sign indicates that the force acts in a direction opposite to
to the amount of work done. The converse of this is also displacement.
true. When the kinetic energy of a body is decreased by a To stretch a spring by a displacement x, we must exert
retarding force, the decrease is equal to the work done by a force F on it, equal but opposite to the force F exerted
the body against the retarding force. Thus kinetic energy by the spring on us. Therefore, the applied force is
and work are equivalent quantities and are, therefore, F =–F=
measured in the same units, namely, joule. Notice that F is a variable force as it depends on x.
Potential Energy: Energy due to Position or Therefore, the work done by the applied force in stretching
An object can have energy not only by the spring through a distance x is given by
virtue of its motion, but also because of its position or x x
W= F dx = ( )dx
0 0
An object held at a x
x
position above the surface of the earth has potential energy x2 1 2
= x dx =
by virtue of its position. 0
2 0
2
Consider a body of mass m. It is lifted vertically to a
It is evident that the work done in compressing the
height above the earth by applying a force F vertically
upward. The force F must be just enough to overcome 1 2
spring by an amount x is also given by W = .
the gravitational attraction, i.e. 2
F = mg
Law of Conservation of Energy
where g is the acceleration due to gravity at that place.
For bodies not too far above the surface of the earth, the The total energy of an isolated system remains constant,
value of g is practically constant. Hence the work done the energy can only change from one form to another.
by a constant force F in displacing a body by a height
can be calculated by the product F = . Thus 6.9
gravitational potential energy of a body of mass m at a A block of mass 0.5 kg is taken from the bottom of
height above the surface of the earth is . an inclined plane to its top and then allowed to slide
Gravitational PE = down to the bottom. The length of the inclined plane
Consider a perfectly
friction between the block and the plane is 0.2. Find
(a) work done by the gravitational force over the
a frictionless horizontal surface as shown in Fig. 6.5. We round trip,
assume that the mass of the spring is negligible compared (b) work done by the applied force over the upward
to the mass of the block. journey,
(c) work done by the frictional force over the round
trip and
(d) the kinetic energy of the block when it reaches
the bottom of the plane. What conclusion will
you draw from your answers to (b), (c) and (d)?
Take g = 10 ms–2.

SOLUTION
Fig. 6.5 Refer to Fig. 6.6.
If we stretch the spring by a distance x, the spring will m = 0.5 kg, = 0.2, (= AC) = 2.5 m and (= AB)
exert a force on us during stretching. This force is due to = 1.5 m
Work, Energy and Power 6.5

W4 = –
Total work done by frictional force over the
round trip is
Wf = W3 + W4 = – 2
= – 2 mg cos
= – 2 0.2 0.5 10 0.8 2.5
=– 4 J
Fig. 6.6 (d) When the block is at A, its initial velocity = 0.
Let v be the velocity when it reaches C. Since f
AB 1.5 acts upwards, the net force on the block when it
sin = = = 0.6
AC 2.5 slides down is
cos = 1 sin 2 = 0.8 F = mg sin – f = mg (sin – cos )
F
(a) Work done by gravitational force to take the Acceleration a = = g (sin – cos )
m
block from C to A is
= 10 (0.6 – 0.2 0.8)
W1 = (– mg sin )
= 4.4 ms –2
The negative sign indicates that the gravitational From v2 – 2
= 2 , we have
force mg sin is opposite to the displacement v2 – 0 = 2 4.4 2.5
which is from C to A. Work done by the gravi- 2 2 –2
tational force to move the block from A to C is v = 22 m s
(because now the gravitational force is in the 1
Kinetic energy at C = mv2
direction of the displacement) 2
W2 = (+ mg sin 1
= 0.5 22
Total work done by the gravitational force 2
over the round trip is = 5.5 J
WG = W1 + W2 = 0 Conclusion
This shows that gravitational force is conserva- Initial kinetic energy at A = 0. Therefore, change in
tive. K.E.= 5.5 – 0 = 5.5 J. Now total work done is
(b) Frictional force f = R = mg cos . The applied W = WG + Wa + Wf
force to move the block from C to A is = 0 + 9.5 – 4 = 5.5 J
F = mg sin + f = mg sin + mg cos Thus, work done = change in kinetic energy. This is
= mg (sin + cos ) the work-energy principle.

Work done by applied force to move the block 6.10


from C to A is A block of mass m = 500 g is placed at the top of
Wa = F AC an inclined plane of inclination = 60°. The length
= mg (sin + cos ) of the plane is 2 m. The block is released from rest.
= 0.5 10 (0.6 + 0.2 0.8) 2.5 Find its speed when it reaches the bottom of the
= 9.5 J plane if
(a) the inclined plane is smooth
(c) Friction always opposes motion. When the block
is moved from C to A, frictional force f is oppo-
the plane is 0.4.
site to direction. Hence work done by frictional
force in the upward journey is Take g = 10 ms–2.
W3 = –
SOLUTION
When the block slides from A to C, f acts up-
Refer to Fig. 6.6 of Example 6.9 above. Height of the
wards along the plane and is opposite to the dis-
inclined plane is = AB = AC sin = 2 sin 60° =
placement. Hence work done by frictional force
in the downward journey is 3 m = 1.73 m
6.6 Comprehensive Physics—JEE Advanced

(a) As the block slides down the plane, it loses po- the same plank before coming to rest if it were mov-
tential energy and gains kinetic energy. From the ing with a speed of 200 ms–1 ?
principle of conservation of energy,
Gain in K.E = loss in P.E. SOLUTION
1 Since the bullet and the plank are the same, the resis-
or mv2 = tive force F exerted on the bullet is the same in the
2
two cases. The kinetic energy is spent in doing work
v= 2 = 2 10 1.73 5.9 ms–1
against friction. Hence
(b) As the block slides down, loss of P.E. = gain in 1
K.E. + work done against friction, i.e., mv12 = Fx1
2
1
= m v2 mg cos AC 1
2 and mv22 = Fx2
2
1
1.73 10 = v 2 0.4 10 cos 60 2 These equations give
2
2
v 5.2 ms–1 v22 200
x2 = x1 = 2 cm
v12 100
6.11 = 8 cm
An elastic spring of negligible mass has a force con-
stant 4 Nm–1 6.13
the wall and the other end touches a block of mass A uniform chain of length L and mass M lies on a
m = 250 g placed on a horizontal surface. The spring frictionless horizontal table with a very small part
is compressed by an amount x = 5 cm as shown in hanging from the edge of the table. The chain begins
to fall under the weight of the hanging part. Obtain
and the horizontal surface is = 0.2. If the system is the expression for the velocity of the chain at the
instant when the length of the hanging part becomes
the spring. L/n where n > 1.

SOLUTION
M
Mass per unit length of chain = . The mass of
L L
length of the chain is
n
Fig. 6.7 M L M
m=
L n n
SOLUTION
As the chain slips down from the table, gravitational
Loss in P.E. of spring + work done against friction = L
gain in K.E. potential energy of the length (hanging part) de-
n
1 2 1
or mgx = mv 2 creases while the part of the chain left on the table
2 2 does not lose any gravitational potential energy. The
1 loss of P.E. of the hanging part gets converted into
4 0.05 2 0.2 0.250 10 0.05
2 K.E. of the entire chain, i.e.
1 Gain of K.E. of the complete chain = loss of P.E. of
= 0.250 v 2 the hanging part. The mass m of the hanging part can
2
be assumed to be concentrated at its centre of mass
which gives v = 0.5 ms–1
L
which is at a height = below the edge of the
6.12 2n
table. If v is the velocity of the slipping chain, then
A bullet moving with a speed of 100 ms–1 travels a
distance of 2 cm in a plank of wood before coming to 1 M L MgL
M v2 = = g
rest. How much distance will the same bullet travel in 2 n 2n 2n 2
Work, Energy and Power 6.7

gL NOTE
v2 =
n2 F is negative if r is opposite to F and positive if r is in
gL the same direction as F.
v=
n 6.14
The force between two point charges q1 and q2
CONSERVATIVE AND NON-CONSERVATIVE 1q2
6.3 FORCES separated by a distance r is F = where is a
r2
constant. Find the potential energy of the system of
(a) Conservative force
charges.
A force is conservative if
(i) the work done by it on a body in moving it from SOLUTION
one position to another depends only on the initial dU
F= dU = – Fdr. Integrating
dr
followed by it between the two positions.
r r
or
(ii) the net work done by the force on a body that moves U= Fdr = 1 q2 r 2 dr
0 0
through any closed path is zero.
The above two conditions are equivalent. Examples of 1 q2
U=
conservative forces are gravitational force, electrostatic r
force and spring force.
(b) Non-conservative force
6.15
A force is non-conservative if
The potential energy U
(i) The work done by it on a body in moving it from
with position r as
one position to another depends on the path fol-
lowed by the body between the two positions. a b
U= 2
or r r
(ii) The work done by the force on a body that moves where a and b are positive constants. Find the position
through a closed path is non zero. r0 where the particle will be in stable equilibrium.
Examples of non-conservative forces are frictional and
viscous forces. SOLUTION
If no force acts on a particle, it will be in stable equi-
Conservative Force and Potential Energy librium, i.e. F = 0 or
For a conservative force F that depends upon position r, dU
F= =0
there is a potential energy function U which also depends dr
on r. When a conservative force does positive work, the d a b
=0
potential of the system decreases, i.e. d r r2 r
Work done = decrease in potential energy 2a b 2a
3 2
=0 r = r0 =
or Fdr = – dU r r b
or F= dU Stable equilibrium corresponds to minimum poten-
dr dU d 2U 2a
tial energy, i.e. = 0 and > 0. If r = r0 = ,
Hence dr dr 2
b
dU
= 0, then U can be minimum or maximum. If
dr
The change in potential energy when the body is
d 2U
displaced from r = a to r = b is > 0 at r = r0, U will be minimum.
b d r2
Ub – Ua = Fdr dU 2a b
Now = 3
a dr r r2
6.8 Comprehensive Physics—JEE Advanced

and TA = tension in the string when the body is at A.


d 2U 6a 2b
=
d r2 r3r4 m v2A
TA = – mg (i)
2a r
At r = r0 = ,
b The body will revolve in the circle if the string does not
4 3 sag, i.e. TA 0. From Eq. (i) it follows that
d 2U b b
= 6a – 2b m v2A
d r2 at r r0
2a 2a – mg 0
r
3 b4 b4 b4
= , which is posi- v2A rg
8 a3 4a 3 3
8a tive.
Therefore, the minimum speed at the top that the body
2a must have so that it can complete the circle is given by
Hence r = r0 = corresponds to stable equilib-
b (vA)min = rg (ii)
rium.
Case (b): When the body is at bottom B of the circle
NOTE
When the body is at B, the net force on the body towards
For stable equilibrium U is minimum and for unstable the centre is (TB – mg). Hence
equilibrium, U is maximum.
dU d 2U m vB2
For stable equilibrium; = 0 and >0 TB – mg =
dr d r2 r
where vB = speed of the body at bottom B
dU d 2U
For unstable equilibrium; = 0 and < 0. and TB = tension in the string when the body is at B.
dr d r2
m vB2
TB = + mg (iii)
6.4 MOTION IN A VERTICAL CIRCLE r
The minimum speed at B that the body must have
Consider a body of mass tied to a string of length r revolved so that it can complete the circle is obtained from the
in a vertical circle with centre O at the other end of the conservation of energy. As the body goes up from B to A,
string as shown in Fig. 6.8. At all positions of the body, it K.E. decreases and P.E. increases.
there are two forces acting on it: its own weight and the Loss in K.E. = gain in P.E.
tension in the string.
1 2 1
v mvB – mv2A = mg AB = mg 2r
2 2
vB = v2A 4gr

(vB)min = ( vA )2min 4 gr (iv)


Using (ii) in (iv) we get
(vB)min = 5gr (v)
The net force towards the centre acting on the body is
v obtained from (iii) by using (v).
m
TB = 5gr + mg = 6 mg
r
Fig. 6.8
NOTE
Case (a): When the body is at top A of the circle 1. When a body moves in a vertical circle, the speed de-
When the body is at top A of the circle, the net force creases as it goes up and increases as it goes down.
towards centre O is TA + mg. Hence [see Fig. 6.8] Hence the body has a non-uniform circular motion.
2. The tension in the string is different positions of the
m vA2 body on the circle. The tension is minimum when the
TA + mg =
r body is at the top of the circle and maximum when it is
where vA = speed of the body at top A at the bottom of the circle.
Work, Energy and Power 6.9

6.16 1 2
min = mvmin min = 2.5r.
A small block of mass m, starts from rest at A and 2
slides on a frictionless track which ends in a circular
loop of radius r. If = 6r 6.5 POWER
when it reaches C as shown in Fig. 6.9. What is the
force exerted on the block by the track when it is at i.e.
C so that the block Work
Power =
is able to complete the circle. Time
The faster a given amount of work is done, the greater
is the power of the agent that does the work.
In the SI system, the unit of power is the watt (symbol
W).
i.e.
1 W = 1 Js–1
Since the watt is a small unit for the measurement of
power, larger units, namely kilowatt (kW) and megawatt
(MW) are often used.
Fig. 6.9
1 kW = 1000 W = 103 W
SOLUTION
Let v be the speed of the block when it reaches C. 1 MW = 1,000,000 W = 106 W
From conservation of energy, gain in K.E. = loss in The power of an agent can also be expressed in terms of
P.E., i.e. the force applied and the velocity of the object on which
1 the force is applied. Now, power is given by
mv2 0 = OB = – mgr W F S
2 = = Fv
1 2 t t
v = g 6r – gr = 5gr S
2 ( = rate of change of displacement = v)
v = 10gr t
Power is a scalar quantity as it is the ratio of two
When the block is at C, the track exerts a normal scalars W and t, or a scalar product of two vectors F
reaction N on the block. Since the block is moving and v.
in a circular path, the necessary centripetal force for
circular motion is provided by the normal reaction 6.17
(Fig. 6.10). An engine pulls a car of mass 1000 kg on a level road
at a constant velocity of 5 ms–1 . If the frictional force
is 500 N, what power does the engine generate? What
extra power must the engine supply to maintain the
same speed up an inclined plane having a gradient of
1 in 10?

SOLUTION
Since the car moves at a constant velocity, its accel-
eration is zero. Hence the engine has to do work only
Fig. 6.10
to overcome the frictional force f .
m v2 m 10 gr Power = f v = 500 5 = 2500 W
N= = = 10 mg
r r = 2.5 kW
Thus the track exerts a force on the block equal to 10 For an inclined plane having a gradient of 1 in 10, sin
times the weight of the block. 1
= . To maintain the same speed up the inclined plane,
To complete the circle, the minimum speed at D must 10
be vmin = 5gr . Hence the engine has to do extra work against the force mg sin .
6.10 Comprehensive Physics—JEE Advanced

Therefore, 1 3 3
v =
Extra power = mg sin v 3 m
1 v3 3
= 100 9.8 5 = 4900 W =
10 3x
= 4.9 kW 3 3
3000 5 2
=
6.18 3 117
= 1000 W = 1 kW

with water. If the tank is 40 m above the ground and 6.20


A car of mass m starts from rest at time t = 0 and is
driven on a straight horizontal road by the engine
Take g = 10 ms–2 . which exerts a constant force If friction is negli-
gible, the car acquires kinetic energy E at time t and
SOLUTION develops a power . Which of the following is/are
Volume of tank V = 2000 litre = 2000 10–3 m3 = 2 m3 correct ?
Mass of water m = V = 1000 2 = 2 103 kg (a) E t (b) E t2
Work done to lift this mass to a height = 40 m is (c) t (d) t2
W= = 2 103 10 40 = 8 105 J
SOLUTION
W 8 105 4 F
Power needed = = = 103 W Since F is constant, acceleration a = is constant.
t 10 60 3 m
If is the total power consumed, the useful power At time t, the velocity of the car is
Ft
0.4 . Hence v= + at = 0 + at = at = .
m
4 Ft 2
0.4 = 103 Kinetic energy E at time t =
1 1
mv2 = m
3 2 2 m
= 3.33 103 W = 3.33 kW F2 2
= t .
m
6.19 Since F is constant, E t2 .
A constant power is supplied to a car of mass Ft F2
m = 3000 kg. The velocity of the car increases from Power at time t = Fv = F = t
m m
= 2 ms–1 to v = 5 ms–1 when the car travels a distance
x = 117 m. Find the value of . Neglect friction. Thus t. So the correct choices are (b) and (c).

SOLUTION 6.21
A body, initially at rest, moves in a straight line
= Fv = mav a=
mv
dv dv d x vd v displacement in time t is proportional to
Now a= (a) t1/2 (b) t
dt dx dt dx
3/2
dv (c) t (d) t2
v =
dx mv
SOLUTION
2
v dv = dx
m dv
= Fv = mav = m v
v x dt
v2 d v = dx vdv = dt
m 0 m
Work, Energy and Power 6.11

v t
2 1/2
Integrating vd v = dt ( = constant) dx = t dt
m0 m
0
x t
v2 dx =
2
t1/2 dt
= t
2 m 0
m 0

2 1/2 4 3/2
v= t x= t
m 3m
dx 2 1/2 So the correct choice is (c).
= t
dt m

Multiple Choice Questions with Only One Choice Correct


1. A particle of mass m is moving in a circular path of 5. Force F acting on a body moving in a straight line
a constant radius r such that its centripetal accelera- varies with the velocity v of the body as F = /v
tion ac is varying with time t as ac = 2 r t2 where where is a constant. The work done by the force
is a constant. The power delivered to the particle in time t is proportional to
by the force acting on it is (a) t (b) t3/2
2 2 2 2 –1/2
(a) 2 rt (b) rt (c) t (d) t–3/2
4 2 5
r t 6. The force F acting on a body varies with its dis-
(c) (d) zero
3 placement x as F = –2/3. The power delivered by
IIT, 1994 the force will be proportional to
2. A stone is tied to a string of length L and whirled in (a) x–3/2 (b) x–1/2
a vertical circle with the other end of the string at (c) x1/2 (d) x3/2
the centre. At a certain instant of time, the stone is 7.
at the lowest position and has a speed . The mag- of 200 ms–1. After passing through the plank, its
nitude of the change in its velocity as it reaches a speed reduces to 180 ms–1. Another bullet, of the
position where the string is horizontal is same mass and size but moving with a speed of 100
(a) 2
2 L (b) 2 gL ms–1
speed of this bullet after passing through the plank?
2 2 Assume that the resistance offered by the plank is
(c) L (d) 2( L)
the same for both the bullets?
IIT, 1998
(a) 48 ms–1 (b) 49 ms–1
3. The power supplied to a body initially at rest var- (c) 50 ms–1 (d) 51 ms–1
ies with time t as = 2 where is a constant. The
velocity of the body at an instant of time t will be 8. If the mass of either bullet in Q. 10 is 7 g and the
proportional to thickness of the wooden plank is 1 m, what is the
average resistance offered by the plank?
(a) t (b) t3/2
(a) 36 N (b) 38 N
(c) t2 (d) t3
(c) 40 N (d) 42 N
4. A force F = (3 i 4 j) newton acts on a particle 9. An engine pulls a car of mass 1500 kg on a level
moving along a line 4y + = 3. The work done by road at a constant speed of 5 ms–1. If the frictional
the force is zero if the value of is force is 1500 N, what power does the engine gener-
(a) 1 (b) 2 ate?
(c) 3 (d) 4
6.12 Comprehensive Physics—JEE Advanced

(a) 5.0 kW (b) 7.5 kW 17.


(c) 10 kW (d) 12.5 kW -
10. In Q. 9, what extra power must the engine develop ing a thickness of 3.5 cm. The total thickness pen-
to maintain the same speed up an inclined plane etrated by the bullet is
having a gradient of 1 in 10? Take g = 10 ms–2. (a) 8 cm (b) 10 cm
(a) 2.5 kW (b) 5.0 kW (c) 12 cm (d) 14 cm
(c) 7.5 kW (d) 10 kW 18.
11. Two identical cylindrical vessels, with their bases
at the same level, each contain a liquid of density . etrating a thickness x of the plank. What is the total
The height of the liquid in one vessel is 1 and that thickness penetrated by the bullet?
in the other is 2. The area of either base is A. What (a) 2x (b) 4x
is the work done by gravity in equalizing the levels (c) 6x (d) 8x
when the vessels are interconnected? 19. A body of mass m, having momentum , is moving
(a) A g ( 1 – 2)2 (b) A g ( 1 + 2)2 on a rough horizontal surface. If it is stopped in a
2 2 distance x
1 2 1 2
(c) A g (d) A g body and the surface is given by
2 2 2 2
12. (a) = (b) =
3 2 gm 2 x 2mgx
with water. If the tank is 60 m above the ground
(c) = (d) =
2mgx 2 gm 2 x
the tank? Take g = 10 ms–2. 20. A uniform chain of mass M and length L is held on
(a) 100 kW (b) 150 kW 1
a horizontal frictionless table with th of its length
(c) 200 kW (d) 250 kW n
hanging over the edge of the table. The work done is
13. The distance x moved by a body of mass 0.5 kg by
pulling the chain up on the table is
a force varies with time t as
Mg Mg
x = 3t2 + 4t + 5 (a) (b)
n 2n
where x is expressed in metre and t in second. What
Mg Mg
(c) 2
(d)
(a) 25 J (b) 50 J n 2 n2
(c) 75 J (d) 100 J 21. A body of mass m = 1 kg is dropped from a height
14. In a hydroelectric power station, the height of the
dam is 10 m. How many kg of water must fall per
second on the blades of a turbine in order to gener- base, as shown in Fig. 6.11. As a result the spring
ate 1 MW of electrical power? Take g = 10 ms–2. is compressed by an amount x = 10 cm. What is the
force constant of the spring. Take g = 10 ms–2.
(a) 103 kgs–1 (b) 104 kgs–1
(a) 600 Nm–1 (b) 800 Nm–1
(c) 105 kgs–1 (d) 106 kgs–1 –1
(c) 1000 Nm (d) 1200 Nm–1
15. A uniform steel rod of mass m and length is
pivoted at one end. If it is inclined with the horizon-
tal at an angle its potential energy will be
1 1
(a) cos (b) sin
2 2
(c) cos (d) sin
16. A bullet, incident normally on a wooden plank,
loses one-tenth of its speed in passing through the
plank. The least number of such planks required to
stop the bullet is
(a) 5 (b) 6
(c) 7 (d) 8 Fig. 6.11
Work, Energy and Power 6.13

22. A force acts on a particle of mass 3 g in such a


(a) zero (b) 20 2 m/s
way that the position of the particle as a function
of time is given by (c) 20 3 m/s (d) 40 m/s
x = 3t – 4t2 + t3
where x is in metres and t is in seconds. The work 29. A force F = – K ( y i + x j ) where K is a positive
constant, acts on a particle moving in the x-y plane.
(a) 570 mJ (b) 450 mJ Starting from the origin, the particle is taken along
the positive x-axis to the point (a, 0) and then par-
(c) 490 mJ (d) 530 mJ
allel to the y-axis to the point (a, a). The total work
23. A sphere of mass m is tied to one end of a string done by the force F on the particle is
of length and rotated through the other end along (a) – 2 Ka2 (b) 2 Ka2
a horizontal circular path with speed v. The work (c) – Ka 2
(d) Ka2
done in one full horizontal circle is
IIT, 1998
(a) zero (b) mg 2
30. A wind-powered generator converts wind energy
mv 2 mv 2 into electrical energy. Assume that the generator
(c) 2 (d)
-
24. The kinetic energy acquired by a mass m in travel- tercepted by its blades into electrical energy. For
ling a certain distance d, starting from rest, under wind speed v, the electrical power output will be
the action of a constant force is proportional to
(a) v (b) v2
(a) directly proportional to m
(c) v 3
(d) v4
(b) independent of m
1 IIT, 2000
(c) directly proportional to 31. A body of mass 6 kg is acted upon by a force which
m
(d) directly proportional to m t2
causes a displacement in it given by x = metre
25. A position dependent force F = 7 – 2x + 3x 2 newton 4
acts on a body of mass 2 kg and displaces it from where t is the time in second. The work done by the
x = 0 to x = 5 m. The work done in joules is force is 2 seconds is
(a) 70 (b) 270 (a) 12 J (b) 9 J
(c) 35 (d) 135 (c) 6 J (d) 3 J
32. A ladder 2.5 m long and of weight 150 N has its
26. A particle is moved from a position r1 = (3 i + centre of gravity 1 m from its bottom. A weight of
2 j – 6 k ) metre to a position r2 = (14 i + 13 j + 40 N is attached to the top end. The work required
to raise the ladder from the horizontal position to
9 k ) metre under the action of a force F = (4 i + j the vertical position is
+ 3 k ). What is the work done? (a) 190 J (b) 250 J
(a) 10 J (b) 100 J (c) 285 J (d) 475 J
(c) 0.01 J (d) 1 J 33. A body of mass 5 kg rests on a rough horizontal
27.
energy increases by pulled through a distance of 10 m by a horizontal
force of 25 N. The kinetic energy acquired by it is
(take g = 10 ms–2)
28. A particle of mass 0.1 kg is subjected to a force (a) 200 J (b) 150 J
which varies with distance as shown in Fig. 6.12. (c) 100 J (d) 50 J
If it starts its journey from rest at x = 0, its velocity 34. A body is moving up an inclined plane of angle
at x = 12 m is with an initial kinetic energy E
friction between the plane and the body is . The
work done against friction before the body comes
to rest is:
E cos
(a) (b) E cos
cos sin
E cos E cos
(c) (d)
Fig. 6.12 cos sin cos sin
6.14 Comprehensive Physics—JEE Advanced

35. A particle moves in a straight line with retardation


proportional to its displacement. Its loss of kinetic
energy for any displacement x is proportional to
(a) x2 (b) ex
(c) x (d) loge x
36. A body of mass m accelerates uniformly from rest
to velocity v1 in time t1. The instantaneous power
delivered to the body as a function of time t is Fig. 6.14
mv1t mv12t 40. An escalator is moving downwards with a uniform
(a) (b)
t1 t12 speed . A man of mass m is running upwards on
it at a uniform speed v. If the height of the escala-
mv12t 2 mv1t 2 tor is , the work done by the man in going up the
(c) (d)
t12 t1 escalator is
(a) zero (b)
37. A force F 5 i 3 j 2k newton is applied to
mg mg v
(c) (d)
a particle which displaces it from its origin to the v v
point r (2 i j) metre. The work done on the 41. The potential energy (in joule) of a body of mass 2
kg moving in the x – y plane is given by
particle (in joule) is
U = 6x + 8y
(a) – 7 (b) + 7 where the position coordinates x and y are measured
(c) + 10 (d) + 13 in metre. If the body is at rest at point (6 m, 4 m) at
38. Two masses M and m (with M > m) are connected by time t = 0, it will cross the y-axis at time t equal to
means of a pulley as shown in Fig. 6.13. The system (a) 1 s (b) 2 s
is released. At the instant (c) 3 s (d) 4 s
when mass M has fallen 42. In Q. 41 above, the speed of the body when it cross-
through a distance , the es the y-axis is
velocity of mass m will (a) zero (b) 5 ms–1
be (c) 10 ms–1 (d) 20 ms–1
(a) 2 43. If W1, W2 and W3 represent the work done in mov-
ing a particle from A to B along three different paths
2 M 1, 2 and 3 (as shown in Fig. 6.15) in the gravitational
(b)
m m
between W1, W2 and W3.
2 M m
(c)
M m

2 M m Fig. 6.13
(d)
M m
39. A mass m, lying on a horizontal frictionless sur-face
is connected to mass M as shown in Fig. 6.14. The
system is now released. The velocity of mass m
when mass M as descended a distance is
2 Mg 2 mg
(a) (b) Fig. 6.15
m M
(a) W1 > W3 > W2 (b) W1 = W2 = W3
2 Mg (c) W1 < W3 < W2 (d) W1 < W2 < W3
(c) (d) 2
M m IIT, 2003
Work, Energy and Power 6.15

44. A particle, which is constrained to move along the


x-axis, is subjected to a force in the same direc-
tion which varies with the distance x of the particle
from the origin as F(x) = – + ax3. Here and
a are positive constants. For x 0, the functional
form of the potential energy U(x) of the particle is
(see Fig. 6.16)

Fig. 6.17
–1
(a) 3 ms (b) 4 ms–1
(c) 5 ms–1 (d) 6 ms–1
48. A body of mass m is dropped from a height above
the ground. The velocity v of the body when it has
lost half its initial potential energy is given by
(a) v = (b) v = 2

(c) v = (d) v = 2
2
49. A body of mass m is thrown vertically upwards with
a velocity v. The height at which the kinetic en-
Fig. 6.16 ergy of the body is half its initial value is given by
IIT, 2002 v2 v2
(a) = (b) =
45. A raindrop of radius r falls from a certain height g 2g
above the ground. The work done by the gravita-
v2 v2
tional force is proportional to (c) = (d) =
3g 4g
(a) r (b) r2
50. A car of mass m moving at a speed v is stopped in
(c) r3 (d) r4
a distance x by the friction between the tyres and
46. A smooth steel ball is moving to and fro about the the road. If the kinetic energy of the car is doubled,
lowest position O of a frictionless hemispherical its stopping distance will be
bowl. The ball attains a maximum height of 20 cm (a) 8x (b) 4x
on either side of O. If g = 10 ms–2, the speed of the (c) 2x (d) x
ball when it passes through O will be 51. A body is allowed to fall freely under gravity from
(a) 2 ms–1
impact with the ground, to what height will it rise
(b) 2 ms–1
after one impact?
(c) 0.2 ms–1 (a) 2.5 m (b) 5.0 m
(d) 0.02 ms–1 (c) 7.5 m (d) none of these
52. In Q. 51, to what height will the body rise after
47. The bob of a pendulum is released from a horizon-
two such impacts with the ground?
tal position A as shown in Fig. 6.17. The length of
(a) 2.5 m (b) 5.0 m
(c) 7.5 m (d) none of these
of the bob is dissipated as heat due to the friction 53. A body of mass m thrown vertically upwards attains
of air, what would be the speed of the bob when it a maximum height . At what height will its kinetic
reaches the lowermost point B? Take g = 10 ms–2.
6.16 Comprehensive Physics—JEE Advanced

displaced towards wall 1 by a small distance x


(a) (b)
6 5
a maximum distance y towards wall 2. Displace-
(c) (d) ments x and y are measured with respect to the equi-
4 3 x
54. A body, having kinetic energy , moving on a librium position of the block B. The ratio is
y
rough horizontal surface, is stopped in a distance (a) 4 (b) 2
x. The force of friction exerted on the body is 1 1
(c) (d)
(a) (b) 2 4
x x

(c) (d)
x
55.
force F = , where is a positive constant. If the
potential energy U is zero at x = 0, the variation
of potential energy with the coordinate x is repre-
sented by [see Fig. 6.18]

Fig. 6.19
IIT, 2008
57. A bob of mass m is suspended by a massless string
of length L. The horizontal velocity v at position A
B. The
angle at which the speed of the bob is half of that
at A

Fig. 6.18
v
IIT, 2004
Fig. 6.20
56. A block (B) is attached to two unstretched springs
S1 and S2 with spring constants and 4 , respec- (a) = (b)
tively (see Figure 6.19). The other ends are attached 4 4 2
to identical supports M1 and M2 not attached to the 3 3
(c) (d)
walls. The springs and supports have negligible 2 4 4
mass. There is no friction anywhere. The blook B is
IIT, 2008

ANSWERS

1. (b) 2. (d) 3. (b) 4. (c) 5. (a) 6. (b)


7. (b) 8. (b) 9. (b) 10. (c) 11. (c) 12. (a)
13. (c) 14. (b) 15. (b) 16. (b) 17. (a) 18. (b)
19. (a) 20. (d) 21. (c) 22. (d) 23. (a) 24. (b)
25. (d) 26. (b) 27. (a) 28. (d) 29. (c) 30. (c)
31. (d) 32. (b) 33. (b) 34. (d) 35. (a) 36. (b)
37. (b) 38. (c) 39. (c) 40. (d) 41. (b) 42. (c)
Work, Energy and Power 6.17

43. (b) 44. (d) 45. (c) 46. (b) 47. (d) 48. (a)
49. (d) 50. (c) 51. (c) 52. (d) 53. (c) 54. (a)
55. (a) 56. (b) 57. (d)

SOLUTIONS
v2 Work done is zero if the force is perpendicular to
1. ac = 2
r t2 = 2 r t2 v= the displacement, i.e. if lines (1) and (2) are perpen-
r
dv dicular each other. Thus the product of their slopes
Tangential acceleration at = =
dt = – 1, i.e.
Tangential force F = m at = 4
=–1 =3
Power = F v = = 2 2
rt 3 4
2. Refer to Fig. 6.21.
5. Power = F v = v = . Therefore, work done
in time t is v
t
W= t = ( = constant)
0
Hence the correct choice is (a).
6. Given F x–2/3. Therefore, acceleration x–2/3, i.e.
dv
v = K x–2/3 (K = constant)
dx
vdv = K x 2/3
dx
Fig. 6.21 v2 x1/3 or v x1/6
From conservation of energy, = F v x–2/3 x1/6 x–1/2
1 2 1 7. Let m be the mass of each bullet. Since the resis-
= mv2 + mgL
2 2 tance offered by the plank is the same for the two
bullets, the amount of work done by the plank is
v= 2
2 L the same for the two bullets. From work-energy
Change in velocity v = v = v ( ) . Thus principle, the decrease in the kinetic energy is the
v is the resultant of v and – . It follows from same for the two bullets.
Fig. 6.21 (b) that
1 1
v = v2 ( )2 = 2
1– mv 12
2 2
2 2 2
= 2 = 2( L) 1 1
= m(200)2 – m (180)2 (i)
3. From work-energy principle, change in K.E. = work 2 2
done. If v2 is the speed of the second bullet after passing
3 through the plank, then
1
mv2 = t = t 2 dt = Decrease in KE of second bullet
2 3
1 1
2 = m 22 – mv 22
v= t3 2 2
3m
1 1
Hence the correct choice is (b). = m(100)2 – mv 22 (ii)
2 2
4. The force F is parallel to the line
Equations (i) and (ii) we have,
4
y= x + C (1) 1 1
3 m(200)2 – m (180)2
The particle moves along the line 2 2
3 1 1
y= – (2) = m(100)2 – mv 22
4 4 2 2
6.18 Comprehensive Physics—JEE Advanced

which gives v 22 = 2400 or v 49 ms –1 Hence


Hence the correct choice is (b). PE after connection = A 1 2
g 1 2

8. Thickness of plank (S ) = 1.0 m 2 2


Mass of bullet (m) = 10 g = 10–2 kg A g 2
= ( 1 + 2)
If F is the average resistive force exerted by the 4
plank on the bullet, the work done by the plank on
A g 2 2 2
the bullet is Change in PE = {( 1 + 2) – 2( 1 + 2)}
W = FS 4
Work done = decrease in KE of the bullet. A g 2
=– ( 1 – 2)
4
1 1
Hence FS = m(200)2 – m(180)2 = 3800 m 2
2 2 1 2
=–A g
2
Putting S = 1 m and m = 10–2 kg, we get F = 38 N.
Hence the correct choice is (b). This must be equal to the work done ‘by’ gravity on
9. Since the car moves at a constant velocity, its the liquid. Thus the work done ‘by’ gravity is
2
acceleration is zero. Hence the engine has to do 1 2
work only to overcome the frictional force (f). A g
2
Since the distance moved in 1 second is v metres,
Hence the correct choice is (c).
the work done per second or the power of the en-
12. Volume (V) = 30 m3, density of water ( ) = 1000 kg
gine is
m–3. Therefore, mass of water to be lifted is
= f v = 1500 5 = 7500
m= V = 1000 30 = 3 104 kg
W = 7.5 kW
Hence the correct choice is (b). Work done to lift this mass of water to a height
10. When the car is being pulled along an inclined plane = 60 m is
1 W= = 3 104 10 60
of gradient 1 in 10 i.e. sin = , the engine has
10 = 1.8 107 J
to do extra work against the component Mg sin
of the weight Mg of the car. The extra work per
work done by the pump is
second or extra power the engine has to develop to
maintain the same speed v is W 100 1.8 107 100
W = =
= Mg sin v 30 30
1 = 6 107 J
= 1500 10 5
10 Time taken t = 10 min = 600 s. Therefore, power
= 7500 W = 7.5 kW consumed is
Hence the correct choice is (c). W 6 107
= =
11. The work done by gravity equals the change in the t 600
potential energy of the system after the vessels are = 100,000 W = 100 kW
interconnected. We may regard the liquid in each
Hence the correct choice is (a).
vessel as equivalent to a point mass kept at their
respective centres of gravity. Remembering that dx d
the mass of the liquid is given by ( ) and that 13. Velocity (v) = = (3t2 + 4t + 5) = 6t + 4.
dt dt
the PE of a mass at a height in earth’s gravity is
, we have dv d
Acceleration is a = = (6t + 4) = 6 ms–2.
1 2 dt dt
Total PE at start = (A 1 )g + (A 2 )g
2 2 Therefore, applied force is F = ma = 0.5 6 = 3 N.
A g Now t = 2s, the distance moved is
= ( 21 + 22)
2 x =3 (2)2 + 4 2 + 5 = 25 m
After the vessels are connected, the height of liquid Work done W = Fx = 3 25 = 75 J. Hence the
in each vessel is ( 1 + 2)/2. correct choice is (c).
Work, Energy and Power 6.19

14. Let M kg of water fall per second. The power is penetrated by the bullet, then
= rate at which work is done = mass per second 2
– v 2 = 2ax
g = 2 2
But = 1 MW = 106 W, = 10 m. Therefore or x = . But a = cms–2.
2a 16
106
M= = = 104 kg s–1, 2
16
10 10 Therefore x = = 8 cm
2
2
which is choice (b).
Hence the correct choice is (a).
15. The weight of the rod acts at the centre of gravity
18. Since the wood offers a constant deceleration
which is at a distance of /2 from the pivoted end.
and hence a constant retardation force, the bullet
When the rod makes an angle with the horizontal,
the vertical height of the centre of gravity is
after penetrating a further distance of 3x. Therefore,
= sin the total distance penetrated by the bullet before
2 it comes to rest = x + 3x = 4x. Hence the correct
i.e. the centre of gravity rises by an amount . choice is (b).
Therefore 19. Force of friction = mg. Therefore, retardation
1 a = mg/m = g.
PE = = sin Also 2ax = v2 or 2am2x = m2v2. But = mv.
2
Therefore,
Hence the correct choice is (b). 2 am 2x = 2
16. Let v
But a = g. Therefore, 2 g m 2x = 2
9v
plank. Its speed after it passes the plank = . If x 2
10
or = . Hence the correct choice is (a).
is the thickness of the plank, the deceleration a due 2 g m2 x
to the resistance of the plank is given by M
2 20. The mass per unit length of the chain m = .
9v 19 v2
L
2ax = v2 – = (i) The mass of the hanging portion of the chain is
10 100
mL
Suppose the bullet is stopped after passing through m = . This mass can be assumed to be con-
n such planks. Then the distance covered by the n
bullet is = nx. Thus, we have centrated at the centre of the hanging portion of the
v2 – 0 = 2 = 2anx chain which is a distance of x =
L
from the edge
2n
v2 v 2 100 100 of the table. Therefore, the work done in pulling the
or n= = = = 5.26
2a x 19 v 2
19 hanging portion of the chain on to the table top is
Thus the minimum number of planks required is 6. mL L
W = m gx = g
Hence the correct choice is (b). n 2n
17. Let cms–1 be the speed of the bullet. Since the
mg L2 M gL
mass of the bullet remains unchanged, its speeds = 2
=
3 2n 2n2
becomes v = cms–1 after it penetrates a distance Hence the correct choice is (d).
4
x = 3.5 cm. The retardation a due to the resistance 21. Since the platform is depressed by an amount x,
of the wooden plank is given by the total work done on the spring is mg ( + x).
2
This work is stored in the spring in the form of
2 3
– v2 = 2ax or 2
– = 2a 3.5 potential energy
1 2
. Equating the two, we
4 2
2
have
which gives a = cms–2. The bullet will come to
16 1 2 2m g
= mg ( + x) or =
rest when its velocity v = 0. If x is the thickness 2 x2
6.20 Comprehensive Physics—JEE Advanced

Given, = 0.4 m, x = 0.1 m, m = 1 kg and 26. The displacement of the particle is


g = 10 ms–2. Substituting these values, we get
r = r2 – r1 = (14 i + 13 j + 9 k )
= 1000 Nm–1. Hence the correct choice is (c).
22. The instantaneous velocity of the particle is – (3 i + 2 j – 6 k )
dx d
v= = (3t – 4t2 + t3) = 3 – 8t + 3t2 = (11 i + 11 j + 15 k )
dt dt
The instantaneous acceleration of the particle is Work done = F r

a=
dv
=
d
(3 – 8t + 3t2) = – 8 + 6t = (11 i + 11 j + 15k ) (4 i + j + 3k )
dt dt = 4 11 + 11 + 3 15 = 100 J
Hence the correct choice is (b).
4 4 2
dx 1
W= Fd x = ma dt 27. Momentum = mv or 2 = m2v2 or =
dt 1 2 m
0 0
mv2. Thus the kinetic energy is
4 2 2
=m (– 8 + 6t) (3 – 8t + 3t 2) dt K =
0 2m
4 If
=m (– 24 + 82t – 72t 2 + 18t3) dt = 1.2 . Therefore the new kinetic energy will
0 be
9 44 1.2 2 2

=m 24t 41t 2 t 24t 3 K = = 1.44 = 1.44 K


2 0 2m 2m
= m (– 96 + 656 – 1536 + 1152) i.e K increases by 0.44 K. The percentage increase
0.44 K
= 176 m = 176 3 10–3 in K is
K
( m = 3 10–3 kg)
28. Work done = area under the (F – x) graph
= 528 10–3 = 528 mJ
( 1 mJ = 10–3 J) =
1
10 4 + 10 4+
1
The closest choice is (d). 2 2
23. The centripetal force, being directed towards the 10 4 = 80 J
centre is always perpendicular to the direction of Now, work done = increase in kinetic energy. If v
displacement which is the direction of the velocity. is the velocity at x = 2 m, then increase in K.E. =
Thus the dot product F S = 0, i.e. work done is zero, 1
which is choice (a). mv 2. Therefore
2
1
24. Kinetic energy K = mv 2. If a is the accelera- 1 80 2 80 2
2 mv 2 = 80 or v2 = =
2 m 0.1
tion, then v2 = 2ad. But a = force/mass = F/m.
2F d 1 2F d = 1600
Therefore, v2 = . Hence K = m or v = 40 ms–1, which is choice (d).
m 2 m
= Fd, which is independent of m. Thus the correct 29. In going from (0, 0) to (a, 0), the x-coordinate varies
choice is (b). from 0 to a while the y-coordinate remains zero.
25. Work done is Work done by force F along this path is ( y = 0)
5 5 a a

W= Fdx = 7 2 x 3x d x2 W1 = F dx = ( K x j) dx i = 0
0 0 0 0

= 7x x2 x3
5 ( j i = 0)
0
In going from (a, 0) to (a, a), the x-coordinate
= 7 5 – (5)2 + (5)3 = 135 J remains constant at x = a while the y-coordinate
Hence the correct choice is (d). changes from 0 to a.
Work, Energy and Power 6.21

Work done by force F along this path is ( x = a) 1


a a
= m 2 = (ii)
2
W2 = F dy = –K yi aj dy j
0 0
a
= – Ka dy = – Ka2
0
( i j = 0, j j = 1)
Since work is a scalar quantity, the total work done
is Fig. 6.22
W = W1 + W2 = 0 – Ka2 = – Ka2
Now, work done is
Hence the correct choice is (c).
W = gain in PE =
30. The power output of the generator is directly pro-
portional to (i) the velocity v of the air molecules It is clear from = sin . There-
in wind and (ii) the average kinetic energy KE = fore,
1 W= sin (iii)
mv2 of the striking molecules (which is propor-
2 From (i), we have
tional v2). Hence a
Power output v v2 v3 g= (iv)
( cos sin )
which is choice (c).
31. The velocity of the body at time t is given by sin
Also = tan =
dx d t2 t cos
v= = =
dt dt 4 2 or sin = cos (v)
At t = 0, v = = 0 and t = 2 s, v = 1 ms–1, Now, Using (iv) and (v) in (iii), we have
work done = increase in KE ma ( cos )
1 1 1 W= (vi)
= mv2 – 2
= mv2 – 0 ( cos sin )
2 2 2 Using (ii) in (vi), we get
1 1
= mv2 = 6 (1)2 E cos
2 2 W= , which is choice (d).
( cos sin )
= 3 J. Hence the correct choice is (d).
32. Work done = increase in potential energy in (i) raising 35. Retardation (– a) is proportional to displacement
the weight 150 N of the ladder through a height 1 m (x), i.e. – a x or a – x. Hence the motion of
and (ii) raising a weight 40 N through 2.5 m the particle is simple harmonic. When the displace-
1
= 150 N 1 m + 40 N 2.5 m ment is x, the kinetic energy = m 2 (A2 – x2),
2
= 250 Nm = 250 J where m, and A are the mass, angular frequency
Hence the correct choice is (b). and amplitude respectively. When displacement
33. Friction force = mg = 0.2 5 10 = 10 N. 1
Effective force F = applied force – frictional force x = 0, the kinetic energy = m 2 A2. Therefore,
2
= 25 – 10 = 15 N. Kinetic energy = work done the loss of kinetic energy for a displacement x is
by force F in pulling the body through a distance
1 1 1
S (= 10 m) = 15 10 = 150 J, which is choice (b). m 2A2 – m 2 (A2 – x2 ) = m 2x2
34. The retardation is given by [see Fig. 6.22] 2 2 2
which is proportional to x2 (since m and are con-
a = g ( cos + sin ) (i)
stants of the motion. Hence the correct choice is (a).
Let be the initial velocity of the body. If it is 36. Power delivered in time t1 is 1 = F v1 = m a v1
stopped after moving a distance up the plane, v1
then Now, acceleration vector is a . Therefore
2 t1
=2
1 m v1 v1 mv12
Kinetic energy = E = 2
1 = ( v1 v1 = v12)
2 t1 t1
6.22 Comprehensive Physics—JEE Advanced

Therefore, the x and y components of acceleration


1
Power delivered per unit time = are
t1 F 6
ax = x = 3 ms–2
1
mv12t m 2
Power delivered at time t = t=
t1 t12 Fy 8
Hence the correct choice is (b). Notice that choices and ay = = 4 ms–2
m 2
(a), (c) and (d) do not have the dimensions of power.
Resultant acceleration a = ax2 a 2y
37. W = F r = 5i 3j 2k 2i j
2 2
= 3 4 = 5 ms–2
= 10 i i 3j j i j k i k j 0 The x and y coordinates of the body at time t are
1 1
= 10 – 3 = 7 x = x0 – a x t2 = 6 – 3 t2
2 2
Hence the correct choice is (b).
38. If mass m falls through a distance , mass m rises up = 6
3 2
t metre
through the same distance . Let v be the common 2
velocity of the masses when this happens. Now,
loss in PE = gain in KE, i.e. 1 1
and y = y0 ay t 2 4 4 t2
1 2 2
– = (M + m) v2
2 = 4 2t 2 metre
2 M m The body will cross the y-axis when x = 0, i.e. at
which gives v = , which is choice
(c). M m 3 2
time t given by 6 t = 0 or t = 2 s. Hence the
39. When M has descended a distance , loss of PE = 2
. If v is the common velocity of the masses, correct choice is (b).
1 42. vx = ax t = 3 ms–2 2 s = 6 ms–1
gain in KE = (M + m) v2. Hence
2 vy = ay t = 4 ms–2 2 s = 8 ms–1
1
(M + m) v2 = v= v 2x v 2y 6 2
8 2
= 10 ms–1
2
Hence the correct choice is (c).
2Mg 43. Gravitational force is conservative. The work done
or v= , which is choice (c).
M m by a conservative force on a particle moving be-
40. Relative speed of man with respect to escalator = tween two points does not depend on the path taken
(v – ). by the particle. Hence the correct choice is (b).
Actual displacement of man per second = 44. The potential energy of the particle is given by
(v – ). U=– Fdx = – (– + ax3) dx
Hence, the actual displacement of man in going up
v x2 x4 x2
the escalator of hight is . Therefore, or U= a = (2 – ax2) (1)
(v ) 2 4 4
v From Eq. (1) it follows that U = 0 at two values of
Work done = mg , which is choice (d)
v v x which are x = 0 and x = 2 / . Hence graphs
41. Given U = 6x + 8y joule and mass m = 2 kg. Force (b) and (c) are not possible. Also U is maximum or
dU
along x-axis is minimum at a value of x given by = 0, i.e.
dU d dx
2
|Fx| = (6x + 8y) = 6 newton d a x4
dx dx 0= = – ax3
Force along y-axis is dx 2 4
dU d = x( – ax2)
|Fy| = (6x + 8y) = 8 newton
dy dy or x= / . At this value of x,
Work, Energy and Power 6.23

d 2U 1
U is maximum if < 0, 49. Initial KE = mv2. Now, gain in PE = loss in KE.
d x2 Thus 2
1
d 2U d = mv2
Now 2
= ( – ax3) = – 3ax2. 4
dx dx
v2
or =
At x= / , 4g
d 2U Hence the correct choice is (d).
= – 3a
dx 2
a 50. If a is the deceleration due to the force of friction f,
then 2ax = v2
= – 3 = – 2 , which is negative.
1
Hence U is maximum at x = / . or mv 2 = max
2
Hence graph (a) is also not possible. Also U is neg- or KE = f x ( f = ma)
ative for x > 2 / . Therefore, the correct graph Thus if KE is doubled, x is also doubled. Hence the
is (d). correct choice is (c).
45. Mass of the drop m = volume density of water = 51. Height = 10 m. PE at this height = . On
4 3 reaching the ground, KE = . Since the body
r , where is the density of water. Work done
3
by gravitational force is after one impact = 0.75 . If v1 is the initial
upward velocity after the impact, we have
4 3 1 3
W= r
= mv 21 = 0.75 =
3 2 4
Thus W r3. Hence the correct choice is (c).
or v 21 = 1.5
46. At the highest point, the energy of the ball is
entirely potential = and at the lowest point, the The height 1 to which the body will rise is
1
energy is entirely kinetic = mv2. Since friction v12 1.5
2 1 = = = 0.75
is absent, the principle of conservation of energy 2g 2g
requires = 0.75 10 = 7.5 m ( = 10 m)
1 Hence the correct choice is (c).
mv2 =
2 52. After the second impact, the initial KE of body =
3 3 2
or v = 2 2 10 0.2 = 2 ms–1. Hence the = , i.e.
4 4
correct choice is (b). 2
1 3
47. PE at A = mv 22 =
2 4
90
at point B = = 0.9 . Therefore, 9
100 or v 22 =
8
1
mv2 = 0.9 The height 2 to which the body will rise after the
2 second impact is
or v2 = 1.8 = 1.8 10 2 = 36 v22 9 9 9 10 45
which gives v = 6 ms–1. Hence the correct choice 2 = = = = = m
2g 8 2g 16 16 8
is (d).
48. Initial PE = . Now, gain in KE = loss in PE. Hence the correct choice is (d).
Thus 53. As the body rises, the initial kinetic energy is
1 1 converted into potential energy. At the maximum
mv2 =
2 2 height , the energy is entirely potential = ,
which is equal to the initial kinetic energy. Let
or v=
Hence the correct choice is (a). its initial value. At this height, the potential energy
6.24 Comprehensive Physics—JEE Advanced

1 2 2
, PE = 0.25 Thus = 0.25 or = =2
2
. Hence the correct choice is (c). y 1
4 which givens = , which is choice (c)
54. Let f be the force of friction and m be the mass of x 2
the body. The retardation a = f/m. If v is the initial 57. Refer to the Fig. 6.23. Here OA = OB = OC = L and
speed of the body, then OD = OC cos = L cos Therefore = OA – OD
2ax = v2 = L – L cos
1
or max = mv2 =
2
But ma = f. Therefore fx = or f = /x. Hence the
v
correct choice is (a).
55. Potential energy function is
x x
U(x) = 1 2
Fd xd x
0 0
2
v
The value of U(x) is always negative for both
Fig. 6.23
positive and negative values of x. Thus the variation
of potential energy with x is an inverted parabola as or = L (1 – cos ). From conservation of energy,
shown in choice (a). total energy at A = total energy at C, i.e.
56. Potential energy stored in spring S1 when the block 2
1 1 v
1 2 mv2 = m + mgL (1 – cos )
B is moved through a distance x is U1 = 1x 2 2 2
1 2 2
= . When the block is released, it moves to the 8 gL
2 v2 =(1 – cos ) (1)
left, compressing the spring S2 through a distance 3
y. The potential energy stored in spring S2 when its The minimum velocity the bob must have at A so
1 2 1 as to reach B is v = 5gL . Putting this in Eq. (1),
compression is y is U2 = 2y = (4 ) y2 = 2 2.
2 2 7 3
we get cos = – . Therefore lies between
Since y is the maximum compression of spring S2, 8 4
from conservation of energy, we have U1 = U2, i.e. and

II

Multiple Choice Questions with One or More Choices Correct


1. In which of the following is no work done by the (b) the displacement of the body
force? (c) the initial velocity of the body
(a) A man carrying a bucket of water, walking (d) the angle between the force vector and the
on a level road with a uniform velocity. displacement vector
(b) A drop of rain falling vertically with a con- 3. A simple pendulum of length L and having a bob
stant velocity. of mass M is oscillating in a plane about a vertical
(c) A man whirling a stone tied to a string in a line between angular limits – and + . At a time
circle with a constant speed when the angular displacement is (< ), the ten-
(d) A man walking up on a staircase. sion in the string is T and the velocity of the bob is
2. The work done by a force on a body does not de- v. Which of the following relations will hold?
pend upon (a) T cos = Mg
(a) the mass of the body (b) T – Mg cos = Mv2/L
Work, Energy and Power 6.25

(c) The magnitude of the tangential acceleration tude of its momentum is . Which of the following
of the bob is |aT| = g sin relations is/are correct?
(d) T = Mg cos 2
4. A particle is acted upon by a force of constant (a) = 2 (b) =
m
magnitude which is always perpendicular to the
velocity of the particle. The motion of the particle 2
takes place in a plane. It follows that (c) 2 = v (d) v =
(a) the velocity of the particle is constant
9. A block of mass m is taken from the bottom of an
(b) the acceleration of the particle is constant
inclined plane to its top and then allowed to slide
(c) the kinetic energy of the particle is constant
down to the bottom again. The length of the inclined
(d) the particle moves in a circular path.
plane is L
IIT, 1987 block and the plane is . The inclination of the plane
5. Two inclined frictionless tracks of different incli- is .
nations meet at A from where two blocks and Q (a) The work done by the gravitational force over
of different masses are allowed to slide down from
the round trip is zero.
rest at the same time, one on each track, as shown
in Fig. 6.24. (b) The work done by the applied force over the
upward journey is mgL (sin + cos ).
(c) The work done by the frictional force over the
round trip is zero.
(d) The kinetic energy of the block when it reaches
the bottom is mgL (sin – cos ).
10. A uniform rod has a mass m and a length . The
potential energy of the rod when
(a) it stands vertically is zero.
(b) it stands vertically is /2
(c) it is inclined at an angle with vertical is
Fig. 6.24 1
cos .
(a) Both blocks will reach the bottom at the same 2
time (d) it is inclined at an angle with the vertical is
(b) Block Q will reach the bottom earlier than 1
block sin .
2
(c) Both blocks reach the bottom with the same
11. A body is subjected to a constant force F in newton
speed given by
(d) Block Q will reach the bottom with a higher
speed than block F = – i + 2 j + 3k
6. Choose the correct statements from the following:
(a) When a conservative force does positive work where i , j and k are unit vectors along x, y and
on a body, its potential energy increases. z axes respectively. The work done by this force in
(b) When a body does work against friction, its moving the body through a distance of
kinetic energy decreases. (a) 4 m along the z-axis is 12 J.
(c) The rate of change of total momentum of a (b) 3 m along the y-axis is 6 J.
many–particle system is proportional to the (c) 4 m along the z-axis and then 3 m along the
net external force acting on the system. y-axis is 18 J.
(d) The rate of change of total momentum of (d) 4 m along the z-axis and then 3 m along the
many–particle system is proportional to the
net internal force acting on the system. y-axis is (12)2 (6) 2 J.
7. Which of the following forces are conservative? 12. Figure 6.25 shows the force F (in newton) acting on
(a) Coulomb force between charged particles at rest a body as a function of x. The work done in moving
(b) Force of a compressed elastic spring the body
(c) Gravitational force between two masses (a) from x = 0 to x = 1 m is 2.5 J.
(d) Frictional force. (b) from x = 1 m to x = 3 m is 10 J.
8. A body of m is moving in a straight line at a con- (c) from x = 0 to x = 4 m is 15 J.
stant speed v. Its kinetic energy is and the magni- (d) from x = 0 to x = 4 m is 12.5 J.
6.26 Comprehensive Physics—JEE Advanced

a constant force is related to time t (in seconds) as


x =t–2
(a) The acceleration of the particle is 1 ms–2.
(b) The acceleration of the particle is 2 ms–2.
(c) The velocity of the particle at t = 3 s is 2 ms–1.
(d) The work done by the force in 5 s is 1.8 J.
18. Two springs 1 and 2 have spring constants 1 and
2 ( 1 > 2). W1 and W2 are the amounts of work
done to increase the lengths of springs 1 and 2 by
Fig. 6.25 the same amount and W3 and W4 are the amounts of
work done when springs 1 and 2 are stretched with
13. A block of mass 2 kg, initially at rest on a horizontal
the same force. Then
(a) W1 > W2 (b) W1 < W2
g = 10 ms–2 (c) W3 > W4 (d) W3 < W4
(a) the work done by the applied force in 4 s is 19. A box of mass m is dragged along a horizontal
240 J. surface with a uniform speed with a force F
(b) the work done by the frictional force in 4 s directed at an angle with the horizontal as shown
is 96 J.
(c) the work done by the net force in 4 s is 336 J. between the box and the surface is .
(d) the change in kinetic energy of the block in (a) The normal reaction on the box is
4 s is 144 J. F cos
14. In which of the following cases is no work done by R= .
the force?
(a) A satellite revolving around the earth in a (b) The normal reaction on the box is
circular orbit. R = (mg – F sin )
(b) The electron revolving around the proton in (c) The work done on the box in dragging it
a hydrogen atom. through a distance x is
(c) A charged particle moving in a circle in a mgx
W= .
(sin cos )
(d) A man carrying a load on his head and walk- (d) The work done on the box in dragging it
ing on a level road at a uniform velocity. through a distance x is
15. A raindrop falls from a certain height above the mgx
ground. Due to the resistance of air, its accelera- W= .
(cos sin )
tion gradually decreases until it becomes zero
when the drop is at half its original height. If
W1 and W2 are the amounts of work done by the

of the journey, then


(a) W1 > W2 (b) W1 < W2
(c) W1 = W2 0 (d) W1 = W2 = 0 Fig. 6.26
16. A particle of mass m is moving in a horizontal
circle of radius r, under a centripetal force F = /r2 20. A force F = (3 i + 4 j ) newton acts on a particle
where is a constant. located at the origin O.
(a) The kinetic energy of the particle is /2r. (a) The work done in taking the particle from O
(b) The potential energy of the particle is – /2r. along the x-axis to a point A (2 m, 0) is 6 J.
(c) The total energy of the particle is – /2r. (b) The work done in taking the particle from A
parallel to the y-axis to a point B (3 m, 2 m)
(d) The total energy of the particle is zero.
is 8 J.
17. The displacement x (in metres) of a particle of mass
(c) The work done in taking the particle from O
100 g moving in a straight line under the action of
to A and then to B is 14 J.
Work, Energy and Power 6.27

(d) The work done in taking particle from O


mg 3mg
directly to B is 10 J. (c) T1 = , T2 =
21. The potential energy of a system varies with 2 2
distance x as 3mg mg
(d) T1 = , T1 =
U = ax2 – bx 2 2
where a and b are positive constants. Then
(a) the potential energy is minimum at x = b/2a
b2
and Umin = – .
4a
(b) the potential energy is maximum at x = b/2a
b2
and Umax = .
4a
(c) The force acting on the system decreases
linearly with x. Fig. 6.27
(d) The force acting on the system is proportional 24. The potential energy function of a particle executing
to x2. linear simple harmonic motion is given by
IIT, 2005 1 2
U (x) =
22. A car is of mass m moving along a circular track of 2
radius r with a speed which increases linearly with where x is the displacement of the particle from
time t as v = , where is a constant. Then the equilibrium position x = 0 and is the force
(a) the instantaneous power delivered by the constant of the oscillator. Figure 6.28 shows the
centripetal force is 3 3
t /r. graph of U(x) against x for = 0.5 Nm–1. If the total
(b) the power delivered by the centripetal force energy of the particle is 1 J, it will turn back when
is zero. it reaches the position
(c) the instantaneous power delivered by the (a) x = – 1 m (b) x = – 2 m
2
tangential force is t. (c) x = + 2 m (d) x = + 1 m
(d) the power delivered by the tangential force
is zero.
23. A body of weight mg is suspended from a rigid
support by two light strings AB and AC as shown
in Fig. 6.27. The tension in string AB is T1 and in
string AC the tension is T2. Then
(a) T1 = 3 T2
(b) T2 = 3 T1
Fig. 6.28

ANSWERS AND SOLUTIONS


1. In choices (a) and (c), no work is done because Relation (a) i.e. T cos = Mg cannot hold for all
the force of gravity in choice (a), and the centrip- values of because if = 0, then T = Mg and the
etal force in choice (c) are perpendicular to the net force along the string is
direction of the displacement. In choice (b), no net F = T – Mg cos 0 = T – Mg = 0
force acts on the raindrop since it is falling with a Since F = 0, there would be no centripetal force
uniform velocity, hence no work is done. In choice and the bob would not oscillate. Hence choice (a)
(d), the man has to do work against gravity. Hence is incorrect.
the correct choices are (a), (b) and (c). The tangential acceleration aT of the bob is caused
2. The correct choices are (a) and (c). by the tangential component Mg sin . Therefore,
3. Referring to Fig. 6.29, the net force along the aT = Mg sin /M = g sin . Hence choice (c) is
string is F = T – Mg cos and this force provides correct. The relation (d) cannot hold because if T
the centripetal force Mv2/L necessary for circular = Mg cos , the net force along the string will be
motion of the bob. Hence choice (b) is correct. zero. Therefore, there will be no centripetal force.
6.28 Comprehensive Physics—JEE Advanced

Hence the correct choices are (b) and (c).

Fig. 6.30
Fig. 6.29 6. The work done by a conservative force is equal
4. The particle moves in a circular path with a uni- to the negative of the potential energy. When the
form speed, because the force has a constant work done is positive, the potential energy de-
magnitude and is perpendicular to the velocity of creases. Thus choice (a) is incorrect. Friction al-
the particle. The velocity and acceleration of the ways opposes motion. Hence, when a body does
particle are not constant because their directions work against friction, its kinetic energy decreases.
are changing as the particle moves in a circular Thus choice (b) is correct. The rate of change of
path. The magnitude of the velocity (i.e. speed) is total momentum of a many–particle system is pro-
1 portional to the net force external to the system;
constant. Therefore, the kinetic energy mv2 is the internal forces between particles cannot
2
constant. Hence choices (c) and (d) are correct and change the momentum of the system. Hence the
choices (a) and (b) are incorrect. correct choices are (b) and (c).
5. Refer to Fig. 6.30. 7. The correct choices are (a), (b) and (c).
2
The accelerations of blocks and Q are 1
8. Now = mv or 2
= m2v2 or = mv2 = or
m g sin 2m 2
a1 = 1 1
= g sin 1 1
m1 = 2 . Also = mv and =mv2.
2
m2 g sin 2
and a2 = = g sin 2 Dividing the two we get 2 = v. Hence the correct
m2 relations are (a) and (c).
Since 2 > 1; a2 > a1. Now PE of block P at A = 9. The gravitational force is conservative. Therefore,
1 the work done by the gravitational force over the
m1 . Its KE on reaching the bottom = m1v 21. round trip is zero.
2
Equating the two we get Refer to Fig. 6.31.
1
m1v 21 = m1
2
or v1 = 2

Similarly, for block Q, v2 = 2 . Since v1 = v2,


both blocks will reach the bottom with the same
speed. Now, v1 = a1t1 ( = 0) and v2 = a2t2. But
v1 = v2. Therefore
Fig. 6.31
a 1t 1 = a 2t 2
When the block is moved from C to A, the force
t1 a
= 2 of friction f acts along the plane in the downward
t2 a1 direction which is in the direction of the component
Since a2 > a1; t1 > t2; i.e. block takes a longer time mg sin of the gravitational force. Hence the applied
to reach the bottom. Hence the correct choices are force F is
(b) and (c). F = mg sin + f
Work, Energy and Power 6.29

frictional force f f 1
Now = = = PE = = cos . The correct choices are
normal reaction R mg cos 2
f = mg cos (b) and (c).
Therefore F = mg sin + mg cos 11. The correct choices are (a), (b) and (c).
= mg(sin + cos )
Displacement along the z-axis is S = 4 k metres.
Work done by the applied force over the upward
Therefore, work done is
journey is
Wa = F L = mg(sin + m cos ) L W1 = F S
The frictional force is non-conservative. Hence the = (– i + 2 j + 3 k ) (4 k )
work done by the frictional force over the round
trip is not zero. = – 4 i k + 8 j k + 12 k k
When the block is at a point A, it is at rest and its
Now i k = 0 and j k = 0 because j and j
initial velocity = 0. It is allowed to slide down the
plane. Let v be the velocity when it reaches the bot- are perpendicular to k . But k k = 1. Therefore,
tom C of the plane. Since the frictional force now W1 = 12 J.
acts upwards, the net force acting on the block when Displacement along the y-axis is S = 3 j metres.
it slides down is Therefore, the work done is
Fn = mg sin – f = mg sin – mg cos
= mg(sin – cos ) W2 = F S = (– i + 2 j + 3 k ) (3 j ) = 6 j j = 6 J
fn Since work is a scalar, the total work done is just
Acceleration of the block, a =
m the algebraic sum of W1 and W2, i.e. W = W1 + W2
= g(sin – cos ). The velocity v is given by = 12 + 6 = 18 J.
v2 – 2 = 2aL (where L = AC) or v2 = 2aL 12. The correct choices are (a),
( = 0) (b) and (c).
Kinetic energy, Work done = area under
1 the F–x graph. Work done
KE = mv2
2 by the force in moving the
1 body from x = 0 to x = 1 m
= m 2AL = maL is (see Fig. 6.33)
2
or KE = mg(sin – cos )L W1 = area of triangle OAD
1
Hence the correct choices are (a), (b) and (d). = AD OD Fig. 6.33
10. The potential energy in the vertical position = work 2
done in raising it from horizontal position to verti- 1
= 5 N 1m = 2.5 J
cal position. In doing so, the mid-point of the rod 2
is raised through a height = /2. Since the entire Hence the correct choice is (a).
mass of the rod can be assumed to be concentrated Work done in moving the body from x = 1 m to
at the mid-point (centre of gravity), the work done x = 3 m is
= = /2. W2 = area of rectangle ABDE
Refer to Fig. 6.32. = AD DE = 5 N 2 m = 10 J,
AD = AB = . In which is choice (c)
the inclined posi- Work done in moving the body from x = 0 to x =
tion, let the centre 4 m is
of gravity C of the W3 = area of (triangle OAD + rectangle ABDE +
rod be at a height triangle BEC)
above the ground,
= 2.5 + 10 + 2.5 = 15 J, which is choice (d).
so that AC = /2.
13. The correct choices are (a), (b) and (d).
In triangle ACE,
Fig. 6.32
Force of friction (f) = mg = 0.2 2 10 = 4N.
we have
Applied force (F) = 10 N. Since friction opposes
= AC sin(90° – ) = cos motion, the net force acting on the body when it is
2 moving is
6.30 Comprehensive Physics—JEE Advanced

F = F – f = 10 – 4 = 6 N dv d
Acceleration a= = (2t – 4) = 2 ms–2
F 6N dt dt
Acceleration a = = = 3 ms–2
M 2 kg Now F = ma = 0.1 kg 2 ms–2
The distance travelled by block in 4s is = 0.2 N
1 2 1 Now distance moved in t = 5 s
S= + at = 0 + 3 (4)2 = 24 m = (5)2 – 4 5 + 4 = 9 m.
2 2
Work done by applied force in 4 s is Work done W = 0.2 N 9 m
W = applied force distance moved in 4 s = 1.8 Nm or J
1
= 10 N 24 m = 240 J 18. If the increase in length is x, W1 = 2
1x and
Work done by the force of friction in 4 s is 2
1 2 W1
W2 = 2x . Therefore, = 1 . Since 1 > 2;
W = f S = 4 N 24 m = 96 J. 2 W2 2
Work done by the net force in 4 s is W1 is greater than W2.
W = F = 6 N 24 m = 144 J Let a force F x1 and the
Velocity acquired by the block in 4 s is x
second by x2. Then F = 1x1 = 2 x2 or 1 = 2 .
v = + at = 0 + 3 4 = 12 ms–1 x2 1
1 2 1 2
Kinetic energy of the block at t = 4 s is Now W3 = x
1 1 and W 4 = x
2 2 . Therefore,
2 2
1 1
KE = mv2 = 2 (12)2 = 144 J 2 2
2 2 W3 x1
= 1 = 1 2
= 2 < 1.
Since the initial KE = 0, the change in KE = 144 J. W4 2 x2 2 1 1
14. All the four choices are correct. In choices (a), Thus the correct choices are (a) and (d).
(b) and (c), the centripetal force (being radial) is 19. The different forces acting on the block are shown
perpendicular to the velocity (and hence displace- in Fig. 6.34. It follows that
ment) vector. In choice (d), the gravitational force R = F cos and R + F sin = mg
(being vertically downwards) is perpendicular to
Eliminating R we get
the displacement.
15. The correct choice is (c). Work done is each half mg
F=
of the journey = where = H/2; H being the cos sin
original height from which the drop fell.
16. The correct choices are (a) and (c).
mv 2
F= = 2 or mv2 = . Therefore,
r r r
1
KE = mv2 =
2 2r
r r r
dr
Now PE = Fdr = 2
dr = Fig. 6.34
r r2
Work done W = Fx
1r Thus the correct choices are (a), (b) and (d).
=– =–
r r
20. WO A = (3 i + 4 j ) (2 i ) = 6 J
Total energy = KE + PE
WA B = (3 i + 4 j ) (2 j ) = 8 J
= – =– WO = 6 + 8 = 14 J
2r r 2r A B
17. The correct choices are (b), (c) and (d). WO = (3 i + 4 j ) (2 i + 2 j ) = 6 + 8 = 14 J
B
x = (t – 2)2 = t2 – 4t + 4
Hence the correct choices are (a), (b) and (c)
dx d 2 d 2U
v= = (t – 4t + 4) = 2 t – 4 21. U is minimum if
dU
= 0 and > 0. Now
dt dt dx dx 2
Work, Energy and Power 6.31

dU d dU mg
= (ax2 – bx) = 2ax – b. So = 0 at x Equations (1) and (2) give T1 = and T2 = 3 T1
dx dx dx 2
3mg
d 2U d = . Thus the correct choices are (b) and (c).
= b/2a. Also 2
= (2ax – b) = 2a, which is 2
dx dx

positive. Hence choice (a) is correct and choice (b)

is wrong. For x = b/2a,


b 2 b b2
Umin = a –b =– .
2a 2a 4a
Force acting on the system is
dU Fig. 6.35
F= – = b – 2ax
dx 24. At any instant during its motion, the energy of the
Hence choice (c) is correct and choice (d) is wrong. particle is partly kinetic and partly potential, but the
mv 2 2 2
t total energy remains constant. If v is the velocity of
22. Centripetal force Fc = = . Since Fc is the particle and x its displacement from the mean
r r
position at an instant of time, then at that instant the
perpendicular to v, = Fc v = 0. Tangential force
total energy is
md v d 1 1 2
Ft = = m ( ) = . Since Ft is parallel to E = K.E. + P.E. = mv2 +
dt dt 2 2
v, = Ft v = = 2t. where m is the mass of the particle. The par-
Hence the correct choices are (b) and (c). ticle will turn back when v = 0 momentarily. At
23. 1 2E 2 1
2
T1 sin 60° = T2 sin 30° (1) that moment E = x= =
2 0.5
and T1 cos 60° + T2 cos 30° = mg (2) = ± 2 m. Hence the correct choices are (b) and (c).

III

Multiple Choice Questions Based on Passage


Questions 1 to 3 are based on the following passage Physicists believe that all laws of physics are invariant
Passage I in all inertial reference frames. For example, the work-
energy principle states that the change in the kinetic en-
Invariance
ergy of a particle is equal to the work done on it by the
Newton’s laws of motion are applicable in all inertial refer- force. Although, work and kinetic energy are not invariant
ence frames. Some physical quantities, when measured by in all reference frames, the work-energy principle remains
observers in different reference frames, have exactly the invariant. Thus even though different observers measuring
same value. Such physical quantities are called -
ant. In Newtonian mechanics mass, time, and force are
invariant quantities. On the other hand, some physical energy principle holds in their respective frames.
quantities, when measured by observers in different refer- 1. Choose the invariant quantities from the following
ence frames, do not have the same value. Such physical (a) mass (b) time
quantities are called not invariant. In Newtonian mechanics (c) velocity (d) displacement
displacement, velocity and work (which is the dot product 2. Which of the following quantities is/are not
of force and displacement) are not invariant. Also kinetic invariant
1 2 (a) Work (b) Kinetic energy
energy mv is not invariant.
2 (c) Torque (d) Displacement
6.32 Comprehensive Physics—JEE Advanced

3. Choose the correct statements form the following. (c) Laws of conservation of energy and momen-
(a) Kinetic energy is not invariant. tum are invariant.
(b) Potential energy is not invariant. (d) All laws of physics are invariant.
ANSWERS
1. The correct choices are (a) and (b). 3. The correct choices are (c) and (d).
2. All choices are correct.

Questions 4 to 8 are based on the following passage 6. A particle of mass m has half the kinetic energy of
Passage II another particle of mass m/2. If the speed of the
heavier particle is increased by 2 ms–1, its new
Mechanical energy exists in two forms: kinetic energy and
kinetic energy equals the original kinetic energy of
potential energy. Kinetic energy is the energy possessed by
the lighter particle. The ratio of the original speeds
a body by virtue of motion. Potential energy is the energy
of the lighter and heavier particles is
These two forms of energy are inter-convertible. If no other (a) 1 : 1 (b) 1 : 2
form of energy is involved in a process, the sum of kinetic (c) 1 : 3 (d) 1 : 4
energy and potential energy always remains constant. 7. In Q.6, what is the original speed of the heavier
4. Two particles of masses m1 and m2 have equal linear particle?
momenta. The ratio of their kinetic energies is
(a) 2(1 + 2 ) ms–1 (b) 2(1 – 2 ) ms–1
m2
(a) 1 (b) (c) (2 2 + 1) ms–1 (d) (2 2 – 1) ms–1
m1
2
8. A uniform rod of mass m and length is made to
m2 m2 stand vertically on one end. The potential energy of
(c) (d)
m1 m1 the rod in this position is
5. Two particles of masses m1 and m2 have equal ki- (a)
mg
(b)
mg
netic energies. The ratio of their linear momenta 4 3
is mg
m1 (c) (d)
(a) 1 (b) 2
m2
2
m1 m1
(c) (d)
m2 m2

SOLUTION
2 Hence the correct choice is (b).
1 1
4. Kinetic energy K = mv2 = (mv)2 = , 7. When the heavier particle is speeded up by
2 2m 2m 1
where = mv is the linear momentum. Thus 2.0 ms–1, its kinetic energy becomes m(v + 2)2.
2
2 2
Since this equals the original kinetic energy of the
K1 = and K2 =
2m1 2m2 lighter particle, we have
K1 m 1 1
= 2 , which is choice (c). m(v + 2)2 = (m/2)(4v2)
K2 m1 2 2
v + 4 + 4v = 2v2 or v2 – 4v – 4 = 0
2
5. K = 2/2m = 2mK . Hence the correct choice
is (b). 4 16 16
v=
6. Let v and v be the original speeds of the heavier and 2
the lighter particles respectively. We then have
4 2 8
1 1 1 m = = 2 2 2
mv2 = v2 2
2 2 2 2
1 The positive root is v = 2+ 2 2 = 2(1 + 2 ).
v2 = v or v = 2v
4 Hence the correct choice is (a).
Work, Energy and Power 6.33

8. The potential energy in the vertical position = work mass of the rod can be assumed to be concentrated
done in raising it from horizontal position to verti- at the mid-point (centre of gravity), the work done
cal position. In doing so, the mid-point of the rod = = /2. Hence the correct choice is (c).
is raised through a height = /2. Since the entire

Questions 9 to 11 are based on the following passage 10. The tension in the rod when it makes an angle
Passage III with the vertical is
(a) Mg cos (b) 2 Mg cos
A light rod of length L having a body of mass M attached (c) 3 Mg cos (d) zero
to its end hangs vertically. It is turned through 90° so that
it is horizontal and then released. 11. The value of when the tension in the rod equals
the weight of the body is given by
9. The centripetal acceleration when the rod makes an 1 1
angle with the vertical is (a) = cos–1 (b) = cos–1
2 3
(a) g cos (b) 2g cos
1 1
(c) g sin (d) 2g sin (c) = sin–1 (d) = sin–1
2 3
SOLUTION
9. The rod is released from the horizontal position OA. v 2 2gL cos
Let OB be the position of the rod when tension in centripetal acceleration = =
L L
the rod is T (Fig. 6.36). = 2g cos ,
which is choice (b).
10. The centripetal force when the body is at B is
M v2
Fc =
L
Thus, we have
M v2
T – Mg cos = (2)
L
Using (1) in (2), we get
Fig. 6.36 M
T – Mg cos 2gL cos = 2 Mg cos
=
Let be the angle with the vertical at this position. L
The loss of PE when the body falls from A to B = or T = 3 Mg cos
Mg OC = MgL cos . If v is the velocity of the Thus the correct choice is (c).
body at B, then 11. T = Mg. Therefore,
1 1
Mv2 = MgL cos or v2 = 2gL cos (1) Mg = 3 Mg cos or cos = , which is choice (b).
2 3

Questions 12 to 14 are based on the following passage 12. The speed of the roller coaster at point B on the
Passage IV track will be
1/ 2
2 2
A small roller coaster starts at point A with a speed on a (a) ( 2 + )1/2 (b)
curved track as shown in Fig. 6.37. The friction between 3
the roller coaster and the track is negligible and it always 3 1/ 2
2 2
remains in contact with the track. (c) ( +2 )1/2 (d)
2
13. The speed of the roller coaster at point C on the
track will be
1/ 2 1/ 2
2 2 2
(a) (b)
3 3
1/ 2
2 4 2
(c) (d) ( +2 )1/2
Fig. 6.37 3
6.34 Comprehensive Physics—JEE Advanced

14. The speed of the roller coaster at point D on the (a) ( 2


+ )1/2 (b) ( 2
+2 )1/2
track will be (c) ( 2
+3 )1/2 (d) ( 2
+4 )1/2
SOLUTION
1 13. Similarly, the speed at point C is given by
12. Total energy at A = KE + PE = 2
+ . If vb 1 1 mg
2 1
2
+ = mv2c + which gives
is the speed at point B, the total energy at B = mv2b 2 2 3
2 1/ 2
+ mg(2 /3). From the principle of conservation of 4
vc = 2
, which is choice (c).
energy, we have 3
1 2 1 2mg 14. At point D, the energy is entirely kinetic. If the
+ = mv2b + speed of the roller coaster at point D is vd, then we
2 2 3
1/ 2
have
2
which gives vb = 2
, 1 1
3 mv2d = + 2
2 2
which is choice (b).
or vd = ( 2
+2 )1/2, which is choice (b).

Questions 15 to 17 are based on the following passage (a) zero (b) 1 m (c) 2 m (d) 3 m
Passage V 16. The acceleration of the particle
The displacement x of a particle moving in one dimension, (a) increases with time
under the action of a constant force is related to time t by (b) decreases with time
the equation (c) increases with time up to t = 3 s and then
t = x +3 decreases with time.
where x is in metre and t is in second. (d) remains constant at 2 ms–2.
15. The displacement of the particle when its velocity 17.
is zero is (a) 1 J (b) 3 J (c) 6 J (d) zero
SOLUTION
17. From Eq. (2), the initial velocity, i.e., velocity at
15. Given t = x + 3 or x = t – 3 or
t = 0 is
x = (t – 3)2 (1)
Differentiating (1) with respect to t, we get v0 = 2(0 – 3) = – 6 ms–1
dx Final velocity, i.e., velocity at t = 6 s is
= 2 (t – 3)
dt v = 2(6 – 3) = 6 ms–1
or v = 2 (t – 3) (2)
From (2) it follows that v = 0 at t = 3 s. Using t =
3 s in (1), we get x = 0. Thus, the displacement of 1 1 1
= mv2 – mv20 = m(v2 – v02 )
the particle is zero when its velocity is zero. Thus 2 2 2
the correct choice is (a).
16. From Eq. (2), we have 1
= m[(6)2 – (– 6)2] = 0, which is choice (d).
dv d 2
a = = [2(t – 3)] = 2 ms–2.
dt dt
Hence the correct choice is (d).

Questions 18 to 20 are based on the following passage position vector and r2


Passage VI If the force is variable, the work done in moving a body
The work done by a constant force acting on a body is from a position r1 to a position r2 is given by
given by r2

W=F r W = F dr
where F is the force vector and r is displacement vector. r1

The displacement vector r = r2 – r1 where r1 is the initial where dr


Work, Energy and Power 6.35

20. A body of mass m is projected from the ground


18. A particle is moved from a position r1 = (3 i + 2 j with a velocity at an angle above the horizontal.
– 4 k ) metre to a position r2 = (5 i + 6 j + 9 k ) The work done by the gravitational force in time
sin
t= is
metre under the action of a force F = ( i + 3 j + k ) g
newton. The work done is 2
(a) 2 sin2 (b) 2
sin2
(a) zero (b) 13 J 2
sin 2
(c) 27 J (d) 35 J (c) (d) zero
2
19. A body of mass m is projected from a tower of
height at an angle above the horizontal. The 21. If = 45°, the work by the gravitational force in
work done by the gravitational force during the 2 sin
Q.20 above in time t = is
time it takes to hit the ground is g
2 2
(a) (1 + cos ) (b) (1 + sin ) (a) (b)
(c) (d) zero 4 2
2
(c) (d) zero
SOLUTION
18. W = F r = F (r2 – r1) Work done W = , which is choice (c).
= ( i + 3 j + k ) [(5 i + 6 j + 9 k ) – (3 i + 2 j 20. t =
sin
g
– 4 k )] 2
sin 2
the body attains maximum height max = .
= ( i + 3 j + k ) (2 i + 4 j + 13 k ) 2g
2
= 2 + 12 + 13 = 27 J, which is choice (c) sin 2
Work done = max = which is
19. Net displacement from the time the body is pro- choice (c). 2
jected to the time it hits the ground is = vertically 21. Net displacement = 0. Hence W = 0. Thus the
downwards. correct choice is (d).

IV

Matching
1. Column I Column II
(a) Force (p) Slope of force-extension graph
(b) Impulse (q) Area under force-time graph
(c) Energy stored in a spring (r) Area under force-extension graph
(d) Force constant of a spring (s) Slope of linear momentum-time graph

ANSWER
(a) (s) (b) (q)
(c) (r) (d) (p)
6.36 Comprehensive Physics—JEE Advanced

Assertion-Reason Type Questions


In the following questions, Statement-1 (Assertion) is 4. Statement-1
followed by Statement-2 (Reason). Each questions has A man carrying a bucket of water and walking on
the following four choices out of which only one choice a rough level road with a uniform velocity does no
is correct. work while carrying the bucket.
(a) Statement-1 is true, Statement-2 is true and State- Statement-2
ment-2 is the correct explanation for Statement-1. The work done on a body by a force F in giving it
(b) Statement-1 is true, Statement-2 is true but a displacement S
Statement-2 is the correct explanation for W = F S = FS cos
Statement-1.
(c) Statement-1 is true, Statement-2 is false. where is the angle between vectors F and S.
5. Statement-1
(d) Statement-1 is false, Statement-2 is true.
A crane lifts a car up to a certain height in 1 min.
1. Statement-1
Another crane Q lifts the same car up to the same
A simple pendulum of length is displayed from
height in 2 min. Then crane consumes two times
its mean position O to position A so that the string
more fuel than crane Q.
makes an angle 1 with the vertical and then re-
Statement-2
leased. If air resistances is neglected, the speed of
Crane supplies two times more power than crane Q.
the bob when the string makes an angle 2 with the
6. Statement-1
vertical is v = 2 (cos 2 cos 1 ) . Two inclined frictionless tracks of different inclina-
Statement-2 tions 1 and 2 meet at A from where two blocks
The total momentum of a system is conserved if no and Q of different masses m1 and m2 are allowed to
external force acts on it. slide down from rest, one on each track as shown in
2. Statement-1 Fig. 6.38. Then blocks and Q will reach the bot-
A uniform rod of mass m and length is held at an tom with the same speed.
angle with the vertical. The potential energy of
1
the rod in this position is cos .
2
Statement-2
The entire mass of the rod can be assumed to be
concentrated at its centre of mass.
3. Statement-1
A block of mass m starts moving on a rough hori-
zontal surface with a velocity v. It stops due to Fig. 6.38
friction between the block and the surface after Statement-2
moving through a certain distance. The surface Blocks and Q have equal accelerations down
is now tilted to an angle of 30° with the hori- their respective tracks.
zontal and the same block is made to go up on 7. Statement-1
the surface with the same initial velocity v. The In Q.6 above, block will take a longer time to
decrease in the mechanical energy in the second reach the bottom than block Q.
situation is smaller than that in the first situa- Statement-2
tion. Block Q has a greater acceleration down the track
Statement-2 than block .
8. Statement-1
the surface decreases with the increase in the angle Comets move around the sun in highly elliptical
of inclination. orbits. The work done by the gravitational force of
IIT, 2007 the sun on a comet over a complete orbit is zero.
Work, Energy and Power 6.37

Statement-2 Statement-2
The gravitational force is conservative. The internal forces can change the kinetic energy of
9. Statement-1 the system of particles but not the linear momentum
The total energy of a system is always conserved of the system.
irrespective of whether external forces act on the 11. Statement-1
system. An elastic spring of force constant is stretched by
Statement-2 a small length The work done in extending the
If external forces act on a system, the total momen- spring by a further length x is 2 2.
tum and energy will increase. Statement-2
10. Statement-1 The work done in extending an elastic spring by a
The rate of change of the total linear momentum length x is proportional to x2.
of a system consisting of many particles is propor-
tional to the vector sum of all the internal forces
due to inter-particle interactions.
SOLUTIONS
1. The correct choice is (b). It is clear from Fig. 6.39 to raise the rod from horizontal position on the
that = cos 1 and = cos 2. Therefore, 1 = ground to the position show =
– cos 1 = (1 – cos 1) and 2 = (1 – cos 2). Let 1
cos .
m be the mass of the bob and v be its speed when 2
it reaches position B. Then, from the principle of 3. Statement-1 is true. The decrease in mechanical
conservation of energy, K.E. at B = loss of P.E. as energy is smaller when the block made to go up on
the bob moves from A to B. the inclined surface because some part of the kinetic
Hence energy is converted into gravitational potential
1
mv2 = 1– 2 friction does not depend on the angle of inclination
2
= mg[ (1 – cos 1) – (1 – cos 2)] of the plane. Hence the correct choice is (c).
4. The correct choice is (a). Since the velocity is uni-
= (cos 2 – cos 1)
form, the man exerts no net force on the bucket in
v= 2 (cos 2 cos 1 ) the direction of motion. The only force he exerts
on the bucket is against gravity (to overcome) the
weight mg of the bucket) and this force is perpen-
dicular to the displacement (i.e. = 90°).
Hence W = FS cos 90° = 0.
5. The two cranes do the same amount of work = .
Hence they consume the same amount of fuel. Crane
does the same amount of work in half the time.
Hence crane supplies two times more power than
crane Q. Thus the correct choice is (d).
6. The acceleration of blocks and Q respectively are
m g sin 1
Fig. 6.39 a1 = 1 = g sin 1
m1
2. The correct choice is (a). Let C be the centre of
mass of the rod AB so that AC = /2. Let be the m2 g sin 2
and a2 = = g sin 2
height of C above the ground. In triangle ACD, we m2
have CD = AC sin (90° – ) Since 2 > 1; a2 > a1. The potential energy of block
(see Fig. 6.40). at A = m1 . When it reaches the bottom B, its
Or = cos . Since the en- 1
2 kinetic energy is m1v21 where v1is its speed when
2
tire mass of the rod can be as- it reaches B. Now P.E. at A = K.E. at B. Hence
sumed to be concentrated at
1
the centre of mass, therefore, m1 = m1v21 v1 = 2 .
potential energy = work done Fig. 6.40 2
6.38 Comprehensive Physics—JEE Advanced

1 9. Statement-1 is false; the total energy of an isolated


Similarly m2 = m2v22 v2 = 2 = v 1. system is conserved. Statement-2 is true. Hence the
2
correct choice is (d).
Hence the correct choice is (c).
10. Statement-1 is false and Statement-2 is true. The
7. The correct choice is (a). If t1 and t2 are the times
rate of change of momentum is proportional to the
taken by and Q to reach the bottom, then
net external force acting on the system. Hence the
v1 = 1 + a 1t 1 = a 1t 1 ( 1 = 0) correct choice is (d).
and v2 = 2 + a 2t 2 = a 2t 2 ( 2 = 0) 11. The correct choice is (d). Potential energy stored in
Now v1 = v2. Hence a1t1 = a2t2. Thus the spring when it is extended by x is U1 =
1 2
t1 a 2
= 2 Potential energy stored in the spring when it is
t2 a1 further extended by x is
Since a2 > a1; t1 > t2. 1
8. The correct choice is (a). For a conservation force, U2 = (x + x)2 = 2 2
2
the work done in moving a body from one point to
another does not depend on the nature of the path Work done = gain in potential energy = U2 – U1
and the work done over a closed path is zero, ir- 1 2 3 2
=2 2– =
respective of the nature of the path. 2 2

VI

Integer Answer Type


1. A particle of mass 1g executes an oscillatory mo-
tion on the concave surface of a spherical dish of ra- system is released from rest.
dius 2 m placed on a horizontal plane. If the motion IIT, 2009
of the particle begins from a point on the dish at a 3. A block of mass 0.18 kg is attached to a spring of
height of 2 cm from the horizontal plane and the

in metre covered by the particle before it comes to block is at rest and the spring is unstretched. An
rest. impulse is given to the block as shown in Fig. 6.42.
IIT, 1990 The block slides a
2. A light inextensible string that goes distance of 0.06 m
and comes to rest for
in Fig. 6.41 connects two blocks of Fig. 6.42
masses 0.36 kg and 0.72 kg. Taking initial velocity of the
g = 10 m/s2 in block in m/s is V = N/10. Then N is
joules) by the string on the block IIT, 2011
Fig. 6.41
SOLUTION

1. Refer to Fig. cal. The particle will keep on oscillating about O


6.43 in which till its initial potential energy is completely
is the angle used up in doing work against the frictional force f
which the nor- and the particle comes to rest. It follows from the
mal to the sur-
face at the lo- f= mg cos
cation of the
Since << R, 0. Therefore, cos 1. Hence
particle makes
with the verti- Fig. 6.43 f = mg
Work, Energy and Power 6.39

If is the distance travelled by the particle before it 3. Refer to Fig. 6.45.


comes to rest, then the work done against friction is
W= =
Now, =
2 cm
or = = = 200 cm = 2m
0.01
2. Refer to Fig. 6.44.
T – m1 g = m1 a
m2 g – T = m2 a
which give
(m2 m1 ) g Fig. 6.45
a=
(m1 m2 )
1
(0.72 0.36) 10 Loss of kinetic energy = mV 2
= 2
(0.72 0.36) Work done against friction = mgx
10 1 2
= m s–2 Gain in potential energy =
3 2
and T = m1(a + g) Fig. 6.44 From work-energy principle,
10 1 1 2
= 0.36 10 = 4.8 N mV 2 = mgx +
3 2 2
Distance moved in t = 1 s is 1
0.18 V2 = 0.1 0.18 10 0.06
1 1 10 2
= + at2 = 0 + (1)2 1
2 2 3 + 2 (0.06)2
5 2
= m 4
3 V = 0.4 = ms–1. Hence N = 4.
5 10
Work done = T S = 4.8 =8J
3
7
Chapter
Conservation of Linear
Momentum and Collisions

REVIEW OF BASIC CONCEPTS 7.2 RECOIL OF A GUN


The gun and the bullet constitute a two-body system.
LAW OF CONSERVATION OF LINEAR
7.1 MOMENTUM at rest. Therefore, the total momentum of the gun-bullet
The law of conservation of linear momentum may be
forward and the gun recoils backwards. Let mb and mg be
stated as ‘when no net external force acts on a system
the masses of the bullet and the gun. If vb and vg are their
consisting of several particles, the total linear momentum
of the system is conserved, the total linear momentum
mb vb + mg vg). From
being the vector sum of the linear momentum of each
the law of conservation of momentum, the total momen-
particle in the system’.
e the same,
According to Newton’s second law of motion, we have
i.e.
for a system of particles,
mb vb + mg vg = 0
dp
Fext = m v
dt or vg = – b b
where p = p1 + p2 + p3 + mg
The negative sign indicates that the gun recoils in a
= m1 v1 + m2 v2 + m3 v3 +
direction opposite to that of the bullet. In terms of mag-
In a system consisting of many particles, there can be nitudes, we have
two kinds of forces: (i) internal forces and (ii) external mv
forces. Internal forces are the forces that the particles ex- vg = b b
ert on each other during their interactions (e.g. collisions). mg
From Newton’s third law, these forces always occur in 7.3 COLLISIONS
pairs of action and reaction. Since these forces are equal
and opposite, they bring about equal and opposite changes Elastic Collisions: If there is no change of kinetic energy
in the momentum of the particles. Thus, internal forces during a collision it is called an elastic collision. The
cannot bring about any net change in the momentum of collision between subatomic particles is generally elastic.
a particle. The external forces, on the other hand, are the The collision between two steel or glass balls is nearly
forces exerted from outside the system. elastic.
Inelastic Collisions: If there is a loss of kinetic energy
If Fext = 0, we have
during a collision, it is called an inelastic collision. Since
dp there is always some loss of kinetic energy in any colli-
=0
dt sion, collisions are generally inelastic. If the loss is negli-
or p = constant gibly small, the collision is very nearly elastic. Perfectly
elastic collisions are not possible. If two bodies stick
Thus, the vector sum p = p1 + p2 + p3 + of the lin-
together, after colliding, the collision is perfectly inelastic,
ear momenta of the particles remains constant if the net
e.g. a bullet striking a block of wood and being embedded
external force is zero.
7.2 Comprehensive Physics—JEE Advanced

in it. The loss of kinetic energy usually results in heat or One-dimensional or Head-on Collision
sound energy.
In may be remembered that the total momentum re- Consider two bodies of masses m1 and m2 moving with
mains conserved in both elastic and inelastic collisions. velocities u1 and u2 in the same straight line (with u1 > u2)
Further, since the interacting forces become effectively colliding with each other. Let v1 and v2 be their respective
zero after the collision, the potential energy remains the velocities after the collision. If velocities u1, u2, v1 and v2
same both before and after the collision. are all along the same straight line, the collision is known
as one-dimensional or head-on collision (Fig. 7.1)

v v

Fig. 7.1

From the law of conservation of momentum Coefficient of Restitution


m 1 u 1 + m 2 u 2 = m 1v 1 + m 2 v 2
Newton proved experimentally that, when two bodies
If momentum along positive x-axis is taken to be positive, collide, the ratio of the relative velocity after collision
the momentum along the negative x-axis is taken to be to the relative velocity before collision is constant for
negative. the two bodies. This constant is known as
restitution and is denoted by letter e.
Two-dimensional or Oblique Collision
velocity of separation after collision
e =–
If the velocities of the colliding bodies are not along the velocity of approach before collision
same straight line, the collision is known as two-dimen-
v 2 v1
sional or oblique collision (Fig. 7.2) or e =–
u 2 u1
v
(i) For a perfectly elastice collision, e = 1.
(ii) For a perfectly in inelastic collision, e = 0, because
the two bodies stick together and hence v2 = v1.
(iii) Perfectly elastic or perfectly inelastic collisions do
not occur in nature. Hence, for any collision, e lies
between 0 and 1.
(iv) For a head-on collision (Fig. 7.1)
velocity of separation
v e =–
velocity of approach
v2 v1
=
u2 u1
Fig. 7.2
(v) For an oblique collision (Fig. 7.2)
In this case, we apply the law of conservation of mome- Velocity of approach = u1 cos 1 – u2 cos 2
ntum separately for x and y components of momenta. The Velocity of separation = v2 cos 2 – v1 cos 1
components of momentum along the positive x-axis and v cos 2 v1 cos 1
positive y-axis are taken to be positive and components of e=– 2
momentum along negative x-axis and negative y-axis are u1 cos 1 u2 cos 2
taken to be negative. Velocities after Head-on Elastic Collision
Momentum conservation of x-components gives
m1u1 cos 1 + m2u2 cos 2 = m1v1 cos 1 + m2v2 cos 2 Refer to Fig. 7.1 again. From the law of conservation of
Momentum conservation of y-components gives momentum, we have
– m1u1 sin 1 + m2u2 sin 2 = m1v1 sin 1 – m2v2 sin 2 m 1u 1 + m 2u 2 = m 1v 1 + m 2v 2 (1)
Conservation of Linear Momentum and Collisions 7.3

v1 = – u1
v v2 and v2 0
e= 1
u2 u1
Thus, if a very light body suffers an elastic colli-
v1 – v2= e (u2 – u1) (2) sion with a very heavy body at rest, the velocity of
Eliminating v2 from (1) and (2), we get the lighter body is reversed on collision, while the
m1 em2 m2 1 e heavier body remains practically at rest.
v1 = u1 u2 (3) A common example of this type of collision is the
m1 m2 m1 m2 dropping of a hard steel ball on a hard concrete
Using (3) in (2), we get
m1 1 e m2 em1 height from where it was dropped while the much
v2 = u1 u2 (4) more massive ground remains at rest.
m1 m2 m1 m2
Finally, if the body at rest is much lighter than the
Perfectly Elastic Collision colliding body, i.e. if m2 << m1, we have
v1 u1 v2 2u1
For perfectly elastic collision, e = 1. Putting e = 1 in Eqs.
(3) and (4) we get i.e. the velocity of the massive body remains practi-
cally unchanged on collision with the lighter body
m1 m2 2m2
v1 = u1 u2 (5) at rest and the lighter body acquires nearly twice
m1 m2 m1 m2 the initial velocity of the massive body.
2m1 m2 m1 (iii) Kinetic energy delivered by incident body to a stati-
and v2 = u1 u2 (6) onary body in perfectly elastic head-on collision.
m1 m2 m1 m2
1
Kinetic energy of m1 before collision is Ki = m1u12
Special Cases 2
1
(i) If both bodies have the same mass, then and after collision is Kf = m1v12 . Therefore
m1 = m2 = m 2
Kf v12
In this case, = 2
Ki u1
v1 = u2
and v2 = u1
Ki Kf v12
or = 1
Ki u12
This means that in a one-dimensional elastic colli-
sion between two bodies of equal mass, the bodies The fractional decrease in kinetic energy of m1 is
merely exchange their velocities after the collision. K v2
(ii) If one of the bodies, say m2, is initially at rest, = 1 12
Ki u1
then
u2 = 0 v1 m m2
If u2 = 0, = 1 . Therefore,
u1 m1 m2
In this case,
2
m1 m2 K m1 m2 4m1m2
v1 = u1 =1 = 2
m1 m2 Ki m1 m2 m1 m2
2m1u1 NOTE
and v2 =
m1 m2
The fraction of kinetic energy lost by mass m1 is maximum
If, in addition, m1 = m2 = m, these equations give if m1 = m2 and minimum if m2 .
v 1= 0 v 2 = u 1 (iv) Change in kinetic energy of a system in a perfectly
Thus, if a body suffers a one-dimensional elastic inelastic head-on collision.
collision with another body of the same mass at In a perfectly inelastic collision, the two bodies stick
rest, together after the collision. Hence v1 = v2 and e = 0.
. Putting e = 0 in Eqs. (3) and (4), we get
However, if the body at rest, namely B, is much
more massive than the colliding body A, i.e. m2 >> m1 m2
v1 = u1 u2
m1, such that m1 is negligibly small, then m1 m2 m1 m2
7.4 Comprehensive Physics—JEE Advanced

m1 m2 velocity of separation
and v2 = u1 u2 e=
m1 m2 m1 m2 velocity of approach
If mass m2 is stationary, u2 = 0. Then v cos j v cos
m1 e= =
v1 = u1 (7) u cos j u cos
m1 m2
vcos = eu cos (10)
m1 Since the impulsive force acts along the normal,
and v2 = u1
m1 m2 the momentum along the normal is not conservved.
Notice that v1 = v2 = v (say) Since the component of the impulsive force along
Total K.E. of the system before collision is the horizontal is zero, the momentum along the
horizontal is conserved. Hence
1
Ki = m1u12 u sin = v sin (11)
2
and after the collision is From Eqs. (10) and (11), we get
1 v = (e2 cos2 + sin2 )1/2 u (12)
Kf = m1 m2 v 2
2
tan
Loss in K.E. of the system is and tan = (13)
e
1 1
Ki – Kf = m1u12 m1 m2 v 2 (8) For a perfectly elastic collision, e =1 and Eqs. (12)
2 2 and (13) give
v m1 v =u
From Eq. (7) = . Using this in
u1 m1 m2 and =
Eq. (8) we get
i.e. for a perfectly elastic collision, the body re-
m1m2u12
Ki – Kf =
2 m1 m2 and at the same angle on the other side of the nor-
mal.
In general, if u2 0, we have
m1m2 2 Direct Impact on a Fixed Plane
Ki – Kf = u1 u2 (9)
2 m1 m2
Oblique Impact on a Fixed Horizontal Plane after impact, then, in this case = = 0. Using this in Eq.
(10) we get [Fig. 7.4]
Consider a body of mass m moving with a velocity u v = eu
making an angle with the normal ON
v

v making an angle with the normal. Since the horizontal

place along the normal. The normal component of u is


u cos along – y direction and the normal component of v
is v cos along the +y directon. Now

Fig. 7.4
v
7.1
A body is dropped from rest from a height h = 5.0 m.

is 0.8?

Fig. 7.3
Conservation of Linear Momentum and Collisions 7.5

SOLUTION
Refer to Fig. 7.5.
= 2gh v
S e 2 gh . This is also
the speed just before the second impact. Therefore,
speed just after second impact = e2 2 gh . This is
the initial speed for the upward motion of the body v
after the second impact, i.e. u = e2 2 gh . Therefore,
height attained after two impacts is
Fig. 7.5
u2 1 2 2
h2 = = e 2 gh = e 4h From conservation of momentum,
2g 2g
mu = mv1 + mv2
= (0.8)4 5 = 2.05 m
u = v1 + v2 (i)
NOTES Taking the scalar product of u with itself, we have
u u = (v1 + v2) (v1 + v2)
(1) Height attained after n impacts is hn = e2nh
(2) Speed of rebound after nth impact is vn = e n 2 gh u2 = v12 2 v1 v 2 v22 (ii)
(3) Total distance travelled before the body comes to rest
Since kinetic energy is also conserved in an elastic
2
1 e collision, we have
= h .
1 e2 1 2 1
mu = mv12
1 2
mv2
2 2 2
7.2
A steel ball of mass m moving with velocity u1 under- u2 = v12 + v22 (iii)
goes a perfectly elastic head-on collision with another Using (iii) in (ii), we get
identical steel ball moving with velocity u2. Show 2 v1 v2 = 0
that, after the collision, they merely exchange their
velocities. v1 v2 = 0
v1 v2 cos =0

Refer to Fig. 7.1 again. From conservation of cos =0 = 90°


momentum,
7.4
mu1 + mu2 = mv1 + mv2
Two steel balls of the same mass m moving in oppo-
u1 + u2 = v1 + v2 (i) site directions with the same speed u collide head-on.
If the collision is perfectly elastic, predict the result
v2 – v1 = e (u1 – u2) of the collision.
For a perfectly elastic collision, e = 1. Hence
v2 – v1 = u1 – u2 (ii)
m1 = m2 = m, u1 = u and u2 = – u. Let v1 and v2 be their
From (i) and (ii) we get velocities after collision.
v1 = u2 and v2 = u1 Total momentum before collision = m1u1 + m2u2
= m (u – u) = 0
7.3 Total momentum after collision = mv1 + mv2
A steel ball of mass m moving with a velocity u = m (v1 + v2)
undergoes a perfectly elastic oblique collision with From conservation of momentum,
another indentical steel ball initially at rest. Show 0 = m(v1 + v2) v2 = – v1
that, after the collision, they move at right angles to
each other.
7.6 Comprehensive Physics—JEE Advanced

Since e = 1, we have
7.6
v2 – v1 = u1 – u2 = u – (– u) = 2u
In Example 7.5, what is the loss of kinetic energy if
Putting v1 = – v2, we get v2 = u. Also v1 = – u. Thus, the ball of mass 4 kg is moving towards the mass of
after the collision, the two balls move in opposite mass 2 kg, their speeds being the same?
directions with equal speeds, each equal to u but their
directions are reversed.

7.5 In this case u2 = – 2 ms–1. Equations (i) and (ii)


A ball of mass 2 kg moving with a velocity of 8 ms–1 become
collides head-on with another ball of mass of 4 kg 4 = v1 + 2v2 (iii)
moving with a velocity of 2 ms–1 moving in the same and v2 – v1 = 5 (iv)

restitution is e = 0.5. Equations (iii) and (iv) give v1 = – 2 ms–1 and


(a) Find the velocities of the balls after the v2 = 3 ms–1
collision. 1 1
Ki = 2 (8)2 + 4 (–2)2 = 72 J
(b) Calculate the loss of kinetic energy due to 2 2
collision. 1 1
Kf = 2 (–2)2 (3)2 = 22J
2 2
Refer to Fig 7.1 again. Loss of K.E = 72 – 22 = 50 J

(a) Given m1 = 2 kg, m2 = 4 kg, u1 = 8 ms–1, u2 = 7.7


2 ms–1 and e = 0.5
Two blocks B and C of masses 1 kg and 2 kg respec-
From conservation of momentum tively are connected by a massless elastic spring of
m 1u 1 + m 2u 2 = m 1v 1 + m 2v 2 spring constant 150 Nm–1 and placed on a horizontal
2 8+4 2 = 2v1 + 4v2 frictionless surface as shown in Fig. 7.6. A third block
A of mass 1 kg moves with a velocity of 3 ms–1 along
12 = v1 + 2v2 (i) the line joining B and C and collides with B. If the
Since e = 0.5, we have collision is perfectly elastic and the natural length
v2 – v1 = e (u1 – u2)
between blocks B and C.
= 0.5 (8 – 2) = 3 (ii)
Eliminating v2 from (i) and (ii) we get v1 = 2 ms–1.
Using this in (i) or (ii), we get v2 = 5 ms–1
(b) Kinetic energy before collision is
1 1 Fig. 7.6
Ki = m 1u 12 + m 2u 22
2 2
1 1 Given mA = mB = 1 kg, mC = 2 kg and u = 3 ms–1.
= 2 (8)2 + 4 (2)2
2 2 Block A will collide with block B. Since they have
= 72 J equal masses and the collision is perfectly elastic, A
will come to rest and B will move to the right with a
Kinetic energy after collision is velocity u. Block B will compress the spring. Hence
1 1 block C will accelarate and block B will retard until
Kf = m 1v 12 + m 2v 22 both B and C move with the same velocity. Let this
2 2 common velocity be v. Since no external force acts,
1 1 the momentum of B and C is conserved, i.e.
= 2 (2)2 + 4 (5)2 = 54 J
2 2 mB u = (mB + mC) v
Loss of K.E. = Ki – Kf = 72 – 54 = 18 J 1 3 = (1 + 2) v v = 1 ms–1
Conservation of Linear Momentum and Collisions 7.7

If x is the maximum compression, then from the


3 u 3m
principle of conservation of energy, mu2 = kx2 x=
1 1 1 4 2 k
mAu2 = (mB + mC) v2 + kx2
2 2 2 7. Useful Formulae and Tips
1 1
1 (3)2 = (1 + 2) (1)2 1. A body of mass m is dropped from a height h. Due
2 2 to the friction of air, it will hit the ground with a
1
+ 150 x2 speed less than 2gh . If v is the speed with which
which gives x = 0.2 m = 20 cm 2
it hits the ground, the work done by friction is
Minimum separation between B and C = 80 cm 1 1
– 20 cm = 60 cm Wf = mv2 – mgh = m(v2–2gh)
2 2
7.8 If friction is absent, Wf = 0, then v = 2gh .
A block of m1= m is moving on a frictionless horizon- 2. Two block A and B of masses m1 and m2 are re-
tal surface with velocity u1 = 2u towards another block leased from the same height at the same time. Block
of mass m2 = 3m moving on the same surface with A slides along an inclined plane of inclination and
velocity u2 = u in the same direction. A massless spring block B falls vertically downwards (Fig. 7.8)
of force constant k is attached to m2 as shown in Fig. If the inclined plane is frictionless, gain in KE =
7.7. When block m1 collides with the spring, show loss in PE, i.e.
that the maximum compression of the spring is given 1
m1v21 = m1gh v1 = 2gh
u 3m 2
by x = . If the air friction is neglected.
2 k
1
m2v22 = m2gh v2 = 2gh
2
Thus both block will hit the ground with the same
speed independent of the mass. But the times taken
Fig. 7.7 to reach the ground will be different.

SOLUTION
When block m1 collides with spring, it begins to get
compressed. As a result m2 gains speed. The compres-
sion of the spring is maximum at the instant when the
relative velocity of m1 with respect to m2 is zero, i.e.
when both m1 and m2 have equal velocities. Let v be
the common velocity of the blocks. From conserva-
tion of momentum,
v
v
m1u1 + m2 u2 = (m1 + m2)v
2mu + 3mu = (m + 3m)v Fig. 7.8

5u 2h
v=
4 For block A, t1 =
g sin 2
From the law of conservation of energy
2h
Loss in K.E. = gain in P.E of spring For block B, t2 =
If x is the maximum compression, then g
3. If a block of mass m in contact with a spring com-
1 1 1 1 2 pressed by a distance x is released, the block will
m1u12 m2u22 – m1 m2 v 2 = kx
2 2 2 2 leave the spring with a velocity v determined from
1 1 1 1 1
m (2u)2 + 3m u2 – (m + 3m) kx2 = mv2
2 2 2 2 2
2
5u 1 k
= kx2 which gives v = x, where k is the spring
4 2 constant. m
7.8 Comprehensive Physics—JEE Advanced

4. If a block of mass m moving with speed u comes in 7. A chain has a length L and mass M. A part L/n is
contact with a relaxed spring of spring constant k, hanging at the edge of the table. The length of the
its velocity v when the spring is compressed by an chain lying on the table is (L – L/n). Then work
amount x is obtained from. done against gravity to pull the hanging part on the
1 1 1 MgL
mu2 = mv2 + kx2 table =
2 2 2 2n 2
1/ 2 8. If a body of mass m moving with velocity v is
k x2 2 stopped in a distance x by a retarding force F, then
which gives v = u
m 1
mv2 = Fx
5. If two springs of spring constants k1 and k2 are 2
stretched by the same force F, then F = k1x1 (a) If two bodies of masses m1 and m2 moving with the
= k2 x2. Potential energy stored is same velocity are subjected to the same retarding
1 1 force, the ratio of the stopping distance is
U1 = k 1x 21 = Fx1 x1
2 2 m1
=
1 1 x2 m2
and U2 = k 2x 22 = Fx2 (b) If the two bodies are moving with equal kinetic
2 2
energy and are stopped by the same retarding force,
U1 x k2
which give = 1 then
U2 x2 k1 x1 = x2
6. If the two springs are stretched by the same amount (c) If the two bodies are moving with equal liner mo-
x, then F1 = k1x and F2 = k2x. mentum and are stopped by the same force, then.
1 1 p2
U1 = k1x2 and U2 = k 2x 2. = Fx
2 2 2m
U1 k F1 x1 m2
= 1 and =
U2 k2 F2 x2 m1

Multiple Choice Questions with only One Choice Correct


1. A ball P moving with a velocity u suffers a one- 3 5
(c) (d)
dimensional collision with another ball Q of the 8 8
same mass but at rest. After the collision the velo- 3. Two particles of the same mass m moving in diff-
city of Q is found to be three times that of P. The erent directions with the same speed v collide and
stick together. After the collision, the speed of the
1 1 composite particle is v/2. The angle between the
(a) (b)
2 3 velocities of the two particles before collision is
1 2 (a) 60° (b) 90°
(c) (d) (c) 120° (d) 150°
4 3
4. Two blocks of masses m1 = m and m2 = 3 m are
2. = 45°.
connected by a spring of force constant k and placed
on a horizontal frictionless surface as shown in
Fig. 7.9. The spring is stretched by an amount x
the collision is
and released. The system executes simple harmonic
1 1
(a) (b) motion. The relative velocity of the blocks when
2 2 2 the spring is at its natural length is
Conservation of Linear Momentum and Collisions 7.9

elastic collision with a body of mass 2 m at rest.


3k k
(a) x (b) 2 x After the collision, the ratio of the kinetic energy
2m m of P to that of Q is
x k k 1 1
(c) (d) 2 x (a) (b)
2 3m 3m 8 4
1 8
(c) (d)
9 9
10. A shell of mass m initially at rest explodes into
three fragments of masses in the ratio 2 : 2 : 1.
Fig. 7.9
mutually perpendicular directions with speed v.
5. A block P moves with an initial velocity of 4 ms–1 The speed of the third (lighter) fragment will be
towards a block Q of the same mass at rest at a dis- (a) v (b) 2v
tance of 2 m on a rough horizontal surface. The co-
(c) 2 2 v (d) 3 2 v
is 0.2. An elastic one-dimensional collision occurs 11. A shell is fired with a speed of 100 ms –1 at an
between the two blocks. If g = 10 ms–2, the separa- angle of 30° with the vertical (y-direction). At the
tion between the blocks when both have come to highest point of its trajectory, the shell explodes
rest is into two fragments of masses in the ratio 1 : 2.
(a) 1 m (b) 2 m
The lighter fragment moves vertically upwards
(c) 3 m (d) 4 m
with an initial speed of 200 ms–1. The speed of the
6. A perfectly elastic oblique collision occurs between
heavier fragment immediately after the explosion
a ball A moving along the x-axis and a ball B at rest
is
and of the same mass as ball A. After the collision,
ball A moves at an angle of 30° with the x-direction (a) 125 ms–1 (b) 150 ms–1
and ball B at an angle with the x-axis. The value (c) 175 ms–1 (d) 200 ms–1
of is 12. A body of mass m moving with a certain speed
(a) 15° (b) 30° suffers an inelastic collision with a body of mass
(c) 45° (d) 60° M
7. A particle A moving with momentum p suffers a system to the initial kinetic energy is
one-dimensional collision with a particle B of the M
m
same mass but at rest. During the collision B im- (a) (b) m M
parts an impulse I to A - m M
tion between A and B is m M m M
(c) (d)
2I 2I m M
(a) (b) –1
p p 13. A neutron moving at a speed v undergoes a head-on
elastic collision with a nucleus of mass number A at
2I I
(c) +1 (d) 2 1 rest. The ratio of the kinetic energies of the neutron
p p after and before collision is
8. A man of mass m stands on one end of a wooden A 1 2 2
A 1
plank of length L and mass M kept initially at rest (a) (b)
on a horizontal frictional surface. If the man walks A 1 A 1
from one end of the plank to the other end at a con- 2 2
A A
stant speed, the resulting displacement of the plank (c) (d)
A 1 A 1
is
14. A radioactive nucleus of mass number A, initially at
mL ML
(a) (b) rest, emits an -particle with a speed v. What will
M m be the recoil speed of the daughter nucleus?
mL mL 2v 2v
(c) (d) (a) (b)
( M m) ( M m) A 4 A 4
9. A body P of mass m moving with kinetic energy 4v 4v
(c) (d)
k and momentum p undergoes a one-dimensional A 4 A 4
7.10 Comprehensive Physics—JEE Advanced

15. A ball is dropped from a height of 10 m. It is 21. A nucleus of mass M amu emits an -particles
embedded 1 m in sand. In this process with a energy K MeV. The total energy of disinte-
(a) only momentum is conserved gration (in MeV) is
(b) only kinetic energy is conserved KM
(c) both momentum and kinetic energy are con- (a) K (b)
M 4
served
(d) neither momentum nor kinetic energy is con- K 1 M KM
(c) (d)
served. M M 4
16. n small balls, each of mass m, impinge elastically 22. A particle falls from a height h
each second on a surface with velocity u. The force plate and rebounds. If e
experienced by the surface will be tution, the total distance travelled by the particle
(a) mnu (b) 2 mnu before it stops rebounding is
1 h(1 e2 )
(c) 4 mnu (d) mnu h(1 e2 )
2 (a) (b)
1 e2 (1 e2 )
17. A rubber ball is dropped from a height of 5 m on a
planet where the acceleration due to gravity is not h(1 e2 ) h(1 e2 )
known. On bouncing it rises to 1.8 m. The ball loses (c) (d)
2(1 e2 ) 2(1 e2 )
its velocity on bouncing by a factor of
16 2 23. A bullet of mass m
(a) (b) velocity v on a wooden block of mass M suspended
25 5
from a support and gets embedded in it. The kinetic
3 9
(c) (d) energy of the bullet + block system is
5 2
1 1
18. An isolated particle of mass m is moving in a hori- (a) mv2 (b) (M + m)v2
zontal plane (x – y), along the x-axis, at a certain 2 2
height above the ground. It suddenly explodes into
Mmv 2 m2 v2
two fragments of masses m/4 and 3m/4. An instant (c) (d)
later, the smaller fragment is at y = + 15 cm. The 2 M m 2 M m
larger fragment at this instant is at 24. A body of mass m moving with a speed v suffers
(a) y = – 5 cm (b) y = + 20 cm an inelastic collision and sticks with another body
(c) y = + 5 cm (d) y = – 20 cm of mass M = 2m at rest. The speed of the composite
IIT, 1997 body will be
19. A body P strikes another body Q of mass that is p v
(a) 3v (b)
times that of body P and moving with a velocity 3
1
that is of the velocity of body P. If body P comes 2v 3v
(c) (d)
q 3 2
25.
p q p q of the system to the initial kinetic energy is
(a) (b)
p q q p 1 1 2
(a) (b)
p q p q 3 3
(c) (d)
p q 1 p q 1 3
(c) 3 (d)
20. Two equal spheres A and B lie on a smoot hori- 2
zontal circular groove at opposite ends of a diam- 26. A body of mass 5 kg is moving along the x-axis
eter. Sphere A is projected along the groove and at with a velocity 2ms–1. Another body of mass 10 kg
the end of time T impinges on sphere B. If e is the
is moving along the y-axis with a velocity 3 ms–1.
occur after a time equal to They collide at the origin and stick together. The
(a) T (b) eT
2T (a) 3 ms–1 (b) 3 1 ms–1
(c) (d) 2 eT
e 4
IIT, 1997 (c) ms–1 (d) none
3
Conservation of Linear Momentum and Collisions 7.11

27. A block of wood of mass M is suspended by means mass M moving with a velocity V in the y-direction.
of a thread. A bullet of mass m The magnitude of the momentum of the composite
into the block with a velocity v. As a result of the body is
impact, the bullet is embedded in the block. The (a) (mv MV ) (b) (m + M) (v + V)
block will rise to vertical height given by
2 2 (c) [(mv)2 + (MV)2]1/2 (d) (Mv + mV)
1 mv 1 mv
(a) (b) 33. In Q. 32 above, the angle subtended by the veloc-
2g M m 2g M m
ity vector of the composite body with the x-axis is
mv 2 mv 2 given by
1 1
MV mv
(c) (d)
2g M m 2g M m (a) = tan–1 (b) = tan–1 MV
mv
28. A moving particle of mass m makes a head-on mV Mv
collision with a particle of mass 2m initially at rest. (c) = tan–1 (d) = tan–1
Mv mV
If the collision is perfectly elastic, the percentage
loss of energy of the colliding particle is 34. A body P of mass m1 moving with a certain velocity
(a) 50% (b) 66.7% collides head-on with a stationary body Q of mass
(c) 88.9% (d) 100% m2. It the collision is elastic, the fraction of kinetic
energy transferred from body P to body Q is
29. A body of mass m moving with a velocity v in the
x-direction collides with a body of mass M mov- 2(m1m2 ) 4(m1m2 )
(a) 2
(b)
ing with a velocity V in the y-direction. They stick (m1 m2 ) (m1 m2 )2
together during collision. Then 2m12 2m22
(a) the magnitude of the momentum of the (c) (d)
(m1 m2 )2 (m1 m2 )2
2 2
composite body is mv MV
35. Two particles, each of mass m, moving along
(b) the composite body moves in a direction different directions with a velocity u making the
MV same angle with the x-axis collide and stick together.
making a angle = tan–1 with the
x-axis. mv The composite particle moves along the x-axis with
a velocity u/2. The angle between their directions
(c) the loss of kinetic energy as a result of col-
of motion before collision is
1 Mm
lision is (V2 + v2) (a) 60° (b) 90°
2 M m (c) 120° (d) 150°
(d) all the above choices are correct. 36. A bullet of mass m moving with a horizontal
30. A body falls from a height h on a horizontal surface velocity u strikes a stationary wooden block of mass
and rebounds. Then it falls again and again rebounds M suspended by a string of length L = 50 cm. The
1 bullet emerges out of the block with speed u/4. If
, the M = 6 m, the minimum value of u so that the block
3
total distance covered by the body before it comes can complete the vertical circle is (take g = 10 ms–2)
to rest is (a) 10 ms–1 (b) 20 ms–1
–1
(c) 30 ms (d) 40 ms–1
h 5h
(a) (b) 37. A compound pendulum consists of a uniform rod of
4 4
length L of negligible mass. A body of mass m1 = m
(c) 2h (d) 3h m2 = 2 m
31. In Q. 30 above, the total time taken by the body to
come to rest is middle of the rod as
shown in Fig. 7.10
2h 2h
(a) (b) 2 The horizontal velo-
g g
city v that must be
2h 2h given to mass m1 to
(c) 3 (d) 4
g g rotate the pendnlum
to the horizontal
32. A body of mass m moving with a velocity v in the
position OC is
x-direction collides and sticks with another body of Fig. 7.10
7.12 Comprehensive Physics—JEE Advanced

(a) 4 (b) 3
2 gL
(a) 2 (b) 2 gL (c) 2 (d) 1
3 IIT, 2009
(c) 2gL (d) gL 40. A ball of mass 0.2 kg rests on a vertical post of
height 5 m. A bullet of mass 0.01 kg, travelling with
38. A ball is thrown will a velocity v1 towards a vertical a velocity V m/s in a horizontal direction, hits the
wall at an angle with the wall. It rebounds with centre of the ball. After the collision, the ball and
a velocity v2 making an angle with the wall as bullet travel independently. The ball hits the ground
at a distance of 20 m and the bullet at a distance of
between the ball and the wall is e, then v2 is given 100 m from the foot of the post. The velocity V of
by the bullet is (see Fig.7.13)
(a) v2 = v1(cos + e sin ) (a) 250 m/s (b) 250 m/s
(b) v2 = v1(sin + e cos ) (c) 400 m/s (d) 500 m/s
(c) v2 = v1 sin 2 e2 cos 2

(d) v2 = v1 cos 2 e2 sin 2

Fig. 7.13

IIT, 2011
41. A ball is thrown from a point O with a velocity
u = 20 ms–1 at an angle = 30° with the horizontal.
It hits a vertical wall which is at a distance x from
O as shown in Fig. 7.14. After rebounding from the
wall, the ball returns to O without retracing its path.
Fig. 7.11 If g = 10 ms–2 e = 0.5,
39. Two small particles of equal masses start moving in the value of x is
opposite directions from a point A in a horizontal (a) 9.6 m (b) 10.3 m
circular orbit. Their tangential velocities are v and (c) 11.5 m (d) 12.8 m
2v, respectively, as shown in Fig. 7.12. Between
collisions, the particles move with constant speeds.
After making how many elastic collisions, other
than that at A, will these two particles again reach
the point A?

v v

Fig. 7.12
Fig 7.14

ANSWERS
1. (a) 2. (c) 3. (c) 4. (d) 5. (b) 6. (d)
7. (b) 8. (c) 9. (a) 10. (c) 11. (a) 12. (a)
13. (a) 14. (c) 15. (a) 16. (b) 17. (b) 18. (a)
19. (d) 20. (c) 21. (b) 22. (a) 23. (d) 24. (b)
25. (a) 26. (c) 27. (a) 28. (c) 29. (c) 30. (b)
31. (b) 32. (c) 33. (a) 34. (b) 35. (c) 36. (d)
37. (a) 38. (d) 39. (c) 40. (d) 41. (c)
Conservation of Linear Momentum and Collisions 7.13

SOLUTIONS
1. Let v1 and v2 be the velocities of P and Q after the m1 m2 m 3m 3m
collision. = = =
(m1 m2 ) (m 3 m) 4
v2 v1
= v2 v1 (1) Substituting in Eq. (1) we get
u 0 u
From conservation of momentum, we have 3 k
kx2 = mv2 v = 2x
mu + 0 = mv1 + mv2 (2) 4 3m
u = v1 + v2 5. Frictional force f = mg. The retardation due to
friction is
From Eqs. (1) and (2), we get
f
a= = g = 0.2 10 = 2 ms–2
u u m
v1 = (1 – e) and v2 = (1 + e)
2 2 Since the blocks have the same mass and the colli-
v2 1 e sion is elastic, after the collision, block P will come
= to rest at the position previously occupied by block
v1 1 e
1 e 1 Q and Q will begin to move with the velocity at
Given v2 = 3v1. Hence 3 = e= which P strikes Q which is given by
1 e 2
2. Let u be the speed of the ball before the collision. v2 = 2as = 2 2 2=8 v= 8 ms–1
After the collision, its speed will be Moving with velocity of 8 ms–1, the block Q will
2 2 come to rest after travelling a distance x given by
u eu
v = v2 8
2 2 x= = =2m
2a 2 2
u2 u2 5 1 Hence the correct choice is (b).
= = u e
2 8 8 2 6. In an oblique elastic collision between two body of
1 1 the same mass, they move at right angles to each
mu 2 mv 2 other after the collision. Hence the correct choice
Fraction of K.E. lost = 2 2
is (d).
1
mu 2 7. Let p1 and p2 be the linear momenta of A and B after
2 the collision. Now, impulse = change in momen-
v2 5 3 tum.
=1– 2 =1– =
u 8 8 For particle B : I = p1
3. Let be the angle between the velocities of the For particle A : I = p – p2 p2 = p – I
two particles before collision. If p1 and p2 are the
v1 v2 m v1 m v2
momenta of the particles before collision and p is e= =
the momentum of the composite particle, then the u mu
conservation of momentum gives p1 p2
p2 = p 21 + p 22 + 2 p1 p2 cos =
p
2
v where m is the mass of each particle and u is the
2m = (mv)2 + (mv)2 + 2 (mv) (mv) cos velocity A before collision and v1 and v2 are the
2
1 velocities of A and B after the collision. Hence
cos =– or = 120°
2 p p I I p I 2I
e= 1 = = –1
4. If v is the relative velocity of the two blocks when p p p
the spring is at its natural length, then from the So the correct choice is (b).
conservation of energy, we have 8. Total initial momentum of the man-plank system is
1
v2 =
1
kx2 (1) zero. If he walks with a speed v on the plank, as a
2 2 result, the plank moves with a speed, say, v in the
where is the reduced mass of the system and is
given by system = mv – (M + m) v . From conservation of
momentum,
7.14 Comprehensive Physics—JEE Advanced

0 = mv – (M + m) v
V = 2 2 v.
v m
= 11. Velocity of the shell at the highest point is v =
v (M m)
100 sin 30° = 50 ms–1 parallel to x-direction. Its
Since the distance moved in proportional to speed momentum is p = mv (Fig. 7.15).
(since there is no acceleration), the displacement L
of the plank is given by choice (c).
9. Total momentum before collision is p = mu + 0
1
= mu and kinetic energy is K = mu2. If v1 and
2
v2 are the velocities of P and Q after the collision,
then, from momentum conservation,
mu = mv1 + (2 m) v2
p = p1 + p2 (1)
From conservation of kinetic energy, Fig. 7.15
K = K1 + K2 m
p1 = v1, where v1 = 200 ms–1. If v2 is the ve-
1 1 3
where K1 = mv 12 and K2 = (2 m) v 22 locity of the heavier fragment, its momentum is
2 2
2 mv2 /3. Conservation of momentum along x and
1 1 y-directions gives
Thus mu2 = mv12 + mv 22
2 2 p = p2 cos

p2 2 m v2
p2 p22 mv = cos
= 1 3
2m 2m 4m
3v = 2v2 cos (1)
2p2 = 2p 21 + p 22 (3)
and p1 = p2 sin
From Eq. (1), p2 = p – p1. Using this in Eq. (3) and
solving we get m 2 m v2
v1 = sin
p 4p 3 3
p1 = – . Hence p2 = v1 = 2v2 sin (2)
3 3
p2 p2 1
Now K1 =
1
mv 12 = 1 = From (1) and (2), we get v2 = (v12 + 9v2)1/2.
2 2m 18 m 2
1
= [(200)2 + 9(50)2]1/2
1 p22
4p 2
2
and K2 = mv 22 = =
2 4m 9m = 125 ms–1
K1 1 12. Initial momentum of the system = mv, since body
= , which is choice (a).
K2 8 of mass M is at rest. After the inelastic collision, the
10. Mass of each heavier fragment = 2 m/5 and of lighter bodies stick together and the mass of the composite
fragment = m/5. Momentum of each heavier frag- body is (m + M). If V is the speed of the composite
ment is p = 2 mv body, its momentum will be (m + M)V. From the
perpendicular directions, their resultant momentum principle of conservation of momentum, we have
mv = (m + M)V
= p2 p2 = 2 p . If V is the speed of the lighter
m
fragment, from the conservation of momentum, we or V = v
m M
have
mV 2 mv 1 1
= = 2 Initial KE = mv 2. Final KE = (m + M)V2.
5 2 p 5 2 2
Conservation of Linear Momentum and Collisions 7.15

Therefore, Let v2 be the velocity with which the ball bounces.


Final KE m+M V m2 It will attain a height h2 given by
= =
Initial KE m v 2
m M v22 = 2 gh2
Hence the correct choice is (a). v2 h2 18
.
13. Mass of neutron (m1) = 1 unit. Mass of nucleus = = 0.6
v1 h1 5
(m2) = A units. Refer to page 7.2. Here u1 = u and
u2 = 0. Therefore the velocity of the neutron after v2
or 1– = 1 – 0.6
the collision is v1
m1 m2 1 A v1 v2 2
v1 = u= u or = 0.4 =
m1 m2 1 A v1 5
1 Hence the correct choice is (b).
KE of neutron after collision = m1v 12 18. Let m and M be the masses of the lighter and
2
2
heaviers fragments respectively. Since the par-
1 1 A ticle is moving along the x-axis, the y-component
= u2
2 1 A of momentum will be zero immediately after and
1 before explosion, i.e.
KE of neutron before collision = m u2 mvy + MVy = 0
2
1 1 2 where vy and Vy are the velocities of the lighter and
= 1 u2 = u. heavier fragments respectively immediately after
2 2
explosion. Thus
1 A 2 m m/ 4
Their ratio is , which is choice (a). Vy = vy = vy
1 A M 3m / 4
14. The total number of nucleus (i.e. protons + neutrons)
in a nucleus is called its mass number. An -particle = 1 vy
3
is a helium nucleus having 2 protons and 2 neutrons.
Since y = + 15 cm, the direction of vy is along
So the mass number of an -particle = 4. When a
the positive y-axis and that of Vy will be along the
nucleus of mass number A emits an -particle, the
negative y-axis. An instant later (say, at time t), it
mass number of the daughter nucleus reduces to
is given that
(A – 4). If V is the recoil speed of the daughter
nucleus, we have, from the law of conservation of y = 15 cm = vy t
momentum, 1 1
Y = Vy t = vy t = y
(A – 4)V – 4v = 0 3 3
1
4v = 15 cm = – 5 cm
or V= 3
A 4
Hence the correct choice is (c). 19. Given mQ = p mP and vQ = vP /q. From the principle
15. The collision is inelastic because the two bodies of conservation of momentum, we have (since body
stick to each other after collision. In an inelastic P comes to rest after collision)
collision, only the momentum is conserved; there mP vP + mQ vQ = mQ v
being a loss in kinetic energy. Hence the correct where v is the velocity of body Q after collision.
choice is (a). Thus
16. Since the collision (impact) is elastic, the ball v
rebounds with the same speed. Therefore, the mP vP + p mP P = p mPv.
q
change in momentum of each ball = 2 mu. The
v p q
change in momentum per second due to n balls = which gives = (i)
2 mnu. But the change in momentum per second is vP pq
the force. Hence the correct choice is (b).
17. A ball dropped from a height h1 on reaching the v v
e=
planet’s surface will have a velocity given by vp vQ vp
vp
v1 = 2 gh1 q
7.16 Comprehensive Physics—JEE Advanced

v e 22. The total distance travelled is


which gives = (q – 1) (ii) S = h + e2h + 2e4h + 2e6h +
vP q
Equating (i) and (ii), we get e =
p q
which is = h + 2h(e2 + e4 + e6 + )
choice (d). p q 1 e
= h + 2h
20. Refer to Fig. 7.16. If sphere A is projected with 1 e2
velocity v, the time taken by it to strike B is equal
2e 2 h(1 e2 )
r = 1 =
to = T or r = Tv. Now, - 1 e2 1 e2
v
23. Initial momentum (p) = momentum of bullet +
titution is given by
momentum of block = mv + 0 = mv. From the
v vA
e= B
v momentum of bullet + block system of mass
(M + m) = Initial momentum p. Now
r p2 m2 v2
KE =
A B 2 M m 2 M m
Hence the correct choice is (d).
24. The correct choice is (b). In an inelastic collision,

Fig. 7.16 composite body, use the principle of conservation


of linear momentum.
where vA and vB are the velocities of A and B 25. The correct choice is (a). Find kinetic energies
after the collision. Thus, vB – vA = ev. The spheres before and after the collision.
travel with this relative velocity. It is clear that one 26. Momentum of 5 kg mass (p1) = 5 2 = 10 kg ms–1
will overtake the other after travelling a distance along the x-axis. Momentum of 10 kg mass (p2)
= 2 r.
= 10 3 kg ms–1 along the y-axis. These two mo-
2 r 2 r 2T v 2T menta are perpendicular to each other. Therefore,
Time taken =
vB v A ev ev e the resultant initial momentum is
2
(since r = Tv). Hence the correct choice is (c). p= p12 p22 10 2
10 3 = 20 kg ms–1
21. Let V be the velocity of the nucleus and v that of
the -particle after disintegration, then from the If v ms–1 is the velocity of the combined mass,
principle of conservation of momentum, we have v = 15 v
(since the mass of an -particle is 4 amu) kg ms–1. Now, from the principle of conservation
4
4v of momentum, we have 15 v = 20 or v = ms–1,
(M – 4) V = 4v or V = (i) 3
M 4 which is choice (c).
1 1 27. Let V be the velocity of the block with the bullet
Total KE = (M – 4) V 2 + 4 v2 (ii) embedded in it at the time of impact. Then from
2 2
the principle of conservation of momentum, we
1 have
Now 4 v2 = K. Using (i) in (ii), we have
2 mv = (M + m) V
1 16v 2
Total KE = (M – 4) 2
+ 2v2 mv
2 M 4 or V= M m (i)
8v 2 If the block, with the bullet embedded in it, rises
= + 2v2
M 4 to a vertical height h, then from the principle of
conservation of energy, we have
4K MK
= +K= ( 2v2 = K) 1
M 4 M 4 (M + m) V 2 = (M + m) gh
2
Hence the correct choice is (b).
or V= 2gh (ii)
Conservation of Linear Momentum and Collisions 7.17

Using (ii) in (i), we get


2h 2h 2h
31. Total time = 2e 2e 2
mv g g g
2gh =
M m
2h 2h
h is given = 2 e e2
correctly by choice (a). g g

4mM 2h 2h e 2h 1 e
28. Percentage loss of energy = 2
100 = 2
M m g g 1 e g 1 e
1
4m 2m 800 1
= 100 = = 88.9% 2h 3 2h
= 2 ,
2m m 2 9 g 1 g
1
Hence the correct choice is (c). 3
29. Refer to Fig. 7.17. Here p = mv and P = MV. The which is choice (b).
resultant of p and P is 32. Refer to Fig. 7.18.
2 2
pr = p2 P2 mv MV
y

P pr P

q
x
p

Fig. 7.17

which is choice (a). The angle which the resultant Fig. 7.18
momentum pr subtends with the x-axis is given by Let v be the velocity of the composite body at an
P MV angle with the x-axis (Fig. 7.11). Equating the
tan = , which is choice (b).
p mv x and y-axes,
Loss of KE we have
mv= (m + M) v cos = p cos (i)
1 1 1 m 2 v 2 M 2V 2
= mv 2 MV 2 and MV = (m + M) v sin = p sin (ii)
2 2 2 M m
where p = (m + M) v is the momentum of the
1 Mm composite body. Find p by squaring and adding (i)
= (V2 + v2), which is choice (c).
2 M m and (ii). The correct choice is (c).
30. Total distance 33. Divide (ii) by (i). The correct choice is (a).
= h + 2 e2 h + 2 e4 h + ... 34. Let u be the velocity of P before collision and v1
and v2 the velocities of P and Q after collision.
= h + 2 e2 h (1 + e2 + ...) From conservation of momentum, we have m1u + 0
2e2 h 1 e2 = m1v1 + m2v2 which gives
=h + h m1(u – v1) = m2v2 (1)
1 e2 1 e2
From the conservation of kinetic energy we have
2
1 1 1 1
1 m1u12 = m1v12 + m2v22
3 5h
=h 2
, 2 2 2
1 4
1 m1(u – v1) = m2v22
2 2
3
m1(u – v1)(u + v1) = m2v22 (2)
which is choice (b).
7.18 Comprehensive Physics—JEE Advanced

Dividing Eq. (2) by Eq. (1) we get 36. Let V be the speed of the block after the bullet
u + v1 = v2 (3) emerges out of it. From conservation of momentum
From Eqs. (1) and (3) we get we have
m1 m2 u
v1 = u (4) mu = MV + m
m1 m2 4
3mu
1 which gives V =
Initial K.E. of P(K) = m1u2, 4M
2
1 Now refer to Q. 27 of Section III of Chapter 4. The
P= m 1v 12 minimum speed the block must have to complete
2
1 1 the vertical circle is
Decrease in K.E. of P( K) = m1u2 – m1v12. V= 5gL
2 2
Fractional decreases in K.E. of P is 3mu
= 5gL
1 1 4M
m1u 2 m1v12
K 2 2 4M
= u= 5 gL
K 1 3m
m u2
2 1 4
= 6 5 10 0.5 = 40 ms–1.
u2 v12 v12 3
= =1–
u2 u2 Hence the correct choice is (d).
2
=1 –
m1 m2
[use Eq. (4)] 37. Since mass m2 is at a distance L/2 from the axis of
m1 m2 rotation, it speed will be v/2 (half that of mass m1).
4m1m2 From the principle of conservation of energy we
= have
(m1 m2 )2
Thus the correct choice is (b). 1 1 v 2
m 1v 2 + m2 = m1gOB + m2gOA
35. Refer to Fig. 7.19. 2 2 2
From conservation of x-component of momentum, 1 1 v2 L
we have mv2 + 2m = mgL + 2mg
2 2 4 2
mucos + mucos = 2mv 2 gL
2 2 which gives v = 2 , which is choice (a).
3
u
=2 m = mu ( v = u/2) 38. The component of velocity parallel to the wall
2 remains unchanged and the component of veloc-
ity perpendicular to the wall reduces by e times its
2 cos =1
2 value before collision. Thus we have
1 v2 cos = v1 cos
which gives cos = 60° = 120°,
2 2 2 and v2 sin = ev1 sin
which is choice (c)
Squaring and adding we get

v2 = v1 (cos2 e2 sin 2 )
Hence the correct choice is (d).
39. Refer to Fig. 7.20. First collision will occur when
angle
r 2 r
=
v 2v
which gives =120°.
B, m2 will move back with
Fig. 7.19 a speed v and will make a second collision with m1
Conservation of Linear Momentum and Collisions 7.19

at C. After the second collision at C, m1 will move Horizontal velocity after rebounding from the wall is
back with a speed v and meet m2 at A. If the third u x = e ux = e u cos .
collision at A is neglected, the particles will make
two collisions before they reach A. Hence the cor- Horizontal displacement from O to A or from A to
rect choice is (c). O = x.
Time taken to go from O to A is
t1 = x (i)
v v
u cos
Time taken to return from A to O is
v v x
t2 = (ii)
eu cos
Since the horizontal and vertical motions are inde-
v v pendent of each other, the net vertical displacement
Sy = 0 since the ball returns to O. If t is the total
Fig. 7.20
time taken by the ball to go from O to A and return
2h 2 5 to O, then from
40. t f) =
= = 1s 1 2
g 10 S y = u yt – gt we have
Horizontal range (R) = horizontal velocity time 2
1 2
0 = (u sin ) t – gt
Horizontal velocities of the bullet and of the ball 2
2u sin
after the collision respectively are t= (iii)
g
100
(v)bullet = = 100 ms–1 Now t = t1 + t2. Using (i) and (ii) in (iii), we have
1
20 2u sin x x
(v)ball = = 20 ms–1 =
1 g u cos eu cos
From conservation of momentum,
u 2 sin(2 )
x=
1
(m)bullet V = (m)bullet (v)bullet + (m)ball 1
e
(v)ball
0.01 V = 0.01 100 + 0.2 20 20 2 sin 60
=
1
V = 500 ms–1 1
0.5
41. Refer to Fig 7.14 on page 7.12. 20
Horizontal velocity before hitting the wall is ux = = 11.5 m, which is choice (c)
u cos 3

II

Multiple Choice Questions with one or More Choices Correct


1. In an inelastic collision of two bodies, which of the (a) In a elastic collision of two bodies, the momen-
following do not change after the collision? tum and energy of each body is conserved.
(a) total kinetic energy (b) The total energy of a system is always
(b) total linear momentum conserved irrespective of whether external
(c) total energy forces act on the system.
(d) total angular momentum (c) The work done by a force in nature on a body,
2. Which of the following statements are true? over a closed loop, is not always zero.
7.20 Comprehensive Physics—JEE Advanced

(d) In an inelastic collision of two bodies, the (b) If the balls stick together, the kinetic energy
of the system immediately after the collision
kinetic energy of the system. becomes half of that before collision.
3. A molecule in a gas container hits the wall with (c) If the collision is perfectly elastic, the bob of
speed v at an angle with the normal and rebounds v2
with the same speed as shown in Fig. 7.21. Which the pendulum will rise to a height of .
2g
of the following statements are true?
(a) The momentum of the system is conserved (d) If the collision is perfectly elastic, the kinetic
in the collision. energy of the system immediately after the
(b) The momentum of the molecule before col- collision is equal to that before collision.
lision with the wall is equal to the momentum 7. A body of mass 1 kg, initially at rest explodes into
of the molecule after collision. three fragments of masses in the ratio of 1 : 1 : 3.
(c) The collision is elastic. -
(d) The collision is inelastic. lar to each other with a speed of 30 ms–1, one along
Wall
the + x direction and the other along the + y direc-
tion. Then
(a) the speed of the heavier fragment will be
v
10 2 ms–1.
(b) the speed of the heavier fragment will be
15 2 ms–1.
(c) the direction of motion of the heavier frag-
v ment will be at angle of 135° with the + y
direction
(d) the direction of motion of the heavier frag-
ment will be at an angle of 45° with the
Fig. 7.21
+ x direction.
4. A particle A suffers an oblique elastic collision with 8. Two bodies A and B of masses m and 2m respec-
a particle B that is at rest initially. If their masses -
are the same, then, after the collision nected by a spring of spring constant k. A third body
(a) they will move in the opposite directions C of mass m moves with a velocity v0 along the line
(b) A continues move in the original direction joining A and B and collides elastically with A as
while B remains at rest shown in Fig. 7.22. At a certain instant of time t0
(c) they will move in the mutually perpendicular after the collision, it is found that A and B have the
directions same velocity v and at this instant, the compression
(d) A comes to rest and B starts moving in the of the spring is x0. Then
direction of the original motion of A
v0 v0
5. Choose the correct statements from the following: (a) v = (b) v =
(a) The general form of Newton’s second law 2 3
of motion is Fext = ma. 2 mv02 3 mv02
(b) A body can have energy and yet no momen- (c) k = (d) k =
3 x02 2 x02
tum.
(c) A body having momentum must necessarily
have kinetic energy.
v0
(d) The relative velocity of two bodies in a head–
on collision remains unchanged in magnitude
and direction Fig. 7.22
6. A ball of mass m moving horizontally at a speed v 9. A block of mass M attached to a light spring of force
collides with the bob of a simple pendulum at rest. constant k rests on a horizontal frictionless surface
The mass of the bob is also m. as shown in Fig. 7.23. A bullet of mass m moving
(a) If the balls stick together, the height to which with a horizontal velocity v strikes the block and
v2 gets embedded in it. The velocity of the block with
the two balls rise after the collision is .
8g the bullet in it just after impact is V. If the impact
compresses the spring by an amount x, then
Conservation of Linear Momentum and Collisions 7.21

x
(a) v = [k(M + m)]1/2
m
1/ 2
2k
(b) v = x
M m
x
(c) V = [2k (M + m)]1/2
m Fig. 7.25
1/ 2
k The track is frictionless and the collision is elastic.
(d) V = x
M m If vA and vB are the velocities of A and B after the
collision, then
(a) vA = 0, vB = 1.4 ms–1
(b) vA = 1 ms–1, vB = 1 ms–1
(c) vA = – 1.4 ms–1, vB = 1.4 ms–1
(d) vA = – 1 ms–1, vB = 1 ms–1
13. Two blocks, each of mass m, moving in opposite
Fig. 7.23 directions with the same speed u, on a horizontal
frictionless surface, collide with each other, stick
10. A body P of mass 1 kg moving with a velocity of
together and come to rest. Then
3 ms–1 along the + x direction collides head-on with
(a) work done by external force on the system
a body Q of mass 2 kg at rest. The collision is elas- is zero.
tic. After the collision (b) work done by the external force on the
(a) P moves along the +x direction with a veloc- system is mu2.
ity of 1 ms–1. (c) work done by the internal force on the
(b) P moves along the –x direction with a veloc- system is zero.
ity of 1 ms–1. (d) work done by the internal force on the
(c) Q moves along the +x direction with a veloc- system is – mu2.
ity of 2 ms–1. 14. A ball P of mass m1 moving with velocity u collides
(d) Q moves along the –x direction with a veloc-
head-on with a stationary ball Q of mass m2. The
ity of 2 ms–1.
collision is perfectly elastic. After the collision
11. A block of mass M with a massless spring of force
(a) if m1 = 2 m2, balls P and Q move in the same
constant k is resting on a horizontal frictionless direction with speeds in the ratio of 1 : 4.
surface (Fig. 7.24). A block of mass m projected (b) if m1 = 3 m2, balls P and Q move in the same
horizontally with a speed u collides and sticks to direction with speeds in the ratio of 1 : 3.
the spring at the point of maximum compression (c) if m2 = 2 m1, balls P and Q move in opposite
of the spring. directions with speeds in the ratio of 1 : 2.
(d) if m2 = 3 m1, balls P and Q move in opposite
directions with equal speeds.
15. A ball P of mass m1 moving with a velocity u
collides obliquely with a stationary ball Q of mass
Fig. 7.24
m2. The collision is perfectly elastic. After the col-
If v is the velocity of the system after mass m sticks
to the spring and n is the fraction of the initial ki- original direction of ball P as shown in Fig. 7.26.
netic energy of mass m that is stored in the spring,
then
v M v m
(a) (b)
u (M m) u (M m)
M m
(c) n = (d) n =
(M m) (M m)
12. A small ball A slides down the quadrant of a circle
as shown in Fig. 7.25 and hits the ball B of equal
mass which is initially at rest.
Fig. 7.26
7.22 Comprehensive Physics—JEE Advanced

(a) if m1 = m2, balls P and Q 19. A body P of mass m1 moving with a velocity
angles to each other with the same speed.
(b) if m1 = m2, balls P and Q u1 = (a i + b j ) collides with a stationary body Q
60° with each other with the same speed. of m2. After the collision body P is found to move
(c) if m1 = 2 m2, balls P and Q with velocity v1 = (c i + d j ) where a, b, c and d
angles to each other with speeds in the ratio
are constants. Then
of 1 : 2.
(d) if m1 = 2 m2, balls P and Q (a) Impulse received by P is m1[(a – c) i +
angle of 60° with each other with speeds in (b – d) j ]
the ratio of 1 : 2.
(b) Impulse received by P is m1[(c – a) i +
16. Which of the following statements is/are incorrect
in the case of an elastic collision between two bod- (d – b) j ]
ies? (c) Impulse imparted to Q is m1[(a – c) i +
(a) If two balls of the same mass moving with (b – d) j ]
the same speed in opposite directions, collide
head-on, then after the collision they move (d) Impulse imparted to Q is m2[(a – c) i +
in opposite direction with the speed each ball (b – d) j ]
had before collision. 20. A billiards ball C of mass m moving with velocity u
(b) If a body suffers a head-on collision with an- collides two identical balls A and B in contact and at
other body of the same mass but at rest, then
rest. After the collision, ball C is stopped dead and
balls A and B move along directions shown in Fig.
dead and the second body moves with the
7.27 with the same speed v. Then
e = 1 for a
perfectly elastic collision.
(d) If a body P collides head-on with a body Q of
the same mass but at rest, then the percentage
fraction of kinetic energy transferred from P
to Q is 50%.
17. Which of the following statements is/are true in the
case of an inelastic collision between two bodies?
(a) The vector sum of the linear momenta of the
two bodies before collision is equal to the
Fig. 7.27
vector sum of the linear momenta after the
u
collision in the case of both one-dimensional (a) v =
and two-dimensional collisions. 3
(b) The total energy of the system is con- u
served. (b) v =
2
(c) The two bodies stick together after inelastic 1
(c) Loss of kinetic energy = mu2
collision. 3
(d) If a body collides with another body of the
1
same mass but at rest and two bodies stick (d) Loss of kinetic energy = mu2
together, the ratio of the total kinetic energy 6
before and after collision is 2 : 1. 21. A U-238 nucleus emits an alpla particle and changes
18. A body P of mass 1 kg moving a velocity of into Th-234. In this process
15 ms–1 collides head-on with a stationary body Q (a) the momentum of Th-234 is equal and oppo-
site to that of the alpha particle.
1/3, then (b) the magnitude of the momentum of Th-234
(a) velocity of P after collision will be 5 ms–1. is greater than that of the alpha particle.
(b) velocity of Q after collision will be 10 ms–1. (c) the kinetic energy of the alpha particle is
(c) the loss of kinetic energy of the system is 50 J. equal to that of Th-234.
(d) the percentage fractional decrease in the (d) the kinetic energy of the alpha particle is
kinetic energy of body P is 50%. greater than that of Th-234.
Conservation of Linear Momentum and Collisions 7.23

22.
is no external force acting on the balls. If p 1 and
collision. In this case
(a) the momentum of the ball just after the colli- p 2 are
sion is the same as that just before collision, option (s) is/are not allowed for any non-zero value
(b) the mechanicial energy of the ball remains of p, a1, a2, b1, b2, c1 and c2
the same in the collision.
(c) the total momentum of the ball and the earth (a) p 1 = a1 i b1 j c1 k ; p 2 a2 i b2 j
is conserved.
(d) the total energy of the ball and the earth is (b) p 1 = c1 k ; p 1 c2 k ;
conserved.
(c) p 1 = a1 i b1 j c1 k ; p 1 a2 i b2 i c1 k
IIT, 1986
(d) p 1 = a1 i b1 j ; p 2 a2 i b1 i
23. Two balls having linear momenta p1 p i and
p2 = – p i undergo a collision in free space. There IIT, 2008

SOLUTIONS

1. The correct choices are (b) and (c). remains unchanged. Hence the total kinetic energy
2. Choice (a) is false. In an elastic collision of two is also conserved. Therefore, the collision is elastic.
bodies, the speeds of the bodies change due to colli- Remember, in an inelastic collision, although the to-
sion. Therefore, the momentum and energy of each tal momentum is conserved, the total kinetic energy
body will change but the total momentum and total is not conserved, it decreases. Hence the correct
energy of the system of two bodies are conserved. choices are (a) and (c).
Choice (b) is also false. The total energy of an iso- 4. From the principle of conservation of momentum,
lated system is conserved. If external forces act on -
the system, the total momentum and energy will mentum.
change. Choice (c) is true. For a non-conservative Momentum conservation is possible in cases (c)
force such as friction, the work done over a closed and (d). In case (c), the two masses should move
loop is not zero. Choice (d) is also true. In an inelas- in mutually perpendicular directions with velocity
tic collision, the two bodies stick together after col- v/ 2 each inclined at 45° with the original direc-
liding. This results in heat or sound energy which is tion of motion of particle A. In case (d), particle B
dissipated at the expense of kinetic energy. Hence must move with velocity v in the original direction
choices (c) and (d) are correct. of motion of A. Hence the correct choices are (c)
3. The system consists of the molecule and the wall. and (d).
Let us assume that initially the wall is stationary so 5. The general form of Newton’s second law is
that its momentum and kinetic energy are both zero
before the collision. Therefore, the total momentum dp d dv dm
Fext = = (mv) = m +v
of the wall + molecule system before the collision is dt dt dt dt
P = 0 + mv, where m is the mass of the molecule.
After the collision the wall acquires a recoil veloc- dm
The form Fext = ma is valid only if = 0, i.e.
ity, say, V and a recoil momentum MV where M is dt
the mass of the wall. After the collision, the recoil
if mass does not change with time. Hence choice
momentum of the wall + momentum of the outgoing
molecule = momentum of the incoming molecule
so that the total momentum of the system is con- (a) is incorrect. Choice (b) is correct because a
served. Notice that the momenta of outgoing and body at rest may have potential energy and yet
incoming molecules are not the same, their direc- no momentum. Choice (c) is also correct. A body
tions are different. has momentum if it has mass and velocity and a
- body having a mass and velocity must have kinetic
mentum produces a negligible velocity so that the energy. Choice (d) is incorrect because the relative
kinetic energy of the wall is negligible after the velocity remains unchanged in magnitude and gets
collision. Since the speed v of the molecule is the reversed in direction; (v2 – v1) = – (u2 – u1). Hence
same before and after collision, its kinetic energy the correct choices are (b) and (c).
7.24 Comprehensive Physics—JEE Advanced

6. In the inelastic collision, two bodies stick together. or = 45° with the x or y axes. From the principle of
After the collision, the speed of the ball and the bob conservation of linear momentum, the magnitude
(sticking together) is v = v/2. The height to which of the momentum of the third fragment is (here v3
they will rise is given by is the speed of the heavier fragment)
v = 2gh p3 = m3 v3 = 6 2

v2 v2 But m3 = 0.6 kg. Therefore,


or h =
2g 8g 6 2
v 3= 10 2 = 14.1 ms–1
Mass of the ball and the bob sticking together is 0.6
1 1 The direction of the velocity of the heavier frag-
m = 2 m. KE after collision = m v 2 = 2m
2 2 ment is inclined with x or y axes at an angle of
135° (see Fig. 7.28). The correct choices are (a)
v 2 1 1
= mv2. KE before collision = mv2. and (c).
2 4 2 8. Initially (i.e. before collision) bodies A and B are at
Therefore, their ratio is 1: 2. In an elastic collision rest and C is moving to the right (towards A) with a
between two bodies of the same mass with one of velocity v0. At a certain instant, say t = 0, C collides
them initially at rest, the moving body is brought to with A. Since the collision is elastic and A and C
rest and the other moves in the same direction with have equal masses, the entire momentum (mv0 ) and
the same speed. Thus the ball will come to rest and 1
the bob of the pendulum acquires a speed v. At this kinetic energy m v02 of C are transferred to A
2
speed, it will rise to height h given by h = v2/2 g. and hence C comes to rest. Thus at t = 0, A moves
Thus all four choices are correct. to the right with a velocity v0 and at this instant the
7. Let m1, m2 and m3 be the masses of the three frag- spring is uncompressed and B is at rest. Hence the
ments. As the total mass is 1 kg and m1 : m2 : m3 = momentum of the system at t = 0 is (mv0). When A
1 : 1 : 3, we have m1 = m2 = 0.2 kg and m3 = 0.6 kg. moves to the right, it compresses the spring and as a
The linear momentum of m1 is result body B begins to move to the right. It is given
p1 = m1 v1 = 0.2 30 = 6 kg ms–1 that at time t = t0, the compression of the spring is
and let it be directed along the x-axis (Fig. 7.28). x0. Let v be the common velocity of A and B at this
instant. From the principle of conservation of linear
momentum, we have momentum of C before colli-
sion = momentum of A after collision + momentum
of B after collision
v
or mv0 = mv + (2m)v or v = 0 (1)
3
From the principle of conservation of energy, we
have
KE of C before collision =(KE of A + KE of B)
Fig. 7.28 after collision + PE in stored spring
The linear momentum of m2 is 1 1 1 1
or mv20 = mv2 + (2m)v2 + kx 20
p2 = m2 v2 = 0.2 30 = 6 kg ms–1 2 2 2 2
and let it be directed along the y-axis. The magni- where k is the spring constant. Thus
tude of the resultant momentum is mv 20 = 3mv 2 + kx 02 (2)
p = ( p 12 + p 22)1/2 = [(6)2 + (6)2]1/2 Using (1) in (2), we get
2
= 6 2 kg ms–1 v0
mv 20 = 3m + kx 02
The direction of the resultant momentum is given 3
by
2 m v02 2 m v02
p or = kx 02 or k =
tan = 2 = 1 3 3 x02
p1
Thus the correct choices are (b) and (c).
Conservation of Linear Momentum and Collisions 7.25

1 1 2
9. PE stored in the spring = kx2. kx
( M m) v 2
2 or 2 =1– (3)
mv = (M + m)V 1
mu 2 mu 2
2
mv
or V = (1) v m
( M + m) From Eq. (1), we have
u ( M m)
After collision, KE of block + bullet in it = PE of
the spring. Thus Using this in Eq. (3), we get
1 2
1 1 kx
M
(M + m)V 2 = kx2 n= 2 =
2 2 1 ( M m)
mu 2
k 2
which gives V = x (2) Thus the correct choices are (b) and (c).
( M m)
12. If uA is the velocity with which A strikes B, then
Using Eq. (2) in (1), we have
1
(M m) V muA2 = mgh
v= 2
m uA = 2gh = 2 9.8 0.1 = 1.4 ms–1.
1/ 2
(M m) k Since the masses of the balls are equal and the
= x
m M m collision is elastic, they exchange their velocities
after collision. Hence the only correct choice is (a).
x 13. Total initial momentum before collision = mu +
= [k (M + m)]1/2
m (– mu
Thus the correct choices are (a) and (d). 0, as the blocks come to rest. Since the change in
10. From conservation of momentum, we have momentum is zero, no external force acts on the
m 1u 1 + m 2u = m 1v 1 + m 2v 2 system. Hence no work is done by the external
force on the system. From work-energy principle,
1 u1 + 0 = 1 v1 + 2 v2
1 1 2 1 2
v2 =
(u1 – v1) (1) initial K.E = 0 – mu mu = – mu2. Hence
2 2 2
From conservation of kinetic energy, we have the correct choices are (a) and (d).
14. Let v1 and v2 be the velocities of balls P and Q
1 1 1 1
m1u12 + m2u22 = m1v12 + m2v22 respectively after the collision. Then we have
2 2 2 2 m 1u = m 1v 1 + m 2v 2 (1)
u1 = v1 + 2v2
2 2 2
1 1 1
v12 = u21 – 2v22 (2) and m1u12 = m1v21 + m2v22
2 2 2
Solving Eqs. (1) and (2) we get v1 = – 1 ms–1 and m 1u 1 = m 1v 1 + m 2v 22
2 2
(2)
v2 = + 2 ms–1.
(a) Putting m1 = 2m2 in Eqs. (1) and (2) and
Hence the correct choices are (b) and (c).
u 4u
11. For conservation of momentum and conservation solving them, we get v1 = and v2 = .
of total energy, we have 3 3
Thus v1 = v2/4.
mu = (M + m) v (1) u 3u
(b) For m1 = 3m2, we get v1 = and v2 = .
1 1 1 2 2
Also mu2 = (M + m)v2 + kx2 (2) Thus v1 = v2 /3.
2 2 2 u 2u
1 (c) For m2 = 2 m1, we get v1 = – and v2 =
Dividing Eq. (2) bv mu2, we get 2 3
2 giving v1 = – v2 /3
1 2 u u
(M m) v 2
kx (d) For m2 = 3m1, we get v1 = – and v2 =
1= 2 2 2
mu 2 1 giving v1 = – v2
mu 2 Hence all the four choices are correct.
2
7.26 Comprehensive Physics—JEE Advanced

15. From conservation of x and y components of 1


momentum we have = 1 (15)2 = 112.5 J
2
m1u = m1v1 cos + m2v2 cos 1 1
Total K.E. after collision = m v12 + m v22
and 0 = m1v1 sin – m2v2 sin 2 2
1
which give m1u = (m1v1 + m2v2) cos (1) = 1 (52+ 102) = 62.5 J
2
and m 1v 1 = m 2v 2 (2) Loss in K.E. = 112.5 – 62.5 = 50 J.
From conservation of kinetic energy, we have Percentage fractional decrease in K.E. of P is
1 1 1 1 2 1 2
m1u2 = m1 v12 + m2 v22 mu v
2 2 2 K
= 2 2 1 100
K 1 2
m1 u2 = m1 v12 + m2 v22 (3) mu
2
If m1 = m2, then from Eqs. (1) and (2), we get
u2 v12
v1 = v2 and u = 2v1 cos = 100
u2
1
Using these in Eq. (3), we get cos2 = (15) 2 (5)2
2 = 100 89%
= 45°. (15) 2
Hence choice (a) is correct and choice (b) is Hence the correct choices are (a), (b) and (c).
wrong.
19.
Similarly putting m1 = 2 m2 in Eqs. (1), (2) and v2 is the velocity of ball Q after the collision, then
(3), we get from the conservation of momentum, we have
v2 m 1u + 0 = m 1 v 1 + m 2 v 2
3
v1 = and cos = = 30°.
2 2 m2 v2 = m1(u – v1)
Hence choice (d) is correct and choice (c) is
wrong. = m1[(a i + b j ) – (c i + d j )]
16. The only incorrect statement is (d). Refer to the
solution of Q.34 of Section I. = m1[(a – c) i + (b – d) j ]

K 4m1m2 4m m Impulse received by P is


= = = 1 or 100 %
K (m1 m2 ) 2
( m m) 2 IP = m1 v1 – m1 u
17. The only incorrect statement is (c). Statements (a), = m1(v1 – u)
(b) and (d) are true.
= m1[(c – a) i + (d – b) j ]
18. Given m = 1 kg, u = 15 ms–1 and e = 1/3. Let v1 and
v2 be the velocities of P and Q after the collision. Impulse imparted to Q is
mu + 0 = m v1 + m v2 IQ = m2 v2 – 0
u = v1 + v2 (1)
v2 v1 = m1[(a – c) i + (b – d) j ]
e= (2)
u Hence the correct choices are (b) and (c).
From Eqs. (1) and (2), we get 20. Conservation of x-component of momentum gives
u 15 1 mu = mv cos 30° + mv cos 30°
v1 = (1 – e) = 1 = 5 ms–1
2 2 3 3
= 2 mv = 3 mv
u 15 1 2
and v2 = (1 + e) = 1 = 10 ms–1 u
2 2 3 v= . Hence choice (c) is wrong.
3
1 1
Total K.E. before collision = mu2 K.E. before collision = mu2
2 2
Conservation of Linear Momentum and Collisions 7.27

1 1 2
K.E. after collision = mv2 + mv2 = mv2 m mTH mTH
2 2 = =
mTH m m
mu 2 Since mTH > m ; K > KTh. Hence the correct choic-
=
3 es are (a) and (d).
1 1 1 22. In a collision, the momentum of indivdual bodies
Loss of K.E. = mu2 – mu2 = mu2. is not conserved. In an inelastic collision, there is
2 3 6
a loss of kinetic energy. Hence choices (a) and (b)
Hence the correct choices are (a) and (d).
are incorrect. The correct choices are (c) and (d).
21. Initially U-238 is at rest and has zero momentum.
23.
When it emits an -particle, the sum of the momenta
of -particle and Th-234 nucleus must be zero, i.e. momentum is pi p1 p2 pi p i = 0. There-
v m
m v + mTh vTh = 0 = – Th c1 = 0
vTh m and in choice (b) and (c) for non-zero values of the
1 1 i and j . Hence the choices (a) and
Now KTh = mTh vTh
2
and K = m v2
2 2 (d) are not allowed.
K m v 2
Hence =
K TH mTH vTH

III

Multiple Choice Questions Based on Passage

Questions 1 to 3 are based on the following passage obtained from the experiments involving collisions.
There are certain collisions called nuclear reactions
Passage I
in which new particles are formed. For example, when a
Collisions:
slow neutron collides with a uranium-235 nucleus, new
In physics we come across many examples of collisions. nuclei baruim-141 and krypton-92 are formed. This colli-
The molecules of a gas collide with one another and with
the walls of the container. The collision of a neutron with deuterium and trituim collide (or fuse) to form a helium
an atom is well known. In a nuclear reactor fast neutrons nucleus with the emission of a neutron.
1. Which one of the following collisions is NOT elas-
down. They are, therefore, made to collide with hydrogen tic?
atoms. The term collision does not necessarily mean that (a) A hard steel ball dropped on a hard concrete
a particle or a body must actually strike another. In fact,
two particles may not even touch each other and yet they (b) Two balls moving in the same direction col-
lide and stick to each other.
the other. When two bodies collide, each body exerts an (c) Collisions between molecules of an ideal gas
equal and opposite force on the other. The fundamental (d) Collisions of fast neutrons with hydrogen
conservation laws of physics are used to determine the
velocities of the bodies after the collision. Collision may 2. Which one of the following statements is true about
be elastic or inelastic. Thus a inelastic collisions?
as an event in which two or more bodies exert relatively (a) The total kinetic energy of the particles after
strong forces on each other for a relatively short time. The collision is equal to that before collision.
forces that the bodies exert on each other are internal to (b) The total kinetic energy of the particles after
the system. collision is less than that before collision.
Almost all the knowledge about the sub-atomic particles (c) The total momentum of the particles after
such as electrons, protons, neutrons, muons, quarks, etc. is collision is less than that before collision.
7.28 Comprehensive Physics—JEE Advanced

(d) Kinetic energy and momentum are both con- (b) only momentum is conserved
served in the collision. (c) neither energy nor momentum is conserved
3. In elastic collisions (d) both energy and momentum are conserved.
(a) only energy is conserved
ANSWERS

1. The correct choice is (b) 3. The correct choice is (d)


2. The correct choice is (b)

Questions 4 and 5 are based on the following passage 4. If M < m, the number of collision between the balls
will be
Passage II
(a) one (b) two
Two balls marked 1 and 2 of the same mass m and a third
(c) three (d) four
ball marked 3 of mass M are arranged over a smooth
5. If M > m, the number of collisions between the balls
horizontal surface as shown in Fig. 7.29. Ball 1 moves
will be
with a velocity v1 towards balls 2 and 3. All collisions are
(a) one (b) two
assumed to be elastic. (c) three (d) four

Fig. 7.29

SOLUTION
4. 2 cannot collide with ball 3 again. Hence there are
both have the same mass, after the collision ball 1 only two collisions. Thus the correct choice is (b).
with come to rest and ball 2 will move with speed 5. If M > m, we have from Eq. (i)
v1. The ball will collide with the stationary ball 3. M m
After this second collision, let v2 and v3 be the speeds v2 = – v1
M m
of balls 2 and 3 respectively. Since the collision are
elastic, v2 and v3 are given by (see page 7.2) The negative sign indicates that, after the second
m M collision, ball 2 will move in opposite direction
v2 = v1 (i) -
m M
lision. Therefore, ball 2 will make another collision
2m with ball 1.
and v3 = v1 (ii)
m M Hence, in this case, there are three collisions in all
If M < m, it follows from (i) and (ii) that v2 < v3 between the balls. Thus the correct choice is (c).
and both have the same direction. Therefore, ball
Questions 6 to 8 are based on the following passage 7. The angle which the direction of the momentum
Passage-III vector of the composite body makes with the
x-axis is
A body of mass m1 = m moving with a velocity v1 = v in the (a) 30° (b) 45°
x-direction collides with another body of the same mass (c) less than 30° (d) greater than 45°
m2 = m moving in the y-direction with the same speed v2 = 8. The fraction of initial kinetic energy transformed
v. They coalesce into one body during the collision. into heat during the collision is
6. The magnitude of the momentum of the composite 1 1
body is (a) (b)
2 4
(a) mv (b) 2 mv
2 1
(c) (d)
(c) 2mv (d) 2 2 mv 3 3
Conservation of Linear Momentum and Collisions 7.29

SOLUTION
6. Refer to Fig. 7.30. Let v be the velocity of the 7. Diving Eq. (2) by Eq. (1), we have tan =1 =
composite body and let be the angle which the 45°, which is choice is (b).
velocity vector v makes with the x-axis. 8. Initial kinetic energy is
1 1
Ki = m1v21 + m 2v 22
2 2
1 1
= mv2 + mv2 = mv2
2 2
From Eq. (3), we have v = v/ 2 . Final kinetic
energy is
1
Kf = (m1 + m2)(v )2
2
1 v2 1
= 2m = mv2
2 2 2
Fig. 7.30 Loss in K.E. = Ki – Kf
Conservation of x and y components of momentum 1
gives = mv2 – mv2
2
m1v1 = (m1 + m2)v cos mv = 2mv cos (1) 1
= mv2
m2v2 = (m1 + m2)v sin mv = 2mv sin (2) 2
Fraction of initial K.E. transformed into heat is
Squaring and adding Eqs. (1) and (2), we get
1 2
mv
2(mv)2 = (2mv )2 2mv = 2 mv (3) K 1
= 2 2 = .
Momentum of composite body is 2 mv . Hence the Ki mv 2
correct choice is (b). Hence the correct choice is (a).

Questions 9 to 11 are based on the following passage 9. The speed v1 of ball P after the collision is
Passage IV u u
(a) (b)
A ball P moving with a velocity u strikes an identical 2 3
stationary ball Q such that after the collision, the direction u u
of motion of balls P and Q make an angle of 30° with the (c) (d)
2 3
original direction of motion of ball P, as shown in Fig. 10. The speed v2 of ball Q after the collision is
7.31.
u u
(a) (b)
3 3
2u 2u
(c) (d)
3 3
11. The ratio of the total kinetic energy of the balls
after collision to that before collision is
1 1
(a) (b)
3 3
2 1
Fig. 7.31 (c) (d)
3 2
SOLUTION
From conservation of x and y components of 3
momentum, u = (v1 + v2) (1)
2
mu = mv1 cos 30° + mv2 cos 30° and 0 = mv1 sin 30° – mv2 sin 30°
7.30 Comprehensive Physics—JEE Advanced

v1 = v2 (2) and after collision


u 1 1
9. Eqs. (1) and (2) give v1 = , which is choice (d). Kf = mv21 + mv22
3 2 2
u
10. v2 = v1 = . Hence the correct choice is (a). 1 u2 1 u2 mu 2
3 = m + m =
11. Total kinetic energy before collision is 2 3 2 3 3
1 Kf
Ki = mu2 =
2
, which is choice (c).
2 Ki 3

Questions 12 to 14 are based on the following passage 13. After the collision, body Q moves along the
Passage V (a) positive x-direction with speed u/3.
(b) negative x-direction with speed u/3.
A body P of mass m moving with a velocity u along the (c) positive x-direction with speed 2u/3.
+ x-direction makes a head-on elastic collision a body (d) negative x-direction with speed 2u/3.
Q of mass 2 m at rest.
14. What fraction of its kinetic energy does body P lose
12. After the collision, body P moves along the
after the collision?
(a) positive x-direction with speed u/3.
(b) negative x-direction with speed u/3. 8 7
(a) (b)
(c) positive x-direction with speed 2u/3. 9 8
(d) negative x-direction with speed 2u/3.
6 5
(c) (d)
7 6
SOLUTION
Let V be the velocity of body Q after the colli- 1
sions. Ki = mu2
2
From the principle of conservation of linear mo- 1
mentum, we have Final kinetic energy, Kf = mv2
2
mu = mv + (2m)V Loss in kinetic energy is
u – v= 2 V 1 1
or (1) K = Ki – Kf = mu2 – mv2
2 2
The conservation of kinetic energy gives
K
1 1 1 Fractional loss =
mu2 = mv2 + (2m)V2 Ki
2 2 2
1 2 1 2
or u2 – v2 = 2V2 mu mv
= 2 2
or (u – v) (u + v) = 2 V 2 (2) 1 2
mu
2
Using Eq (1) in Eq (2), we have
2
2V(u + v) = 2V2 or u + v = V u2 v2 v
= =1–
or 2(u + v) = 2V (3) u2 u
2
12. From Eqs. (1) and (3), we get v = – u/3. Hence the 1
=1– ( v = – u/3)
correct choice is (b). 3
13. Using v = – u/3 in Eq. (1), we get V = 2u/3. Hence
the correct choice is (c). 8
= , which is choice (c).
14. Initial skinetic energy of the colliding mass is 9

Questions 15 to 17 are based on the following passage 15. After the collision, body B will move with the
greatest speed if
Passage VI
(a) m1 >> m2 (b) m1 << m2
A body A of mass m1 moving with a velocity u makes a head- (c) m1 = m2 (d) m1 = 2 m2
on elastic collision with a body B of mass m2 initially at rest.
Conservation of Linear Momentum and Collisions 7.31

16. After the collision, body B will move with the 17. After the collision, body B will move with the
greatest momentum if greatest kinetic energy if
(a) m1 >> m2 (b) m1 << m2 (a) m1 >> m2 (b) m1 << m2
(c) m1 = m2 (d) m1 = 2 m1 (c) m1 = m2 (d) m2 = 2 m1

SOLUTION

Since the collision is elastic, both linear momentum and 2


kinetic energy are conserved. Thus = m1 m2 + 2 m1m2
m1 u1 = m1 v1 + m2 v2 (1) 2m1m2u
p2 = 2
1 1 1 m1 m2 2 m1m2
and m1 u21 = m1v21 + m2v22 (2)
2 2 2 Momentum p2 will be maximum if the denominator
Solving for v1 and v2, we get 2
is the minimum, i.e. if m1 m2 = 0 since a
m1 m2
v1 = u1 (3) perfect square can never be negative. Thus for p2 to
m1 m2
be maximum m1 m2 or m1 = m2. Hence body
2m1 B will move with the greatest momentum if its mass
and v2 = u1 (4)
m1 m2 is equal to the mass of body A. Hence the correct
choice is (c).
15. Equation (4) gives the recoil speed of body B which
17. The kinetic energy of body B after the collision is
can rewritten as
2
2u1 1 1 2m1u
v2 = K2 = m2v22 = m2
m2 2 2 m1 m2
1
m1 2m12 m2u 2
=
Now v2 will be maximum if the denominator is the m1 m2 2
m
minimum, i.e. if 2 << 1 or m2 << m1. Thus body The denominator (m1 + m2)2 can be rewritten as
m1
B will move with the greatest speed if its mass is (m1 + m2)2 = (m1 + m2)2 – 4m1 m2 + 4m1 m2
very small compared to the mass of body A. Hence = (m1 – m2)2 + 4 m1 m2
the correct choice is (a).
16. The momentum of body B after the collision is 2m12 m2u 2
K2 = 2
2m1m2u m1 m2 4m1m2
p2 = m2 v2 =
m1 m2 Energy K2 will be maximum if the denominator
The denominator (m1 + m2) can rewritten as is the minimum, i.e. if (m1 – m2)2 = 0 or m1 = m2.
2 2 Hence body B will move with maximum energy if
m1 + m2 = m1 + m2 – 2 m1m2
ita mass is equal to the mass of body A. Thus the
+ 2 m1m2 correct choice is (c).
Questions 18 to 20 are based on the following passage 19. If M = 2m, the speed of body P after the collision
Passage VII will be
A body P of mass m moving along the positive x-direction u u
with velocity u collides with a body Q of mass M initially (a) (b)
2 2
at rest. After the collision body P moves along the positive u
u
y-direction and body Q moves along a direction making (c) (d)
an angle below the x-axis. The collision is assumed to 3 3
be elastic. 20. If M = 2m, the kinetic energy gained by body Q due
18. The angle is give by to collision is
M m 2 1
(a) = tan–1 (b) = tan–1 (a) mu2 (b) mu2
m M 3 3
M m M m 1
(c) = tan–1 (d) = tan–1 (c) mu2 (d) mu2
M m M m 2
7.32 Comprehensive Physics—JEE Advanced

SOLUTION
Refer to Fig. 7.32. 18. Using Eqs. (1) and (2) in Eq. (3), we get
M m 1/ 2 M m 1/ 2
sin = and cos =
2M 2M
Hence the correct choice is (c).
1/ 2
2m m 1
19. If M = 2m, then sin = =
2 2m 2
= 30°
Dividing Eq. (2) by Eq. (1), we get
u
Fig. 7.32 v = u tan = u tan (30°) =
3
Conservation of x and y components of momentum Hence the correct choice is (d).
gives u
20. Similarly we get V = . Therefore, the increase
mu = MV cos (1) 3
0 = mv – MV sin 1 1
in K.E. of Q is MV2 – 0 = MV2
mv = MV sin (2) 2 2
From conservation of kinetic energy we have 1 u 2 1
= 2m = mu2.
1 1 1 2 3 3
mu2 = mv2 + MV2 (3)
2 2 2 Hence the correct choice is (b).
Questions 21 to 24 are based on the following passage 1/ 2
2h 1 e 2 2h 1 e 2
Passage VIII (c) (d)
g 1 e g 1 e2
A ball of mass m is dropped from a height h on a smooth 23. The total momentum imparted by the ball to the
e.
21. The total distance covered by the ball before it
comes to rest is 1 e2 1 e
2 (a) m 2hg 2
(b) m 2hg
1 e 1 e 1 e 1 e
(a) h (b) h
1 e 1 e
1/ 2
(c) m 2hg (d) me 2hg
1 e2 1 e2 24. The average force exerted by the ball on the wall is
(c) h (d) h
1 e2 1 e2 1 e
22. The total time taken by the ball to come to rest is (a) mg (b) mg
1 e
2h 1 e 2h 1 e 2 1 e2
(a) (b) (c) mg (d) zero
g 1 e g 1 e2 1 e2

SOLUTION

21. v = 2gh . Between the second and the third impact, the dis-
It will rebounce with a speed v1 = ev and will rise tance covered = 2e4 h and so on. Therefore, the total
to a height h1 given by distance covered by the ball before it stops is
v21 = 2g h1 H = h + 2e2 h + 2e4 h + ...
= h + 2e2 h(1 + e2 + ....)
v2 e2 v2 e2 2 gh
which gives h1 = 1 = = = e2h.
2g 2g 2g 2e2 h 1 e2
=h + =h
1 e2 1 e2
2
h1 = 2e h. Hence the correct choice is (c).
Conservation of Linear Momentum and Collisions 7.33

2h 2h 2h 2ev 1 e
22. Total time taken t = + 2e + 2e2 + =m v = mv
g g g 1 e 1 e
1 e
2h 2h = m 2gh
= +2 e (1 + e + ....) 1 e
g g
Thus the correct choice is (b).
2h 2h e 2h 1 e momentum transferred p
= +2 = 24. Average force =
g g 1 e g 1 e time taken t
Hence the correct choice is (a).
1 e
23. Total momentum transferred is m 2 gh
1 e = mg
p = m[v + 2(v1 + v2 + ...)] =
2h 1 e
= m[v + 2(ev + e2v + ...)] g 1 e
= m[v + 2ev(1 + e + ...)] Hence the correct choice is (a).

Questions 25 to27 are based on the following passage 25. The speed of the block at point B immediately after
Passage IX it strikes the second incline is
(a) 60 ms–1 (b) 45 ms–1
A small block of mass M
moves on a frictionless (c) 30 ms–1 (d) 15 ms–1
surface of an inclined 26. The speed of the block at point C, immediately be-
plane, as shown in Fig. fore it leaves the second incline is
7.33. The angle of the
incline suddenly change (a) 120 ms–1 (b) 105 ms–1
from 60° to 30° at point (c) 90 ms–1 (d) 75 ms–1
B. The block is initially
27. If collision between the block and the incline is
at rest at A. Assume that
completely elastic, then the vertical (upward)
collisions between the
component of the velocity of the block at point B,
block and the incline are
Fig. 7.33 immediately after it strikes the second incline is
totally inelastic (g = 10
m/s2) (a) 30 ms–1 (b) 15 ms–1
IIT, 2008 (c) 0 (d) 15 ms–1
SOLUTION
25. Let vB be the speed of the block just before it strikes
the second incline.
1
mvB2 = mg AD = mg BD tan 60°
2
1/ 2
vB = 2 10 3 tan 60 = 60 ms–1
This velocity can be resolved into two components.
vB cos 30° along the second incline and vB sin 30°
perpendicular to it (see Fig.7.34) Fig. 7.34
In an inelastic collision, the perpendicular compo-
nent becomes zero after the collision. Hence the 26. Let vC be the speed at C. From conservation of
speed of the block at point B immediately after the energy, gain in K.E. = loss in P.E., i.e.
3 1 2 1
collision is vB cos 30° = 60 45 ms–1. mvC – mvB2 = mg BE = mg EC tan 30°
2 2 2
7.34 Comprehensive Physics—JEE Advanced

vC2 = vB2 + 2g 3 3 tan 30°


1
= 45 + 2 10 3 3 = 105
3
vC = 105 ms–1
27. If the collision is perfectly elastic, the perpendicu-
1
lar component vB sin 30° = 60 = 15 ms–1 is
2
reversed after the collision as shown in the Fig.7.35.
The resultant vertical component is
Fig. 7.35
15 sin 60° – 45 sin 30° = 0, which is choice (c).

IV

Matching
Match the Statements in Column I with the Processes in Column II
Column I Column II
(a) Collision of two light nuclei to form a heavier nucleus (p) Elastic collision
(b) A speeding bullet getting embedded (q) Inelastic collision
in a wooden plank

(d) Collision in which there is no loss (s) Nuclear fusion


of kinetic energy
ANSWER
(a) (s) (b) (q)
(c) (r) (d) (b)

Reason-Assertion Type Questions


In the following questions, Statement-1 (Assertion) is 1. Statement-1
followed by Statement-2 (Reason). Each question has the Two identical balls B and C lie on a horizontal
following four choices out of which only one choice is smooth straight groove so that they are touching.
correct. A third identical ball A moves at a speed v along
(a) Statement-1 is true; Statement-2 is true and State- the groove and collides with B (see Fig. 7.36). If
ment-2 is the correct explanation for Statement-1. the collisions are perfectly elastic, then after the
(b) Statement-1 is true; Statement-2 is true but State- collision, balls A and B will come to rest and ball C
ment-2 is not the correct explanation for State- moves with velocity v to the right.
ment-1.
(c) Statement-1 is true; Statement-2 is false.
(d) Statement-1 is false; Statement-2 is true.
Fig. 7.36
Conservation of Linear Momentum and Collisions 7.35

Statement-2 Statement-2
In an elastic collision, linear momentum and kinetic If no external force acts, the total linear momentum
energy are both conserved. of a system is conserved.
2. Statement-1 8. Statement-1
Two bodies A and B of masses m and 2 m respec- In an elastic collision between two bodies, the en-
- ergy of each body is conserved.
nected by a spring. A third body C of mass m moves Statement-2
with a velocity u and collides elastically with A as The total energy of an isolated system is con-
shown in Fig. 7.37. At a certain instant t0 after the served.
collision, it is found that the velocities of A and B 9. Statement-1
are the same = u/3. The total energy of a system is always conserved
irrespective of whether external forces act on the
system.
v0 Statement-2
The total energy of an isolated system is always
Fig. 7.37 conserved.
10. Statement-1
Statement-2
A body P of mass M moving with speed u collides
In an elastic collision, the kinetic energy of the sys-
head-on and elastically with a body Q of m initially
tem is conserved.
at rest. If m << M, body Q will have a maximum
3. Statement-1
speed equal to 2 u after the collision.
In an inelastic collision between two bodies, the
total energy does not change after the collision but Statement-2
the kinetic energy of the system decreases. In an elastic collision, the momentum and kinetic
Statement-2 energy are both conserved.
The loss of kinetic energy appears as heat in the 11. Statement-1
system. A block of mass m starts moving on a rough
4. Statement-1 horizontal surface with a velocity v. It stops due
In a collision between two bodies, each body to friction between the block and the surface
exerts an equal and opposite force on the other at after moving through a certain distance. The
each instant of time during the collision. surface is now tilted to an angle of 30° with the
Statement-2 horizontal and the same block is made to go up
The total energy of the system is conserved. on the surface with the same initial velocity v.
5. Statement-1 The decrease in the mechanical energy in the
The term ‘collision’ between two bodies does not second situation is smaller than that in the first
necessarily mean that the two bodies actually strike situation.
against each other. Statement-2
Statement-2
In physics, a collision is said to take place if the one the surface decreases with the increase in the angle
of inclination.
6. Statement-1 IIT, 2007
In an inelastic collision, the two colliding bodies
12. Statement-1
stick to each other after the collision and move with
In an elastic collision between two bodies, the rela-
a common velocity.
tive speed of the bodies after collision is equal to
Statement-2
the relative speed before the collision.
There is a lost of total kinetic energy in an inelastic
Statement-2
collision.
In an elastic collision, the linear momentum of the
7. Statement-1
system is conserved.
In a collision between two bodies, the linear
momentum each body remains constant. IIT, 2007
7.36 Comprehensive Physics—JEE Advanced

SOLUTIONS
1. The correct choice is (a). Linear momentum will be but the total energy of the system of two bodies is
conserved if A comes to rest and B and C move to conserved.
the right with a velocity v/2 each or A, B and C all 9. The correct choice is (d). If an external force acts
move to the right with velocity v/3 each. It is easy on a system, it is accelerated which will increase the
to see that in these two cases, the kinetic energy is total energy.
not conserved. Hence the only result of the colli- 10. The correct choice is (a). If v and V are the veloci-
sion is the one given in Statement-1. ties of Q and P after the collision, then from conser-
2. The correct choice is (b). Let C collide with A at vation of momentum and kinetic energy, we have
t = 0. Since the collision in elastic and A and C have Mu = mv + MV M(u – V ) = mv (1)
equal masses, C will come to rest and A will move to 1 1 1
the right with velocity u and at this instant the spring Mu2 = mv2 + MV2
2 2 2
is uncompressed and B is at rest. Hence at t = 0, the
momentum of the system = mu. When A moves to M(u – V) (u + V) = mv2 (2)
the right, it compresses the spring and as a result B From Eqs. (1), (2), we get
beings to move to the right. Let v be the common 2Mu 2u
velocity of A and B at time t0. From the principle of v= =
M m m
conservation of linear momentum, we have 1
M
Momentum of C before collision = momentum of If M >> m, then v is maximum equal to 2u (since
A after collision + momentum of B after collision m
u zero).
or mu = mv + (2m) v v= . M
3 11. Statement-1 is true. The decrease in mechanical
3. The correct choice is (a). The total energy (which energy is smaller when the block is made to go
includes all forms of energy) is conserved in any up on the inclined surface because some part of
process. the kinetic energy is converted into gravitational
4. The correct choice is (b). Statement-1 follows from potential energy. Statement-2 is false. The coef-
Newton’s third law of motion.
5. The correct choice is (a). inclination of the plane.
6. The correct choice is (d). The two colliding body 12. For elastic collisions both the statements are true
need not get stuck after an inelastic collision. but statement-2 is not the correct explanation of
7. The correct choice is (d). Since the velocities of statement-1. Linear momentum is conserved in
the two bodies change due to collision, the linear elastic as well as inelastic collision. But in an elas-
momentum of each body will change but the total tic collision the total kinetic energy of the system
linear momentum of the system of two bodies is is also conserved. Statement-1 is obtained if we use
conserved. both the conservation of linear momentum as well
8. The correct choice is (d). Due to change in veloc- as the conservation of kinetic energy in the case of
ity, the energy of each body changes on collision an elastic collision.

VI

Integer Answer Type


1. A shell of mass 4 kg, initially at rest explodes into the wall to a horizontal position
three fragments. Two of the fragments, each of mass and released (see Fig. 7.38). The
1 kg are found to move with a speed of 2 ms –1 each bob hits the wall, the restitution
in mutually perpendicular directions. Find the total co 2 / 5 . What is
energy (in joule) released in the explosion. the minimum number of impacts
IIT, 1987 with wall after which the ampli-
2. A simple pendulum is suspended from a peg on a tude becomes less than 60°?
vertical wall. The pendulum is pulled away from IIT, 1988 Fig. 7.38
Conservation of Linear Momentum and Collisions 7.37

3. A cart is moving along the + x direction with a 4. Three objects A, B and C are kept in a straight line
velocity of 4 ms–1. A person on the cart throws a on a frictionless horizontal surface. These have
stone with a velocity of 6 ms–1 relative to him- masses m, 2m and m, respectively. The object A
self. In the reference frame of the cart, the stone moves towards B with a speed 9 m/s and makes an
is thrown in the y-z plane making an angle of 30° elastic collision with it. Thereafter B makes com-
with vertical z-axis. At the highest point of its pletely inelastic collision with C. All motions occur
trajectory, the stone hits an object of equal mass in
hung vertically from the branch of a tree. A com- m/s) of the object C. [see Fig. 7.39]
pletely inelastic collision occurs in which the stone
gets embedded in the object. Find the speed in ms–1
of the combined mass immediately after the colli-
sion with respect to an observer on the ground. Fig. 7.39
IIT, 1997 IIT, 2009
SOLUTIONS
1. Refer to Fig. 7.40. where e
p1 = 1 kg 2 ms –1
= 2 kg ms –1 of the bob after the second rebound will be v2 = ev1
–1 = e2v. Thus, the speed of the bob after n rebounds
p2 = p1 = 2 kg ms will be
v n = env
p= p12 p22 = 2 2 kg ms–1
If the ball rises to a position C at a height h = BD
From the conservation of momentum, we have after n rebounds, then from the principle of conser-
p3 p =0 vation of energy, we have
1
p3 = – p = 2 2 kg ms–1 mvn2 = mgh
2
2
2 v= 2 2 v=– 2 ms–1 vn2 en v e2n v2
h= (2)
Total energy released 2g 2g 2g
1 1 1 2 Using Eq. (1) in (2), we get
= 1 22 1 2 2
2 2
2 2 2 2 gle2 n
h= le2 n (3)
= 2+2+2=6J 2g
For n is the angle the string subtends with vertical
after n
h = OB – OD = l – l cos n = l (1 – cos n) (4)
From Eqs. (3) and (4), we have
le2n = l (1 – cos n) or e2n = 1 – cos n)

Fig. 7.40

2. Refer to Fig. 7.41. When the bob is at the horizon-


tal position A, its height above B = length of the
string = l. Therefore, potential energy at A = mgl.
When the bob is released, it hits the wall at B and
the entire potential energy is converted into kinetic
energy. If v is the velocity with which the bob hits Fig. 7.41
the wall, then For n to be less than 60°, i.e. cos n is greater than
1 1 1
mv2 = mgl or v = 2gl (i) , (1 – cos n) must be less than . Thus
2 2 2
1
v1 = ev e2n <
2
7.38 Comprehensive Physics—JEE Advanced

2 4. Elastic collision of A with B. (Fig. 7.43)


Given, e = . Therefore,
5
2n n
2 1 4 1
< or <
5 2 5 2
1
or (0.8)n < or n log (0.8) < log (0.5)
2
0.3010 v v
or – (0.0969)n < – 0.03010 or n > or
n > 3.1 0.0969

The least integer greater than 3.1 is 4. Hence n = 4. Fig. 7.43


3. The x, y and z components of the initial velocity of Momentum conservation gives
the stone are (Fig. 7.42)
mu1 + 2 mu2 = mv1 + 2 mv2
ux = 4 ms–1
uy = (6 ms–1) sin 30° = 3 ms–1 9m + 0 = mv1 + 2 mv2

uz = (6 ms–1) cos 30° = 3 3 ms–1 9 = v1 + 2 v2 (1)


When the stone reaches the highest point A of its Conservation of kinetic energy gives
parabolic path, the vertical i.e. z component of its 1 1 1
velocity becomes zero but the x and y components m (9)2 = mv12 2m v22
remain unchanged, i.e. 2 2 2
vx = ux = 4 ms–1 81 = v12 2v22 (2)
vy = uy = 3 ms–1
Solving (1) and (2), we get we get v1 = – 3 ms–1 and
vz = 0 v2 = 6 ms–1
The speed of the stone at the highest point on its Inelastic collision between B and C (Fig. 7.44)
trajectory with respect to an observer on the ground
is In a perfectly inelastic collision the two bodies stick
together.
v = (v2x + v2y + v2z)1/2
= (42 + 32 + 02)1/2 = 5 ms–1
v v

6 ms–1

v
30°

Fig. 7.44

Momentum conservation gives


4 ms–1 2 mv2 + 0 = (2m + m) v
2 2
v= v2 = 6 = 4 ms–1
3 3
Fig. 7.42
8
Chapter
Rigid Body Rotation

REVIEW OF BASIC CONCEPTS


8.1
8.1 CENTRE OF MASS OF DISCRETE PARTICLES Three particles of masses m1 = m, and m2 = 2m and
m3 = 3m are placed at the corners of an equilateral
triangle of side a as shown in Fig. 8.1. Locate the
that point where the entire mass of the system is imagined centre of mass of the system.
to be concentrated, for considerations of its translational
motion.
If r1, r2, r3, ... rn are the position vectors of masses
m1, m2, m3, ...mn respectively, the centre of mass of the
system is
m r m2r2 m3r3 ......... mn rn
rCM = 1 1
m1 m2 m3 ........ mn
n n
mn rn mn rn
N 1 N 1 Fig. 8.1
= n
=
M
mn
N 1
SOLUTION
where M is the total mass of the system of particles. Take any one particle to be located at the origin O.
rCM is the weighted average of all the position vectors The x and y coordinates of m1, m2, and m3 respectively
of the particles of the system, the contribution of each a
are x1 = 0 and y1 = 0, x2 = a and y2 = 0 and x3 = and
particle being proportional to its mass. 2
For a system consisting of two particles, the centre of 3a
y3 =
mass is 2
m r m2r2
rcm = 1 1 The x and y coordinates of the centre of mass are
m1 m2
If the masses are equal, i.e. m1 = m2, then xCM = m1 x1 m2 x2 m3 x3
m1 m2 m3
1
rCM = (r1 + r2) a
2 m 0 2m a 3m
which means that the centre of mass lies exactly in the 2 7a
=
middle of the line joining the two masses. m 2m 3m 12
8.2 Comprehensive Physics—JEE Advanced

m1 y1 m2 y2 m3 y3 8.3
and yCM =
m1 m2 m3 Locate the centre of mass of a uniform rod of mass
3a M and length L.
m 0 2m 0 3m
= 2
m 2m 3m

3a
= Fig. 8.3
2

8.2 SOLUTION
Four particles of masses 1 kg, 1 kg, 2 kg, and 2 kg, We assume that the rod lies along the x-axis with one
are placed at the corners of a square of side 12 cm end at origin O.
as shown in Fig. 8.2. Find the position vector of the M
centre of mass of the system. Mass per unit length of the rod =
L
Mass of element of length dx is [Fig 8.3]
M
dm = dx
L
The x-coordinate of the centre of mass is
L
x dm L
xCM = 0 1 M L
x dx
L
M 0
L 2
Fig. 8.2 dm
0
SOLUTION The centre of mass is at the centre of the rod.
1 0 1 12 2 12 2 0
xCM = = 6 cm
1 1 2 2 8.4
1 0 1 0 2 12 2 12 A non-uniform rod of length L is lying along the
yCM = = 8 cm x-axis with one end at origin O as shown in Fig. 8.4.
1 1 2 2
The linear mass density (i.e. mass per unit length)
The position vector of the centre of mass is varies with x as = a + bx, where a and b are
constants. Find the distance of the centre of mass
rCM = 6 i 8 j cm
from origin O.
CENTRE OF MASS OF A BODY HAVING
8.2 CONTINUOUS DISTRIBUTION OF MASS
If a body has continuous distribution of mass, the position
of its centre of mass is determined by dividing the body Fig. 8.4
into a very large number of extremely small elements. If
dm is the mass of the element and it is at a distance x and SOLUTION
y from the origin of chosen coordinate system, then x and
y coordinates of the centre of mass are given by Mass of element is dm = dx = (a + bx) dx
x dm L
xCM = x dm
dm
xCM = 0 (i)
L
y dm
and yCM = dm
dm 0
Rigid Body Rotation 8.3

L L
x dm = a bx x dx R2 sin d
0 0
ydm 0 R2
yCM = | cos |0
L L dm R
2
= a x x3 aL2 bL3 (ii)
R d
b 0
2 0 3 0 2 3
R 2R
L
L2 =– cos cos 0
x dm = 3a 2bL
6
0 2R
L L 2
Thus, the centre of mass is at a distance of
And bL L from origin O on the y-axis.
dm = a bx dx aL 2a bL
0 0
2 2
FINDING CENTRE OF MASS OF A SYSTEM
(iii) 8.3 WHEN A PART OF ITS MASS IS REMOVED
Using (ii) and (iii) in (i), we get
L 3a 2bL Consider of system of mass M. If a mass m is removed, the
xCM = remaining mass = M – m which may written as M + (–m).
3 2a bL
Then the x and y coordinates of the centre of mass of the
8.5 remaining portion are given by
Locate the centre of mass of a uniform semicircular xCM = Mx mx
ring of radius R and linear mass density . M m

SOLUTION and yCM = My my


M m
Let us take the centre of the ring at origin O. consider
where x and y are the coordinates of the centre of mass of
a small element of arc length dl of the ring. Let
the complete part and x and y are the coordinates of the
be the angle which the radius vector of the element
centre of mass of the removed part.
makes with the x-axis as shown in Fig. 8.5.
8.6
Four particles, each of mass 1 kg, are placed at the
corners of a square of side 12 cm. If mass m3 is

system (see Fig. 8.6).

Fig. 8.5

Let d be the angle subtended by the element at


the centre. Then dl = R d . Mass of the element
is
dm = dl = Rd Fig. 8.6
The x and y components of radius vector R are
x = R cos and y = R sin . Then SOLUTION
As shown in Example 2, the x and y coordinates of
R2 cos d the original system are xCM = 6 cm and yCM = 6 cm
x dm 0 R2 If mass m3 is removed, the x and y coordinates of
xCM = | sin |0 the centre of mass of the remaining system are
dm R
R d
0 x CM = 1 0 1 12 1 0 = 4 cm
R 1 1 1
= (sin – sin 0) = 0
8.4 Comprehensive Physics—JEE Advanced

The x and y coordinates of the remaining portion


y CM = 1 0 1 0 1 12 = 4 cm (shown shaded) are
1 1 1
Mx mx
xCM =
Thus r = 6i 6j M m
M R
M 0
and r = 4i 4j = 4 2 R
M 6
M
shift r = r r 6i 6j 4i 4j 4
M
M 0 0
= 2i 2j yCM = My my 4 0
M m M
M
4
8.7
The negative sign of xCM indicates that the centre of
From a uniform thin disc of radius R and mass M, a mass of the remaining portion is located at a distance
circular portion of radius r =R/2 is removed as shown R/6 towards the left of the center O of the complete
in Fig. 8.7. Find the centre of mass of the remaining disc.
part of the disc.
Alternative Method
It is clear that centre of mass of the remaining portion of
the disc will shift to the left of O. Let G be the centre of
mass of the remaining portion of the disc (Fig. 8.8). Let
OG = x. Equating the moments of Mg and mg about G,
we have

Fig. 8.7

SOLUTION
Let the centre of the disc be at origin O. The centre of
mass of the complete disc will be at O (by symmetry)
and its x and y coordinates are (0, 0). Mass per unit Fig. 8.8
M OG = mg
area of the disc = . Therefore, mass of removed Mg OG
R2
portion is M R
M x= x
M 4 2
m= 2
r2 .
R R
x=
2 6
=
M R M
. Thus the centre of mass shifts by R/6 to the left of O.
2
R 2 4
8.8
The x and y coordinates of the centre of mass of
the removed portion are From a thin uniform square lamina of side a a square
of side a/2 is removed from its corner as shown in
R Fig. 8.9. Find the centre of mass of the remaining
x = r and y 0
2 portion (shown shaded) of the lamina.
Rigid Body Rotation 8.5

The velocity of the centre of mass is given by


drCM
vCM =
dt
dr dr dr
m1 1 m2 2 ... mn n
dt dt dt
=
m1 m2 ... mn
m1 v1 m2 v 2 ... mn v n
vCM
m1 m2 ... mn
The acceleration of the centre of mass is given by
d v CM
aCM =
Fig. 8.9 dt
dv d v2 d vn
m1 1 m2 mn
SOLUTION = dt dt dt
Let M be the mass of the complete lamina. Mass of m1 m2 mn

the removed portion is m


M
. The centre of mass aCM = m1a1 m2a 2 mn a n
4 m1 m2 mn
of the complete lamina is taken to be at origin O (x = Fext
F1 F2 Fn
0, y = 0). The coordinates of the centre of mass O of or aCM =
a a m1 m2 mn M
the removed portion are x = and y .
4 4
If Fext 0, then aCM = 0, i.e vCM = constant. Hence if no
The x and y coordinates of the centre of mass of the
remaining portion are net external force acts on a system, its centre of mass will
remain at rest or will move with a constant velocity.
Mx mx
xCM =
M m 8.9
M a A boy of mass m = 50 kg stands at the end A
M 0 plank AB of wood of mass M = 100 kg and length l =
= 4 4 a
M 12 lake. The end B of
M the plank is at a distance of 30 m from the shore of the
4
lake as shown in Fig. 8.10. The boy walks a distance
and yCM =
My my of 6 m on the plank towards the shore. How far is the
M m boy from the shore now? Neglect viscosity of water.
M a
M 0
= 4 4 a
M 12
M
4

VELOCITY AND ACCELERATION OF CENTRE


8.4 OF MASS OF A SYSTEM OF PARTICLES Fig. 8.10
If r1, r2, ... rn are the position vectors of masses m1, m2, ...
mn, respectively, th\e position vector of the centre of mass SOLUTION
of the system of particles is given by Initially the centre of mass of the system (plank +
boy) is at rest. To walk, the boy exerts a force in
rCM = m1r1 m2r2 mn rn
m1 m2 mn the backward direction. The plank in turns exerts a
reaction force on the boy in the forward direction.
8.6 Comprehensive Physics—JEE Advanced

These forces are internal to the system. Since no exter- dP


nal force acts, the centre of mass of the system remains Fext =
dt
at rest even when the boy walks on the plank.
where P = MvCM is the total linear momentum of the
Let the shore be at the origin O (x = 0). Initially let
system of particles which is equal to the product of the
x be the distance of the centre of mass of the plank
total mass of the system and the velocity of the centre of
from O. Then the distance of the centre of mass of the
mass.
system (plank + boy) from O will be
dP
M x m 30 10 If Fext = 0, =0 P = constant.
xCM = dt
M m
Thus, if no net external force acts on a system, the total
100 x 50 40 100 x 2000 linear momentum of the system remains constant; the
= =
100 50 150 total linear momentum being the vector sum of linear
Since the boy moves towards the shore and the momentum of individual particles, i.e.
centre of mass of the system has to remain at rest, P = P1 + P2 + + Pn
the plank will move away from the shore. If x is
the distance moved by the plank, the distance of the
centre of mass from the shore when the boy walks 8.11
6 m on the plank is given by A boy of mass m = 40 kg is standing on a stationary
100( x x ) 50(40 6 x) long plank of mass M
xCM = in a lake. He starts running with a velocity v = 6 ms–1
100 50 relative to the plank. Find the velocity of the boy rela-
100 x 150 x 1700 tive to a stationary observer on the bank of the lake.
=
150
SOLUTION
Since xCM = xCM,
Since the system (boy + plank) is initially at rest, its
100 x 2000 100 x 150 x 1700 momentum is zero. Since no external force acts on
=
150 150 the system, the momentum of the system will remain
300 zero. Let us assume that the boy runs in the positive
x = =2m x-direction.
150
Distance of the boy from the shore = 40 – 6 + 2 Velocity of boy relative to the plank is
= 36 m.
vbp = v i
8.10 Now vbp= vb – vp
A car of mass 1000 kg is moving with a velocity of where vb = velocity of boy relative to ground
10 ms–1 towards another car of mass 1500 kg moving
and vp = velocity of plank relative to ground
with a velocity of 15 ms–1 in the same direction. Find
the velocity of the centre of mass of the two cars. Hence vb= vbp + vp

SOLUTION = vi vp i

vCM = m1v1 m2 v2 = (v vp ) i
m1 m2
1000 10 1500 15 Total momentum of plank + boy = 0
= = 13 ms–1
1000 1500 Mvp + mvb= 0
MOMENTUM CONSERVATION AND
8.5 Mvp i + mvb i = 0
CENTRE OF MASS MOTION
We have seen that Mvp i + m (v + vp) i = 0
d v CM d mv
Fext = MaCM M M v CM which gives vp =
dt dt M m
Rigid Body Rotation 8.7

Velocity of boy is

mv Mv
vb = v i = i
M m M m
260 6
= i
260 40 Fig. 8.12
1
= 5.2 i ms = magnitude of either force perpendicular distance
Hence the velocity of boy relative to a stationary observer is between the two antiparallel forces
5.2 ms–1 in the direction along which the boy is running.
Work done by torque
If a force F acts on a rigid body at perpendicular r from
8.6 TORQUE the axis of rotation, the work done by the force in rotating
If a force F acts on a particle P whose position vector with the body through an angle is given by
respect to the origin of an inertial reference frame is r, the W = Fr =
torque acting on the particle with respect to the origin is
= magnitude of torque angular displacement
=r F dW d
Power = = =
In terms of magnitudes, dt dt
where is the angular velocity.
= r F sin = F (r sin ) = Fr
where is the angle between vectors r and F.
8.12
Torque is a vector quantity. Its magnitude is given by
= rF sin ; its direction is normal to the plane containing
vectors r and F and can be determined by the right–hand A force F = 2 i 3 j newton acts on a particle
screw rule. whose position vector with respect to origin O
Unit of Torque Torque has the same dimensions as those is r = 4 i 5 j metre. Find the magnitude and
of work (both being force times distance) viz. ML2T–2. direction of the torque.
The two are, however, very different quantities. Work is a
scalar, torque is a vector. To distinguish between the two we SOLUTION
express work in joule and torque in newton–metre (N m).
=r F

= 4i 5j 2i 3j

=4i 2i 12 i j 10 j i 15 j j

= 0 12 k 10 k 0

= 22 k newton metre
The magnitude of torque is 22 Nm and its direction
is along the positive z-axis.

8.13
Fig. 8.11 A rectangular plate OPQR of dimensions 2 m 3m
Couple
Two equal antiparallel forces having different lines of lies in the x-y plane as shown in Fig. 8.13. A force
action constitute a couple.
The moment of couple or torque = Fr (Fig. 8.12) F = 3i 5 j newton is applied at point Q. Find the
8.8 Comprehensive Physics—JEE Advanced

torque of F (a) about origin O, (b) about point P and where r is the position vector of the particle and p its
(c) about x-axis, y-axis and z-axis. linear momentum. In terms of magnitudes,
L = rp sin r p
where is the angle between vectors r and p. The
dimensions of angular momentum are (ML2 T–1) and its
SI unit is kg m2s–1.
The direction of L is perpendicular to the plane
containing the vectors r and p and its sense is given by
the right–hand rule.

Fig. 8.13

SOLUTION

(a) r = OQ = 3i 2 j metre
Torque about O is

O =r F
Fig. 8.14
= 3i 2j 3i 5j
8.14
= 9i i 15 i j 6j i 10 j j
A body of mass m = 200 g is moving parallel to the
x-axis with a velocity v = 30 cms–1 in the x-y plane
= 0 15 k 6k 0 9 k Nm
as shown in Fig. 8.15. Calculate the magnitude of its
(b)Torque about P is angular momentum about origin O at any time t.

P = PQ F

= 2j F

= 2j 3i 2j = – 6 k Nm
(c)Torque about x-axis is
Fig. 8.15
x= i = 9k i = 0
Torque about y-axis is SOLUTION
Magnitude of angular momentum = r p
y = j = 9k j = 0 = r mv
Torque about z-axis is = OB m v
= 0.15 0.2 0.3
z = z O k = 9k k = 9 Nm = 9 10–3 kg m2s–1

8.7 ANGULAR MOMENTUM 8.15


A body of mass m = 200 g is projected with a velocity
The angular momentum L of a particle P with respect to u = 5 ms–1at an angle = 30° with the horizontal.
Calculate the magnitude of the angular momentum of
8.14] the body about the point of projection when it is at the
L= r p highest point of its trajectory. Take g = 10 ms–2.
Rigid Body Rotation 8.9

SOLUTION dp
= dt
dt
= d p = change in linear momentum
In rotational motion, angular impulse J
J= dt

dL
= dt
Fig. 8.16
dt
= dL = change in angular momentum
Refer to Fig. 8.16. At the highest point A, the body
has only horizontal velocity
LAW OF CONSERVATION OF ANGULAR
v = u cos 8.10 MOMENTUM
u 2 sin 2
hmax = If no external torque acts, the total angular momentum of
2g a body or a system of particles is conserved.
Magnitude of angular momentum of the body about We have seen that the rate of change of angular
O when it is at point A is momentum of a particle is equal to the torque produced
L = mv OB by the total force. If, in a certain situation, the torque
= mv hmax itself vanishes, then it follows that the angular momentum
of the particle will remain constant. This is the law of
u 2 sin 2 conservation of the angular momentum of a particle. One
= m u cos
2g trivial situation is when the force vanishes. Then the torque
mu3 sin 2 cos vanishes too. The particle then moves freely in a straight
=
2g
both linear and angular momenta are conserved.
3
0.2 5 sin 2 30 cos 30 A general situation is when the torque vanishes without
=
2 10 the force itself vanishing. The torque will vanish if the
component F (the angular component) of F vanishes but
1 3
0.2 125 the radial component FII does not. The radial component FII
= 4 2 is the component of F along the radius (or position) vector
20 r. Hence, if the force acting on the particle is purely radial
= 0.27 kg m2s–1 (i.e. if it is directed along or against its position vector) then
the torque acting on the particle vanishes and its angular
RELATION BETWEEN TORQUE AND momentum is conserved and so is its areal velocity.
8.8 ANGULAR MOMENTUM
8.11 MOMENT OF INERTIA
In linear motion, the relation between force F and linear
momentum p is The moment of inertia of a rigid body about a particular
dp axis may be
F= of all the particles constituting the body and the squares of
dt
their respective distances from the axis of rotation, i.e.
In rotational motion, the relation between torque and
angular momentum L is I = m1r 21 + m2r 22 + m3r 23 + + mnr2n
dL n
= = mn rn2
dt
N=1
which states that the torque acting on a particle is equal
Its value depends upon the particular axis about which
to the rate of change of angular momentum.
the body rotates and the way the mass is distributed in the
body with respect to the axis of rotation.
8.9 ANGULAR IMPULSE
In the case of a body which does not consist of separate,
In linear motion, impulse I discrete particles but has a continuous and homogeneous
distribution of matter in it, the summation is replaced by
I= F dt
integration, so that
8.10 Comprehensive Physics—JEE Advanced

I= r 2d m MOMENT OF INERTIA AND ANGULAR


8.15 MOMENTUM
where dm
of the body at a distance r from the axis of rotation. The magnitude of angular momentum of a rotating body
Moment of inertia is a scalar quantity. Its SI unit is is given by L = I
kg m2 and its dimensions are (ML2).
8.16 PARALLEL AXES THEOREM
8.12 RADIUS OF GYRATION
If M is the total mass of a body and h the distance
The radius of gyration of a body about its axis of rotation between two parallel axes, then according to parallel axes
theorem I = ICM + Mh2
which, if the entire mass of the body were concentrated,
its moment of inertia about the given axis would be the 8.17 PERPENDICULAR AXES THEOREM
same as with its actual distribution of mass. It is usually
denoted by the letter K. If M is the mass of the body, its The theorem of perpendicular axes for a body of plane
moment of inertia I in terms of its radius of gyration K can lamina states that the moment of inertia of a plane
be written I = MK2 lamina about an axis perpendicular to the plane of the
lamina is equal to the sum of the moments of inertia
MOMENT OF INERTIA AND ROTATIONAL of the lamina about any two mutually perpendicular
8.13 KINETIC ENERGY axes in its own plane and intersecting each other at
the point where the perpendicular axis passes through
Kinetic energy of a rotating body is related to moment of it.
inertia as If Ix and Iy are the moments of inertia of a plane lamina
1 about the perpendicular axes x and y respectively which
KE = I 2
2 lie in the plane of the lamina and intersect each other at O,
where is the angular velocity (or frequency) of the body. then the moment of inertia I of the lamina about an axis
passing through O and perpendicular to its plane is given
8.14 MOMENT OF INERTIA AND TORQUE by
I = Ix + Iy
The magnitude of torque is given by = I
where is the angular acceleration of the body.

EXPRESSIONS FOR MOMENT OF INERTIA OF BODIES OF REGULAR SHAPES ABOUT


8.18 PARTICULAR AXES OF ROTATION

Shape of body Axis of Rotation Expression for Moment of Inertia


1. Circular ring of mass (i) through centre, perpendicular to plane of ring M R2
M and radius R (ii) any diameter (1/2) M R2
(iii) any tangent in the plane of ring (3/2) M R2
(iv) any tangent perpendicular to plane of ring 2 M R2
2. Circular disc of mass M (i) through centre, perpendicular to plane of disc (1/2) M R2
and radius R (ii) any diameter (1/4) M R2
(iii) tangent in the plane of the disc (5/4) M R2
(iv) tangent perpendicular to plane of disc (3/2) M R2
3. Sphere of mass M and (i) any diameter (2/5) M R2
radius R (ii) any tangent plane (7/5) M R2
4. Cylinder of mass M, (i) own axis (1/2) M R2
radius R and length L R2 L2
(ii) through centre perpendicular to length M
4 12

R2 L2
(iii) through end faces and to length M
4 3
Contd.
Rigid Body Rotation 8.11

Shape of body Axis of Rotation Expression for Moment of Inertia


5. One dimensional rod (i) centre of rod and to length M L 2/12
of mass M and length L
(ii) one end and to length M L2/3
6. Rectangular lamina of (i) length of lamina and in its plane M B2/3
mass M, length L and (ii) breadth of lamina and in its plane ML2/3
breadth B (iii) centre of lamina and parallel
M B 2 or M L2
to length or breadth in its plane 12 12

L2 B2
(iv) centre of lamina and to its plane M
12

L2 B2
(v) centre of length and to its plane M
12 3

L2 B2
(vi) centre of breadth and to its plane M
3 12
7. Rectangular block of mass (i) through centre or block and parallel to length or
B2 H2 or
M, length L, breadth B and breadth or height of the block M
12
height H
H 2 L2 or M L
2
B2
M
12 12

(ii) through end face and


H2 B2 or
parallel to length or breadth M
3 12
or height of the block L2 H2 2
L2
M or M B
3 12 3 12

KINEMATICS OF ROTATIONAL (a) about any diameter


(b) about an axis passing through a point on the
8.19 MOTION WITH CONSTANT ANGULAR
edge of the disc and perpendicular to the disc
ACCELERATION (c) about a tangent in the plane of the disc.
Consider a body rotating with an initial angular velocity 0.
It is given a constant angular acceleration (by applying a SOLUTION
constant torque) for a time t The plane of the disc is the x-y plane.
angular velocity and suffers an angular displacement
(a) Using perpendicular axes theorem [Fig. 8.17(a)]
in time t. The equations of rotational motion are
= 0+ t Ix + Iy = Iz

1 2 Now Iz = IC = 1 MR2 (given). From symmetry


= 0t t 2
2 I y = I x.
2 2
and 2 =
2Ix = 1 MR2 Ix = Iy = 1 MR2
0

2 4
8.16 (b) using parallel axes theorem [Fig. 8.17(b)]
The moment of inertia of a uniform circular disc of
IAB = Iz + M(CD)2
mass M and radius R about an axis passing through its
1 = IC + MR2
centre and perpendicular to its plane is MR2 . Find
2 1 3
the moment of inertia of the disc = MR2 MR2 = MR2
2 2
8.12 Comprehensive Physics—JEE Advanced

(c) using parallel axes theorem [Fig. 8.17(c)]


IEF = Iy + MR2
1 5
= MR 2 MR 2 = MR 2
4 4

Fig. 8.17

M R4 r4 1
8.17 = = M R2 r2
A thin uniform disc of mass M and radius R has con- 2 R2 r 2
2
centric hole of radius r. Find the moment of inertia of
the disc about an axis passing through its centre and 8.18
perpendicular to its plane. Find the moment of inertia of a thin uniform rod of
mass M and length L about an axis passing through its
SOLUTION centre and making an angle with the rod.
Mass per unit area of the
SOLUTION
disc is [Fig. 8.18]
Divide the rod into a very large number of extremely
M small elements each of length dx. Consider one such
m=
R 2
r2 element at a distance x from the centre O of the rod
(Fig. 8.19).
Mass of the disc if it was
complete (i.e. without
hole) is
M1 = m R2
Fig. 8.18
2
M M R
= R2 =
R 2
r 2
R2 r2

Mass of the removed portion is

Mr 2
M2 = m r2 =
R2 r2 Fig. 8.19
M
Since the two portions are concentric, the moment of Mass of element is dm = dx
inertia of the given disc about the given axis is L
Perpendicular distance of the element from the axis
1 1 of rotation is
I= M1R 2 M2 r2
2 2 r = OA = x sin
Moment of inertia of the rod about the given axis is
1 MR4 Mr 4
= M
2 R2 r 2 R2 r2 I = dm r 2 = dx x sin 2
L
Rigid Body Rotation 8.13

x L where is in radian [Fig 8.22].


M sin 2 2
2
= x dx
L x L
2

L/2
M sin 2 x3
=
L 3 L/2
2
ML
= sin 2
12
ML2 Fig. 8.22
If = 90°, I =
12
M
Mass per unit area =
8.19 A
The radius of gyration K of a hollow sphere of mass 2M
=
M and radius R about a certain axis is equal to R. R2
Find the distance of that axis from the centre of the 2
Area of strip is dA = x dx x2
sphere. 2

SOLUTION 2 xdx = (xdx)


2
Let x be the distance of the 2M 2M
axis from the centre of the Mass of strip is dm = 2
dA = xd x
R R2
sphere [Fig. 8.20]. From par-
allel axes theorem 2M
xd x=
IXY = IAB + Mx2 R2
Moment of inertia of the section about the given
2 axis is
MK2 = MR 2 Mx2
3 2M
R
2
Given K = R. Hence I = dmx = 2
x3 dx
R 0
2 2 1
R2 = R x2 = MR 2
3 Fig. 8.20
2
R
x= 8.21
3
Two particles of masses m1= 1 kg and m2 = 2 kg
are connected by a rigid bar of length L = 1.2 m of
8.20 negligible mass. The system rotates about an axis
Figure 8.21 shows perpendicular to the rod and at a distance x from
a section (a part) of mass m1. Find the value of x for which the moment
circular disc of radius of inertia about the given axis is minimum. what is
R. The mass of the the minimum moment of inertia?
section is M. Find the
moment of inertia of SOLUTION
the section of the disc From Fig 8.23, it follows that
about an axis passing
through its centre O
and perpendicular to Fig. 8.21
its plane.

SOLUTION
R2
Area of the section A = R2 = Fig. 8.23
2 2
8.14 Comprehensive Physics—JEE Advanced

I = m1x2 + m2 (L – x)2 (i) = a + b is applied to it, where is the angular


2 displacement and a and b are positive constants.
dI d I
I will be minimum if = 0 and 0 Obtain the expression for the angular velocity of the
dx d x2 disc as a function of .
Differentiating (i)
dI SOLUTION
= 2m1x + 2m2 (L – x) (–1) = 0 (ii)
dx =I
m2 L 2 1.2
x= = = 0.8 m or =
m1 m2 1 2 I
Differentiating (ii) d a b
2
=
d I dt I
= 2m1 + 2m2, which is positive for any value of x.
dx2 d d a b
=
Substituting x in (i), we have d dt I
2 2 a b a b
m2 L m2 L d = d = d d
Imin = m1 m2 L I I I
m1 m2 m1 m2 Integrating
2
m1m2 L2 1 2 1.2
= = = 0.96 kg m2 d =
a
d
b
d
m1 m2 1 2 I I
0 0 0
2
8.22 a 2 b
=
A giant wheel of radius 2.0 m and mass 100 kg is 2 I 2 I
initially at rest. (a) What torque should be applied
1 2
to it so that it acquires an angular frequency of 300 = a 2b
r.p.m. in 10 s? (b) Find the kinetic energy when it is I
rotating at 300 r.p.m. 2 2
= 2
a 2b
SOLUTION MR
300 1
Given 0 = 0, = 300 r.p.m. = = 5 Hz. Therefore I MR 2
60 2
=2 = 10 rad s–1 and t = 10 s
8.24
Using = 0 + t, we have 10 = 0 + 10
A thin uniform rod AB of mass M and length L is
= rad s–2 hinged at one end A
1 it stands vertically. It is allowed to fall freely in a
(a) Torque required is = I = MR2 vertical plane.
2
1 (a) What is the angular acceleration of the rod when
= 100 22 it is at an angle with the vertical?
2
= 200 = 628 Nm (b) With what linear speed will the end B hit the

1 2 1 1 2
(b) K.E. = I = 100 22 10
2 2 2 SOLUTION
= 104 2
= 9.9 104 J (a) The entire mass of the rod acts at its centre of
mass C. AC = L/2. The magnitude of the torque
8.23 due to weight Mg is [Fig. 8.24]
A stationary horizontal uniform disc of mass M and = Mg r
radius R is free to rotate about an axis passing through L
its centre and perpendicular to its plane. A torque = Mg AD = Mg sin
2
Rigid Body Rotation 8.15

(b) Find the magnitude of the angular momentum of


the particle about O at this time t.
dL
(c) Show that, in this example, =
dt
SOLUTION

(a) The torque is due to force of gravity F = mg. The


magnitude of the torque of F about O is
= rF sin = r mg sin
x x0
Fig. 8.24 = rmg 0 sin
r r
ML2 = mg x0
Moment of inertia of the rod about A is I =
3 From right hand rule, the direction of the torque is
MgL sin 3 into the page .
Angular acceleration = =
I 2 ML2 (b) The magnitude of angular momentum about O is
3g sin L = rp sin = rmv sin
= From v = u + at we have v = 0 + gt = gt. Therefore
L
(b) When the end B - L = r m gt
x0
= mg x0t
tance through which C falls is L/2. From the law r
of conservation of energy, dL d
(c) = mgx0 t = mgx0 =
Loss in P.E. = gain in K.E dt dt
L 1 2 1 ML2 2
Mg = I = 8.20 ROLLING MOTION WITHOUT SLIPPING
2 2 2 3
3g (1) Total Kinetic Energy
=
L The total kinetic energy of a body which is moving as well
Linear speed of end B = L = 3gL as rotating is equal to the sum of translational K.E. and
rotational K.E., i.e.
8.25 1 1
K = m v2 I 2
A particle of mass m is released from rest at point P 2 2
located at a distance x0 from origin O on the x-axis where m = mass of the body, v = linear velocity of its
as shown in Fig. 8.25. It falls vertically along the centre of mass, I = moment of inertia of the body about an
negative y-axis. axis passing through its centre of mass and = angular
velocity of rotation.
(2) Instantaneous Velocity of a point on a Rolling
Body
Consider a wheel of radius R(= AC = BC) rolling without
slipping on a horizontal rough surface (Fig. 8.26).

Fig. 8.25

(a) Find the magnitude and direction of the torque


acting on the particle at time t when it reaches
point Q whose position vector with respect to O
is r. Fig. 8.26
8.16 Comprehensive Physics—JEE Advanced

Every point on the wheel has instantaneous velocity. F


For a point P at a distance r from the centre of mass C, For a hollow cylinder I = MR2 aCM =
2M
the instantaneous velocity is the vector sum of velocity v
of the centre of mass and tangential velocity vt of point P 2 5F
For a solid sphere I = MR 2 aCM =
relative to the centre of mass, i.e. 5 7M
vP = v + vt 2 3F
For a hollow sphere I = MR 2 aCM =
where vt = r . Vector vt is directed along the tangent to the 3 5M
circle of radius r about C. If the force is applied tangentially to the body as shown
For point A, vt = R and v = r . Since these velocities in Fig. 8.28, then
are in the same direction, the instantaneous velocity of
A is
vA = v + vt = R + R = 2 R
For point B in contact with the horizontal surface,
vB = v + vt = R – R = 0
If a body rolls on a surface without slipping, the instan-
taneous velocity of the point of contact with the surface Fig. 8.28
is zero.
2F
(3) A body Rolling without slipping on a Rough aCM =
I
Horizontal surface M 1
MR 2
Horizontal force F is applied at the centre of mass of a
I
body (disc, ring, cylinder or sphere) of mass M and radius F 1
R (Fig. 8.27) on a rough and f= MR 2
I
horizontal surface. If f is 1
the frictional force, and the MR 2
body rolls without slipping (4) A body Rolling without slipping on a Rough
aCM = R. Inclined Plane
F – f = MaCM (i) A body (ring, disc, cylinder or sphere) of mass M and radius
= fR = I R is rolling (without slipping) down a rough inclined plane
of inclination (Fig. 8.29).
I I aCM Fig. 8.27
f= = 2
(ii)
R R
From (i) and (ii), acceleration of centre of mass is
F
aCM =
I
M 1
MR2
F Fig. 8.29
and f= 2
MR For linear motion parallel to the plane
1
I Mg sin – f = Ma (i)
where a = linear acceleration of the centre of mass.
1 2F
For a disc I = MR2 aCM = For rotational motion about the axis through the centre
2 3M of mass
F =I
For a ring I = MR2 aCM = I I
2M Rf = I f= =( a=R )
R R2 (ii)
1 2F
For a solid cylinder I = MR2 aCM =
2 3M I is the moment of inertia about the centre of mass.
Rigid Body Rotation 8.17

Using (i) and (ii), we get


SOLUTION
g sin The acceleration of the cylinder is
a=
I
1 g sin 2 g sin
MR 2 a= =
I 3
g sin 1
For a ring I = MR2 a= MR 2 1
2 I MR 2
2
1 2 g sin
For a disc I = MR2 a=
2 3 Distance travelled is s = AB =
h
1 2 g sin sin
For a solid cylinder I = MR2 a= 2 2
2 3 (a) Using v – u = 2as, we have
g sin
For a hollow cylinder I = MR2 a= 2 g sin h
2 v2 – 0 = 2
3 sin
2 3g sin
For a hollow sphere I = MR 2 a= 4 gh
3 5 v=
2 5 g sin 3
For a solid sphere I = MR 2 a=
5 7 Speed v can also be found from the law of conserva-
tion of energy. As the cylinder moves from A to B, it
Ia Mg sin
Frictional force is f = 2
= loses P.E. and gains K.E.
R MR 2
1 Loss in P.E. = gain in K.E.
I
1 1
Condition for rolling without slipping or Mgh = = Mv2 + I 2
2 2
To prevent slipping, f μN, where μ
1 v
of static friction between the body and the plane and Now I= MR2 and = . Therefore
N = Mg cos is the normal reaction. 2 R
Hence to avoid slipping, 1 1 1 v 2
Mgh = Mv2 + MR2
Mg sin 2 2 2 R
μ Mg cos
MR 2 3
1 = M v2
I 4
tan
μ 4 gh
MR 2 v=
1 3
I
(b) From v = u + at, we have

8.26 4 gh 2 g sin
=0 + t
3 3
A solid cylinder of mass M and radius R is released
from rest from top A of an inclined plane of height h 3h 1
t=
and inclination as shown in Fig. 8.30. The cylinder g sin
rolls without slipping. Find (a) the speed at which it
reaches bottom B of the plane and (b) the time it takes 8.27
to reach B.
A billiard ball has mass M = 250 g and radius
R = 2.5 cm and is
initially at rest. A rod
held horizontal at a
height h above centre
C hits the ball. The
ball begins to roll
without slipping. Find
the value of h [see Fig.
Fig. 8.30 8.31]. Fig. 8.31
8.18 Comprehensive Physics—JEE Advanced

SOLUTION 8.29
The horizontal force F imparts a linear impulse A uniform rod AB of length L = 1 m is sliding along
I= Fdt = change in linear momentum two mutually perpendicular surfaces OP and OQ as
shown in Fig. 8.32. When the rod subtends an angle
I = Mv – 0 = Mv = MR (i) = 30° with OQ, the end B has a velocity 3 ms–1.
Find the velocity of end A at that time.
where v is the velocity of the centre of mass of the
ball. Since it rolls without slipping, v = R , where
is the angular velocity. v
The torque due to F imparts an angular impulse
J = I h = change in angular momentum
v
=I –0=I
2 2
or Ih = M R2 I MR 2 (ii) Fig. 8.32
5 5
Dividing (ii) by (i)
SOLUTION
2
M R2 OB = x, OA = y and x2 + y2 = L2 and x = L cos
5 2R 2 2.5 cm
h= = = Differentiating x2 + y2 = L2 with respect to t we
MR 5 5
have
= 1.0 cm dx dy
2x + 2y =0
dt dt
8.28 2xvB + 2yvA = 0
A turntable of radius R = 10 m is rotating making x
98 revolutions in 10 s with a boy of mass m = 60 vA = – vB
y
kg standing at its centre. He starts running along a
x
radius. Find the frequency of the turntable when the | vA | = vB = vB cot
boy is 4 m from the centre. The moment of inertia of y
the turntable about its axis is 1000 kg m2. = 3 cot 30°
1
SOLUTION = 3 = 1 ms–1
3
Initial moment of inertia of the system is
M1 = M.I. of turntable + M.I. of boy at the centre 8.30
= 1000 + 0 = 1000 kg m2 A rope is wound around a hollow cylinder of mass M
= 3 kg and radius R = 40 cm. If the rope is pulled with
Initial frequency 1 = 9.8 rev/sec
a force F
Final moment of the system is of the cylinder and (b) the linear acceleration of the
M2 = M.I. of turntable + M.I. of boy at a distance 4 m rope.
from the centre of turn table
= 1000 + 60 (4)2 = 1960 kg m2 SOLUTION
Since no external torque acts, the angular momentum For a hollow cylinder I = MR2
of the system is conserved, i.e.
(a) Torque on cylinder is = FR I = FR.
I2 2 = I1 1 I2 2 = I1 1 Therefore
I1 1 1000 9.8 FR FR F 30
= = = 5 rev/s a= = 2
= =
2
I2 1960 I MR MR 3 0.4
= 5 Hz = 25 rad s–2
Rigid Body Rotation 8.19

(b) Linear acceleration of rope is


a = R = 0.4 25 = 10 ms–2 8.32
A uniform rod AB of mass M and length L is hinged
8.31 at one end A. It is released from rest at a horizontal
position. Find the angular acceleration of the rod and
A uniform rod AB of mass M = 0.4 kg and length the linear acceleration of its centre of mass as it falls.
L = 1 m lies on a horizontal frictionless table with its
end A pivoted to the table. A ball of mass m = 0.2 kg SOLUTION
moving along the surface of the table with velocity
u = 4 ms–1 perpendicular to the rod collides with the free Refer to Fig. 8.34.
end B
velocity of the ball immediately after the collision and
(b) the angular velocity of the rod after collision.

SOLUTION
Refer to Fig. 8.33.
Fig. 8.34

L MgL
Torque about A = Mg I =
v 2 2

MgL 3g ML2
or = = I
Fig. 8.33 2I 2L 3
(a) Let v be the velocity of the ball just after col- Linear acceleration of centre of mass is
lision. Since the collision is perfectly elasitc,
L L 3g 3g
e = 1, i.e. aCM = = =
Velocity of approach = velocity of separation 2 2 2L 4
or u= L–v
TRANSLATIONAL AND ROTATIONAL
u v 8.21
= (i) EQUILIBRIUM
L
Since there is no external torque, the angular A body is said to be in equilibrium if its state of motion
mementum about A is conserved, i.e. does not change with time.
mu L = mvL + I Translational Equilibrium
2
= MvL +
ML A body is in translational equilibrium if the total force
3 acting on it is zero; the total force is equal to the vector sum
3 (u v)m (resultant) of the individual forces acting on the body,
= (ii) i.e.
ML
Ftotal = F1 + F2 + … + Fn
From (i) and (ii), we get
If a body is in translational equilibrium, the linear
3m M momentum does not change with time, i.e. p = constant
v= u (iii)
3m M
Rotational Equilibrium
3 0.2 0.4
= 4 = 0.8 ms–1 A body is in rotational equilibrium if the total torque
3 0.2 0.4 acting on the body is zero; the total torque is equal to the
(b) Using (iii) in (i), we get vector sum of the individual torques acting on the body,
i.e.
6 mu
= total = 1 + 2 +……+ n
(3 m M ) L
If the body is in rotational equilibrium, the angular
6 0.2 4 momentum about the axis of rotation does not change with
= = 4.8 rad s–1
(3 0.2 0.4) 1 time, i.e. L = constant
8.20 Comprehensive Physics—JEE Advanced

Consider a uniform rigid rod AB of negligible mass. For translational equilibrium Ftotal = 0 R– F1 – F2 = 0
Two parallel forces of equal magnitude F are applied For rotational equilibrium total = 0 F 1 a 1 – F 2a 2 = 0
perpendicular to the rod at ends A and B as shown in Fig. or F 2a 2 = F 1a 1
8.35(a). Let C be the mid-point of rod.
i.e. anticlockwise torque = clockwise torque.
This is the principle of moments.

8.33
A uniform metal bar AB of length 100 cm and mass
M = 2kg is supported on two knife-edges placed 20 cm
from each end. A mass of m = 3kg is suspended at
a distance of 40 cm from end A. Find the normal
reactions at the knife-edges. Take g = 10 ms–2.

SOLUTION
Refer to Fig. 8.37

Fig. 8.35

Taking the anticlockwise torque to be positive and


clockwise torque to be negative, anticlockwise torque
about C = Fa and clockwise torque about C = – Fa.
Total torque about C = Fa – Fa = 0. Total Force = F + F
= 2F. Hence the rod is in rotational equilibrium but not in Fig. 8.37
translational equilibrium.
If the forces act in opposite directions as shown in Let R1 and R2 be the normal reaction at K1 and K2.
Fig. 8.34(b), total torque = Fa + Fa = 2Fa and total force The rod is in translational as well as rotational equi-
= F – F = 0. Hence, in this case, the rod is in translational librium.
equilibrium but not in rotational equilibrium. For translational equilibrium, Ftotal = 0, Hence
A pair of equal and opposite forces having different mg + mg – R1 – R2 = 0
lines of action is known as a couple. A couple produces R1 + R2 = (m + m)g = (3 + 2) 10 = 50 N (i)
a torque which produces rotation without translation. For
example, opening the cap of a bottle or opening a tap. For rotational equilibrium, total = 0, Hence
Torque is also called moment of force. Clockwise moment about G = anticlockwise moment
Principle of Moments about G

Consider a uniform rod AB of negligible mass pivoted R1 (K1G) = R2 (K2G) + mg (PG)


at a point along its length as shown in Fig. 8.36. Forces R1 30 = R2 30 + 3 10 10
F1 and F2 act at ends A and B are such that the rod is in
R1 = R2 + 10
translational as well as rotational equilibrium. Let R be the
normal reaction of the pivot. R1 – R2 = 10 (ii)
From Eqs. (i) and (ii) we get R1 = 30 N and R2 = 20 N

8.34
A uniform rod AB of length 1.0 m and mass 5.0 kg
leans on a frictionless vertical wall and a rough hori-
B touching the wall and end A
at a distance of 40 cm from the wall as shown in Fig.
Fig. 8.36 8.38(a) Find
Rigid Body Rotation 8.21

(a) Normal reaction of the wall and normal reaction For translational equilibrium, Ftotal = 0. Hence
R1 = f (horizontal direction)
(b) the frictional force at end A,
and R2 = Mg (vertical direction)
end A, and For rotational equilibrium, total = 0. Hence clock-
(d) the reaction force at A. wise moment about A = anticlockwise moment
about A
R1 BC = R2 0 + Mg AD + f 0
R1 0.92 = Mg 0.2 = 5 10 0.2 = 10
R1 = 10.9 N
Also R2 = Mg = 5 10 = 50 N

[Note that the perpendicular distance between R2


and the axis of rotation at A is zero and perpen-
dicular distance of f from the axis of rotation at
A is also zero]
(b) Frictional force f = R1 = 10.9 N
Fig. 8.38

end A is
SOLUTION
f 10.9
μ= = = 0.22
Refer to Fig. 8.38(b). AB = 1.0 m, AG = BG = 0.5 m, R2 50
AC = 0.4m. G is the centre of mass of the rod. (d) The reaction force F at A is the resultant of f
2 2 2 2 and R2
BC = AB AC = 1.0 0.4 = 0.92 m
F= f2 R22
The forces acting on the rod are
(i) normal reactions R1 and R2 at B and A 2 2
= 10.9 50 = 51.2 N
(ii) weight Mg of rod acting at G and
(iii) frictional force f A of NOTE
the rod. At end B, reaction force = normal reaction R1 because
(a) Since the frictional force f prevents the rod from the wall is frictionless.
sliding away from the wall, force f must be di-
rected towards the wall.

Multiple Choice Questions with Only One Choice Correct


1. A gun of mass M is initially at rest on a horizontal (b) moves with a velocity mv cos /M in the
m with horizontal direction
a velocity v at an angle with the horizontal. After (c) moves with a velocity mv sin /M in the
horizontal direction
(a) moves with a velocity mv/M opposite to the (d) remains at rest
direction of motion of the bullet.
8.22 Comprehensive Physics—JEE Advanced

2. A thin uniform circular disc has a radius R. A square 8. A solid cylinder of mass M and radius R is rolling
portion of diagonal equal to R is cut out from it. without slipping on a horizontal plane with a speed
The distance between the centre of mass of the re- v
maining portion of the disc from the centre of the to a maximum height given by
complete disc is v2 v2
(a) (b) sin
(a)
R
(b)
R 2g 2g
2 (2 1)
3v 2 3v 2
R R (c) (d) sin
(c) (d) 4g 4g
(2 1) 2(2 1)
9. A uniform rod AB of mass m and length L is sus-
3. A carpet of mass M is rolled along its length in the
pended by two strings C and D of negligible mass
form of a cylinder of radius R and kept on a rough
as shown in Fig. 8.39. When string D
tension in string C will be
radius R
1 3 (a)
mg
(b) mg
(a) MgR (b) MgR
2 4 4
5 7 (c) 2 mg (d) 4 mg
(c) MgR (d) MgR
8 8
4. A ring of radius r has its mass non-uniformly
distributed over its circumference with centre at
x is the distance of the centre of mass

(a) x = r (b) x < r


(c) x > r (d) 0 x r Fig. 8.39
5. Two particles of equal mass have velocities v1 = a i
10. A body of mass m is projected with a velocity u at
and v2 = a j an angle of 60° with the horizontal. The magnitude
a1 = b ( i j) where a and b of the angular momentum about the point of pro-
- jection when the body is at the highest point of its
tre of mass of the two particles moves along a trajectory is
(a) straight line (b) circle mu 3 2mu 3
(c) ellipse (d) parabola (a) (b)
2g 5g
6.
an inclined plane of inclination 1 3mu 3 mu 3
(c) (d)
sliding and reaches the bottom with speed v1 and 16 g 4g
its time of descent is t1. The same sphere is then 11. A cubical block of side L and mass m rests on a
released from rest from the top of another inclined rough horizontal surface. A horizontal force F is
plane of inclination 2 applied normal to one of its faces at a point that is
without sliding and reaches the bottom with speed directly above the centre of the face at a height 2L/3
v2 and its time of descent is t2 2> 1 above the base. The minimum force F required to
(a) v2 > v1 ; t2 < t1
topple the block before sliding is
(b) v2 = v1 ; t2 < t1
(c) v2 < v1 ; t2 > t1 2mg 3mg
(d) v2 = v1 ; t2 = t1 (a) (b)
3 2
7. The moment of inertia of a uniform rod of mass
M and length L about an axis passing through its 3mg 4mg
(c) (d)
centre and inclined to it at an angle = 60° is 4 3
ML2 ML2 12.
(a) (b)
3 4
duration of the new day will be
ML2 ML2 (a) 6 hours (b) 12 hours
(c) (d)
12 16 (c) 18 hours (d) 30 hours
Rigid Body Rotation 8.23

13. A circular ring of mass M and radius R is rotating of friction between the coin and the record is
about its axis at an angular frequency . Two the minimum angular frequency of the record for
m
the opposite ends of a diameter of the ring. The
angular frequency of the ring becomes . The ratio 2 g g
(a) (b)
/ is r 2r
M 2M g g
(a) (b) (c) (d) 2
M 2m M 2m r r
2m M 19.
(c) (d)
M 2m
14. A solid sphere rolls down from the top of an in-

the plane is v. When the same sphere slides down g 2 g


(a) (b)
r r
the bottom is v . The ratio v /v is g 2 g
3 (c) 2 (d) 2
(a) (b) 1 r r
5 20. m
7 3 corners of a right angled triangle as shown in Fig.
(c) (d) OA = a and OB = b
5 5 the centre of mass is (here i and j are unit vectors
15. A circular disc is rolling down an inclined plane along x and y axes respectively).
1 1
acceleration of the disc down the inclined plane is (a) (ai + bj) (b) (ai – bj)
g 3 3
(a) g (b) 2 2
2 (c) (ai + bj) (d) (ai – bj)
3 3
g 2
(c) (d) g
3 3
16. A block of mass M is released from the top of an

of the plane is v. A circular disc of the same mass


M
velocity on reaching the bottom is v . The ratio
v /v will be
1 2
(a) (b)
3 3 Fig. 8.40
2 2 21. A sphere of mass M and Radius R is released from
(c) 1 (d)
3 the top of an inclined plane of inclination . The
17. Two circular loops A and B of radii R and 2R
respectively are made of the same wire. Their and the sphere so that it rolls down the plane without
moments of inertia about the axis passing through sliding is given by
the centre and perpendicular to their plane are IA and 2
IB respectively. The ratio IA/IB is (a) = tan (b) = tan
3
1
(a) 1 (b) 2 2
2 (c) = tan (d) = tan
5 7
1 1
(c) (d) 22. M and length
4 8 L
18. A small coin is placed at a distance r from the centre moment of inertia of the composite structure about
of a gramophone record. The rotational speed of an axis passing through the centre of mass of the
structure and perpendicular to its plane is
8.24 Comprehensive Physics—JEE Advanced

26. I1 I2 I3 and I4 are respectively the moments of inertia


ML2 ML2
(a) (b) of a thin square plate ABCD of uniform thickness
2 4
ML2 ML2 the plate (Fig. 8.43). The moment of inertia of the
(c) (d) plate about an axis passing through thecentre O and
8 12
perpendicular to the plane of the plate is
23. M and length L
are welded to form a square ABCD as shown in Fig. (a) 2(I1 + I2) (b) 2(I3 + I4)
8.41. What is the moment of inertia of the composite (c) I1 + I3 (d) I1 + I2 + I3 + I4
structure about a line which bisects rods AB and CD

ML2 ML2
(a) (b)
6 3
ML2 2 ML2
(c) (d)
2 3

Fig. 8.43

Fig. 8.41 27. The speed of a homogeneous solid sphere after roll-
ing down an inclined plane of vertical height h
24 . A thin uniform metallic triangular sheet of mass from rest without sliding is
M has sides AB = BC = L. What is its moment of
inertia about axis AC 10
(a) gh (b) gh
(Fig. 8.42) 7
(a) M L2 6 4
(c) gh (d) gh
12 5 3
M L2 28.
(b) energy to the total kinetic energy is given by
6
2 (a) 7 : 10 (b) 2 : 5
ML
(c) (c) 10 : 7 (d) 2 : 7
3
29. A cart of mass M is tied at one end of a massless
2M L2 rope of length 10 m. The other end of the rope is in
(d) Fig. 8.42
3 the hands of a man of mass M. The entire system
25. M and radius is on a smooth horizontal surface. The man is at
R x = 0 and the cart at x
rolling and partly sliding. During this kind of
motion of the sphere a point
(a) total kinetic energy is conserved (a) x = 0
(b) the angular momentum of the sphere about the (b) x = 5 m
point of contact with the plane is conserved (c) x = 10 m
(d) they will never meet.
(c) only the rotational kinetic energy about the
centre of mass is conserved 30. A mass m is moving with a constant velocity along
(d) the angular momentum about the centre of a line parallel to the x
mass is conserved. angular momentum with respect to the origin
Rigid Body Rotation 8.25

(a) is zero (b) remains constant


(c) goes on increasing (d) goes on decreasing.

31. Let I be the moment of inertia of a uniform square


plate about an axis AB that passes through its centre
and is parallel to two of its sides. CD is a line in the
plane of the plate that passes through the centre of the Fig. 8.45
plate and makes an angle with AB. The moment of 35. A thin wire of length L and uniform linear mass
inertia of the plate about the axis CD is then equal to density is bent into a circular loop with centre at
(a) I (b) I sin2 O as shown in Fig. 8.46. The moment of inertia of
(c) I cos2 (d) I cos2 the loop about the axis XX is
2 L3 L3
(a) 2
(b) 2
8 16
32. A smooth sphere A is moving on a frictionless
horizontal surface with angular speed and centre 5 L3 3 L3
(c) (d)
of mass velocity v 16 2 8 2
on with an identical sphere B at rest. Neglect friction

are A and B respectively. Then


(a) A < B (b) A = B
(c) A = (d) B =

33. A disc of mass M and radius R is rolling with


angular speed on a horizontal plane as shown in Fig. 8.46
Fig. 8.44. The magnitude of angular momentum of 36. A tube of length L
the disc about the origin O is incompressible liquid of mass M and closed at both
the ends. The tube is then rotated in a horizontal plane
about one of its ends with a uniform angular velocity
. The force exerted by the liquid at the other end is
2
M L 2
(a) (b) M L
2
2 2
M L M L2
(c) (d)
4 2
Fig. 8.44

1 37. An equilateral triangle


(a) MR2 (b) MR2
2 ABC formed from a
3 uniform wire has two
(c) MR2 (d) 2 MR2 small identical beads
2 initially located at A. The
34. A cubical block of side a is moving with a velocity triangle is set rotating
v on a horizontal smooth plane as shown in about the vertical axis
O. The angular AO. Then the beads
speed of the block after it hits the ridge at O is are released from rest
3v 3v simultaneously and
(a) (b) Fig. 8.47
4a 2a one along AB and the
3v other along AC as shown in Fig. 8.47. Neglecting
(c) (d) zero
2a
8.26 Comprehensive Physics—JEE Advanced

(a) angular velocity and total energy (kinetic and (a) M (b) 2MA
potential). (c) A M 2
(d) AM2
(b) total angular momentum and total energy. 43. One end of a thin uniform rod of length L and mass
(c) angular velocity and moment of inertia about M1 is rivetted to the centre of a uniform circular
the axis of rotation. disc of radius ‘r’ and mass M2 so that both are
(d) total angular momentum and moment of coplanar. The centre of mass of the combination
inertia about the axis of rotation. from the centre of the disc is:
(Assume that the point of attachment is at the origin)
38. A cubical block of side L rests on a rough horizontal
. A horizontal L ( M1 M 2 ) LM1
(a) (b)
force F is applied on the block as shown in Fig. 2 M1 2( M1 M 2 )
2 ( M1 M 2 ) 2 LM1
(c) (d)
LM1 ( M1 M 2 )
minimum force required to topple the block is
mg/4 44. Two circular loops A and B of radii rA and rB respec-
tively are made from a uniform wire. The ratio of
(c) mg/2 (d) mg(1 — )
their moments of inertia about axes passing through
their centres and perpendicular to their planes is
IB rB
is equal to
IA rA
(a) 2 (b) 4
Fig. 8.48 (c) 6 (d) 8
39. The angular velocity of a body changes from 1 to 45. A body of mass ‘m’ is tied to one end of a spring and
whirled round in a horizontal plane with a constant
2 without applying a torque but by changing the
moment of inertia about its axis of rotation. The angular velocity. The elongation in the spring is one
ratio of the corresponding radii of gyration is
elongation in the spring is 5 cm. The original length
(a) 1 : 2 (b) 1: 2 of the spring is:
(c) : (d) :
2 1 2 1 (a) 16 cm (b) 15 cm
40. A thin uniform rod AB of mass m and length L is (c) 14 cm (d) 13 cm
hinged at one end A
46. A particle performs uniform circular motion with
stands vertically and is allowed to fall freely to the
an angular momentum L
the rod when its end B
ular momentum becomes:
1/ 2
mg mg (a) 4L (b) 2L
(a) (b)
L 3L (c) L (d) L
3 g 1/ 2 2 4
g
(c) (d) 47. A uniform rod of length 1 metre is bent at its mid-
L L point to make 90° angle. The distance of the centre
41. Moment of inertia of uniform horizontal solid of mass from the centre of the rod is
cylinder of mass M about an axis passing through (a) 36.1 cm (b) 25.2 cm
its edge and perpendicular to the axis of the
cylinder when its length is 6 times its radius R is: (c) 17.7 cm (d) zero
48. A mass is whirled in a circular path with constant
39 MR 2 39 MR
(a) (b) angular velocity and its angular momentum is L
4 4 the string is now halved keeping the angular veloc-
49 MR 49 MR 2
(c) (d)
4 4 (a) L (b) L
42. A is the areal velocity of a planet of mass M 4 2
angular momentum is (c) L (d) 2L
Rigid Body Rotation 8.27

49. - two beads are at the centre of the rod and the system
is rotating with angular velocity 0 about its axis
perpendicular to the rod and passing through its
mid point (see Fig. 8.50). There are no external
(a) Moment of inertia
(b) Angular momentum the angular velocity of the system is
(c) Angular velocity M 0 M 0
(d) Rotational kinetic energy (a) (b)
M 3m M 6m
50. A solid sphere is rotating about its diameter. Due to
(c) M 6m 0 (d) 0
M
speed of the sphere will
1
(a) increase by nearly %
3
1
(b) decrease by nearly %
3
1
(c) increase by nearly %
2
Fig. 8.50
2
(d) decrease by nearly % 54. Two particles A and B
3
each other under a mutual force of attraction. At the
51. The height of a solid cylinder is four times its radi-
instant when the speed of A is V and that of B is 2V
t = 0 on a belt which
the speed of the centre of mass of the system is
is moving in the horizontal direction with a velocity
(a) 0 (b) V
v = 2.45 t2 where v is in ms–1 and t
(c) 1.5V (d) 3V
t equal to
(a) 1 s (b) 2 s 55. One quarter sector is cut from a uniform circular
(c) 3 s (d) 4 s disc of radius R. This sector has mass M
52. A circular portion of diameter R is cut out from a made to rotate about a line perpendicular to
uniform circular disc of mass M and radius R as its plane and passing through the center of the
shown in Fig. 8.49. The moment of inertia of the
remaining (shaded) portion of the disc about an axis rotation is (Fig. 8.51)
passing through the centre O of the disc and per- 1
pendicular to its plane is (a) MR2
2
15 7 1
(a) MR2 (b) MR2 (b) MR2
32 16 4
13 3 1
(c) MR 2 (d) MR2 (c) MR 2
32 8 8 Fig. 8.51
(d) 2 MR 2

56. Two blocks of masses 10 kg and 4 kg are connected


by a spring of negligible mass and placed on a
frictionless horizontal surface. An impulse gives
a velocity of 14 m/s to the heavier block in the
direction of the lighter block. The velocity of the
Fig. 8.49 center of mass is
(a) 30 m/s (b) 20 m/s
53. A smooth uniform rod of length L and mass M has
(c) 10 m/s (d) 5 m/s
m
8.28 Comprehensive Physics—JEE Advanced

57.
A B
(a) (b)
throughout these motions). The directions of the B A
frictional force acting on the cylinder are:
A B
(a) up the incline while ascending and down the (c) (d)
incline while descending B A

(b) up the incline while ascending as well as 63.


descending
(c) down the incline while ascending and up the with speed vr
incline while descending -
(d) down the incline while ascending as well as out rolling and reaches the bottom with speed vs.
descending. Then
(a) vr > vs (b) vr < vs
58. The angular momentum of a particle moving in (c) vr = vs (d) vr = vs = 0
a circular orbit with a constant speed remains 64. ABCD shown in
conserved about a = AB = BC/2. The moment of inertia of
(a) any point on the circumference of the circle the lamina is the minimum along the axis passing
(b) any point inside the circle through
(c) any point outside the circle
(a) BC (b) AB
(d) the centre of the circle
(c) HF (d) EG

59. A particle moves in a circular orbit with uniform

orbit is itself rotating at a constant angular speed.


We may then say
(a) the angular velocity as well as the angular
acceleration of the particle are both constant
Fig. 8.52
(b) neither the angular velocity nor the angular
acceleration of the particle are constant 65. A uniform rod of length L is suspended from one
(c) the angular velocity of the particle varies but end such that it is free to rotate about an axis
its angular acceleration is a constant passing through that end and perpendicular to the
length. What minimum speed must be imparted to
(d) the angular velocity of the particle remains
the lower end so that the rod completes one full
constant but its angular acceleration varies
60. A cylinder of mass m and radius r is rotating about its
axis with a constant speed v (a) 2gL (b) 2 gL
(a) 2 mv2 (b) mv2 (c) 6gL (d) 2 2gL
1 1 66. A circular disc of radius R is free to oscillate about
(c) mv2 (d) mv2
2 4 an axis passing through a point on its rim and per-
61. A circular disc of mass m and radius r is rolling on pendicular to its plane. The disc is turned through
a smooth horizontal surface with a constant speed
v when it reaches the equilibrium position will be
1 1 g 2g
(a) mv2 (b) mv2 (a) (b)
4 2 3R 3R
3 2g 2g
(c) mv2 (d) mv2 (c) (d) 2
4 R R
62. Two solid spheres A and B R 67. A massless and inextensible cord is wound round
made of materials of densities A and B respec- the circumference of a circular ring of mass M and
tively. Their moments of inertia about a diameter radius R. The ring is free to rotate about an axis
are IA and IB respectively. The ratio IA /IB is passing through its centre and perpendicular to its
Rigid Body Rotation 8.29

plane. A mass m is attached at the free end of the 71. A particle is moving in the x – y plane with a
cord and is at rest. The angular speed of the ring constant velocity along a line parallel to the x
when mass m has fallen through at height h is away from the origin. The magnitude of its angular
2 gh 2mgh momentum about the origin.
(a) (b) (a) is zero
R2 MR 2 (b) remains constant
2mgh 2mgh (c) goes on increasing
(c) 2
(d) (d) goes on decreasing
M m R M 2m R 2
68. The moment of inertia of a hollow sphere of mass
M and internal and external radii R and 2R about an 72. A thin uniform disc has mass M and radius R. A
axis passing through its centre and perpendicular to circular hole of radius R/3 is made in the disc as shown
its plane is in Fig. 8.54. The moment of inertia of the remaining
portion of disc about an axis passing through O and
3 13 perpendicular to the plane of the disc is
(a) MR 2 (b) MR 2
2 32 1 2
31 62 (a) MR2 (b) MR2
(c) MR2 (d) MR2 9 9
35 35 1 4
69. (c) MR2 (d) MR2
3 9
angular frequency with his arms outstretched. He

with folded arms is 75% of that with outstretched

(a) increase by 33.3%


(b) decrease by 33.3%
(c) increase by 25%
(d) decrease by 25%

70. A uniform disc of radius R is rolling (without Fig. 8.54


slipping) on a horizontal surface with an angular
speed as shown in Fig. 8.53. O is the centre of
A and C are located on its rim and 73. A solid metallic sphere of radius R having moment
R of inertia equal to I about its diameter is melted
point B is at a distance from O and recast into a solid disc of radius r of a uniform
2 thickness. The moment of inertia of the disc about
the points A B and C lie on the vertical diameter an axis passing through its edge and perpendicular
vA vB and vC are the to its plane is also equal to I. The ratio r/R is
linear speeds of points A B and C respectively at
2 2
(a) (b)
(a) vA = vB = vC (b) vA > vB > vC 15 10
4 2 1
(c) vA vC = vB (d) vA vC = 2 vB (c) (d)
3 5 2

74. A small object of uniform density rolls up a curved


surface with an initial velocity v
3v 2
a maximum length h =
4g
initial position. The object is (see Fig. 8.55)
(a) ring (b) solid sphere
Fig. 8.53 (c) hollow sphere (d) disc
8.30 Comprehensive Physics—JEE Advanced

Fig. 8.55

75. The mass per unit length of a non-uniform rod OP


of length L varies as
m=k x Fig. 8.56
L
where k is a constant and x is the distance of any 78. A gramophone record of mass M and radius R is
point on the rod from end O. The distance of the rotating at an angular velocity . A coin of mass m
centre of mass of the rod from end O is is gently placed on the record at a distance r = R/2
from its centre. The new angular velocity of the
(a) L (b) 2 L system is
3 3 2 M 2 M
L 2L (a) (b)
(c) (d) ( 2 M m) ( M 2m)
2 3
76. A tube of length L - M
(c) (d)
compressible liquid of mass M and closed at both M
ends. The tube is then rotated in a horizontal plane 79. A bolck of base 10 cm 10 cm and height 15 cm
about one of its ends with a uniform angular veloc-
ity . The force exerted by the liquid at the other friction between them is 3 . The inclination
end is. of this inclined plane from the horizontal plane is
(a) 2 M 2L (b) M 2L gradually increased from 0°. Then
1 3
(c) M 2L (d) M 2L (a) at
2 2 the plane
77. A uniform thin rod of mass M and length L is hinged (b) the bock will remain at rest on the plane up
by a frictionless pivot at its end O to certain and then it will topple
8.56. A bullet of mass m moving horizontally with
(c) at
a velocity v strikes the free end of the rod and gets
the plane and continue to do so at higher
embedded in it. The angular velocity of the system angles
about O just after the collision is
mv 2m v (d) at
(a) (b) the plane and on further increasing
L ( M m) L ( M 2m)
topple at certain .
3mv mv
(c) (d)
L( M 3m) LM

ANSWERS

1. (d) 2. (d) 3. (d) 4. (d) 5. (a) 6. (b)


7. (d) 8. (c) 9. (a) 10. (c) 11. (c) 12. (a)
13. (a) 14. (c) 15. (c) 16. (b) 17. (d) 18. (c)
19. (a) 20. (a) 21. (d) 22. (a) 23. (d) 24. (a)
25. (b) 26. (c) 27. (a) 28. (d) 29. (b) 30. (b)
31. (a) 32. (c) 33. (c) 34. (a) 35. (d) 36. (a)
37. (b) 38. (c) 39. (c) 40. (d) 41. (d) 42. (b)
43. (b) 44. (a) 45. (b) 46. (d) 47. (c) 48. (a)
49. (b) 50. (b) 51. (a) 52. (c) 53. (b) 54. (a)
55. (a) 56. (c) 57. (b) 58. (d) 59. (c) 60. (d)
Rigid Body Rotation 8.31

61. (c) 62. (c) 63. (b) 64. (c) 65. (c) 66. (b)
67. (c) 68. (d) 69. (a) 70. (c) 71. (b) 72. (d)
73. (a) 74. (d) 75. (b) 76. (c) 77. (c) 78. (a)
79. (b)

SOLUTIONS
1. Since there is no external force acting on the gun- 4. Since the mass is not distributed uniformly over the

remains at rest. lie anywhere between its centre and the circumfer-
2. Let = mass per unit area of the disc. Mass of the ence. Hence the correct choice is (d).
cut-out portion m1 = R2 /2 and mass of the remain- m v m2 v 2
ing portion is (see Fig. 8.57) 5. vCM = 1 1
m1 m2
R2
m2 = R2 1
2 = (v1 + v2) ( m 1 = m 2)
2
Let O1 be the centre of mass of the remaining por- a
tion. The centre of mass of the square is at O2. Tak- = i j
ing moments of m1 g and m2 g about O 2
m1 g x1 = m2 g x2 m1a1 m2 a2 a
aCM = = 1 ( a2 = 0)
2 2
m1 m2 2
R R
x1 g = R2 x2 g b
2 2 =i j
2
R Since vectors vCM and aCM are parallel to each
x2 = ( x1 = R /2)
2(2 1)
line.
6.
is conserved. Since both the inclined planes are of
v2 = v1. The acceleration of the
sphere rolling down the plane is
g sin
a= I
1
MR 2
Since 2 > 1; a2 > a1. Hence
Fig. 8.57 t1 sin 2
=
3. The entire mass M of the carpet can be assumed to t2 sin 1
be concentrated at its centre of mass which is origi- Thus t1 > t2. Hence the correct choice is (b).
nally at a height R M
P.E. = MgR. When the carpet is unrolled to a radius 7. Mass of element of length dx is dm = dx.
R R/2 above L
Perpendicular distance of the element from the axis
of rotation = x sin . Therefore moment of inertia
M ( R / 2) 2 M about the axis of rotation AB is (Fig. 8.58)
m= 2
R 4 L/2

M R MgR I= dm( x sin ) 2


and its P.E. = mg (R/2) = g L/2
4 2 8
L/2
7 M
Decrease in P.E. = MgR –
MgR
MgR = sin2 x 2 dx
8 8 L L/2
8.32 Comprehensive Physics—JEE Advanced

ML2 ML2 ML2 Also a =


L
(3)
= sin2 = sin2 (60°) = 2
12 12 16 3g
a= . Then from Eq. (1)
4
3mg mg
mg – T = T=
4 4
So the correct choice is (a).
10.
v = u cos 60° = u/2 along the horizontal direction.
Hence the perpendicular distance of its linear mo-
mentum from the point of projection = maximum
height attained.
Fig. 8.58 u 2 sin 2 60 3u 2
r1 = hmax = =
2g 8g
1 1
8. K.E. = Mv 2 + I 2
Magnitude of angular momentum = m r1 v
2 2
1 1 1 v 2 3u 2 u
= Mv 2 + MR 2 =m
2 2 2 R 8g 2
1 1 3 3 mu 3
= Mv 2 + Mv2 = Mv2 =
2 4 4 16 g
11. Torque of F about A is [see Fig. 8.60]
Loss in K.E. = gain in P.E.
3 2L
or Mv 2 = Mgh 1 = F
4 3
3v 2 Since the weight mg
h= torque of the weight about A is
4g
9. When string D A. L
Let a be the linear acceleration of the centre of mass 2 = mg
2
and T the tension in string C
The minimum force required to topple the block is
mg – T = ma (1) given by
2L L
Torque of force mg about A is = Mg
L
. ( 1)min = 2 or Fmin = mg
2 3 2
3mg
Fmin =
4

Fig. 8.60
Fig. 8.59

Moment of inertia of the rod about A is I = ML2/3. 12. Let M be the mass and R the initial radius of the
is the angular velocity of the rotation of
A is T of the day is
g 2
= = (2) T=
I 2L
Rigid Body Rotation 8.33

Let R be the radius of the earth after contraction 1


and its angular velocity. From the conservation I= MR2. Using this and
2
a=g
I =I
16. The acceleration of the block sliding down the
2 2 plane is
where I M R 2 and I M R 2 are the
5 5 a = g sin
moments of inertia of the earth before and after where l is the length

2 2 R2 reaching the bottom is given by


MR 2 = MR 2 or = 2
=4 v2 = 2 al = 2g sin l
5 5 R
( R = R/2) or v = 2gl sin
The duration T of the new day will be The acceleration of the disc rolling down the plane
is (as shown above)
2 2 T 2
T = = = a = g sin
4 4 3
24 hours bottom is given by
T = = 6 hours
4
4
v 2 = 2a l = gl sin
13. From the law of conservation of angular momen- 3
gl sin v 2
or v = 2 =
3 v 3
I =I
Hence the correct choice is (b).
Here I = MR2 and I = (M + 2m) R2. Therefore 17. Let be the mass per unit length of the wire. The
I M mass of loop A is MA = 2 R and mass of loop B is
= =
I M 2m MB = 4 R Their moments of inertia respectively
Hence the correct choice is (a). are IA = MA R A2 = 2 R R2 = 2 R3
and IB = MBR 2B = 4 R (2R)2
1 1
14. For rolling : Mgh = Mv2 + I 2
= 16 R3
2 2
IA 1
1 1 2 v2 =
= Mv2 + M R2 IB 8
2 2 5 R2 Hence the correct choice is (d).
1 1 7 18. Let m
= Mv2 + Mv2 = Mv2
2 5 10 the centripetal force mr 2 just exceeds the force of
friction mg. The minimum is given by
2 v
I = MR 2 and =
5 R mr 2
= mg or =
g
1 r
For sliding : Mgh = Mv 2. Therefore Hence the correct choice is (c).
2
19. The minimum angular frequency is independent of
1 7
Mv 2 = Mv2 the mass. Hence the correct answer is still g/r
2 10 which is choice (a).
v 7 20. The (x y) co-ordinates of the masses at O A and B
or =
v 5 respectively are (refer to Fig. 8.40 on page 8.23)
15. The acceleration down the plane is given by (x1 y1 x2 = a y2 = 0) and (x3 y3 = b)
g sin The (x y) co-ordinates of the centre of mass are
a=
I m1 x1 m2 x2 m3 x3
1 xCM =
MR 2 m1 m2 m3
8.34 Comprehensive Physics—JEE Advanced

m 0 m a m 0 a
= =
m m m 3
m1 y1 m2 y2 m3 y3
yCM =
m1 m2 m3
m 0 m 0 m b b
= =
m m m 3
The position vector of the centre of mass is xCM i +
yCM j
a b 1
= i+ j = (ai + bj)
3 3 3
21. - Fig. 8.61
3
PS = PQ sin 60° = L sin 60° = L.
g sin 2
a=
I PS 1 3 L
1 h= = L=
MR 2 3 3 2 2 3
where K is the radius of gyration of the sphere
its moment of inertia about an axis passing through
the sphere about its diameter is its centre of mass C and perpendicular to its plane
2
I= MR2
5 Ic = 3 (I + Mh2)
5 where I is the moment of inertia of each rod about
g sin
a= = g sin (i) the axis passing through its centre and perpendicu-
2 7
1
5 2
M L2 L M L2
f I= Also Mh2 = M =
provides the necessary torque which is given by 12 2 3 12
= force moment arm = fR M L2 M L2 2 2
But = I is the angular acceleration of Ic = 3 =3 ML = ML
12 12 6 2
I = fR
Hence the correct choice is (a).
a= R
23. Refer to Fig. 8.41 on page 8.24. Moment of inertia
I Ia 2 2 is a scalar quantity. So the moment of inertia of the
f = = 2 = Ma IMR 2
R R 5 5 structure is the sum of the moments of inertia of the
normal reaction
2 I = I1 + I2 + I3 + I4
= Mg cos . Thus Mg cos = Ma where I1= moment of inertia of rod 1 about an axis
5
passing through its centre E and perpendicular to its
5
or a= g cos (ii) M L2
2 plane =
Equating (i) and (ii) we have 12
I2= moment of inertia of rod 2 about an axis passing
5 5 2
g sin = g cos or = tan through its centre F and perpendicular to its plane
7 2 7
M L2
Hence the correct choice is (d). =
22. Given PQ = QR = RP = L. The centre of mass is 12
located at centroid C which cuts lines PS QT and I3 = moment of inertia of rod 3 about a parallel axis
UR in the ratio 2 : 1. Let h = CS = CT = UC L L 2 M L2
at a distance from it = M =
PQS 2 2 4
Rigid Body Rotation 8.35

I4 = moment of inertia of rod 4 about a parallel axis


L M L2 linear velocity v and an angular velocity
at a distance from it = . kinetic energy at any time t = translational KE +
2 4
rotational KE
M L2 M L2 M L2 M L2
I= 1 1
12 12 4 4 or KE = mv2 + I 2
2 2
2
= ML2 2 v
3 Here I= MR2 and =
24. 5 R
1 1 1 2 v2
the moment of inertia of triangular sheet ABC = KE = Mv2 + MR2
2 2 2 5 R2
moment of inertia of a square sheet ABCD about
1 1 1 7
its diagonal AC or It = Is = Mv2 + Mv2 = Mv2
2 2 5 10
sheet = M + M = 2M
L2 M L2 7 10 g h
Is = (2M) = or Mgh = Mv2 which gives v =
12 6 10 7
Is M L2 Hence the correct choice is (a).
It = = 28.
2 12
the total (translational + rotational) kinetic energy
respectively are
1 1 1
Kr = I 2 and Kt = mv2 + I 2
2 2 2
The moment of inertia of a sphere of mass m and
2
radius r about its centre is I = mr2. Also =
5
2 v2 2
v/r. Therefore I 2 = mr2 2
= mv2. Thus
5 r 5
1 1 1 7
Kr = mv2 and Kt = mv2 + mv2 = mv2.
5 2 5 10
Fig. 8.62 Hence
Hence the correct choice is (a). 1
m v2
25. The correct choice is (b) because the angular
Kr 5 2
=
momentum of the sphere about the point of con- Kt 7 7
m v2
tact with plane surface also includes the angular 10
momentum about the centre of mass. Hence the correct choice is (d).
26. I 1 = I 2. 29.
I3 = I4. From perpendicular axes theo- man will meet at their centre of mass. Since their
at
plate about an axis passing through the centre and x = 5 m. Hence
perpendicular to the plane of the plate is the correct choice is (b).
I = I1 + I2 = I3 + I4 = 2I1 = 2I3 30. Refer to Fig. 8.63. The angular momentum of the
( I 1 = I 2 I 3 = I 4) mass at point P(x y) about origin O
or I 1 = I 3.
Thus I = I 1 + I 2 = I 3 + I 4 = I 1 + I 3. L = mr v = m (x i y j) (v i )
Hence the correct choice is (c).
27. Let the mass of the sphere be M and R its radius. = myv ( k )

energy is entirely potential given by PE = Mgh ( i i = 0 and j i = –k )


8.36 Comprehensive Physics—JEE Advanced

32. Since there is no friction between the sphere and


the horizontal surface and also between the spheres

momentum from sphere A to sphere B due to the


collision. Since the collision is elastic and the
A only
transfers its linear velocity v to sphere B. Sphere A
will continue to rotate with the same angular speed
Fig. 8.63
is (c).
Now m and v are constants. Also y remains constant 33. Refer to Fig. 8.65. Let OC = RC and let vc be the
as the mass moves parallel to the x-axis. Hence L velocity of the centre of mass of the disc. The linear
remains constant. Thus the correct choice is (b). momentum of the centre of mass is pc = Mvc
31. Refer to Fig. 8.64. According to the perpendicular Lc is the angular momentum of the disc about C
then the angular momentum about origin O is
about the z-axis is
L0 = Lc + Rc pc
Iz = Ix + Iy
with Ix = Iy. The square plate lies in the x-y plane.
Since the directions of the x and y
the only restriction being that the angle between

will not change if the axes are rotated through any


angle in the plane of the plate. This can be proved
as follows.
Fig. 8.65

Magnitude of L0 = Ic + Rc Mvc sin


1
= MR2 + M Rcvc sin
2
1
Ic M R2
2
1
= MR2 + MR R
Fig. 8.64 2
Consider a particle of the plate of mass mn located at ( Rc sin = R and vc = R )
a point P(xn yn) in the x-y plane. Then IAB = mn y2n
The moment of inertia about line CD will be 3
= MR2
ICD = mn y 2n 2
The coordinates (xn yn) and (xn yn) are related as Hence the correct choice is (c).
xn = xn cos – yn sin 34. When the block hits the ridge at point O
yn = yn sin – yn cos start rotating about an axis passing through O and
perpendicular to the plane of the paper (Fig. 8.66).
Now ICD = mn y 2n = mn(xn sin – yn cos )2
= (mn x2n) sin2 + (mn y 2n ) cos2
– 2 (mn xn yn) sin cos
IAB = mn x2n
= mn y2n and mn xn yn = 0. Hence
ICD = IAB (sin2 + cos2 ) + 0 = IAB = I
Thus the correct choice is (a).
Fig. 8.66
Rigid Body Rotation 8.37

36. The entire mass of the liquid can be regarded as be-


an-gular momentum is conserved. Angular momen- ing concentrated at the centre of mass of the tube
tum of the block before it hits the ridge is L
which is at a distance of r = from the axis of
a 1 2
Li = Mv AC = Mv = Mva (1) revolution. The force exerted by the liquid at the
2 2
other end of the tube is the centripetal force of a
Angular momentum of the block after it hits the L
ridge is mass M revolving in a circle of radius r = . Thus
2
Lf = IO (2)
M v 2 M (r )2 ML 2
where IO is the moment of inertia of the block about Fc = = Mr 2 =
an axis passing through O and perpendicular to r r 2
the plane of the block and is the angular speed Hence the correct choice is (a).
of rotation of the block. From the parallel axes 37.
gular momentum L is conserved. As the beads slide
2
IO = Ic + Mr
1 change. From L = I will also
C is Ic = Ma2
2
6
a a2
a 2 correct choice is (b).
and r2 = + =
. Hence 38. Torque due to F about A is 1 = FL
2 22
Since the weight mg acts through the centre of mass
1 1 2
IO = Ma2 + Ma2 = Ma2 (3) of the block (which is at a distance of L/2 from
6 2 3 the slide of the block) the torque due to weight mg
2 about A is
Lf = Ma2
3 L
Li = Lf 2 = mg
2
The minimum force required to topple the block is
2 2 3v obtained when 1 is slightly greater than 2
Mva = Ma2 or = limit
3 3 4a L
Hence the correct choice is (a). ( 1)min = 2 or Fmin L = mg
2
35. Let m be the mass of the loop and r its radius. The mg
moment of inertia of the loop about an axis passing or Fmin =
2
through the centre O is (Fig. 8.67)
Hence the correct choice is (c).
39. The magnitude of angular momentum of a rotat-
ing body is given by L = I
I = con-
stant. Hence I1 1 = I2 2 K1 and K2 are the cor-
I1 = MK12 and
I2 = MK22. Hence
Fig. 8.67
1 MK21 1 = MK22 2
IO = mr2
2 K1 2
- or =
K2 1
ertia bout XX is
40. Since the centre of gravity of the rod is at its cen-
1 3
I = IO + mr2 = mr2 + mr2 = mr2
2 2 L 1 1
m = L and radius r = L/2 . mg = mg L. Gain in KE = I 2 where
2 2 2
Hence I is the moment of inertia of the rod about an axis
2
3 L 3 L3
I= L 2
passing through its end and perpendicular to its
2 2 8 1
length which is given by I = mL2. Now gain in
Thus the correct choice is (d). 3
8.38 Comprehensive Physics—JEE Advanced

mB r
= B (ii)
1 1 1
mL2 2
= mgL mA rA
2 3 2 r
Using (ii) in (i) and putting B
have rA
3g
which gives = 3
L rB rB
8= or
41. Given: l = 6R rA rA
moment of inertia about the given axis is given by
45. Let L cm be the original length of the spring and k
R2 l2 be the spring constant. Then
I= M
4 3 m(L + x1) 21 = kx1
2
R2 (6 R ) 2 and m(L + x2) 2 = kx2
=M
4 3
2
R2 36 R 2 2 L x1 x1
= 49 MR
1
=M = (i)
4 3 4
L x2 2 x2

Hence the correct choice is (d). Given x1 x2 = 5 cm and 2 =2 1. Using


L
area swept by radius vector
42. Areal velocity A = choice (b).
time taken
Assuming that the orbit of the planet is a circle of 1 2
I K 1
radius R 46. L = I = 2 = K= I 2
is the
2 2 2 2
R
A=
T is doubled and K
2 value of L becomes one-fourth. Hence the correct
T= . Hence choice is (d).
47. Rod POQ of length l = 100 cm is bent at its mid-
R2 R2 point O so that POQ = 90° (see Fig. 8.68).
A= =
2 / 2 P
2A
or =
R2 OP = OQ = l/2
2 2A
Angular momentum L =I = (MR )
R2 A (0, l/4)
= 2 MA
Hence the correct choice is (b).
C (l/8, l/8)
43. The mass of the rod can be considered to be con- D
centrated at its centre (x = L/2) where x = 0 is the
origin. Hence O
E (l/4, 0) Q
M L/2 0 LM1
RCM = 1 =
M1 M 2 2 ( M1 M 2 ) Fig. 8.68

Hence the correct choice is (b). The mass of part PO of length l/2 can be taken to be
44. IA = mA rA2 and IB = mB rB2 . Hence concentrated at its mid-point A whose coordinates
l/4) and of part OQ of length l/2 at its mid-
2
IB mB rB point B whose coordinates are (l
= (i)
IA mA rA of mass of these two equal masses is at mid-point C
Let k be the mass per unit length of the wire. Then between A and B. The coordinates of C are (l l/8).
the masses of loops A and B are l 2
l 2
OC = (OE ) 2 (CE )2 =
mA = (2 rA)k and mB = (2 rB)k 8 8
Rigid Body Rotation 8.39

l 100 cm Now v = 2.45 t2


= = dv d
32 32 a= 2 . 45t 2 = 4.9t (ii)
dt dt
48. L = mr . For given m and L r2 r
2

the angular momentum L becomes one-fourth. g 9 .8


t= = = 1 s.
Hence the correct choice is (a). 2 4 .9 9 .8
49. Hence the correct choice is (a).
momentum L = (I I is 52. Moment of inertia of complete disc about O is
the moment of inertia and is the angular velocity 1 M
I= MR 2. Mass of the cut-out part is m = .
2 4
I and hence
The moment of inertia of the cut-out portion about
1 2
kinetic energy will also change. Hence 1 R 2 =
mr2 = 1 M
I
2 its own centre I0 =
2 2 4 2
the correct choice is (b). 1 2
4 4 MR because r = R/2. From the parallel axes
50. V = r3 or log V = log + 3 log r. 32
3 3 -
Differ We have tion about O is
V r r 1 V 1 1 2
3 or 0 . 5% % 1 M R
V r r 3 V 3 6 Ic = I0 + mr2 = MR 2 +
32 4 2
2
I = constant or 3
2 2
mr = constant or r = constant (c) 5 = MR 2
32
or 2 log r + log = log c Moment of inertia of the shaded portion about
2 r 1 3 13
=0 O is Is = I – Ic = MR 2 – MR 2 = MR2
r 2 32 32
r 1 1 which is choice (c).
or 2 2 % %
r 6 3 53. From the principle of conservation of angular
The negative sign indicates that decreases. Hence I0 0 = I I0 and 0 are the
the correct choice is (b). moment of inertia and angular velocity when the
51. The cylinder will topple when the torque mgr equals beads are at the centre of the rod and I and those
the torque ma
h
(see Fig. 8.69) when the beads are at the ends of the rod.
2 ML2
I0 =
12
ML2 mL2 mL2 L2
and I= (M + 6m)
12 4 4 12

ML2 M 6m L2
0 =
12 12
M 0
or =
M 6m
Hence the correct choice is (b).
54.
centre of mass will remain at rest. Hence the correct
choice is (a).
Fig. 8.69 55. Area of complete disc = R 2. Area of one quarter
1
2 gr g sector OAB = R 2. Mass of this sector = M.
or a = ( h = 4r) (i) 4
h 2 Mass per unit area of the sector is
8.40 Comprehensive Physics—JEE Advanced

M 4M 1 2
mo = 2 2
(1) energy = I .
R /4 R 2
Divide the sector into a 1 1
KE = mv 2 + I 2
large number of cylin- 2 2
drical shells. Consider 1 1 1 2 v 2

an element of mass dm = mv2 + mr


2 2 2 r
at a distance r from the
1
centre O and having ( I= mr 2 )
thickness dr (see Fig. 2
8.70). Then 1 1 3
= mv2 + mv2 = mv2
2 rdr Fig. 8.70 2 4 4
dm = mo
4 Hence the correct choice is (c).
The moment of inertia of the sector about the axis 4
62. Mass of sphere A MA = R3 A
of rotation is 3
4 2 2
R
2 m0
R
m0 R 4 B MB = R3 B IA = MAR2 and IB =
I= r 2 dm r 3 dr (2) 3 5 5
0
4 0
8 2
MB R
1
I= MR2 IA
=
MA
=
A
(a). 2 IB MB B
56. The velocity vCM of the centre of mass can be Hence the correct choice is (c).
obtained by using the principle of conservation of 63. -

MV = (M + m) vCM 2
1 ar = g sin
MV 10 kg 14 ms 3
or vCM = = 10 ms–1
M m 10 4 kg -
Hence the correct choice is (c). celeration is
57. as = g sin
its angular acceleration is always directed down where is the inclination of the plane.
the plane. Hence the frictional force acts up the h
inclined plate when the cylinder rolls up or down on reaching the bottom are given by
vr = 2ar h and vs = 2as h
58. The correct choice is (d).
59. Since as > ar vs > vr
- choice (b).
2
m AB ma 2
- 64. IBC =
stant. Hence the correct choice is (c). 3 3
2
1 2 m BC 4
60. The kinetic energy (which is rotational) is I . IAB = ma 2
2 3 3
1 2 2
I = mr and m AB ma 2
2 IHF =
2 12 12
v 1 1 v
= mr2 m BC 2
ma 2
r 2 2 r IEG =
1 12 3
= mv2 HF is the mini-
4
61. The kinetic energy of a rolling disc consists of two
1 65. MgL
parts: translational energy = mv2 and rotational where M
2
Rigid Body Rotation 8.41

69. Let I1 and 1 be the moment of inertia and angular


1 2 1 ML2 2
MgL = I frequency when his arms are outstretched and I2
2 2 3 and 2 those when his arms are folded. Then
6g . Now v = L = 6g I1 1 = I2 2
or = L 6 gL
L L 3 3
Given I2 = I1. Hence I1 1 = I1 2
Hence the correct choice is (c). 4 4
66. PE at = 60° is Mgh (1 – cos ) where h is the 4
distance between the axis of rotation and the centre or 2 = 1.
3
of mass of the disc. Thus h = R. Gain in KE when 1 2
1 K1 = I1 1
the disc reaches the equilibrium position = I 2 2
2 2
1 2 1 3I1 4 1
2 1 2 2 3 K2 = I2 =
where I = IC.M. + Mx = MR MR MR2. 2
2
2 4 3
2 2
Here x is the distance between the centre of mass 4 1 2 4
= I1 1 K1
x = R. 3 2 3
Now PE = KE gives K2 K1
3 Percentage increase in KE = 100
MgR (1 – cos 60°) = MR 2 2 K1
4 4
2g K1 K1 100
which gives = = 3 100 = = 33.3%
3R K1 3
67. Loss in PE = gain in rotational KE. Thus Hence the correct choice is (a).
1 1 70. The disc is rolling about the point O. Thus the
mgh = (I + mR 2) 2 = (MR2 + mR2) 2
2 2 axis of rotation passes through the point A and is
1 2 perpendicular to the plane of the disc. From the
= R (M + m) 2
2 relation v = r where r is the distance of the point
2mgh
or = . Hence the correct choice is (c).
M m R2 3R
vA vB = (AB) =
68. We obtain the given hollow sphere as if a solid sphere 2
and vC = (AC) = 2R
of radius R has been removed from a solid sphere of
radius 2R. The mass of the given hollow sphere is v B 3R 1 3
Hence = . Thus the correct
(here is the density of the material of the sphere) vc 2 2R 4
M = M1 – M2 choice is (c).
4 4 71. Refer to Fig. 8.71.
where M1 = (2R)3 and M2 = R3 are the
3 3 The angular momen-
masses of spheres of radii 2R and R respectively. tum of the particle at
point P(x y) about
M = 28 R3 (i) origin O is given by
3
L=mr v
The moment of inertia of the given hollow sphere is
2 2 =m xi yj
I= M1 (2R)2 – M2 R2
5 5 Fig. 8.71
= – myv k
2 4 3 2 2 4
= 2R 2R R3 R2
5 3 5 3 ( i i = 0 and j i =– k)
2 4 m and velocity v are constant. Also y
= (32 – 1) R5 (ii)
5 3 remain constant as the particle moves parallel to the
62 x-axis. Hence L remains constant. Thus the correct
I = MR 2
choice (d). 35 choice is (b).
8.42 Comprehensive Physics—JEE Advanced

M mR 2
72. Mass per unit area of the disc = 2
. Therefore, On solving, we get I = . Hence the object is a
R 2
mass of the removed portion (hole of radius R/3) is disc, which is choice (d).
75. Consider a rod OP of length L lying along the x-axis
M R 2 M
m= 2 = with O as the origin (Fig. 8.72). Consider a small
R 3 9 element AB of length dx at a distance x from O.
The moment of inertia of the complete disc about an
axis passing through its centre O and perpendicular
to its plane is
1
I= MR2
2 Fig. 8.72
Using the parallel axes theorem, the moment of iner- k
tia of the removed portion of the disc about the axis Mass of element AB (= dm) = mdx = (xdx)
L
passing through O and perpendicular to the plane of The distance of the centre of mass from O is given by
the disc is L
I = MI of mass m about O + m OO k
x2 d x
2 2
( d m) x L0
1 R 2R xCM = L
= m +m ( d m) k
2 3 3 xd x
L0
1 M R2 M 4R2 1
= + = MR2 x3
L
2 9 9 9 9 18
Therefore, the moment of inertia of the remaining
3 0 L3 / 3 2L
=
2 L 2
L /2 3
1 x
portion of the disc about O = I – I = MR2 –
2 2 0
1 4
MR2 = MR2. Hence the correct choice is (d). The correct choice is (b).
18 9
76. When the tube AB is rotated about its end A in a
73. Let M be the mass of the sphere. The mass of the
horizontal plane with a uniform angular velocity, all
disc will also be M. The moment of inertia of the
points of the tube rotate with same angular velocity.
sphere about its diameter is
Consider a small element of the liquid of length dr
2 at a distance r from the axis of rotation. The mass
Is = MR2
5 of this element is (see Fig. 8.73).
The moment of inertia of the disc about its edge M
m= dr
and perpendicular to its plane is (using parallel axes L
theorem)
Id = Icm + Mh2 = 1 Mr2 + Mr2 = 3 Mr2
2 2
Given Is = Id. Hence, we have
2 3
MR2 = Mr2
5 2
r 2
which gives = , which is choice (a).
R 15
74. From the principle of conservation of mechanical
energy, we have
1 1
mv2 + I 2 = mgh
2 2
1 1 v 2
3v 2
mv2 + I = mg
2 2 R 4g
Fig. 8.73
Rigid Body Rotation 8.43

Force exerted by the element is Since no external torque acts on the system, the
M angular momentum is conserved, i.e.
2 2
dF = m r = r dr L = L or (I + mr2) = I
L
Total force exerted by the liquid at end B is 1
M R2
L M
I 2
2 M 2 L or =
F= r dr r dr I m r2 1
0 L L 0 M R2 m r2
L
2
M 2 r2 1 2
= M L ,which is choice (c). or =
L 2 0
2 2 m r2
1
77. Let be the angular velocity acquired by the sys- M R2
tem (rod + bullet) immediately after the collision. Putting r = R
Since no external torque acts, the angular momen-
79. The block will just begin to slide if the downward
tum of the system is conserved. Thus
force mg sin just overcomes the frictional force,
mvL = I (1)
i.e. if mg sin = μN = μ mg cos tan = μ =
where I is the moment of inertia of the system about
3 = 60° [see Fig. 8.74]
an axis passing through O and perpendicular to the
rod. Thus The block will topple if the torque due to normal
I = M.I. of rod about O + M.I. of bullet stuck at its reaction N about O just exceeds the torque due to
lower end about O mg sin about 0, i.e.
1 1 N OA = mg sin OB
= ML2 + mL2 = (M + 3m)L2 (2)
3 3 15
Using Eq. (1) in Eq. (2), we have mg cos 5 cm = mg sin cm
2
1 2
m vL = (M + 3m) L2 tan = 34°
3 3
3m v Since for toppling is less than for sliding, the
or =
L ( M 3m ) correct choice is (b).
Hence the correct choice is (c).
78. The initial angular momentum of the rotating re-
cord is
L =I
1
where I = MR2.
2
Let be the angular velocity of the record when
the coin of mass m is placed on it at a distance
r from its centre. The angular momentum of the
system becomes Fig. 8.74

L = (I + mr2)

II

Multiple Choice Questions with One or More Choice Correct


1. In the HCl molecule, the separation between the 35.5 1.27
nuclei of hydrogen and chlorine atoms is 1.27 Å. (a) Å from the hydrogen atom
36.5
If the mass of a chlorine atom is 35.5 times that
35.5 1.27
of a hydrogen atom, the centre of mass of the HCl (b) Å from the chlorine atom
molecule is at a distance of 36.5
8.44 Comprehensive Physics—JEE Advanced

1.27
(c) Å from the hydrogen atom
36.5
1.27
(d) Å from the chlorine atom
36.5
2. Choose the correct statements from the following:
(a) The position of the centre of mass of a system
of particles does not depend upon the internal
forces between particles.
(b) The centre of mass of a solid may lie outside Fig. 8.75
the body of the solid.
(c) A body tied to a string is whirled in a circle 6. The moment of inertia of a uniform circular disc of
with a uniform speed. If the string is suddenly mass M and radius R about its centre and normal
1
cut, the angular momentum of the body will to its plane is MR2. Then
not change from its initial value. 2
(d) The angular momentum of a comet revolving (a) its radius of gyration about the centre is R.
around a massive star, remains constant over (b) the moment of inertia of the disc about its di-
1
the entire orbit. ameter is MR2.
3. Which of the following statements are correct? 4
(c) the moment of inertia of the disc about an axis
(a) When a body rolls on a surface, the force passing through a point on its edge and normal
of friction acts in the same direction as the 3
direction of motion of the centre of mass of to the disc is MR2.
2
the body.
(d) the moment of inertia of the disc about a tangent
(b) During rolling, the instantaneous speed of the 5
point of contact is zero. in the plane of the disc is MR2.
4
(c) During rolling, the instantaneous acceleration
7. A molecule consists of two atoms, each of mass m,
of the point of contact is zero.
separated by a distance a. The rotational kinetic en-
(d) A wheel moving down a perfectly frictionless
ergy of the molecule is K and its angular frequency
inclined plane will slip and not roll on the
is . I is the moment of inertia of the molecule about
plane.
its centre of mass. Then
4. In which of the following is the angular momentum 1
conserved? (a) I = ma2 (b) I = ma2
2
(a) The planet Neptune moves in an elliptical
orbit round the sun with the sun at one of 1 K 2 K
(c) = (d) =
the foci of the ellipse a m a m
8. A circular ring of mass m and radius r rolls down
elliptical orbit round the nucleus an inclined plane of height h. When it reaches the
(c) An -particle, approaching a nucleus, is scat- bottom of the plane its angular velocity is and its
tered by the force of electrostatic repulsion rotational kinetic energy is K. Then
between the two 1 1
(d) A boy whirls a stone, tied to a string, in a (a) = gh (b) = gh
r 2r
horizontal circle
1
5. Four tiny masses are connected by a rod of (c) K = mgh (d) K = mgh
negligible mass as shown in Fig. 8.75. 2
9. A rope is wound round a solid cylinder of mass M
(a) The moment of inertia of the system about axis
and radius R. If the rope is pulled with a force F, the
AB is 50 ma2
(b) The radius of gyration od the system about cylinder acquires an anglular acceleration and the
axis AB is 5 a. rope acquires a linear acceleration a. Then
(c) The moment of inertia of the system about F 2F
(a) = (b) =
axis CD is 10 ma2 MR MR
(d) The radius of gyration of the system about 2F F
axis CD is a. (c) a = (d) a =
M M
Rigid Body Rotation 8.45

10. A solid sphere rotating about its diameter at an (c) Frictional force = mg cos , where m is the
angular frequency has rotational kinetic energy K. mass of the cylinder and
1 friction between the cylinder and the plane.
When it is cooled so that its radius reduces to of
n (d) Frictional force helps rotational motion of
its original value, the new values of and K become the cylinder but opposes its translational mo-
and K respectively. Then tion.
13. A spherical ball is released from rest from point A
(a) =n (b) = n2 on a hemispherical surface and it rises up to a point
C as shown in Fig. 8.78. Part AB of the surface is
K K rough and the ball rolls from A to B without slip-
(c) =n (d) = n2
K K ping. Part BC of the surface is frictionless. KA, KB
11. Three forces act on a wheel of radius 20 cm as shown and KC are kinetic energies of the ball at points A,
in Fig. 8.76. B and C respectively. Which of the following is/are
correct?
(a) KA = KC, hA = hC (b) KB > KA, hA < hC
(c) KB > KC, hA > hC (d) KC > KA, hA > hC

Fig. 8.78

14. The moment of inertia of a uniform disc about its


diameter is I. Then the moment of inertia about an
axis
(a) passing through its centre and perpendicular
to its plane is 2I.
Fig. 8.76 (b) tangential to its plane is 5I.
(a) The torque produced by the force of 8N is (c) tangential and perpendicular to its plane is 6I.
0.8 Nm clockwise. (d) all the above choices are correct.
(b) The torque produced by the force of 4 N is 15. A wire of mass M, length L and density is bent to
0.8 Nm anticlockwise. form a circular ring of radius R. Then, the moment
of inertia of the ring about its diameter is
(c) The torque produced by the force of 9 N is 1
zero. (a) MR2 (b) R4L /2
2
(d) The net torque produced by the forces is ML2 R 2 L3
1.8 Nm clockwise. (c) (d)
12. A solid cylinder rolls down a rough inclined plane 8 2 8
without slipping as shown in Fig. 8.77. Choose the 16. A uniform solid sphere and a solid cylinder of the
correct statement (s) from the following. same mass and the same diameter are released from
rest on the top of an inclined plane of inclination .
If they roll down the plane without slipping, then
(a) the acceleration of each down the plane is
g sin .
(b) the ratio of the accelerations of the sphere
and the cylinder is 15:14.
(c) the ratio of the times taken by the sphere and
Fig. 8.77 the cylinder to reach the bottom of the plane
(a) If is decreased, the force of friction will is 14 : 15 .
decrease. (d) the sphere and the cylinder will reach the
(b) Frictional force is dissipative. bottom with the same speed.
8.46 Comprehensive Physics—JEE Advanced

17. A rope is wound around a hollow cylinder of mass (a) the velocity of centre of mass is zero.
3 kg and radius 20 cm. The rope is pulled with a (b) the angular speed of the bar with the masses
constant force of 30 N. If is the angular accelera- v
tion of the cylinder and a the linear acceleration of stuck to it is .
5a
the rope, then
(c) the moment of inertia of the bar with masses
(a) = 50 rad s–2 (b) = 40 rad s–2
–2 stuck to it about the axis passing through the
(c) a = 30 ms (d) a = 10 ms–2
18. A rope of negligible mass is wound around the end of the bar and perpendicular to its plane
circumference of a bicycle wheel (without tyre) of is 30 ma2.
3
diameter 1 m. A mass of 2 kg is attached to the end (d) the total energy of the bar mv2.
of the rope and is allowed to fall from rest. The mass 5
falls 2 m is 4 s. The axle of the wheel is horizontal IIT, 1991
and the wheel rotates in the vertical plane. Take g = 22. A disc of mass M and radius R is rolling with
10 ms–2 and neglect the friction due to air. angular speed on a horizontal surface as shown
(a) the linear acceleration of the wheel is in Fig. 8.79. The magnitude of angular momentum
0.25 m s–2. of the disc about the origin O is (here v is the linear
(b) the angular acceleration of the wheel is 0.5 velocity of the disc)
rad s–2.
(c) the magnitude of the torque acting on the
wheel is 10 Nm.
(d) the moment of inertia of the wheel about the
horizontal axle is 20 kg m2.
19. A smooth sphere A is moving on a horizontal friction-
less surface with angular speed and centre of mass
velocity v. It collides head-on with an identical sphere Fig. 8.79
B at rest. After the collision their angular speeds are 3
A and B respectively. If the collision is elastic and
(a) MR2 (b) MR2
2
the friction is neglected, then 3
(a) A = (b) B = 0 (c) MRv (d) MRv
(c) A < B (d) A = B 2
IIT, 1999 IIT, 1999
20. The torque acting on a body about a given point is 23. Which of the following statements is/are correct
about a particle moving in a circle with a constant
given by = A L where A is a constant vector speed?
and L is the angular momentum of the body about (a) The linear velocity and acceleration vectors
that point. It follows that are perpendicular to each other.
dL (b) The linear velocity vector is always perpen-
(a) is perpendicular to L at all instants of
dt dicular to the angular velocity vector.
time. (c) The force acting on the particle is radial.
(b) the component of L in the direction of A (d) The force does no work on the particle.
does not change with time. 24. The position vector of a particle with respect
(c) the magnitude of L does not change with toorigin O is r . If the torque acting on the particle
time. is zero, then
(d) All the above choices are correct. (a) the linear momentum of the particle remains
IIT, 1998 constant.
21. A uniform bar of length 6a and mass 8 m lies on (b) the angular momentum of the particle about
a horizontal frictionless table. Two point masses O remains constant.
m and 2m moving in opposite directions but in
(c) the force applied to the particle is perpendicular
the same horizontal plane with speeds 2v and v
respectively strike the bar at distance a and 2a from to r .
one end and stick to the bar after the collision. Then (d) the force applied to the particle is parallel to
after the collision r.
Rigid Body Rotation 8.47

25. A block of mass m is connected to a spring of a mass m which is moved in a horizontal circle of
spring constant k radius R to form a conical pendulum. If the string
r as shown in Fig. 8.80. The mass of the pulley is makes an angle = 60° with the vertical, then
2m. The block is pulled down by x0 from the (a) the speed of the body along the circle is
equilibrium position and released. The spring has v = Rg .
negligible mass. Then (b) the tension in the string is 2mg.
1 2 (c) the horizontal component of the angular mom-
(a) the total energy of the system is kx0 .
2 3 Lg
(b) the velocity of the block when it is at a entum of the body point O is mL .
2 2
distance x from the equilibrium position is (d) the magnitude of the torque acting on the
1/ 2
k
x02 x 2 . 3
2m body about point O is mgL.
2
(c) the velocity of the block is maximum when 29. A solid cylindrical roller of mass M and radius R is
x = x 0. rolled on a rough horizontal surface by applying a
(d) the velocity of the block is zero when x = x0. horizontal force F. If aCM is the linear acceleration
of the centre of mass and f is the frictional force
between the roller and the surface, then
F 2F
(a) aCM = (b) aCM =
M 3M
F
(c) f = (d) f = zero
3
30. If the resultant of all the external forces acting on
a system of particles is zero, then for an inertial
frame, one can surely say that
(a) linear momentum of the system does not
change in time
Fig. 8.80 (b) kinetic energy of the system does not change
26. A rod of mass M = 0.9 kg and in time
length L = 1 m is suspended (c) angular momentum of the system does not
at O as shown in Fig. 8.81. change in time
A bullet of mass m = 100 g (d) potential energy of the system does not
moving with velocity v = 80 change in time
ms–1 in the horizontal direction IIT, 2009
strikes the end P of the rod and 31.
gets embedded in it. If I is the horizontal plane surface. In Fig. 8.82, A is the point
moment of inertia of the system of contact, B is the centre of the sphere and C is its
and is the angular velocity topmost point. Then,
immediately after the collision, (a) VC VA = 2 V B VC
then
(a) I = 0.4 kgm2 (b) VC VB = VB VA
(b) I = 0.3 kgm2 Fig. 8.81 (c) VC VA = 2 VB VC
(c) = 20 rad s–1
(d) = 26.7 rad s–1 (d) VC VA = VB
27. A wheel is initially at rest. A constant torque acts
on it for a time t. As a result, the wheel acquires an
angular acceleration and angular velocity . If
the angular displacement produced is , then
(a) t2 (b) = constant
(c) t (d) power Fig. 8.82
28. A string of length L and of negligible mass hangs
from a support O. The other end of the string carries IIT, 2009
8.48 Comprehensive Physics—JEE Advanced

32. A thin ring of mass 2 kg and radius 0.5 m is rolling (d) there is no friction between the ring and the
without slipping on a horizontal plane with velocity ground.
1 m/s. A small ball of mass 0.1 kg, moving with velocity
20 m/s in the opposite direction hits the ring at a height
of 0.75 m and goes vertically up with velocity 10 m/s.
Immediately after the collision [See Fig. 8.83]
(a) the ring has pure rotation about its stationary
CM.
(b) the ring comes to a complete stop. Fig. 8.83
(c) friction between the ring and the ground is
IIT, 2011
to the left.

ANSWERS AND SOLUTIONS


1. Since, most of the mass of an atom is concentrated acceleration of the point of contact is not zero.
in its nucleus and the size of a nucleus (which is of Statement (d) is correct. Rolling cannot take place
the order of 10–15 m) is very small compared to the in the absence of friction because it is the friction-
separation between the hydrogen and chlorine atom al force that provides the necessary torque which
(which is 1.27 10–10 m), the atoms can be treated makes the body roll on a surface. Hence the correct
as point masses. choices are (a), (b) and (d).
The centre of mass of HCl molecule will be on 4. The angular momentum is conserved in the four
the line joining the two atoms. Let us say that the cases. The object, in each case, is moving under
H atom is located at, say, x = 0 and the Cl atom the action of a central (radial) force. The torque
at x = x. Let xCM be the position of the centre of dL
mass between x = 0 and x = x. If m1 and m2 are the due to a radial force is zero. Since = ; the
dt
masses of H and Cl angular momentum L does not change with time.
of centre of mass, we have
5. All four choices are correct.
m 0 m2 x m2 x The moment of inertia about AB is
xCM = 1 =
m1 m2 m1 m2 I = m1r 21 + m2r 22 + m3r23 + m4r24
Now x = separation between H and Cl atoms = 1.27 = m 0 + 2m (a)2 + 3m (2a)2 + 4m (3a)2
Å and m2 = 35.5 m1. Hence = 0 + 2 ma2 + 12ma2 + 36ma2 = 50ma2
35.5 m1 1.27 Å The radius of gyration K is given by
xCM = = 35.5 1.27Å
m1 35.5 m1 36.5 I = MK2 = (m1 + m2 + m3 + m4)K2
This gives the distance of the centre of mass from = (m + 2m + 3m + 4m)K2 = 10mK2
the hydrogen atom. The distance of the centre of I = 50ma 2 =
mass from the chlorine atom is or K= 5a
10m 10m
1.27 Å – 35.5 1.27 Å = 1.27Å The moment of inertia about CD is
36.5 36.5 I = m (2a)2 + 2 m (a)2 + 3m 0 + 4 m (a)2
Hence the correct choices are (a) and (d). = 4 ma2 + 2ma2 + 0 + 4ma2 = 10ma2
2. The only incorrect statement is (c). Since no external The radius of gyration is
torque acts on the body even after the string is cut,
the angular momentum will remain unchanged. I 10ma 2
= = =a
3. Statement (a) is correct. The direction of motion of 10ma 10m
the centre of mass is the direction along which the 6. The correct choices are (b), (c) and (d).
body rolls. Since the force of friction is opposite to Let us consider two perpendicular diameters, one
the direction of the velocity of the point of contact, along the x-axis and the other along the y-axis.
the force of friction acts in the direction of motion Then
of the centre of mass. Statement (b) is also correct. 1
Ix = Iy = MR2
At each instant of time, the point of contact is mo- 4
mentarily at rest. Statement (c) is incorrect. Since According to the perpendicular axes theorem, the
the body is rotating while it is rolling, the direction moment of inertia of the disc about an axis passing
of the velocity is changing with time. Hence the through the centre is
Rigid Body Rotation 8.49

1 1 FR FR 2F
Ic = Ix + Iy = MR2 + MR2 = = =
4 4 I 1 2 MR
MR
1 2
= MR2
2 2F
Also a= R=
which is choice (b). M
Since the disc is uniform, its centre of mass co- Thus the correct choices are (b) and (c).
incides with its centre. Therefore, the moment of 10. The correct choices are (b) and (d).
inertia of the disc about an axis passing through its r 2 I
2 2
centre of mass and normal to its plane is I =I ,I= mr2, I = m = 2.
1 5 5 n n
ICM = IC = MR 2 I
2 Hence I = 2
According to the theorem of parallel axes, the mo- n
ment of inertia of the disc about an axis passing or = n2
through a point on its edge and normal to its plane 1 1 1 I
is given by K = I 2, K = I 2 = 2
(n2 )2
1 2 2 2 n
Ie = ICM + M h2 = MR 2 + MR2 1 2 2
2 ( h = R) = I n = n2K.
2
3 2
= MR . 11. The torques produced by forces 8 N, 4 N and 9 N
2
From the parallel axes theorem, the moment of in- respectively are
1 1 = 8 N 0.2 m sin 30° = 0.8 Nm (clockwise)
ertia of the disc about a tangent is MR2 + MR2 = 2 = 4 N 0.2 m sin 90° = 0.8 Nm (anticlockwise)
4
5 2 3 = 9 N 0.2 m sin 90° = 1.8 Nm (clockwise)
MR .
4 Net torque = 0.8 – 0.8 + 1.8 = 1.8 Nm clockwise
7. The correct choices are (b) and (d). Hence the correct choices are (a), (b) and (d).
Since the two atoms have the same mass, the cen- 12. Refer to the Fig. 8.84. Here f is the frictional force.
tre of mass is at a distance of a/2 from each atom. The linear acceleration of the centre of mass of the
Therefore, the moment of inertia of the molecule rolling cylinder is given by
about its centre of mass is g sin
2 2
acm = (1)
a a ma 2 I cm
I=m =m = 1
2 2 2 mR 2
where R is the radius of the cylinder and Icm is the
1 2 moment of inertia of the cylinder about the centre
Kinetic energy is k = I , which gives
2 of mass which is given by
2k 2k 2 2 k 1
= = = Icm = mR2
I ma 2
1 a m 2
1 1 1 1
8. mgh = mv2 + I 2
= mv2 + (mr2) 2
2 2 2 2
( I = mr2)
1 2 1 2
= mr 2
+ mr 2
( v=r )
2 2
Fig. 8.84
= mr2 2
1 Using this in Eq. (1), we have
= gh . Also
r g sin
1 2 1 gh 1 2 g sin
K= I = (mr2) mgh acm = mR 2 = (2)
2 = 1 3
2 2 r 2 2mR 2
Hence the correct choices are (a) and (d). Now, for linear motion, we have
1
9. I = MR2 and torque = FR = I . Hence mgsin – f = macm (3)
2
8.50 Comprehensive Physics—JEE Advanced

Using Eq. (2) in Eq. (3), we get law of conservation of energy EA = EC, i.e. mg hA
2mg sin 1 1
mgsin – f = = I 2 + mg hC or mg (hA – hC) = I 2. Since the
3 2 2
mg sin right hand side of this equation is positive, hA > hC.
which gives f = (4) Hence the correct choices are (c) and (d).
3
14. From perpendicular axes theorem, Ix + Iy = Ic. Hence
It follows from Eq. (4) that if is decreased, f will
Ic = I + I = 2 I. [see Fig. 8.85(a)]
also decrease. Hence choice (a) is correct. As the
1
cylinder is rolling down, the point of application of we knows that Ic = MR2 MR2 = 2Ic = 4I.
the frictional force is at rest at any given instant. 2
Hence no work is done by the frictional force, i.e.
the frictional force is not dissipative. Therefore,
statement (b) is wrong. Statement (c) is also wrong
as f is given by Eq. (4). Statement (d) is correct
because the frictional force provides torque = f R
to help rotational motion but it will oppose transla-
tional motion. Hence the correct choice are (a) and
(d).
13. The ball is at rest at point A. Hence its kinetic en-
ergy KA (rotational + translational) is zero, it has
only gravitational potential energy mghA. As it is
released at A, it begins to roll (due to friction in part
AB) thus acquiring kinetic energy at the expense of
gravitational potential energy. When it reaches point
B, its kinetic energy consists of both rotational and
translational energy. At point B its potential energy
1
is zero. At point B, the rotational kinetic energy is
2
I 2, where is the angular velocity at point B and I Fig. 8.85
is the moment of inertia of the ball about its centre.
Since part BC is frictionless, the torque on the ball is From parallel axes theorem [see Fig. 8.84 (b)], we
zero. Hence its angular momentum L = I remains have
constant in part BC. Hence, the angular velocity IAB = Ix + MR2 = I + 4 I = 5 I.
of the ball remains constant in part BC. The transla- Using parallel axes theorem [see Fig. 8.84 (c)], we
tional kinetic energy is converted into gravitational have
potential energy. At point C, the translational kinetic IPQ = Ic + MR2 = 2I + 4I = 6 I.
energy of the ball is zero; it has rotational kinetic Hence the correct choice is (d).
1 M
energy I 2 and gravitational potential energy 15. = M = R2L (1)
2 R2 L
mg hC. Thus L
KA = 0 (1) L =2 R R= (2)
2
1 1 The moment of inertia of a ring about an axis
KB = mv2 + I 2 (2)
2 2 passing through its centre and perpendicular to its
where v is the linear velocity of the centre of mass plane = MR2. From the parallel axes theorem, the
of the ball and moment of inertia of the ring about its diameter =
1 1
KC = I 2 (3) MR2
2 2
It follows from (1), (2) and (3) that KB > KC > KA. four choices are correct.
Now total energy at A is EA = 0 + mghA = mghA and g sin
16. Acceleration a =
1 I
total energy at C is EC = I 2 + mg hC. From the 1
2 MR 2
Rigid Body Rotation 8.51

If s is the distance along the inclined plane, the Differentiating, we have


velocity when the body reaches the bottom of the d d 2
(L L) = (L )
plane is given by v2 – 0 = 2as v = 2as and the dt dt
time taken to reach the bottom is
1/ 2 dL dL dL
I L + L =2L
2s 1 dt dt dt
t = 2s = MR 2
a g sin dL dL
2L =2L
dt dt
1
For cylinder: Ic = MR2 dL dL
2 Since L ; L = 0. Hence
2 dt dt
For sphere: Is = MR2
5 dL dL
L =0 = 0 L = constant. Hence choice
2 g sin 5 g sin dt dt
Hence ac = and as =
3 7 dL
(c) is correct. Since A , choice (b) is also cor-
as 15 dt
giving = rect. Thus the correct choice is (d).
ac 14
21. Since no external force is applied, the linear mo-
vs as t a
Also = and s = c mentum is conserved. Hence
vc ac tc as
(8 m + m + 2 m)vCM = 2m(– v) + m(2v) + 8m 0
Thus the correct choices are (b) and (c).
where vCM is the velocity of the centre of mass.
17. I = MR2 and = FR. This gives vCM = 0.
FR F 30 The moment of inertia of the system is
= = 2 = = = 50 rad s–2
I MR MR 3 0.2 1
I = 2ma2 + m(2a)2 + 8m (6a)2
a = R = 0.2 50 = 10 ms–2 12
The correct choices are (a) and (d). = 2ma2 + 4ma2 + 24ma2 = 30 ma2
1 1 Since no external torque is applied, the angular
18. s = ut + at2 2=0+ a (4)2
2 2 momentum of the system is conserved. If is the
a = 0.25 ms–2. angular speed, then
a 0.25 2 mv a + m 2v 2a = I 2
= = = 0.5 rad s–2 v
R 0.5 6mva = 30ma2 2 =
= mg moment arm = 2 10 0.5 = 10 Nm 5a
The system has no translational K.E. Hence
10
I= = = 20 kg m2 Total K.E. = K.E. of rotation
0.5
1 2
Hence all the four choices are correct. = I
19. Since the collision is elastic and head-on and the 2
spheres have the same mass, they will exchange 1 v 2 = 3 mv2
their velocity after the collision, i.e. A comes to =30 ma2
2 5a 5
rest and B moves with velocity v. Since there is no
Hence all the four choices are correct.
friction, the torque on each sphere is zero. Hence
22. The angular momentum about O is
their angular speeds remain unchanged on colli-
sion. Thus the correct choices are (a) and (b). LO = LCM + M R v
20. Given = A L (1) Its magnitude is R v and LCM = I
dL dL
We know that = . Hence = A L . This LO = I + MRv
dt dt
1
dL = MR 2 + MR R ( v=R )
means that is perpendicular to both L and A . 2
dt
Hence choice (a) is correct. 3
= MR2
Now L L = L2, where L is the magnitude of L . 2
8.52 Comprehensive Physics—JEE Advanced

3 v 3 28. Refer to Fig. 8.86. OA = OB = OC = L


= MR2 = MRv R = l sin , OD = L cos
2 R 2
Hence the correct choices are (a) and (d).
23. All the four choices are correct. The linear veloc-
ity v , angular velocity and radius vector r are
related as
v = r
Hence v is perpendicular to both and r . Hence
choice (b) is correct. The acceleration (and hence
force) is centripetal (towards the centre) and v is
r . Hence choice (a) and (c) are correct. Since F
is r , the work done is zero. Hence choice (d) is
also correct. Fig. 8.86
dL
24. = r F and = . Hence the correct choices T sin = mg (1)
dt
are (b) and (d). mv 2
T cos = (2)
1 R
25. Total energy before the block is released = kx20.
2 Dividing (1) by (2), we get
After the block is released, the total energy when v = Rg tan = Rg tan 60 = 1.73Rg
it is at a distance x from the equilibrium position
Hence choice (a) is incorrect. From (1) and (2) we
= K.E. of block + rotational K.E. of pulley + P.E.
get
1 1 1
stored in the spring = mv2 + Ip 2 + kx2. m2
2 2 2 T 2 = (mg)2 + (v2)2
R2
1 1
where Ip = m pr 2 = (2m)r2 = mr2. m2
2 2 = (mg)2 + (Rg tan 60°)2
From the principle of conservation of energy, we R2
have ( v = r ) = (mg)2 + 3(mg)2 = 4(mg)2
1 2 1 1 v 2 1 2 T = 2 mg
kx0 = mv2 + (mr2) + kx
2 2 2 r 2 Horizontal component of angular momentum about
1/ 2 O
k
which gives v = x02 x 2 . = mv OD = mv L cos
2m
Thus v is maximum when x = 0. Hence the correct =m Rg tan L cos
choices are (a), (b) and (d).
= mL cos Lg tan sin
1 1
26. I = mL + ML2 = 0.1 (1)2 +
2
0.9 (1)2 = mL sin Lg cos
3 3
= 0.4 kgm2 = mL sin 60° Lg cos 60
From conservation of angular momentum, we have
mvL = I 3 Lg
= mL , which is choice (c).
mv L 0.1 80 1 2 2
= = = 20 rad s–1.
I 0.4 Torque about O = mgR = mgL sin
The correct choices are (a) and (c). 3
= mgL sin 60° = mgL
27. = 0+ t = 0 + t = t. Since is constant 2
Hence the correct choices are (b), (c) and (d).
and = I , it follows that is constant. Hence t.
29. Since the frictional force acts opposite to the direction
1 2
Also = t . Hence t2. Also power P = . of motion, the equation of translational motion is
2
Hence P . Thus all the four choices are correct. F – f = MaCM (1)
Rigid Body Rotation 8.53

For rotational motion, we have


VB V A = Vi
=I
Since the torque is due to frictional force, = f R. Hence the correct choice are (b) and (c)
1 a CM 32. Let M be the mass of the ring and m that of the ball
Hence fR = MR 2
2 R and let V and v be their velocity before collision.
1 aCM The initial momentum of the system (ring and ball)
I MR 2 ,
2 R in the horizontal direction is
MaCM pi = MV mv
or f= (2)
2 =2 1 + 0.1 (–20)
2F F
Equations (1) and (2) give aCM = and f = . =2 – 2 = 0
3M 3
So the correct choices are (b) and (c). -

30. The only correct choice is (a) tum of the system pf = 0 in the horizontal direc-
31. Refer to Fig. 8.87. tion. Hence Vcm = 0 for the ring, i.e. the ring has
pure rotation about its centre of mass. So choice (a)
is correct.
The total initial angular momentum of the system
about the point of collision is
Li = mvr – I
V
= mvr – MR2
R
= mvr – MRV
Fig. 8.87
= 0.1 20 0.75 – 2 0.5 1
2 –1
VA = 0, Let V B = V i , then V C = 2V i = 1.5 – 1 = 0.5 kg m s
V C – V A = 2V i – 0 = 2V i From the conservation of angular momentum, the

V C – V B = 2V i V i = V i the friction between the ring and the ground is to


VB VC = Vi the left. So the correct choices are (a) and (c).

III

Multiple Choice Questions Based on Passage


Questions 1 to 2 are based on the following passage angular momentum of a fundamental particle is quantized
Passage I and its value is given by
h
Quantization S=n
2
In physics some measurable quantities are quantized. A where h is the Planck’s constant = 6.63 10–34 Js and n is
physical quantity is said to be quantized if it can have a number called the spin quantum number. The value of
only discrete (not continuous) values. Some examples of n for electrons, protons, positrons and antiprotons can
quantities which are quantized are mass, charge and energy. 1 1
For sub-atomic particles (called fundamental particles) be + and – . Pions have n = 0.
2 2
the angular momentum is quantized. Fundamental
particles such as electrons and protons have a certain 1. Which of the following is not quantized ?
intrinsic angular momentum of their own. This angular (a) Mass (b) Energy
momentum is called spin angular momentum. The spin (c) Linear momentum (d) Charge
8.54 Comprehensive Physics—JEE Advanced

2. Choose the correct statements from the following. h h


(a) Electric charge on a charged body can only be can be + or – .
4 4
an integral multiple of the smallest possible
charge. (d) The spin angular momentum of a pion is
(b) Energy can have only discrete values. h h
(c) The spin angular momentum of an electron + or – .
2 2
ANSWERS
1. The correct choice is (c).
2. The correct choices are (a), (b) and (c). The spin
angular momentum of a pion is zero.

Questions 3 to 5 are based on the following passage (a) retards the motion of the sphere
Passage II (b) makes the sphere move faster
(c) has no effect on the motion of the sphere
A hollow sphere of mass M and radius R is initially at rest (d) is independent of the velocity of the sphere.
on a horizontal rough surface. It moves under the action of 4. The linear acceleration of the sphere is
a constant horizontal force F as shown in Fig. 8.88.
10 F 7F
(a) a = (b) a =
7M 5M
6F F
(c) a = (d) a =
5M M
5. The frictional force between the sphere and the
surface is
F F
Fig. 8.88 (a) (b)
2 3
3. The frictional force between the sphere and the F F
surface (c) (d)
4 5
SOLUTIONS
3. If the horizontal force F is applied at the centre of F + f = Ma (1)
mass of the sphere, then the frictional force opposes The magnitude of the net torque acting on the
the translational motion of the sphere. If force F is sphere = FR – f R. Hence, for rotational motion the
applied above the centre of mass, the torque due equation is
to frictional force tends to rotate the sphere faster. Ia
FR – f R = I = ( a = R)
Hence, in this case, frictional force f acts in the di- R
rection of motion, as shown in Fig. 8.89. Thus the 2
For a hollow sphere, I = MR2. Hence
correct choice is (b). 3
2 a 2
FR – f R = MR2 = MRa
3 R 3
2
F – f = Ma (2)
3
6F
Equations (1) and (2) give a = , which is choice
Fig. 8.89 (c) 5M
4. Let a and be the linear and angular accelera- Ma F
5. From Eqs. (1) and (2) we get f = = . Hence
tions of the sphere respectively. For translational 6 5
motion, the correct choice is (d).
Rigid Body Rotation 8.55

Questions 6 to 11 are based on the following passage 8. The moment of inertia of sphere B about side AD
Passage III is
Four solid spheres each of mass m and radius r are located 2 2 m
(a) mr (b) (5a2 + 2r2)
with their centres on four corners of a square ABCD of 5 5
side a as shown in Fig. 8.90. m m
(c) (2r2 + a2) (d) (3a2 + 5r2)
5 5
9. The moment of inertia of sphere D about side
AD is
2 2 2
(a) mr (b) m(r2 + a2)
5 3
m m
(c) (3a2 + 4r2) (d) (5a2 + 2r2)
5 5
Fig. 8.90 10. The moment of inertia of the system of four spheres
6. The moment of inertia of sphere A about diagonal about diagonal AB is
AB is m m
(a) (8r2 + 5a2) (b) (7r2 + 4a2)
2 2 2 2 5 5
(a) mr (b) mr
3 5 m m
(c) (5r2 + 8a2) (d) (3r2 + 5a2)
a2 a2 5 5
(c) m r2 (d) m r2
4 4 11. The moment of inertia of the system of four spheres
7. The moment of inertia of sphere C about diagonal about side AD is
AB is 2m m
(a) (2r2 + 5a2) (b) (7r2 + 5a2)
2 2 2 5 5
(a) mr (b) (2r2 + 3a2)
5 5 2m m
(c) (4r2 + 5a2) (d) (3r2 + 5a2)
m m 5 5
(c) (5r2 + 3a2) (d) (4r2 + 5a2)
5 10

SOLUTION
Refer to Fig. 8.91. 2
their centre and parallel to AB = mr2 each. The distance
5
of this axis (shown by broken lines) from the diagonal AB
= a/ 2 . From the parallel axes theorem, the moment of
inertia of spheres C and D about diagonal AB is
2 2 a 2 2 m a2
mr2 + m(CO)2 = mr2 + m = mr2 +
5 5 2 5 2
6. The correct choice is (b).
7. The correct choice is (d).
8. The correct choice is (b).
9. The correct choice is (a).
10. The moment of inertia of the system of four spheres
about diagonal AB is
IAB = MI of A about AB + MI of B about AB
Fig. 8.91
+ MI of C about AB + MI of D about AB
The moment of inertia of spheres A and B about their 2 2 2 1
2 = m r2 m r2 m r2 m a2
common diameter AB = mr2 each. Also the moment of 5 5 5 2
5 2 1
inertia of spheres C and D about an axis passing through + mr 2 ma 2
5 2
8.56 Comprehensive Physics—JEE Advanced

8r 2 IAD = MI of A about AD + MI of D
8 a2
= mr2 + ma2 = m 5
about AD + MI of B about AD +
5 MI of C about AD
The correct choice is (a).
11. Moment of inertia of sphere A about side AD = =
2
m r2
2
m r2
2
m r 2 m a2
moment of inertia of sphere D about side AD = 5 5 5
2 2
mr2. Using the parallel axes theorem, moment of m r 2 m a2
5 5
inertia of sphere C about AD = moment of inertia
8 8r2
of sphere B about AD =
2
mr2 + ma2. Hence the = mr2 + 2 ma2 = m 2 a2
5 5 5
moment of inertia of the system of four spheres
about side AD is The correct choice is (c).

Questions 12 to 15 are based on the following passage 1 2


Passage IV (a) s (b) s
5 5
A small sphere rolls down without slipping from the top (c) 0.1 s (d) 0.2 s
of a track in a vertical plane. The track has a elevated 14. If g = 10 ms , the distance on the ground with
–2
section and a horizontal part. The horizontal part is 1.0 m respect to point B (which is vertically below the end
above the ground and the top of the track is 2.4 m above A of the track) is
the ground. (See Fig. 8.92) (a) 1.0 m (b) 1.4 m
(c) 2.0 m (d) 2.8 m
15. Choose the correct statement/statements from the
following.
(a) During its motion as a projectile after the
sphere leaves the track at A, it will stop ro-
tating.
(b) During its motion as a projectile after point
Fig. 8.92 A, the sphere will continue to rotate about its
12. If g = 10 ms , the horizontal velocity when the
–2 centre of mass.
sphere reaches point A is (c) Due to rotation, the horizontal range of the
(a) 5 ms–1 (b) 2 5 ms–1 sphere will be less than that found in Q.14
above.
(c) 7 ms–1 (d) 2 7 ms–1
(d) The rotation of the sphere has no effect on
13. If g = 10 ms , the time taken by the sphere to fall
–2
the horizontal range found in Q.14.
through h = 1.0 m is

SOLUTION

12. The loss in potential energy when the sphere moves 1 1 7


from the top of the track to point A = gain in total = Mv2 + Mv2 = Mv2
2 5 10
kinetic energy (translational and rotation), i.e. 1/ 2
1 1 or v = 10( H h) g
Mg(H – h) = Mv2 + I 2 7
2 2
10 (2.4 1.0) 10 1 / 2
2 v = =2 5 ms–1
where I = MR2 and = . Thus 7
5 R
The correct choice is (b).
1 1 2 2
Mg(H – h) = Mv2 + MR2 v 13. Since the vertical component of velocity is zero, the
2 2 5 R
Rigid Body Rotation 8.57

2h 2 1.0 1
t= = = s, which is choice (a). -
g 10 5
1
14. Horizontal range = vt = 2 5 = 2.0 m, which
5 -
is choice (c).
15. A and (c).

Questions 16 to 18 are based on the following passage


Passage V in case (b) is
M and radius R 2 2
(a) (b)
s and 3
inclination
5 7
(c) (d)
7 3
16. 18.
(a) to that in case (b) is the bottom in case (a) to that in case (b) as
2
(a) 1 (b) 3
3
5 7 (a) 1 (b) 2
(c) (d)
7 9 2
(c) 2 (d)
17. 3
SOLUTION
16. 17. Using v2 = 2as
g sin v1 5
I 5 2 = , which is choice (c).
a1 = 1 = g sin I MR 2 v2 7
MR 2 7 5
1 2
In case (b), the acceleration is 18. From s = at
a2 = g sin 2
is (d).
Hence the correct choice is (c).

Questions 19 to 22 are based on the following passage 20. Tension T1 is


Passage VI mg 3 mg
(a) (b)
m1 = 3 m and m2 = m are attached to 5 5
7 mg 9 mg
M = 2 m and radius (c) (d)
5 5
R) as shown in Fig. 8.93. The 21. Tension T2 is
masses are then released.
9 mg 7 mg
19. (a) (b)
5 5
the system is 3 mg
(a) 2 g (c) mg (d)
5
(b)
2g 22.
3 2 mgR
(a) 2 mgR (b)
2g 3
(c)
5 2 mgR
(c) (d) 3 mgR
3g 5
(d)
7
Fig. 8.93
8.58 Comprehensive Physics—JEE Advanced

SOLUTION
= (T1 – T2)R (3)
1
Also, = I where I = MR2 and is the angular
2

a
=
R
1 a 1
= MR2 = MRa = m Ra
2 R 2 ( M = 2m)

T1 – T2 = ma (5)
19.
2g
get a = . Hence the correct choices is (c).
5
20. T1 = 3m(g – a) =
2g 9 mg
Fig. 8.94 3m g = ; which is choice (d).
5 5
2g
tensions T1 and T2 a is the acceleration 21. T2 = m(g + a) = m g
5
7 mg
m 1g – T 1 = m 1a 3mg – T1 = 3ma (1) = so choice (b) is correct.
5
and T 2 – m 2 g = m 2a T2 – mg = ma (2)
2 g 2mgR
The resultant tension (T1 – T2 22. = mRa = mR = , which
5 5
is choice (c).

Questions 23 to 26 are based on the following passage


(a) 2gR (b) 3gR
Passage VII
As m (c) 5gR (d) 7gR
24.
R P Q is
at a height H R
(a) mg (b) 2 mg
(c) 3 mg (d) 5 mg
25.
Q is
(a) . mg (b) 5 mg
(c) 26 mg (d) 3 3 mg
26. H so that the
A
R (b) 2.5 R
Fig. 8.95
(c) 2.6 R (d) 2.7 R
23. Q on
the track is
Rigid Body Rotation 8.59

SOLUTION
23. Q = Gain in K.E.
mv2 m
1 1 Fh = = (5 gR) = 5 mg, which is choice (d).
mg(H – R) = mv2 + I 2
R R
2 2 25.
2
1 2 1 2 v Fv mg
mg R – R) = mv + mR 2
2 2 5 R F= Fh2
= Fv2 26 mg
1 2 1 2 7 Thus the correct choice is (c).
= mv + mv = mv2 26. A
2 5 10
A
v= 5gR . Hence the correct choice m v 2A
is (c). = mg vA = Rg
R
24. Q H
Q 7 7
mg(Hmin – 2R) = m v 2A = mRg
the centre O 10 10
H = 2 R + 0.7R = 2.7R, which is choice (d).

Questions 27 to 30 are based on the following passage (c) Mg sin


Passage VIII 29.
(a) MgR (b) MgR sin
M and radius R
with the horizontal. 2 MgR sin MgR sin
(c) (d)
27. 3 3
30.
2 g sin
(a) g sin (b) M and the same radius R
3
g sin 2 g cos 3
(c) (d)
3 3 (a) (b) 2
2
28. (c) (d) 1
2
Mg sin 2 Mg sin
(a) (b)
3 3
SOLUTION
Mg sin – f = Ma (1)
is the angular acceleration and I
f f
a is the linear acceleration acting on it is
=I
is (see Fig. 8.96)
1 a
Now I= MR2, = and = f R. Hence
2 R
1 a 1
fR= M R2 = M Ra
2 R 2
1
or f= Ma (2)
2
2 g sin
27. a= , which
is choice (b). 3
Fig. 8.96
8.60 Comprehensive Physics—JEE Advanced

1 Mg sin MgR sin


28. f = Ma = , which is choice (a). 29. =fR= , which is choice (d).
2 3 3
30. For a ring, I = M R2. The correct choice is (a).

Questions 31 to 33 are based on the following passage mg 2mg


Passage IX (c) (d)
( M m) ( M 2m)
M and radius R is mounted on
32. The tension in the string is
Mmg
(a) mg (b)
( M m)
2 Mmg Mmg
m is hung (c) (d)
( M 2m) ( M 2m)
Fig. 8.97 33.
rest. to hn, where h is the height through which the mass
31. n is
(a) zero (b) 1
mg
(a) g (b) 1
M (c) (d) 2
2
SOLUTION
a mM g
32. T= , which
( M 2m)
ma = mg – T (1)
is choice (d).
where T is the tension in the string. 33.

= TR = I 1 2 1 2
mgh = mv + I
I 2 2
T=
R 1 1 1
= m R2 2
+ MR 2 2
1 a 2 2 2
= MR 2
2 R2 1
= (2m + M) R2 2
1
= Ma (2)
2 1/ 2
Fig. 8.98 mgh
31. =
( M 2m) R 2
2 mg
a= , which is choice (d). Thus h1/2. Hence the correct choice is (c).
( M 2m)

Questions 34 to 36 are based on the following passage x2. Both the discs
Passage X rotate in the clockwise direction.
IIT, 2007
Two discs A and B
34. The ratio x1/x2 is
I and 2I
1
A (a) 2 (b)
2
x1. Disc B 1
(c) 2 (d)
2
Rigid Body Rotation 8.61

35. When disc B is brought in contact with disc A, they 36.


t. The is
2 2
I I
(a) (b)
2 3
2I 9I
(a) (b) I 2
I 2
3t 2t (c) (d)
9I 3I 6
(c) (d)
t 2t
SOLUTION
34. A is A
1 1 2I
kx12 = I (2 )2 (1) = I (2 ) – I =
2 2 3 3
For disc B, the energy is A is
1 2 1
kx 2 = (2I)2 (2)
2 2
changein angular momentum 2I
x12 x1 = =
From (1) and (2) we get =2 = = 2. Time 3t
x22 x2
36.
35. When disc B is brought in contact with disc A, let
1 1
I the E1 = I(2 )2 + (2I) 2
= 3I 2
2 2

2
1 1 8
I = I (2 ) + (2I) I E2 = I ( )2 = (3I) = I 2
2 2 3 3
I = I + 2I = 3I E 1– E 2
8 1
3I I = = 3I 2 – I 2= I 2
3 3 3

Questions 37 to 39 are based on the following passage


Passage XI
M and radius R

constant k

L Fig. 8.99
L IIT, 2008
V0 V0 i . 38.
μ.
37. to
x k 2k
(a) (b)
M M
(a) –kx (b) – 2kx
2kx kx 2k k
(c) (d) (c) (d)
3 3 3M 3M
8.62 Comprehensive Physics—JEE Advanced

39. V0 3M 5M
(c) μg (d) μg
k 2k
M M
(a) μg (b) μg
k 2k

SOLUTION
37. Let kx
translational motion (see Fig. 8.100) 38. Acceleration a = =– 2
x. Hence the motion
3M
Ma = f – 2kx (1)
Let k
= .
3M
39. When V0 x x ) and
f
= μMg.

Ma = μMg – 2kx (3)


1
and – μMgR = MRa
Fig. 8.100 2
=I Eliminating a
1 a 3
or – fR = MR 2 ( a= R) (2) kx = Mg (5)
2 R 2
3
Eliminating f MV2 = kx2
kx
a =–
3M 3M
kx V = μg
F = Ma = . k
3

IV

Assertion-Reason Type Questions

Mg h/3.
Statement-2
correct.
- throughout its motion.
2. Statement-1
Two bodies A and B m
and m
-
A is v B is 2v,
v/3.
Statement-2
1. Statement-1
M and radius R rolls down
h. The rotational kinetic
Rigid Body Rotation 8.63

3. Statement-1
B
- is 3g / L .
circle. Statement-2
Statement-2

8. Statement-1
4. Statement-1
M and m (with M > m)

- anticlockwise along the circle.


V Statement-2

MV 9. Statement-1
v =
( M m)
Statement-2
Statement-2

5. Statement-1 angular momentum.


L -
10. Statement-1
M and closed at both
the ends. The tube is then rotated in a horizontal
mass M and the same radius R

2
other end is ML .
Statement-2
Statement-2
-
11. Statement-1
6. Statement-1
-
L and mass m is bent into a
-
r as shown in Fig. 8.101.
X X is
3mL2/8 2.
Statement-2

12. Statement-1

Fig. 8.101
same time.
Statement-2
Statement-2
-
XX
2
about YY + mr .
13. Statement-1
7. Statement-1
AB M and length L is
hinged at one end A -
8.64 Comprehensive Physics—JEE Advanced

Statement-2

14. Statement-1
suddenly

. Fig. 8.102
Statement-2
Statement-2

17. Statement-1
15. Statement-1
- zero.
Statement-2
Statement-2
18. Statement-1
-
16. Statement-1 tact is non zero.
Statement-2

SOLUTIONS
1. The correct choice is (a). 4. -

Potential energy = Translational kinetic energy MV = (M + m)vCM


+ Rotational kinetic energy MV
1 1 vCM =
or Mgh = Mv2 + I 2 ( M m)
2 2 5.
1 1 1
or Mgh = MR2 2 + M R2 2
2 2 2
3 L
= MR2 2 r=
2
2 gh
or =
3R 2 1 is the M
Now the rotational kinetic energy = I 2. L
2 circle or radius r = .
2
and I 2
Thus
1 1 gh
Rotational kinetic energy = MR 2 M v2 M (r )2 ML 2
2 2 3R 2 Fc = = = Mr 2
=
r r 2
M gh
= 6.
3
2. O is
1
IO = mr2
initially at rest) will remain at rest. 2
3. -
ertia about X X is
1 2 3
I = IO + mr2 =
mr + mr2 = mr2
2 2
Now L = 2 r or r = L/2
Rigid Body Rotation 8.65

3m L 2
3mL2 g sin 5 g sin 2
I= = a= I = I MR 2
2 2 8 2 1 7 5
7. The correct choice is (c). Loss in P.E. = gain in
MR 2

MgL
through a distance L/2, the loss in P.E. = . radius will reach at the same time.
2 2 13.
1 2 1 ML 2
Gain in K.E. = I =
2 2 3
a = g(sin – cos )

MgL ML2 2 3g g, and .


= = 14. I -
2 6 L
8. The correct choice is (d). I L = I is constant,
9. .
15. The correct choice is (a).
angular momentum will remain unchanged.
1 16.
10. The correct choice is (a). For cylinder, Ic = MR2 A -
2
2 2
Is = MR . Also = I or = .
5 I at A
, I.
Hence 17.
s
Ic
=
c
Is -
Ic > Is ; s > c R
both, as > ac > Hence no work is done.
s c.
11. 18. -
-
ity is changing with time. Hence the instantaneous
12. The correct choice is (a). The linear acceleration is

Integer Answer Type


1. - 2. L and mass 300 g rests on
F is

- L

F
g = 10 ms–2
8.103. Find the distance
3.

Fig. 8.103

IIT, 1980
8.66 Comprehensive Physics—JEE Advanced

Fig. 8.106
IIT, 1997
Fig. 8.104
7.
g = 10 ms–2 0.5 m with a stick as shown in Fig. 8.107. The stick

B 2
.
A
IIT, 1987 ring is large enough that rolling always occurs and the
4. m
(P P. Take g = 10 ms–2.

the circle is r, the tension in the string is T = Ar–n,


where A n.
IIT, 1993
5. A block X m = 0.5 kg is held by a string on
= 30°.

drum Y M = 2 kg and radius R = 0.2 m as


Fig. 8.107
X IIT, 2011
–2
g = 10ms 8. 5 cm and
newton) in the string during the motion. -

N 10 kg m2, then N is (see Fig. 8.108)

Fig. 8.105

6. M and radius R
u = 6 ms–1 on a rough

sliding motion at t t0
–1
v )
t 0.
Fig. 8.108

IIT, 2011
SOLUTION
1. O O2
O1 W1
Rigid Body Rotation 8.67

W2 3L L
Fmin = mg
2
2mg 2 0.3 10
Fmin = = = 2N.
3 3

Fig. 8.110

3.
A = gain in total
kinetic energy (translational and rotation), i.e.
1 1
Mg(H – h) = Mv2 + I 2
2 2
2 v
Fig. 8.109 Where I = MR2 and = . Thus
5 R
p (21)2 cm2 2
1 1 2 v
cm2 Mg(H – h) = Mv2 + MR2
2 2 5 R
(28)2 cm2
[(28)2 – (21)2 1 1 7
= Mv2 + Mv2 = Mv2
cm2 2 5 10
12
area. Hence 10 H h g
or v=
7
W1 mg m
= 1 = 1 = = 9 12
W2 m2 g m2 7 10 . 1.0 10
=
Taking moments about O 7
W1 OO1 = W2 OO2 = 2 5 ms –1

W1
OO2 = OO1
W2
9 2h 2 1.0 1
= 7 cm = 9 cm t= = = s
7 g 10 5
2. F about A is (Fig. 8.110) Horizontal range = vt = 2 5 = 2.0 m.
3L 4. Angular momentum is mvr = constant (say k).
1 = F
k
v= .
mg mr
mv 2 mk 2
A is Also T= = 2 3 = m–1k2r–3 = Ar–3
r m r
L –1 2
2 = mg where A = m k
2
T = Ar–n, we get n = 3
5. X is as shown in Fig.
X, then
( 1)min = 2 ma = T – mg sin (1)
8.68 Comprehensive Physics—JEE Advanced

f = μmg
a =R P is
= RT = I F R – fR = I
1 Where I = mR2 + mR2 = 2mR2
MR 2
I 2 1 a
T= = = MR
R R 2 R
and a =R
1
T = Ma (2) Also f = μmg
2
Using (2) in (1) a
F R – μmgR = (2 mR2) = 2maR
R
1 F – μmg = 2 ma
ma = Ma – mg sin 30°
2 2 – μ 2 10 = 2 2 0.3
mg 0.8 .
a= (3) μ= = . Hence P
M 2m 2 10 10
M mg 8. Let M R its radius.
T=
2 M 2m
2 0.5 10 diagonal AB
= =5N
2 2 2 0.5 I = I1 + I2 + I3 + I
where I1, I2, I3 and I
AB.
2
I1 = I2 = MR2
5

2
I3 = I = MR2 + Mr2
5
Fig. 8.111 2
2 a
6. = MR2 + M
5 2
2 2 Ma 2
I =2 MR 2 2 MR 2
1 5 5 2
MuR = MvR + I 2
2
= 8 MR 2 + Ma2
1 v
= MvR + (MR2) 5
2 R 2
v 8 5 2
u =v + = 0.5 10 + 0.5
2 5 2
2u 2 6 –1
10–2)2
v= = =9 10 kgm2. Hence N = 9.
3 3
7.

Fig. 8.112
9
Chapter
Gravitation

REVIEW OF BASIC CONCEPTS to all other masses is obtained from the principle of
superposition which states that ‘the gravitational force
9.1 NEWTON’S LAW OF GRAVITATION experienced by one mass is equal to the vector sum of the
gravitational forces exerted on it by all other masses taken
Newton’s law of universal gravitation states as follows:
one at a time.’
‘Any two particles of matter anywhere in the universe
attract each other with a force which is directly proportional
to the product of their masses and inversely proportional
to the square of the distance between them, the direction
of the force being along the line joining the particles, i.e.
(Fig. 9.1)

Fig. 9.1

m1 m2 Fig. 9.2
F
r2
The gravitational force on mass m due to masses m1, m 2,
where F is the magnitude of the force of attraction m3, ... mn is given by (Fig. 9.2)
between two particles of masses m1 and m2 separated by F = F1 + F2 + F3 + + Fn
a distance r.
In the form of an equation the law is written as NOTE
Gm1 m2 (1) Gravitational force is always attractive.
F= 2 (2) Gravitational force between two masses does not de-
r
pend the medium between them.
where G is a constant called the universal gravitation (3) Gravitational force acts along the straight line joining
constant. The value of this constant is to be determined the centres of the two bodies.
experimentally and is found to be
G = 6.67 10–11 N m2 kg–2. 9.1
Two bodies A and B of masses m 1 = 1 kg and m 2 =
GRAVITATIONAL FORCE DUE TO MULTIPLE 16 kg respectively are placed 1.0 m apart. A third
9.2 MASSES body C of mass m = 3 kg is placed on the line joining
A and B. At what distance from A should C be placed
If a system consists of more than two masses, the so that it experiences no gravitational force?
gravitational force experienced by a given mass due
9.2 Comprehensive Physics—JEE Advanced

The magnitude of the resultant force is


SOLUTION
Let x metre be the distance between A and C (Fig. Fr = F12 F22 2 F1 F2 cos
9.3)
= F2 F2 2 F 2 cos 60

Gm 2
= 3F = 3 , directed vertically
a2 downwards.
Fig. 9.3

Force exerted by A on C is 9.3 ACCELERATION DUE TO GRAVITY


G m 1m
F1 = directed towards A Considering the earth as an isolated mass, a force is
x2
experienced by a body near it. This force is directed
Force exerted by B on C is towards the centre of the earth and has a magnitude mg,
G m2 m where g is the acceleration due to gravity.
F2 = directed towards B
(1 x) 2 GmM
F = mg =
C will experience no force if F1 = F2, i.e. R2
G m1m G m2 m where M is the mass of the earth and R its radius (nearly
2
= constant for a body in the vicinity of the earth)
x (1 x) 2
GM
m2 (1 x) 2 g=
= R2
m1 x2 All bodies near the surface of the earth fall with the
(1 x) 2 same acceleration which is directed towards the centre of
16 = the earth.
x2
1 x 9.4 VARIATION OF G
4=
x
1. Variation with altitude The acceleration due to gravity
which gives x = 0.2 m. Note that if body C is placed of a body at a height h above the surface of the earth is
to the left of body A or to the right of body B, it will given by
2
R
gh = g
9.2 R h
Three bodies, each of where g is the acceleration due to gravity on the surface
mass m, are placed at of the earth. If h is very small compared to R, we can use
the vertices of an equi- binomial expansion and retain terms of order h/R. We
lateral triangle of side then get
a as shown in Fig. 9.4. 2h
gh = g 1
Find the magnitude and R
direction of the force ex- Thus, the acceleration due to gravity decreases as the
perienced by the body at altitude (h) is increased.
vertex A. Fig. 9.4
2. Variation with depth The acceleration due to gravity
SOLUTION at a depth d below the surface of the earth is given by
The forces exerted on the d
gd = g 1
body at A by bodies at B R
and C are shown in Fig. This equation shows that the acceleration due to gravity
9.5. decreases with depth. At the centre of the earth where
F1 = F2 d = R, gd = 0. Thus the acceleration due to gravity is
maximum at the surface of the earth, decreases with
Gm 2 increase in depth and becomes zero at the centre of the
= = F (say)
a2 Fig. 9.5 earth.
Gravitation 9.3

3. Variation with Latitude Due to the rotation of earth


SOLUTION
about its axis, the value of g varies with latitude, i.e. from
2
one place to another on the earth’s surface. At poles, the g R
=
effect of rotation on g is negligible. At the equator, the g R h
effects of rotation on g is the maximum. In general, the 2
value of acceleration due to gravity at a place decreases 1 R
=
with the decrease in the latitude of the place. 2 R h
The acceleration due to gravity at a place on earth
1 R
where the latitude is is given by =
g = g – 2 R cos2 ; 2 R h

= angular velocity of rotation of earth. which gives h= 2 1 R.


At equator, =0 g e = g – R 2 (minimum)
°
At poles, = 90 gp = g (maximum) 9.5
g = g – 0.0337 cos 2 A body weighs 63 N on the surface of the earth. How
Thus the value of g varies slightly from place to place much will it weigh at a height equal to half the radius
on earth. Variation of g with altitude and depth is shown of the earth?
in Fig. 9.6.
SOLUTION
W = mg = 63 N
2
R
W = mg = mg
R h
2
R
W =W
R h
2
R
= 63 = 28 N
R R/2
Fig. 9.6 Variation of g (gravitational acceleration)
9.6
9.3 Assuming the earth to be a sphere of uniform mass
The acceleration due to gravity on the surface of the
moon is 1.67 ms–2. The mass of the earth is about of a body when it is taken to the end of a tunnel
80 times that of the moon. Estimate the ratio of the 32 km below the surface of the earth. Radius of earth
radius of the earth to that of the moon. = 6400 km.

SOLUTION SOLUTION
G Me G Mm d 32 199 g
ge = and gm = g =g 1 =g 1 =
Re2 Rm2 R 6400 200
1/ 2 Decrease in weight = mg – mg
Re Me gm
= 199 mg
Rm Mm gm = mg 1 =
200 200
1/ 2
1.67 mg / 200
= 80 3.7 Percentage decrease = 100 = 0.5%
9.8 mg

9.4 9.7
Assuming that the earth is a sphere of radius R, at At what depth below the surface of the earth will the
what altitude will the value of acceleration due to value of acceleration due to gravity become 90% of
gravity be half its value at the surface of the earth? its value at the surface? R = 6.4 106 m.
9.4 Comprehensive Physics—JEE Advanced

SOLUTION
intensity is given by
d
g = 0.9 g. g =g 1 I= dI
R
d Gravitational Field due to some continuous Mass
0.9 = 1 –
R Distributions
d = 0.1 R = 6.4 105 m
M and radius R at a point at a distance r from the
9.5 GRAVITATIONAL FIELD INTENSITY centre and on the axis of the ring is given by
GM r
I= 2
(R r 2 )3 / 2

of mass M and radius R at a point P at a distance


r > R from the centre of shell,
experienced by a unit mass placed at that point. GM
M. I = 2 (outside the shell)
r
Inside the shell, I = 0
a point P at a distance r from M,
M and
we place a small mass m at P. The
radius R is
gravitational force exerted on m
by M is (Fig. 9.7) Fig. 9.7 GM
I = 2 for r > R
GM m r
F= GM
r2 I= for r = R
R2
M at
P is given by GMr
I= for r < R
F GM R3
I= = 2
m r
I is a vector quantity. In vector form
GM
I= – r
r2
where r is a unit vector directed from M to P, i.e radially
away from M. The negative sign indicates that I directed
radially inwards towards M. Fig. 9.8
The SI unit of I is N kg–1.
In three dimensions, if mass M is located at the origin,
P (x, y, z) is given by of a solid sphere.
GM 9.6 GRAVITATIONAL POTENTIAL ENERGY
I= – r
r2
Gravitational potential energy of a system of two masses
where r = x i + y j z k represents the position of point P M and m held a distance r
with respect to mass M at the origin.
For a many body system, the principle of superposition respective locations along any path and without any
acceleration. (see Fig. 9.9)
gravitational forces, i.e.
I = I1 + I2 + I3 + + In
where I1, I2, ... In
point due to bodies of masses M1, M2, ... Mn.
For continuous mass distributions (i.e rigid bodies),
Fig. 9.9
Gravitation 9.5

Work done to bring mass M A is W1 = 0. GMm GMm


Work done to bring mass m from r = to r = r is =– –
( R h) R
r
W2 = F dr =
GmM h
R ( R h)
r
GmM h
= F dr cos If h < < R ; U= = mgh
R2
Since mass M will attract mass m, angle between F GM Fig. 9.11
and dr is zero. Hence 2
g
R
r
GM m
W2 = dr
r2 9.7 GRAVITATIONAL POTENTIAL
Gravitational potential at a point P in
r
GMm
= GMm r 2dr = – M
r
GMm Fig. 9.12
Total work W = W1 + W2 = –
r that point along any path and without
Gravitational potential energy of the system is any acceleration, i.e., (Fig. 9.12)
GMm W GM m
U= W = – V= =–
r m r m
The zero of potential energy is assumed to be at r = . GM
The negative sign indicates the potential energy is negative V= –
r
Potential V is a scalar quantity. Hence the gravitational
potential at a point P due to a number of masses m 1, m2, ... mn
Expression for Increase in Gravitational Potential at distances r 1, r 2, ... rn respectively from P is given by
Energy V = V1 + V2 + + Vn
If the body m is moved away from M, the potential energy
m1 m2 mn
of the system increases. [see Fig. 9.10] =–G
r1 r2 rn
The SI unit of V is J kg–1.
Relation between Gravitational Field Intensity (I)
Fig. 9.10 and Gravitational Potential (V)

GMm
P.E. at B = – a point are related as
r1
dV
GMm I=
P.E. at C = – dr
r2 Gravitational Potential due to a Spherical Shell
GM m GM m
Increase in P.E. = – – (i) At a point outside the shell, V =
GM
(for
r2 r1 r
1 1 r > R) where M is the mass and R is the radius of
= GMm the shell.
r1 r2 GM
(ii) At a point on the surface of the shell, V =
If the body of mass M is the earth, then the increase in R
gravitational P.E. when a body of mass m is taken from GM
(iii) At a point inside the shell, V = (for r < R)
the surface of the earth to a height h above the surface is R
given by (see Fig. 9.11), R = radius of the earth. Figure 9.13 shows the variation V with r for a
U = P.E. at Q – P.E at P spherical shell.
9.6 Comprehensive Physics—JEE Advanced

P will be zero if
Gm1 Gm2
I1 = I2
r 12 r 22
2 2
m2 r1 r1
or = 81 =
m1 r2 1 r1
( r2 = r – r1 and r = 1 m)

Fig. 9.13 r1
9= r1 = 0.1 m
1 r1
Gravitational Potential due to a Solid sphere of mass M
and radius R Gravitational potential at P is
(i) For points outside the sphere (r > R),
m1 m2
V = V1 + V2 = – G
GM r1 r2
V=
r
11 100 8100
(ii) For points inside the sphere (r < R), = 6.67 10
0.1 0.9
3G M R 2 r2 = – 6.67 10–7 J kg–1
V=
R3 2 6
(b) Gravitational potential energy of the system is
(ii) At the centre of the sphere (r = 0) G m1 m2
3G M G.P.E. =
V= r
2R
11
(iv) On the surface of the sphere (r = R) 6.67 10 100 8100
=
GM 1
V=
R = – 5.4 10–5 J

9.8 9.9
Two masses m1 = 100 kg and m2 = 8100 kg are held Three equal masses, each equal to m, are kept at the
1 m apart. vertices of an equilateral triangle of side a. Find the
(a) At what point on the line joining them is the
- centroid of the triangle.
tational potential at that point.
SOLUTION
(b) Find the gravitational potential energy of the
system. Given G = 6.67 10–11 Nm2 kg–2. Refer to Fig. 9.15.

SOLUTION
P due to m1 is [Fig. 9.14]

Fig. 9.14

Gm1
I1 = directed towards m1
r 12
P due to m2 is
Gm2
I2 = directed towards m2 Fig. 9.15
r 22
Gravitation 9.7

The centroid O divides the lines AD, BE and CF in


9.11
the ratio 2:1. Also AO = BO = CO = r (say). Now AO
2 Two particles of masses m1 and m2 are initially at rest
= AD and -
3
wards each other due to gravitational attraction. Find
a2 3a the ratio of their accelerations and speeds when the
AD = a2 =
4 2 separation between them becomes r.
23a a a
AO = = , i.e. r = SOLUTION
3 2 3 3 Since no external force acts on the system, the accel-
O due to masses m at A, B eration of their centre of mass is zero, i.e.
Gm
and C are IA = IB = IC = 2 = I. Their directions are m1 a1 m2 a2
r aCM =
shown in Fig. 9.15. The angle between any two of m1 m2
them is = 120°.
m1a1 m2 a2
The resultant of IB and IC is 0=
m1 m2
I = I2 I2 2 I 2 cos 120 m 1a 1 = – m 2 a 2
= I directed vertically down. a1 m2
=
I will cancel with IA a2 m1
at O is zero. The negative sign indicates that they move in oppo-
The gravitational potential at O is site directions.
V = V1 + V2 + V3 Let v1 and v2 be the speeds of two masses when they
Gm Gm Gm are at a distance r. Due to gravitational attraction,
= they gain speed as they approach each other. Hence
r r r
their kinetic energy increases and gravitational poten-
3Gm Gm a tial energy (G.P.E) decreases. From the conservation
= = 3 3 r
r a 3 of energy,
Loss in G.P.E. = gain in K.E.
9.10 G.P.E.)i – (G.P.E.)f = (K.E.)f – (K.E.)i
The gravitational potential at a height h = 1600 km
above the surface of the earth is – 4.0 107 J kg–1. Gm1m2 1 1
0 = m1 v 12 m2 v 22 0
Assuming the earth to be a sphere of radius R = 6400 r 2 2
G m1 m2 1 1
at that height. = m1v12 m2 v 22 (i)
r 2 2
SOLUTION Since no external force acts, the total momentum of
Let r = R + h. Then the system is conserved, i.e., pi = pf or
GM 0 = m 1v 1 – m 2v 2 (ii)
V=
r From Eqs. (i) and (ii), we get
GM 1/ 2
and g = 2G m22
r 2 v1 =
r m1 m2
g GM / r 2 1 1
= = = 1/ 2
V GM / r r R h 2G m 12
and v2 =
7 1
r m1 m2
V 4.0 10 J kg
g = =
R h 6.4 1.6 106 m v1 m
= 2
v2 m1
= 5.0 J kg–1 m–1 = 5.0 ms–2
9.8 Comprehensive Physics—JEE Advanced

9.8 ESCAPE VELOCITY (R + h) by R. The error involved in this approximation is


negligible since the radius of the earth (R) = 6.4 106 m
The escape velocity is the minimum velocity with which = 6400 km.
a body must be projected in order that it may escape the Thus, we may write
earth’s gravitational pull. The magnitude of the escape
velocity is given by v= gR 9.8 6.4 106
2MG = 7.9 103 ms–1 8 km s–1
ve =
R Periodic Time The periodic time T of a satellite is
the time it takes to complete one revolution and it is
where M is the mass of the earth and R its radius. given by (since r = R + h)
Substituting the known values of G, M and R, we get
ve = 11.2 kms–1. The expression for the escape velocity 2 r r3 ( R h )3
can be written in terms of g as T= 2 2
v GM gR 2
ve = 2gR
If h << R, we have v = g R . Hence
The escape velocity is independent of the mass of the body
and the direction of projection. 3
R h R
T= 2 2
2
9.9 SATELLITES gR g

A body moving in an orbit around a much larger and Total Energy of a Satellite
massive body is called a satellite. The moon is the natural Total energyE = K.E. + P.E.
satellite of the earth.
1 Gm M
Orbital Velocity Let us assume that a satellite of mass m = m v2
2 r
goes around the earth in a circular orbit of radius r with a
uniform speed v. If the height of the satellite above the =
Gm M Gm M
v=
GM
earth’s surface is h, then r = (R + h), where R is the mean 2r r r
mv2
radius of the earth. The centripetal force necessary GmM Gm M
r E= =
to keep the satellite in its circular orbit is provided by 2r 2 R h
GmM NOTE
the gravitational force between the earth and the
r2 (1) E =
P.E.
satellite. This means that 2
(2) E = – (K.E.)
mv2 mM
=G 2 (3) The total energy is negative which implies that the sat-
r r
where M is the mass of the earth. Thus Gm M
The binding energy = . This energy must be
GM GM 2r
v=
r ( R h)
Now the acceleration due to gravity on earth’s surface Angular Momentum
is given by
GM The magnitude of angular momentum of a satellite is
g= given by
R2 L = mvr
or GM = gR2 GM
= m r
Substituting for GM we get r
g L = m GMr
v= R
R h
Geostationary Satellites A Geostationary satellite is
If the satellite is a few hundred kilometres above a particular type used in communication. A number of
the earth’s surface (say 100 to 300 km), we can replace communication satellites are launched which remain in
Gravitation 9.9

They are called geostationary or synchronous satellites


used in international communication.

certain place on the earth, it must corotate with the earth


so that its orbital period around the earth is exactly equal
to the rotational period of the earth about its axis of
rotation.

at a height of about 36,000 km above the earth’s surface.


Trajectory of a Satellite for Different Speeds
Orbital speed of a satellite is
GM
v0 =
r
Escape velocity is
Fig. 9.16
2GM
ve = According to Kepler’s second law, if the time
r
interval between P1 and P2 equals the time interval
Let v be the speed given to a satellite.
between P3 and P4, then area A1 must be equal to
(i) If v < v0, the satellite will follow an elliptical path
area A2. Also the planet has the greater speed in its
with the centre of the earth as the farthest focus.
path from P1 to P2 than in its path from P3 to P4.
The satellite will not complete the orbit and will
3. Law of periods The squares of the periods of the
fall to the earth.
planets are proportional to the cubes of their mean
(ii) If v = v0, the satellite will follow a circular path
distances from the sun. If T1 represents the period
with the centre of the earth as the centre of the cir-
of a planet about the sun, and r1 its mean distance,
cular orbit.
then
(iii) If v0 < v < ve, the satellite will follow an ellipti-
cal orbit with the centre of the earth as the nearer T12 r13
focus. If T2 represents the period of a second planet about the
(iv) If v = ve, the satellite will escape the gravitational sun, and r2 its mean distance, then for this planet
pull of the earth following a parabolic path. T 22 r32
(v) If v > ve, the satellite will escape the gravitational These two relations can be combined since the factor
pull of the earth following a hyperbolic path. of proportionality is the same for both. Thus

9.10 KEPLER’S LAWS OF PLANETARY MOTION T12 r13


=
T22 r23
Johannes Kepler formulated three laws which describe
planetary motion. They are as follows:
9.12
1. Law of orbits Each planet revolves about the sun
The mass of Jupiter is 318 times that of the earth and
in an elliptical orbit with the sun at one of the focii
its radius is 11.2 times that of the earth. Calculate
of the ellipse. The orbit of a planet is shown in Fig.
the escape velocity from Jupiter’s surface. Given the
9.16(a) in which the two focii F1 and F2, are far
escape velocity from earth’surface = 11.2 km s–1.
apart. For the planet earth, F1 and F2 are very close
together. In fact, the orbit of the earth is practically SOLUTION
circular.
2M J G
2. Law of areas A line drawn from the sun to the For Jupiter : vJ =
planet (termed the radius) sweeps out equal areas RJ
in equal intervals of time. In Fig. 9.16(b) P1, P2, P3 2M E G
and P4 represent positions of a planet at different For Earth : vJ =
RE
times in its orbit and S, the position of the sun.
9.10 Comprehensive Physics—JEE Advanced

vJ MJ At the highest point, v = 0. If h is the maximum height


RE
= attained, the energy of the rocket at height h is
vE ME RJ
GmM
Ef =
1 ( R h)
= 318 5.33
11.2 From conservation of energy, Ei = Ef , i.e.,
vJ = 5.33 vE = 5.33 11.2 = 59.7 km s–1
1 2 GmM GmM
mv =
2 R ( R h)
9.13
Calculate the escape velocity of a body at a height 2GM h h
v= = ve
1600 km above the surface of the earth. Radius of R R h R h
earth = 6400 km.
ve v R R
Given v = . Hence e = ve h=
SOLUTION 2 2 R h 3
Work required to move a body of mass m from r =
R + h to r = is 9.15
R
W= Fdr A body is dropped from a height h equal to , where
2
R h R is the radius of the earth. Show that it will hit the
dr Gm M surface of the earth with a speed v = ve / 3 , where ve
= Gm M 2
= is the escape velocity from the surface of the earth.
R h r R h
If ve is the escape velocity, then SOLUTION
1 Gm M Total energy of the body at height h is
mve2 =
2 R h Gm M
Ei = (i)
R h
2 GM 2 gR 2
ve = Total energy when it hits the surface of the earth is
R h R h
GM 1 Gm M
g m v2
Ef = (ii)
R 2 2 R
Given R = 6.4 106 m, and h = 1.6 6
10 m and From conservation of energy, Ei = Ef , i.e.
g = 9.8 ms–2 Gm M 1 GmM
6 2
= m v2
2 9.8 (6.4 10 ) R h 2 R
ve =
(6.4 1.6) 106 2GM h
–1
v=
= 10 10 ms R R h
= 10 km s–1
R
= ve
9.14 R h
A rocket is launched vertically from the surface of the For h =
R v
,v= e .
earth with an initial velocity equal to half the escape 2 3
velocity. Find the maximum height attained by it in
terms of R where R is the radius of the earth. Ignore
9.16
atmospheric resistance.
Show that the minimum energy required to launch
SOLUTION a satellite of mass m from the surface of the earth in
a circular orbit at an altitude h = R, where R is the
On the surface of the earth, the total energy of the
3mgR
rocket is radius of the earth is where M is the mass of
1 2 GmM 4
Ei = K.E. + P.E. = mv the earth.
2 R
Gravitation 9.11

SOLUTION NOTE
Total energy of the satellite orbiting the earth is Since Fg = Fc, the satellite is a freely falling body and is,
Gm M therefore, weightless.
E1 =
2r
9.18
Gm M GmM
= ( h = R) A body projected vertically upwards from the surface
2( R h) 4R of the earth with a certain velocity rises to a height of
Total energy when the satellite was at rest on the sur- 10 m. How high will it rise if it is projected with the
face of the earth is same velocity vertically upwards from a planet whose
density is one-third that of the earth and radius half
E2 = K.E. + P.E.
that of earth? Ignore atmospheric resistance.
Gm M Gm M
= 0 = SOLUTION
R R
Minimum energy required is Since the kinetic energy of the body is the same in
Emin = E1 – E2 both the cases, loss in K.E. = gain in P.E. will be equal,
i.e.,
=
Gm M GmR mgphp = mgehe
4R R
ge
3Gm M 3 hp = he
= = mg R gp
4R 4
GM G 4 4
9.17 Now g= 2
= 2
R3 = GR
R R 3 3
A satellite of mass m = 100 kg is in a circular orbit at
a height h = R above the surface of the earth where R ge R
= e e
=2 3=6
is the radius of the earth. Find gp Rp p

(a) the acceleration due to gravity at any point on hp = 6he = 6 10 = 60 m


the path of the satellite,
(b) the gravitational force on the satellite and 9.19
(c) the centripetal force on the satellite. A satellite of mass 2000 kg is orbiting the earth at an
altitude R/2, where R is the radius of the earth. What
SOLUTION extra energy must be given to the satellite to increase
2 2
its altitude to R? Given R = 6.4 106 m.
R R 9.8
(a) g = g = 9.8 =
R h R R 4 SOLUTION
= 2.45 ms–2 R 3R
r1 = R = and in the second
(b) Gravitational force on satellite is 2 2
Fg = mg = 100 2.45 = 245 N case, r2 = R + R = 2R.
Required energy = E2 – E1
m v2 Gm M
(c) Centripetal force Fc = = Gm M Gm M
r r2 =
2 r2 2 r1
GM
g=
R2 =
Gm M GmM
Gm M R2 4R 3R
or Fc = 2 GmM
R r2 =
2
12 R
R
= mg = mg = 245 N. mgR GM
R h = g=
12 R2
9.12 Comprehensive Physics—JEE Advanced

2000 9.8 6.4 106 SOLUTION


=
12 From Kepler’s law of periods, T2 r3. Therefore
8
= 1.04 10 J Tp2 rp3
= = (10)3 = 1000
Te2 r 3e
9.20
Two bodies of masses m1 and m2 are held at a distance Tp = Te 1000 = 1 year 31.6 = 31.6 years
r apart. Show that at the point where the gravitational
9.22
is given by A satellite is revolving in a circular orbit close to the
G surface of the earth with a speed v. What minimum
V= m1 m2 2 m1m2
r additional speed must be imparted to it so that it es-
capes the gravitational pull of the earth? Radius of
SOLUTION earth = 6.4 106 m.
P (Fig.
9.17). Then SOLUTION

G m1 G m2 v= g R and ve = 2g R
=
r 12 r 22 Additional speed required is
ve – v = 2 1 gR
r1 m1
= Fig. 9.17
r2 m2 = 0.414 9.8 6.4 106
= 3.28 103 ms–1
r1 m1
= = 3.28 km s–1
r r1 m2

r m1 9.23
r1 = (i)
m1 m2 A body of mass m is placed at the centre of a spherical
shell of radius R and mass M. Find the gravitational
Also r2 = r – r1
potential on the surface of the shell.
r m1
= r SOLUTION
m1 m2
Gravitational potential on the surface of the shell due
r m2 to body of mass m is
= (ii) Gm
m1 m2 Vb =
R
Gravitational potential at P is Gravitational potential on the surface of the shell due
Gm1 Gm2 to shell itself is
V= (iii) GM
r1 r2 Vs =
R
Using (i) and (ii) in (iii) and simplifying, we get G
V = Vb + Vs = m M
G R
V= m1 m2 2 m1m2
r 9.24
A tunnel is drilled from the surface of the earth to its
9.21 centre. A body of mass m is dropped into the tunnel.
The distance of a planet from the sun is 10 times Find the speed with which the body hits the bottom
that of the earth. Find the period of revolution of the of the tunnel. The mass of earth is M and its radius
planet around the sun. is R.
Gravitation 9.13

Hence
SOLUTION
1 Gm M 3 GmM
Let v be the required speed. Gain in K.E. = loss in P.E. mv2 =
2 R 2 R
= P.E. at the surface – P.E. at the centre. The poten-
3 GmM v=
GM
=
GM
tial energy at the centre of the sphere = . gR g=
2 R R R2

Multiple Choice Questions with Only One Choice Correct


1. A body of mass m is placed at the centre of a spheri- of the earth and E2 is the energy required to put it
cal shell of radius R and mass M. The gravitational into a circular orbit at that height, then the ratio E1/
potential on the surface of the shell is E2 is
G G 1
(a) – (M + m) (b) – (M – m) (a) 1 (b)
R R 2
1 2
G mM G mM (c) (d)
(c) – (d) – 3 3
R M m R M m
6. If g is the acceleration due to gravity on the surface
2. The magnitude of angular momentum of the of the earth, the gain in potential energy of a satel-
earth revolving round the sun is proportional to lite of mass m raised from the earth’s surface to a
R n where R is the distance between the earth and height equal to the radius R of the earth is
the sun. The value of n is (assume the orbit to be (a) mgR/4 (b) mgR/2
circular). (c) mgR (d) 2 mgR
(a) 1 (b) 1 7. The escape velocity of a body projected vertically
2 upwards from the surface of the earth is v. If the
body is projected at an angle of 30° with the hori-
(c) 3 (d) 2 zontal, the escape velocity would be
2
(a) v/2 (b) 3 v/2
3. A tunnel is drilled from the surface of the earth to
(c) 2 v (d) v
its centre. The radius of the earth is R and g is the
acceleration due to gravity on its surface. A body of 8. The escape velocity of a body at a height h above
mass m is dropped into the tunnel. If M is the mass the surface of the earth is (g = acceleration due to
of the earth, the speed with which the body hits the gravity on the surface of the earth and R = radius of
bottom of the tunnel is the earth) is

m (a) 2 gR (b) 2 g R h
(a) gR (b) M gR
M m 2 gR 2 gR 2
(c) (d)
R h 2 R h
(c) 2 gR (d) gR
9. A small planet is revolving around a very massive
4. A satellite is orbiting at a height R above the star in a circular orbit of radius r with a period of
surface of the earth where R is the radius of the revolution T. If the gravitational force between the
earth. By what percentage must the energy of the planet and the star were proportional to r –5/2, then
satellite be increased so that it orbits at a height 2 R T would be proportional to
above the surface of the earth? (a) r 3/2 (b) r 5/3
7/4
(a) 25% (b) 33.3% (c) r (d) r 3
(c) 50% (d) 66.7% 10. If both the mass and the radius of the earth decrease
5. If E1 is the energy required to raise a satellite to a by 1%, the acceleration due to gravity on the sur-
height h = R (radius of the earth) above the surface face of the earth will
9.14 Comprehensive Physics—JEE Advanced

(a) decrease by 1% (b) increase by 1% 16. An extremely small and dense neutron star of mass
(c) increase by 2% (d) remain unchanged. M and radius R is rotating at an angular frequency
11. Two masses M1 and M2 are separated by a distance . If an object is placed at its equator, it will remain
r. A particle of mass m is placed exactly mid-way stuck to it due to gravity if
between them. The minimum speed with which R R2 2
(a) M > (b) M >
the particle should be projected so as to escape to G G
3 2
1/ 2
R R 32
G M1 M 2 (c) M > (d) M >
(a) v = 2 G G
r 17. What is the minimum energy required to launch a
1/ 2
2 G M1 M 2 satellite of mass m from the surface of the earth of
(b) v =
r radius R in a circular orbit at an altitude of 2R?
2 1/ 2
5GmM 2GmM
G M1 M 2 (a) (b)
(c) v = 2 6R 3R
mr GmM GmM
(c) (d)
2 1/ 2 2R 3R
2 G M1 M 2
(d) v = 18. Two stars, each of mass m and radius R are ap-
mr proaching each other for a head-on collision. They
12. The areal velocity of a planet of mass m moving start approaching each other when their separation
along an elliptical orbit around the sun is is r >> R. If their speeds at this separation are negli-
(a) proportional to m (b) proportional to m gible, the speed with which they collide would be
1 1 1
(c) proportional to (d) independent of m (a) v = Gm
m R r
13. A planet of mass m revolves around the sun in an
1 1
elliptical orbit of semimajor axis a. If M is the mass (b) v = Gm
of the sun, the speed of the planet when it is at a 2R r
distance x from the sun is 1 1
(c) v = Gm
1 1 1 1 R r
(a) GM (b) GM
x 2a 2x a 1 1
(d) v = Gm
2 1 GM a 2R r
(c) GM (d)
x a 2 x2 19. Consider a particle of mass m suspended vertically
by a string at the equator. Let R and M denote the
14. A satellite revolves around a planet with a speed radius and the mass of the earth respectively. If
v in a circular orbit of radius r. If R is the radius is the angular velocity of earth’s rotation about its
of the planet, the acceleration due to gravity on its own axis, the tension in the string is equal to
surface is
mM mM
v2 r v2 R (a) G (b) G
(a) g = 2 (b) g = 2 2R2 R2
R r
mM 2 mM 2
v2 v2 (c) G 2 –m R (d) G 2 +m R
(c) g = (d) g = R R
r R
20. m, are
15. Three spheres, each of mass M and radius R, are placed along a straight line at distances of r, 2r, 4r,
kept such that each touches the other two. The mag- 8r, etc. from a reference point O. The gravitational
nitude of the gravitational force on any one sphere O will be
due to the other two is
5 Gm 4 Gm
GM 2 3 GM 2 (a) (b)
(a) (b) 4r 2
3r 2
2R2 2 R2
3 Gm 2 Gm
3 GM 2 3 GM 2 (c) (d)
(c) (d) 2r 2
r2
2 R2 4 R2
Gravitation 9.15

21. In Q. 20, the magnitude of the gravitational poten- attraction. Their relative velocity of approach at a
tial at point O will be separation distance r between them is
Gm Gm 1/ 2
(a) (b) 2G m1 m2
2r r (a)
r
3 Gm 2 Gm
(c) (d) 1/ 2
2r r 2G m1 m2
22. A satellite in force-free space sweeps stationary (b) r 2
interplanetary dust at a rate dM/dt = v, where M is 1/ 2
the mass and v is the velocity of the satellite and r
(c)
is a constant. The acceleration of the satellite is 2G m1 m2
2 v v2 2G 1/ 2
(a) (b) (d) m1 m2
M M r
2
v
(c) (d) – v2 26. If the distance between the earth and the sun were
M half its present value, the number of days in a year
23. The time of revolution of a satellite is T. Its kinetic would have been
energy is proportional to (a) 64.5 (b) 129
1 1 (c) 182.5 (d) 730
(a) (b) 2
T T IIT, 1996
1 27.
(c) 3 (d) T –2/3
T around the earth has a total (kinetic + potential)
energy E0. Its potential energy is
24. A solid sphere of uniform density and radius 4 units
(a) – E0 (b) 1.5 E0
is located with its centre at origin O of coordinates.
(c) 2 E0 (d) E0
Two spheres of equal radii 1 unit, with their centres
at A (– 2, 0, 0) and B(2, 0, 0) respectively are taken IIT, 1997
out of the solid leaving behind spherical cavities as 28. A satellite S is moving in an elliptical orbit around
shown in Fig. 9.18. Choose the incorrect statement the earth. The mass of the satellite is very small
from the following. compared to the mass of the earth. Which of the
following statements is correct?
(a) The acceleration of S is always directed
towards the centre of the earth.
(b) The angular momentum of S about the cen-
tre of the earth changes in direction, but its
magnitude remains constant.
(c) The total mechanical energy of S remains
constant.
(d) The linear momentum of S remains constant
in magnitude.
IIT, 1998
Fig. 9.18 29. The distance between the sun and the earth is r and
(a) the gravitational force due to this object at the earth takes time T to make one complete revolu-
the origin is zero. tion around the sun. Assuming the orbit of the earth
(b) the gravitational force at point B (2, 0, 0) is zero. around the sun to be circular, the mass of the sun
(c) the gravitational potential is the same at all will be proportional to
points of the circle y2 + z2 = 36. r2 r2
(d) the gravitational potential is the same at all (a) (b)
T T2
points of the circle y2 + z2 = 4.
IIT, 1993 r3 r3
(c) (d)
25. Two bodies of masses m1 and m2 are initially at rest T2 T3
30. A meteor of mass M breaks up into two parts. The
move toward each other under mutual gravitational mass of one part is m. For a given separation r the
9.16 Comprehensive Physics—JEE Advanced

mutual gravitational force between the two parts


2 2 GmM GmM
will be the maximum if (a) (b)
27 R 2 8R 2
M M
(a) m = (b) m =
2 3 GmM 2 GmM
(c) 2 (d)
M M 9R 9 9R2
(c) m = (d) m =
2 2 2
31. A body of mass m is raised to a height h above the
surface of the earth of mass M and radius R until
its gravitational potential energy increases by
1
mgR. The value of h is
3
R R
(a) (b) Fig. 9.20
3 2
35. Two objects of masses m and 4m are at rest at
mR mR
(c) (d)
M m M under mutual gravitational attraction. Then, at a
32. Two balls A and B are thrown vertically upwards separation r, which of the following is true?
from the same location on the surface of the earth (a) The total energy of the system is not zero.
gR 2 gR (b) The force between them is not zero.
with velocities 2 and respectively, (c) The centre of mass of the system is at rest.
3 3
(d) All the above are true.
where R is the radius of the earth and g is the
acceleration due to gravity on the surface of the IIT, 1994
earth. The ratio of the maximum height attained by 36. A satellite is launched into a circular orbit of radius
A to that attained by B is R around the earth. A second satellite is launched
(a) 2 (b) 4 into an orbit of radius 1.01 R. The period of the
(c) 8 (d) 4 2
33. A uniform sphere of mass M and radius R exerts a approximately
force F on a small mass m situated at a distance of (a) 0.5% (b) 1.0%
2R from the centre O of the sphere. A spherical por- (c) 1.5% (d) 3.0%
tion of diameter R is cut from the sphere as shown IIT, 1995
in Fig. 9.19. The force of attraction between the 37. A simple pendulum has a time period T1 when on
remaining part of the sphere and the mass m will the earth’s surface, and T2 when taken to a height
be R above the earth’s surface, where R is the radius
7F 2F of the earth. The value of T2/T1 is
(a) (b)
9 3 (a) 1 (b) 2
4F F (c) 4 (d) 2
(c) (d)
9 3 IIT, 2001
38. An ideal spring with spring-constant k is hung from
the ceiling and a block of mass M is attached to its
lower end. The mass is released with the spring
initially unstretched. Then the maximum extension
in the spring is
(a) 4 Mg/k (b) 2 Mg/k
(c) Mg/k (d) Mg/2k
IIT,2002
Fig. 9.19
39. A geo-stationary satellite orbits around the earth
34. The centres of a ring of mass m and a sphere of in a circular orbit of radius 36000 km. Then, the
mass M of equal radius R, are at a distance 8 R time period of a spy satellite orbiting a few hundred
apart as shown in Fig. 9.20. The force of attraction kilometers above the earth’s surface (REarth = 6400
between the ring and the sphere is km) will approximately be
Gravitation 9.17

(a) (1/2) h (b) 1 h gR 2 gR


(c) 2 h (d) 4 h (a) – (b) –
10 10
IIT, 2002
3gR 4 gR
40. A satellite of mass m is moving in a circular orbit (c) – (d) –
of radius R above the surface of a planet of mass M 10 10
and radius R. The amount of work done to shift the 46. The radius of the earth is R and g is the acceleration
satellite to a higher orbit of radius 2R is (here g is due to gravity on its surface. What should be the
the acceleration due to gravity on planet’s surface) angular speed of the earth so that bodies lying on
mgR the equator may appear weightless?
(a) mgR (b)
6 g 2g
mMgR mMgR (a) (b)
(c) (d) R R
M m 6 M m
g g
41. The change in the gravitational potential energy (c) (d) 2
2R R
when a body of mass m is raised to a height nR
47. If W1, W2 and W3 represent the work done in
above the surface of the earth is (here R is the
moving a particle from A to B along three differ-
radius of the earth)
ent paths 1, 2 and 3 (as shown in Fig. 9.21) in
n n m
(a) mgR (b) mgR
n 1 n 1 correct relation between W1, W2 and W3.

(c) nmgR (d)


mgR (a) W1 > W3 > W2 (b) W1 = W2 = W3
n (c) W1 < W3 < W2 (d) W1 < W2 < W3
42. A body of mass m is dropped from a height nR
IIT, 2003
above the surface of the earth (here R is the radius
of the earth). The speed at which the body hits the
surface of the earth is
2 gR 2 gR
(a) (b)
n 1 n 1
2 gRn 2 gRn
(c) (d)
n 1 n 1
43. Two solid spheres of radii r and 2r, made of the
same material, are kept in contact. The mutual
gravitational force of attraction between them is Fig. 9.21
proportional to
1 1 48. A binary star system consists of two stars of
(a) 4 (b) 2 masses M1 and M2 revolving in circular orbits
r r of radii R1 and R2 respectively. If their respective
(c) r2 (d) r4 time periods are T1 and T2, then
44. A comet is moving in a highly elliptical orbit round
(a) T1 > T2 if R1 > R2
the sun. When it is closest to the sun, its distance
from the sun is r and its speed is v. When it is (b) T1 > T2 if M1 > M2
farthest from the sun, its distance from the sun is (c) T1 = T2
R and its speed will be
3/ 2
1/ 2 T1 R1
r r (d)
(a) v (b) v T2 R2
R R IIT, 2006
r 3/ 2
r 2 49. A spherically symmetric gravitational system of
(c) v (d) v 0 for r R
R R particles has a mass density = where
45. The value of the acceleration due to gravity at the 0 for r R
surface of the earth of radius R is g. It decreases 0 is a constant. A test mass can undergo circular
by 10% at a height h above the surface of the earth.
The gravitational potential at this height is of particles. Its speed v as a function of distance
9.18 Comprehensive Physics—JEE Advanced

r(0 < r < ) from the centre of the system is repre- 50. A satellite is moving with a constant speed ‘V ’ in a
sented by (see Fig. 9.22) circular orbit about the earth. An object of mass ‘m’
IIT, 2008 is ejected from the satellite such that it just escapes
from the gravitational pull of the earth. At the time
of its ejection, the kinetic energy of the object is
1
(a) mV 2 (b) mV2
2
3
(c) mV 2 (d) 2mV2
2
IIT, 2011

Fig. 9.22

ANSWERS

1. (a) 2. (a) 3. (d) 4. (b) 5. (d) 6. (b)


7. (d) 8. (c) 9. (c) 10. (b) 11. (a) 12. (d)
13. (c) 14. (a) 15. (d) 16. (c) 17. (a) 18. (b)
19. (c) 20. (b) 21. (d) 22. (b) 23. (d) 24. (b)
25. (a) 26. (b) 27. (c) 28. (a) 29. (c) 30. (a)
31. (b) 32. (c) 33. (a) 34. (a) 35. (d) 36. (c)
37. (d) 38. (b) 39. (c) 40. (b) 41. (a) 42. (d)
43. (d) 44. (b) 45. (c) 46. (a) 47. (b) 48. (c)
49. (c) 50. (b)

SOLUTIONS
1. Gravitational potential on the surface of the shell 3. Let v be the required speed.
due to the body of mass m is Gain in K.E. = loss in P.E.
Vb = –
Gm = P.E. at the surface – P.E. at
R the centre of the earth
Gravitational potential on the surface of the shell 1 GmM 3 GmM
due to shell itself is mv2 = – –
2 R 2 R
GM
Vs = – GM GM
R v= = gR g
G R R2
V = Vb + Vs = – (M + m), which is choice (a). 4. The total energy of a satellite in orbital radius r is
R
m v2 1 GMm
2. =
GmM
v=
GM E = K.E. + P.E. = mv2 –
R R2 R 2 r
GM GMm GMm GM
Angular momentum L = mvR = m R = v
R 2r r r
= m(GMR)1/2. GMm
i.e. L R1/2. So the correct choice is (a). =
2r
Gravitation 9.19

r = R + R = 2R. Hence
k r 3/ 2
GMm v=
E= m
4R
2 r m
In the second case, r = R + 2R = 3R. Hence T= =2 r
v kr 3/ 2
GMm 7/4
E = i.e. T r , which is choice (c)
6R
GM
Increase in energy E = E – E 10. g =
R2
GMm GMm
= – g 2 R
6R 4R =
m

g M R
GMm
= = – 1% – 2 (– 1%) = + 1%
12 R
Hence g will increase by 1%.
E
Percentage increase = 100 11. Distance of m from M1 or M2 = r/2. Therefore,
|E|
G m M1 G m M2
GMm / 12 R Total P.E. =
= 100 = 33.3% r/2 r/2
GMm / 4 R
2Gm
5. E1 = P.E. at h = R – P.E. at h = 0 = (M1 + M2)
r
= P.E. at r = 2R – P.E. at r = R
If v is the required velocity of projection, the total
GMm GMm GMm
= initial energy is
2R R 2R
1 2Gm
GMm GMm 3 GMm Ei = mv 2 – (M1 + M 2)
E2 = = 2 r
4R R 4R
Ef = 0. Putting Ei = Ef , we get
E1 2
= G M1 M2
1/ 2
E2 3 v= 2
GMm r
6. P.E. on the surface of the earth =
R dA L
12. A real velocity = , where L is the magni-
GMm dt 2m
P.E. at a height h (= R) = tude of angular momentum of the planet about the
2R
sun. L = mvr sin . Hence,
GMm GMm
Gain in P.E. = dA mvr sin v r sin
2R R = =
dt 2m 2
GMm 1 GM So the correct choice is (d).
= = mgR g
2R 2 R2 13. Total energy of the planet in an elliptical orbit of
7. The correct choice is (d) because the escape veloc- semimajor axis a is
ity is independent of the direction along which the GmM
body is projected. E1 = –
2a
8. Escape velocity at height h = 2g R h , where
2 Total energy of the planet when it is at a distance
g =g
R
. Hence the correct choice is (c). x from the sun (here v = speed of the plane at that
R h instant) is
9. Since the gravitational force provides the necessary E2 = K.E. + P.E.
centripetal force,
1 GmM
mv2 mv2 –
=
r –5/2 2 x
r From conservation of energy, E1 = E2, i.e.
mv2 GmM 1 GmM
= kr–5/2, k = constant – = mv2 –
r 2a 2 x
9.20 Comprehensive Physics—JEE Advanced

Fg > Fc
2 1
which gives v = GM , which is choice (c).
x a GmM 2 R3 2
or > mR or M >
R2 G
mv 2 GmM Hence the correct choice is (c).
14. =
r r2 17. Consider a satellite of mass m moving with a speed
GM = v2 r v at an altitude r (measured from the centre of the
earth). Then
GM v2 r
g= = 1
R2 R2 Kinetic energy (KE) = mv 2
2 GmM
So the correct choice is (a). Gravitational potential energy (PE) = –
15. Force between any two spheres is r
where M is the mass of the earth.
For a satellite in circular orbit, we have
GM 2 GM 2
F= 2 = mv 2
2R 4R2 =
GmM
or v2 =
GM
r r2 r
This is the force exerted on any sphere (say A) by
the other two spheres B and C (Fig. 9.23). Thus the 1 GmM
or mv2 =
resultant force on sphere A is 2 2r
Fr = F2 F2 2 F 2 cos 60 i.e. KE =
GmM
2r
3 GM 2 Thus the KE of a satellite in a circular orbit is nu-
= 3F = , which is choice (d).
4R2 merically half its PE but opposite in sign. The total
energy of the satellite in orbit is
GmM GmM
E = KE + PE = –
2r r
GmM
=–
2r
It is given that r = 2R + R = 3R, where R is the
radius of the earth.
GmM
E= –
6R
GmM
Now PE on the surface of the earth = –
R
Minimum energy required (E min)
Fig. 9.23
GmM GmM
16. An object of mass m, placed at the equator of the =– –
star, will experience two forces: (i) an attractive 6R R
force due to gravity towards the centre of the star 5GmM
=
and (ii) an outward centrifugal force due to the 6R
rotation of the star. The centrifugal force arises Hence the correct choice is (a).
because the object is in a rotating (non-inertial) 18. The speeds of stars at separation r are negligible.
frame; this force is equal to the inward centripetal Therefore, their energy is entirely potential at this
force but opposite in direction. Force on object due separation (since KE = 0)
to gravity is Gm1 m2
GmM E1 = (PE at r) = –
Fg = r
R2
Gm 2
Centrifugal force on the object is =–
r
Fc = mR 2
As the stars approach each other under gravitational
attraction, they begin to acquire speed and hence
off if
Gravitation 9.21

kinetic energy at the expense of potential energy. Gm 1 1 1


When they eventually collide, the separation be- = 1
r 2 4 8
tween their centres is
r = R + R = 2R Gm 1 1 1 1
=
r 20 21
2 2
23
At r = 2R, the total energy is
E2 = PE at (r = 2R) + KE at (r = 2R) Gm 1 2Gm
= =
Gm 2
1 1 r 1 r
=– + mv2 + mv2 1
2R 2 2 2
Hence the correct choice is (d).
Gm 2
or E2 = – + mv2 22. From Newton’s second law of motion, force is the
2R rate of change of momentum, i.e.
From the principle of conservation of energy,
d dM
E1 = E2, i.e. F= (Mv) = v= v2
Gm 2 Gm 2 dt dt
– =– + mv2 dM
r 2R v
F v2 dt
1 1 Retardation = or acceleration =
which gives v = Gm M M
2R r v2
. Hence the correct choice is (b).
Hence the correct choice is (b). M
19. The acceleration due to gravity at a place on the 23. The orbital speed of a satellite at a height r from the
surface of the earth is given by GM
g = g – 2R cos centre of the earth is given by v =
r
where is the latitude of the place. At the equator, where M is the mass of the earth. If m is the mass
= 0. Therefore of the satellite, its kinetic energy is
g = g – 2R
1 1 GM
Therefore, the tension in the string will be K= mv2 = m
mg = mg – m 2R 2 2 r
GmM 1
= – m 2R Thus, K is proportional to . Now, the time period
R2 r
Hence the correct choice is (c). r3
of the satellite is given by T = 2
20. O will be g R2
1 1 1 1 where R is the radius of the earth. Thus T r3/2. But
I = Gm 1
r2 2r 2
4r 2
8r 2
K . Hence K T–2/3, which is choice (d).
r
Gm 1 1 1 24. The distance of each cavity from the centre O is the
= 2
1 2 2 2
r 2 4 8 same. Since the two cavities are symmetrical with
respect to the centre O and the mass of the sphere
Gm 1 1 1 1
= 2 0 2 4
can be regarded as being concentrated at the centre
r 2 2 2 26 O, the gravitational force due to the sphere is zero
at the centre. Hence choice (a) is correct. For the
Gm 1
Gm 1 4Gm same reason, the gravitational potential is the same
= 2 1 = 2 = at all points of the circle y2 + z2 = 36 whose radius
r 1 r 1 1 3r 2
2 2
4 is 6 units and at all points of the circle y2 + z2 = 4.
Hence choices (c) and (d) are also correct. But the
Hence the correct choice is (b).
gravitational force at point B cannot be zero.
21. The gravitational potential, in magnitude, at point 25. -
O is tance from each other, their gravitational potential
1 1 1 1
V = Gm energy is zero. When they are at a distance r from
r 2r 4r 8r each other the gravitational P.E. is
9.22 Comprehensive Physics—JEE Advanced

G m1 m2 linear momentum vary periodically. Hence choices


PE = – (c) and (d) are also incorrect.
r
29. Let m and M be the masses of the earth and the sun
The minus sign indicates that there is a decrease
respectively and v the speed of the earth in circular
in P.E. This gives rise to an increase in kinetic
orbit. To keep the earth in circular orbit, the gravi-
energy. If v1 and v2 are their respective velocities GmM
when they are a distance r apart, then, from the tational force must balance the centripetal
law of conservation of energy, we have r2
2
mv
force , i.e.
1 G m1 m2
m1v21 = r
2 r GmM m v2
=
2 G m2 r2 r
or v1 =
r v2 r
or M=
1 G m1 m2 G
and m2v22 =
2 r 2 r 4 2 r3
Also v = . Using this, we get M = or
2 G m1 T T 2G
or v2 = r3
r M .
Therefore, their relative velocity of approach is T2
Hence the correct choice is (c).
2 G m2 2 G m1 30. Mass of the second part = M m. Gravitational
v1 + v2 =
r r force between the two parts is
2G G(M m) m G
=
(m2 m1 ) F= 2
= 2
(Mm m 2)
r r r
Hence the correct choice is (a). dF d 2F
26. According to Kepler’s law of periods, F will be maximum if = 0 and is nega-
tive.
dm d m2
3/ 2 3/ 2
T1 R1 R1
= dF G dF
T2 R2 R1 / 2 Now, = 2 (M 2m). Setting = 0, we
dm r dm
3/ 2
= 2 2 2 get M 2m = 0 or m =
M
. Now
2
T1 365 days
T2 = = 129 days. d 2F 2G
2 2 2 2 = – 2 , which is negative.
d m2 r
27. For a satellite, we have
GmM Hence the correct choice is (a).
Kinetic energy = GM m
2r 31. PE on the surface of earth = –
R
GmM
Potential energy = – PE at a height h above the surface of earth =
r GM m
Total energy E0 = KE + PE –
( R h) GM m GM m
Gm M Gm M Gm M PE Increase in PE = – –
= ( R h) R
2r r 2r 2
or PE = 2E0. Hence the correct choice is (c). 1 1 h
= GMm = GMm
28. For elliptical orbit, the earth is at one focus of the R R h R ( R h)
ellipse. For spherical bodies, the gravitational force g Rmh GM
is central (or radial). Hence statement (a) is correct. = g
( R h) R2
The gravitational force exerts no torque on the sat-
ellite. Hence the angular momentum of S remains 1
PE will increase by mgR at a value of h given
constant in magnitude as well as direction. Hence by 3
choice (b) is incorrect. For elliptical orbit, the dis- g Rmh 1
tance of the satellite from the earth varies periodi- = mgR
( R h) 3
cally. Hence potential energy, kinetic energy and
Gravitation 9.23

h 1 R because dx = × L = m, the mass of the ring.


or = or h = , which is choice (b).
R h 3 2 Hence the correct choice is (a).
32. If h is the maximum height attained, then we have
1 GM m GM m
mv2 – =–
2 R ( R h)
2g h R GM
which gives v2 = g
( R h) R2
4g R 2g hA R
For ball A, we have = hA = 4R Fig. 9.24
3 ( R hA )
35.
2g R 2g hB R R
For ball B, we have = hB = the system of two masses and the force between
3 ( R hB ) 2
hA rest initially and there is no external force, the cen-
= 8, which is choice (c).
hB tre of mass cannot move. Hence the correct choice
33. The force of attraction between the complete sphere is (d).
and mass m is 36. According to Kepler’s law of period, T2 = kR3 where
GmM GmM k is a constant. Taking logarithm of both sides, we
F= 2 = (i) have
(2 R) 4R2
4 3 2 log T = log k + 3 log R
Mass of complete sphere is M = R . Mass
3 Differentiating, we get
4 R 3 T R
of the cut out portion is m0 = . Thus, 2 =0+3
3 2 T R
M
m0 = . The distance between the centre of the T 3 R 3 1.01 R R
8 or = = × 100
R 3R T 2 R 2 R
cut out portion and mass m = 2R = .
2 2 = 1.5%
Hence the force of attraction between the cut out Hence the correct choice is (c).
portion and mass m is 37. The acceleration due to gravity at a height h above
G m0 m G ( M/8) m GmM 2 the surface of the earth is given by
f= 2
= 2
= 2 2
(3R / 2) 9 R /4 4R 9 g2 = g1
R
2F R h
Using (i), we get f = . Therefore, the force of where g1 is the value at the surface of the earth.
9
attraction between the remaining part of the sphere Now
2F 7F
and mass m = F f = F = which is l l
choice (a). 9 9 T2 = 2 and T1 = 2
g2 g1
34. Refer to Fig. 9.24. Let be the mass per unit
length of the ring. L = 2 R is the length of the ring. T2 g1 R h R R
Consider a small element of length dx of the ring = = = =2 ( h = R)
T1 g2 R R
located at C. Then
G M dx Hence the correct choice is (d).
Force along BC is f = . Therefore, force
(3R) 2 38. Let x be the extension in the spring when it is loaded
G M dx 8R with mass M. The change in gravitational potential
along BA is dF = f cos = =
9R 2
3R energy = Mgx. This must be the energy stored in the
8 GM dx 1 2
spring which is given by kx . Thus
27 R2 2
8 GM 8 GMm 1 2 2Mg
Total force = dx = kx = Mg x or x = , which is choice (b).
27 R 2 27 R 2 2 k
9.24 Comprehensive Physics—JEE Advanced

39. For a satellite of mass m moving with a velocity v in Distance between their centres is d = r + 2r = 3r.
a circular orbit of radius r around the earth of mass 4 4
M, we have G r3 ( 2 r )3
G M1 M 2 3 3
Now F = =
mv 2 GmM GM d2 9r 2
= or v =
r r2 r which gives F r4, which is choice (d).
44. The angular momentum of the comet is con-
2 r 2 r GM stant over the entire orbit. Hence vr = VR or
Now v= . Thus = or T r3/2.
T T r r
V=v , which is choice (b).
3/ 2 R
T2 r2
= (1) GM
T1 r1 45. gh =
( R h) 2
Given r2 = 6400 km and r1 = 36000 km. For a
geostationary satellite T1 = 24 h. Using these values GM gh R2
Also g = . Thus = .
64 3 / 2 R2 g ( R h) 2
in (1), we have get T2 = 24 × = 1.8 h.
Hence the closest choice is (c). 360 90 g
Given gh =
40. PE at a distance r from the centre of the planet 100
GM m
=– R2 9 10 R
r = or (R + h) =
( R h) 2 10 3
GM m GM m
Initial PE = – =–
R R 2R GM GMR 2 gR 2
Potential = – =– 2 =–
GM m GM m ( R h) R ( R h) ( R h)
Final PE = – =–
R 2R 3R 3gR
=–
Now, work done = increase in PE 10
Hence the correct choice is (c).
GM m 1 1 GM m 1 GM 46. At the equator, the value of g is
= = = mgR g
R 2 3 6R 6 R2 g =g–R 2
Hence the correct choice is (b). where is the angular speed of the earth. For bodies
to appear weightless at the equator, g = 0, i.e.
GM m
41. Change in PE =
GM m
– =
n
mgR g – R 2= 0
R ( n 1 ) R n 1
Hence the correct choice is (a). g
which gives = . Hence the correct choice is (a).
42. From the principle of conservation of energy, we R
have 47. Gravitational force is conservative. The work done
1 GM m GM m by a conservative force on a particle moving be-
mv2 =– tween two points does not depend on the path taken
2 R ( R n R)
by the particle. Hence the correct choice is (b).
2 n R GM 2n R g
which gives v2 = = 48. In a binary star system, the two stars move under
(n 1) R 2 (n 1) their mutual gravitational force. Therefore, their
GM angular velocities and hence their time periods
g are equal. Thus the correct choice is (c).
R2
49. If M is the total mass of the system of particles, the
Hence the correct choice is (d). orbital speed of the test mass is
43. If is the density of the material of each
sphere, then the mass of the sphere of radius r is GM
v=
4 3 r
M1 = r and the mass of the sphere of radius
3 4 3
G r
4 3 0
2r is M2 = (2r)3 . For r R, v = which gives v r,
3 r
Gravitation 9.25

i.e. v increases linearly with r up to r = R. Hence 50. Let M be the mass of the earth and r be the
choices (b) and (d) are wrong. orbital radius of the satellite. The energy needed
For r > R, the whole mass of the system is so that the object of mass m escapes from x = r
4 to x = is
M = R3 0, which is constant. Hence for
3 dx GMm
E = GMm (i)
r > R, x 2
r
r

v=
GM The orbital speed V is given by
r mV 2 GMm GMm
1 = mV2 = (ii)
i.e, v . Hence the correct choice is (c) r r2 r
r
Using (ii) in (i), E = mV2.

II

Multiple Choice Questions with one or More Choices Correct


1. A satellite is moving around the earth in a stable 5. Choose the correct statements from the following:
circular orbit. Choose the correct statements from (a) The magnitude of the gravitational force
the following. between two bodies of mass 1 kg each and
(a) It is moving at a constant speed. separated by a distance of 1 m is 9.8 N.
(b) Its angular momentum remains constant. (b) Higher the value of the escape velocity for a
(c) It is acted upon by a force directed away planet, the higher is the abundance of lighter
from the centre of the earth which counter gases in its atmosphere.
balances the attraction by the earth. (c) The gravitational force of attraction between
(d) It behaves as if it were as freely falling body. two bodies of ordinary mass is not noticeable
2. The escape velocity from a planet depends upon because the value of the gravitation constant
(a) the mass of the body is extremely small.
(b) the mass of the planet (d) Force of friction arises due to gravitational
(c) the average radius of the planet attraction.
(d) the average density of the planet 6. Choose the wrong statements from the following.
3. The orbital velocity of a body in a stable orbit (a) It is possible to shield a body from the gravi-
around a planet depends upon
(a) the average radius of the planet shielding material between them.
(b) the height of the body above the planet (b) The escape velocity of a body is independent
(c) the acceleration due to gravity of the mass of the body and the angle of
(d) the mass of the orbiting body projection.
4. Choose the correct statements from the following: (c) The acceleration due to gravity increases due
(a) The equivalence of inertial and gravitational to the rotation of the earth.
mass has provided a clue to the deeper under- (d) The gravitational force exerted by the earth
standing of gravitation . on a body is greater than that exerted by the
(b) At poles, the effect of rotation of earth on body on the earth.
the value of g is the minimum. 7. A comet is revolving around the sun in a highly
(c) Very massive rockets and extremely tiny par- elliptical orbit. Which of the following will remain
ticles, such as the molecules of a gas, require the constant throughout its orbit?
same initial velocity to escape from the earth. (a) Kinetic energy
(d) A geostationary satellite, if imparted the nec- (b) Potential energy
essary velocity, can be put in orbit at any (c) Total energy
height above the earth. (d) Angular momentum
9.26 Comprehensive Physics—JEE Advanced

8. A satellite is orbiting the earth. If its distance from 14. Two stars of masses m and 2 m are co-rotating about
the earth is increased, its their centre of mass. Their centres are at a distance
(a) angular velocity would increase r apart. If r is much larger than the size of the stars,
(d) linear velocity would increase then their
(c) angular velocity would decrease (a) common period of revolution is proportional to
(d) time period would increase. r3/2.
9. For two satellites at distance R and 7R above the (b) orbital velocities are in the ratio 2 : 1.
earth’s surface, the ratio of their (c) kinetic energies are in the ratio 1 : 2 .
(a) total energies is 4 and potential and kinetic (d) angular momenta are in the ratio 1 : 4.
energies is 2 15. A space-ship is orbiting close to the surface of the
(b) potential energies is 4 earth at a speed v. The radius of the earth is R and
(c) kinetic energies is 4 g is the acceleration due to gravity close to the
(d) total energies is 4. surface of the earth. An additional speed of v0 is to
10. A satellite is orbiting the earth in a circular orbit of be imparted to the space-ship so that it overcomes
radius r. Its the gravitational pull of the earth. Then
(a) kinetic energy varies as 1/r (a) v = Rg (b) v = 2Rg
(b) angular momentum varies as 1/ r 2 1
(c) v0 = Rg (d) v0 = Rg 2 1
(c) linear momentum varies as 1/ r
(d) frequency of revolution varies as 1/r3/2. 16. A satellite of mass m is moving in a circular orbit
11. If both the mass and radius of the earth decrease by of radius r around a planet of mass M.
1%, the value of (a) The magnitude of angular momentum with
(a) acceleration due to gravity would decrease respect to the centre of the orbit is m GMr ,
by nearly 1% where G is the gravitation constant.
(b) acceleration due to gravity would increase by 1% (b) The magnitude of the angular momentum is
(c) escape velocity from the earth’s surface would mR 2gr where g is the acceleration due to
decrease by 1% gravity on the surface of the planet.
(d) the gravitational potential energy of a body (c) The direction of angular momentum is paral-
on earth’s surface remains unchanged. lel to the plane of the orbit.
12. An object is taken from a point P to another point (d) The direction of angular momentum is per-
Q pendicular to the plane of the orbit.
(a) assuming the earth to be spherical, if both P and Q IIT, 1997
lie on earth’s surface the work done is zero. 17. Two bodies of masses m1 = m and m2 = 4 m are
(b) If P is on earth’s surface and Q above it, the placed at a distance r
work done is minimum when it is taken along is zero at a point P at a distance x from mass m1.
the straight line PQ. The gravitational potential at point P is V. Then
(c) The work done depends only on the positions (U is the gravitational P.E. of the system)
of P and Q and is independent of the path r 2r
along which the particle is taken. (a) x = (b) x =
3 3
(d) there is no net work done if the object is
taken from P to Q and then brought back to 7G 9G
(c) U = – (d) U = –
P, along any path. r r
13. A satellite is moving in a circular orbit around the 18. A small planet is revolving with speed v around
earth with a speed equal to half the escape velocity a very massive star in a circular orbit of radius
from the earth. The radius of the earth is R and h is r with a period of revolution T. If the gravita-
the height of the satellite above the surface of the tional force between the planet and the star were
earth. If the satellite is suddenly stopped in its orbit inversely proportional to r, then
and allowed to fall freely, it will hit the surface of (a) v is independent of r.
the earth with a speed v. Then (b) v decreases if r is increased.
R (c) T increases if r is increased.
(a) h = (b) h = R
2 (d) T is independent of r.
19. The total energy of a satellite of mass m moving
(c) v = 2gR (d) v = gR with speed v around the earth of mass M in a
Gravitation 9.27

circular orbit of radius r is directly proportional to 25. A satellite of mass m is launched from the surface
(a) m (b) M of the earth in a circular orbit at a height h = R
(c) v (d) r above the surface of the earth; R being the radius
20. Assuming the earth to be a sphere of radius R of the earth. If g is the acceleration due to gravity
and uniform density, if the acceleration due to on the surface of the earth and the air resistance
gravity at a point at a distance r from the centre is neglected,
of the earth is g, then (a) the increase in gravitational potential energy
1 is mgR
(a) g r for r < R (b) g for r < R
r (b) the velocity with which it was projected from
1 the earth is gR
(c) g r2 for r > R (d) g for r > R (c) the total energy spent in launching the satel-
r2 3
21. A satellite is revolving around a planet in a lite in the circular orbit is mgR
4
circular orbit. During its motion, the satellite (d) the acceleration due to gravity at the site of
begins to experience a resistive force (possibly the satellite is g/4.
due to cosmic dust). As a result 26. Two satellites of the same mass are put in the
(a) its kinetic energy will decrease same orbit round the earth but they revolve in
(b) its angular speed will decrease opposite directions. They undergo an inelastic
(c) its time period of revolution will increase collision and stick together.
(d) its angular momentum will decrease. (a) The ratio of the potential energy of the system
22. If a satellite revolving around the earth is moved before and just after the collision is 2 : 1
from one stable orbit to another stable orbit of a (b) The ratio of the total energy of the system
higher orbital radius, then its before and just after the collision is 1 : 2
(a) time period will increase (c) After the collision, the combined mass will
(b) centripetal force will decrease. continue moving in the same orbit
(c) gravitational potential energy will increase (d) After the collision, the combined mass will
(d) angular momentum will remain unchanged fall freely under the gravity of the earth.
23. A comet is moving around the sun in a highly 27. A solid sphere of uniform density and radius 4 units
elliptical orbit. Its is located with its centre at origin O of coordinates.
(a) kinetic energy and gravitational potential Two spheres of equal radii 1 unit, with their centres
energy both change over the orbit at A (– 2, 0, 0) and B(2, 0, 0) respectively are taken
(b) total energy remains constant throughout the out of the solid leaving behind spherical cavities as
orbit shown in Fig. 9.25.
(c) linear momentum changes in magnitude as
well as direction over the orbit
(d) angular momentum remains constant over the
orbit.
IIT, 1998
24. A body of mass m is released from rest from
above at a distance r from the centre of the earth
of mass M and radius R. If the air resistance is
neglected, it will strike the surface of the earth
with a velocity v. If g is the acceleration due to
gravity on earth s surface,
Fig. 9.25
(a) v = 2Rg if r >> R
(a) the gravitational force due to this object at the
(b) v = Rg if r = 2R origin is zero.
(b) the gravitational force at point B (2,0,0) is zero.
(c) v =
Rg
if r = 4R (c) the gravitational potential is the same at all
2 points of the circle y 2 + z2 = 36.
1 (d) the gravitational potential is the same at all
(d) v = Rg if r = 8R points of the circle y2 + z2 = 4.
2
IIT, 1993
9.28 Comprehensive Physics—JEE Advanced

28. -
F1 r12
tances r1 and r2 from the centre of a uniform sphere (c) if r1 < R and r2 < R
of radius R and mass M are F1 and F2 respectively. F2 r22
Then F1 r2
(d) if r1 > R and r2 > R
F r1 F2 r1
(a) 1 if r1 < R and r2 < R
F2 r2 IIT, 1994
F1 r22
(b) if r1 > R and r2 > R
F2 r12

ANSWERS AND SOLUTIONS


1. The correct choices are (a), (b) and (d). Thus decreases with increase in r. The time
2. The correct choices are (b), (c) and (d). period of the satellite is given by
3. The correct choices are (a), (b) and (c).
4. The only incorrect statement is (d). A geostation- r3
T =2
ary satellite can be put in orbit only at a height GM
of 35,870 km above the earth. Thus T increases as r is increased. Hence the
5. Statement (a) is incorrect; the magnitude of the correct choices are (c) and (d).
force is 6.67 10–11 N. Statement (b) is correct. 9. Distances of the two satellites from the centre of
If the escape velocity for a planet is high, the the earth are r1 = 2R and r2 = 8R respectively. R =
thermal speeds of lighter gases are less than the earth’s radius. Their potential energies are
escape velocity. Therefore, lighter gases are not
able to escape. Statement (c) is also correct. State- GmM
V1 = –
ment (d) is incorrect; force of friction arises due r1
to electrical forces. Hence correct choices are (b) GmM
and (c). and V2 = –
r2
6. Statement (a) is incorrect; there is no method by
which a body can be shielded from the gravi- V1 r2 8R
Their ratio is = = = 4.
correct. Statement (c) is wrong. As the earth
V2 r1 2R
rotates about its axis, a body on the surface of The kinetic energy of a satellite can be obtained
the earth also rotates with it. Since the body is from relation
in a rotating (non–inertial) frame, it experiences mv 2 GmM
an outward centrifugal force against the inward =
r r2
force of gravity. As a result, the acceleration due
1 GmM
to gravity decreases due to rotation. Statement (d) or K= mv2 =
is incorrect, the forces are equal in magnitude but 2 2r
opposite in direction. Hence choices (a), (c) and GmM GmM
(d) are wrong. Thus K1 = and K2 =
2r1 2r2
7. The correct choices are (c) and (d). The kinetic
The ratio of their kinetic energies is
energy of the comet changes because its speed
in the orbit keeps changing. The potential energy
K1 r2 8R
= = = 4.
changes because the distance of the comet from the K2 r1 2R
Their total energies are
sun keeps changing for an elliptical orbit.
GM GmM GmM GmM
8. Linear velocity or orbital velocity is v = , E1 = – =–
r r1 2r1 2r1
where r is the distance of the satellite from the
GmM GmM GmM
centre of the earth. Therefore v decreases as r is and E2 = – =–
increased. Also v = r , where is the angular r2 2r2 2r2
velocity. Therefore E1 r2 8R
Their ratio is = = = 4.
v 1 GM GM E2 r1 2R
= = =
r r r r3 Hence the correct choices are (b), (c) and (d).
Gravitation 9.29

1 GmM 1 1 1 2GM
10. Kinetic energy mv2 = or KE . Thus But v= ve =
2 2r r 2 2 R
choice (a) is correct. Angular momentum = mvr =
GM GM
m r = m GM r , which is proportional =
r 2R
to r . Hence choice (b) is wrong. Linear momen- GM GM
GM =
tum mv = m , which is proportional to r 2R
1 r
. Hence choice (c) is correct. The frequency or r = 2R. Height above earth = 2R – R = R.
r Potential energy of the satellite in its orbit is
of revolution is GmM GmM
E1 = – =– ( r = 2R)
1 1 GM 1 r 2R
= = 3
i.e. 3/ 2 The kinetic energy is zero because the satellite
T 2 r r
is stopped. Potential energy of the satellite on the
Hence the correct choices are (a), (c) and (d).
surface of the earth is
11. The acceleration due to gravity is
GmM
GM E2 = –
g= R
R2 Loss of PE = E1 – E2
The new value of g would be
GmM GmM
G 0.99 M GM =– –
g = 2 1.01 1.01 g 2R R
0.99 R R2 GmM
Thus g would increase by about 1%. The new =
2R
escape velocity would be
This is converted into kinetic energy. If v is the
2 0.99 M G 2M G speed with which the satellite hits the surface of the
ve = = = ve earth, then from the law of conservation of energy,
0.99 R R
we have
Thus the escape velocity will remain unchanged. 1 GmM
The potential energy of a body of mass m on mv2 =
2 2R
earth’s surface would be
GM GM
GM 0.99 M or v2 = = gR g
– =–
GmM R R2
0.99 R R The correct choices are (b) and (d).
Thus the potential energy will also remain un- 14. The distance x of the star of mass m from the centre
changed. Hence the correct choices are (b) and(d). of mass is given by
12. Work done is independent of the path chosen and m 2m
=
x (r x)
object. Also the work done on any closed path in a
r
which gives x = . The orbital speed v1 of the star
the surface of the earth is at the same potential, no 3
work is done for points on the surface of the earth. of mass m1 = m is given by (here m2 = 2 m)
Hence the correct choices are (a), (c) and (d).
Gm1 m2 m1v12 m1v12
13. If M is the mass of the earth, the escape velocity = =
is r2 x r /3
2GM 2Gm
ve = which gives v1 =
Gm2
= (i)
R 3r 3r
For a satellite of mass m and orbital radius r (= its
2 x 2 r 3r
distance from the centre of the earth), the orbital Time period (T) of m = =
speed v is given by v1 3 2GM
mv 2 GmM GM =
2
(r)3/2
= 2
or v =
r r r 3GM
9.30 Comprehensive Physics—JEE Advanced

or T r3/2.
The orbital speed v2 of the star of mass m2 = 2m is
given by
Gm1m2 m2 v22 Gm(2m) 2m v22
= or =
r2 (r x) r2 2r / 3
v
2Gm
or v2 = = v1 [see Eq. (i)]
3r
1 1 1
Now K1 = m1 v12 and K2 = m2 v22 = (2m) v22
2 2 2
2
Fig. 9.26
K1 v1 1
= = , since v1 = v2. The magnitude of angular momentum is
K2 2v22 2
mv1r L = mrv sin
Angular momentum of m1 is L1 = m1v1 x = where is the angle between vectors r and v. For a
3
circular orbit, = 90°. Therefore
Angular momentum of m2 is L2 = m2v2(r – x) L = mrv (1)
2 mv 2 2 r The gravitational force of attraction on the satellite
= is GMm/r2 which provides the necessary centripetal
3
L1 1 force mv2/r, i.e.
(since v1 = v2) m v2
L2 4 GMm
=
The correct choices are (a), (c) and (d). r2 r
15. The orbital velocity of the space-ship of mass M or (mrv) = m GMr (2)
and orbital radius r is given by From Eqs. (1) and (2), we have
GM L = m GMr
v=
r This gives the magnitude of angular momentum.
The direction of angular momentum is perpendicu-
where r = R + h; R being the radius of the earth and
lar to the plane of the orbit.
h the height of the space-ship above the surface
The correct choices are (a) and (d).
of the earth. For a space-ship close to the earth s
17. Consider a point P at a distance x from mass m1; its
surface r = R. Therefore
distance from mass m2 is (r – x). The net gravita-
GM GM P will be zero if
v= = Rg g
R R2 Gm1 Gm2
The space-ship will overcome the gravitational pull 2 =
x (r x)2
of the earth if it is given an additional v0 such that or
v + v0 = ve where ve is the escape velocity which is
m1 m2
given by =
x (r x)
2GM
ve = = 2Rg
R r m1
which gives x= (1)
The required addition velocity is m1 m2
v0 = ve – v = 2Rg – Rg r m2
(r – x) = (2)
= m1 m2
Rg 2 1
The gravitational potential at point P is given by
The correct choices are (a) and (d).
Gm1 Gm2
16. At a certain instant of time let r be the radius vec- U=– (3)
tor of the satellite from the centre of its circular x r x
orbit. If the velocity of the satellite is v as shown in Using (1) and (2) in (3), we have
Fig. 9.26, its angular momentum is given by m1 m2 m1 m2
L = r (mv) U = – G m1 m2
r m1 r m2
Gravitation 9.31

G When the body reaches the earth s surface, its


=– m1 m1 m2 m2 m1 m2 velocity is v and its distance from the centre of the
r
earth is the earth s radius R. Therefore
G
=– m1 m2 2 m1m2 (4) 1 Mm
r Ef = mv2 – G
2 R
Putting m1 = m and m2 = 4m in Eqs. (1) and (4) we
From energy conservation, Ei = Ef, i.e.
1 Mm Mm
18. F =
k
, where k is a constant. mv2 – G =–G
r 2 R r
which gives
mv 2 k k 1/ 2
= v= , which is independent of r. 1 1
r r m v = R 2g
R r
2 r m
T= = 2 r T r.
v k choices are (a) and (b).
Hence the correct choices are (a) and (c). GmM GmM
25. (a) Increase in P.E. = –
( R h) R
GmM
19. Total energy E = – . Hence the correct choic- GMm
es are (a) and (b). 2r = ( h = R)
2R
20. The correct choices are (a) and (d).
21. Due to friction, the satellite will slow down. Hence 1 GM
= mgR g
its kinetic energy will decrease. Now v = R . 2 R2
Therefore, a decrease in v results in a decrease in (b) If v is the velocity of projection, then from
. Since T = 2 / , time period T will increase. The energy conservation, we have
angular momentum will remain unchanged because 1 GMm GMm
the torque due to a radial (central) force is zero. mv2 – =–
2 R R h
Hence the correct choices are (a), (b) and (c).
Using h = R, this equation gives v = gR .
GM
22. Orbital speed v = . If r increases, v will (c) Total energy of the satellite orbiting at h = R is
r
2 r r3 E1 = K.E. + P.E. =
GMm

GMm
=–
GMm
decrease. Time period T = =2 . Hence 4R
v GM 4R 2R
T will increase. Centripetal force = gravitational Total energy when the satellite is at rest on
GmM the surface of the earth is
force = . Hence if r increases, centripetal GMm
r2 GM E2 = –
force will decrease. Gravitational P.E. = – . R
r Energy required is E = E1 – E2
If r increase, P.E. becomes less negative, i.e. it
increases. Since no external torque acts, the angular GMm GMm 3 GMm 3
=– – = = mgR
momentum is conserved. Hence all the four choices 4R R 4 R 4
are correct. 2
23. The speed of the comet is greater when it is closer R g
(d) Value of g at h (= R) = g . =
to the sun than when it is farther from it. Hence R h 4
both v and r change for an elliptical orbit. Thus Hence the correct choices are (b), (c) and (d).
choice (a) is correct. The total energy is conserved. GMm
The linear momentum changes because the velocity 26. (a) P.E. of a satellite in orbit = – . There-
r
changes in magnitude as well as direction. Since
fore, P.E. of two satellites before collision is
the force is central, the angular momentum is
conserved. Hence all the four choices are correct. GMm 2GMm
(P.E.)i = 2 =–
24. Since the initial velocity of the body is zero, its total r r
energy is After the collision, they stick together. So the
Ei = – G
Mm combined mass is 2m. Therefore, P.E. just
r after collision is
9.32 Comprehensive Physics—JEE Advanced

GM 2m 2GMm the combined mass will not move in an orbit; it will


(P.E.)f = – =– fall freely under the gravity of the earth.
r r
Hence the correct choices are (b) and (c).
Hence the ratio of P.E. before and after collision is 27. The distance of each cavity from the centre O is
1 : 1. the same. Since the two cavities are symmetrical
(b) Total energy of a satellite in orbit is with respect to the centre O and the mass of the
GMm GMm GMm sphere can be regarded as being concentrated at the
K.E. + P.E. = + =–
2r r 2r centre O, the gravitational force due to the sphere is
zero at the centre. Hence choice (a) is correct. For the
Therefore, the total energy of the two satellites be-
same reason, the gravitational potential is the same
fore collision is
at all points of the circle y 2 + z 2 = 36 whose radius
GMm GMm
Ei = 2 =– is 6 units and at all points of the circle y2 + z2 = 4.
2r r Hence choices (c) and (d) are also correct. But the
Let V be the velocity of the combined mass. Final gravitational force at point B cannot be zero.
momentum = (2m)V. Since the satellites have op-
posite velocities, initial momentum = mv – mv = 0. GM
28. (i) For r > R, F = . Therefore,
r2
2 mV = 0 or V = 0. Hence the combined mass has no GM GM F1 r22
kinetic energy; it has only potential energy. Thus, F1 = and F2 = . Hence =
r12 r22 F2 r12
total energy of the system just after the collision is
So choice (b) is correct and choice (d) is wrong.
GM (2m) 2GMm
Ef = – =– GMr
r r (ii) For r < R, F = . Therefore,
R3
Ei 1
= GMr1 GMr2 F1 r
Ef 2 F1 = 3
, F2 = 3
. Hence = 1
R R F2 r2
Since the combined mass comes to rest, the centrip- So choice (a) is correct and choice (c) is
etal force disappears ( velocity V = 0). Therefore, wrong.

III

Multiple Choice Questions Based on Passage


Questions 1 to 3 are based on the following passage 2. The magnitude of angular momentum of the satel-
lite is
Passage I m
A satellite of mass m is revolving in a circular orbit of (a) m GMR (b) GMR
2
radius r around the earth of mass M. The speed of the m
satellite in its orbit is one-fourth the escape velocity from (c) GMR (d) 2m GMR
2 2
the surface of the earth.
1. The height of the satellite above the surface of the 3. If the total energy of the satellite is E, its potential
earth is (R = radius of earth) energy is
(a) 2R (b) 3R (a) – E (b) E
(c) 5R (d) 7R (c) 2E (d) – 2E
SOLUTION

GM GM ve 1 2GM
1. v = = (1) v= = (2)
r ( R h) 4 4 R
Gravitation 9.33

Equations (1) and (2) give h = 7R, which is choice (d). The correct choice is (c).
3. The correct choice is (c).
GM m
2. L = mvR = m R= GMR
8R 2 2
( h = 7 R)

Questions 4 to 8 are based on the following passage 5. A body weighs 63 N on the surface of the earth.
Passage II How much will it weigh at a height equal to half
Considering the earth as an isolated mass, a force is the radius of the earth?
experienced by a body at any distance from it. This force (a) 63 N (b) 32.5 N
is directed towards the centre of the earth and has a
(c) 28 N (d) none of these
magnitude mg, where m is the mass of the body and g is the
acceleration due to gravity. The value of the acceleration 6. Assuming the earth to be a sphere of uniform mass
due to gravity decreases with increase in the height above density, the weight of a body when it is taken to
the surface of the earth and with increase in the depth the end of a tunnel 32 km below the surface will
below the surface of the earth. Even on the surface of (radius of earth = 6400 km)
the earth, the value of g varies from place to place and (a) decrease by 0.5%
decreases with decrease in the latitude of the place. (b) decrease by 1%
4. Assuming the earth to be a sphere of uniform mass (c) increase by 0.5%
density, which of the graphs shown in Fig. 9.27
(d) increase by 1%
represents the variation g with distance r from the
7. If gp is the acceleration due to gravity at the poles
centre of the earth (R = radius of earth)
and ge that at the equator, then
(a) gp < ge (b) gp > ge
(c) gp = qe (d) ge = 0
8. If a tunnel is dug along a diameter of the earth and
a body is dropped from one end of the tunnel,
(a) it will fall and come to rest at the centre of
the earth where its weight becomes zero.
(b) it will emerge from the other end of the tunnel.
(c) it will execute simple harmonic motion about
the centre of the earth.
(d) it will accelerate till it reaches the centre and
decelerate after that eventually coming to rest
at the other end of the tunnel.

Fig. 9.27

SOLUTION
4. At a height h above the surface of the earth, R 2
R 2
5. W = W0 = 63 = 28 N
g0 R 2 R h
R
R
gh = 2
( R h) 2
d 32 199 g0
At a depth d below the surface of the earth 6. g = g0 1 = g0 1 =
R 6400 200
d
gd = g0 1 Decrease in weight = mg0 – mg
R
199 mg 0
where g0 = acceleration due to gravity at the surface = mg0 1 =
200 200
of the earth. Hence the correct choices is (d).
Hence the correct choice is (a).
9.34 Comprehensive Physics—JEE Advanced

7. Due to the rotation of the earth about its axis, 8. The correct choice is (c).
g p > g e.

Questions 9 to 12 are based on the following passage 10. The mass of Jupiter is about 319 times that of the
earth and its radius is about 11 times that of the
Passage III
earth. The ratio of the escape velocity on Jupiter to
The escape velocity on a planet or a satellite is the that on earth is
minimum velocity with which a body must be projected
from that planet so that it escapes the gravitational pull of (a) 29 (b) 29
the planet goes into outer space. We obtain the expression 1 1
for the escape velocity by equating the work required to (c) (d)
29 29
with the initial kinetic energy given to the body. The 11. If R is the radius of the earth and g the acceleration
escape velocity from a planet of mass M and radius R is due to gravity on its surface, the escape velocity
given by of a body projected from a satellite orbiting the
earth at a height h = R from the surface of the earth
2MG will be
ve = = 2gR
R (a) gR (b) 2gR
where g is the acceleration due to gravity on the surface of
the planet and G is the gravitation constant. (c) 3gR (d) 2 gR
9. Choose the only incorrect statement from the 12. A body is dropped from a height equal to half the
following. The escape velocity from a planet radius of the earth. If ve is the escape velocity on
(a) is independent of the mass of the body which earth and air resistance is neglected, it will strike
is projected. the surface of the earth with a speed
(b) is independent of the direction in which the ve v
body is projected. (a) (b) e
2 2
(c) depends on the mass and radius of the planet.
ve ve
(d) will be less than the value given by the ex- (c) (d)
3 3
2MG
pression ve = if the planet has an
atmosphere. R

SOLUTION
9. The only incorrect statement is (d). Due to atmo- where g is the acceleration due to gravity at
spheric friction, a part of the initial kinetic energy height h,
is lost as heat. Hence the actual value of the escape R 2
velocity is greater than the value obtained from the g =g
R h
given expression.
For h = R, we get ve = gR , which is choice (a).
vJ MJ RE 1 12. The correct choice is (c). Use conservation of
10. = = 319 = 29 energy, i.e.
vE ME RJ 11
Total energy at h (= R/2)
Hence the correct choice is (a).
= Total energy when the body strikes the earth
11. The escape velocity at a height h is given by
GmM 1 GmM
ve = 2g R h – = mv2 –
( R h) 2 R
Gravitation 9.35

IV

Assertion Reason Type Questions

In the following questions, Statement-1 (Assertion) is Statement-2


followed by Statement-2 (Reason). Each question has the The gain in the kinetic energy of each body equals
following four choices out of which only one choice is the loss in its gravitational potential energy.
correct. 4. Statement-1
(a) Statement-1 is true, Statement-2 is true and State-
An astronaut inside a massive space-ship orbiting
ment-2 is the correct explanation for Statement-1.
(b) Statement-1 is true, Statement-2 is true but State-
gravitational force.
ment-2 is not the correct explanation for State-
Statement-2
ment-1.
(c) Statement-1 is true, Statement-2 is false. The centripetal force necessary to keep the space-
(d) Statement-1 is false, Statement-2 is true. ship in orbit around the earth is provided by the
1. Statement-1 gravitational force between the earth and the space-
ship.
A body is projected up with a velocity equal to half
the escape velocity from the surface of the earth. If 5. Statement-1
R is the radius of the earth and atmospheric resis- The escape velocity varies with altitude and lati-
tance is neglected, it will attain a height h = R/3. tude of the place from where it is projected.
Statement-2 Statement-2
The gravitational potential is – GM/R on the surface The escape velocity depends on the gravitational
of the earth and it increases with height; M being potential at the point of projection.
the mass of the earth. 6. Statement-1
2. Statement-1 A comet orbits the sun in a highly elliptical orbit.
The total energy (kinetic + potential) of a satellite Its potential energy and kinetic energy both change
moving in a circular orbit around the earth is half over the orbit but the total energy remains constant
its potential energy. throughout the orbit.
Statement-2 Statement-2
The gravitational force obeys the inverse square For a comet, only the angular momentum remains
law of distance. constant throughout the orbit.
3. Statement-1 7. Statement-1
Two bodies of masses m1 = m and m2 = 3m are ini- The acceleration due to gravity decreases due to
rotation of the earth.
allowed to move towards each other under mutual Statement-2
gravitational attraction. Their relative velocity of A body on the surface of the earth also rotates with
approach when they are at a separation it in a circular path. A body in a rotating (non-in-
2Gm ertial) frame experiences an outward centrifugal
r is v = force against the inward force of gravity.
r
SOLUTIONS

2Gm
1. The correct choice is (b). Use ve = and gravitational force. Since the gravitational force
R obeys the inverse square law of distance, the orbital
total energy at r (= R + h) = total initial energy, i.e. velocity of the satellite is given by
GmM 1 GmM
– = mv2 – GM
r 2 R v=
2. The correct choice is (a). The centripetal force r
needed for circular motion is provided by the where M = mass of earth and r = orbital radius.
9.36 Comprehensive Physics—JEE Advanced

Therefore
1 Gm1m2 2Gm2
1 GmM m1v12 or v1
Kinetic energy = mv2 = 2 r r
2 2r
1 Gm1m2 2Gm1
where m = mass of the satellite. From the inverse and m1v22 or v2
2 r r
the satellite is given by Therefore, their relative velocity of approach is

Potential energy = –
GmM 2Gm2 2Gm1
r v = v1 + v2 = +
r r
Total energy E = K.E. + P.E.
2G
=
GmM
+
GmM = (m2 m1 )
2r r r
2Gm
Putting m1 = m and m2 = 3m, we get v = 2
GmM P.E.
=– = r
2r 2 4. The correct choice is (b). Because the centripetal
3. The correct choice is (d). Initially when the two force equals the gravitational force exerted by the
masses are at an distance from each other, their earth on the space-ship, the astronaut does not
gravitational potential energy is zero. When they experience any gravitational force of the earth. The
are at a distance r from each other the gravitational only force of gravity that an astronaut in an orbiting
P.E. is space-ship experiences is that which is due to the
Gm1m2 gravitational force exerted by the space-ship. Since
P.E = –
r
but very small.
The minus sign indicates that there is a decrease
5. The correct choice is (a). The gravitational poten-
in P.E. This gives rise to an increase in kinetic
tial at a point varies with the altitude and latitude
energy. If v1 and v2 are their respective velocities
of the place.
when they are at a distance r apart, then, from the
6. The correct choice is (c).
law of conservation of energy, we have
7. The correct choice is (a).
10
Chapter
Elasticity

REVIEW OF BASIC CONCEPTS in it. Thus the ratio of stress to strain is a constant. This
constant is called the modulus of elasticity. Thus
10.1 ELASTICITY stress
Modulus of elasticity =
The ability of a body to regain its original shape and strain
size when the deforming force is withdrawn, is known as Since strain has no unit, the unit of the modulus of elasticity
elasticity. is the same as that of stress, namely, Nm–2.

10.2 STRESS 10.5 YOUNG’S MODULUS


When a deforming force is applied to a body, it reacts to the Suppose that a rod of length L and a uniform cross-
applied force by developing a reaction (or restoring) force sectional area A is subjected to a longitudinal pull. In
which, from Newton’s third law, is equal in magnitude and other words, two equal and opposite forces are applied
opposite in direction to the applied force. The reaction at its ends.
force per unit area of the body which is called into play F
due to the action of the applied force is called stress. Stress =
A
Stress is measured in units of force per unit area, i.e. The stress in the present case is called linear stress,
Nm–2. Thus tensile stress, or extensional stress. If the direction of
F
Stress = the force is reversed so that L is negative, we speak
A of compressional strain and compressional stress. If the
where F is the applied force and A is the area over which
elastic limit is not exceeded, then from Hooke’s law
it acts.
Stress strain
10.3 STRAIN or Stress = Y strain
When a deforming force is applied to a body, it may stress F L
or Y = (1)
strain A L
ratio of the change in size or shape to the original size or where Y, the constant of proportionality, is called the
shape of the body. Strain is a number; it has no units or Young’s modulus of the material of the rod and may be
dimensions. ratio of the linear stress to linear strain,
The ratio of the change in length to the original length provided the elastic limit is not exceeded. Since strain has
is called longitudinal strain. The ratio of the change in no unit, the unit of Y is Nm–2.
volume to the original volume is called volume strain. The
strain resulting from a change in shape is called shearing 10.6 BULK MODULUS
strain.
Solids, liquids and gases can be deformed by subjecting
10.4 HOOKES’ LAW them to a uniform normal pressure P in all directions.
Hookes’ law states that, within the elastic limit, the stress Stress and pressure have the same dimension (force
developed in a body is proportional to the strain produced per unit area), but pressure is not the same thing as
10.2 Comprehensive Physics—JEE Advanced

stress. Pressure is the force per unit area acting on the surface Suppose the lower face PQRS
of a system, the force being everywhere perpendicular to F is applied parallel to the upper face MNUV. As a result
the surface so that, for a uniform pressure, the force per of this, the lines joining the two faces turn through an
unit area is the same. angle . We say that the face MNRS is sheared through an
Pressure is a particular kind of stress which changes angle (measured in radians). The angle is called the
only the volume of the substance and not its shape. The shear strain or the angle of shear and is a measure of the
substance may be a solid, liquid or gas. A small increase degree of deformation.
in pressure P applied to a substance decreases its If A is the area of the face MNUV, the ratio F/A is the
volume from, say, V to V – V so that V is the small shearing stress. It is found that for small deformation, the
decrease in volume. The volume strain is given by shearing stress is proportional to the shear strain, i.e.
V
Volume strain = – F
,
F
= ,n=
F
V A A A
pressure and the corresponding volume strain, i.e. The quantity n is called the shear modulus or the
modulus of rigidity. Referring to Fig. 10.1, if is small,
P PV
B= (2) L
V V tan =
V L
If P is positive, V will be negative and vice versa. The so that =
F L
B ensures A L
that B is always positive. The SI unit of B is Nm–2. The This equation looks similar to Eq. (1) for Young’s
reciprocal of B is known as compressibility. modulus with the difference that F/A here is the tangential
The bulk modulus of a gas depends on the pressure. stress and not longitudinal stress.
Under isothermal conditions (i.e. when the temperature
is kept constant), the bulk modulus of a gas is equal to 10.8 POISSON’S RATIO
its pressure P. Under adiabatic conditions (i.e. when heat
When a wire is stretched with a force, apart from an increase
is not allowed to leave or enter the system), the bulk
in its length, there is a slight decrease in its diameter, i.e.
modulus is equal to P, where (= Cp/Cv) is the ratio of
both shape and volume change under longitudinal stress.
the molar heat capacities of the gas at constant pressure
The ratio of the decrease D in diameter to the original
and constant volume. Thus
diameter D is called lateral strain, i.e. strain at right
Isothermal bulk modulus = P
angles to the deforming force. Thus
Adiabatic bulk modulus = P.
change in diameter D
Lateral strain =
SHEAR MODULUS OR MODULUS OF original diameter D
10.7 RIGIDITY change in length L
Also Longitudinal strain =
Shear is a particular kind of stress which only solids can original length L
withstand. The solid is deformed by changing its shape The ratio of the two is called Poisson’s ratio and is
without changing its size. The body does not move or denoted by . Hence,
rotate as a whole: there is a relative displacement of its Lateral strain D/ D
contiguous layers. =
Longitudinal strain L/ L
Consider a solid in the form of a rectangular cube as in
Fig. 10.1. =
D L
D L
Since it is a ratio between two types of strain, is
dimensionless. Theoretically, one can show that it
must be less than 0.5. For most solids it lies between
1/4 and 1/3, and for rubber it is very nearly 0.5.

ENERGY STORED IN A STRAINED WIRE:


10.9 STRAIN ENERGY
If a wire is stretched, the energy stored per unit volume
Fig. 10.1 is given by
Elasticity 10.3

1 10.1
U = stress strain
2 A steel wire (original length = 2 m, diameter = 1mm)
1 and a copper wire (original length = 1m, diameter
= S = 2 mm) are loaded as shown in Fig. 10.2. Find the
2
ratio of extension of steel wire to that of copper wire.
where S = stress, = strain Young’s modulus of steel = 2 1011 Nm–2 and that of
S copper = 1 1011 Nm–2.
Since Y =

1 1 2 1 S2
U = S = Y =
2 2 2 Y

10.10 THERMAL STRESSES

then due to expansion or contraction, tensile or compressive


stress is set up in the rod. These stresses are called thermal Fig. 10.2
stresses. If a rod of length L is free to expand or contract
and its temperature is changed by T, the change in its SOLUTION
length is given by FL
Y=
A L
L = L T
FL 4F L
where L= =
YA Y d2
Now from Eq. (1), we have
For steel wire Fs = (6 + 4) 9.8 = 98 N
FL
L = For copper wire Fe = 6 9.8 = 58.8 N
AY
4 Fs Ls 4 Fc Lc
F = AY T ( L)s = 2
and ( L)c =
Ys d s Yc d c2
Thus, the thermal stress in the rod is
2
Ls F Ls Yc dc
F
= Y T = s
A Lc Fc Lc Ys ds
98 2 1 2
= 2
its volume cannot change, then a change in temperature 58.8 1 2
results in a change in pressure. The thermal stress is then
20
given by =
P = B T 3
where B is its 10.2
When the pressure on all sides of a metal cube is in-
10.11 TORQUE creased by 107 Pa, its volume decreases by 0.015% .
Find the bulk modulus of the metal.
If a rod or wire of length l and radius r
and the other end is twisted through an angle (in radian) SOLUTION
by applying a torque, the restoring torque is given by V 0.015
P = 107 Pa, =
r4 V 100
=
2l
P 107
B= =
Where is the shear modulus of the material of the rod V /V 0.015 /100
or wire.
= 6.67 1010 Pa or Nm–2
10.4 Comprehensive Physics—JEE Advanced

10.3 10.5
A rubber cube is subjected to a pressure of 106 Pa on In this example, Statement-I is followed by State-
all sides. As a result, each side of the cube decreases ment-II. Choose
by 1%. Calculate the bulk modulus of rubber. (a) if statement-II is true and statement-II is true
and statement-II is the correct explanation for
SOLUTION statement-I.
Volume of cube is V = L3. Therefore (b) if statement-I is true and statement-II is true but
V 3 L 3 statement-II is not the correct explanation for
= =3 – 1% = – 3% = statement-I.
V L 100
(c) if statement-I is true and statement-II is false.
P 106 (d) if statement-I is false and statement-II is true.
B= = = 3.33 107 Nm–2
V/V 3/100 (i) Statement-I: A wire of
length L extends by an
10.4 amount L when a block
The base of a rubber cube of side 3.0 cm is clamped. A of weight W is hung from
horizontal force F of 2.7 N is applied on the top face. it as shown in Fig. 10.4(a).
Calculate (a) the angle of shear and (b) the horizon- If the same wire goes over
tal displacement of the top face of the eraser. Shear a frictionless pulley and
modulus of rubber = 1.5 105 Nm–2. two blocks each of weight
W are hung at its ends as
SOLUTION shown in Fig 10.4(b), the
wire will extend by 2 L,
Refer to Fig. 10.1 and also to Fig. 10.3. if the elastic limit is not Fig. 10.4
exceeded.
Statement-II: For a given load, the extension is
proportional to the length of the wire.
(ii) Statement-I: Steel is more elastic than rubber.
Statement-II: For a given deforming force, a
steel wire is extended by an amount less than
rubber band of the same radius and the same
length.
(iii) Statement-I: Figure 10.5 shows the stress-strain
Fig. 10.3
curves for two materials 1 and 2. It follows from
Area of top face (A) = 3 cm 3 cm the graphs that material 1 has higher Young’s
modulus then material 2.
= 9 cm2 = 9 10–4 m2
Statement-II: Within elastic limit, the slope
F 2.7 of the stress-strain graph is greater for material
Shearing Stress = = 3 103 Nm–2
A 9 10 4 1 than for material 2.
shearing stress
(a) Angle of Shear =
shear modulus

3 103
= 5
= 2.0 10–2 rad
1.5 10
x
(b) tan = x = L tan = L ( is small)
L
= 3 cm 2.0 10–2
= 6 10–2 cm
= 0.6 mm
Fig. 10.5
Elasticity 10.5

(iv) Statement-I: It follows from Fig. 10.5 than mate-


r2 2
rial 1 is more brittle than material 2. h
= (r1 r22 )
2L
Statement-II: The plastic range is more for mate-
rial 1 than for material 2. r22
= s
1 ( r1 = r)
(v) Statement-I: A hollow shaft is stronger than a r2
solid shaft both made of the same material and i.e. > s. So the correct choice is (a).
the same external dimensions. h

Statement-II: To produce a given twist, a greater 10.6


torque is needed for the hollow shaft than for the A heavy and thick rubber rope is hung frame support.
solid shaft. It is stretched by its own weight. The strain in the
rope is
SOLUTIONS
(a) the maximum near the support.
(i) Statement-II is true. Since the length of the wire (b) the maximum near the free end of the rope
and tension in wire in cases (a) and (b) shown in
(c) the maximum at the centre of the rope
Fig. 10.4 are the same, the extension in the wire
will be the same equal to L in both case. So (d) the same everywhere.
Statement-I is false. The correct choice is (d)
SOLUTION
FL FL
(ii) Ys > Yr, i.e. < or ( L)s < ( L)r If M is mass of the rope and L its length,
AYs AYr
then the tension at a point P at a distance
So Statement-II is true. This means than steel
y from the free end is (see Fig. 10.6)
develops less strain that rubber and hence steel,
can withstand greater stress than rubber. So State- Tp =
Mg
y
ment-I is also true. The correct choice is (a). L
(iii) Young’s modulus = slope of the straight portion Thus the tension is maximum at A and
of the stress – strain graph. Hence Young’s mod- zero at B. Now
ulus of material 1 is greater than that of material L T
Fig. 10.6
2. So the correct choice is (a). Strain =
L AY
(iv) The plastic range AB of material 2 is less than
Hence the strain is maximum near A and minimum
that of material 1. So the breaking strain of material
near B. So the correct choice is (a).
2 is less than that of material 1. Hence material
2 is more brittle than material 1. So, Statement-I
10.7
is false and Statement-I is true.
(v) The torque per unit angular twist for a solid shaft A metal rod of length 1.0 m and cross-sectional area
is 1.0 mm2
supports. The tension in the rod is zero when the
r4 temperature is 40°C. Find the tension in the rod when
s
= (1)
2L the temperature falls to 20°C. Young’s modulus of
If r1 and r2 are the external and internal radii metal = 2.0 1011 Nm–2 -
of the hollow shaft, the torque per unit angular pansion = 1.0 10–5 K–1.
twist is
SOLUTION
(r14 r24 )
h
= Thermal stress = Y T
2L
T
= Y T
= (r12 r22 )(r12 r22 ) (2) A
2L
T =A Y T
Since the volumes are the same, (r12 r22 ) L
= r2L. = (1.0 10–5) (1.0 10–4)
(2.0 1011) (40 – 20)
Using this eq. (2), we get
= 40 N
10.6 Comprehensive Physics—JEE Advanced

10.8 TL 18.87 2.0


Now L= 4
A bob of mass 1 kg is suspended by a rubber cord
AY (1.0 10 ) (5 108 )
2.0 m long and of cross-sectional area 1.0 cm2 . It is = 7.5 10–4 m = 0.75 mm
revolved in a horizontal circle of radius 1.0 m at a
speed of 4 ms–1. If Young’s modulus of rubber is 5
108 Nm–2, calculate the extension of the cord. Take 10.9
g = 10 ms–2.
A wire of length L is suspended from a support. Its
length becomes L1 when its is loaded with a block
of mass m1 and L2 when it is loaded with a block of
SOLUTION
mass m2.
Refer to Fig. 10.7. Let T be the tension in the cord. Then
1
(a) L L1 L2 (b) L ( L1 L2 )
2

L1m2 L2 m1 L1m2 L2 m1
(c) L (d) L
(m1 m2 ) (m2 m1 )
v
SOLUTION
mgL
L=
AY

Fig. 10.7 m1 gL
L1 =
AY
Figure 10.7(b) shows the free body diagram of the
bob. From the diagram, it follows that m1 gL
L1 – L = (i)
AY
mv 2
T sin = (centripetal force)
r m2 gL
and L2 =
and T cos = mg AY
Squaring and adding these equations, we get
m2 gL
2
L2 – L = (ii)
mv 2 AY
2
T= (mg )
r Dividing (i) by (ii)
2 L1 L m1 L1m2 L2 m1
1.0 42 = L
= (1.0 10) 2 L2 L m2 m2 m1
1.0
= 18.87 N So the correct choice is (d).

Multiple Choice Questions with Only One Choice Correct


1. The following four wires are made of the same (b) Length = 100 cm, diameter = 1 mm
material. Which of these will have the largest (c) Length = 200 cm, diameter = 2 mm
extension when the same tension is applied? (d) Length = 300 cm, diameter = 3 mm
(a) Length = 50 cm, diameter = 0.5 mm IIT, 1981
Elasticity 10.7

2. When a wire of length L is stretched with a tension


(a) 3 : 2 (b) 3 : 2
F, it extends by l. If the elastic limit is not exceeded,
the amount of energy stored in the wire is (c) 3 3 : 2 2 (d) 9 : 4
1 9. A wire of length L and cross-sectional area A is made
(a) F l (b) Fl of a material of Young’s modulus Y. The work one
2
in stretching the wire by an amount x is given by
Fl 2 Fl 2
(c) (d) YA x 2 YA x 2
L 2L (a) (b)
L 2L
IIT, 1990 YA L2 YA L2
3. A wire, suspended from one end, is stretched by (c) (d)
x 2x
attaching a mass of 2 kg to the lower end. The wire
stretches by 1 mm. The ratio of the energy gained IIT, 1987
by the wire to the gravitational energy lost by the 10. A solid sphere of radius R and made of a material of
mass in dropping a distance of 1 mm is (Take g = bulk modulus K is completely immersed in a liquid
10 ms–2) in a cylindrical container. A massless piston of area
(a) 1 (b) 1/2 A
(c) 1/3 (d) 1/4 M is placed on the piston to compress the liquid, the
4. When the pressure on a metal cube is increased by fractional change in the radius of the sphere, R/R
107 Pa, its volume decreases by 0.015%. The bulk is given by
modulus of the metal (in Nm–2) is Mg Mg
(a) 1.5 1010 (b) 3.33 1010 (a) (b)
10 KA 2K A
(c) 6.67 10 (d) 7.5 1010
Mg Mg
5. A rubber cube is subjected to a pressure of 106 Pa (c) (d)
on all sides. As a result, each side of the cube 3K A 4K A
decreases by 1%. The bulk modulus of rubber is 11. A steel wire of cross-sectional area 3 10–6 m2
(in Nm–2) can withstand a maximum strain of 10–3. Young’s
(a) 1 108 (b) 3 107 modulus of steel is 2 1011 Nm–2. The maximum
7
(c) 3.33 10 (d) 6.67 107 mass the wire can hold is (Take g = 10 ms–2)
6. Two wires A and B made of the same material have
(a) 40 kg (b) 60 kg
their lengths in the ratio 1 : 2 and their diameters in
the ratio 2 : 1. If they are stretched with the same (c) 80 kg (d) 100 kg
force, the ratio of the increase in the length of A to 12. A rubber eraser 3 cm 1 cm 8 cm is clamped at
that of B is one end with the 8 cm edge vertical. A horizontal
1 force of 2.4 N is applied at the free end (the top
(a) 1 (b)
2 face). If the shear modulus of rubber is 1.6 105
1 1 Nm–2, the horizontal displacement of the top face
(c) (d) will be
4 8
7. A copper wire of length 1.0 m and a steel wire of (a) 1 mm (b) 2 mm
length 0.5 m having equal cross-sectional areas are (c) 3 mm (d) 4 mm
joined end to end. The composite wire is stretched 13. The density of water at the surface of the ocean is .
by a certain load which stretches the copper wire by If the bulk modulus of water is B, what is the den-
1 mm. If the Young’s modulii of copper and steel sity of ocean water at a depth where the pressure is
are 1.0 1011 Nm–2 and 2.0 1011 Nm–2, the total nP0, where P0 is the atmospheric pressure?
extension of the composite wire is B B
(a) 1.25 mm (b) 1.50 mm (a) (b)
B n 1 P0 B n 1 P0
(c) 1.75 mm (d) 2.0 mm
B B
8. Two springs of equal lengths and equal cross- (c) (d)
B nP0 B nP0
sectional areas are made of materials whose Young’s
modulii are in the ratio of 3 : 2. They are suspended 14. One end of a uniform rod of mass M and cross-
and loaded with the same mass. When stretched sectional area A is suspended from rigid support
and released, they will oscillate with time periods and an equal mass M is suspended from the other
in the ratio of end. The stress at the mid-point of the rod will be
10.8 Comprehensive Physics—JEE Advanced

2M g 3M g 20. The Poisson’s ratio of a material is 0.4. If a force is


(a) (b) applied to a wire of this material, there is a decrease
A 2A
of cross-sectional area by 2%. The percentage
Mg increase in its length is :
(c) (d) zero
A (a) 3% (b) 2.5%
15. (c) 1% (d) 0.5%
of linear expansion 1 and 2 and Young’s modulii
21. Fig. 10.8 shows a graph of the extension ( l) of
Y1 and Y2
a wire of length 1 m suspended from the top of a
rods are heated to the same temperature. There is
roof at one end and
no bending of rods. If 1 : 2 = 2 : 3, the thermal
with a load W con-
stresses developed in the two rods will be equal
nected to the other
provided Y1 : Y2 is equal to
end. If the cross-
(a) 2 : 3 (b) 1 : 1
sectional area of the
(c) 3 : 2 (d) 4 : 9
wire is 10–6 m2, the
16. One end of a uniform wire of length L and of weight Young’s modulus of
W is attached rigidly to a point in the ceiling and a the material of the
weight W1 is suspended from its lower end. If S is wire is Fig. 10.8
the area of cross-section of the wire, the stress in
the wire at a height 3L/4 from its lower end is (a) 2 1011 N/m2 (b) 2 10–11 N/m2
W (c) 3 1012 N/m2 (d) 3 10–12 N/m2
(a) W1/S (b) W1 S
4 IIT, 2003
3W 22. A light rod of length L is suspended from a support
(c) W1 S (d) (W1 + W)/S
4 horizontally by means of two vertical wires A and
B of equal length as shown in Fig. 10.9. The cross-
17. A uniform wire (Young’s modulus 2 1011 Nm–2)
sectional area of A is half that of B and the Young’s
is subjected to a longitudinal tensile stress of
modulus of A is twice that of B. A weight W is hung
5 107 Nm–2. If the overall volume change in the
as shown. The value of x so that W produces equal
wire is 0.02%, the fractional decrease in the radius
stress in wires A and B is
of the wire is
(a) 1.5 10–4 (b) 1.0 10–4 L L
(c) 0.5 10 –4
(d) 0.25 10–4 (a) (b)
3 2
IIT, 1993
2L 3L
18. An elastic spring of unstretched length L and force (c) (d)
constant k is stretched by a small length x. It is 3 4
further stretched by another small length y. The
work done in the second stretching is
1 1
(a) ky2 (b) k (x2 + y2) A B
2 2 TA TB

1 1
(c) k(x + y)2 (d) ky(2x + y) L
2 2 C
IIT, 1994 C
19. The length of an elastic string is a metre when the x
longitudinal tension is 4 N and b metre when the
W
tension is 5 N. The length of the string (in metre)
when the longitudinal tension is 9 N is Fig. 10.9
(a) a – b (b) 5b – 4a 23. In Q. 22 above, the value of x at which W produces
a equal strain in wires A and B is
(c) 2b – (d) 4a – 3b
2 L L
(a) (b)
IIT, 1987 4 2
Elasticity 10.9

2L 4L 1
(c) (d) (c) (d) 1 + B/p
3 5 1 p/ B
24. A wire breaks when subjected to a stress S. If is
the density of the material of the wire and g, the 27. A thin uniform metallic rod of mass M and length L
acceleration due to gravity, then the length of the wire is rotated with a angular velocity in a horizontal
so that it breaks by its own weight is plane about a vertical axis passing through one of
its ends. The tension in the middle of the rod is
g
(a) gS (b) 1 1
2 2
S (a) ML (b) ML
gS S 2 4
(c) (d) 1 3
g 2 2
(c) ML (d) ML
25. A stone of mass m is attached to one end of a wire of 8 8
cross-sectional area A and Young’s modulus Y. The 28. A rubber cord of mass M, length L and cross-sec-
stone is revolved in a horizontal circle at a speed such tional area A is hung vertically from a ceiling. The
that the wire makes an angle with the vertical. The Young’s modulus of rubber is Y. If the change in
strain produced in the wire will be. the diameter of the cord due to its own weight is
neglected, the increase in its length due to its own
mg cos mg weight is
(a) (b)
AY AY cos
MgL MgL
(a) (b)
mg sin mg AY 2 AY
(c) (d)
AY AY sin 2MgL MgL
26. The density of a metel at a normal pressure is . Its (c) (d)
AY 4 AY
density when it is subjected to an excess pressure p
is . If B is the bulk modulus of the metal, the ratio 29. The volume of a wire remains unchanged when the
/ is wire is subjected to a certain tension. The Poisson’s
ratio of the material of the wire is
1 p (a) 0.25 (b) 0.3
(a) (b) 1
1 p/ B B (c) 0.4 (d) 0.5

ANSWERS
1. (a) 2. (b) 3. (b) 4. (c) 5. (b) 6. (d)
7. (a) 8. (a) 9. (b) 10. (c) 11. (b) 12. (d)
13. (a) 14. (b) 15. (c) 16. (c) 17. (d) 18. (d)
19. (b) 20. (b) 21. (a) 22. (c) 23. (b) 24. (d)
25. (b) 26. (a) 27. (d) 28. (b) 29. (d)
SOLUTIONS
FL 4 FL 1
1. Extension L= = U= (stress strain) volume of wire
AY d 2Y 2
Since the wires are made of the same material, Y is 1 F l
= AL
the same for all wires. For a given tension F, 2 A L
L
L 1
d2 = Fl
2
L 3. Energy gained by the wire is
The value of 2 is the largest for L = 50 cm and
d = 0.5 mm. d 1 1
1 U1 = Fl = mg l
2. Energy stored per unit volume = (stress strain). 2 2
2 1
If A is the cross-sectional area of the wire, the total = 2 10 (1 10–3) = 0.01 J
2
energy store in the wire is
10.10 Comprehensive Physics—JEE Advanced

Gravitational P.E. lost by the mass is = Mg is applied. Now, the time period of vertical
U2 = mgh = 2 10 (1 10–3) = 0.02 J oscillations is given by
Hence the correct choice is (b). M ML
T= 2 =2
V 0.015 k YA
4. P = 107 Pa, = = – 1.5 10–4
V 100 T1 Y2 3
= =
P T2 Y1 2
Bulk modulus B = –
V /V Hence the correct choice is (a).
9. Work done per unit volume of the wire
107 1
=– = 6.67 1010 Nm–2 = (stress strain).
1.5 10 4 2
5. V = L3. Therefore stress
Y =
V 3 L strain
= = 3 (– 1%) = – 3% stress = Y strain
V L
Work done per unit volume
P 106
B=– =– = 3.33 107 Nm–2 1 1 x 2
V /V 3% = Y (strain)2 = Y
2 2 L
FL 4 FL But, volume of the wire = A L
6. L= = 2
AY d 2Y 1 x
Work done = Y A L
Since F and Y are the same for wires A and B, 2 L
L LA dB
2
1 1 2
1 Y Ax 2
A
= = = =
L LB dA 2 2 8 2L
B
Hence the correct choice is (b).
7. When a wire of length L, cross-sectional area A 10. Pressure exerted by the piston on the liquid when
and Young’s modulus Y is stretched with a force F, a mass M is placed on the piston, P = Mg/A. This
the extension l in the wire is given by pressure is exerted by the liquid equally in all direc-
FL tions. Therefore, the surface of the sphere experienc-
l =
AY es a force P per unit area. The stress on the sphere
Since F and A are the same for the two wires, we is P = Mg/A. Now, the volume of the sphere is
have
F Lc 4 R3
For copper wire lc = V =
AYc 3
Due to stress, the change in the volume of the sphere
F Ls
For steel wire, ls = is
AYs
4 R3 4
Ls Yc V = = . 3R2 R
ls = lc 3 3
Lc Ys = 4 R2 R
0.5 m 1.0 1011 Nm 2 V 3 R
= 1 mm Volume strain =
1.0 m 2.0 1011 Nm 2 V R
= 0.25 mm
stress M g /A
Total extension = 1 + 0.25 = 1.25 mm. K= =
strain 3 R/R
Hence the correct choice is (a).
R Mg
F L or =
8. Young’s modulus Y = R 3K A
A l
Hence the correct choice is (c).
Force constant k =
F YA 11. Maximum stress = Young’s modulus maximum
l L strain
where l is the extension in the spring of original =2 1011 10–3 = 2 108 Nm–2
length L and cross-sectional area A when a force F
Elasticity 10.11

Maximum force (F) = maximum stress area V P


8 –6 V =
=2
10 3 10 = 600 N B
F 600 Volume of the same mass M of water at the
Maximum mass = = = 60 kg, which is given depth is
choice (b). g 10
V P
12. Figure 10.10 shows a rubber eraser ABCD clamped V =V– V=V–
B
at end AB and a horizontal F applied at the free face
DC resulting in a displacement DD = CC . The P V
=V 1 = (B – P)
shear angle is given by B B
DD Density of water at that depth is
tan =
AD M V V
= = =
V V V
B P
B
B B
= =
B P B n 1 P0
Hence the correct choice is (a).
14. Since the rod is uniform, half its weight can be
taken to act at its mid-point. Therefore, stress at
mid-point is
weight of suspended mass + weight of half the rod
cross-secttional area
1
Mg + Mg
2 3M g
Fig. 10.10 = = , which is choice (b).
A 2A
DD
Since is small, tan = = stress l
AD 15. Young’s modulus Y = = , where =
strain L
Area of top face = 3 cm 1 cm Now, if the temperature of a rod is increased by ,
= 3 cm2 = 3 10–4 m2 the increase in its length due to thermal expansion is
F 2.4 l = L
Shearing stress = = 4 l
A 3 10 Strain = = . Now stress is
L
=8 103 Nm–2 =Y =Y
shearing stress For the two rods, the stress is
Shear angle =
shear modulus 1 = Y1 1 and 2 = Y2 2
But 1 = 2 (given). Hence Y1 1 = Y2 2
8 103
= = 0.05 rad Y1 3
1.6 105 or = 2 = . Hence the correct choice is (c).
Y2 1 2
DD
or = 0.05 or DD = 0.05 AD 16. Since the wire is uniform, i.e. its mass per unit
AD length is constant over its entire length L, the
= 0.05 8 cm = 0.4 cm = 4 mm
total donward weight = the weight of the sus-
Hence the correct choice is (d). 3L
pended mass + weight of length of the wire or
13. Pressure at the surface of the ocean = P0, the atmos- 4
3W
pheric pressure. Pressure at a depth = nP0 (given). F = W1 + .
4
Increase in pressure ( P) = nP0 – P0
3
= (n – 1)P0 W1 W
force F 4
Stress = = =
Let V be the volume of a certain mass M of water at area S S
the surface, so that M = V. The decrease in volume Hence the correct choice is (c)
under pressure P is
10.12 Comprehensive Physics—JEE Advanced

20.
L stress 5 107
17. = 2.5 10 –4. Given
L Y 2 1011 d /d l d /d
= or (i)
V 0.02 l /l l
= 2 10–4 where d is the diameter and l is the length of the
V 100
wire. The area of cross-section is
Now, V = r2L. Therefore,
d2
V ( r 2 L) 2 r rL r2L L A = r2 =
= = 4
V r2 L r2 L
or log A = log + 2 log d
= 2 r L 4
r L Differentiating, we have
2 r V L
or = = 2 10–4 – 2.5 10–4 A
=2
d
r V L A d
= – 0.5 10–4 d 1 A 1
r or = × 2% = 1%
or = – 0.25 10–4 d 2 A 2
r
Fractional decrease in r = 0.25 10–4 which is A
( = 2%, given)
choice (d). A
18. Potential energy stored in the spring when it is ex- Using this in (i) we have
tended by x is l 1%
1 = = 2.5% ( = 0.4)
U1 = kx2 l 0.4
2 Hence the correct choice is (b).
Potential energy stored in the spring when it is fur- force W l
ther extended by y is 21. Stress = . Strain = . Young’s modu-
area A l
1
U2 = k(x + y)2 lus is given by
2
stress W/A W l
1 1 Y= = (i)
Work done = U2 – U1 = k(x + y)2 – kx2 strain l /l l A
2 2
l 4 10 4
1 Now, slope of graph is = m/N. Using
= ky(2x + y) W 80
2 this in (i), we get (given l = 1 m and A = 10–6 m2)
19. If L is the initial length, then the increase in length
by a tension F is given by 80 1
Y= 4
= 2 1011 N/m2
FL 4 10 10 6
l= 22. Let TA and TB be the tensions in wires A and B
r 2Y
respectively. If aA and aB are the respective cross-
4L
Hence a= L + l = L = L + 4c (1) sectional areas, then
r 2Y
TA
9L Stress in wire A =
and b= L + = L + 5c (2) aA
r 2Y
TB
Stress in wire B =
L aB
where c = . Solving (1) and (2) for L and
r 2Y The stress in wires A and B will be equal if
c, we get L = 5a – 4b and c = b – a. For F =
= B or A = A = 1 (given).
TA T T a
9N, we have aA aB TB aB 2
9L Since the system is in equilibruim, the moments
x= L + = L + 9c
r 2Y of forces TA and TB about C will be equal (see
= (5a – 4b) + 9(b – a) = 5b – 4a Fig. 10.3), i.e.
Hence the correct choice is (b). TA x = TB (L – x)
Elasticity 10.13

TA 1 L x 26. Let V be the volume of the metal at normal pressure.


or = L x or = p
TB x 2 x From B = – , the decrease in the volume when
V/V
2L the metal is subjected to an excess pressure p is given
which gives x = . Hence the correct choice
is (c). 3 by
pV
Stress T | V| =
23. Strain = , = B
Young s modulus aY pV p
New volume of metal is V = V – =V 1
Strain in wires A and B will be equal if B B
TA T Now mass of the metal is m = V. Therefore, its new
= B
a AYA aBYB density is
m V
TA Y aA 1 = = =
or = A =2 =1 V p p
TB YB aB 2 V 1 1
B B
which gives TA = TB. Equating moments about C, we 1
have or = , which is choice (a).
1 p/ B
TA x = TB (L – x)
L 27. Let m be the mass per unit length of the rod. Then
Which gives x = L – x or x = ( TA = TB). Hence M = mL.
the correct choice is (b). 2 Consider a small element of length dx located at C
24. Let L be the required length of wire. If A is its area at a distance x from A. (Fig. 10.12)
of cross-section and is the density of its material,
the weight of the wire is
W = mg = Al g
weight AL g
Stress = = =L g
area A
or S = L g L = S/ g
25. Refer to Fig. 10.11. For vertical equilibruim
T cos = mg
mg
or T=
cos Fig. 10.12
If L is the original length of the wire, the increase in
The mass of element of length dx = mdx. The
length is
centripetal force at C is
TL
l= dF = (mdx) x 2 (1)
AY
The force (or tension) at point C is due to the outer
Strain =
l
=
T
=
mg portion CB of the rod, i.e. portion from x = x to
L AY AY cos x = L. Therefore tension at point C is
x L
2
F= (mdx) x
x x
x L
2 1 2
= m xdx = m (L2 – x2)
x x
2
M
Now, m = . Therefore,
L
1M 2 2
F= (L – x2)
2 L
1 2 x2
Fig. 10.11 = ML 1
2 L2
10.14 Comprehensive Physics—JEE Advanced

If r is the radius of the rod and its density, the To obtain the total elongation l of the cord, we
mass of the rod M = r2L . Therefore, integrate from y = 0 to y = L. Thus
L
1 x2 l = dl =
Mg
(L – y) dy
F = r2 L2 2 1 2 (2)
2 L LAY 0
Thus the tension in the rod varies with x, it is zero L
at x = L, i.e. at point B and maximum at x = 0, i.e. Mg y2
= Ly
at A where the rod is pivoted. LAY 2 0
L
Tension in the middle x of the rod is Mg 2 L2 MgL
2 = L
LAY 2 2 AY
1 1
F = r2 L2 2 1
2 4 The correct choice is (b).
3 29. Poisson’s ratio
= r2 L2 2
8 lateralstrain r /r
Thus the correct choice is (d). = =
longitudinalstrain l /l
28. Let M be the mass of the
rubber cord, L its length, where r is the radius of the wire and l its length and
A its cross-sectional area r is the change in r and l the change in l when
(assumed constant). Let the wire is subjected to tension.
- Volume of wire before elongation is
tion dl of an element AB V 1 = r 2l
of length dy at a distance Volume of wire after elongation is
y
(Fig. 10.13). The force V2 = (r – r)2 (l + l)
due to the weight of the Given V1 = V2. Thus
cord is maximum at P(y
= 0) and zero at Q(y = r 2l = (r – r)2 (l + l)
L). Therefore, the force = [r2 – 2r( r) + ( r)2] (l + l)
acting at the lower end Fig. 10.13
B of the element is = r 2(l + l) – 2 r r (l + l)
Mg + ( r)2 (l + l)
F= (L – y) (1)
L Since r and l are very small, terms of order r
This force is responsible for the elongation of ele- l and ( r)2 and higher can be ignored. Then, we
ment AB. Now have
stress F / A Fdy r2l = r 2l + r2 l – 2 rl r
Y =
strain dl / dy Adl l r
Fdy or r l=2l r or =2
or dl = (2) l r
AY
r /r 1
Using Eq. (1) in Eq. (2), we get = = = 0.5,
l /l 2
Mg
dl = (L – y)dy which is choice (d).
LAY
Elasticity 10.15

II

Multiple Choice Questions with One or More Choices Correct


1. Figure 10.14 shows the stress–strain graphs for ma- (c) A can withstand a greater load before break-
terials A and B. ing than B
(d) A and B will withstand the same load before
breaking
5. Choose the correct statements from the following:
(a) Steel is more elastic than rubber.
(b) The stretching of a coil spring is determined
by the Young’s modulus of the wire of the
spring.
(c) The frequency of a tuning fork is determined by
the shear modulus of the material of the fork.
(d) When a material is subjected to a tensile
(stretching) stress, the restoring forces are
caused by interatomic attraction.
6. A uniform wire of Young’s modulus Y is stretched
by a force within the elastic limit. If S is the stress
Fig. 10.14 produced in the wire and is the strain in it, the po-
tential energy stored per unit volume is given by
From the graph it follows that 1 1 2
(a) material A has a higher Young’s modulus (a) S (b) Y
2 2
(b) material B is more ductile
(c) material A is more brittle S2 1
(c) (d) Y S
(d) material A can withstand a greater stress. 2Y 2
2. Two wires A and B have the same cross-section 7. A wire AB of length 2l and cross-sectional area a
and are made of the same material, but the length of A
wire A is twice that of B. Then, for a given load and B (Fig. 10.15). The wire is pulled at the centre
(a) the extension of A will be twice that of B into shape ACB such that d << l. The tension in the
(b) the extensions of A and B will be equal string is T and the strain produced in it is . If the
(c) the strain in A will be half that in B Young’s modulus of the material of wire is Y. Then
(d) the strains in A and B will be equal. d d2
3. Two wires A and B have equal lengths and are made (a) = (b) = 2
2l 2l
of the same material, but the diameter of wire A is
twice that of wire B. Then, for a given load d d2
(c) T = aY (d) T = aY
(a) the extension of B will be four times that of A 2l 2l 2
(b) the extensions of A and B will be equal
(c) the strain in B is four times that in A
(d) the strains in A and B will be equal.
4. In Q. 3, above
(a) Young’s modulus of A is twice that of B
(b) Young’s modulus is the same for both A and B Fig. 10.15

ANSWERS AND SOLUTIONS


1. The slope of the linear portion of the curve gives that of the linear portion OR for material B. Hence
the Young’s modulus of the material. The slope of statement (a) is correct. The plastic region for
the linear portion OP for material A is greater than material A (from P to Q) is greater than that (from
10.16 Comprehensive Physics—JEE Advanced

R to S ) for material B, which indicates that ma- ing load = breaking stress cross-sectional area,
terial A is more ductile. Hence statement (b) is the wire having a greater area of cross-section can
incorrect. The breaking stress for material B (i.e. withstand a greater load. Hence the correct choices
stress corresponding to point S) is less than that for are (b) and (c).
ma-terial A (i.e. stress corresponding to point Q), 5. Statement (a) is correct because the Young’s modu-
which implies that material B can break move eas- lus of steel is greater than that of rubber. Statement
ily than material A. Thus material B more brittle. (b) is incorrect. If a spring is stretched, the total
Hence choice (c) is also incorrect. Material A is length of the wire in the coil and the volume of
stronger than material B because it can withstand a the wire, both do not change. Only the shape of
greater stress before it breaks. The breaking stress the coils of the wire undergoes a change. Hence it
is the stress corresponding to point Q for material is the shear modulus that determines the stretch-
A and to point S for material B. Hence the correct ing of a coil. Statement (c) is also incorrect. The
choice are (a) and (d). bending moment of the prongs of a tuning fork is
2. We know that determined by the Young’s modulus of its material.
FL Hence the restoring force on the prongs depends on
Y= Young’s modulus, which determines the frequency
Al
of the fork. Statement (d) is correct. When the ma-
l=
FL terial is not subjected to any stress, its atoms are in
AY their normal (equilibrium) positions. When a ten-
Since the two wires are made of the same material, sile stress is applied, the separation R between the
the Young’s modulus Y is the same for both. Since atoms becomes greater than the equilibrium sepa-
load F and the cross-sectional A are the same, ration R0. For R > R0, the interatomic forces are
l L attractive.
i.e. extension is proportional to the length of the 6. The correct choices are (a), (b) and (c).
wire. Hence choice (a) is correct. The strain in a 7. AC + CB = 2 AC = 2(l2 + d2)1/2. Increase in length
wire is given by is
l F L = AC + CB – AB = 2(l2 + d2)1/2 – 2l
=
L AY T
Stress = .
Since F, A and Y are the same, the strain in wires a
A and B will be equal. Thus the correct choices are L 2(l 2 d 2 )1 / 2 2l
(a) and (d). Strain = =
L 2l
3. Area of cross-section A = d 2/4; where d is the
1/ 2
diameter of the wire. Therefore d2 d2
If d << l, (l2 + d2 )1/2 = l 1 =l 1 .
4F L l2 2l 2
l= Therefore
d 2Y d2 d2
Since F, L and Y are the same for wires A and B, 2(l2 + d2)1/2 – 2l = 2l 1 – 2l = . Thus
2l 2 l
l 1/d 2, i.e. the extension is inversely proportional
to the square of the diameter. Hence choice (a) is L d2
Strain = = 2
correct. The strain is L 2l
l 4F stress T /a
= Now Y= = 2 2
L d 2Y strain d / 2l
2
Thus, strain 1/d . Hence the correct choices are
(a) and (c). d2
or T=aY
4. Since wires A and B are made of the same material 2l 2
they have the same Young’s modulus. Now break- Hence the correct choices are (b) and (d).
Elasticity 10.17

III

Multiple Choice Questions Based on Passage


Questions 1 to 4 are based on the following passage 2. In Q.1 above, the metal shows plastic behaviour
Passage I beyond point
(a) A (b) B
Elasticity: The atoms in solids are held together by inter-
(c) C (d) D
atomic forces. The average locations of the atoms in a
3. Figure 10.17 shows the strain-stress graphs for
lattice do not change with time. Since the atoms are almost
materials A and B. From the graph it follows that
lacking in mobility, their kinetic energy is negligibly small.
(a) material A has a higher Young’s modulus than
It is this lack of mobility which makes a solid rigid. This
B.
rigidity is the cause of elasticity in solids. In some solids
(b) material B has a higher Young’s modulus than
such as steel, the atoms are bound together by larger inter-
A.
atomic forces than in solids such as aluminium. Thus, the
(c) for a given strain, material B can withstand
greater stress than A.
exhibit elasticity. All material bodies get deformed when
(d) for a given stress, the strain produced in
subjected to a suitable force. The ability of a body to regain
material B is more than that in material A.
its original shape and size is called elasticity. The deform-
ing force per unit area is called stress. The change in the
dimension (length, shape or volume) divided by the original
dimension is called strain. The three kinds of stresses are
tensile stress, shearing stress and volumetric stress. The
corresponding strains are called tensile strain, shearing
strain and volume strain. According to Hookes’ law, within
the elastic limit stress is proportional to strain. The ratio
stress/strain is called the modulus of elasticity.
1. Figure 10.16 is the load-extension curve for a
metallic wire. Over which region is Hookes’ law
obeyed?
(a) OA (b) AB Fig. 10.17
(c) BC (d) CD
4. Choose the correct statements from the following.
(a) Steel is more elastic than rubber.
(b) Fluids have Young’s modulus as well as shear
modulus.
(c) Solids have Young’s modulus, bulk modulus
as well as shear modulus.
(d) Bulk modulus of water is greater than that of
copper.
Fig. 10.16

ANSWERS

1. Hooke’s law is obeyed in the linear portion of the Stress


graph. Hence the correct choice is (a). 3. Young’s modulus = , which is, therefore, given
Strain
2. The correct choice is (b). by the reciprocal of the strain versus stress graph. The
correct choices are (b) and (c).
4. The correct choices are (a) and (c).
10.18 Comprehensive Physics—JEE Advanced

Questions 5 to 7 are based on the following passage (d) None of these


Passage II 6. The new length of the rod P is
Two rods P and Q of different metals having the same F
area A and the same length L are placed between two rigid (a) L1 = L 1 1T
AY1
expansion of P and Q are 1 and 2 respectively and their (b) L1 = L 1
F
1T
Young’s modulii are Y1 and Y2. The temperature of both AY1
rods is now raised by T degrees.
F
(c) L1 = L 1 1T
AY1
F
(d) L1 = L 1 1T
AY1
7. The new length of rod Q is
F
(a) L2 = L 1 2T
AY2
Fig. 10.18
F
5. The force exerted by one rod on the other is (b) L2 = L 1 2T
AY2
TA ( 1 2)
(a) F = F
1 1 (c) L2 = L 1 2T
AY2
Y1 Y2
(b) F = TA Y1Y2( 1 + 2) (d) L2 = L 1
F
2T
(c) F = TA(Y1 + Y2) AY2
1 2

SOLUTION
L 1 = L 1T and L 2 = L 2T 5. Since the total length cannot change, the increase in
total length due to heating = decrease in total length
L1 + L2 = LT( 1 + 2) (1)
due to the compressive force F. Equating (1) and
Since the walls are rigid, this increase in length cannot
occur. Hence a force F appears between them. The decrease 6. New length of rod P = original length + increase in
in length of the rods due to this force is length due to heating – decrease in length due to F.
FL FL Hence.
L1 = and L2 = FL
Y1 A Y2 A L 1 = L + L 1T –
AY1
FL 1 1 So the correct choice is (c).
L1 + L2 = (2)
A Y1 Y2 7. The correct choice is (c).

Questions 8 to 11 are based on the following passage 9. The tension T in the string is
Passage III mg mg
(a) (b)
One end of a string of length L and cross-sectional area cos sin
A mg
of mass m. The bob is revolved in a horizontal circle of (c) (d) m(g2 + r2 4 1/2
)
tan
radius r with an angular velocity such that the string
10. The increase L in length of the string is
makes an angle with the vertical.
8. The angular velocity is equal to TL MgL
(a) (b)
AY AY cos
g sin g cos
(a) (b)
r r MgL MgL
(c) (d)
AY sin AY
g tan g cot
(c) (d) 11. The stress in the string is
r r
Elasticity 10.19

mg mg r mg r mg r
(a) (b) 1 (c) 1 (d)
A A L A L A L

SOLUTION
Refer to Fig. 10.19. The
g tan
vertical component T cos or =
of tension T balances r
the weight mg and the Hence the correct choice is (c).
horizontal component 9. -
T sin provides the es are (a) and (d).
necessary centripetal T L stress
force. Thus 10. Stress = . Also strain =
A L Y
T cos = mg (1)
2
T sin = mr (2) L=
TL
=
mg L
8. Dividing (1) and AY cos AY
(2) Hence the correct choices are (a) and (b)
r 2 Fig. 10.19 11. The correct choice is (a).
tan =
g

Questions 12 to 14 are based on the following passage (c) 4 10–4m (d) 5 10–4m
Passage IV 13. What mass must be attached at the lower end of the
A thin rod of negligible mass and cross-sectional area rod so that the rod is prevented from contracting on
4 10–6m2, suspended vertically from one end, has a cooling?
length of 0.5 m at 100°C. The rod is cooled to 0°C. Young’s (a) 40 kg (b) 30 kg
modulus = 1011 Nm–2 (c) 20 kg (d) 10 kg
10–5 K–1 and g = 10 ms–2. 14. The total energy stored in the rod is
12. The decrease in the length of the rod on cooling is (a) 0.1 J (b) 0.2 J
(a) 2 10–4m (b) 3 10–4m (c) 0.3 J (d) 0.4 J

SOLUTION
12. The decrease in the length of the rod on cooling is YA L 1011 4 10 6 5 10 4

L =L t = 0.5 10–5 100 or m= =


gL 10 0.5
=5 10–4 m = 40 kg, which is choice (a).
So the correct choice is (d). 14. Energy stored in the rod is
13. If the rod is to be prevented from contracting, the 1 1
mass m attached at the lower end must increase its U= F L= mg L
2 2
length by an amount L = 5 10–4 m. Now
1
mgL = 40 10 5 10–4
Y = 2
A L = 0.1 J.
Thus the correct choice is (a).

Questions 15 to 17 are based on the following passage 15. The common acceleration of the blocks is
Passage V g
(a) g (b)
Two blocks of masses m and M = 2 m are connected by 3
means of a metal wire of cross-sectional area A passing
2g 3g
(c) (d)
system is then released. 3 2
10.20 Comprehensive Physics—JEE Advanced

16. The stress produced in the wire is


mg 2mg
(a) (b)
A 3A
3mg 4mg
(c) (d)
4A 3A
17. If m = 1 kg, A = 8 10–9 m2, the breaking stress =
2 109 Nm–2 and g = 10 ms–2, the maximum value
of M for which the wire will not break is
(a) 4 kg (b) 6 kg
(c) 8 kg (d) 10 kg

Fig. 10.20

SOLUTION
If a is the common acceleration of the blocks and T the T 4 mg
tension in the wire. Then the equations of motion of the Stress = = , which is choice (d).
A 3A
blocks are
Mg – T = Ma (1) 17. Breaking stress is the maximum stress the wire can
withstand. From Eqs. (1) and (2)
and T – mg = ma (2)
2 mMg
15. Adding Eq. (1) and Eq. (2), we get T =
( M m)
( M m) g 2 mg
a = Breaking stress =
T
= .
( M m) A m
A1
For M = 2 m, we get a = g/3, which is choice (b). M max
16. From Eqs. (1) and (2), T = m(g + a) = m(g + g/3) Using the given values, we get Mmax = 4 kg, which
= 4 mg/3. is choice (a).

IV

Assertion-Reason Type Questions


In the following questions, Statement-1 (Assertion) is tension is 5 N. The length of the string (in metre)
followed by Statement-2 (Reason). Each question has the when the tension is 9 N is (a + b – L)
following four choices out of which only one choice is Statement-2
correct. The extension of an elastic string is proportional to
(a) Statement-1 is true, Statement-2 is true and State- the initial length of the string.
ment-2 is the correct explanation for Statement-1. 2. Statement-1
(b) Statement-1 is true, Statement-2 is true but Steel is more elastic than rubber.
Statement-2 is not the correct explanation for Statement-2
Statement-1. The Young’s modulus of steel is greater than that of
rubber.
(c) Statement-1 is true, Statement-2 is false. 3. Statement-1
(d) Statement-1 is false, Statement-2 is true. The stretching of an elastic spring is determined by
1. Statement-1 the shear modulus of the material of the spring.
The length of an elastic string of initial length L is a Statement-2
metre when the tension is 4 N and b metre when the For a given stretching force, the amount of stretching
depends on the force constant of the spring.
Elasticity 10.21

4. Statement-1 7. Statement-1
Figure 10.21 shows that stress-strain curves for two Two wires A and B have equal lengths and are made
different types of rubber. Rubber A rather than rub- of the same material but the diameter of wire A is
ber B should be used as a car tyre. twice that of B. For a given load, the extension of
B will be four times that of A.
Statement-2
For a given load, the extension of a wire is inversely
proportional to its area of cross-section.
8. Statement-1
Two wires A and B are made of the same material.
The length of wire A is twice that of B but the di-
ameter of A is half that of B. For a given load, the
strain produced in B will be twice that in A.
Fig. 10.21
Statement-2
Statement-2 For a given load, the extension produced in a wire
Rubber A dissipates larger amount heat energy than is directly proportional to its length and inversely
rubber B. proportional to the area of cross-section.
5. Statement-1 9. Statement-1
Two wires A and B have the same cross-sectional When a material is subjected to a tensile (stretch-
area and are made of the same material but the ing) stress, the restoring forces are caused by inter-
length of wire A is twice that of B. For a given atomic attraction.
load, the extension of A will be twice that of B.
Statement-2
Statement-2 Restoring force is called into play in an elastic ma-
For a given load the extension of a wire is propor- terial by an inherent property of the material and
tional to its length. not due to interatomic attraction.
6. Statement-1 10. Statement-1
Two wires A and B have the same cross-sectional When a material is subjected to a compressional
area and are made of the same material but the stress, the restoring forces are caused by interatomic
length of wire A is the twice that B. For a given repulsion.
load, the strain in wire A is twice that in B.
Statement-2
Statement-2 The atoms of a material never repel.
For a given load, the extension in a wire is propor-
tional to its length.

ANSWERS
1. The correct choice is (d). If L is the initial length, 9L
then the increase in length by a tension F is given x =L+ = L + 9c
r 2Y
by
FL = (5a – 4b) + 9(b – a) = 5b – 4a
l=
r 2Y 2. The correct choice is (a).
4L 3. The correct choice is (b). If a spring is stretched, the
Hence a=L+l=L+ = L + 4c (1) total length of the wire of the coil and the volume
r 2Y of the wire, both do not change. Only the shape (or
5L
and b=L+ = L + 5c (2)
r 2Y change. Hence the stretching of a spring is not de-
termined by Young’s modulus or bulk modulus. It is
L determined by the shear modulus.
where c = . Solving (1) and (2) for L and c,
r 2Y 4. The correct choice is (c). The area of the hysteresis
we get L = 5a – 4b and c = b – a. For F = 9 N, we loop for rubber A much smaller than that for rubber
have B. This implies that rubber A dissipates a smaller
10.22 Comprehensive Physics—JEE Advanced

amount of heat energy than rubber B. Consequent- Hence the correct choice is (a).
ly, tyres made of rubber A will not get heated to a L 4F 1
high temperature. This prevents wear and tear of 8. Strain = 2
. Thus strain .
L d Y d2
tyres.
Hence strain in B will be four times that in A. Thus
FL
5. L= . Since the two wires are made of the the correct choice is (d).
AY 9. The correct choice is (c). When the material is not
same material, the Young’s modulus Y is the same.
subjected to any stress, its atoms are in their nor-
Since F and A also the same, L L. Hence the
mal (equilibrium) positions. When a tensile stress
correct choice is (a).
is applied, the distance R between atoms becomes
6. The correct choice is (d).
greater than their equilibrium separation R0. For
Strain =
L
=
F R > R0, the interatomic force is attractive and this
L AY force provides the restoring force under which the
Since F, A and Y are the same for the two wires, the material regains its original shape and size when
strains in them are equal. the stress is removed.
7. Area of cross-section A = d 2/4, where d is the 10. The correct choice is (c). When the material is
diameter of the wire. Thus subjected to a compressional stress, R becomes less
4F L 1 than R0 and in this case the interatomic force is
L = 2
or L repulsive which causes the restoring force.
d Y d2

Integer Answer Type


1. A light rod AB of length 2m is suspended from the 2.
ceiling horizontally by means of two vertical wires the strains in the two wires are the same.
as shown in Fig. 10.22. One of the wires is made of IIT, 1980
steel of cross-section 0.1 cm2 and the other of brass 3. A metal wire of negligible mass, length 1 m and
of cross-section 0.2 cm2. The Young’s modulus of cross-sectional area 10–6 m2 is kept on a smooth
brass is 1.0 1011 Nm–2 and of steel is 2.0 1011
Nm–2. A weight W is hung at point C at a distance ball of mass 2 kg is attached to the other end of the
x from end A. It is found that the stress in the two wire. When the wire and the ball are rotated with
n
wires is the same when x = metre. Find the value angular velocity of 20 rad s–1, it is found that the
of n. 3 wire is elongated by 10–3 m. If the Young’s modulus
of the metal is n 1011 Nm–2 n.
IIT, 1980
IIT, 1992
4. In Q.10 above, if the angular velocity is gradually
increased to 100 rad s–1, the wire breaks. If the
breaking stress is x 1010 Nm–2 x.
IIT, 1992
5. A body of mass 3.14 kg is suspended from one end
of a wire of length 10.0 m. The radius of the wire
is changing uniformly from 9.8 10–4 m at one end
to 5.0 10–4 m at the other end. Find the change in
the length of the wire in mm. Young’s modulus of
the material of wire is 2 1011 Nm–2.
IIT, 1994
Fig. 10.22 6. Steel wire of length ‘L’ at 40°C is suspended from
the ceiling and then a mass ‘m’ is hung from its free
Elasticity 10.23

end. The wire is cooled down from 40°C to 30°C of the wire is 1 mm. Assume that L >> diameter of
to regain its original length ‘L the wire. Then the value of ‘m’ in kg is nearly.
linear thermal expansion of the steel is 10–5/°C, IIT, 2011
Young’s modulus of steel is 1011 N/m2 and radius

SOLUTIONS
1. If Ts and Tb are the tensions in steel and brass wires, Given m = 2 kg, R = 1 m, = 20 rad s–1, A = 10–6m2
then the stress in them will be the same if and L = 10–3 m. Substituting these values in (1)
Tb Ts As 0.1 cm 2 1 and solving, we get
Ts
= 2 Y = 8 1011 Nm–2
As Ab Tb Ab 0.2 cm 2
Thus n = 8.
Since the system is in equilibrium, the moments of
4. If the wire breaks at max = 100 rad s–1, then the
forces Ts and Tb about C will be equal, i.e. (see Fig.
breaking stress is
10.23)
Ts x = Tb (2 – x) Fmax mR 2 max 2 1 (100)2
= 2 1010 Nm 2 .
4 A A 10 6
x = 2(2 – x) x= metre Thus x = 2.
3
( Tb = 2Ts) 5. Figure 10.24 shows a wire PQ of length L having
Thus the value of n = 4 radius r1 at end P increasing uniformly to a radius
r2 at end Q. Consider an element AB of the wire of
length dy at a distance y from end P. Let r be the
radius of the wire at y and (r + dr) at (y + dy). As
the radius increases uniformly from P to Q.
dr
= constant, say C
dy
r2 r1
where C= . Thus
L
dr
dy =
C
Fig. 10.23

stress T
2. Strain =
Y AY
The strain in the two wires will be equal if
Ts Tb
=
AsYs AbYb
Ts A Ys 0.1 2.0 1011
= s 1
Tb Ab Yb 0.2 1.0 1011
Equating moments about C, we have
Ts x = Tb (2 – x)
which gives x = 1 metre ( Ts = Tb)
3. Let R = L = length of the wire = radius of the circle.
The centrifugal force F = mR 2 produces the stress
as at result of which the wire elongates.
FL mR 2 L mR 2 2
Thus Y= (1) Fig. 10.24
A L A L A L
10.24 Comprehensive Physics—JEE Advanced

dl produced in the r2 r1
element of length dy due to a force F = Mg. Stress in Now F = Mg and C = . Hence
L
the element = F/ r2 and strain in it = dl/dy. Hence
MgL
stress F / r2 Fdy l=
Y= Yr1r2
strain dl / dy r 2 dl
Substituting the given values of M, L, Y, r1 and r2
But dy =
dr
. Therefore, and g = 9.8 ms–2, we have
C
3.14 9.8 10.0
Fdr Fdr l=
Y= 2 or dl = 3.14 2 1011 9.8 10 4
5.0 10 4
r Cdl CYr 2
= 10–3 m = 1 mm.
Integrating from r = r1 to r = r2, we obtain the
total extension l produced in the wire by a force F 6. Change in length L = L T (i)
which is given by mgL mgL
r2
Also Y = L= (ii)
F dr A L YA
l= dl Equation (i) and (ii) we get ( A = r2)
CY r1 r2
TY r2
F 1 r2
F 1 1 m=
=– = g
CY r r1 CY r1 r2
(10 5 ) (10) (1011 ) 3.14 (1 10 3 ) 2
=
F (r2 r1 ) 9.8
=
CYr1r2 = 3.2 kg 3 kg
Hydrostatics (Fluid Pressure and Buoyancy) 11.1

11
Chapter
Hydrostatics (Fluid Pressure
and Buoyancy)

REVIEW OF BASIC CONCEPTS

11.1 FLUID PRESSURE

P0 105 Pa
F
P= 11.5 GAUGE PRESSURE
A
SI
–2

–2
P= g
g
11.2 PASCAL’S LAW

pressure
11.6 HYDR0STATIC PRESSURE

11.3 DENSITY AND RELATIVE DENSITY P0


g

–3
–3 –3 P = P0 + g

11.1
density e
density

–2
g

11.4 ATMOSPHERIC PRESSURE SOLUTION


103 –3

g
11.2 Comprehensive Physics—JEE Advanced

103) 10
4 SOLUTION
10 Pa –3
1
–3
( 2 A
2
=5 10–4 2

104)
p1 = 1g
104
p1 = P0 + 1g (P0
10 4 sure)

1 P2 = P0 + 2g
cal side = 104 104 P
2 P2 – P1 = ( 2 – 1)
2
P 2 – P 1) A = ( 2 – 1)
= (1500 – 1000) 5 10–4
104)
104

11.2 11.4
A OP L
r A O
P

O
is P0 P
L
SOLUTION
SOLUTION
F = (P0 + g) A + at
a distance r O
=
F
P= = P0 + g+
A A
11.3

Fig. 11.2

2 2
= =A
r = L/2 to r = L
L
r2
L
2 2 3 2 2
F=A r =A = A L
L/2
2 L/2
=
F 3 2 2
Pressure at P = = L
A
Fig. 11.1
Hydrostatics (Fluid Pressure and Buoyancy) 11.3

L
11.5
A
= L A
acceleration a

P1 A – P2 A= = ( L A) a
–3 P1 and P2
–3 –3

P1 – P2 = La (i)
SOLUTION
( 1 – 2) g = La
La
1 – 2 = (ii)
g
P1 > P2
1 2 a
tan = =
L g

11.6

A
°

Fig. 11.3

Pressure at A = P0 + 0 0g + g (i)
SOLUTION
Pressure at B = P0 + g (ii)
a
tan =
g

0 0 + = a
tan 30° =
0 0 +( – 0) = g

0) 1300 –2
0 a tan 30° = =
( = 3
g
11.8 ARCHIMEDES' PRINCIPLE
0

PRESSURE DIFFERENCE IN AN ‘
11.7 ACCELERATED LIQUID

Law of Floatation

11.9 APPLICATIONS

V g V
Fig. 11.4
11.4 Comprehensive Physics—JEE Advanced

Case
<
V0 outside liquid and Vi inside liquid
V
Vi g =V g
Vi V0
= and = ( V = V0 + Vi )
V V
Case =
V0
Case >

T
a liquid ( >
Case V
and 1
V1 in liquid 1 and V2 in liquid 2
T= V g–V g 2
1< < 2)
= V( – )g
Case

T =V ( – )g Fig. 11.5

a
Fig. 11.7
g = g + a and T = V ( – ) (g + a)

a (< g) V g = V1 1g + V2 2g (i)
g = g – a and T = V( – ) (g – a) Also V1 + V2 = V (ii)

a = g and T = 0
V 2 V 1
< V1 = V2 =
2 1 2 1

V2 1
and =
V1 2

T =V ( – ) g
Fig. 11.6
11.7
a
T =V ( – ) g g = (g + a)

a
T =V ( – g = (g – a)
T =0
Fig. 11.8
(a) is at rest
–2

–2
–2
Take g
Hydrostatics (Fluid Pressure and Buoyancy) 11.5

SOLUTION SOLUTION
–3
Let
–3

3
V
–4 3
10
W)
W= = Vg
U Vg
Fig. 11.10
–2 –2 2
(5 – ) 10 (5 10 ) 1000 g =
(5 10–2 )3 g

10–2 k
= =
k 10–2
U = (5 10–2 )3 1000 g
–2
Fig. 11.9 ( g )

T+U=W Fig.11.11
W+ =U + (i)
T = W – U = Vg – Vg
W = (5 10–2 )3 =k 10–2
= ( – ) Vg U
= (7000 – 1000) 10–4) 10
–2
k
g =g+a k –1

T = W – U = Vg – Vg
= ( – ) V (g + a) 11.9
R M is
= (7000 – 100) 10–4) (10 + 2)

g =g–a
T = ( – ) V (g – a)
= (7000 – 1000) 10–4) (10 – 2) SOLUTION

11.8
–3

constant k
Take g –2 Fig. 11.12
11.6 Comprehensive Physics—JEE Advanced

P1 = 1 g 2

P2 = 2 g
A
103) 103g = 2 103 g
F = ( p2 – p1) A = ( 2 – 1) Ag 2) 103 g
2
= Ag = V g
V 11.12

11.10

radius R

Fig. 11.13 SOLUTION

SOLUTION

V 103 –3

=( R2 ) g+V g
= g [V + R 2 ss
V= = = 10–3 3
density 103
11.11 V
2
V
1000 g
103 –3
103 2
–2
10 3
= 1000 g g
2

SOLUTION
10 10 = 103 3
= 10–3 3

(a) Let 1 SURFACE TENSION AND SURFACE


11.10 ENERGY

1) 103g 103)
103 g
1
Hydrostatics (Fluid Pressure and Buoyancy) 11.7

–1
–2
ABCD
PQ L AB and
CD
PBCQ
L
PQ is

Fig. 11.15

SOLUTION
F 1 5 10 2
F =2 L = =
2L 2 03
Fig. 11.14
10–2 –1
F= 2L=2 L

11.11 EXCESS PRESSURE


PQ

PQ P Q is r is
W =F = (2L )= A
2
A p=
r
–2
r is
11.13 4
p=
r
r in
3 10–5
2
p=
SOLUTION r
P inside
2
P+
2 r
=1 10–4 2
2
–4 2
= P0 + g+
A=2 10 r
5
W 3 10 –2 –1
= = 4
A 2 10
WORK DONE IN BLOWING A SOAP
11.14 11.12 BUBBLE

10–2 r1 to r2
(4 r21
=2 (4 r22 )
11.8 Comprehensive Physics—JEE Advanced

or W (r22 – r21)
SOLUTION
r is
(since r2 = r and r1 = 0)
W r2 is R = 2 r
4 r2 – 4 R2
r1 = r to r2 = 2r is = 32 r2 = 4 (2r)2
W [(2r)2 – r2 r2 = 16 r2
R into n Q= A = 16 r2
r 4 32 3
= R3 = r
3 3
R2
n 4 r2 Q= T
32
W = 4 (nr2 – R2) 16 r2 = r3 s T
3
3
4 3 4 3 T=
r n= R 2r s
3 3
or n1/3 r = R
11.17
W = 4 R2 (n1/3 –1)

11.15
–1
–3
105

–1 SOLUTION
2
P = P0 + + g
SOLUTION R
2 0 05
r=
R 105 + 3
1200
2 1 10
R2 105 Pa
2
4 r
2 11.13 ANGLE OF CONTACT
R
4 R2
2
A R2 – 4 R2 = 4 R2
A
4 10–2)2
10–5

11.16
r

Fig. 11.16
Hydrostatics (Fluid Pressure and Buoyancy) 11.9

–2

SOLUTION
2 cos 2 0 07 os 0
(a) = = 3
r g 0 5 10 103 10
–2
10

NOTE

11.14 CAPILLARITY

and density r is Fig. 11.17

2 cos = =2
= cos 60
rg
11.19

2
=
rg

SOLUTION
NOTE
1 1 r 2 1 r
1 = and 2 =
cos 1 2 cos 2

2 2 2 cos 1
11.18 =
1 1 1 cos 2

26 cos 0
–1
=
10 0 cos135
26 1
=
10 0 707

Multiple Choice Questions with Only One Choice Correct


1.
z
11.10 Comprehensive Physics—JEE Advanced

(a)
7. A and B
(c) (d)
eration a (< g
2. A
B

L
2 La Lg
(a) (a)
3 g a
–3 a a
3. (c) L 1 (d) L 1
g g

4. a and

Fig. 11.19
(a)
A a 8.

(c) (d)
A a
5.
9. k
g

(a) g (1 – k g (1 + k)
1 1
(c) g 1 (d) g 1
k k
6.
10.
A
a (< g
1 2 > 1

2 2
(a) 2 Ag ( 2 – 1) Ag ( 2 – 1)

1 2 1 2
(c) Ag ( 2 – 1) (d) Ag ( 2 – 1)
2 4
Fig. 11.18 11. r
Hydrostatics (Fluid Pressure and Buoyancy) 11.11

r 17.
(a) = =r
2
density > 1
(c) = 2r (d) = 4r
12. r
2
2/ 1 is

(a) 1 1

2 2 (c) (d)
(a) + –
r r
18. A and B
4 4 1
(c) + (d) – A
r r 4
13. is B
2
and area A 3
A and B is

A 2 A
(a) 19. 1
and 2
4 A A
(c) (d)
1
14. (a) ( 1 + 2 1 2
2
( >
1 2 2 1 2
(c) (d)
1 2 1 2
(a) g g
20. 1
and 2
(c) g (d) g
1
15. T (a) ( 1 + 2 1 + 2)
2
1 2
(c) 1 2 (d)
1 2
21. r
1/ 2 1/ 2
(a) T T
r
1/ 2 1/ 2
(c) T (d) T

16. and density is tied to a 22. r

>

(a) (a) 2
(c) (d) 2
23.
(c) (d)
11.12 Comprehensive Physics—JEE Advanced

30.

24.
2
E=
31.

25.

32.

–3
26.
–3 33. rises

–3 2
is 2
(a) 2

2
(c) (d)
27.
34.

2
2
(a) g
g
2
28. 2 2
(c) 2 g (d)

= 0 e– 35. and density


r
0= k is a con
or equal to

0g 0g
2 2
(a) (a)
2k k g
2 0g 0g 2
(c) (d) (c) (d)
k 2k g g
29. 36.

10–2 –1
)
Hydrostatics (Fluid Pressure and Buoyancy) 11.13

–5 –5
44.
–7 –7

37. r1 and r2 (r2 > r1)

(a) 2 2

r1 r2 r1 r2 1
(a) (c) 2 1 (d) 2 (2 1)
r2 r1 r2 r1 2
1 45. D
(c) (r1 + r2) (d) (r2 – r1)
2
D/ is
38.
( 1)
(a)
–2 –1
g ( 1)
–2
1/ 3 1/ 3
1
(c) (d)
1
39. 46.
radius r
R
s its
ion)
1 1 3 1 1
(a)
s r R s r R
1 1 3 1 1 47.
(c) (d)
s r R s r R
40. W
V R
is
V
(a) 2 W 2 W (a) 1 1
(c) (21/3) W (d) (4 ) W1/3 R R
41. R
(c) (d)

48. r
(a) R R
(c) 4 R R 5r
42.

(a) R R 49.
(c) 4 R R a
43. r1 and r2 a

(a)
4 2
(c) 2 (d) 4
(a) 2 (r1 + r2) (r1 – r2) 50. W
(c) 4 (r1 + r2) (d) 4 (r1 – r2) V
11.14 Comprehensive Physics—JEE Advanced

V
(a) 2 W W
W (d) 16 W
51.

Fig. 11.21
2
4 6 54.
R
52. R
A V
M
H
densities and 2
2

H A
L L
2 5
Fig. 11.22
(a) Mg
L
liq Mg – V g
4 (c) Mg + R2
(d) g (V + R2 )
3 4
(a)
2 3 55.
5 and
(c) (d) 3d
4

Fig. 11.23

(a) decreases and increases


increases and decreases
Fig. 11.20
and increase
53. and decrease

56.
R
T
P0
ABCD
Hydrostatics (Fluid Pressure and Buoyancy) 11.15

(a) | 2P0 + R2 – 2 RT|


2
P0 +R – 2 RT| r
(c) | P0 R2 +R 2
– 2 RT|
(d) | P0 R2 +R 2
+ 2 RT|

Fig. 11.25

Fig. 11.24

57. r

ANSWERS
1. (d) 2. 3. (c) 4. (a) 5. (c) 6. (c)
7. (a) 8. (c) 9. (c) 10. (d) 11. 12. (a)
13. 14. (c) 15. (a) 16. 17. 18. (c)
19. (d) 20. (a) 21. (c) 22. 23. (a) 24. (d)
25. 26. (c) 27. 28. 29. (c) 30. (a)
31. 32. (a) 33. (c) 34. 35. (d) 36. (d)
37. (a) 38. (c) 39. 40. (d) 41. (c) 42.
43. (a) 44. (c) 45. (c) 46. (c) 47. (a) 48. (c)
49. 50. 51. 52. (c) 53. (c) 54. (c)
55. (d) 56. 57.

SOLUTIONS
1. V U W
a=
– =V g (1) ( V)
and – z =V g (2) V g V g 1 / g
= =
V /
1 0 75 g
a= g=
= 0 75 3

v= 2g H
2. v2 – u2 = 2aS
–1
v= 2 = 2 g 1 = 2g 0 – 2g = 2
g
H
3
U V g H
V 3. Let V
W=V g –3
V=
11.16 Comprehensive Physics—JEE Advanced

8.
is

a=
ss 3
= 200 or
V g V g 1000 g 250 g
= = =3g
V 250 9. V
Vg =
v= 2 = 2 3g 1 = 6g g=
k
H
H k= /
/k
v2 6g
H= =
2g 2g – /k
4. Let 1
= 1 a
g= k

( = ) 1
= 1
k
1
or a =g 1
k
F=
10.
is
1
F ( 2 – 1
= 2
A A
1
5. g ( 2 – 1)
2

6. 1
W= ( 2 – 1)
2

1
= V= ( 2 – 1) A
2
7. Let p1 and p2 1 1
W= ( 2 – 1) A g ( 2 – 1)
A and B 2 2
p 1 – p 2= g (
1 2
A = Ag( 2 – 1)
4
11. r
F = (p1 – p2) A = gA (1)
F= = AL
L p1 =
F= ALa (2) p2 = 0
La
=
g p1 p2
p=
2
Hydrostatics (Fluid Pressure and Buoyancy) 11.17

or p= /2
or =

area
or g = g
= 2
2
2
= r
p1 r2 = r2 T =2 =2
g g
2
r = r2 or =r 1/ 2
12. = T T 2
g
13.

16.
2
r r /
r=
/ g=

p= = =2
r /2
=

F
2 A T= – =
=

17. W= = Vg U=
14. Let V
Vg
= V
W= = Vg
F =U – W = ( – ) Vg
V
= V
U
uid is
U =V g
F Vg
a= = = g
V
F = W – U = ( – ) Vg
is
F Vg v= 2 1 (i)
a= =
V
2
= g v= 2 2 (ii)
a 1 = 2
15. or 2
=
a
= = 1
1 g

F = ( – ) Vg =
18. Let V1 A and 1

( = V)
11.18 Comprehensive Physics—JEE Advanced

1
1 V1 g = 1 V1 g r2 = r2 g
4
0 0
3 2
or 1 =
4 = r2 g
2
2
B =( r ) /2 = /2
2V2 2
2 V2 g = g or 2 = 23. Let V1 P1
3 3
1 3 3
= =
2 4 2 P1 = P0 + g

19.
total ss P0 V2
= P2
total lu

= 1 2 3 n P2 = P0
1 2 3 n V2 = 2V1
1 2 3 n P1V1 = P2V2 or (P0 + g) V1
= P0 2V1
= 1 2
= 1 2 1 2 P0 0 75 40 3
1 2
= =
1 2 2 1 g
1 2
2 1 2
1 = 2 = =
2 1
24.
g
20. p = 2 /R R
1 2 1V1 2 V2
= =
V1 V2 V1 V2
g
V1 and V2

1 25.
V1 = V2 = V = ( 1 + 2
2

21. = r2
r r
= = 26.
2 2

r r = 2r = or V g=
V
g+
V
g
2 2
13 6 0 –3
= r2 = (2r)2 or =
2 2
2
=2 r =2 =2
27.
22. and 2 cos
=
+ is r g
= r2
28.
is
Hydrostatics (Fluid Pressure and Buoyancy) 11.19

=( ) g r
r2 = r 21 + r 22 or r = r12 r22
as
= 3 0) 2 (4 0 2
= 0 e–
33. W) =
= 0g e– (1)
= r2 g
P g
= or gr = 2
2
= 0 to =
W =2 r= ( = 2r)
e
P= 0g = 0g
0
k 0 34.

0g 0g
=– (e– – e–0) =
k k is
29. Let
( g )2
is PE = = r2 g = (1)
2 2 2 2g
g 2
W= 2
g = or = (2)
2

2
2 2 2
= g = g 2 2 2
2 PE = =
2g g
4 4
1 1000
= =
2 2 35.
liquid is
30.
P1 =
r

2
g r g
P2 = = =
(2 ) 2 2
P1 = P2
r g
31. =
r 2
2
or r=
g

36. F = 2 W = FS = 2
2
32. air =2 10 10 7
4 4
P1 = and P2 = r1 37. Let r
r1 r2
r2

P1 V1 + P2 V2 = PV
4 4 3 4 4 3 4 4 3 4 4 4
or r1 r2 = r (1) – =
r1 3 r2 3 r 3 r1 r2 r
11.20 Comprehensive Physics—JEE Advanced

r=
r1 r2 1
or v= 2 1
r2 r1 2
38. 45. Let 0
2 2 4
= g or = (R 2
r 3
r r g 3
2 1 3
r = 103 3
4
and g 2 R3
3
10 2
39.
4 4
2 1/3 2 R (R3 r 3) g= R3 0 g
W =4 R (n 1) = 4 R 1 (1) 3 3
r
( R = n1/3 r) or (R3 r 3) = R3
0
4 3
Q= T= R 3
s T (2) or (R r ) = R3
3
3
1/ 3
R
T =
r 1
4
40. W r2 V= r3
3 46. Let A
2/3
3V
W W V2/3
4

41. L=2 R
L=2 R=4 R

42. Tension = r=2 r g


47. =
43. Force = 2 r1 + 2 r2 = 2 (r1 + r2) 2
g
44. and 0 =
2
g 1
g R
= =
g g
2
v g r2 0 =1–
= 2
R

=g – 0 48.

P1 V1 = P2 V2
=g 1 3
4 4 5r
or ( + 10) r3 = (10)
v 3 3 4
or =g 1
125
or ( + 10) = 10
v 64
or v =g 1

49. a = r2
v2 2
r= a/ a
=g =g
2 2 0
2
Hydrostatics (Fluid Pressure and Buoyancy) 11.21

2 cos 2 cos
= = FB
r g a g

a a is A L A 3L
= 2 g+ g
5 4 5 4

50. r is 1 3 1
2
= =
W =4 r 10 20 4
4
V= r3
3 W = FB
2/3
3V 1 1 5
W=4 or ALDG = or D =
4 5 4 4
V
is 53. V
2/3
V
W =4
4
a
2/3
3V 2/3
and F
=4 =4W T
4
F
51. Let r R
6
)

4 r3 4 R3
V = 106 =
3 3
R3 = 106 r 3 or R = 100 r
r is

S = 4 r2
Fig. 11.26

E1 = 4 r 2 106 54.

E2 = 4 R 2
= Mg + g = Mg + R2 g
2 2
E2 R 1 100 r 1 1 55.
= = =
E1 r 106 r 106 102

52.
density in
A 1 56.
= L D= ALD
5 5 ABCD F 1 due
1 ABCD
W) = ALDg 2
5
11.22 Comprehensive Physics—JEE Advanced

1 ABCD 57.
1
p
= P0 g 2 r
2 r
2 AB
=T 2R = 2RT

F = P0 g 2 – 2RT
2
2
= [2P0 +R – 2RT

II

Multiple Choice Questions with One or More Choices Correct


1. 6.
–1
T–2

2.

3. 7.

4.

8.

(a) rises

5. A 9.
B

(a) rises
A
B
A
A
B 10.
Hydrostatics (Fluid Pressure and Buoyancy) 11.23

(a) PA = PB PC = PD
1 2
(c) 1 > 2 (d)
2 1
15. L
M

11.
Mg
n

1
is Mg 1
n

loaded is L1 = L –
12. (d) L1 = L – /n
16.
1
2

103 –3
103 –3

1 and 2 are
(a) 1 1 17.
2 2
13. P
W1 is F

reads W2 (a) P F
P and F

(a) W1 W1
(c) W2 W2 18. a
14. A until
densities 1 and 2 PA PB
PC and PD A B C and D
F
U

Fig. 11.28
Fig. 11.27
11.24 Comprehensive Physics—JEE Advanced

(a) F = 2a F=a g
(c) U = 2a 2
(d) U = a 2 =
2sin
19. and
aera A ga
<< a =
1 and 2
2
and ga
<< a =
liquid are 1 and 2

Fig. 11.29 Fig. 11.30

1 2
(a) = 1 1 – 2 1
21. A and B
A and B
1 2
= 1 1 + 2 1
F

1 (a) A F B F
F= 2 (c) (d)
A F A B F

1 2
cylinder is a =
1 2

20. a

Fig. 11.31
g
(a) =
2 cos

ANSWERS AND SOLUTIONS

1.
2.
3.
4.

5.
Hydrostatics (Fluid Pressure and Buoyancy) 11.25

C
AC is due to
B V

6.
7. Let M and V = (2)

V= (1) V
=
V

V >V

M
V =
V <V
s
( = V =V
Vs =
s
9. 1
2
V = V + Vs ( +
1 2
1
= M (2) V =( +
1 2)/
s
>
1/
s
V <V

2/ s s

1 2
V =
s
8. con
1 1
A s >
s
B AB V <V
V
10.

= V g or V = (1)
11.
g

12. 10 10 = 103 3

= 10–3 3

(a) Let 1

Fig. 11.32
11.26 Comprehensive Physics—JEE Advanced

3
1)
1 2
PC = PD 1 1 = 2 2 or =
2 1
1) 103 g
1> 2 1< 2

M
is 15. V =
103 10–3 g g V
1) 103 g
M
g W =V g= g
1 Mg
2
=
n
n=
Mg 1
Mg – = Mg 1
g= 1000 g n n
2
– 2) 103 g
L1
2
F L
Y=
13. A L
L = (L – L1)
and F = Hence
MgL1
Y= (1)
A L L1
103 –3
L = (L – L1 – )
1
ss 72 and F = Mg 1
V= = 10–3 3 n
density 7 2 103 3
V 1
Mg 1 L
2 Y= n 1 (2)
A L 1

L 1= L –
g
V 16.
= 1000 g
2 in air
= = =6
1 loss in r
= g
2
( V = 10–3 3
)
lo iquid
=
lo

= =

14. Pressure at A and B 17.


Hence P and F

P0

PC = P0 + g and PD = P0 + g 18.
1 1 2 2
Hydrostatics (Fluid Pressure and Buoyancy) 11.27

is = 2a g
F= = 2a
tan =
a
<< a
sin
= sin = (2)
2 a
(a ) =a
19.
ga
=
( 1 + 2) A g= 1 1A + 2 2A
2

21. Let V T

1
A is UA = FVg
1
B is UB = FVg
equal to 1
A is WA = AVg
in 2 B is WB = BVg
2 is F A and B
= 2

M=( 1+ 2) UA = T + WA and UB + T = WB
F 2g =T + (i)
Acceleration a = = FVg AVg
M 1 2 and FVg + T = (ii)
BVg

2
= T
1 2 F= + A F> A
Vg
Hence
20. Let

W=
=

Fig. 11.33
Fig. 11.34
cos
sin T
B = + F B > F
Vg
2 sin =W
or 2 sin = T
g 2 F = A + B
or = (1)
2 sin
11.28 Comprehensive Physics—JEE Advanced

III

Multiple Choice Questions Based on Passage


Questions 1 to 4 are based on the following passage 3.
Passage I P

P
P as
/
P = P0 e 0

P0 0)
and 0
1. 0 is
0 0 0 0
LT L T0
0 0 Fig. 11.35
T
2. / n 4.
n is
1 (a) = 0 =2 0
(c) = e(2) (d) = 0e–1/2
2 0

(c) 1 (d) 2
SOLUTION

1. 3. P
0
4. P = P0
P0 P P0 /
2. =– e / 0
=– = P0 e 0

2
0 0
1 /
= e 0
k k 2
PV = k P= = /
V 2= e 0

( = /V)
k e(2) =
=– 0
0
= 0 e

Questions 5 to 10 are based on the following passage


Passage II
Hydrostatics (Fluid Pressure and Buoyancy) 11.29

9.

5.

10.

1 and 2
1 and 2 is

6.

7.
Fig. 11.36
–3
1
3 3 (a) = ( 1 + 2)
3 3 2
8. R =2 1 + 2
r 1
(c) = (2 1 + 2)
3
1
(d) = ( 1 + 2 2)
3

SOLUTION

5. 30
or = =
5 30 5

7.
6.
3

264 3
is =
1 g = ( – 10) g
3
– 15 or
8. Let its density and
let
1 g = ( – 5) g
4
or = ( – 5) = (R3 – r3)
3
11.30 Comprehensive Physics—JEE Advanced

4 3 1
and = r V
3 3
R3 r3
= (i) 1
r3 V g= V or =
3 3
4
V= R3 Let
3

(1 –
V
or + g = V 0g or + =V 0
V g= g + (1 – ) V g
0
4 4 3 4 or = + (1 – )
( R3 r3 ) r = R3 0
3 3 3
or (R3 – r3) + r3 = R3 (ii) =
3
= / 0 and = / 0 1
= + (1 – )
3

R3 ( ) 1
= or = (1 – )
3
r3 ( 1)
R3 ( )
or 3
1= –1
r ( 1)
3
R r3 (1 ) 1
or 3
= (iii) )
r ( 1) 3

=
10.
1 +
(1 ) 1 0 3) 24
= 4 2
( 1) 2 4 1) 03 (3 )3 g = (3 )2 2 + (3 )2
1g 2g

1
9. V and its den = (2 1 + 2)
V g 3

Questions 11 to 13 are based on the following passage


Passage III
h1
4r
H r
B B h
4r H h2

H
1
liquid is 2r

Fig. 11.37
Hydrostatics (Fluid Pressure and Buoyancy) 11.31

11. 4
(a)
1 3
2 5
2 3 (c) (d)
(a) 3
3 4
13. 2
5
(c) (d) 2
3
2 =
12. 3
2
2 2 =
4

2 =
5
SOLUTION

11. (2r)2 12. U equals


2
=4 r =A W
(4r)2 = 16 r2
2 3A = 4A
= 4A 3
Liquid
A – A = 3A level
h2

Fig. 11.38
U W
U = F1 – F2
= g( + ) 3A – 4A Fig. 11.39
1 1

4
W= 4A 2 =
3
U =W 13. 2
4
g( 1 + )3A – 4 g 1A = g
3
5
1 =
3

IV

Assertion-Reason Type Questions


I

1. Statement-1

Statement-2
11.32 Comprehensive Physics—JEE Advanced

2. Statement-2 5. Statement-1

n
Statement-2

n 6. Statement-1
Statement-2

Statement-2
3. Statement-3

7. Statement-1
R is
2 /R
R is 4 /
Statement-2

Statement-2

8. Statement-1
4. Statement-1
n us R
4
P0
R
n and P0
Statement-2
Statement-2

SOLUTION
1. 5.

2.
g

g 6.

3.

7.
4. . 8.

g 2
= P0 + + g
R
n
12
Chapter
Hydrodynamics (Bernoulli’s
Theorem and Viscosity)

REVIEW OF BASIC CONCEPTS


vt r
12.1 VISCOSITY

2 r2 g
vt =
9
vt r2
dv
F= – A
dx
dv
A
dx

–2
Fig. 12.1
–1 –1
[ML
NOTE
12.2 STOKES’ LAW

-
r v

F= 6 rv 12.4 POISEUILLI’S FORMULA

r
12.3 TERMINAL VELOCITY
p

pr4
Q=
8 l
F l
W–U W U
12.2 Comprehensive Physics—JEE Advanced

Capillaries Connected in Series v 15 2


–1
l1 l2 r1 r2 = 3
x
p
R v –4
F= A
8 l1 8 l2 x
R = R1 + R2 =
r14 r24
12.2

3 –3

p
Q = Q1 = Q 2 = –5
R1 R2

p1 p2

P = p1 + p 2
SOLUTION
Capillaries Connected in Parallel

p 2 r2 g
vt =
9
1 1 1 R1 R 2
= R= 3 3 2
2 12 1 98
R R1 R2 R1 R2 =
5
9 18 1
8 l1 8 l2 –2 –1 –1
R1 = R2 =
r 14 r 42
F=6 r vt
–5
r1 =6
–3 –2

p
Q1 =
R1
12.3
r2
-
p
Q2 = –1
R2 -

12.1
SOLUTION
r R
-
–1
4 4
8 r3 = R3 R = 2r
3 3
2
SOLUTION
2
A –1 –1
–4 2

12.4

–1
v = 15
–2 –1 –3
x
12.3

h1
–2
g h2

SOLUTION
–3

–3

V m V
W mg = V g
U V g
F=W–U – Vg

F F Vg
a= = =
m V V
Fig. 12.3
= g
A p1 = h 1 g
1 2 B p2 = h 2 g
s = ut + at
2 p = p1 – p2 h1 – h2 g
1 a
h a t2
2 AB F = a p h1 – h2 a g
2h 2h
1/ 2 d m = adx
t= =
a
x1 x1
1/ 2 2 2
2 1 Fe = dm x = a xd x
= x2
x2

1 2
= a x12 x 22
2
12.5
=
–3 2
h1 – h2 = x12 x 22
2g
x1 x2 2
–2 2 2
g ]
2 1
–1

12.6

-
–3 –1

3 –3
–2
Fig. 12.2
SOLUTION
SOLUTION
–2 –2
AB l r
dx x
12.4 Comprehensive Physics—JEE Advanced

3 14 1 3 1 12.8 BERNOULLI’S THEOREM


Q= 3 3
12 the total energy of an
6 incompressible and non-viscous liquid in a streamline
3 14 1 3 –1
=
12 energy being the sum of pressure energy, potential energy
and kinetic energy of the liquid.
8 lQ 1
p= PV + m g h + mv 2
r4 2
p 2 1 m
P+ gh + v2 =
2 V
12.5 STREAMLINE OR LAMINAR FLOW
12.9 VELOCITY OF EFFLUX
H
h

CRITICAL VELOCITY AND REYNOLD’S


12.6 NUMBER
Fig. 12.4

k V
vc =
r
A B
1 1 2
r PA + V2 + gH = PB + v + g H–h
k 2 2
PA = PB = P AV = a v
A
a
1 a 2v 2 1 2
k k P + 2
+ gH = P + v + g H–h
2 A 2
k
1/ 2

2g h
12.7 EQUATION OF CONTINUITY OF FLOW v =
a2
1
a1 a2 A2
v1 v2
A>>a v = 2g h
a 1 v1 = a2 v 2 av

vice versa H h
t=
g
R = vt = 2 h H h
12.5

NOTE SOLUTION
H
h =
R =H 2
k
v =
h h r
–1
= 2
12.7 1 1
–1

12.10

SOLUTION –2
g

31 4 1 3 3 SOLUTION
Q
v = 2gh = 2 1 25
4 –1
3 14 1 3 –1
=

A= r2 –2 2 2
12.11
4
Q 3 14 1
= 2 2
A 3 14 1
–1 h = PQ
A a
12.8

–1

SOLUTION
a 1= r 21 a2 = r 22
Fig. 12.5
a 1 v 1 = a 2 v2
a1 v1 r 12 SOLUTION
v2 = = v1
a2 r 22 V P v Q
AV = a v
2
r1
= v1 1 1 1
r2 PP + gh + V 2 = PQ + g v2
2 2 2
2
= 1 –1 –1 PP = PQ = P
1 1
P + gh + V2 = P + v2
12.9 2 2
V 2 = v 2 – 2g h
AV
–2
AV = a v v =
a
12.6 Comprehensive Physics—JEE Advanced

1/ 2
2 g ha 2
V= av
A2 a2 Q= = av = a 2gh
1/ 2
2g h F = Qv = Q 2gh
AV = aA 2 2
A a

12.12 12.13

A - a =
a h 2

Q
v
Q

A >> a L O
2 Fig. 12.6
av agh vQ Q 2gh
SOLUTION
Q
SOLUTION v=
a
v 2gh A >> a
Q2
F = av2 =
a
av 36 3
Q
av v = av2 –5 3
=6
O=F L
2
Q L
F av2 =
a
3 5 2
6 1 14
2 = 4
F = av = a 2gh = 2a gh 2 1
v= 2gh –3

Multiple Choice Questions with Only One Choice Correct

1. T

- T T
–2 2 2
g
T T
2 2 4
2. 3. -
12.7

v
v v2
4. v3 v4
v 10. r
v
v r
v
2
v v v v
5. 4 2
h v v
11. V
vt
R V
-
vt
h g h g vt
2
h g h g
vt vt
6.
A 12.
L
a L
L t1
h - t2
W1 W2

ghA ghA t1 < t2 ; W1 > W2 t1 > t2 ; W1 < W2


gha gha t1 = t2 ; W1 = W2 t1 > t2 ; W1 = W2
7. 13.
r Q
P

r -
P 14.
Q Q
Q Q
4 8
8. A B r
l l
P -
A Q
15. R R
P

2Q
Q
3 16. V
Q 3Q P
d l
3 4 d l -
9. P
a
12.8 Comprehensive Physics—JEE Advanced

V h1
V h2
4
V V
8 16 h1
17. h2
–1
–4 2

23. d
–2 h
g
–4 2 –5 2
–5 2 –5 2

R
18. d
2
L y R 4 R2
R h h
y
R2 h2
4h R
R 24. A B
L B
L A -
2
L A B
L
2

25. m x
19. H
y
h
mg x mg y
R R
y x
R= h H h R= h H h
y x
mg 1 mg 1
R= 2 h H h R= 2 h H h x y
20. R 26. l
H H r r
h= h= p
4 3
H
h= h=H l
2
21.
1/4
r r
r r
27.

–2 2 h
g

22. H
12.9

2 3 2h g
1 2 R= V R= V
g 2h
2 1 aV 2h AV 2h
3 2 R= R=
A g a g
28.
A
a
-

m
t

ga gA
A a
2ga gA
A 2a Fig. 12.7

29. 32. r

3mg 2mg
4A A -
mg mg
2 r2 r3
A 2A
r4 r5
30. A
33. r
a h
a
-
A h
2
–2 r r3
g
–1 –1
r4 r5
–1 –1
34.

31. -
h
–2
A a g
V 2 –1 4 –1
R 5 5
6 –1 8 –1
5 5

ANSWERS
1. 2. 3. 4. 5. 6.
7. 8. 9 10 11. 12.
13. 14. 15. 16. 17. 18.
19. 20. 21. 22. 23. 24.
25. 26. 27. 28. 29. 30.
31. 32. 33. 34.
12.10 Comprehensive Physics—JEE Advanced

SOLUTIONS

1. 1
P1 – P2 = v 22 – v 12
W1 = mgh V gh = ghV 2
1
P = v 2
– v 2]
W2 = P V = h gV 2
3
W = W1 + W2 = 2 h gV = v2
2
W
P= 3
t = 2gh = 3 hg
2
W 2 h gV 6. h1 h2
t= =
P P
3 3 3
2 3 av1 av2 v1 = 2 gh 1 v2 =
= 3
2 gh 2
m1 = av1 m2 = av2
2. h
v = p1 = m1 v1
2gh
p2 = m2 v2

v 1 gh F
va = 2 gh
2 2 2 m 1v 1 – m 2v 2
V
= av 12 – av22
= a 2 gh1 – a 2gh2
V 2V
T = Av = 2 ag h1 – h2
a A gh
A =2 a h
V h
V = h = Pr 4
2 2 7. Q = ; -
8 l
l
2V / 2 V T l Q Pr4 P
T = r
A gh / 2 A gh 2
Q
8.
2 A
3. = –1
] B Pa = 2P Pb = P
ML 1 1
l -

4. r Q = Qa + Qb
2
r r
v Pr 4 2P / 3 r 4 P /3 r 4
=
8 l 8 2l 8 l
5. v = 2gh
v 1 1 1
=
l 3l 3l
12.11

3l mgh = mgL
l =
2
Pr 4 2 Pr 4 2Q 13. l
Q = x
8 3l / 2 3 8 l 3 2
lx
9. l l
V W= l 2 x g d x = l2 g xd x
o o

W W V l2
P= = l2 g
t V t 2
l4 g 14 98
= =
= 2 2
e
1 2
= v av
2
14. -
1 3
= av
2
P v3
10. -
a 1v 1 = a 2v 2 r 21v1 = r 22v2
v2 r2 r2 15. m r -
= 12 =4
v1 r2 r 2 2

v2 = 4v1 = 4v vt -
W–U
F = 6 rvt W
11. 4 r3
= mg U = g r3
2 gr 2 3
vt = = kr2 -
9
2 g 4 r3
k= 6 rvt = mg – g
9 3
4 = mg 1
V= r3
3
2/3 4 r3 m
2 3V
r = 3
4
m
3 2/3 1
vt = k V2/3 rv t vt
4 r
V
2/3
3 2/3
Vt = k V = 4vt 16.
4
[ 2/3
= 4] P r4 Pd4 d
V= r=
8 l 128 l 2
12. - d4
v v V l l
l
2 2 d
v t1 > t2 V
2
12.12 Comprehensive Physics—JEE Advanced

V
V V

17. v 1a 1 = v 2 a 2
–1 –4 2
v1 a1 v2
h v2

v22 = v21 + 2 gh

–1
v2
v1 a1 4
a2 =
v2
–5 2

18.
Fig. 12.8
2 R
V1 v a v1 L
= 1 1
V2 v2 a2 v2 R 2
R = vt
v t
v1 = 2gy v2 = 2g 4 y
v1 2 gy 1 2 H h
= R= 2g h 2 h H h
v2 8 gy 2 g

1 L2 dR
V1 20. R h
= 2 dh
V2 R2
d2 R
1 L2 d h2
V1 = V2
2 R2 R hH – h2 1/2

L
R = h
2 dR
=2 1 hH – h2 –1/2
H – 2h
19. h dh 2
H 2h
= 1/ 2
v hH h2
v= 2g h dR
h
dh
S=H–h H – 2h
t H
h =
2
1 2
S= gt d 2R
2
d h2
1 2
H–h= gt h h
2

2 H h d 2R 2h 1 H 2h
t= 2
=– 1/ 2
1
g dh H h 4h H h
12.13

h= H
P1r14 P2 r24
2 =
d R 8 l1 8 l2
=– 2H
d h2 h H /2 P1 4
l r2 1
= 1 = 24 = 8
R P2 l2 r1 2
h=H R
25. -
21.
1
v2 = p = gh
2 r
v 2 4 3 m y
h= F= r x y g= x y g = mg 1
2g 3 x x
v=r =r
2 2 2
26. R
2 r
h= p R4 p r4 p 2r 4
g =
2 8 l 8 l 8 l
r g
2
h R4 = r4 + 16r4 R 1/4
r

22. 27.
h

R=2 h H h = =
e
H
h1
2 = v
1v1 = 2v2
H
h2
2 A1v1 = A2v2
1 v2 A1 A1 1
H H =
h1 h2 2 v1 A2 2 A1 2
2 2
28. h
H H H H
2 h1 h1 = 2 h2 h2
2 2 2 2
h1 = h2 A

23. v 2 gd
m = Ah
2h
t= Fig. 12.9
g
R vt
2h v= 2gh
= 2 gd = 4dh
g av
2
R
R2 = 4 dh d= av
4h
P1r14 P2 r24 av v = av2
24. Q1 = Q2 =
8 l1 8 l2
Q1 = Q2 t
12.14 Comprehensive Physics—JEE Advanced

AV = av
e av2 a v2
= = AV
Ah Ah v =
a
a 2 gh
= v 2 gh t
Ah
2ga t
=
A R = vt
29. 1
h =h h = gt2
2
2h
v 2gh t=
g
h gh
= 2g
4 2
2h
m R = v
h= g
A
gm
v =
2A AV 2h
R=
30. v a g
V
32.
a v = AV 2 r2g
vt =
av 9
V=
A

P = f vt f=6 rvt
P= 6 r vt × vt
2
2 r2g
=6 r
9

8 g2 2
Fig. 12.10
= r5

P r5
1 1
P + V 2 + gH = P + v2 + gh 33.
2 2
-
v2 = V 2 + 2g H – h
mg
2g H h 2 98 2
v =
2
2
= 25
a 2 6 rv = mg
1
A rv
v –1 v

31. v
4 4
m= r3 6 rv = r3 g
3 3
12.15

h ag = h ag + hk kag
2 r2 g
v=
9 k
h= h h
v2
v2 – gh h =
2g
h r4

34. k - v= 2gh

h ag + hk ag h hk - 8 –1
k = 2 1 64
a 5

II

Multiple Choice Questions with Two or More Choices Correct

1.

6.
2.

3.

7.

8.

4.

9. OP L
a O
P
5. - O
L

P p
v
12.16 Comprehensive Physics—JEE Advanced

3
3 2 2 3 2 2
p2 – p1
p= L p= L 4
4 8
4
3 3 L
v= L v= 14.
2 2
h
10.

–1 –2
g

–1

–1
5 2

11.
PA PB -
A B vA vB
A B
PA = PB A B

PA PB
vA vB
vA vB

Fig. 12.12

15. -
Fig. 12.11
r
12.
- ,
v1 v2 h
p1 p2 pc -
Fig. 12.13
v1 > v2 v1 < v2 ps
p1 > p2 p1 < p2
13. v1 –1
v 2= –1
pc > ps pc < ps
–3 2 r2 2 r2 2
h= h=
p2 – p1 3 2g g

ANSWERS AND SOLUTIONS


1.
2.
3.
12.17

2 2
- dF = mr =a rdr
F
l
r L–l r=
L
av v L L
2 2 r2
a F= a rdr = a
- L l
2 L l

1 2 l
= a [L2 L – l 2] = a 2
l L
2 2
-
2 2
3a L
l=L F=
4. 8

5.
A B
A a1
Fig. 12.14
v1 p1
a2 v2 p2 F 3
P p= = 2 2
L v
B a 8

1 2 1 2 1 2 3
p1 + v1 = p2 + v2 v =p= 2 2
L
2 2 2 8
1
p1 – p2 v22 v12 3 L
2 v=
a2 < a1 2
a1v1 = a2v2 v1 > v2
p2 < p1 10.
11. vA < vB -
6.
1 1 2
PA + v2A = PB + vB
2 2
p r4 vB > vA pB < pA
Q=
8 l
7. 12.

F= 6 rv
8.
-
2 r2g
vt =
9
= 13.
1
9. p = p2 – p1 = v21 – v22
2
dr r O 1 2 2 3
m = a dr =
2
p
12.18 Comprehensive Physics—JEE Advanced

3 4

D2
14. A= ; D vs = r
4
r
d2
a= ;d
4
AV = av 1 2 1 2
ps + v s = pc + vc
AV D 2 2 2
v= V 2 –1
ps pc
a d2
vc
2h 2 1 25 vc
t=
g 1 2 1 2 2
pc – ps = v = r
R = vt = 5 2 2
vx = v –1 pc ps
t v = gt h
–1

v2x +vy2 1/2 2


+ 2
]
–1 pc – ps = gh
= 5 2

15. 1 2 2 r2 2
1 gh = r h=
p+ v2 2 2g
2

III

Multiple Choice Questions Based on Passage

Questions 1 to 4 are based on the following passage F=6 rv


Passage I
1. F=6 rv

–1 –1 –1 –2

–2 –1 –2 –2

2.

-
-
-
-
terminal velocity

r v
-
F
12.19

3. 4.
-

ANSWERS

1. mg
2. vt =
6 r
3. -
4.

Questions 5 to 7 are based on the following passage f -


Passage II
f

6. -
dv
F=– A
dx –1 –1 –1
]
–2 –2 –1 –2
dv ]
A
dx 7. -
–1
–3 –2

–4 –2 –4 –2
5. f
–4 –2 –4 –2
F

SOLUTION
5. 2
ML
= 1
= [ML–1 –1
]
- L 2
L
7.
6.
F F dv –3 2 –4 –2
= = = =4
A d v dx A dx 5

Questions 8 to 11 are based on the following passage a A


Passage IV
m
A. h
12.20 Comprehensive Physics—JEE Advanced

8. 10.

agh Agh Ag 2ag


2a A
agh Agh
11.
9.

a A gh
mg mg gh
A a 2

2a 2A 3gh 3gh
mg mg 4 2
A a

SOLUTIONS
8. P=h g 10.

F P a h ga

2a
= mg -
A
v= 2gh
av e
=
= av 2a
g
A
av v = av2
11.
h =h
av2 = a 2gh = 2 agh

9. agh = v 2gh
2a
mg m = Ah h gh
A = 2g
4 2

Questions 12 to 14 are based on the following passage


2gh g H2 + h2 1/2 1/2
]
Passage V
2g H h 2 gH
13.
A
R
H
hH 2hH
2 hH h2 H2
a
14.

h. A h A 2h
Fig. 12.15
a 2g a g
12. A H a 2H
a 2g A g
12.21

SOLUTION
A dx/dt A
12. vx = 2gh
-
x

vy = 2gH u2 = v2 + 2 as
dx
a 2g x = – A
dt
Ad x
v= v 2x v 2y = 2g H h dt = –
a 2 gx
A
13. =– x–1/2 dx
a 2g
2H
t= t
g
x=h
2H x=h
R = vx t= 2gh 2 hH h/4
g A
t= – x–1/2 dx
a 2g h
14. - h/4
x x1 / 2
A
=–
t a 2 g 1/ 2 h

1/ 2
vx = 2g x 2A h
= – h1/ 2
a a 2g 4

A h
= avx = a 2g x =
a 2g

Questions 15 to 17 are based on the following passage 16.


Passage VI
1
gh 2 gh
a1 h. 2
a2 a1 = 3a2 3
gh 2 2 gh
2
15.
17.
2h h
2gh gh 4
g g
gh 1 2h 2h
gh 6 8
2 2 g g
SOLUTIONS
15. v1 a 1v 1 = a 2v 2
a2 v
v1 = v2 = 2
v2 a1 3
A B,
1 1 2
P + gh + v12 = P + v2
2 2
Fig. 12.16 v12 = v22 – 2gh
12.22 Comprehensive Physics—JEE Advanced

1
v1 = gh h=h h
2
2 1/ 2
3 t=– h dh
16. v2 = 3v1 = gh g h
2
17. h =–
2
2 h 4
h
dh g g
v1 = –
dt
dh 2
dt = – = – h–1/2 dh
v1 g

Questions 18 to 20 are based on the following passage 18.


Passage VII
A
2 2 gH 2 gH
d d 2 gH gH
H/2
a A 19. x
h=H H
H
2
H H
2 2 2
20. h
x
2H 3H
3 2
3H H
4 2
Fig. 12.17

SOLUTION
18. v1 h = H v2 = gH
v2
a
Av1 = av2 v1 = v2 2h
A 19. t=
a << A, v2 a g

1/ 2
3 2h
x = v2 t = H 2h g
1 2 H H 2 g
P + dv1 + dg + d g
2 2 2 1/2
= [h H – 4h
1
=P + d v22+ d gh H
2 h = H x =
v1 2
1/ 2
3
v2 = H 2h g
2
12.23

20. h x Hh – 4h2 –1/2

h x x
dx xm h
dh
3
h= H
d 8
[3Hh – 4h2]1/2
dh h
1/ 2
1 3 12 3
[3Hh – 4h2]–1/2 H – 8h xm = H 3H H = H
2 8 8 4
3
H – 8h h= H
8

IV

Assertion-Reason Type Questions


4. Statement-1

- Statement-2

not
5. Statement-1

1. Statement-1

Statement-2
Statement-2 -

6. Statement-1
2. Statement-1

Statement-2
Statement-2 -

7. Statement-1
3. Statement-1

Statement-2
Statement-2
12.24 Comprehensive Physics—JEE Advanced

8. Statement-1 9. Statement-1

Statement-2

10. Statement-1

Fig. 12.18

Statement-2
Statement-2

SOLUTION
1.
F=6 rv
6.
- 7.
-

8.
2.
-

P
3. P

- 9.
- 10. Av

4.
5.

Integer Answer Type


1.
2
–2
g
-
12.25

SOLUTION
1. vc
1 2 1 2 1 2 2
p v pc – ps = vs r
2 2 2
pc ps
h

vs = r pc – ps = gh
r
1 2 2 r2 2
gh = r h=
2 2g
=2 n n
1 2 1 2
ps + v s = pc + vc Fig. 12.19 2 2 2 2 2
2 2 4 r n 4 1 2
h=
ps pc 2g 2 1
vc
13
Chapter
Simple Harmonic Motion

REVIEW OF BASIC CONCEPTS We will use the sine function. A cosine function is
equally valid.
13.1 SIMPLE HARMONIC MOTION (1) Amplitude The amplitude of SHM is the maxi-
mum (positive or negative) value of the displace-
Simple harmonic motion (SHM) is the simplest kind of
ment from the equilibrium position. Quantity
oscillatory motion in which a body, displaced from its
A
stable equilibrium position, oscillates to and fro about
function) is the amplitude of SHM.
the position when released. If the displacement (x) from
the equilibrium position is small, the restoring force (F) (2) Time period The smallest time interval during
acting on the body is given by which the motion repeats itself is called time period
or simply period of SHM. The angular frequency
F =–kx is related to time period T as
where k is a positive constant, known as the force
2
constant. In the SI system k is expressed in newton =
per metre (N m–1). The acceleration (a) of the body is T
given by Frequency of SHM is the number of complete
F k oscillations completed in 1 second.
a = =– x=– 2x
m m 1
Thus, = =
k T 2
where m is the mass of the body and = 2
m or = =2
Thus acceleration (a) = – constant displacement x. T
The resulting motion is called simple harmonic motion. k
Thus, a simple harmonic motion is a motion in which the Since = , we have
m
acceleration (i) is proportional to the displacement from
the equilibrium position and (ii) is directed towards the 1 1 k
equilibrium position. =
T 2 m
13.2 CHARACTERISTICS OF SHM Angular frequency t in
the sine or cosine function.
The displacement x in SHM at time t is given by
(3) Phase The quantity ( t + ) is called the phase
x = A sin ( t+ ) of SHM at time t; it describes the state of motion at
where the three constants A, and characterize the that instant. The quantity is the phase at time t = 0
SHM, i.e., they distinguish one SHM from another. A and is called the phase constant or initial phase or
SHM can also be described by a cosine function as epoch of the SHM. The phase constant is the time-
x = A cos ( t + ) independent term in the cosine or sine function.
13.2 Comprehensive Physics—JEE Advanced

13.3 VELOCITY AND ACCELERATION IN SHM 1


mA2 2 cos 2 ( t
= ) sin 2 ( t )
2
The velocity V or the particle in SHM is given by
1
dx = mA2 2
V = =A cos ( t+ ) 2
dt E is also obtained as follows:
x E = K.E. + P.E.
But sin ( t+ ) = 1 1
A = m 2(A2–x2) + m 2x2)
1/ 2 2 2
x2
cos ( t+ ) = 1 1
A2 = mA2 2
2
1/ 2
x2 Thus, although the kinetic energy and potential energy
Hence velocity V =A 1
A2 of a simple harmonic oscillator both change with time
t and displacement x, the total energy is independent of
= (A2 – x2)1/2 both x and t and hence remains constant. This is expected
The acceleration a of the particle in SHM is given by because friction has been neglected.
d2 x NOTE
a = 2 1
dt (1) At the mean position, x = 0, K.E. is maximum = mA2 2
2 2 and P.E. = 0 2
=–A sin ( t+ )=– x
(2) At the extreme position, x = A, K.E. = 0 and P.E. is
Notice that, when the displacement is maximum, i.e., 1
when x = |A|, the velocity is zero but the acceleration is maximum = mA2 2.
2
maximum = | 2 A|. But when the displacement is zero 1
(x = 0), the velocity is maximum = | A| and the acceleration (3) E is constant = mA2 2
for all values of x.
2
is zero.
Figure 13.1 shows the variation K.E., P.E. and E with t for
13.4 ENERGY IN SHM the case = 0.
At any instant of time t, the kinetic energy of the oscillator Kinetic Potential Total energy
is given by 1 2 2
2
1 1
K.E. = mV 2 mA2 2
cos 2 t
Energy

2 2
At any displacement x from the equilibrium position,
1
K.E. = m 2 ( A2 x 2 )
2
0 3 time
x2
because cos2 t =1 – sin2 t =1– 2 . 4 2 4
A Fig. 13.1
At that instant t, the potential energy of the oscillator is
given by The energy of an oscillator may decrease with time not
only by friction (damping) but also due to radiation. The
1 2 1 2 oscillating body imparts periodic motion to the particles
P.E. = kx = m A2 sin 2 ( t )
2 2 of the medium in which it oscillates, thus producing
waves. For example, a tuning fork or a string produces
2
because k = m and x = A sin t . In terms of sound waves in the medium which results in a decrease
displacement x, in energy.
1 2 1 2 2
P.E. = kx = m x 13.1
2 2
The displacement x of a body varies with time t as
Total energy of the oscillator is x = a sin (ct) + b cos (ct)
E = K.E. + P.E. where a, b and c are constants.
Simple Harmonic Motion 13.3

(a) Show that the motion of the body is simple har-


SOLUTION
monic.
(b) Find the amplitude A, time period T and phase (i) sin kt + cos kt represents simple harmonic mo-
constant of the motion in terms a, b and c. tion since we can write it as 2 sin kt
4
SOLUTION or 2 cos kt t in the
4 2
(a) Given x = a sin (ct) + b cos (ct) (i) argument of the sine or cosine function = ,
T
Differentiating (i) w.r.t. time t, we get velocity 2 2
where T is the period. Hence k = or T = ,
dx T k
V= = ac cos (ct) – bc sin (ct)
dt 2
i.e. the period of the function is .
The acceleration is given by k
dV d (ii) Each term in (sin t + 2cos 2 t + 3sin 3 t) repre-
a= ac cos(ct ) bc sin(ct )
dt dt sents SHM.
= – c2 [a sin (ct) + b cos (ct)] The period T of the term sin t is given by
a = –c2x [use Eq. (i)] 2
= or T = 2s. The period of the term
T
Since a (–x), the motion is simple harmonic. 2 cos 2 t is 1 s, i.e. T/2 and the period of the term
(b) Since the motion is simple harmonic; 3 sin 3 t is 2/3 s, i.e. T/3. The sum of the three
x = A sin ( t + ) terms, however, does not represent SHM; it rep-
= A [sin ( t) cos + cos( t) sin ]
term completes one cycle, the second term com-
x = (A cos ) sin( t) + (A sin ) cos( t) (ii) pletes two cycles and the third term completes
Comparing (i) and (ii), we get three cycles. This shows that the sum represents
A cos = a (iii) a periodic motion with period T = 2s.

A sin =b (iv) (iii) cos 2kt represents an SHM whose period


3
2 2 T is given by
and =c =c T=
T c 2
2k =
Squaring and adding (iii) and (iv), we get T
A= a2 b2 or T=
k
Dividing (iv) by (iii) we get
b b (iv) cos kt + 2 sin2kt = cos kt + (1 – cos2kt) =
–1
tan = = tan 1 + cos kt – cos 2kt. The period of cos kt is
a a 2 T
T= and that of cos 2kt is = . These
k k 2
13.2 2
two terms together have a period T =
k
Which of the following functions of time represent
as explained in (ii). The other term 1 is a
(a) simple harmonic, (b) periodic but not simple har-
monic, and (c) nonperiodic motion? Find the period constant independent of t and hence does not
for each periodic motion. Here k is a positive real affect the period of the sum. Hence (coskt +
constant. 2sin2kt) represents a periodic motion with period
2
(i) sin kt + cos kt, (ii) sin t + 2cos 2 t + 3 sin 3 t, T= .
k
(iii) cos 2kt , (iv) cos kt + 2 sin2kt, (v) e–kt and (v) e–kt decreases monotonically to zero as t . It
3 is an exponential function with a negative expo-
(vi) log kt. nent of e, where e 2.71828. It is non-periodic.
13.4 Comprehensive Physics—JEE Advanced

(vi) Function log (kt) increases monotonically with


SOLUTION
time. Therefore, it never repeats itself and is a
non-periodic function. The displacement of the particle is given by

13.3 x = 2 cos 0.5 t cm


3
An oscillator has a time period T = 4s. If it is oscil-
-
tion, we compare this equation with
energy.
x = A cos ( t + )
SOLUTION (a) Amplitude A = 2 cm
The kinetic energy of a simple harmonic oscillator (b) Angular frequency = 0.5 rad s–1
2 2 2
varies with time as sin2( t), where = . or T= = = 4s
T 0.5
1 1
Now sin2 ( t) = (1 – cos 2 t) = (1 – cos t), (c) Maximum velocity Vmax = | A |
2 2
2 2 T = 2 0.5 = cm s–1
where = 2 = 2 T = .
T T 2 = 3.14 cm s–1

Hence K.E. varies periodically with period


T
= 2 s. (d) Maximum acceleration amax = | – 2 A |
2 = 2A
NOTE
= (0.5 )2 2
T
The P.E. also varies periodically with period . 2
2 = cm s–2
2
13.4
= 4.935 cm s–2
A body oscillates harmonically with amplitude 0.05 m.
At a certain instant of time its displacement is +0.01 m 13.6
and acceleration is –1.0 ms–2. Find (a) the velocity A particle executes SHM of amplitude 25 cm and
of the oscillator at this instant and (b) the maximum time period 3s. What is the minimum time required
velocity. for the particle to move between two points located at
12.5 cm on either side of the mean position?
SOLUTION
a=– 2x –1.0 = – 2 0.01 = 10 rad s–1 SOLUTION
Velocity when displacement x = 0.01 m is Let A and B be the two extreme positions of the par-
2
V = (A – x ) 2 1/2 ticle with O as the mean position. Displacements to
the right of O are taken as positive while those to the
= 10 [(0.05)2 – (0.01)2]1/2
left of O are taken as negative (Fig. 13.2)
= 0.49 ms–1
Vmax = A = 0.05 10 = 0.5 ms–1

13.5
Fig. 13.2
The displacement x (in centimetres) of an oscillating
particle varies with time t (in seconds) according to Let the displacement of the particle in SHM be given
the equation by
x = 2 cos 0.5 t x(t) = A sin ( t + ) (i)
3
Find 2 2
where A = 25 cm and = = rad s–1
(a) the amplitude of oscillation T 3
(b) the time period of oscillation Let us suppose that at time t = 0, the particle is at
(c) the maximum velocity of the particle extreme position B. Setting x = A at t = 0 in Eq. (i)
(d) the maximum acceleration of the particle. we have
Simple Harmonic Motion 13.5

A = A sin SOLUTION
giving = /2.
The block will not be detatched from the platform,
Putting = /2 in Eq. (i), we get if the frequency of platform’s SHM is such that the
x(t) = A cos t (ii) maximum acceleration of the platform equals the ac-
Now let us say that the particle reaches point C at celeration due to gravity, i.e.
t = t1 and point D at t = t2. At C, the displacement A 2max = g
x (t1) = + 12.5 cm and at D, it is x(t2) = – 12.5 cm (see
Fig. 10.2). So from (ii) we have g 9.8
max = = 3.13 rad s–1
+ 12.5 = 25 cos t1 A 1.0
and – 12.5 = 25 cos t2 max 3.13
max = 0.5 Hz.
or cos t1 = + 0.5 or t1 = /3 2 2 3.14
2
and cos t2 = – 0.5 or t2 =
3 13.9
2 A body of mass m is attached by a string to a suspend-
Hence (t2 – t1) = – =
3 3 3 ed spring of spring constant k. Both the string and the
T 2 spring have negligible mass. The body is pulled down
t2 – t1 = = a distance A and released. Assuming that the string
3 6 T
3 (a) the maximum downward acceleration of the os-
or (t2 – t1)min= = 0.5 s
6 cillating body and
4 (b) the maximum amplitude for which the string re-
Notice that cos t2 = – 0.5 even for t2 = . This
3 mains taut.
value of t2 does not correspond to the minimum time
because this is the time at which the particle, moving SOLUTION
to left, reaches A and then returns to D. (a) As long as the string remains taut, the restoring
force will be proportional to displacement from
13.7
the mean position. The motion of the body is
A particle is executing linear simple harmonic motion simple harmonic whose amplitude is A and an-
of amplitude A. At what displacement is the energy gular frequency is
half kinetic and half potential?
k
SOLUTION =
m
The energy will be half kinetic and half potential at a The maximum acceleration is
kA
2
A=
value of displacement x when K.E. = P.E., i.e. m
(b) If the tension in the string is T, the downward
1 2 1
m (A2 – x2) = m 2
x2 acceleration is
2 2
T
A2 – x2 = x2 a =g –
m
A For the string to remain taut T > 0. Therefore,
x=
2 the maximum downward acceleration cannot
exceed g, i.e.
13.8 amax = g
A horizontal platform is executing SHM in the verti- 2
Amax = g
cal direction with an amplitude of 1.0 m. A block
of mass 5 kg is placed on the platform. What is the g mg 2 k
Amax = =
maximum frequency of platform’s SHM so that the 2
k m
block is not detatched from the platform?
13.6 Comprehensive Physics—JEE Advanced

The block is pulled to the right by a small distance x from


13.10
the equilibrium position and released. The restoring force
At time t = 0, the displacement of a simple harmonic on the block is F = – kx. The acceleration of the block is
oscillator from the mean position is x0 and the veloc-
F k
ity is v0. Obtain the expressions for the amplitude and a= = x (i)
phase constant of the oscillator in terms of x0, v0 and m m
where is the angular frequency of the oscillator. Since a (–x), the motion of the block is simple harmonic.
Comparing Eq. (i) with a = – 2x, we get
SOLUTION
For a simple harmonic oscillator 2 k k
= =
x = A sin ( t + ) (i) m m

dx 2 m
and v= = A cos ( t + ) (ii) Time period T = =2
dt k
Putting t = 0 and x = x0 and v = v0 in (i) and (ii), we
have (2) Vertical Oscillations of a Mass-spring System
x0 = A sin (iii) Consider a massless spring suspended from a support.
and v0 = A cos (iv) [Fig. 13.4 (a)]. A block of mass m is attached at the lower
end, as a result, the string extends by an amount d given
From (iii) and (iv), we get by [Fig. 13.4 (b)].
1/ 2 F = kd mg = kd (i)
v02
A= x02 2 This is the equilibrium state of the system.

x0 1 x0
and tan = = tan
v0 v0

NOTE
If at t = 0, x = x0 and v0 = 0, then A = x0 and = . On
2 v
the other hand, if at t = 0, x = 0 and v = v0 then A = 0
and = 0

EXPRESSIONS FOR TIME PERIOD OF


13.5 MASS-SPRING SYSTEM
(1) Horizontal Oscillations of a Mass-Spring System
Consider a block of mass m placed one horizontal
frictionless surface and attached to a spring of negligible Fig. 13.4
mass and spring constant k as shown in Fig. 13.3.
When the body is pulled through a distance y from this
position and released [Fig. 10.4 (c)], the restoring force is
F = – ky and the acceleration of the block is
F k
a= =– y
m m
Hence the motion is simple harmonic whose time period is

m
T= 2 (ii)
k
Equation (i) determines k. Time period given by Eq. (ii) is
the same as for horizontal oscillation. It depends only on
m and k and is independent of gravity.
Fig. 13.3
Simple Harmonic Motion 13.7

(3) Parallel Combination of Springs If the block is displaced through a distance x, say to
Figure 13.5 shows the equilibrium the right, the spring k1 is extended by x and spring k2 is
state of a block connected to compressed by x so that the restoring force exerted by
two springs which are joined in each spring on the block is in the same direction (along
parallel. If the block is pulled the left). If F1 and F2 are the restoring forces, F1 = – k1x
down through a distance x, the and F2 = – k2 x, the total restoring force is
extension produced in each spring F = – (k1 + k2)x = – kx
will be x. The restoring forces in where k = (k1 + k2) is the effective force constant of the
the springs are F1 = – k1 x and system. The time period is
Fig. 13.5
F2 = – k2 x.
Total restoring force F = F1 + F2 = – (k1 + k2)x = – (kp) x m m
T= 2 =2
where kp = k1 + k2 is the effective force constant of the k k1 k2
parallel combination. The time period is given by
13.11
m m
T= 2 =2 If a spring of force constant k is cut into two equal
kp (k1 k2 )
halves, what is the force constant of each half.
(4) Series Combination of Springs
SOLUTION
Figure 13.6 shows the equilibrium state of
a block connected to two springs which are If a force F produces an extension x in the spring,
joined in series. The block is pulled down by then
F = kx (i)
a distance x. Let x1 and x2 be the extensions
produced in the springs. The restoring force Since the extension produced by a force is propor-
in each spring will be the same equal to tional to the length of the spring, if a spring is cut into
two equal halves, the same force F will produce an
F = – k 1x 1 = – k 2x 2
extension x = x/2 in half the spring. If k is the force
F F constant of half the spring,
so that x1 = – and x2 = –
k1 k2 x
F= k x =k (ii)
Total extension x = x1 + x2 2
Fig. 13.6
From Eqs. (i) and (ii), we get k = 2k.
1 1
= –F
k1 k2 13.12
k1 k2 Two blocks, each of mass m, are connected by a
= –F spring of force constant k and placed on a horizontal
k1k2
frictionless surface as shown in Fig. 13.8. Equal force
k1k2 F is applied to each block as shown. Find time period
or F= – x = – ks x
k1 k2 of the system when the force is removed.
k1k2
where ks = is the effective force constant of the SOLUTION
k1 k2
series combination. The time period is

m m(k1 k2 )
T= 2 =2
ks k1k2 Fig. 13.8

(5) A Block Connected between Two Springs When a force F is applied at each end of a spring,
every coil of the spring is not elongated. The coil
Figure 13.7 shows the equilibrium state of a block
at point O in the middle of the spring is not elon-
connected between two springs
gated. This situation is the same as two springs each
of length l/2 (where l is the length of the complete
spring) joined to each other at point O. If k is the force
constant of the complete spring, the force constant of
Fig. 13.7 each half = 2k. Hence
13.8 Comprehensive Physics—JEE Advanced

The radius of the circle along which the trolley moves


m
T= 2 is
2k r = 40 cm = 0.4 m
When the table is rotated, the tension in the spring is
13.13
equal to the centripetal force, i.e.
A spring has a natural length of 50 cm and a force
constant of 2.0 103 Nm–1. A body of mass 10 kg is mv 2
T= = mr 2
suspended from the spring. (i) What is the stretched r
length of the spring? (ii) If the body is pulled down = 2.0 0.4 (10 )2 = 790 N
further stretching the spring to a length of 58 cm, and
Now, extension produced in the spring by this force
then released, what is the frequency and amplitude of
oscillation. Neglect the mass of the spring. = 40 – 35
= 5 cm = 0.05 m
SOLUTION
force 790 N
Natural length of the spring = 50 cm = 0.50 m Force constant of spring =
extension 0.05 m
k = 2.0 103 N m–1, m = 10 kg
= 1.58 104 Nm–1
(i) The extension produced in the spring is 1.6 104 Nm–1
mg 10 9.8
y= =
k 2.0 103 13.15
= 0.049 m = 4.9 cm A tray of mass M = 12 kg is supported on two identi-
cal springs as shown in Fig. 13.9. When the tray is
Stretched length of the spring = 0.50 + 0.049
depressed a little and released, it executes an SHM
= 0.549 m = 54.9 cm
of period 1.5s. (a) Find the force constant of each
(ii) The frequency of oscillation is given by spring. (b) When a block of mass m is placed on the
1 k 1 2.0 103 tray, the period of the SHM becomes 3.0s. Find the
= value of m.
2 m 2 10
= 2.25 Hz

The length of the spring in the equilibrium posi-


tion = 54.9 cm. If it is pulled to a length of 58
cm, the maximum displacement from the equi-
librium position = 58 – 54.9 = 3.1 cm, which is
the amplitude of oscillation.

13.14
A small trolley of mass 2.0 kg resting on a horizontal
frictionless turntable is connected by a light spring Fig. 13.9
to the centre of the table. The relaxed length of the
spring is 35 cm. When the turntable is rotated at a SOLUTION
speed of 300 rev/min, the length of the spring be- (a) The time period of oscillation of M is given by
comes 40 cm. Find the force constant of the spring.
M
T =2
SOLUTION 2k
Mass of trolley (m) = 2.0 kg where k is the force constant of each spring.
–1 300 Therefore
Frequency of rotation ( ) = 300 rev. min = =
5 rev. s–1 60 2 2
M 2 (3.142) 2 12
k= =
Angular frequency ( ) = 2 = 2 5 = 10 rad s–1 T2 (1.5) 2
= 105.3 N m–1
Simple Harmonic Motion 13.9

(b) The new period T1 is now given by vertical oscillations. Let L, A and be respectively, the
M m length of the liquid column, area of cross-section of the
T1 = 2 tube and density of the liquid. We shall neglect viscous
2k
effects. Since the right-hand side column is higher by 2y,
T12 k with respect to the column on the left-hand side, the mass
so that m + M =
2 2 of this column of liquid is m = 2A y. The restoring force
(which is a gravitational force) is given by
(3)2 105.3
or m + 12 = = 48 F = – mg = – 2A gy = – Ky
2 (3.142) 2
where the force constant K = 2A g. The angular frequency
or m = 36 kg of the harmonic oscillation is
K
EXPRESSIONS FOR TIME PERIOD OF =
13.6 SOME OTHER SYSTEMS
M
where M = AL is the total mass of the liquid in oscillation.
(1) A Ball Oscillating in a Concave Mirror Thus
A small spherical steel 2A g 2g
ball is placed a little =
AL L
away from the centre of
a concave mirror whose
radius of curvature is R.
When the ball is released,
it begins to oscillate about
the centre.
Place a small steel ball at
A, a little away from the Fig. 13.10
centre O of a concave
mirror of radius of curvature R (= OC = AC) as shown in Fig. 13.11 Oscillations of a liquid column
Fig. 13.10. Let ACO = . If m is the mass of the ball,
its weight mg acts vertically downwards at A. This force The time period of oscillation is
is resolved into two rectangular components: mg cos L
(which is balanced by the reaction of the mirror) and mg T =2
2g
sin (which provides the restoring force F). Thus
F = – mg sin It is interesting to note that the period of oscillation does
not depend on the density of the liquid or the area of cross-
= – mg section of the tube.
(since is small, R being very large) (3) Oscillation of Floating Vertical Cylindrical Body
mg x A cylindrical piece of cork of
=
R height h and density c
( x = R ; x being the arc OA) a liquid of density l. The cork is
depressed slightly and released.
= – Kx
Let A be the cross-sectional
where force constant K = mg/R. Thus the motion is area of the cork and M its mass.
harmonic and the angular frequency is given by Fig. 13.12 (a) shows the static
k g equilibrium, the weight of the
= = cork being balanced by the
m R
weight of the liquid it displaces. Fig. 13.12
R If the cork is depressed through
T= 2
g a distance x, as shown in Fig. 13.12 (b), the buoyant force
on it increases by l Agx, because l Ax is the mass of the
(2) Oscillation of a Liquid in a U-tube
liquid displaced by dipping, g being the acceleration due
The column of the liquid is displaced through y by gently to gravity. If viscous effects are neglected, the restoring
blowing into the tube (Fig. 13.11). The columns exhibit force on the cork is given by
13.10 Comprehensive Physics—JEE Advanced

F= – l Agx = – Kx If I is the moment of inertia of the system about the axis


where K = l Ag. Since F – x, the motion of the cork is passing through O and perpendicular to the plane of the
simple harmonic. The time period of the motion is rod, the angular acceleration is
mgL
M =
T= 2 I I
K
where M is the mass of the cork = Ah c. Hence since – , the motion is simple harmonic whose
angular frequency is (compare with = – 2 ) given by
Ah c h mgL mgL
T= 2 2 c 2
= =
l Ag g l I I
(4) A Simple Pendulum I
A simple pendulum consists of a massless inextensible Thus T= 2
mgL
O
and having a small bob at ML2 1
where I= + mL2 = (M + 3m)L2.
the other end (Fig. 13.13). 3 3
When the bob is displaced
from equilibrium position (M 3m) L
Thus T= 2
A to a position B and 3mg
released, the component
L
mg cos of its weight If M << m, T = 2 , the same as that of a simple
balances with tension pendulum. g
T and it returns under a
Fig. 13.13 (6) Horizontal Oscillations of a Cylinder-Spring System
restoring force
F = – mg sin A solid cylinder of mass M
and radius R is connected to
x
If is small, sin . Also = a spring of force constant k as
L shown in Fig. 13.15.
mg
F= x Case (a): Cylinder slips without
L rolling.
Acceleration a =
F
=
g
x. The total energy of the system Fig. 13.15
m L is translational K.E. + P.E. If
x is the instantaneous displacement and v the velocity of
L the cylinder, the total energy is
Time period T = 2
g
1 1
NOTE E= Mv2 + kx2 (i)
2 2
T is independent of the mass of the bob. dE
If friction is neglected, E = constant. Hence = 0.
(5) A Compound Pendulum dt
dE
A compound pendulum Differentiating (i) w.r.t. time t and setting = 0, we
get dt
is a rigid body capable of
oscillating about an axis. The dv dx
0 = Mv + kx
pendulum consists of a rod of dt dt
mass M and length L which is
dv dx
pivoted at O and carries a bob 0 = Mva + kxv ( a= ,v= )
of mass m at the other end as dt dt
shown in Fig. 13.14. The rod k
is displaced and released. Fig. 13.14
a= x
M

restoring torque M
T= 2
= – mgL sin = – mgL (for small ) k
Simple Harmonic Motion 13.11

Case (b): Cylinder rolls without slipping 1 1 1


In this case, the total energy of the system is E= m M v2 + kx2
2 2 2
1 1 1
E= Mv2 + I 2 + kx2 dE
2 2 2 Setting = 0, we have
dt
1 I 1 M dv dx
= Mv2 1 + kx2 0= m v + kx
2 MR 2
2 2 dt dt
v dv kx dx
Acceleration a = =– v
R dt M dt
dE (m )
Setting = 0, we get 2
dt 2
dv I dx = – x
0 = Mv 1 2
+ kx
dt MR dt M
m
I = 2
0 = Mva 1 + kxv
MR 2 k
k
T= 2
k M
a= – x m
M
I 2
R2 If the pulley has a mass M << m, then
1\ 2
I k
M T= 2
T= 2 R2 m
k
(8) A Block Attached to Three Springs
1 A block of mass m is connected to three identical springs
This the general formula. For a solid cylinder, I = MR2 as shown in Fig. 13.17. The block is pushed towards
and then we get 2
spring 1 through a small distance so that spring 1 is
3M compressed by x and springs 2 and 3 are extended by x2 = x3
T= 2
2k = x cos 45° = x / 2 . When the block is released, the
(7) Vertical Oscillations Mass-spring System Connected restoring force acting on it is
by a Pulley F = F1 F2 F3
The block is pulled out through a small distance x and
released. If M is the mass of the pulley and R its radius,
the total energy of the system is [Fig. 13.16]

Fig. 13.17

The magnitude of F along the vertical direction is


Fig. 13.16 F = – (F1 + F2 cos 45° + F3 cos 45°)
1 1 1 2 1 1
E= mv2 + I 2
+ kx = kx kx2 kx3
2 2 2 2 2
kx kx
1 v = kx
where I= MR2 and = . Thus 2 2
2 R
= – 2 kx
13.12 Comprehensive Physics—JEE Advanced

F 2k (viii) A pole of mass M and area of cross-section A


2
Acceleration a= = x =– x. Hence in a liquid of density
m m
M
m T= 2
T= 2 Ag
2k
(ix) LC circuit consisting of inductance L and capacitance
IMPORTANT TIPS AND ADDITIONAL C
13.7 FORMULAE T= 2 LC
(x) Simple pendulum immersed in a liquid
(i) Mass m suspended from a spring of force constant k
l
m T= 2 where g = g 1
T= 2 g
k
(ii) If a spring of spring constant k is cut into n equal parts, = density of liquid, = density of bob.
the spring constant of each part becomes n k. If each (xi) Simple pendulum in a lift moving up with acceleration a
part is loaded with a mass m, the time period of each
l
will be T= 2
g a
m
T= 2 If the lift is moving down with acceleration a (< g)
nk
(iii) If two springs of spring constants k1 and k2 are con- l
T= 2
nected in parallel, the spring constant of the combina- g a
tion is k = k1 + k2. If the combination is loaded with If the lift is falling freely a = g and T = , frequency
mass m, then = 0.
1
m 2 (xii) Simple pendulum in a trolley moving with acceleration
T= 2 a in horizontal direction.
k1 k2
(iv) If two springs of spring constants k1 and k2 are con- l
T= 2 where g = g2 a2
nected in series, the spring constant of the combination g
is given by
(xiii) Simple pendulum in a trolley moving down an inclined
1 1 1 plane of inclination
=
k k1 k2
in which case l
T= 2
1 g sin
m k1 k2 2
T= (xiv) For a seconds pendulum T = 2s. Hence l = 99 cm 1 m.
k1k2
(xv) An extended body pivoted at point P at a distance h
(v) If two masses m1 and m2 and connected to the ends of a from its centre of mass (Fig. 13.18) and oscillating in
spring of force constant k, the time period of the oscil- the vertical plane.
lations of the system is
I
T= 2
T= 2 mgh
k
m1m2
where = is the reduced mass.
m1 m2
(vi) Simple pendulum of length l
l
T= 2
g
(vii) Liquid column of length L in a U-tube

L
T= 2
2g
Fig. 13.18
Simple Harmonic Motion 13.13

where I = moment of inertia of the


4m
body about the pivot. Fig. 13.21 (b) Tb = 2
k
(a) If the body is a rod of mass m and length l
l (Fig. 13.19) pivoted at its end, then
m
Fig. 13.21 (c) Tc = 2
l ml 2 4k
h= and I = .
2 3 l
m k1 k2
2l Fig. 13.21 (d) Td = 2
Hence T= 2 4k1k2
3g
Fig. 13.19
(xvii) Horizontal oscillation (with-
(b) If the body is a ring of mass m and
out slipping) of a disc, ring,
radius r pivoted at a point P on its
cylinder or sphere of mass
periphery (Fig. 13.20), then
M and radius R rolling on
h = r and I = mr2 + mr2 = 2mr2. a horizontal surface (Fig. Fig. 13.22
13.22).
2r
Hence T =2 I
g M
Fig. 13.20 T= 2 R2
For a disc pivoted at a point on its k
periphery,
1 3
h = r and I = mr2 + mr2 = mr2.
2 2 13.8 DAMPED OSCILLATIONS
Hence

T= 2 3r
force are called damped oscillations. Due to damping the
2g amplitude (and hence energy) of the oscillator keeps on
(xvi) Some cases of oscillation of spring-mass-pulley decreasing with time and eventually the oscillator comes
system. The mass of pulley is negligible. to rest. Damping also decreases the frequency of the
oscillator.

13.9 FORCED OSCILLATIONS AND RESONANCE

external periodic force are called forced oscillations. The


external force maintains the oscillations of a damped
oscillator. The amplitude of these oscillations remains
constant.
If the frequency of the externally applied force is equal
to the natural frequency of the oscillator, resonance is said
Fig. 13.21 to occur. If damping is small, the amplitude of resonant
oscillations will become very large. At resonance, the
m oscillator absorbs maximum energy supplied by the
Fig. 13.21 (a) Ta = 2 external force.
k

Multiple Choice Questions with Only One Choice Correct


1. A uniform rod AB of length L is pivoted at one end oscillations about an axis passing through A and
A and hangs vertically. The time period of small perpendicular to the rod is
13.14 Comprehensive Physics—JEE Advanced

6. A body is executing simple harmonic motion of


L L
(a) 2 (b) 2 time period T. Its kinetic energy varies periodically
g 2g with time period equal to
T
L L (a) (b) T
(c) 2 (d) 2
3g 2g
(c) 2 T (d) 2 T
2. The displacement of a particle executing simple
harmonic motion is given by 7. A particle is executing a linear simple harmonic
x = a sin t + a cos t motion. At time t = 0, it is at one extreme position
The total energy of the particle is
1 cm in the next second moving in the same direction.
(a) m a2 2 (b) m a2 2 The amplitude of the motion is
2
(a) 8 cm (b) 10 cm
1
(c) m a2 2
(d) 2 m a 2 2
(c) 12 cm (d) 14 cm
4
8. A body of mass 200 g executing SHM has a veloc-
3. In Q.2 above, the phase constant of the simple ity of 3 cm s–1 when its displacement is 4 cm and a
harmonic motion is velocity of 4 cm s–1 when its displacement is 3 cm.
(a) zero (b) 30° The total energy of the oscillator is
(c) 45° (d) 90°
(a) 2.5 10–4 J (b) 2.5 10–2 J
4. The displacement of a particle executing simple
(c) 2.5 J (d) 250 J
harmonic motion varies with time t as
9. The percentage change in the time period of a sim-
x = a sin t + a sin t ple pendulum if its length is increased by 2% is
3
The amplitude of oscillation is (a) 4% (b) 2%

(a) a (b) 2a (c) 1% (d) 2 %


10. Two springs of equal lengths and equal cross-
(c) 3 a (d) 2a sectional areas are made of materials whose Young’s
5. A block of mass m is attached to a spring of force modulii are in the ratio of 3 : 2. They are suspended
constant k by means of a string going over a and loaded with the same mass. When stretched
frictionless pulley as shown in Fig. 13.23. The block and released, they will oscillate with time periods
is held in position so that the spring is unstretched. in the ratio of
The block is then released and it begins to oscillate
with a small amplitude. The maximum velocity of (a) 3 : 2 (b) 3 : 2
the block during oscillation is
(c) 3 3 : 2 2 (d) 9 : 4
mg mg
(a) (b) 11. Two bodies A and B of equal masses are sus-
k k
pended from two separate springs of force con-
k m stants k1 and k2 respectively. If the two bodies
(c) g (d) g
m k oscillate such that their maximum velocities are
equal, the ratio of the amplitudes of oscillation of
A and B will be
(a) k1 / k2 (b) k1 / k2

(c) k2 / k1 (d) k2 / k1
IIT, 1988
12. A spring of force constant k is cut into three equal
pieces. If these three pieces are connected in paral-
lel, the force constant of the combination will be
(a) k/3 (b) k/9
Fig. 13.23 (c) 3k (d) 9k
Simple Harmonic Motion 13.15

13. The displacement x (in centimetres) of an oscillat- spring is 35 cm. When the turntable is rotated with
ing particle varies with time t (in seconds) as an angular frequency of 10 rad s–1, the length of
the spring becomes 40 cm. What is the force
x = 2 cos 0.5 t constant of the spring?
3
(a) 1.2 103 Nm–1 (b) 1.6 103 Nm–1
The magnitude of the maximum acceleration of the
particle in cms–2 is (c) 2.0 103 Nm–1 (d) 2.4 103 Nm–1
19. A simple pendulum of length l and bob mass m
(a) (b) is displaced from its equilibrium position O to a
2 4
2 2 position P so that the height of P above O is h. It is
(c) (d) then released. What is the tension in the string
2 4 when the bob passes through the equilibrium
14. A particle is executing linear simple harmonic mo- position O? Neglect friction.
tion of amplitude A. What fraction of the total en- 2 mg h
ergy is kinetic when the displacement is half the (a) mg (b)
l
amplitude?
1 1 h 2h
(a) (b) (c) mg 1 (d) mg 1
4 l l
2 2
1 3 20. When a mass m is hung from the lower end of a
(c) (d) spring of negligible mass, an extension x is pro-
2 4
duced in the spring. The mass is set into vertical
15. In Q.14, at what displacement is the energy of the
oscillations. The time period of oscillation is
oscillator half potential and half kinetic?
A A x gx
(a) (b) (a) T = 2 (b) T = 2
4 2 mg m
A A x
(c) (d) (c) T = 2
x
(d) T = 2
2 3 g 2g
16. A horizontal platform is executing simple harmonic
21. A small spherical steel ball is placed a little away
motion in the vertical direction of frequency . A
from the centre of a large concave mirror of radius
block of mass m is placed on the platform. What is
of curvature R = 2.5 m. The ball is then released.
the maximum amplitude of the platform so that the
What is the time period of the motion? Neglect fric-
block is not detached from it?
tion and take g = 10 ms–2.
g mg
(a) (b)
4 2 2
4 2 2 (a) sec (b) sec
4 2
(c)
g
(d)
mg (c) sec (d) 2 sec
2 2 2 2
2 2 22. The potential energy of a particle executing simple
17. A spring of negligible mass having a force con- harmonic motion at a distance x from the equilib-
stant k extends by an amount y when a mass m is rium position is proportional to
hung from it. The mass is pulled down a little and (a) x (b) x
released. The system begins to execute simple (c) x2 (d) x3
harmonic motion of amplitude A and angular fre-
23. The displacement y of a particle executing simple
quency . The total energy of the mass–spring
harmonic motion is given by
system will be
t
1 1 2 1 y = 4 cos2 sin (1000 t)
(a) mA2 2
(b) mA 2
+ k y2 2
2 2 2 This expression may be considered to be a result
1 2 1 1 of the superposition of how many simple harmonic
(c) ky (d) mA2 2 – ky2 motions?
2 2 2
18. A small trolley of mass 2 kg resting on a horizontal (a) two (b) three
frictionless turntable is connected by a light spring
to the centre of the table. The relaxed length of the IIT, 1992
13.16 Comprehensive Physics—JEE Advanced

24. The kinetic energy of a particle executing S.H.M. 29. A rigid cubical block A of mass M and side L is
is 16 J, when it is at its mean position. If the am-
plitude of oscillation is 25 cm and the mass of the same dimensions and of modulus of rigidity such
particle is 5.12 kg, the time period of oscillation is that the lower face of A completely covers the
upper face of B. The lower face of B is rigidly
(a) sec. (b) 2 sec. held on a horizontal surface. A small F is applied
5
(c) 20 sec. (d) 5 sec. perpendicular to one of the side faces of A. After the
force is withdrawn, block A executes small oscilla-
25. The time taken by a particle executing simple tions, the time period of which is
harmonic motion of time period T to move from
the mean position to half the maximum displa- M
(a) 2 M L (b) 2
cement is L
T T ML M
(a) (b) (c) 2 (d) 2
2 4 L

(c)
T
(d)
T IIT, 1992
8 12 30. A particle free to move along the x-axis has poten-
IIT, 1992 tial energy given by
26. One end of a long metallic wire of length L is tied U(x) = k[1 – exp (– x2)] for – x +
to the ceiling. The other end is tied to a massless where k is a constant of appropriate dimensions.
spring of spring constant K. A mass m hangs freely Then
from the free end of the spring. The area of cross- (a) at points away from the origin, the particle
section and Young’s modulus of the wire are A and is in unstable equilibrium
Y respectively. If the mass is slightly pulled down x, there is a
and released, it will oscillate with a time period force directed away from the origin
given by (c) if its total mechanical energy is k/2, it has its
minimum kinetic energy at the origin
m
(a) 2 (d) for small displacements from x = 0, the mo-
K tion is simple harmonic.
YA + KL m 1/ 2 31. The period of oscillation of a simple pendulum of
(b) 2 length L suspended from the roof of a vehicle which
YAK moves without friction down an inclined plane of
m YA inclination , is given by
(c) 2
KL L L
(a) 2 (b) 2
mL g cos g sin
(d) 2
YA IIT, 1993 L L
(c) 2 (d) 2
27. A particle is executing simple harmonic motion g g tan
along the x–axis with amplitude 4 cm and time IIT, 2000
period 1.2 s. The minimum time taken by the par- 32. A spring of force constant k is cut into two pieces
ticle to move from x = + 2 cm to x = + 4 cm and such that one piece is double the length of the other.
back again is Then the longer piece will have a force constant
(a) 0.6 s (b) 0.4 s of
(c) 0.3 s (d) 0.2 s 2 3
(a) k (b) k
28. If a spring extends by x on loading, then the energy 3 2
stored in the spring is (T is the tension in the spring (c) 3 k (d) 6 k
and k its force constant) IIT, 1999
T2 T2 33. A body of mass 1 kg is executing simple harmonic
(a) (b) motion. Its displacement x (in cm) at time t (in
2x 2k
second) is given by
2
2k
(c) (d) 2 T x = 6 sin 100 t
2
T k 4
Simple Harmonic Motion 13.17

The maximum kinetic energy of the body is 40. A horizontal platform with an object placed on it
(a) 6 J (b) 18 J is executing SHM in the vertical direction. The am-
(c) 24 J (d) 36 J plitude of oscillation is 2.5 cm. What must be the
34. Two particles P and Q start from the origin and least period of these oscillations so that the object
execute simple harmonic motion along x-axis with is not detached from the platform? Take g = 10 m
the same amplitude and time periods 3 s and 6 s s–2.
respectively. The ratio of the velocities of P and Q (a) 0.1 sec (b) 0.5 sec
when they meet is
(c) sec (d) 2 sec
(a) 1 : 2 (b) 2 : 1
(c) 2 : 3 (d) 3 : 2 41. The ends of a rod of length l and mass m are at-
tached to two identical springs as shown in Fig.
35. A body is executing simple harmonic motion. At
13.24. The rod is free to rotate about its centre O.
a displacement x, its potential energy is E1 and at
The rod is depressed slightly at end A and released.
a displacement y, its potential energy is E2. The
The time period of the resulting oscillation is
potential energy E at a displacement (x + y) is
m 2m
(a) E1 + E2 (b) E12 E22 (a) 2 (b) 2
2k k
(c) E1 + E2 + 2 E1 E2 (d) E1 E2
2m 3m
36. A body executes simple harmonic motion under the (c) (d)
3k 2k
4
action of a force F1 with a time period s. If the
5
force is changed to F2 it executes S.H.M. with time
3
period s. If both the forces F1 and F2 act simul-
5
taneously in the same direction on the body, its time
period in seconds is:
12 24
(a) (b)
25 25
35 15 Fig. 13.24
(c) (d)
24 12 IIT, 2009
37. If the displacement (x) and velocity (v) of a par-
42. A uniform metre scale of length 1 m is balanced on
ticle executing simple harmonic motion are related
through the expression 4v2 = 25 – x2, then its time
shown in Fig. 13.25. One end of the scale is slightly
period is
depressed and released. The time period (in
(a) (b) 2
(c) 4 (d) 6 seconds) of the resulting simple harmonic motion
is (Take g = 10 ms–2)
38. The bob of a simple pendulum executes simple
harmonic motion in water with a period t, while (a) (b)
2
the period of oscillation of the bob in air is t0. If
the density of the material of the bob is (4/3) × (c) (d)
1000 kg m–3, and the viscosity of water is neglect- 3 4
ed, the relationship between t and t0 is
t
(a) t = t0 (b) t = 0
2
(c) t = 2t0 (d) t = 4t0
39. A body at the end of a spring executes S.H.M. with
a period t1, while the corresponding period for an-
other spring is t2. If the period of oscillation with
the two springs in series is T, then Fig. 13.25

(a) T = t1 + t2 (b) T 2 = t12 + t22 43. A simple pendulum of length l is suspended


1 1 1 from the ceiling of a train which is moving in the
1 1 1
(c) (d) 2 horizontal direction with a constant acceleration
T t1 t2 T t12 t22
13.18 Comprehensive Physics—JEE Advanced

a. The time period of the pendulum is given by 48. A simple pendulum is oscillating without damping.
l When the displacement of the bob is less than max-
T=2 where geff is given by imum, its acceleration vector is correctly shown in
g eff
(see Fig. 13.27).
(a) g (b) (g + a)
(c) (g – a) (d) g 2 a 2
44. A simple pendulum of length l is suspended from
the ceiling of a trolley which is moving, without
friction, down an inclined plane of inclination
. The time period of the pendulum is given by
l
T=2 , where geff is given by
g eff
(a) g (b) g sin
(c) g cos (d) g tan
45. One end of a massless spring of relaxed length
50 cm and spring constant k
frictionless inclined plane of inclination = 30º
as shown in Fig. 13.26. When a mass m = 1.5 kg
is attached at the other end, the spring extends by
2.5 cm. The mass is displaced slightly and re-
leased. The time period (in seconds) of the resulting Fig. 13.27
oscillation will be IIT, 2002
49. For a particle executing simple harmonic motion,
(a) the displacement x is given by x = A cos t. Iden-
7
tify the graph which represents the variation of
2 potential energy (PE) as a function of time t and
(b)
7 displacement x. (see Fig. 13.28).
(c) (a) I, III (b) II, IV
5 (c) II, III (d) I, IV
2 IIT, 2003
(d) Fig. 13.26
5
46. Two particles are executing simple harmonic
motions of the same amplitude and the same
frequency along the same straight line and about
the same mean position. If the maximum separation
between them is 2 times the amplitude, the phase
difference between them is

(a) (b)
2
(c) (d)
3 4
47. A particle executes simple harmonic motion be-
tween x = – A and x = + A. The time taken for it to
go from 0 to A/2 is T1 and to go from A/2 to A is Fig. 13.28
T2. Then
50. The time period of a particle in simple harmon-
(a) T1 < T2 (b) T1 > T2 ic motion is 8 seconds. At t = 0 it is at the mean
(c) T1 = T2 (d) T1 = 2T2 position. The ratio of the distances travelled by it in
IIT, 2001
Simple Harmonic Motion 13.19

1 1 54. A block is kept on a horizontal table. The table


(a) (b) is executing simple harmonic motion of time pe-
2 2
riod T
1 1 static friction between the block and the table is .
(c) (d)
2 1 3 The maximum amplitude of the table for which the
51. A particle is executing linear simple harmonic mo- block does not slip on the surface of the table is
tion about the origin x = 0. Which of the graphs gT gT 2
shown in Fig. 13.29 represents the variation of the (a) (b) 2
2 2
potential energy function U (x) versus x?
gT 2
(c) (d) gT2
4 2
55. A particle executes a linear simple harmonic
motion of amplitude 25 cm and time period 3 s.
What is the minimum time required for the particle
to move between two points located at 12.5 cm on
either side of equilibrium position?
(a) 0.5 s (b) 1.0 s
(c) 1.5 s (d) 2.0 s
56. One end of a light spring of force constant k
to a block A of mass M placed on a horizontal fric-

to a wall (Fig. 13.31). A smaller block B of mass m


is placed on block A. The system is displaced by a
small amount and released. What is the maximum
amplitude of the resulting simple harmonic motion
of the system so that the upper block does not slip
-
Fig. 13.29
tion between the two blocks is .
IIT, 2004
52. A simple pendulum has a time period of 3.0 s. If
the point of suspension of the pendulum starts
moving vertically upward with a velocity v = Kt
where K = 4.4 ms–2, the new time period will be
(Take g = 10 ms–2) Fig. 13.31
9
(a) s (b) 5 s
4 3 Mg mg
(a) Amax = (b) Amax =
(c) 2.5 s (d) 4.4 s k k
IIT, 2005 (M m) g
(c) Amax = (d) None of these
53. A simple pendulum is k
moving simple har- 57. A block A of mass m
monically with a period is placed on a fric-
of 6 s between two tionless horizontal
extreme positions B surface. Another
and C as shown in Fig. block B of the same Fig. 13.32
13.30. If the angular mass is kept on A
distance between B and and connected to the wall with the help of a spring
C is 10 cm, how long of force constant k, as shown in Fig. 13.32. The
will the pendulum take A and B is
Fig. 13.30
to move from position . The blocks move together executing simple har-
C to position D exactly midway between O and C. monic motion of amplitude a. The maximum value
(a) 1 s (b) 2 s of frictional force between A and B is
(c) 3 s (d) 4 s
13.20 Comprehensive Physics—JEE Advanced

(a) ka (b) ka/2 k1 A k2 A


(c) zero (d) mg (c) (d)
k1 k2 k1 k2
IIT, 2005
58. A simple pendulum attached to the ceiling of a M IIT, 2009
stationary lift has a time period T. The distance y
60. A wooden block performs SHM on a frictionless
covered by the lift moving upwards varies with
surface with frequency, v0. The block carries a
time t as y = t2 where y is in metre and t in second.
charge +Q on its surface. If now a uniform electric
If g = 10 ms–2, the time period of the pendulum
will be E is switched-on as shown in Fig. 13.34, then
the SHM of the block will be
4 5
(a) T (b) T (a) of the same frequency and with shifted mean
5 6
position.
5 6 (b) of the same frequency and with the same
(c) T (d) T
4 5 mean position
IIT, 2007 (c) of changed frequency and with shifted mean
59. The mass M shown in Fig. 13.33 oscillates in simple position.
harmonic motion with amplitude A. The amplitude (d) of changed frequency and with the same
of the point P is mean position.
IIT, 2011

Fig. 13.33

k1 A k2 A Fig. 13.34
(a) (b)
k2 k1

ANSWERS

1. (c) 2. (b) 3. (c) 4. (c) 5. (d) 6. (a)


7. (a) 8. (a) 9. (c) 10. (a) 11. (d) 12. (d)
13. (c) 14. (d) 15. (c) 16. (a) 17. (b) 18. (b)
19. (d) 20. (c) 21. (c) 22. (c) 23. (b) 24. (a)
25. (d) 26. (b) 27. (b) 28. (b) 29. (d) 30. (d)
31. (a) 32. (b) 33. (b) 34. (b) 35. (c) 36. (a)
37. (c) 38. (c) 39. (b) 40. (a) 41. (c) 42. (c)
43. (d) 44. (b) 45. (a) 46. (b) 47. (a) 48. (b)
49. (a) 50. (c) 51. (d) 52. (c) 53. (a) 54. (c)
55. (a) 56. (c) 57. (b) 58. (b) 59. (d) 60. (a)

SOLUTIONS

I L
1. T = 2 T=2 , which is choice (c).
MgL 3g
where M is mass of the rod and I is its moment of 2. Given x = a sin t + a cos t (1)
inertia about the axis passing through its end and The displacement equation of a simple harmonic
perpendicular to its length. motion is
x = A sin ( t + )
1
I= ML2 where A = amplitude and = phase constant.
3
Simple Harmonic Motion 13.21

x = A sin t cos + A cos t sin (2) 7. Since the particle is at extreme position at time
Comparing (1) with (2) we get t = 0,
x = A cos t
A cos = a (3)
At t = 0, x0 = A
and A sin = a (4)
At t = 1 s, x1 = A cos
Squaring and adding Eqs. (3) and (4) we get At t = 2 s, x2 = A cos 2
A2 = a2 + a2 A2 = 2a2 2
If A is in cm, 2 = A – A cos cos = 1 –
A
1
Energy = m 2 A2 = m 2 a2 and 2 + 5 = A – A cos 2
2
So the correct choice is (b). 7 = A [1 – (2 cos2 – 1)]
3. Dividing Eq. (4) by Eq. (3) we get = 2 A (1 – cos2 )
tan = 1 = 45°, which is choice (c). 2 2
=2 A 1 1
4. Given x = a sin t + a sin t A
3
4 4 8
7 = 2A 1 1 2
=8–
= a sin t + a sin t cos + a cos t sin A A A
3 3
which gives A = 8 cm.
3a 3a
x= sin t + cos t (1) 8. V = 2
(A 2 – x 2)
2 2
For a simple harmonic motion 9 = 2 (A2 – 16) (1)
x = A sin ( t + ) and 16 = 2 (A2 – 9) (2)
x = A sin t cos + A cos t sin (2) Solving Eqs. (1) and (2), we get A = 5 cm and
Comparing (1) and (2) we get = 1 rad s–1.
1
3a E= m A2 2
A cos = 2
2
1
3a = (200 10–3) (5 10–2)2 (1)2
and A sin = 2
2 = 2.5 10–4 J
2 2
3a
2 3a
Hence A = = 3a2 9. T = 2
L
T2 = 4 2 L
2 2 g g
A = 3 a, which is choice (c). 2 T L
5. When the block is released, the spring extends by =
T L
an amount x given by
T 1 L 1
kx = mg x =
mg
, which is also the amplitude of or = = 2% = 1%
k T 2 L 2
mg F L
the oscillation, i.e. A = 10. Young’s modulus Y =
k A l
F YA
mg k m Force constant k = =
vmax = A = = g l L
k m k
where l is the extension in the spring of original
1
6. Kinetic energy = m A2 2
cos2 ( t + ) length L and cross-sectional area A when a force F
2 = Mg is applied. Now, the time period of vertical
1 oscillations is given by
= m A2 2
[1 + cos (2 t + 2 )]
4 M ML
2 T T =2 =2
Time period of kinetic energy = k YA
2 2
T1 Y2 3
2 = =
T2 Y1 2
T
So the correct choice is (a). Hence the correct choice is (a).
13.22 Comprehensive Physics—JEE Advanced

11. The velocity of an oscillating body is maximum 1


when it is at the equilibrium position where the 14. Kinetic energy (KE) = m 2(A2 – x2)
2
potential energy is zero and the energy is entirely
1
kinetic. At the extreme positions, the kinetic energy Potential energy (PE) = m 2x 2
is zero and the energy is entirely potential. There- 2
fore, the kinetic energy at equilibrium position = 1
Total energy (E) = m 2A 2
potential energy at extreme positions = total en- 2
ergy. Since the maximum velocities (i.e. velocities When x = A/ 2,
at equilibrium position) are equal for the two equal 1 A2
masses, their kinetic energies are also equal = their KE = m 2 A2
2 4
potential energies at extreme positions where the 3 2 2
displacement is maximum = amplitude. If x1 and x2 = m A
8
are amplitudes of bodies A and B, we have
1 KE 3
1 1 x k2 E= m 2A 2
k1 x 21 = k2 x 22 or 1 2 E 4
2 2 x2 k1
15. The energy will be half kinetic and half potential at
Hence the correct choice is (d).
a value of x when KE = PE, i.e.
12. If a force F is applied to a spring of force contant
k and the spring extends by an amount x, then 1 1
m 2(A2 – x2) = m 2x 2
F = kx 2 2
The extension x produced in a spring is propor- A
or A2 – x2 = x2 or x =
tional to its length. Thus, if the spring is cut into 2
three equal pieces, the same force F will produce 16. The block will not be detached from the platform,
an extension x/3 in a piece. If k is the force if the amplitude of the platform’s SHM is such that
constant of the piece, we have the maximum acceleration equals the acceleration
F = k x/3 due to gravity, i.e.
k 2 g g
Therefore = k or k = 3k. Thus, the force Amax = g or Amax = 2
3 4 2v2
constant of each piece is 3k. When springs are Hence the correct choice is (a). Notice that Amax is
connected in parallel, the force constant of the com- independent of the mass of the block.
bination is equal to the sum of the individual force
17. Let L be the relaxed length of the spring and y the
constants of the springs so connected. Therefore,
extension produced in it due to force mg so that
the force constant of the combination = 3k + 3k +
ky = mg (i)
3k = 9k. Hence the correct choice is (d).
The displacement of the mass during oscillation is
13. Given x = 2 cos 0.5 t given by
3 x = A sin ( t + ) (ii)
dx At the instant when the displacement is x
Velocity V =
dt 2
KE of mass = 1 mV2 = 1 m
dx
=–2 0.5 sin 0.5 t 2 2 dt
3
Acceleration a =
dV = 1 mA2 2 cos2 ( t + ) (iii)
dt 2
PE of spring = 1 k(y + x)2
=–2 0.5 0.5 cos 0.5 t
3 2
Maximum acceleration, 1
2 = k (y2 + 2yx + x2)
amax = – 2 0.5 0.5 =– cms–2 2
2
2 2
= k y2 + kyx + 1 kx 2
1
2 2
Magnitude is amax = = cms–2
2 2
k
Hence the correct choice is (c). Using (i) and (ii) and = , we have
m
Simple Harmonic Motion 13.23

At position O, the tension F in the string is given by


PE of spring = 1 ky2 + mgx + 1 m 2
A2
2 2 mV 2
sin2 ( t + ) (iv) F – mg = centripetal force =
l
Taking gravitational PE at the mean position to be mV 2
zero, or F = mg +
l
Gravitational PE at x = – mg x (v) 2 mg h
Adding (iii), (iv) and (v), we get = mg + ( V 2 = 2gh)
l
Total energy of mass–spring system 2h
or F = mg 1
= 1 mA2 2
cos2 ( t + ) + 1 ky2 + l
2 2 Hence the correct choices is (d).
mgx + 1 mA2 2 sin2 ( t + ) – mgx 20. If k is the force constant, we have
2 mg = k x
= 1 mA 2 2
+ 1 ky2 or
m
=
x
2 2 k g
18. The radius of the circle along which the trolley m x
moves is T =2 2
r = 40 cm = 0.4 m k g
When the table is rotated, the tension in the spring Hence the correct choice is (c).
is equal to the centripetal force, i.e. 21. Let the steel ball be placed at A, a little away from
mv2 the centre O of a concave mirror of radius of cur-
F = = mr 2 vature R (= OC = AC), as shown in Fig. 13.36. Let
r
= 2 0.4 (10)2 = 80 N ACO = . If m is the mass of the ball, its weight
mg acts vertically downwards at A. This force is
The extension in the spring is x = 40 – 35 = 5 cm =
resolved into two rectangular components: mg cos
0.05 m
(which is balanced by the normal reaction N of the
F
Force constant k = mirror) and m g sin (which provides the restoring
x force F). Thus
0 F = – m g sin
= = 1.6 103 N m–1
0.05 =–mg
Hence the correct choice is (b). ( is small; x/R being very small)
19. P.E. at point P = mgh. If friction is neglected,
the potential energy is completely converted into
kinetic energy when the bob reaches the equi-
librium position O (see Fig. 13.35). If V is the
velocity of the bob at O, then
1
mV 2 = mgh
2
or V 2 = 2gh

Fig. 13.36

x
=–mg
R
( x = R ; x being the arc OA)
or F = – kx
mg
where k = is the force constant.
Fig. 13.35 R
13.24 Comprehensive Physics—JEE Advanced

m R 2 t1 T
Hence T =2 =2 or = or t1 =
k g T 6 12
2.5 T T
=2 = seconds. t1 – t0 = –0= .
10 12 12
Hence the correct choice is (c). Hence the correct choice is (d).
22. The potential energy of a particle of mass m execut- 26. If the wire extends by an amount x when a force F
ing simple harmonic motion of angular frequency is applied to it, then
at a distance x from the equilibrium position is F/A FL
Y = =
given by x /L Ax
1 1
P.E. = m 2 x2 = k x2 or F =
YA
x = k x; k =
YA
2 2 L L
where k = m 2 is constant. Hence the correct choice
is (c). Thus the force constant of the wire is k. If K is the
t t force constant of the spring, then the force con-
23. We can write, 4 cos2 = 2 2 cos2 =2 stant of the series combination of the wire and the
2 2
(1 + cos t). Therefore, spring is given by
y = 2 (1 + cos t) sin (1000 t) 1 1 1
=
= 2 sin (1000 t) + 2 cos t sin (1000 t) k k K
= 2 sin (1000 t) + sin (1001 t) + sin (999 t) YA
kK K
Thus y is a superposition of three simple harmon- or k = = L
k K YA
ic motions of angular frequencies 999, 1000 and K
1001 rad s–1. Hence the correct choice is (b). But L
a superposition of two or more simple harmonic Y AK
= (i)
motions of different frequency does not produce Y A KL
a simple harmonic motion. The statement of the The time period of the combination is
question is incorrect.
m
24. At the mean position, the velocity of the particle is T =2 (ii)
k
v = A . Therefore
1 1 2 2
K.E. = mA 2 2 = mA2 27. Let the displacement of the particle be given by
2 2 T
2 2 t
2 m A2 x = A sin
= T
T2
where A = 4 cm and T = 1.2 s. If t1 is the time
2m taken by the particle to move from x = 0 to x =
or T= A
K.E. 2 cm, then
1/ 2 2 t1
2 5.12 2 = 4 sin
= 0.25 T
16
which gives t1 = T/12. If t2 is the time taken to
= 0.2 second
move from x = 0 to x = 4 cm, then
Hence the correct choice is (a).
2 t2
25. Let the displacement of the particle be given by 4 = 4 sin
T
2 t
x = A sin t = A sin T
T which gives t2 = . Therefore, time taken to
4
i.e. when x = 0, t0 = 0. When x = A/2, the value of T
t is given by move from x = 2 cm to x = 4 cm is t2 – t1 = –
4
A 2 t1 T T 1.2 s
= A sin = 0.2 s. Therefore, time taken
2 T 12 6 6
2 t1 1 by the particle to move from x = 2 cm to x = 4 cm
or sin =
T 2 and back = 0.4 s. Hence the correct choice is (b).
Simple Harmonic Motion 13.25

28. T = kx. Therefore x = T/k. Now energy stored in The effective acceleration due to gravity acting on
1 1 T 2 T2 the bob is given by
the spring = k x2 = k , which is
2 2 k 2k g2eff = a 2x + (g – ay)2
choice (b).
29. Let the force F produce a deformation x. When = a 2x + g2 + a2y – 2gay
this force is withdrawn, the force that tries to = g2 sin2 cos2 + g2 + g2 sin4
restore block A to its equilibrium position is
– 2g2 sin2
proportional to x and is given by
f =– L x
d2 x f L 2
Acceleration 2
= x x
dt M M

L
where = . The angular frequency of this
M
2
simple harmonic frequency is . Now T = . Fig. 13.38

Therefore = g2 sin2 (cos2 + sin2 ) + g2


M – 2g2 sin2
T =2
L = g2 (1 – sin2 ) = g2 cos2
or geff = g cos
30. Figure 13.37 shows the plot of U(x) versus x.
At x = 0, potential energy U(0) = k[1 – exp (0)] = Now T = 2
L
2
L
k(1 – 1) = 0 and it has a maximum value = k at x = geff g cos
± since Hence the correct choice is (a).
U (± ) = k[1 – exp (– ± ) 2] 32. The force constant of a spring is inversely propor-
= k (1 – 0) = k tional to its length. If a spring of length L is cut into
two pieces of lengths x and (L – x), such that
2L
x = 2 (L – x) or x = ,
3
then the force constant of the spring of length x
is related to the force constant k of the complete
spring of length L as

Fig. 13.37 k1 L L 3 3
= or k1 = k
k x 2 L /3 2 2
Since the total mechanical energy has a constant
value = (k/2), the kinetic energy will be maximum which is choice (b).
at x = 0 and minimum at x = ± . At x = 0 33. Velocity of the body at time t is
dx d
dU
= [2 kx exp (– x2)]at =0 v= 6 sin 100t
dx
x=0 dt dt 4
x 0

Hence the particle is in stable equilibrium at x = 0 = 600 cos 100t cm s–1


4
(origin) and would oscillate about x = 0 (for small
displacements) simple harmonically. Hence (d) is vmax = 600 cm s–1 = 6 ms–1
the only correct choice. 1 1
Maximum K.E. = mv2max = 1 (6)2 = 18 J
31. The acceleration of the vehicle down the plane is 2 2
g sin . The reaction force acting on the pendu- Hence the correct choice is (b).
lum bob gives it an acceleration a = g sin up 34. Since the particles start from x = 0 and have the
the plane. This acceleration has two rectangular same amplitude but different time periods, they will
components ax = a cos = g sin cos and y = meet again at x = 0 where their velocities are maxi-
a sin = g sin2 as shown in Fig. 13.38. mum equal to a 1 and a 2, i.e
13.26 Comprehensive Physics—JEE Advanced

v1 2 T2 T2 6 37. Given 4v2 = 25 – x2. Differentiating with respect to


= 1 =2 time t, we have
v2 2 T1 2 T1 3
dv dx
Hence the correct choice is (b). 8v = 0 – 2x
dt dt
1
35. E1 = m 2 x 2 dv dx
2 or 8va = – 2xv a ,v
dt dt
1 2
or E1 = x m (1) where a is the acceleration. Thus
2
1 1
E2 = 2 2 a= x (i)
2
m y 4
For a simple harmonic motion
1 2
or E2 = y m (2) a= – 2x (ii)
2
Comparing (i) and (ii), we have
1 2
E= m (x + y)2 1
2 =
2
1 2 1
or E = (x + y) m 2 (3) or = or T = 4 , which is choice (c).
2 T 2
From (1), (2) and (3), it follows that 38. Since the density of water is 3/4 of the density of
the bob, the effective acceleration due to gravity
E = E1 E2
when the bob is in water decreases (due to buoy-
3g g
or E = E1 + E2 + 2 E1 E2 ancy) from g to g = g . Now
4 4
which is choice (c).
36. Let the body be displaced by a distance x. If the l
t0 = 2
restoring force is F1, then the angular frequency of g
the resulting simple harmonic motion is given by
l
K Kx F1
2
and t= 2
= 1 (i) g
m mx mx Dividing, we get
where m is the mass of the body. For force F2,
we have t g g
= = 2 or t = 2t0
F2 t0 g g /4
2
2 = (ii) Hence the correct choice is (c).
mx
F2 F1 39. Let m be the mass of the body and k1 and k2 the
2
= (iii) force constants of the two springs. Then
mx
From (i), (ii) and (iii) we get m
t1 = 2 (1)
2 2 2 k1
= 1 + 2
2 2 2 m
2 2 2 and t2 = 2 (2)
or = k2
T T1 T2
If the two springs are connected in series, the
1 1 1 1 1 force constant of the combination is given by
or = 1 1 1
T 2 T12 T22 4 2
3 2
=
5 5 k k1 k2
1 25 25 25 25 25 25 or k=
k1k2
or =
T2 16 9 16 9 144 (k1 k2 )

144 12 m m (k1 k2 )
or T= s, T=2 =2 (3)
25 25 25 k k1k2
which is choice (a).
Simple Harmonic Motion 13.27

Squaring and adding (1) and (2), we get


2m
or T = , which is choice (c).
1 1 3k
t12 + t22 = 4 2
m
k1 k2
2 (k1 k2 )
=4 m
k1k2
= T2 [Use Eq. (3)]
40. The object will not detach from the platform, if the
angular frequency is such that, during the down-
ward motion, the maximum acceleration equals the
acceleration due to gravity, i.e.,
2
max A = g
g
or max =
A Fig. 13.39
2 A 42. Refer to Fig. 13.40. The magnitude of the restoring
or Tmin = =2
max g torque = force × perpendicular distance
Now A = 2.5 cm = 2.5 10–2 m and g = 10 ms–2. = mg × AB = mg × R sin
Substituting these values we get, T = 0.1 sec. Since is small, sin . here is expressed in
Hence the correct choice is (a). radian. The equation of motion of the scale is
41. Let the rod be depressed by a small amount x
d2
(Fig. 13.39). Both the springs are compressed by I = – mgR
x. When the rod is released, the restoring torque is dt 2
given by d2 mgR
l l or 2
=
= (kx) × + (kx) × = (kx)l dt I
2 2
mgR 2 mgR I
x 2x = or = or T = 2 .
Now sin = = . Since is small, sin I T I mgR
l/2 l
, where is expressed in radian. Thus = 2x/l mL2
Now I = . Hence
or x = l/2. Hence 12
L
l k l2 T=
=k ×l= 3gR
2 2
If I is the moment of inertia of the rod about O, then Using the values L = 1 m, g = 10 ms–2 and R =
0.3 m, we get T = /3 second. Hence the correct
d2 kl 2
I =– choice is (c).
d t2 2
d2 kl 2
or 2 =–
dt 2I
d2
Since (– ), the motion is simple harmonic
d t2
whose angular frequency is given by
kl 2
=
2I
2 ml2 Fig. 13.40
Now = and I = . Therefore, we have
T 12 43. There are two perpendicular acceleration vectors; g
kl 2 12 acting vertically downwards and a acting horizon-
2 6k
= 2 = tally. The resultant acceleration is given by
T 2 ml m
13.28 Comprehensive Physics—JEE Advanced

48. In the simple harmonic motion of a pendulum, the


geff = g2 a2 restoring force vector (and hence the acceleration
Hence the correct choice is (d). vector) is tangential to the path of the bob and is
44. The correct choice is (b) since the component of directed towards the mean position. Hence the cor-
the acceleration due to gravity along the plane is rect choice is (b).
g sin . 49. Given x = A cos t. As a function of x, the PE is
45. The force which increases the length of the spring given by
by x = 2.5 cm is F = mg sin . Therefore, the
1
spring constant is PE = m 2 x2
F mg sin 2
k= At x = 0, PE = 0. Hence the correct graph is III. As
x x
a function of t, the PE is given by
m m
Now time period T = 2 2 1 2
k mg sin / x PE = m (A cos t)2
2
x
=2 1 2
g sin = m A2 cos2 t
2
–2 –2
Putting x = 2.5 cm = 2.5 × 10 m, g = 9.8 ms
and = 30º, we get T = /7 second, which is 1
At t = 0, PE is maximum equal to m 2 A 2.
choice (a). 2
Hence the correct graph is I. Thus, the correct
46. Let the motions be given by x1 = A sin t and x2 =
choice is (a).
A sin ( t + ). Separation between the particles at
time t is given by 50. Since the particle is at the mean position at t = 0,
x = x2 – x1 = A sin ( t + ) – A sin t (1) the phase of the motion is zero. Therefore, the
dx displacement of the particle is given by
The separation will be maximum when = 0, i.e.
dt 2 t
when x = A sin ( t) = A sin
A cos ( t + ) – A cos t = 0 T
or cos ( t + ) = cos t t
= A sin ( T = 8s) (i)
or t+ =± t 4
[ cos (– ) = cos ]
t=
The plus sign is not possible because then = 0 1s in (i)]
which implies that the separation x is always zero.
Therefore, t + = – t or t = – /2. Using this A
x1 = A sin =
value in (1), we have 4 2
t
xmax=A sin – A sin = 2 A sin = 2 s in (i)]
2 2 2
Given xmax = 2 A. Hence x = A sin =A
2
2 A = 2 A sin Distance travelled in the IInd second is
2
1 A 1
or sin = or = or = . x2 = x – x1 = A – =A 1
2 2 2 4 2 2 2
Hence the correct choice is (b). A
x1 2 1
47. In simple harmonic motion, the speed of the par- = =
ticle is the maximum at the mean position x = 0, x2 1 2 1
A 1
decreases as it moves towards the extreme posi- 2
tion becoming zero at the extreme position x = A. Hence the correct choice is (c).
Hence the particle will take shorter time to move 51. In simple harmonic motion, the force acting on the
A A
from x = 0 to x = than to move from x = to particle (restoring force) is given by
2 2
x = A. Thus, the correct choice is (a). F = – kx
Simple Harmonic Motion 13.29

where k is a positive constant. Now OD = 2.5 cm


dU Let the displacement of the pendulum be given by
F=
dx x = A sin ( t + )
2 2
dU where A = 5 cm and = rad s–1
Therefore, – kx = T 6 3
dx
Let us suppose that at t = 0, the pendulum is at C,
or dU = kdx i.e. at t = 0, x = A,
x so that
1 A = A sin ( 0+ )
U (x) = kdx = kx 2 + c
2
0 or A = A sin or sin = 1 or =
where c is a constant of integration. 2
Thus the motion of the pendulum is given by
In simple harmonic motion, the potential energy of
the oscillator is zero at the mean position, i.e. U (0) putting
2
= 0. Hence c = 0. Therefore
1 x = A sin t
U (x) = kx 2 2
2 = A cos t = 5 cos t
which is the equation of a parabola. Since U (x) is The value of t for which x = 2.5 cm is given by
positive for all values (positive and negative) of x, 2.5 = 5 cos t
the correct graph is (d). 1
or cos t = or t =
52. Original time period is 2 3
2 t T
l or = , or t = = 1 s.
T1 = 2 (1) T 3 6
g
54. Refer to Fig. 13.41. Let the mass of the block be
When the pendulum is moving upwards, the m and let, at a certain instant of time, the direction
effective value of g is of acceleration a of the table (executing simple
geff = g + a harmonic motion) be along the positive x-direction.
where a is the acceleration of the pendulum which As a result, the block will experience a force ma
is given by directed along the negative x-axis. Consequently,
the force of friction mg will act along the positive
dv d
a= = (Kt) = K = 4.4 ms–2 x-axis. The weight mg of the block will be balanced
dt dt
by the normal reaction R. The block will not slip on
geff = g + a = 10 + 4.4 = 14.4 ms–2. the surface of the table, if the acceleration a of the
motion of the table is such that
Therefore, the new time period is
mg ma or g a
l
T2 = 2 (2)
g eff
From (1) and (2), we get

T2 g 10 1
=
T1 geff 14.4 1.2
Fig. 13.41
T 3
or T2 = 1 = 2.5 s Therefore, for no slipping, the table can have a
1.2 1.2
maximum acceleration amax = g. We know that,
Hence the correct choice is (c). for a simple harmonic motion, amax = 2A, where
1 is the angular frequency and A the amplitude of
53. Amplitude = OC = OB = BC
2 the motion of the table. Therefore, the maximum
10 amplitude is given by
= = 5 cm
2
13.30 Comprehensive Physics—JEE Advanced

2
Amax = g The upper block of mass m will not slip over the
g gT 2 lower block of mass M if the maximum force on the
or Amax = 2
= 2
, upper block fmax does not exceed the frictional force
4 mg between the two blocks. Now
which is choice (c).
fmax = mamax = m 2Amax (2)
55. Let A and B be the two extreme positions of the
where amax is the maximum acceleration and Amax is
particle with O as the equilibrium position. Dis- the maximum amplitude. Using (1) in (2), we get
placements to the right of O are take as posi-
tive, while to the left of O are taken as negative fmax =
mk Amax
(Fig. 13.42) M m
For no slipping, fmax = mg
mk Amax M m g
or = mg or Amax = ,
Fig. 13.42
M m k
which is choice (c).
Let the displacement of the particle in SHM be
given by 57. The blocks will move together as long as the fric-
x(t) = A sin ( t + ) (i) tional force of block B = mass of block B maxi-
2 2 mum acceleration of its S.H.M., i.e.
where A = 25 cm and = = rad s–1
T 3 f = m 2a
Let us suppose that at time t = 0, the particle is at
extreme position B. Setting x = A at t = 0 in Eq. (i) k k
where =
we have ( m m) 2m
A = A sin giving = /2
k
Putting = /2 in Eq. (i), we get Thus f=m a
x(t) = A cos t (ii) 2m
where A = 25 cm. = ka/2, which is choice (b).
Now let us say that the particle reaches point C at 2
t = t1 and point D at t = t2. At C, the displacement 58. Given y = t . The velocity of the lift varies with t as
x(t1) = + 12.5 cm and at D, it is x(t2) = – 12.5 cm dy
(see Fig. 13.12). So from (ii) we have v = = 2t
dt
+ 12.5 = 25 cos t1 and
– 12.5 = 25 cos t2 dv
Acceleration a = = 2 ms–2, directed up-
or cos t1 = + 0.5 or t1 = /3 dt
2 wards, Hence
and cos t2 = – 0.5 or t2 =
3 l
2 T =2
Hence (t2 – t1) = g a
3 3 3
T 2 and T=2
l
t2 – t1 = =
3 6 T g
3
or (t2 – t1)min = = 0.5 s T g 10 5
6 = = =
T g a (10 2) 6
4
Notice that cos t2 = – 0.5 even for t2 =
3 The correct choice is (b)
which gives t2 = 2 T/3 = 4 s which does not 59. If a force F is applied to M, say to the right, let A be
correspond to minimum value of (t2 – t1). Thus the the distance moved by M. If the system is released,
correct choice is (a). it executes simple harmonic motion of amplitude A.
56. The angular frequency of the system is If A1 and A2 are the extensions in springs k1 and k2
1/ 2 then A = (A1 + A2) and
k
= (1) F = k1 A1 = k2 A2
M m
Simple Harmonic Motion 13.31

F F The amplitude of point P = amplitude of oscilla-


A1 = and A2 = tions of spring k1 which is
k1 k2
F k2 A
1 1 A1 = =
A = A1 + A2 = F k1 (k1 k2 )
k1 k2
60. The force exerted on charge +Q
F (k1 k2 ) E is
=
k1 k2 F = QE
k1 k2 A in the direction of E . Since F is constant, a con-
F= stant force is added to the applied force. Hence only
(k1 k2 )
the mean position will change and the frequency of
oscillation will remain the same.

II

Multiple Choice Questions with One or More Choices Correct

1. Which of the following expressions represent sim- Fig. 13.43. The trolley is displaced from its equi-
ple harmonic motion? librium position by a distance x and released. The
(a) x = a sin ( t + ) trolley executes simple harmonic motion of period
(b) x = a cos ( t + ) T. After some time it comes to rest due to friction.
(c) x = a sin t + b cos t The total energy dissipated as heat is (assume the
(d) x = a sin t cos t damping force to be weak)
2. Choose the correct statements from the follow- 1
(a) kx2 (b) kx2
ing in which k is a real, positive constant. 2
(a) Function f (t) = sin kt + cos kt is simple 2 2
mx 2 mx 2
harmonic having a period 2 / k. (c) (d)
T2 T2
(b) Function f (t) = sin t + 2 cos 2 t +
3 sin 3 t is periodic but not simple harmonic
having a period of 2 s.
(c) Function f (t) = cos kt + 2 sin2 kt is simple
harmonic having a period 2 / k.
(d) Function f (t) = e–kt is not periodic.
Fig. 13.43
3. A simple pendulum of length l and bob mass m
is displaced from its equilibrium position O to a 5. In order to execute simple harmonic motion, a
position P so that the height of P above O is h. It is system must have
then released. What is the tension in the string (a) inertia (b) moment of inertia
when the bob passes through the equilibrium posi- (c) elasticity (d) buoyancy
tion O? Neglect friction. V is the velocity of the
6. The time period of a system executing simple har-
bob at O.
monic motion depends upon
V2 2 mg h (a) mass of the system
(a) m g (b)
l l (b) force constant of the system
h 2h (c) amplitude of the oscillator
(c) mg 1 (d) mg 1
l l (d) phase constant of the oscillator.
4. A trolley of mass m is connected to two identi- 7. The amplitude of a particle executing simple har-
cal springs, each of force constant k, as shown in monic motion depends upon
13.32 Comprehensive Physics—JEE Advanced

(a) initial displacement 10. A simple pendulum is oscillating between extreme


(b) initial velocity positions P and Q about the mean position O. Which
(c) initial acceleration of the following statements are correct about the
(d) initial phase. motion of the pendulum?
(a) At point O, the acceleration of the bob is
8. The displacement (x) of a particle as a function of
different from zero
time (t) is given by
(b) The acceleration of the bob is constant
x = a sin (bt + c) throughout the oscillation
where a, b and c are constants of motion. Choose (c) The tension in the string is constant through-
the correct statements from the following: out the oscillation
(d) The tension is the maximum at O and the
(a) The motion repeats itself in a time interval
minimum at A or B.
2 /b
11. Two springs A and B have force constants k1 and k2
(b) The energy of the particle remains constant
respectively. The ratio of the work done on A to that
(c) The velocity of the particle is zero at done on B in increasing their lengths by the same
x=±a amount is and the ratio of the work done on A
(d) The acceleration of the particle is zero at to that done on B when they are stretched with the
x=±a same force is . Then
9. Figure 13.44 (a) shows a spring of force constant k k
(a) = 1 (b) = 2
k m at the k2 k1
other end placed on a horizontal frictionless sur-
k1 k2
face. The spring is stretched by a force F. Figure (c) = (d) =
13.44 (b) shows the same spring with both ends free k2 k1
and a mass m 12. A body of mass 50 g executing linear simple har-
stretched by the same force F. The mass in case (a) monic motion has a velocity of 3 cms–1 when its
and the masses in case (b) are then released. displacement is 4 cm and a velocity of 4 cms–1 when
Which of the following statements are true? its displacement is 3 cm.
(a) While oscillating, the maximum extension of (a) The amplitude of oscillation is 5 cm.
the spring is more in case (a) than in case (b). (b) The angular frequency of oscillation is
(b) The maximum extension of the spring is the 1 rad s–1.
same in both cases.
(c) The maximum kinetic energy of the oscillator
(c) The time period of oscillation is the same in
is 6.25 10–5 J.
both cases.
(d) The time period of oscillation in case (a) is (d) The maximum potential energy of the oscillator
2 times that in case (b). is 6.25 10–5 J.
13. The displacement of an oscillating particle is
given by
x = a sin(ct) + b cos(ct)
where a, b and c are constants. If A is the amplitude
of the oscillations and T their time period, then
(a) A = a + b (b) A = a2 b2
2 2 a
(c) T = (d) T =
c bc
14. All the springs shown in Fig. 13.45 (a), (b) and (c)
are identical, each having a force constant k. If Ta,
Fig. 13.44 Tb and Tc are the time periods of oscillations of the
three systems respectively, then
Simple Harmonic Motion 13.33

1/ 2
L
(d) 2
v2
g
R
17. A solid sphere of density and radius R -
ing in a liquid of density with half its volume
submerged. When the sphere pressed down slightly
and released, it executes simple harmonic motion
of time period T. If viscous effect is ignored, then
(a) =2 (b) =2
2R 3R
(c) T = 2 (d) T = 2
3g 2g
IIT, 2004
18. Two simple harmonic motions are represented by
Fig. 13.45
the following equations
Tb
(a) Ta = (b) Tb = 2Tc y1 = 10 sin (12t + 1)
2 4
Tc
(c) Ta = 2 Tc (d) Ta = 2Tb = and y2 = 5 (sin 3 t + 3 cos 3 t)
2
15. Two masses m1 and m2 are suspended together by The ratio of their amplitudes is m and the ratio of
a massless spring of spring constant k (see Fig. their time periods is n. Then
13.46). When the masses are in equilibrium, m1 is 2
removed without disturbing the system. If is the (a) m = (b) m = 1
3
angular frequency of oscillation and x2 the ampli-
tude of oscillation of mass m2, then 3
(c) n = (d) n = 1
k 2
(a) = IIT, 1986
m2
19. A solid cylinder of mass m and radius R is attached
k to a massless spring of force constant k as shown
(b) =
(m2 m1 ) in Fig. 13.47. The cylinder is pushed to the right
m1 g a little and released. It executes simple harmonic
(c) x2 = motion. The cylinder rolls on the horizontal surface
k
without slipping.
m g
(d) x2 = 2 Fig. 13.46
k
IIT, 1981
16. A simple pendulum of length L and mass m is sus-
pended from the ceiling of the compartment of a
train that is travelling at a constant speed v around Fig. 13.47
a circular track of radius R. The tension in the string
is T. The time period of the pendulum is The time period of oscillation is T. When the
instantaneous displacement of the cylinder from
mL
(a) 2 the mean position is x, the total energy of the
T system is E. Then
L
(b) 2 2k m
g (a) T = 2 (b) T = 2
3m k
1/ 2
L
(c) 2 1/ 2 3 2 1 2 1 2 1 2
v4 (c) E = mv + kx (d) E = mv + kx
g2 4 2 2 2
R2
13.34 Comprehensive Physics—JEE Advanced

20. A man is standing on a weighing machine placed 21. The displacement x of a particle varies with time t
on a horizontal platform that is executing vertical as
simple harmonic motion of angular frequency . x = A sin2 t + B cos2 t + C sin t cos t
The maximum and minimum readings of the ma-
chine are m1 kg and m2 kg respectively. If the true For what values of A, B and C is the motion simple
mass of the man is m and A is the amplitude of the harmonic?
motion then (a) All values of A, B and C with C 0.
1 m1m2 (b) A = B, C = 2B
(a) m = (m1 + m2) (b) m = (c) A = – B, C = 2B
2 (m1 m2 ) (d) A = B, C = 0
(m1 m2 ) g (m1 m2 ) g IIT, 2007
(c) A = (d) A =
(m1 m2 ) 2 (m1 m2 ) 2

ANSWERS AND SOLUTIONS


1. Simple harmonic motion is represented by a sine 3. P.E. at point P = mgh. If friction is neglected,
function or a cosine function or a linear combi- the potential is completely converted into kinetic
nation of them. Hence the correct choices are (a), energy when the bob reaches the equilibrium posi-
(b) and (c). The choice (d) which is a product tion O (see Fig 13.48). If V is the velocity of the bob
of the two functions does not represent a simple at O, then
harmonic motion. 1
2. Statement (a) is correct. The function f (t) = sin kt mV 2 = mgh
2
+ cos kt can be written as f (t) = 2 sin kt or V 2 = 2gh
4
At position O, the tension F
or 2 cos kt both of which are simple in the string is given by
4
t in the argument F – mg = centripetal force
of the sine or cosine function = 2 /T where T is the mV 2
period. Hence k = 2 / T or T = 2 /k. =
l
Statement (b) is also correct. Each term repre-
sents simple harmonic motion. The period T of term mV 2
or F = mg + Fig. 13.48
sin t is = 2 / T or T = 2s. The period of term l
2 cos 2 t is 1s, i.e. T/2 and the period of term 3 2 mg h
= mg + ( V 2 = 2gh)
sin 3 t is 2/3 s, i.e. T/3. The sum of two or more l
simple harmonic motions of different periods is not 2h
simple harmonic. The sum, however, is periodic. or F = mg 1
l
Hence the correct choices are (a) and (d).
second term completes two cycles and the third
term completes three cycles. Thus the sum has a 4. In general, the motion of a damped oscillator is not
period of 2s. simple harmonic. If the damping forces are weak,
Statement (c) is incorrect. We can write the motion is very nearly simple harmonic and all
formulae of SHM apply. The amplitude A = x. The
f(t) = cos kt + 2 sin2kt
time period T is
as f (t) = cos kt + (1 – cos 2kt)
m
= 1 + cos k t – cos 2kt T=2 (i)
2k
The period of cos kt is T = 2 /k and of cos 2kt is
If the trolley eventually comes to rest, the entire
/k which is T/2. As explained above, the period of
energy of oscillation is dissipated as heat due to
the two terms together is T = 2 /k. The term 1 is a
friction. Hence, the total energy dissipated as heat is
constant independent of time.
Statement (d) is correct. Function e–kt decreases 1 1 2 2
2 2
mx 2
monotonically to zero at t ; it never becomes E= mA 2 2
= mx2 =
2 2 T T2
negative. Hence it is non-periodic.
(ii)
Simple Harmonic Motion 13.35

which is choice (c). Using (i) in (ii) we get tional to the length of the spring, the force F applied
E = kx 2 at end B produces an extension x/2 in the part OB
of the spring and the force F applied at A produces
Hence choice (b) is also correct. an extension x /2 in the part OA. The total extension
5. The correct choices are (a) and (c). x x
in the spring is = x.
6. The correct choices are (a) and (b). 2 2
7. The correct choices are (a) and (b). Thus, the maximum extension produced in the
8. The motion of the particle is simple harmonic. The spring in cases (a) and (b) is the same.
displacement at time t is Now, the force constant of half the spring is twice
that of the complete spring. In case (b) the force
x = a sin (bt + c) constant = 2 k. Hence the time period of oscillation
2 will be
Displacement at time t is
b T =2
m
2k
2 2
x at t = a sin b t c T
b b = 2
T
= a sin (bt + c + 2 ) Hence the correct choices are (b) and (d).
= a sin (bt + c) 10. Statement (a) is correct. At any position between
O and P or between O and Q, there are two
= x at time t accelerations–a tangential acceleration g sin and a
Hence statement (a) is correct. Statement (b) is centripetal acceleration v2/l (because the pendulum
also correct since the same displacement is recov- moves along the arc of a circle of radius l) where
ered after a time interval of 2 / b. Statement (c) is l is the length of the pendulum and v its speed at
correct because the velocity is zero when the dis- that position. When the bob is at the mean position
O, the angle = 0, therefore sin = 0; hence the
placement = ± amplitude a, i.e. at the extreme ends
tangential acceleration is zero. But at O, speed v
of the motion. Statement (d) is incorrect, the accel-
is maximum and the centripetal acceleration v2/l
eration is maximum (in magnitude) at x = ± a.
is directed radially towards the point of support.
9. The maximum extension x produced in the spring When the bob is at the end points P and Q, the speed
in Fig. 13.25 a is given by v is zero, hence the centripetal acceleration is zero
F = kx at the end points, but the tangential acceleration is
maximum and is directed along the tangent to the
or x = F/k
curve at P and Q. The tension in the string is not
The time period of oscillation is constant throughout the oscillation. At any position
between O and end points P or Q, the tension in the
mass m
T=2 =2 string is given by T = mg cos . At the end points
force constant k P or Q, the value of is the greatest, hence the
In case (a) one end A tension is the least. At the mean position O, = 0
wall. When a force F is applied to the free end B and cos = 1 which is the greatest; hence tension
in the direction shown in Fig. 13.44a the spring is is the greatest at the mean position.
stretched exerting a force on the wall which in turn 11. F1 = k1x, F2 = k2x.
exerts an equal and opposite reaction force on the 1 1
spring, as a result of which every coil of the spring Work done W1 = k1x2 and W2 = k2x2
2 2
is elongated producing a total extension x. In case W k
(b) shown in Fig. 13.44b, both ends of the spring = 1 = 1
W2 k2
are free. Therefore, the reaction force is absent, as
a result of which every coil of the spring is not When the springs are stretched by the same force
elongated when force F is applied at each end in op- F, the extensions in springs A and B are x1 and x2
posite directions. The coil at point O in the middle respectively which are given by
of the spring is not elongated. This situation can be F = k1 x1 = k2 x2 or
x1
= 2
k
(i)
visualized as two springs each of length l/2 (where x2 k1
l is the length of the complete spring) are joined to 1 1
each other at point O. Since extension is propor- Work done W1 = k1 x12 and W2 = k2 x22
2 2
13.36 Comprehensive Physics—JEE Advanced

W1 k x12
= 1 (ii)
W2 k2 x22
Using (i) and (ii) we get
W1 k1 k22 k2
= = 2
W2 k2 k1 k1
Hence the correct choices are (a) and (d).
12. In SHM, the velocity V at a displacement x is given
by
V= (A2 – x2)1/2
or V2 = 2
(A2 – x2)
Now V = 3 cm s–1 when x = 4 cm. Therefore,
2
9= (A2 – 16) (i)
Also V = 4 cm s–1 when x = 3 cm. Therefore,
2 2 Fig. 13.49
16 = (A – 9) (ii)
Simultaneous solution of Eqs. (i) and (ii) gives Subtracting (1) and (2) we have
m1g = kx2 (3)
Amplitude A = 5 cm
Putting A = 5 cm in Eq. (i) above we get = Hence, Restoring force on m2 = – kx2
1 rad s–1. F k
accelerating of m2 = x2
Maximum kinetic energy = maximum potential m2 m2
1
energy = total energy which is E = mA2 2 i.e., acceleration – displacement
2
1 k
= (50 10–3) (5 10–2)2 (1)2 Angular frequency is =
2 m2
= 6.25 10–5 J Frequency of oscillation is
Hence all the four choices are correct. 1 k
n=
13. The correct choices are (b) and (c). 2 2 m2
14. The correct choices are (a), (b) and (c). In case It is clear that A is the equilibrium position of m2
(b), the equivalent force constant of the series and B its maximum displacement position. Hence
combination is ks = k/2 and in case (b), the equiva- AB = x2 is the amplitude of oscillation of m2 which
lent force constant of the parallel combination is from Eq. (3) is given by
kp = 2k. mg
Amplitude = x2 = 1
15. Let x1 be the extension produced in the spring k
when it is loaded with mass m2 alone and x2 be the Thus the correct choices are (a) and (d).
further extension when mass m1 is added to mass 16. Since the train is moving in a circle, it is in an
m2 so that x = x1 + x2 is the total extension produced accelerated (non-inertial) frame of reference.
by m1 + m2 (see Fig. 13.49). Thus we have, mv2/R is to
For equilibrium state of m2 be introduced as shown in Fig. 13.50. This force
m2 g = kx1 (1) is horizontal. Consequently, the equilibrium posi-
For equilibrium state of (m1 + m2) tion of the pendulum will not be vertical; it will be
(m1 + m2)g = k(x) = k(x1 + x2) (2) inclined at an angle with the vertical. If T is the
tension in the string in this position, it follows from
When the mass m1 is removed, the mass m2 will
move upwards under the unbalanced force = m1 g.
m v2
Hence T cos = mg and T sin =
Restoring force (F) on m2 = – m1g R
Simple Harmonic Motion 13.37

F= – R 2 gx

Fig. 13.50
Squaring and adding we get
2
2 2 m v2
T = (mg) +
R
1/ 2 Fig. 13.51
2 v4
or T= m g = mge Therefore, the acceleration of the sphere is [use
R2
Eq. (1)]
where the effective value of acceleration due to
gravity is F R2 g x 3g x
1/ 2 a=
v4 m 4 4 R
ge = g 2 R3
R2 3
1/ 2 Using Eq. (2) we get
L mL
Time period = 2 =2 3g
ge T 2
a= x =– x (3)
1/ 2 2R
L
=2
v4
1/ 2 2R
2 which gives T = 2 . Hence the correct choices
g 3g
R2 are (a) and (c).
The correct choices are (a) and (c). 18. The two simple harmonic motions are represented
17. Mass of sphere is by equations
4 12 t
m= R3 = V (1) y1 = 10 sin
3 4 4
where V is the volume of the sphere. The volume
= 10 sin 3 t (1)
of sphere under water = volume of water displaced 4
V
= . If is the density of water, the upthrust is and y2 = 5 sin 3 t + 5 3 cos 3 t (2)
2
1 Refer to the solution of Q.13 of this section. The
U = (V g)
2 correct choices are (b) and (d). The amplitude of
2
each = 10 units and the time period of each =
sphere, i.e. U = mg or 3
second.
1
(V g) = V g 19. Total energy = translational K.E. + rotational K.E.
2
or =2 (2) + P.E. stored in spring
If the sphere is pressed down through a small dis- 1 1 1
= mv2 + I 2 + kx2
tance x (see Fig. 13.51), the volume of water dis- 2 2 2
placed due to this pressing = volume of a disc of 2
1 2 1 1 v 1 2
radius R (since the half the sphere is submerged) = mv + mR 2 + kx
and thickness x which is R 2 x. Hence, upthrust 2 2 2 R 2
due to this pressing is R 2 x g, which provides the 3 1
restoring force. Hence the restoring force acting on E= mv2 + kx2 (1)
4 2
the sphere when it is released is given by
13.38 Comprehensive Physics—JEE Advanced

Now, the total energy of the system must remain 1 1


dE or x= (A + B) + (B – A)cos2 t
constant, i.e. = 0. 2 2
dt
Differentiating Eq. (1) with respect to time t and C
+ sin2 t (1)
dE 2
setting = 0, we have
dt Choice (a): Equation (1) can be written as
x = x0 + a cos2 t + b sin2 t (2)
dE 3 dv 1 dx
= 0 = m 2v + k 2x 1 1 C
dt 4 dt 2 dt where x0 = (A + B), a = (B – A) and b = .
2 2 2
dv dx Equation (2) can be recast as
Now acceleration a = and velocity v = .
dt dt x = x0 + A0sin(2 + ) (3)
3
Therefore, mva + k vx = 0 2 2 1/2
where A0 = (a + b ) and tan = a/b. Equation
2
(3) represents a simple harmonic motion of angu-
3 lar frequency 2 , amplitude = x0 + A0 and phase
or v ma kx = 0 constant .
2
Since v 0, we have Choice (b): For A = B and C = 2B, Eq. (1) be-
3 comes
ma + kx = 0 x = B + B sin2 t = B(1 + sin2 t)
2
2k 2 This equation represents a simple harmonic motion
or a=– x=– x (2)
3m of amplitude 2 B and angular frequency 2 .

2k 2k Choice (c): For A = – B and C = 2B, Eq. (1)


where = . Hence T = 2 becomes
3m m
x = B cos2 t + B sin2 t
The correct choices are (a) and (c).
which represents a simple harmonic motion of
20. mg + m 2A = m1g and mg – m 2 A = m2g. Form amplitude B, angular frequency 2 and phase
constant /4.
(a) and (d).
21. The displacement equation can be rewritten as Choice (d): For A = B and C = 0, Eq. (1) reduces to
x=A
A B C which does not represent simple harmonic motion.
x= (1 – cos2 t) + (1 + cos2 t) + sin2 t
2 2 2 Hence the correct choices are (a), (b) and (c).

III

Multiple Choice Questions Based on Passage

Questions 1 to 3 are based on the following passage


Passage I
Two light springs of force-constants k1 = 1.8 Nm–1 and k2
= 3.2 Nm–1 and a block of mass m = 200 g are arranged on
a horizontal frictionless table as shown in Fig. 13.52. One
Fig. 13.52
end is free. The distance CD between the free ends of 1. When the block moves towards the spring k2, the
the springs is 60 cm and the block moves with a velocity time taken by it to move from D upto the maximum
v = 120 cm s–1 between the springs. compression of spring k2 is (in seconds)
IIT, 1985
Simple Harmonic Motion 13.39

(a) (b) (c) (d)


2 3 4
3. The period of oscillation of the block between the
(c) (d) springs is (in second)
3 4
2. When the block moves towards spring k1, the time 5 7
(a) 1 (b) 1
taken by it to move from C upto the maximum com- 6 6
pression of k1 is (in seconds)
5 7
2 (c) 1 (d) 1
(a) (b) 12 12
3

SOLUTION
1. Time taken by block to move from D upto the maxi- 1
T1 1 m 0.2
mum compression of spring k2 = half the time pe- 2. Similarly t1 = = 2 = 2
riod of oscillation of the block if it were attached to 2 2 k1 2 1.8
the free end of k2, i.e. second =
3
T2 1 m 3. Time period of oscillation of block is
t2 = = 2
2 2 k2 T = time taken by the block to move freely from C
to D and from D to C = t1 + t2
1 0.2
= 2 = second 2 60 cm 7
2 3.2 4 = 1
+ = 1 ,
120 cms 3 4 12
The correct choice is (d). which is choice (d).

Questions 4 to 6 are based on the following passage (a) – Mgx (b) – (k + A g)x
Passage II 1 1
(c) = – k A g x (d) – k A g x
A uniform cylinder of length L and mass M having cross- 2 2
sectional area A is suspended, with its length vertical,
6. The time period T of the vertical oscillations of the
half submerged in a liquid of density at equilibrium cylinder is
position. When the cylinder is given a small downward M
push and released, it starts oscillating vertically with a (a) 2
small amplitude. k
IIT, 1990 M 1/ 2

4. The extension x0 of the spring when it is in equilib- (b) 2 1


rium is k A g
2
Mg g
(a) (b) (M – LA ) 1/ 2
k k M
(c) 2
g 1 g 1 (k A g )
(c) M LA (d) M LA
k 2 k 2 M 1/ 2
(d) 2
5. If the cylinder is given a small downward displace- k A g
ment x from the equilibrium position and released,
the restoring force F acting on it is
13.40 Comprehensive Physics—JEE Advanced

SOLUTION
4. The upthrust on the cylinder with half its length L
submerged in the liquid is given by is x A g and the force in the spring is
2
U = weight of the liquid displaced by k (x0 + x). Hence, the restoring force on the
a length L/2 of the cylinder cylinder is
L L L
=A g= (A g) F=– k ( x0 x) Mg x A g (2)
2 2 2
Let x0 be the extension of the spring when it is in Using Eq. (1) in Eq. (2), we have
equilibrium. Then
F = – (k x + A gx)
L
kx0 = Mg – (A g) (1) or F = – (k + A g)x
2
Thus the correct choice is (b)
The correct choice is (c).
6. The acceleration of the cylinder is
5. Let x be the small downward displacement given
to the cylinder so that the submerged length of F k A g
a= =– x (3)
L M M
the cylinder is now x and the extension Comparing Eq. (3) with a = – 2x, where =
2
2 /T
of the spring is now (x0 + x). The upthrust now

Questions 7 to 9 are based on the following passage


MA ( M m) A 1 / 2
Passage III (c) (d)
mA mA
One end of a light spring of force constant k 8. If v and v are the velocities before and after the
a block of mass M placed on a horizontal frictionless object is placed on the block, then the ratio v /v is

The spring-block system is executing simple harmonic M M m


(a) (b)
motion of amplitude A and frequency . When the block (M m) m
is passing through the equilibrium position, an object of M m A M m A
a mass m is gently placed on the block. As a result, the (c) (d)
M m A M m A
frequency of the system becomes and the amplitude
becomes A . 9. The ratio A /A is
1/ 2 1/ 2
7. The ratio / is M m Am
(a) (b)
1/ 2 1/ 2
m A ( M m)
M m
(a) (b) M 1/ 2
A ( M m)
1/ 2
M m M m (c) (d)
M m AM

SOLUTION

7. The frequency of the system before the object is 1/ 2


M
placed on the block is given by = ,
M m
1 k 1/ 2 which is choice (a)
=
2 M 8. From conservation of momentum, we have
After the object of mass m is placed on the block, Mv = (M + m)v
the new frequency of the system becomes
v M
1 k 1/ 2 or = .
= v ( M m)
2 M m
So the correct choice is (a).
Simple Harmonic Motion 13.41

9. From the principle of conservation of energy we 1/ 2


A v (M m)
know that the kinetic energy of the block when =
it is passing through the equilibrium position A v M
= potential energy of the spring when the displace- 1/ 2
ment is equal to the amplitude. M M m
=
Thus we have M m M
1 1 1/ 2
Mv2 = k A2 =
M
,
2 2 M m
1 1 which is choice (c).
and (M + m)v 2 = kA 2
2 2

Questions 10 to 12 are based on the following passage 3a


(c) 3a (d)
Passage IV 2
Three simple harmonic motions in the same direction 11. The phase of the resultant motion relative to the
having the same amplitude a and the same period are
superposed. Each motion differs from the preceding (a) 30° (b) 45°
motion by a phase = 45°. (c) 60° (d) 90°
IIT, 1999 12. If the energy associated with each motion is E, the
10. The amplitude of the resulting simple harmonic total energy of the resultant motion is
motion is (a) 3E (b) (1 + 2 )E
(a) (1 + 2 )a (b) (1 + 3 )a
(c) (3 + 2 )E (d) (3 + 2 2 )E

SOLUTION
10. x1 = a cos t a sin ( N / 2) ( N 1)
where R= and =
x2 = a cos ( t + ) sin ( / 2) 2
x3 = a cos ( t + 2 ) a sin (3 / 2)
For N = 3, R =
sin ( / 2)
where = 45°. From the principle of superposition,
the resultant motion is given by a (sin cos / 2 cos sin / 2
=
sin / 2
x = x1 + x2 + x3
= a (sin 45° cot 22.5° + cos 45°)
x = a cos t + a cos ( t + )+
a cos ( t + 2 ) = a ( 2 + 1)
1
It can be shown that the resultant displacement x for 11. = (3 – 1) 45° = 45°
N collinear simple harmonic motions of the same 2
amplitude a and differing in phase by is given R2
12. Total energy = energy of any one motion
by a2
x = R cos ( t + ) = ( 2 + 1)2E = (3 + 2 2 )E

Questions 13 to 15 are based on the following passage in studying the changes in motion as initial position and
Passage V momentum are changed. Here we consider some simple
dynamical systems in one-dimension. For such systems,
Phase space diagrams are useful tools in analyzing all
phase space is a plane in which position is plotted along
kinds of dynamical problems. They are especially useful
13.42 Comprehensive Physics—JEE Advanced

horizontal axis and momentum is plotted along vertical


axis. The phase space diagram is x(t) vs. p(t) curve in
this plane. The arrow on the curve indicates the time

moving with constant velocity is a straight line as shown


in the Fig. 13.53. We use the sign convention in which
position or momentum upwards (or to right) is positive
and downwards (or to left) is negative.
IIT, 2011

13. The phase space diagram for a ball thrown verti-


cally up from ground is

Fig. 13.54

15. Consider the spring-mass system, with the mass


submerged in water, as shown in Fig. 13.55. The
phase space diagram for one cycle of this system is

Fig. 13.55

Fig. 13.53

14. The phase space diagram for simple harmonic


motion is a circle centered at the origin. In Fig.
13.54, the two circles represent the same oscilla-
tor but for different initial condition, and E1 and
E2 are the total mechanical energies respectively.
Then
(a) E1 = 2 E2
(b) E1 = 2E2
(c) E1 = 4E2
(d) E1 = 16E2
Fig. 13.56

SOLUTION

13. According to the given sign convention, position when the body is moving upwards and becomes
(x) remains positive and momentum is positive zero when it reaches the highest point after which
Simple Harmonic Motion 13.43

the momentum (p) becomes negative. Hence the 2


E1 2a
correct graph is (d). = = 4. So E1 = 4E2
E2 a
14. Energy of simple harmonic oscillator is
1 2 15. Due to upthrust, the spring will be compressed.
E= kA
2
will be smaller than the initial position. Hence
where k is the force constant and A the amplitude
choices (c) and (d) are not possible. Due to
of the oscillator. Since the oscillator is the same,
buoyancy, the block will move upwards. Hence,
the value of k is the same. Hence
according to the given sign convention, posi-
1 1 tion (x) is positive initially. When the system is
E1 = kA12 and E2 = kA22
2 2 released, x will decrease and momentum (p) will
2 increase becoming maximum when the system
E1 A1
= reaches the mean position (x = 0) after which the
E2 A2 momentum will decrease to zero when the oscil-
Now A1 = maximum value of displacement of lator reaches the extreme position, after which
oscillator having energy E1 = 2a and A2 = a. the momentum becomes negative. Hence the
Therefore correct graph is (b).

IV

Matrix Match Type


1. Column I describes some situations in which a small object moves. Column II describes some characteristics
of these motions. Match the situations in Column I with the characteristics in Column II.
Column I Column II
(a) The object moves on the x- axis under a (p) The object executes a simple harmonic
conservative force in such a way that its “speed” motion
and “position” satisfy v = c1 c2 x 2 where c1
and c2 are positive constants.
(b) The object moves on the x-axis in such a way (q) The object does not change its direction
that its velocity and its displacement from the
origin satisfy v = – kx, where k is a positive
constant.
(c) The object is attached to one end of a mass- (r) The kinetic energy of the object keeps on
less spring of a given spring constant. The other decreasing.
end of the spring is attached to the ceiling of
an elevator. Initially everything is at rest. The
elevator starts going upwards with a constant
acceleration a. The motion of the object is
observed from the elevator during the period it
maintains this acceleration.
(d) The object is projected from the earth’s surface (s) The object can change its direction only
vertically upwards with a speed 2 GM e / Re , once.
where Me is the mass of the earth and Re is the
radius of the earth. Neglect forces from objects
other than the earth.
IIT, 2007
13.44 Comprehensive Physics—JEE Advanced

2. Column I gives a list of possible set of parameters measured in some experiments. The variations of the param-
eters in the form of graphs are shown in Column II. Match the set of parameters given in Column I with the
graphs given in Column II.
Column I Column II

(a) Potential energy of a simple pendulum (y axis) as (p)


a function of displacement (x axis)

(b) Displacement (y axis) as a function of time (q)


(x axis) for a one dimensional motion at zero or
constant acceleration when the body is moving
along the positive x-direction

(c) Range of a projectile (y axis) as a function of its (r)


velocity (x

(d) The square of the time peroid (y-axis) of a simple (s)


pendulum as a function of its length (x axis).

IIT, 2008

ANSWERS
1. (a) The velocity v of a body executing simple harmonic motion is related to displacement x as
v= A2 x2
where ands A are positive constants. The given equation v = c1 c2 x 2 is similar to the above equa-
tion. Hence the body executes simple harmonic, motion, which is choice (p).
(b) It follows from equation v = – kx that v = 0 at x = 0, i.e. object comes to rest at x = 0. Thus the object
starting from a negative x value comes to rest at x = 0; its velocity (and hence kinetic energy) decrease
with time. Hence correct choices are (q) and (r).
(c) For an observer in the reference frame of the elevator, a constant force acts on the object. This is a
pseudo force. Hence the motion of the object remains simple harmonic, which is choice (p).

(d) Since the speed of the object is 2 times the escape velocity ve 2GM e / Re , the direction of motion
of the object does not change (since the forces exerted by objects other than the earth are neglected) and
the speed of the object keeps decreasing. Hence the correct choices are (q) and (r).
(a) (p) (b) (q), (r) (c) (p) (d) (q), (r)
1
2. (a) The potential energy of a simple pendulum is given by U = kx2 where x is the displacement and k
2
is the force constant. U is maximum where x = + A, where A is the amplitude and minimum at x = 0.
Hence the correct graph is (p).
Simple Harmonic Motion 13.45

(b) For one-dimensional motion with constant velocity (a = 0), displacement S = vt (i.e. s t) and with
1 2 2
constant acceleration S = ut + at , i.e. S t . Hence the correct choices are (q) and (s).
2
(c) For a projectile, range R u2, where u is the speed of projection. Hence the correct graph is (s).
4 2l
(d) For a simple pendulum, T2 = . Hence the correct choice is (q).
g
(a) (p) (b) (q), (s) (c) (s) (d) (q)

Assertion-Reason Type Questions


In the following questions, Statement-1(Assertion) is Statement-2
followed by Statement-2 (Reason). Each question has the For a body executing simple harmonic motion, the
following four choices out of which only one choice is potential energy is proportional to the square of its
correct. displacement from the mean position.
(a) Statement-1 is true, Statement-2 is true and State- 4. Statement-1
ment-2 is the correct explanation for Statement-1.
The time period of a simple harmonic oscillator
(b) Statement-1 is true, Statement-2 is true but State-
depends upon its amplitude and force constant.
ment-2 is not the correct explanation for Statement-1
(c) Statement-1 is true, Statement-2 is false. Statement-2
(d) Statement-1 is false, Statement-2 is true. The frequency of a simple harmonic oscillator is
1. Statement-1 determined by elasticity and inertia.
If a spring of force constant k is cut into two equal 5. Statement-1
halves, the force constant of each half is 2k. The amplitude and phase constant of a particle in
Statement-2 SHM are determined from its initial displacement
When an elastic spring is extended by an amount x, and initial velocity.
1 Statement-2
the work done is k x 2.
2 The amplitude and phase constant of SHM depend
2. Statement-1 on the magnitude of the restoring force.
A particle executes simple harmonic motion be-
6. Statement-1
tween x = – A and x = + A. The time taken for it to
go from x = 0 to x = A/2 will be less than the time The displacement of a particle is given by
taken for it to go from x = A/2 to x = A. x = a sin (bt + c)
Statement-2 This equation suggests that the time period of mo-
In simple harmonic motion, the speed of the par- tion of 2 /b.
ticle is the maximum at the mean position x = 0 and Statement-2
decreases as it moves towards the extreme position The same value of x is obtained at t = t and at t = t
becoming zero at x = A. + 2 /b.
3. Statement-1 7. Statement-1
A body is executing simple harmonic motion. At The displacement of a simple harmonic oscillator
a displacement x, its potential energy is E1 and at is given by
a displacement y, its potential energy is E2. The x = A sin( t + )
potential energy at a displacement (x + y) is This equation suggests that the energy of the oscil-
E = E12 E22 . lator remains constant.
13.46 Comprehensive Physics—JEE Advanced

Statement-2 Statement-2
The same value of x is obtained after an interval For larger amplitude, the speed of the bob is greater
equal to the time period of the oscillator. when it passes through the mean position.
8. Statement-1 10. Statement-1
The time period of a simple pendulum is indepen- For an oscillating simple pendulum, the tension in
dent of the mass of the bob. the string is constant at T = mg for all positions of
Statement-2 the bob.
The restoring force does not depend on the mass of Statement-2
the bob. The tension in the string will not remain constant
9. Statement-1 at T = mg because the speed of the bob is different
For small amplitudes, the motion of a simple at different positions.
pendulum is simple harmonic of time period
T = 2 l / g . For larger amplitudes, the time period
is greater than 2 l /g .

SOLUTION

1. The correct choice is (b). The force required to 4. The correct choice is (d).
extend the spring by an amount x is given by 5. The correct choice is (c).
F = kx (1) 6. The correct choice is (a). The value of x at t = t +
If the spring cut into two equal halves, the same 2 /b is
force F will produce half the extension because the 2
extension is directly proportional to the length of x = a sin b t c
b
the spring. Hence
x = a sin[bt + 2 + c]
F =k x =k (2)
2 = a sin(bt + c) = x [ sin( + 2 ) = sin ]
From (1) and (2), we get k = 2k. 7. The correct choice is (a).
2. The correct choice is (a). 8. The correct choice is (c). The restoring force when
3. The correct choice is (d). the string makes an angle with the vertical is giv-
1 m 2 en by F = – mg sin , which depends upon m.
2
E1 = m x2 E1 = x = xk (1) 9. The correct choice is (b). Restoring force is F
2 2
= – mg sin or F = – mge where ge = g sin .
m 2 For small oscillations, is small so that sin
where k= (here is in radian) and the effective value of g is
2
ge . For larger oscillations g sin is less than g
1 2 2 because sin < . Hence T is greater than 2 l / g .
E2 = m y E2 = yk (2)
2 10. The correct choice is (d). The tension in the string
1 2 2 is given by
and E = m (x + y) E = (x + y)k (3)
2 mv2
T = mg cos +
From (1), (2) and (3), we get l

E = E1 E2 where l = length of the pendulum, v its speed when


the string makes an angle with the vertical.
E = E1 + E2 + 2 E1 E2
Simple Harmonic Motion 13.47

VI

Integer Answer Type

1. A mass M attached to a spring oscillates with a 3. A block is kept on a horizontal table. The table is
period of 1s. If the mass is increased by 3 kg, the undergoing simple harmonic motion of frequency
period increases by 1s. Find the value of M (in kg)
assuming that Hookes’ law is obeyed. friction between the block and the table is 0.72.
Find the maximum amplitude in cm of the table for
IIT, 1979
which the block does not slip on the surface of the
2. An object of mass 0.2 kg executes simple har-
table. Take g = 10 ms–2.
monic motion along the x-axis with a frequency of
25/ Hz. At the position x = 0.04 m, the object IIT, 1996
has kinetic energy of 0.5 J and potential energy of 4. A particle executes simple harmonic motion be-
0.4 J. Find the amplitude of oscillations in cm. tween x = – A and x = + A. It takes time t1 to go
from 0 to A/2 and t2 to go from A/2 to A. Find the
IIT, 1994
ratio T2/T1.
IIT, 2001
SOLUTION

M M 3 the table, if the acceleration a of the motion of the


1. 1 = 2 and 2 = 2 table is such that [Fig. 13.57]
k k
mg ma or g a
Dividing these equations we get
M 3
4= M = 1 kg
M
25
2. Angular frequency = 2 = 2 = 50 rad s–1
1
Kinetic energy K = m 2 (A2 – x2)
2
Fig. 13.57
1 2 2
Potential energy U = m x Therefore, for no slipping, the table can have a max-
Dividing we get 2 imum acceleration amax = g. Where amax = 2A.
K A2 x 2 Therefore, the maximum amplitude is given by
= 2
Amax = mg
U x2
0.5 A2 x 2 or Amax =
g
=
g
( =2 )
= 2 2 2
0.4 x2 4
3x 3 0.04 m 0.72 10
A= = = 2 = 0.02 m = 2 cm
2 2 4 (3)2
= 0.06 m = 6 cm 4. From x = A sin t, we have
3. Let the mass of the block be m and let, at a cer- A
tain instant of time, the direction of acceleration = A sin t1 t1 = (1)
2 6
a of the table (executing simple harmonic motion) Also A = A sin (t1 + t2)
be along the positive x-direction. As a result, the
block will experience a force ma directed along the (t1 + t2) = (2)
negative x-axis. Consequently, the force of friction 2
mg will act along the positive x-axis. The weight t1 1
From (1) and (2), we get = which gives
mg of the block will be balanced by the normal t2 t1 t2 3
reaction R. The block will not slip on the surface of =2
t1
14
Chapter

REVIEW OF BASIC CONCEPTS of wave) between two nearest particles which are
in the same phase of vibration.
14.1 WAVE MOTION
DISPLACEMENT EQUATION FOR A
Wave motion involves the transport of energy without 14.4
any transport of matter. In case of mechanical waves, the TRAVELLING WAVE
disturbance is the physical displacement of particles of When a plane wave travels in a medium along the positive
a medium. In case of electromagnetic waves, the distur- x-direction, the displacement y of a particle located at x at
time t is given by
14.2 TYPES OF WAVES y = A sin ( t – kx)
where A = amplitude of the wave, (= 2 ) is the
There are two types of wave motions: (1) transverse and
angular frequency (in rad s–1), is the frequency
(2) longitudinal.
2
(1) Transverse Waves In transverse waves the parti- (in Hz) and k = is the angular wave number and
cles of the medium vibrate at right angles to the
is the wavelength of the wave.
direction in which the wave propagates. Waves on
For a wave travelling along the negative x-direction,
strings, surface water waves and electromagnetic
waves are transverse waves. y = A sin ( t + kx)
(2) Longitudinal Waves In longitudinal waves the par- The wave velocity is given by
ticles of the medium vibrate along the direction of
wave propagation. Sound waves are longitudinal. v= =
k
CHARACTERISTICS OF A HARMONIC
14.3 WAVE 14.5 PHASE AND PHASE DIFFERENCE

(1) Amplitude The amplitude of a wave is the maxi- The argument of the sine (or cosine) function which
mum displacement of the particles of the medium represents a wave is called the phase of the wave. For a
from their mean position. wave travelling along the positive x-direction, the phase
(2) Period The time period of a wave is the period of at a space point x at time t is given by
harmonic oscillations of particles of the medium. = t – kx
The frequency of a wave is the reciprocal of the It is clear that the phase changes with time t as well as
time period. space point x.
(3) Wave Velocity Wave velocity is the distance trav-
elled by the wave in one second. Phase Change with Time
(4) Wavelength - The phase of a given particle (i.e. x
tance (measured along the direction of propagation time. As time changes from t to (t + t), the phase of a
14.2 Comprehensive Physics—JEE Advanced

particle oscillation changes from to ( + ) where 2 2


is given by (c) = x= 50 = – = – 180°
100
= { (t + t) – kx} – { t – kx}
The negative sign indicates that the particle at
2 x = 50 m lags behind the particle at x = 0 by a
= t= t phase angle of 180°.
T
where T is the time period of particle oscillation. If 14.2
t = T, = 2 .
A travelling wave on a string is given by
Phase Change with Position
At a given instant of time t, the phase of particles of the y = 7.5 sin 0.005 x 12t
4
medium varies with position x of the particles. The phase
difference at an instant t between two particles separated where x and y are in cm and t in second.
by x and (x + x) is given by (a) Find the displacement and velocity of the par-
ticle at a point x = 1 cm at t = 1s. Is this equal to
= { t – k(x + x)} – ( t – kx) the wave velocity? Given sin (12.7°) = 0.22.
2 (b) Locate the points on the string which have the
= –k x= x same transverse displacement and velocity as
the x = 1 cm point has at t = 2s, 5s and 11s.
The minus sign indicates that, for a wave travelling along
the positive x-direction, the particles located at higher SOLUTION
values of x lag behind in phase. If x = , | |=2 .
Hence Given y = 7.5 sin 0.005 x 12t (i)
two particles whose phases differ by 2 . 4
Putting x = 1 cm and t = 1s in Eq. (i) we have
14.1
3.142
When a plane wave travels in a medium, the displace- y = 7.5 sin 0.005 1 12 1
ments of particles are given by 4
= 7.5 sin (12.79 rad)
y = 0.01 sin [2 (2t – 0.01x)]
= 7.5 sin (732.7°)
where x and y are in metre and t in second. Find ( rad = 180°)
(a) the amplitude, wavelength, wave velocity and
= 7.5 sin (4 + 12.7°)
frequency of the wave,
(b) the phase difference between two positions of = 7.5 sin (12.7°)
the same particle in a time interval of 0.25s and = 7.5 0.22 = 1.65 cm
(c) the phase difference at a given instant of time Particle velocity is obtained by differentiating (i)
between two particles 50 m apart. w.r.t. time t.
dy
SOLUTION V= = 7.5 12 cos 0.005 x 12t
dt 4
(a) The given equation is Putting x = 1 cm and t = 1s,
y = 0.01 sin (4 t – 0.01 2 x) V = 7.5 12 cos 12.7°
Comparing this equation with = 87.8 cm s–1
y = A sin ( t – kx), we get
Comparing Eq. (i) with
A = 0.01 m
y = A sin (kx + t+ 0)
=4 2 =4 = 2 Hz we get k = 0.005 rad cm and = 12 rad s–1 giving
–1

2 2 12
k = 0.01 2 = v= = = 2400 cm s–1
100 k 0.005
= 100 m Particle velocity V is not equal to wave velocity v.
–1 ), all points on
v= =2 100 = 200 ms
the string which are located at x = , 2 , ...
(b) =2 t=2 2 (0.25) = = 180°
Waves and Doppler’s Effect 14.3

= n (n = 0, 1, 2, ...) away from x = 1 cm have (c) For liquids: E = B, the bulk modulus of the
the same displacement and velocity as those at liquid. Thus
x = 1 cm for all values of t. B
2 v=
From k = 0.005 = 0.005 = 12.67 cm.
2. The velocity of sound in a gas is independent of the
Hence the positions of the required points are
pressure but is directly proportional to the square
x = 1 cm + (12.67 cm) n = (1 + 12.67 n) cm where n
root of the absolute temperature.
is an integer.
vt T t 273 1 / 2
= =
14.3 v0 T0 273
A tuning fork vibrating at 200 Hz produces a sound where t is the temperature in °C.
wave which travels in air at a speed 340 ms–1. Find 3. The velocity of sound increases with increase in
the distance travelled by sound during the time the humidity. Sound travels faster in moist air than in
fork makes 60 vibrations. dry air at the same temperature.
4. Velocity of a transverse wave on a stretched string
SOLUTION is given by
= 200 Hz, v = 340 ms–1 T
v=
v 340 m
= = = 1.7 m
200 where T = tension in the string and m = mass per
) is the distance travelled unit length of the string. For a string of diameter d
by the wave in 1 complete vibration (= time period) and density , we have
of the fork. Hence d2 2 T
m = Thus, v =
x = 60 1.7 = 102 m 4 d

EXPRESSIONS FOR WAVE VELOCITY IN 14.4


14.6 DIFFERENT MEDIA Uniform rope AB of mass M and length L hangs verti-
cally with end A
1. Velocity of sound in an elastic medium is given is created at the free end A. The speed of the pulse is
by (a) the same at every point of the rope
(b) maximum near end A
E (c) maximum near end B
v=
(d) zero at the mid-point of the rope.
where E = modulus of elasticity of the medium
and SOLUTION
= density of the medium. -
(a) For gases: ent at different points on the rope. Hence the speed of
E= P the pulse is different at different points.
where g = Cp/Cv M
Mass per unit length is m = .
of the gas at constant pressure and that at constant L
volume and P is the pressure of the gas. Tension at point P (Fig. 14.1) is
Thus Mgx
T= = mgx
P L x
v=
Speed of the pulse is
(b) For solids: E = Y; the Young’s modulus of
T mgx Fig. 14.1
the solid. Thus v= gx
m m
Y
v= i.e. v x . Hence the correct choice is (b).
The speed of the pulse is
14.4 Comprehensive Physics—JEE Advanced

3
14.5 A vA dB 0.5 10
Hence = 3
= 0.5
A long uniform steel wire has a diameter of 2.0 mm. B vB dA 10
What should be the tension in the wire, so that the
speed of the transverse wave on it equals the speed 14.7
of sound at STP (= 320 m s–1)? The density of steel Compare the velocities of sound in hydrogen (H2) and
is 7800 kg m–3. carbon dioxide (CO2). The ratio (
of H2 and CO2 is respectively 1.4 and 1.3.
SOLUTION
The volume of a wire of length L and diameter d is SOLUTION
V = r2L = d2L/4
The mass of the wire is 1P 2P
v1 = and v2 =
d 2L 1 2
M = volume density = v1
4 = 1 2
2 v2 2 1
M d
Mass per unit length (m) = =
L 4 Since density of a gas is proportional to its molecular
T weight,
Now v=
m 2
= 44.01 = 21.83
d 2 v2 1 2.016
T = mv2 =
4 v1 1.4 1/2

Substituting the values of d, and v and solving we = 21.83 = 4.85


v2 1.3
get
T = 25.1 N Velocity of sound in hydrogen is 4.85 times that in
carbon dioxide.
14.6
14.8
Transverse waves are generated in two uniform steel
wires A and B of diameters 10–3 m and 0.5 10–3 m At what temperature will sound travel in hydrogen
respectively, by attaching their free end to a vibrat- with the same speed as in oxygen at 927°C?
ing source of frequency 500 Hz. Find the ratio of
the wavelengths if they are stretched with the same SOLUTION
tension.
v1 1 2
=
SOLUTION v2 2 1
The density of a wire of mass M, length L and diam- Hydrogen and oxygen are both diatomic gases. Hence,
eter d is given by 1 = 2. Also
4M 4m 2 molecular mass of oxygen
= 2
= =
d L d2 1 molecular mass of hydrogen
T 32
now vA = = = 16
mA 2
v1
T = 16 = 4
and vB = v2
mB
1/2
vt t 273
vA mB d =
= = B v0 273
vB mA dA
1/2 1/2
but vA = and vB = vt 927 273 1200
A B, being the frequency of the For oxygen = = (i)
source. v2 273 273
Waves and Doppler’s Effect 14.5

1/2 to the algebraic sum of the individual displacements. This


vt 273 t
For hydrogen = (ii) is called the principle of superposition.
v1 273
y = y1 + y2 + ... + yn
Dividing (i) and (ii)
The following three cases of superposition are of practical
vt v1 1200
1/2 importance.
=
vt v2 273 t
14.8 INTERFERENCE
1/2
vt 1200 The superposition of two waves of the same frequency
4=
vt 273 t travelling in the same direction in a medium is called
Given v t = vt. Hence interference. Consider two waves of equal amplitude a,
equal angular frequency and and equal angular wave
1/2
1200 number k but having a phase difference travelling along
4=
273 t the positive x-direction. The displacements y1 and y2 of a
particle located at x at time t are
1200 y1 = a sin ( t – kx)
16 =
273 t and y2 = a sin ( t – kx + )
t = – 198°C According to the superposition principle, the resultant
displacement is given by
14.9 y = y1 + y2
In a laboratory experiment (room temperature being = a [sin ( t – kx) + sin ( t – kx + )]
15°C) the wavelength of a note of sound of frequency
500 Hz is found to be 0.68 m. If the density of air at Using sin + sin = 2 sin cos , we
get 2 2
STP is 1.29 kg m–3
heats of air. y = 2a cos sin t kx
2 2
SOLUTION
y = A sin t kx
Speed of sound at 15°C is v1 = = 500 0.68 2
= 340 ms–1
273 where A = 2a cos is the resultant amplitude. The
Speed of sound at 0°C is v0 = 340 2
273 15 frequency and wavelength of the resultant wave remain
= 331 ms–1
–3 the same as those of individual waves.
At STP, 0 = 1.29 kgm and P0 = 1.01 105 N m–2
(a) Constructive Interference: If A is maximum (positive
P0 Cp or maximum), the interference is constructive, the Amax =
Now v0 = ; = 2a. This happens if
0 Cv
cos = 1
v2 (331) 2 1.29 2
= 0 0 = = 1.39
P0 1.01 105 = 0, , 2 , ...
2
SUPERPOSITION OF WAVES (THE = 2n ; (n = 0, 1, 2, ...)
14.7 SUPERPOSITION PRINCIPLE) (b) Destructive Interference: If A = 0, the interference is
destructive. This happens if
When a wave reaches a particle of a medium, it imparts a
displacement to that particle. If two or more waves arrive cos =0
at a particle, the resultant displacement of the particle is 2
equal to the vector sum of individual displacements. In 3 5
= ,, ,
the particular case when the waves travelling in the same 2 2 2 2
straight line superpose, the resultant displacement is equal = (2n + 1) ; n = 0, 1, 2, ...
14.6 Comprehensive Physics—JEE Advanced

Equation (i) represents a standing wave. It does not


14.10
represent a travelling wave since it does not involve the
Two waves each of amplitude 2 cm and wavelength combination ( t kx) in the argument of the sine or
5 cm and frequency 10 Hz have a constant phase dif- cosine function. Equation (ii) tells us each particle has
ference of 60°. Travelling in the same direction, they a simple harmonic motion and Eq. (iii) tells us that the
superpose at a particle of the medium. What is the amplitude of motion is different for different particles (i.e.
resultant amplitude of the oscillations of the particle? for different values of x). Such simple harmonic motions
- of the particles of a medium are called normal modes.
tant wave.
Nodes
SOLUTION There are certain points in the medium which are
Resultant amplitude is A = 2a cos permanently at rest. These points are called nodes. The
2 position of nodes is given by
60
=2 (2 cm) cos A =0
2
= 2 3 cm 2a sin (kx) = 0
The frequency and wavelength of the resultant wave sin kx = 0
are 10 Hz and 5 cm respectively, the same as those of kx = 0, , 2 , ...
the individual interfering waves.
2
x = 0, , 2 , ...
14.9 STANDING (OR STATIONARY) WAVES
Standing (or stationary) waves are produced when two , , ... and so on
x = 0,
2
waves of the same frequency travelling in opposite
directions in a medium superpose. In actual practice, Distance between two consecutive nodes = .
2
we do not send two independent waves in a medium in
Antinodes
There are certain points in the medium which have
length or a rod or a column of gas or liquid. The wave maximum (positive or negative) amplitude. These points
are called antinodes. Their position is given by
A= 1
rise to standing waves.
2a sin (kx) = 1
undergoes a reversal of amplitude (which implies a phase sin kx = 1
change of ). Consider a wave travelling in the negative
3 5
x-direction towards a boundary at x = 0, where it is kx = , , ,
2 2 2
2 3 5
x= , , ,
yi = a sin ( t + kx) 2 2 2
and yr = – a sin ( t – kx) 3 5
x= , , ,
4 4 4
From the superposition principle, the resultant displacement
is Distance between two consecutive antinodes = .
2
y = yi + yr
NOTE
= a [sin ( t + kx) – sin ( t – kx)] 1. Exactly mid-way between two nodes is an antinode and
vice versa.
Using sin – sin = 2 sin cos , we get 2. The distance between a node and the next antinode
2 2
y = 2a sin (kx) cos ( t) (i) = .
4
y = A cos t (ii) 3. There is no transfer of energy along the medium.
where A = 2a sin(kx) (iii)
Waves and Doppler’s Effect 14.7

The frequency of the fundamental mode is


14.11
v v
Standing waves are produced by the superposition of 1 = =
two waves 2L
y1 = 5 sin (3 t – 2 x)
T
and y2 = 5 sin (3 t + 2 x) where v=
m
where y and x are in cm and t in second. Find the
(b) Second Harmonic. [Fig. 14.2 (b)]
amplitude of the particle at x = 2 cm.
For the second harmonic, =L = L.
SOLUTION 2 2
y = y1 + y2 v v
2 = = =2 1
= 5 [sin (3 t – 2 x) + sin (3 t + 2 x)] L
(c) Third Harmonic. [Fig.14.2(c)]
Using sin ( + ) + sin ( – ) = 2 sin cos , we
get 2L
For the third harmonic, =L =
y = 10 cos (2 x) sin (3 t) 2 2 2 3
y = A sin (3 t), where A = 10 cos (2 x) v 3v
3 = = =3 1
Amplitude A at x = 2 cm is 2L
10 cos (2 2) = 10 cos 4 = 10 cm In general, for a string vibrating in the nth harmonic,
the frequency of vibration is
Normal Modes of a String Fixed at Both Ends
nv n T
Consider a uniform string of length L stretched with a n = =
tension T x = 0 and x = L. 2L 2L m
The string can vibrate in a number of modes. Figure 14.2
14.12

ends is given by
2 x
y = 0.06 sin cos (120 t) (i)
3
where y and x are in metre and t in second. The length
of the string is 1.5 m and its mass is 3 10–2 kg.
(a) What is the amplitude at point x = 0.5 m?
(b) What is the velocity of the particle at x = 0.75 m
at t = 0.25 s?
(c) Write down the equation of the component
waves whose superposition gives the vibration
given in Eq. (i) above. What is the wavelength,
frequency and speed of each wave?
(d) Determine the tension in the string.
(e) Do all points on the string vibrate with the same
(i) frequency, (ii) phase and (iii) amplitude?

SOLUTION
(a) Displacement is maximum when cos (120 t) = 1.
Fig. 14.2 2 x
Hence the amplitude = 0.06 sin .
(a) Fundamental Mode (or First Harmonic) 3
In this mode, the string vibrates in one segment. 2
[Fig. 14.2 (a)] At x = 0.5 m, the amplitude = 0.06 sin 0.5
3
=L = 2L = 0.06 sin = 0.052 m
2 3
14.8 Comprehensive Physics—JEE Advanced

(b) The velocity at point x at time t is obtained by


T
differentiating Eq. (i) with respect to t. We know that v = , where T is tension in the
string. m
dy
V= T = mv2 = 2.0 10–2 (180)2 = 648 N
dt
2 x (e) In a stationary wave on a string, all points on
= – (0.06 120 ) sin sin(120 t) the string vibrate with the same frequency and
3
the same phase, but the amplitude is different at
Now at t = 0.25s, sin(120 t) = sin (30 ) = 0. different points (i.e. different values of x). The
Hence at t = 0.25s, the velocity is zero for all amplitude is zero at nodes and maximum at an-
values of x including x = 0.75 m. tinodes.
(c) The stationary wave in the question may be writ-
ten as 14.13
y = 2A sin cos A wire of density 9 g cm–3 is stretched between two
2 x clamps 100 cm apart while being subjected to an
where A = 0.03 m, = and = 120 t. extension of 0.05 cm.. What is the lowest frequency
3
of transverse vibrations of the wire, assuming that
Now 2A sin cos = A sin( + ) + A sin( – ) the Young’s modulus of the material of the wire
2 x = 0.9 1011 N m–2.
= 0.03 sin 120 t
3 SOLUTION
2 x Young’s modulus
+ 0.03 sin 120 t
3 stress T /A TL
i.e. y = y1 + y2 Y= = =
strain l/L Al
Hence the two component waves are
where T = tension, L = original length = 100 cm,
2 x l = extension = 0.05 cm
y1 = 0.03 sin 120 t
3 and A = area of cross-section of the wire.
2 x T Yl 0.9 1011 0.05
and y2 = 0.03 sin 120 t Hence, = =
3 A L 100
2 x = 4.5 107
= – 0.03 sin 120 t
3 Now mass per unit length
Let be the wavelength, the frequency and mass
m=
v the speed of each wave. Then length
2 volume density
x in the argument =
2 length
of the sine function =
3 area length density
or = 3 m. =
length
Also =2 t in the = area density
argument of the sine function =A
= 120 Density = 9 g cm–3 = 9000 kg m–3. The lowest fre-
quency is the frequency of the fundamental mode.
which gives = 60 Hz.
1 T 1 T
Hence v= = 60 3 = 180 m s–1 1 = =
2L m 2L A
3.0 10 2 1 4.5 107
(d) Mass per unit length (m) = =
1.5 2 1 9000
= 2.0 10–2 kg m–1 = 35.3 Hz
Waves and Doppler’s Effect 14.9

(a) Fundamental Mode (or First Harmonic) [Fig.


14.14
14.3 (a)]
A wire having a linear density of 0.05 g cm –1 is
stretched between two rigid supports with a tension =L L=4
4
of 4.5 102 N. It is observed that the wire resonates
v v
at a frequency of 420 Hz. The next higher frequency 1 = =
at which the wire resonates is 490 Hz. Determine the 4L
length of the wire. P
where v = is the speed of sound in the
SOLUTION gas.
Let 420 Hz be the nth harmonic, then 490 Hz is the (b) Third Harmonic (or First Overtone) [Fig.
(n + 1)th harmonic. Therefore 14.3 (b)]
4L
n T =L =
420 = (i) 2 4 3
2L m
v 3v
(n 1) T 3 = = =3 1
and 490 = (ii) 4L
2L m
(c) Fifth Harmonic (or Second Overtone) [Fig.
Dividing (i) and (ii) we get n = 6. Putting n = 6 in (i) 14.3 (c)]
we get
4L
6 4.5 102 =L =
420 = L = 2.14 m 2 2 4 5
2 L 0.05 10 1 v 5v
5 = = =5 1
Normal Modes in Air Columns in a Pipe 4L
A gas column in a pipe can oscillate in a number of In general, for nth harmonic
modes
nv n P
Case 1: Closed Pipe n = =
Consider a pipe of length L open at one end and closed 4L 4L
at the other. The closed end is a node and the open end is where n = 1, 3, 5, ... etc.

closed pipe. NOTE


In a closed pipe only odd harmonics are present; all the
even harmonics are absent.
Case 2: Open Pipe

Fig. 14.3
14.10 Comprehensive Physics—JEE Advanced

14.15
A pipe of length 20 cm is closed at one end. Which
harmonic mode of the pipe is resonantly excited by a
430 Hz source? Will the same source be in resonance
with the pipe if both ends are open? Speed of sound
= 340 ms–1.
Fig. 14.4
SOLUTION
(a) Fundamental Mode (or First Harmonic) [Fig.
14.4 (a)] Let N be the frequency of the source and m that of
the mth harmonic of the closed pipe, where m = 1, 3,
=L = 2L 5, .... Resonance will occur if
4 4 mv
v v N= m=
1 = = 4L
2L
(b) Second Harmonic (or First Overtone) [Fig. m 340
430 =
14.4 (b)] 4 0.2

=L =L which gives m = 1.01 1. The source of frequency


4 2 4
v v fundamental mode) of the closed pipe.
2 = = =2 1
2L For an open pipe, the condition of resonance with the
same source will be
(c) Third Harmonic (or Second Overtone) [Fig.
14.4 (c)] nv
N= n= , where n = 1, 2, 3, ...
2L 2L
=L = N 2L 430 2 0.2
4 2 2 4 3 n= = = 0.5
v 3v v 340
3 = = =3 1 which is not an integer. Hence the source will not be
2L
For nth harmonic in resonance with any harmonic of the open pipe.

n = n 1; n = 1, 2, 3, ...
14.16
NOTE A half-metre long tube open at one end, with a mov-
In an open pipe, all harmonics (even as well as odd) are able piston shows resonance with a tuning fork of
present. frequency 512 Hz when the tube length is 16.0 cm
and 49.0 cm. Calculate the speed of sound at the tem-
End Correction perature of the experiment and determine the end-
We have taken the open end of a pipe to be an antinode. correction.
This is not strictly true. In fact, the particles of air just at the
SOLUTION
open end are not perfectly free because of the restriction
imposed by the pipe. The true antinode is slightly away The tuning fork is in resonance at two lengths of the
from the open end as shown in Fig. 14.5. pipe, viz. 16 cm and 49 cm. Since the second reso-

(i.e. the fundamental mode) when L1 = 16 cm and


with the third harmonic when L2 = 49 cm. Thus we
have
Fig. 14.5 v
N= 1= (i)
4( L1 e)
The distance e is called the end correction. The effective
length of the pipe is (L + e). 3v
Also N= 3 = (ii)
4( L2 e)
Waves and Doppler’s Effect 14.11

where N is the frequency of tuning fork and e is the Hence the frequency of beats is
end-correction. Dividing (ii) by (i) we get b = 1 – 2
3(16.0 + e) = 49.0 + e or e = 0.5 cm Thus
Then the speed of sound as obtained from (i) is Beat frequency = difference between the frequencies of
v = 4N(L1 + e) interfering waves.
= 4 512 (16.0 + 0.5)
= 33792 cm s–1 = 337.92 m s–1 14.17
338 m s–1 Two tuning forks A and B produce 10 beats per sec-
ond when sounded together. On loading fork A with
14.10 BEATS a little wax it is observed that 5 beats per second are
produced. If the frequency of fork B
The periodic rise and fall of intensity of the wave the frequency of fork A (a) before loading and (b)
resulting from the superposition of two waves of different after loading.
frequencies is called the phenomenon of beats.
Consider two waves of angular frequencies 1 and 2. SOLUTION
For simplicity, we assume that they have equal amplitude There are two possibilities (i) A < B or
a and that the observation point is at x = 0. Then
(ii) A > B.
y1 = a sin 1t; 1 = 2 1
Case (i) A < B ; B – A = b 480 – A = 10
and y2 = a sin 2t; 2 = 2 2
A = 470 Hz.
Using the superposition principle,
On loading with a little wax, the frequency of a fork
y = y1 + y2 decreases slightly, i.e. A becomes slightly less than
= a (sin 1t + sin 2t)
470 Hz. Hence the number of beats per second must
increase. But b decreases to 5. Hence A cannot be
= 2a cos 1 2
t sin 1 2
t less than B.
2 2 Case (ii) A > B. In this case A = B + b = 480 + 10
1 = 490 Hz. On loading A, A decreases. Hence b = A
y = A sin( av t); av = ( 1 + 2)
2 – B will decrease. Since b is observed to decrease
1 2 to 5, A must be greater than B.
where A = 2a cos t
2 (a) Hence before loading, A = 490 Hz
(b) After loading A = 490 – 5 = 485 Hz
Now, intensity is proportional to A2. Therefore, the
resultant intensity is maximum when
14.18
1 2
cos t= 1 A metal wire of diameter 1.5 mm is held on two knife
2
edges separated by a distance of 50 cm. The tension in
1 2
t = 0, , 2 , ... the wire is 100 N. The wire vibrating with its funda-
2 mental frequency and a vibrating tuning fork together
1 2
2 t = 0, , 2 , ... produce 5 beats per second. The tension in the wire is
2
then reduced to 81 N. When the two are excited, beats
1 2 are heard at the same rate. Calculate (a) the frequency
t = 0, , ,
( 1 2) ( 1 2) of the fork, and (b) the density of the material of the
wire.
The time interval between two consecutive maxima is
1 SOLUTION
tb =
1 2
(a) Let N be the frequency of the tuning fork. Then,
Therefore frequency of maxima is the frequency of the wire, when the tension is
1 100 N will be (N + 5) and when the tension is
b = = 1 – 2 81 N, it is (N – 5); since in each case 5 beats are
tb
heard per second. Hence
Similarly, we can show that the frequency of minima = b.
14.12 Comprehensive Physics—JEE Advanced

6. Both receding from each other


1 T1 1 100 10
N+5= = = v u0
2L m 2 0.5 m m =
6
(i) v us
7. Source approaching a receding observer
1 T2 1 81 9
and N – 5 = = = v u0
2L m 2 0.5 m m 7 =
v us
(ii)
8. Observer approaching a receding source
Subtracting (ii) from (i) we have
1 v u0
10 = or m = 0.01 kg m–1 8 =
v us
m
When the source of sound goes pass stationary
Using this values of m in (i) or (ii) gives N = 95 Hz. observer, the apparent change in the frequency of
d2 sound is given by
(b) Now m = r2 =
4 2 vus
=
Putting d = 1.5 10–3 m and m = 0.01 kg m–1, v 2 us2
we get
2us
= 5.7 103 kg m–3 If us << v, then v2 – u2s v2, then =
v
When the observer goes past a stationary source of sound,
14.11 DOPPLER EFFECT IN SOUND the apparent change in frequency of sound is given by
The apparent change in frequency of sound heard by an 2 u0
=
observer due to a relative motion between the observer v
and the source of sound is called the Doppler effect.
NOTE
The expressions for the apparent frequencies are as
follows: (1) If the source of sound moves, the apparent change in
1. Source approaching a stationary observer frequency is due to change in wavelength; the speed of
sound remaining the same.
v (2) If the observer moves, the apparent change in frequency
1 = ,
v us is due to change in the speed of sound relative to ob-
where = real frequency server; the wavelength of sound remaining the same.
v = velocity of sound
Effect of the Motion of the Medium The velocity of
us = velocity of source
material or mechanical waves is affected by the motion
2. Source receding from a stationary observer of the medium. If the medium is moving with a velocity
v um in the direction of propagation of sound, the effective
2 = velocity of sound is increased from v to (v + um). In
v + us
3. Observer approaching a stationary source of this case, v is replaced by (v + um) in the above expres-
sound sions. On the other hand, if the medium is moving with
a velocity um in a direction opposite to the direction of
v + u0
3 = , wave propagation, v is replaced by (v – um).
v
where u0 = velocity of observer
14.12 DOPPLER EFFECT IN LIGHT
4. Observer receding from a stationary source of
sound If a star emitting light of frequency goes away from the
v u0 earth with a speed v, the apparent frequency of the light
4 = reaching the earth is given by
v
5. Both approaching each other c
=
v u0 c v
5 = where c is the speed of light. Since = c/ and = c/ ,
v us
we have
Waves and Doppler’s Effect 14.13

c c c 14.20
=
c v A train standing at a platform is blowing a whis-
v v tle of frequency 500 Hz in still air. The speed of
or =1+ or = sound in still air is 340 m s–1. (i) What is the fre-
c c
quency of the sound of the whistle heard by a man
v on a trolley which is moving (a) towards the en-
or =
c gine with a speed of 20 m s–1 and (b) away from
The apparent change in wavelength is called the the engine with a speed of 20 m s–1? (ii) What is
Doppler shift. the speed of sound heard by the man in each case?
a moving object decreases, the object is moving towards (iii) What is the wavelength of sound heard by the
the observer and vice versa. The wavelength of light man in each case?

called the red shift which indicates that the galaxy is SOLUTION
receding from us. The red shift indicates that the universe = 500 Hz, u0 = 20 m s–1, v = 340 m s–1
is expanding.
(i) (a) Observer approaching

14.19 (v u0 ) 500 (340 20)


2 = =
A train standing at the outer signal of a railway station v 340
is blowing a whistle of frequency 500 Hz. The speed = 529 Hz
of sound is 340 m s–1.
(b) Observer receding
(i) Find the frequency of the sound of the whistle
heard by a man standing on the platform when (v u0 ) 500 (340 20)
2 = =
the train (a) approaches the platform with a v 340
speed of 20 ms–1 and (b) recedes from the plat- = 471 Hz
form with a speed of 20 ms–1.
(ii) What is the speed of sound in each case? (ii) Since the observer is moving, the speed of sound
(iii) What is the wavelength of sound heard by the heard by him in case (a) is (v + u0) = 340 + 20 =
man in each case? 360 m s–1 and case (b) it is (v – u0) = 340 – 20 =
320 m s–1,
SOLUTION speed of sound relative observer
–1 –1 (iii) Wavelength =
= 500 Hz, us = 20 m s , v = 340 m s frequency of sound heard byy him

(i) (a) Train approaching v u0 360


In case (a) 2 = = = 0.68 m
v 500 340 2 529
1 = = = 531 Hz
v us 340 20
v u0 320
(b) Train receding In case (b) 2 = = = 0.68 m
2 471
v 500 340
1 = = = 472 Hz Notice that 2 = 2.
v us 340 20
(ii) Since the observer is at rest, the speed of sound
relative to him is v = 340 m s–1 in cases (a) and 14.21
(b). An ambulance blowing a siren of frequency 700 Hz
(iii) Now, wavelength
speed of 2 ms–1. If the speed of sound is 350 m s–1,
speed of sound relative to observer calculate the number of beats heard in one second
=
frequency of sound heardd by him by
v 340 (a) the driver of the ambulance,
In case (a) 1= = = 0.64 m
1 531 (b) a person standing between the ambulance and
v 340 the wall, and
In case (b) 1= = = 0.72 m (c) a person standing behind the ambulance.
1 472
14.14 Comprehensive Physics—JEE Advanced

(c) A person standing at B will hear two sounds, one


SOLUTION
from the siren receding from him and the other
(a) The driver will hear two sounds, one coming from the mirror image of the siren approaching
him with the same speed.
the wall or coming from the acoustic image of
the car. = frequency of direct sound = 700 Hz. v 350
1 = = 700
- v us 350 2
ing from the mirror image (shown dotted in Fig. = 696 Hz
14.6).
v 350
2 = = 700
v us 350 2
= 704 Hz
Beat frequency = 2 – 1 = 704 – 696 = 8 Hz

INTENSITY, QUALITY AND PITCH OF


14.13 SOUND

1. Intensity The intensity of a travelling wave is

per unit area of a surface held perpendicular to the


direction of propagation. Intensity I of a wave in a
medium of density is given by
Fig. 14.6
I = 2 2 v A2 2
The observer (driver) is approaching this where is the frequency of particle oscillation, A
image-source which is also approaching him their amplitude and v is the velocity of the wave in
with the same speed. Hence, the frequency of that medium.
sound heard by him is given by (u0 = us) The SI unit of intensity is watt per square metre
v u0 350 2 (W m–2).
= = 700
v us 350 2
an arbitrary scale. The zero of the scale is taken at
= 708 Hz
the sound wave intensity
Number of beats heard per second = – =
I0 = 1 10–12 W m–2
708 – 700 = 8
Beat frequency = 8 Hz which corresponds to the faintest audible sound.
(b) A person standing at A will hear two sounds, The unit of intensity level is called decibel (dB).
one from the siren approaching him and the Intensity level in decibels of a sound of intensity
other from the mirror image of the siren also I
approaching him with the same speed. Hence he I = 10 log
I0
will hear no beats.

Multiple Choice Questions with Only One Choice Correct

1. When a plane harmonic wave of wavelength


travels in a medium, the particle speed will always (a) less then (b) less then
2
be less then the wave speed if the amplitude of the
wave is (c) greater then (d) greater then
Waves and Doppler’s Effect 14.15

2. An open pipe and a closed pipe have the same graphs shown in Fig. 14.7 correctly represents the
length L. The ratio of the frequencies of their n th variation of the apparent frequency of sound as
overtones is heard by the observer with time t?
2n n
(a) (b)
n 1 n 1
2 n 1 2 n 1
(c) (d)
n 2 2n 1
3.

also found that a vibrating source resonates with the


n th harmonic of the closed pipe and with the m th
harmonic of the open pipe. The ratio n/m is
2 3
(a) (b)
3 2
3 5
(c) (d)
5 3
4. A standing wave is produced due to a superposition
Fig. 14.7
a boundary. It is observed that the amplitude at
antinode is 9 times that at node. The percentage of 8. A train is moving with a constant speed u along
a circular track. A siren in its engine is emitting a
is sound of frequency . If u = v/10, where v is the
(a) 36% (b) 64% speed of sound, the apparent frequency of sound as
(c) 72% (d) 81% heard by a passenger at the rear end of the train is
5. A source emitting a sound of wavelength is in 9 11
(a) (b)
10 10
Another source emitting a sound of wavelength 9
is in resonance with the second overtone of the (c) (d)
11
same pipe. The ratio / is
9. A progressive wave in a medium is represented by
5 4 the equation
(a) (b)
3 3 5
y = 0.1 sin 10 t x
3 5 11
(c) (d)
2 2
where y and x are in cm and t in seconds. The
6. The displacements of a particle located at x at time maximum speed of a particle of the medium due
t due to two waves are given by to the waves is
y1 = a sin ( t – kx) (a) 1cm s–1 (b) 10 cm s–1
And y2 = a sin ( t – k x + ) (c) cm s–1 (d) 10 cm s–1
If the amplitude of the resultant wave formed by 10. A tuning fork of frequency 340 Hz is sounded
the superposition of these two waves is a, the phase above a cylindrical tube 1 m high. Water is slowly
constant is equal to poured into the tube. If the speed of sound is 340
ms–1, at what levels of water in the tube will the
(a) zero (b)
2 (a) 25 cm, 75 cm (b) 20 cm, 80 cm
2 3 (c) 15 cm, 85 cm (d) 17 cm, 83 cm
(c) (d)
3 4 11. A sonometer wire, 65 cm long, is in resonance
7. Starting from rest, an observer moves with a con- with a tuning fork of frequency N. If the length
stant acceleration a towards a stationary source of the wire is decreased by 1 cm and it is vibrated
emitting a sound of frequency 0. Which of the with the same tuning fork, 8 beats are heard per
second. What is the value of N?
14.16 Comprehensive Physics—JEE Advanced

(a) 256 Hz (b) 384 Hz (a) 318 Hz (b) 321 Hz


(c) 480 Hz (d) 512 Hz (c) 327 Hz (d) 330 Hz
12. Two organ pipes, each closed at one end, give 5 19. A metal wire of diameter 1 mm is held on two knife
beats per second when emitting their fundamental edges separated by a distance of 50 cm. The tension
notes. If their lengths are in the ratio of 50 : 51, their in the wire is 100 N. The wire vibrating with its
fundamental frequencies (in Hz) are fundamental frequency and a vibrating tuning fork
(a) 250, 255 (b) 255, 260 together produce 5 beats per second. The tension
(c) 260, 265 (d) 265, 270 in the wire is then reduced to 81 N. When the two
13. Two sources A and B are sounding notes of are excited, beats are heard at the same rate. What
frequency 680 Hz. A listener moves from A to B is the frequency of the fork?
with a constant velocity u. If the speed of sound (a) 90 Hz (b) 95 Hz
is 340 ms–1, what must be the value of u so that he (c) 100 Hz (d) 105 Hz
hears 10 beats per second? 20. -
(a) 2.0 ms–1 (b) 2.5 ms–1 gation and Ranging) system which operates at an
(c) 3.0 ms–1 (d) 3.5 ms–1 ultrasonic frequency of 42 kHz. An enemy subma-
14. Standing waves are produced by the superposition rine is moving towards the SONAR with a speed
of two waves of 200 ms–1. If the speed of sound in seawater is
y1 = 0.05 sin (3 t – 2x) 1400 ms–1, what is the frequency of sound received
-
and y2 = 0.05 sin (3 t + 2x)
tion from the enemy submarine?
where x and y are expressed in metres and t is in (a) 36 kHz (b) 42 kHz
seconds. What is the amplitude of a particle at x =
0.5 m. Given cos (57.3°) = 0.54. (c) 48 kHz (d) 56 kHz
(a) 2.7 cm (b) 5.4 cm 21. A machine gun is mounted on a tank moving at
(c) 8.1 cm (d) 10.8 cm a speed of 20 ms–1 towards a target with the gun
15. pointing in the direction of motion of the tank.
both ends is given by The muzzle speed of the bullet equals the speed
of sound = 340 ms–1
2 x
y = 0.06 sin cos (120 t) target is 500 m away from the tank, then
3
where x and y are in metres and t is in seconds. (a) the sound arrives at the target later than the
The length of the string is 1.5 m and its mass is bullet
3.0 10–2 kg. What is the tension in the string? (b) the sound arrives at the target earlier than the
(a) 648 N (b) 724 N bullet
(c) both sound and bullet arrive at the target at
(c) 832 N (d) 980 N
the same time
16 . A pipe of length 20 cm is closed at one end. Which
(d) the bullet will never arrive at the target.
harmonic mode of the pipe is resonantly excited by
a 425 Hz source? The speed of sound = 340 ms–1. 22. Out of the four choices given in Q. 21 above,
(a) First harmonic (b) Second harmonic choose the correct choice, if the gun points in a
direction opposite to the direction of motion of the
(c) Third harmonic (d) Fourth harmonic
tank.
17. A pipe of length 20 cm is open at both ends. Which
harmonic mode of the pipe is resonantly excited by 23. Three sound waves of equal amplitudes have
a 1700 Hz source? The speed of sound = 340 ms–1. frequencies ( – 1), ( ) and ( + 1). They superpose
(a) First harmonic (b) Second harmonic to give beats. The number of beats produced per
second will be
(c) Third harmonic (d) Fourth harmonic
18. Two sitar strings A and B are slightly out of tune (a) (b)
and produce beats of frequency 6 Hz. When the 2
tension in string A is slightly decreased, the beat (c) 2 (d) 1
frequency is found to be reduced to 3 Hz. If the 24. The speed of sound in hydrogen at STP is v. The
original frequency of A is 324 Hz, what is the speed of sound in a mixture containing 3 parts of
frequency of B? hydrogen and 2 parts of oxygen at STP will be
Waves and Doppler’s Effect 14.17

v v (a) approaching the earth with a speed 3 107


(a) (b) ms–1
2 5
(b) going away from the earth with a speed
v 3 107 ms–1
(c) 7 v (d)
7 (c) approaching the earth with a speed 1.5 107
25. The speed of sound in hydrogen at STP is v. What ms–1
is the speed of sound in helium at STP? (d) going away from the earth with a speed
v v 1.5 107 ms–1
(a) (b) 32. Radiowaves of frequency 600 MHz are sent by a
2 2
radar towards an enemy aircraft. The frequency
(c) 2 v (d) 2 v
26. Nine tuning forks are arranged in order of increas- measured at the radar station is found to increase
ing frequency. Each tuning fork produces 4 beats by 6 kHz. It follows that the aircraft is
per second when sounded with either of its neigh- (a) approaching the radar station with a speed
bours. If the frequency of the 9th tuning fork is 1.5 kms–1
(b) going away from the radar station with a
speed 1.5 kms–1
(c) approaching the radar station with a speed
(a) 32 Hz (b) 40 Hz 3 kms–1
(c) 48 Hz (d) 56 Hz (d) going away from the radar station with a
27. A sonometer wire of length 120 cm is divided into speed 3 kms–1.
three segments of lengths in the ratio of 1 : 2 : 3. 33. An observer moves towards a stationary source
What is the ratio of their fundamental frequencies? of sound with a velocity one-tenth the velocity of
(a) 3 : 2 : 1 (b) 4 : 2 : 1 sound. The apparent increase in frequency is
(c) 5 : 3 : 2 (d) 6 : 3 : 2 (a) zero (b) 5%
28. A tuning fork produces 4 beats per second when (c) 10% (d) 0.1%
sounded with a sonometer wire of vibrating length 34. A wave represented by the equation y = a cos (kx
48 cm. It produces 4 beats per second also when the – t) is superposed with another wave to form a
vibrating length is 50 cm. What is the frequency of stationary wave such that the point x = 0 is a node.
the tuning fork? The equation of the other wave is
(a) 196 Hz (b) 284 Hz (a) y = a sin (kx + t)
(c) 375 Hz (d) 460 Hz (b) y = – a cos (kx – t)
29. Two identical strings of a stringed musical instru- (c) y = – a cos (kx + t)
ment are in unison when stretched with the same (d) y = – a sin (kx – t)
tension. When the tension in one string is increased IIT, 1983
by 1%, the musician hears 4 beats per second. What 35. Two wires of the same material and radii r and 2r
was the frequency of the note when the strings were are welded together end to end. The combination
in unison? is used as a sonometer wire and kept under tension
(a) 796 Hz (b) 800 Hz T. The welded point is mid-way between the two
(c) 804 Hz (d) 808 Hz bridges. When stationary waves are set up in the
composite wire, the joint is a node. Then the ratio
30.
of the number of loops formed in the thinner to
frequency 300 Hz at 27°C. If the temperature of the
thicker wire is
(a) 2 : 3 (b) 1 : 2
beats heard per second will be
(c) 2 : 1 (d) 5 : 4
(a) 1 (b) 2
36. A uniform rope of length 12 m and mass 6 kg
(c) 3 (d) 4
hangs vertically from a rigid support. A block of
31. The wavelength of light of a particular wavelength mass 2 kg is attached to the free end of the rope. A
received from a galaxy is measured on earth and transverse pulse of wavelength 0.06 m is produced
is found to be 5% more that its wavelength. It in the lower end of the rope. What is the wavelength
follows that the galaxy is of the pulse when it reaches the top of the rope?
14.18 Comprehensive Physics—JEE Advanced

(a) 0.06 m (b) 0.03 m 43. An organ pipe P1, closed at one end vibrating in
(c) 0.12 m (d) 0.09 m P2, open at
IIT, 1984 both ends vibrating in its third harmonic, are in
37. A tube closed at one end containing air, produces, resonance with a given tuning fork. The ratio of the
when excited, the fundamental note of frequency lengths of P1 and P2 is
512 Hz. If the tube is open at both ends, the funda- 8 1
(a) (b)
mental frequency that can be excited is (in Hz) 3 6
(a) 1024 (b) 512 1 1
(c) (d)
(c) 256 (c) 128 2 3
IIT, 1988
IIT, 1986
38. Two sound waves of equal intensity I produce beats. 44. The extension in a string, obeying Hookes’ law, is
The maximum intensity of sound produced in beats x. The speed of the wave in the stretched string is
will be v. If the extension in the string is increased to 1.5
(a) I (b) 4I x, the speed of the wave in the string will be
(c) 2I (d) I/2 (a) 1.22 v (b) 0.61 v
39. Two travelling waves (c) 1.50 v (d) 0.75 v
y1 = A sin [k(x + ct)] IIT, 1996
and y2 = A sin [k(x – ct)] 45. A travelling wave in a stretched string is described
are superposed on a string. The distance between by the equation
y = A sin (kx – t)
adjacent nodes is
ct ct The maximum particle velocity is
(a) (b)
2 (a) A (b)
k
d x
(c) (d) (c) (d)
2k k dk t
IIT, 1992 IIT, 1997
40. Three waves of equal frequency having amplitudes 46. A transverse sinusoidal wave of amplitude a, wave-
10 m, 4 m and 7 m arrive at a given point with length and frequency f is travelling on a stretched
a successive phase difference of /2. The amplitude string. The maximum speed at any point on the
of the resulting wave in m is given by string is v/10, where v is the speed of propagation
(a) 7 (b) 6 of the wave. If a = 10–3 m and v = 10 ms–1, then
(c) 5 (d) 4 and f are given by
41. A transverse wave is represented by 103
(a) =2 10–2 m, f = Hz
2 2
y = y0 sin vt x (b) = 10–2 m, f = 103 Hz
(c) = 10–3 m, f = 104 Hz
For what value of is the maximum particle veloc-
2
ity equal to twice the wave velocity? 10
(d) = m, f=2 103 Hz
2 y0 2
(a) = 2 y0 (b) = IIT, 1998
3
y0 47. The ratio of the speed of sound in nitrogen gas to
(c) = (d) = y0 that in helium gas at 300 K is
2
42. A train blowing its whistle moves with a constant 2 1
(a) (b)
velocity u away from the observer on the ground. 7 7
The ratio of the actual frequency of the whistle to 3 6
that measured by the observer is found to be 1.2. If (c) (d)
5 5
the train is at rest and the observer moves away from
IIT, 1999
it at the same velocity, the ratio would be given by
(a) 0.51 (b) 1.25 48. Two vibrating strings of the same material but
(c) 1.52 (d) 2.05 lengths L and 2L have radii 2r and r, respectively.
They are stretched under the same tension. Both the
IIT, 1993
Waves and Doppler’s Effect 14.19

strings vibrate in their fundamental modes, the one pulses will be


of length L with frequency 1 and the other with (a) zero
frequency 2. The ratio 1/ 2 is given by (b) purely kinetic
(a) 2 (b) 4 (c) purely potential
(c) 8 (d) 1 (d) partly kinetic and partly potential
IIT, 2000 IIT, 2001
49. Two sounds of wavelengths 5 m and 6 m, travel-
ling in a medium produce 10 beats per second. The
speed of sound in the medium is
(a) 300 ms–1 (b) 320 ms–1
–1
(c) 350 ms (d) 1200 ms–1 Fig. 14.8

50. The frequencies of tuning forks A and B are 55. A siren placed at a railway platform is emitting
respectively 3% more and 2% less than the fre- sound of frequency 5 kHz. A passenger sitting in
quency of fork C. When A and B are simultane- a moving train A records a frequency of 5.5 kHz
ously excited 5 beats per second are produced. The while the train approaches the siren. During his
frequency (in Hz) of fork A is return journey in a different train B he records a
(a) 98 (b) 100 frequency of 6.0 kHz while approaching the same
(c) 103 (d) 105 siren. The ratio of the velocity of train B to that of
51. A metallic wire with tension T and at temperature train A is
30ºC vibrates with its fundamental frequency of (a) 242/252 (b) 2
1 kHz. The same wire with the same tension but (c) 5/6 (d) 11/6
at 10ºC temperature vibrates with a fundamental IIT, 2002
56. A sonometer wire resonates with a given tuning
expansion of the wire is
(a) 2 × 10–4 ºC –1 (b) 1.5 × 10–4 ºC–1 between the two bridges when a mass of 9 kg is
–4 –1 –4 –1
(c) 1 × 10 ºC (d) 0.5 × 10 ºC suspended from the wire. When this mass is re-
52. A knife-edge divides a sonometer wire into two placed by a mass M, the wire resonates with the
parts. The fundamental frequencies of the two same tuning fork forming three antinodes for the
parts are 1 and 2. The fundamental frequency same positions of the bridges. The value of M is
of the sonometer wire when the knife-edge is (a) 25 kg (b) 5 kg
removed will be (c) 12.5 kg (d) (1/25) kg
1
(a) 1 + 2 (b) ( 1 + 2) IIT, 2002
2
57. A police van, moving at 22 ms–1, chases a motor-
(c) (d) 1 2 cyclist. The policeman sounds his horn at 176 Hz,
1 2
1 2 while both of them move towards a stationary si-
53. A sonometer wire is stretched by a hanging metal
bob. Its fundamental frequency is 1. When the
bob is completely immersed in water, the
frequency becomes 2. The relative density of the
metal is If the motorcyclist does not observe any beats, his
2 2
(a) 1
(b) 2 speed must be (take the speed of sound = 330 ms–1)
2 2 2 2
1 2 1 2 (a) 33 ms–1 (b) 22 ms–1
(c) 1
(d) 2 (c) zero (d) 11 ms–1
1 2 1 2 IIT, 2003
54. Two pulses in a stretched string whose centers 58. In the experiment for the determination of the speed
are initially 8 cm apart are moving towards each of sound in air using the resonance column, it is
other as shown in Fig. 14.8. The speed of each pulse observed that 0.1 m of air column resonates with
is 2 cm/s. After 2 seconds, the total energy of the a tuning fork in the fundamental mode. When the
14.20 Comprehensive Physics—JEE Advanced

length of the air column is changed to 0.35 m, the


(a) 1 (b) 3
3
What is the end correction?
(a) 0.0125 m (b) 0.025 m 1 1 1 2
(c) (d)
(c) 0.05 m (d) 0.075 m 6 2
6 1
IIT, 2004
IIT, 2003
59. Transverse waves are generated in two uniform 65. A source of sound of frequency 600 Hz is placed
steel wires A and B of diameters 10–3 m and 0.5 inside water. The speed of sound in water is 1500
10–3 m respectively, by attaching their free end to a ms–1 and in air it is 300 ms–1. The frequency and
vibrating source of frequency 500 Hz. The ratio of wavelength of sound recorded by an observer who
the wavelengths if they are stretched with the same is standing in air respectively are
tension is (a) 600 Hz, 0.5 m (b) 600 Hz, 2.5 m
1 1 (c) 3000 Hz, 0.4 m (d) 120 Hz, 2 m
(a) (b)
2 2 IIT, 2004
66. In the resonance tube experiment for determining
(c) 2 (d) 2
the speed of sound in air using a tuning fork of
60. A wire is stretched between two rigid supports with a
certain tension. It is observed that the wire resonates at 17.7 cm of air column and the second at 53.1 cm.
in the nth harmonic at a frequency of 420 Hz. The The maximum possible error in the speed of sound
next higher frequency at which the wire resonates is in air is
490 Hz. The value of n is (a) 192 cm s–1 (b) 96 cm s–1
(a) 2 (b) 4 (c) 64 cm s –1
(d) 48 cm s–1
(c) 6 (d) 8 IIT, 2005
61. The fundamental frequency of a sonometer wire
67. A rod AB of length L is hung from two identical
increases by 6 Hz if its tension is increased by 44%, wires 1 and 2. A block of mass m is hung at point
keeping the length constant. The frequency of the O of the rod as shown in Fig. 14.9. The value of x
wire is so that a tuning fork excites the fundamental mode
(a) 24 Hz (b) 30 Hz in wire 1 and the second harmonic in wire 2 is
(c) 36 Hz (d) 42 Hz
L L
62. Which of the following functons represents a trav- (a) (b)
5 4
elling wave? Here a, b and c are constants.
L 2L
(a) y = a cos (bx) sin (ct) (c) (d)
3 3
(b) y = a sin (bx) cos (ct)
(c) y = a sin (bx + ct) – a sin (bx – ct)
(d) y = a sin (bx + ct) Wire 1 Wire 2
63. A sonometer wire is vibrating with a frequency of L

30 Hz in the fundamental mode. If the length of A


x
B
the wire is increased by 20%, the change in the O
frequency of the fundamental mode is
(a) 5 Hz (b) 10 Hz
(c) 15 Hz (d) 20 Hz m

64. An organ pipe P1, closed at one end and containing Fig. 14.9
a gas of density 1
Another organ pipe P 2, open at both ends and 68. Two tuning forks, each of frequency , move rela-
containing a gas of density 2 is vibrating in its third tive to a stationary observer. One fork moves away
harmonic. Both the pipes are in resonance with a from the observer with a speed u while the other
given tuning fork. If the compressibility of gases fork moves towards him at the same speed. The
is equal in both pipes, the ratio of the lengths of P1 speed of sound is v. If u << v, the observer hears
and P2 is beats of frequency
Waves and Doppler’s Effect 14.21

u (a) 344 (b) 336


(a) zero (b) (c) 117.3 (d) 109.3
v
IIT, 2008
2 u u 73. A transverse sinusoidal wave moves along a string
(c) (d)
v 2v in the postive x-direction at a speed of 10 cm s–1.
69. The wavelength of the wave is 0.5 m and its ampli-
both ends are represented by tude is 10 cm. At a particular time t, the snap-shot
x of the wave is shown in Fig. 14.10. The velocity of
y = 4 sin cos (96 t) point P when its displacement is 5 cm is
15
where x and y are in cm and t in second. The
particle velocity at x = 7.5 cm and t = 0.25 s is
(a) zero (b) 10 cm s–1
(c) 100 cm s–1 (d) (4 96) cm s–1
IIT, 1985
70. A uniform rope of mass M hangs vertically from a
rigid support. A block of mass m is attached to the Fig. 14.10
free end of the rope. A transverse pulse of wave-
length is produced at the lower end of the rope. 3
3 1 1
The wavelength of the pulse when it reaches the top (a) j ms (b) – j ms
of the rope is 50 50
1/ 2
m M m M 3 1 3 1
(a) (b) (c) i ms (d) – i ms
m m 50 50
m IIT, 2009
(c) (d)
M 74. A point mass is subjected to two simultaneous sinu-
IIT, 1984 soidal displacements in x-direction, x1(t) = A sin t
71. A band playing music at a frequency f is moving 2
towards a wall at a speed u. A motorist is following and x2(t) = A sin t . Adding a third sinu-
3
the band with the same speed u. If v is the speed of soidal displacement x3(t) = B sin( t + ) brings the
sound, the beat frequency heard by the motorist is mass to a complete rest. The values of B and are
2f u 3 4
(a) f (b) (a) 2 A, (b) A,
(v +u) 4 3
f (u v) 2 fu 5
(c) (d) (c) 3 A, (d) A,
(v u) (v u) 6 3
72. A vibrating string of certain length L under a ten- IIT, 2011
sion T resonates with a mode corresponding to the 75. A police car with a siren of frequency 8 kHz is
moving with uniform velocity 36 km/hr towards a
length 75 cm inside a tube closed at one end. The
string also generates 4 beats per second when ex- speed of sound in air is 320 m/s. The frequency of
cited along with a tuning fork of frequency n. Now the siren heard by the car driver is
when the tension of the string is slighty increased (a) 8.50 kHz (b) 8.25 kHz
the number of beats reduces to 2 per second. As- (c) 7.75 kHz (d) 7.50 kHz
suming the velocity of sound in air to be 340 m/s,
IIT, 2011
the frequency n of the tuning fork in Hz is
14.22 Comprehensive Physics—JEE Advanced

ANSWERS

1. (a) 2. (d) 3. (b) 4. (b) 5. (a) 6. (c)


7. (d) 8. (d) 9. (c) 10. (a) 11. (d) 12. (a)
13. (b) 14. (b) 15. (a) 16. (a) 17. (b) 18. (a)
19. (b) 20. (d) 21. (a) 22. (b) 23. (d) 24. (d)
25. (a) 26. (a) 27. (d) 28. (a) 29. (b) 30. (b)
31. (d) 32. (a) 33. (c) 34. (c) 35. (b) 36. (b)
37. (a) 38. (b) 39. (d) 40. (c) 41. (d) 42. (b)
43. (b) 44. (a) 45. (a) 46. (a) 47. (c) 48. (d)
49. (a) 50. (c) 51. (d) 52. (d) 53. (a) 54. (b)
55. (b) 56. (a) 57. (b) 58. (b) 59. (a) 60. (c)
61. (b) 62. (d) 63. (a) 64. (d) 65. (a) 66. (a)
67. (a) 68. (c) 69. (a) 70. (b) 71. (d) 72. (a)
73. (a) 74. (b) 75. (a)

SOLUTION
1. For a plane harmonic wave, the particle displace- 3. Let Lc be the length of the closed pipe and Lo of the
ment is given by open pipe. It is given that
y = A sin ( t – k x) v v
dy 3 =2
Particle speed V = = A cos ( t – kx) 4Lc 2Lo
dt
Vmax = A = 2 A Lc 3
=
Wave speed v = Lo 4
Particle speed will always less than wave speed if Further, the frequency of the n th harmonic of the
2 A < closed pipe = frequency of the mth harmonic of the
open pipe, i.e,
or A< n 1, 3, 5, etc.
2 nv mv
= ;
2. In an open pipe, all harmonics (even as well as odd) 4 Lc 2 Lo m 1, 2, 3, etc.
are present, but in a closed pipe, only odd harmonics n 2 Lc 3 3
are present. Thus, in an open pipe, the second = =2 =
m Lo 4 2
is the second overtone and so on. In a closed pipe, 4. Let Ai and Ar be the amplitudes of the incident and

harmonic is the second overtone and so on. If v is Amplitude at antinode = Ai + Ar


the speed of sound, the frequency of the n th over- Amplitude at node = Ai – Ar
tone in an open pipe is Given
v Ai Ar Ar
0 = (n + 1) =9 = 0.8
2L Ai Ar Ai
In a closed pipe, the frequency of the nth overtone
is Intensity is proportional to the square of the
amplitude.
v Hence
c = (2n + 1)
4L 2
Ir Ar
0 2 n +1 = = (0.8)2 = 0.64 = 64%
= Ii Ai
c 2n 1
Waves and Doppler’s Effect 14.23

5. Let L be the length of the pipe. given u0 = us = u. Thus, the source is receding from
the observer with a speed u cos and the observer
3 2L
L= = is approaching the source with the same speed
2 3 u cos
5 2L v + u cos
For second overtone; L = = Apparent frequency = =
2 5 v + u cos
5 Hence the correct chose is (d)
= , which is choice (a)
3 9. The speed of particle is given by
6. The resultant amplitude is given by
dy 5
A 2 = a 21 + a 22 + 2a1 a2 cos (1) V= = 0.1 10 cos 10 t x
dt 11
Given A = a1 = a2 = a. Substituting these in Eq
Vmax = 0.1 10 = cms–1.
(1),
We have Hence the correct choice is (c).
a2 = a2 + a2 + 2a2 cos 10. Frequency of sound emitted by a closed pipe of
1 2 v
cos =– = length L in the fundamental mode is = . For
2 3 4L
So the correct chose is (c). resonance = where is the frequency of
7. Speed of observer at time t is u0 = 0 + at = at 340
the tuning fork. Thus 340 = or 4 L = 1 m or
4L 4
v + u0
Apparent frequency = 0 L = 25 cm. The next resonance occurs at L=1
v m or L = 75 cm. 3
v + at Hence the correct choice is (a).
= 0
v
1 T 1 T
a 0 11. N = . Given N = .
= 0+ t 2 65 1 m 2 65 m
v Therefore
This is the equation of a straight line with a positive N 2 65 65
a 0 = = Also N – N = 8
slope m = and a positive intercept c = 0 N 2 65 1 64
v
Hence the correct choice is (d). or N = N + 8.
8. Refer to Fig. 14.11. 8 65N
= which gives N = 512 Hz.
N 64
Hence the correct choice is (d).
1 p 1 p
12. 1 = , 2 = . Therefore
4 L1 4 L2

1 L2 50
= = . It is given that 2 = 1 + 5.
2 L1 51

1 50
Therefore = which gives 1 = 250 Hz.
1 5 51
Hence 2 = 250 + 5 = 255 Hz which is choice (a).
13. The listener moves away from A and approaches B.
Hence the apparent frequencies are
u
Fig. 14.11 1 = 1
v
The components of u o and u s along the direction and = 1
u
2
of the velocity of sound are uo cos and us cos v
14.24 Comprehensive Physics—JEE Advanced

2 – 1 = 2 u/v. It is given that for any value of p = 1, 3, 5, .... Now for a closed
2 – 1 = 10, v = 340 ms–1 and = 680 Hz. pipe, we know that
Substituting these values we get pv
p =
2 680 u 4L
10 = or u = 2.5 ms–1. Therefore, for resonance,
340
Hence the correct choice is (b). pv
N=
14. The resultant displacement is given by 4L
y = y1 + y2 4NL 4 425 0.2
or p= = =1
v 340
= 0.05 {sin (3 t – 2 x) + sin (3 t + 2x)}
Hence the correct choice is (a).
Using the trigonometric relation
17. In an open pipe, the condition of resonance is
sin ( + ) + sin ( – ) = 2 sin cos
we have y = 0.1 cos 2 x sin 3 t pv
N= p = ; p = 1, 2, 3, ....
or y = R sin 3 t 2L
where R, the amplitude of standing waves, is given 2NL 2 1700 0.2
or p= = =2
by R = 0.1 cos 2 x with v 340
x = 0.5 m Hence, the correct choice is (b).
cos 2x = cos (2 0.5 rad) 18. = 324 Hz, b = 6 Hz.
A
= cos (1 rad) = cos 57.3° The frequency of string B is B = A B = 324 ± 6
= 0.54 = 330 or 318 Hz
Amplitude R at x = 0.5 is 0.1 0.54 Now, the frequency of a string is proportional to
the square root of tension. Hence, if the tension
= 0.054 m = 5.4 cm
in A is slightly decreased, its frequency will be
15. Let be the wavelength, the frequency and v the slightly reduced, i.e. it will become less than
speed of each wave. Then 324 Hz. If the frequency of string B is 330 Hz, the
2 beat frequency would increase to a value greater
x in the argument of the sine
2 than 6 Hz if the tension in A is reduced. But the
function = beat frequency is found to decrease to 3 Hz. Hence,
3
the frequency of B cannot be 330 Hz; it is, there-
or = 3 m. fore 318 Hz. When the tension in A is reduced, its
Also =2 = c t in the frequency becomes 324 – 3 = 321 Hz which will
argument of the sine function produce beats of frequency 3 Hz with string B of
= 120 frequency 318 Hz. Hence the correct choice is (a).
which gives = 60 Hz. 19. Let N be the frequency of the tuning fork. Then,
Hence v= = 60 3 = 180 m s–1 the frequency of the wire, when the tension is
100 N will be (N + 5) and when the tension is 81
3.0 10 2
Mass per unit length (m) = N, it is (N – 5); since in each case 5 beats are heard
1.5 per second. Hence
= 2.0 10–2 kg m–1
1 T1 1 100
T N+5= =
We know that v = , where T is tension in 2L m 2 0.5 m
the string. m
T = mv2 = 2.0 10–2 (180)2 10
= (i)
= 648 N m
16. Let N be the frequency of the source and p that of 1 T2 1 81
the pth harmonic of a closed pipe. The source will and N–5= =
2L m 2 0.5 m
resonantly excite that harmonic mode of the pipe
for which 9
= (ii)
N= p m
Waves and Doppler’s Effect 14.25

Subtracting (ii) from (i) we have 23. When the three waves superpose at a point, then
1 from the superposition principle, the resultant par-
10 = ticle displacement at that point is given by
m
or m = 0.01 kg m–1 y = y1 + y2 + y3
= a sin {2 ( – 1)t} + a sin (2 t)
Using this value of m in (i) or (ii) gives N = 95 Hz.
+ a sin {2 ( + 1) t}
Hence the correct choice is (b).
20. The frequency of ultrasonic (sound) waves sent Now sin {2 ( – 1) t} + sin {2 ( + 1) t} =
out from the SONAR undergoes a change in two 2 cos 2 t sin 2 t
Therefore,
received by the enemy submarine which is appro- y = y1 + y2 + y3
aching the SONAR with a speed u0 = 200 ms–1 is y = a (1 + 2 cos 2 t) sin 2 t
given by
v + u0 or y = A sin 2 t
=
v where A = a (1 + 2 cos 2 t) is the resultant
3 amplitude.
42 10 1400 200
= Now, the resultant intensity A2. Now A2 will be
1400
maximum when
= 48 103 Hz
cos 2 t = + 1
(ii) The wave of frequency or 2 t = 0, 2 , 4 , ... etc.
enemy submarine, which acts as a virtual image or t = 0, 1s, 2s,... etc.
source of frequency , approaching the SONAR
Time period of beats = time interval between
with a speed us = 200 ms–1. Hence the frequency
two consecutive maxima = 1 s. Hence the beat fre-
of sound received back at the SONAR will be
quency is 1 Hz, i.e. one beat is heard per second
(observer stationary, source approaching)
which is choice (d).
v
= 24. Let the density of hydrogen be = 2 , then the
v us density of oxygen will be 32 . The density of the
1400 48 103 mixture will be
=
1400 200 3 2
= 2 + 32 = 14
= 56 103 Hz = 56 kHz 5 5
Hence the correct choice is (d). Since the pressure is the same, we have
21. Since only the source of sound, i.e. the gun is in v 2 1
motion, the speed of sound remains unchanged at =
v 14 7
340 ms–1. Therefore, the time taken by the sound of
v
= . Hence the correct choice is (d).
500 25 7
ts = = s 25. The density of helium at STP = 2 times the density
340 17
If the gun points in the direction of motion of the of hydrogen at STP. Since v 1/ , the speed of
tank, the effective speed of the bullet = 340 + 20 = sound in helium will be v/ 2 . Hence the correct
360 ms–1. Therefore, the time taken by the bullet to choice is (a).
reach the target is 26. . The
500 25 frequency of the second will be ( + 4) and of the
tb = = s third will be ( + 8) and so on. Now + 8 = +
360 18
Since ts > tb, the correct choice is (a). (3 – 1) 4. Therefore, the frequency of the 9th
tuning fork = + (9 – 1) 4 = + 32. It is given that
22. In this case, the effective speed of the bullet = 340
+ 32 = 2 . Hence = 32 Hz which is choice (a).
– 20 = 320 ms–1. Therefore,
500 25 27. The frequency of the fundamental mode is given by
tb = = s
320 16 1 T
Thus ts < tb. Hence the correct choice is (b). =
2L m
14.26 Comprehensive Physics—JEE Advanced

1 1 1 1 Now, frequency speed of sound. Hence


. Hence 1 : 2 : 3 = : : . v 151
2
L L1 L2 L3 = 2 =
Now L1 = 20 cm, L2 = 40 cm and L3 = 60 cm. 1 v1 150
Therefore, 151 300 151
2 = 1 = = 302 Hz.
1 1 1 150 150
1 : 2 : 3 = : : =6:3:2
20 40 60 Hence beat frequency = 302 – 300 = 2
Hence, the correct choice is (d). Thus the correct choice is (b).
28. Let 1 be the frequency of the wire when its vibrat-
31. If a source emitting light of wavelength goes
ing length is L1 = 48 cm and 2 when L2 = 50 cm.
1 away from the earth, the apparent wavelength of
Since ; 1 > 2. If is the frequency of the the light reaching the earth is given by
L
tuning fork, then v
=1+
1 = + 4 and 2 = – 4 c
1 4 where v is the speed of the source of light and c the
=
2 4 speed of light. The increase in wavelength =
– is given by
1 L2 50 25
But = = v
2 L1 48 24 =
c
4 25 5
Thus = Here = 5% = and c = 3 108 ms–1.
4 24 100
which gives = 196 Hz. Hence the correct choice
is (a). Therefore,
29. Let T1 be the tension in each string when they are 5
v=3 108 = 1.5 107 ms–1
in unison. Let T2 be the tension in each string when 100
they are not in unison; then since T ; 2> 1 Hence the correct choice is (d).
such that 32. If the aircraft is approaching the radar station with
2
= 2
T a speed u, the apparent frequency of radiowaves re-
1 T1
Now T2 = 1.01 T1. Therefore is given by
1/ 2 2u
2 1 = 1
= 1.01 = 1 c
1 100
Apparent increase in frequency is
Expanding binomially, we have
2u
1 1/ 2
1 201 = – =
1 1+
= . Thus c
100 200 200 Given, = 600 MHz = 600 106 Hz
2 201 and = 6 kHz = 6 103 Hz. Thus
=
1 200 c 3 108 6 103
Also, – 1= 4 u= =
2 2 2 600 106
or 2 = 1 + 4. Therefore, we have
= 1.5 103 ms–1 = 1.5 kms–1
1 4 201
= Hence, the correct choice is (a).
1 200
u0 1
which gives 1 = 800 Hz. Hence the correct 33. = 1 . Given u0/v = . Therefore
choice is (b). v 10
30. Let v1 be the speed of sound at 27°C and v2 at 31°C. 11
=
Then 10
1/ 2 11
v2 273 31 304 1 / 2 4 1/ 2
or – = – = .
1 10 10
v1 273 27 300 300
The percentage increase in is
1 4 151
1
2 300 150 100 = 10% which is choice (c).
Waves and Doppler’s Effect 14.27

34. To form a stationary wave, waves y and y must i.e. a1 = a2 = a. Thus A = 2a. Therefore, A2 = 4a2 or
travel in opposite directions. Wave y = a cos (kx Imax = 4I. Hence the correct choice is (b).
– t) travels along the positive x-direction. Waves 39. Distance between adjacent nodes (or antinodes) =
y = – a cos (kx – t) and y = – a sin (kx – t) in /2. Also
choices (b) and (d) travel along positive x-direction. 2
Hence choices (b) and (d) are not possible. Choice x in the argument
of the sine function = k
(a) is also incorrect because at x = 0
2
y = a sin t or = .
k
and y = a cos (– t) = a cos t Hence, the distance between adjacent nodes
Therefore, the resultant displacement at x = 0 which
is y + y = a sin t + a cos t is not zero, i.e. these . =
k
waves do not produce a node at x = 0. Choice (c) is 40. The amplitude of the three waves are a1 = 10 m,
correct because at x = 0, y + y = 0. a2 = 4 m and a3 = 7
35. The frequency of a string of length L, mass m per wave be zero. Then the wave of the second wave
unit length, stretched with a tension T and vibrating = /2 and of third wave = . Therefore,
in p segments, is given by 2 2
p T third wave is . Hence their resultant is
=
2L m a = a1 – a3 = 10 – 7 = 3 m
If the radius of the wire is r and its density, then
The phase difference between this resultant (of
mass r2 L
m= = = r2 . /2.
length L Therefore, the resultant of a and a3 is

=
p T A = (a2 + a 22)1/2 = (32 + 42)1/2 = 5 m.
2Lr
Hence the correct choice is (c).
Since , T, L and are the same for both wires,
p 41. Wave velocity = v. Particle velocity is
= constant or p r. Hence the number of loops dy
r V=
formed in the thicker wire will be two times that in dt
the thinner wire. Hence the correct choice is (b).
2 v 2
36. = y0 cos vt x
the rope is different at different points on the rope.
- 2 v
sion = weight of the rope + the weight attached to Vmax = y0
the free end of the rope = 6 kg + 2 kg = 8 kg wt.
Tension at the free end of the rope = 2 kg wt. Since 2 v
Now, Vmax = 2 v, if y0 = 2v, which gives
T , if the tension becomes 4 times, the fre-
v 1 = y0. Hence the correct choice is (d).
quency is doubled. Since = ; . Hence 42. If the train is going away from the observer, the
the wavelength is halved. Thus the correct choice apparent frequency is
is (b). v
v 1 = (i)
37. For a closed tube = . For an open tube = v u u
4 L 1
v v
. Hence = 2 = 2 512 = 1024 Hz. Thus the
2L It is observed that = 1.2 1. In the second case, the
correct choice is (a). apparent frequency is
38. When two waves of amplitudes a1 and a2 super-
pose to produce beats, the resultant amplitude of v u u
2 = = 1
the maxima of intensity is v v
A = a1 + a2 1
2
or = (ii)
Now, intensity (amplitude) . Since the two waves 2 1
u
have the same intensity, their amplitudes are equal, v
14.28 Comprehensive Physics—JEE Advanced

u 47. For a gas, adiabatic elasticity E = P where =


Now, from (i) we have = 1 + u or 1.2 = 1 + , Cp/Cv and P is the pressure. The speed of sound in
1 v v
u the gas is given by
or u = 0.2 v or = 0.2. Using this in (ii), we get
v E
RT P
5 v=
= 1.25. Hence the correct choice is (b). M
2 4
where M is the molecular mass and R is the gas
v constant. Thus
43. For pipe P1 : 1 =
4 L1 vN M He 4 1
3v =
For pipe P2 : = vHe MN 28 7
3
2L2 48. Frequency of the fundamental mode is given by
It is given that 1 = 3. Therefore, L1/L2 = 1/6 which 1 T
is choice (b). =;
2L m
44. The speed of the wave in the string is given by T = tension and m = mass per unit length
T
v= 1 1/ 2L T / m1 m2
m = 2
According to Hookes’law, tension (T) extension (x). 2 1/ 4L T / m2 m1
Hence v x . Therefore M1 2r 2 L
Now m1 = = =4 r2
v 15
. x L L
= . = 1.22
15
v x M2 r2 2L
and m2 = = r2
Hence the correct choice is (a). 2L 2L
dy d
45. Particle velocity V = [A sin (kx – t)] 1 r2
dt dt Hence = 2 =1
= – A cos (kx – t) 2 4 r2
Hence Vmax = A 30
49. Beat frequency 0 = = 10 per second. Now
46. Standing waves are formed on the string. Particle 3
displacements are given by = – 2=
v v
b 1
2 x 1 2
y = a sin cos (2 f t)
1 2 b 5 6 10
Particle velocity or v=
( 2 1) (6 5)
dy = 300 ms–1
V=
dt Hence the correct choice is (a).
2 x
= (– 2 fa) sin sin (2 ft) 50. Let the frequency of fork C be n. Then nA = n + 0.03 n
= 1.03n and nB = n – 0.02 n = 0.98 n. The beat
(V)max = 2 fa
frequency is
v nb = nA – nB
Given(V)max = . Therefore,
10 or 5 = 1.03 n – 0.98 n = 0.05 n
v which gives n = 100 Hz. Hence nA = 1.03 100
2 fa =
10 = 103 Hz which is choice (c).
1 51. Given t1 = 10ºC, n1= 1.001 kHz, t2 = 30ºC, n2 =
v 10 ms 103 1 kHz. Let l1 and l2 be the vibrating lengths of the
or f= Hz
20 a 20 10 3 m 2 wire at 10ºC and 30ºC respectively. Since tension
1
T is kept constant.
v 10 ms n 1l 1 = n 2 l 2 (i)
Now = 3
=2 10–2 m
f 10 /2 Hz
l2 n 1.001
Hence the correct choice is (a). = 1 1.001
l1 n2 1
Waves and Doppler’s Effect 14.29

If v uA v uA
l2 = l1 (1 + t) 55. For train A: A = or 5.5 = 5
uA = 0.1 v v v
l2
or = 1 + a t = 1 + a (30–10) v uB v uB
l1 For train B: B = or 6.0 = 5
= 1+ 20a (ii) v v
From (i) and (ii) we have uB = v/5 = 0.2 v
1 + 20 = 1.001 uB 0.2 v
= = 2, which is choice (b).
which gives = 0.5 × 10 per ºC, which is –4 uA 0.1v
choice (d). 56. Let L be the length of the wire between the bridges
52. Let L1 and L2 be the lengths of the two parts of and let m be the mass per unit length of the wire.
5
sonometer wire. Five antinodes on a length L implies that L = 1
2L 2
1 T 1 T or 1 = . Thus in this case we have
Given 1 = and 2 = 5
2 L1 m 2 L2 m

1 T v1 5 T1
or L1 = L2 = = constant, say k. 1 = , where T1 = M1 g
1 2
2 m 1 2L m
Thus
Three antinodes on a length L implies that
3 2L
L1 =
k
and L2 =
k L= 2 or 2 = . In this case, we have
2 3
1 2
The fundamental frequency of the complete sono- v2 3 T2
= , where T2 = M2 g
meter wire is 2
2 2L m
1 T k Given 1 = 2, M1 = 9 kg and M2 = M. Therefore,
= =
2( L1 L2 ) m ( L1 L2 ) we have
1 L1 L2 1 1 5 9g 3 Mg
or = = 2L m 2L m
k k 1 2
which gives M = 25 kg. Hence the correct choice
or = 1 2
, which is choice (d). is (a).
1 2
57. Given v = 330 ms–1 and up = 22 ms–1. The apparent
53. Let W1 be the weight of the bob in air and W2 when frequency of the police man’s horn of frequency
it is immersed in water. Given 176 Hz as heard by the motorcyclist is given by
1 W1 1 W2 330 um 176
1 = and 2 = 1 = 176 = (330 – um) (i)
2L m 2L m 330 22 308
2
W1 1 The apparent frequency of the stationary siren of
=
W2 2 frequency 165 Hz as heard by the motorcyclist is
2
given by
weight in air 330 um
Relative density =
loss of weight in water 2 = 165 (ii)
330
W1 2
1
Since the motorcyclist does not observe any beats,
= =
W1 W2 2 2 1 = 2. Equating (i) and (ii) and solving for um we
1 2
get um = 22 ms–1.
Hence the correct choice is (a). Hence the correct choice is (b).
54. After 2 seconds, both the pulses will be at the same 58. Let be the frequency of the tuning fork and e the
location on the string and will superpose on each end correction. Given L1 = 0.1 m and L2 = 0.35 m
other. Since their amplitudes are equal and oppo-
site, they cancel each other and the string becomes 1 P
= 1 = (i)
straight. Hence the string has no potential energy, 4 ( L1 e)
i.e. the total energy is purely kinetic. Thus the 3 P
correct choice is (b). = 3 = (ii)
4 ( L2 e)
14.30 Comprehensive Physics—JEE Advanced

Equating (i) and (ii), we get 6


1 3 which gives = = 30 Hz, which is choice (b).
= 0.2
L1 e L2 e 62. A travelling wave is characterized by wave functions
1 3 of the type y = f (vt + x) or y = f (vt – x) where f stands
or = for sine or cosine function. Hence the correct choice
01. e 0.35 e
is (d). Choices (a), (b) and (c) represent a stationary
which gives e = 0.025 m. Hence the correct choice or standing wave.
is (b).
63. If the length is increased by 20%, the new length is
59. The density of a wire of mass M, length L and
20 L
diameter d is given by L =L+ = 1.2 L
4M 4m 100
= 2
= The original frequency is
d L d2
1 T
T T =
Now vA = and vB = 2L m
mA mB Since T and m are the same, the new frequency will be
vA mB d 1 T 1 T
= = B = = =
vB mA dA 2L m 1.2 2 L m 1.2
but vA = n A and vB = n B, n being the frequency of 30
Now = 30 Hz. Therefore, =
= 25 Hz. Thus,
the source. 1.2
v d 0.5 10 3 the frequency decreases by 5 Hz. Hence the correct
Hence A
= A = B = = 0.5, choice is (a).
B vB dA 10 3
nv
which is choice (a). 64. For a closed pipe, n = ; n = 1, 3, 5, etc
60. The frequency of the nth hormonic is 420 Hz. The 4L
frequency of the (n +1)th harmonic is 490 Hz. nv
For an open pipe, n = ; n = 1, 2, 3, etc
Therefore, 2L
n T
420 = where v =
P
.
2L m
(n 1) T
and 490 =
2L m (n = 1), we have
Dividing, we have 1 P
490 (n 1) 1 = (1)
= which gives n = 6. 4 L1 1
420 n For the open pipe vibrating in the third harmonic
61. The fundamental frequency of the vibration of a (n = 3), we have
wire of length L, mass m per unit length and under
3 P
tension T is given by 3 = (2)
2 L2
1 T 2
= (1) Given . Equating (1) and (2), we get
2L m 1 3

If the tension is increased by 44%, the new tension is L1 1 2


= .
44 L2 6
T =T+ T = T + 0.44 T = 1.44 T 1
100 Hence the correct choice is (d).
Since L is kept constant, the new fundamental 65. The frequency of sound is a characteristic of its
frequency is source. Hence frequency of sound is the same in
1 T 1 1.44T 1.2 T air as is water. Therefore, the observer in air will
= = =
2L m 2L m 2L m receive a sound of frequency 600 Hz. If a and w
Comparing this with Eq. (1), we have are the wavelengths in air and water resptively,
then
= 1.2 v vw
n= a
Given – = 6 Hz. Hence + 6 = 1.2 a w
Waves and Doppler’s Effect 14.31

va 300 For rotational equilibrium of the rod about O, we


which gives a= = 0.5 m have
600
T1 AO = T2 BO or T1 x = T2 (L – x),
Hence the correct choice is (a). The wavelength is
water is T ( L x) T
which gives 1 = . But 1 = 4. Hence
vw 1500 T2 x T2
w= = = 2.5 m
600 ( L x)
4=
66. Refer to Fig. 14.12. x
3 L
L1 = and L2 = which gives x = . Hence the correct choice
4 4 5
is (a).
v
N= 1 = (1) 68. When the source moves towards from the station-
4 ( L1 e)
ary observer, its apparent frequency is
3v v
and N= 3 = (2) = (1)
4 ( L2 e) v u
where e is the end correction. Eliminating e from
When the source moves away from the stationary
Eqs (1) and (2), we get
observer, its apparent frequency is
v = 2N (L2 – L1) (3)
v
Lengths L1 and L2 are measured with a metre scale = (2)
v u
whose least count is 0.1 cm. Thus L1 = (17.7
When both the forks are moving relative to station-
0.1) cm and L2 = (53.1 ± 0.1) cm. The maximum
ary observer, the number of beats heard by him per
error in (L2 – L1) is ± 0.2 (for maximum error, the
second = –
errors in individual measurements add up). Thus
Since us << v
L2 – L1 = (53.1 – 17.7) = 35.4 cm. Hence L2 – L1 =
follows.
(35.4 ± 0.2) cm. Using this in Eq. (3), we have
u 1 u
v = 2 480 (35.4 ± 0.2) = (33984 ± 192) = 1 1
v v
cms–1
1
u u
Hence maximum error = 192 cm s–1, which is = 1 1
choice (a). v v
where terms of order u2/v2 have been neglected in
the binomial expansion. Thus,
u u u
– = 1 1 = 2
v v v
Thus the correct choice is (c)
69. Particle velocity
dy d x
V= = 4 sin cos (96 t )
Fig. 14.12
dt dt 15

67. Let T1 and T2 be the tensions in wires 1 and 2 x


=–4 96 sin sin (96 t)
respectively. Let m be the mass per unit length 15
of each wire and let l be the length of each wire. At x = 7.5 cm and t = 0.25 s, the particle velocity is
Given
7.5
1 T1 2 T2 V= – 4 96 sin sin (96 0.25)
= = 15
2l 2m 2l m
= – (4 96) sin (0.5 ) sin (24 )
T1 =0 [ sin (24 ) = 0]
which gives =4
T2 The correct choice is (a).
14.32 Comprehensive Physics—JEE Advanced

70. The tension at the lower end of the rope is T = mg. than 340. If n = 336 Hz, this will result in a greater
T number of beats per second. But the number of
The speed of pulse at this point is v = , = mass beats decreases to 2 per second. Hence n is not 336
per unit length of the rope. If is the frequency of Hz; it is 344 Hz. So the correct choice is (a).
the pulse, then 73. In a transverse wave, the particle displacement and
particle velocity are perpendicular to the direction
v 1 T of propagation of the wave. Hence choices (c)
= = (1)
and (d) are wrong. The particle displacements is
The tension at the upper end of the rope is T = given by
(m + M) g. Let be the wavelength of the pulse 2
when it reaches the upper end of the rope. Then, y = a sin (vt x) (1)
since frequency of the pulse remains the same,
Putting y = 5 cm and a = 10 cm in Eq. (1), we get
1 T
= (2) 2 1
sin (vt x) =
2
From Eqs. (1) and (2), we have
2
(vt – x) =
T (m M ) g M m 6
= = =
T mg m Particle velocity is
Hence the correct choice is (b). dy 2 2
V= =a v cos (vt x) (2)
71. Let um = speed of motorist and ub = speed of the dt
band. The motorist will hear two sounds–one com-
Given a = 0.1 m, v = 0.1 ms–1, a = 0.1 m and
2
from the wall. The apparent frequency of the direct = 0.5 m. Also (vt – x) = . Putting these
sound is given by (the observer is approaching a 6
values in Eq. (2), we get
receding source of sound)
3
v um V= ms–1 along the y-axis.
f1 = f =f ( um = ub = u) 50
v ub
74. Since the mass is brought to rest, the total displace-
ment is zero, i.e.
the mirror image which is approaching the motorist x1(t) + x2(t) + x3(t) = 0
with a speed vb. Hence, the apparent frequency of
2
A sin t + A sin t + B sin( t + )
source of sound are both approaching each other) 3
=0
v um v u
f2 = f = =f
v ub v u Using sin + sin = 2sin cos ,
2 2
v u we get
Beat frequency (= f2 – f1) = f –f 4 4
v u 2A sin t cos + B sin ( t + ) = 0
3 3
2fu
= 4
v u – A sin t + B sin( t + ) + 0
3
Thus the correct choice is (d).
4 1
72. The frequency of the third harmonic of a closed cos
pipe is 3 2
3v 3 340 4
f= = = 340 Hz. B sin( t + ) = A sin t
4L 4 0.75 3
4
Beat frequency fb = 4. Therefore n = f fb = 340 which gives B = A and . =
4 or n = 336 Hz or 344 Hz. The frequency of string 3
75. u = 36 km h–1 = 10 ms–1,
T . Hence if tension T is slightly increased, the
v = 320 ms–1, = 8 kHz
frequency will slightly increase and become greater
Waves and Doppler’s Effect 14.33

v u
imagined to be coming from the mirror image. =
The driver is approaching the image-source which v u
is also approaching him with the same speed. 320 10
Hence the frequency of sound heard by the driver = 8 kHz = 8.5 kHz
320 10
is

II

Multiple Choice Questions with One or More Choices Correct

1. (c) y = a sin (b x + ct)


medium, which of the following will change? (d) y = a sin (b x + ct) + a sin (bx – c t)
(a) Amplitude (b) Frequency IIT, 1987
(c) Wavelength (d) Phase
6. When a wave travels in a medium, the particle dis-
2. When a wave is refracted into another medium, placements are given by
which of the following will change? y = a sin 2 (bt – cx)
(a) Amplitude (b) Velocity
where a, b and c are constants.
(c) Frequency (d) Phase
(a) The wavelength of wave is c.
3. Choose the correct statements from the following. b
(a) Any function of the form y (x, t) = f(vt + x) (b) The velocity of the wave is .
c
represents a travelling wave.
(c) The maximum particle velocity is twice the
(b) The velocity, wavelength and frequency of a 1
wave do not undergo any change when it is wave velocity if c =
a
(c) When an ultrasonic wave travels from air (d) The frequency of the wave is b.
into water, it bends towards the normal to 7. Two persons A and B, each carrying a source of
the air–water interface. sound of frequency 90 Hz are standing a few metres
(d) The velocity of sound is generally greater in B with
solids than in gases at STP. a speed u = v/10, where v is the speed of sound.
4. Which of the following statements are correct? Then
(a) The decrease in the speed of sound at high (a) A will hear 9 beats per second
altitudes is due to a fall is pressure. (b) A will hear 6 beats per second
(b) The standing wave on a string under (c) B will hear 12 beats per second
(d) B will hear 10 beats per second.
8. A wire of length L having linear density of 1.0
(c) The phenomenon of beats is not observed in 10–3 kg/m is stretched between two rigid supports
the case of visible light waves. with a tension of 40 N. It is observed that the wire,
(d) The apparent frequency is 1 when a source vibrating in p segments resonates at a frequency
of sound approaches a stationary observer of 420 Hz. The next higher frequency at which the
with a speed u and is 2 when the observer wire resonates is 490 Hz. The values of p and L are
approaches the same stationary source with (a) p = 6 (b) p = 7
the same speed. Then 2 < 1, if u < v, (c) L = (60/49) m (d) L = (10/7) m
where v is the speed of sound.
9.
5. Which of the following functions represent a sta- Lo
tionary wave? Here a, b and c are constants. pipe (of length Lc) with a beat frequency of 10 Hz.
(a) y = a cos (bx) sin (c t) The fundamental frequency of the closed pipe is
(b) y = a sin (b x) cos (ct) 110 Hz. If the speed of sound is 330 ms–1, then
14.34 Comprehensive Physics—JEE Advanced

(a) Lc = 0.75 m (c) he will hear two sounds of frequencies 403


(b) Lo = (33/34) m or (33/35) m Hz and 480 Hz.
(c) Lo = (33/32) m or 1 m (d) he will hear a range of frequencies between
(d) Lo= (33/32) m or (33/34) m 403 Hz and 480 Hz.
IIT, 1997 14. A wave is represented by the equation
10. In a resonance tube experiment, a tuning fork of y = A sin (10 x + 15 t + /3)
frequency 480 Hz resonates in the fundamental where x and y are in metre and t in second. The
mode with an air column of length 16 cm in a tube expression represents a wave
closed at one end. If the speed of sound in air is 336 (a) travelling in the positive x-direction
ms–1, the diameter of the tube is (b) travelling in the negative x-direction
(a) 5.6 cm (c) of wavelength 0.2 m
(b) 5.0 cm (d) of velocity 1.5 ms–1.
(c) either 5.0 cm or 5.6 cm IIT, 1990
(d) neither 5.0 cm nor 5.6 cm. 15. Two identical wires as a stretched with tensions
IIT, 2003 T1 and T2 with T1 > T2. They produce 6 beats per
second when vibrated. If the tension in one of them
11.
is changed slightly, it is observed that the beat fre-
all points on the string vibrate with
quency remains unchanged. Which of the follow-
(a) the same frequency and the same phase but
ing is/are possible?
different amplitude.
(a) T1 was increased (b) T1 was decreased
(b) the same frequency and the same amplitude (c) T2 was increased (d) T2 was decreased
but different phase.
(c) the same frequency but different phase and IIT, 1991
different amplitude. 16. A plane progressive wave of frequency 25 Hz, am-
(d) the same frequency, the same phase and the plitude 2.5 10–5 m and initial phase zero propa-
same amplitude. gates in a non-absorbing medium along the nega-
12. Sound waves travel from location A at absolute tive x-direction with a velocity of 300 ms–1. Then
temperature T1 to location B at absolute temperature (a) Wavelength of the wave is 12 m
T2 in time t. The air temperature increases lin- (b) The phase difference between the oscillations
early form T1 to T2. The speed of sound varies with at two points 6 m apart is
absolute temperature T as v = k T where k is a posi- (c) The corresponding amplitude difference is
tive constant. The distance between A and B is L. 2.5 10–5 m
dT T T (d) The corresponding amplitude difference is zero.
(a) The temperature gradient = 2 1. IIT, 1997
dx L
17. The (x, y) co-ordinates of the corners of a square
dT 1 plate are (0, 0), (L, 0), (L, L) and (0, L). The edges
(b) The temperature gradient = (T2 – T1).
dx 2L of the plate are clamped and transverse standing
kt waves are set up in it. If u(x, y) denotes the dis-
(c) L = T2 T1 placement of the plate at the point (x, y) at a certain
2
instant of time, the possible expression(s) for
kt u(x, y) is/are (a = positive constant)
(d) L = T2 T1
2 x y
13. A whistle emitting a sound of frequency 440 Hz (a) u(x, y) = a cos cos
2L 2L
is tied to a string of length 1.5 m and rotated with x y
an angular velocity of 20 rad s–1 in the horizontal (b) u(x, y) = a sin sin
L L
plane. An observer is stationed at a large distance
from the whistle. If the speed of sound is 330 ms–1, x 2 y
(c) u(x, y) = a sin sin
(a) he will hear two sounds of frequencies 403 L L
Hz and 440 Hz. 2 x y
(b) he will hear two sounds of frequencies 484 (d) u(x, y) = a cos sin
L 2L
Hz and 440 Hz.
IIT, 1998
Waves and Doppler’s Effect 14.35

18. A transverse sinusoidal wave of amplitude a, wave- 21. In a wave motion y = a sin (k x – t), y can represent
length and frequency f is travelling on a stretched
string. The maximum speed of any point on the (c) displacement (d) pressure
string is v/10, where v is the speed of propagation IIT, 1999
of the wave. If a = 10–3 m and v = 10 ms–1, then 22. Standing waves can be produced
(a) =2 10–2 m (b) = 10–3 m (a) on a string clamped at both the ends
103 (b) on a string clamped at one end and free at the
(c) f = Hz (d) f = 104 Hz
2 other
IIT, 1998
19. As a wave propagates in a non-absorbing medium, wall
(a) the wave intensity remains constant for a plane (d) when two incident waves with a phase differ-
wave ence of are moving in the same direction.
(b) the wave intensity decreases as the inverse of IIT, 1999
the distance from the source for a spherical 23. A student performed the experiment to measure the
wave speed of sound in air using resonance air-column
(c) the wave intensity decreases as the inverse method. Two resonances in the air-column were ob-
square of the distance from the source for a tained by lowering the water level. The resonance
spherical wave
(d) the total intensity of a spherical wave over a and that with the longer air-column is the second
spherical surface centred at the source remains resonance. Then,
constant at all times.
IIT, 1999 resonance was more than that at the second
20. A moving pulse is represented by the expression resonance
0.8 (b) the prongs of the tuning fork were kept in a
y(x, t) = horizontal plane above the resonance tube
[(4 x 5t ) 2 5]
(c) the amplitude of vibration of the ends of the
where x and y are in metre and t in second. Then prongs is typically around 1 cm
(a) the pulse is moving in the + x direction -
(b) in 2s it will travel a distance of 2.5 m nance was somewhat shorter than 1/4th of the
(c) its maximum displacement is 0.16 m wavelength of the sound in air
(d) it is a symmetric pulse.
IIT, 2009
IIT, 1999

ANSWERS AND SOLUTIONS


1. When a wave travelling in a medium falls on the v
=

into the second medium. Therefore, the intensity


same as that of the incident wave. But the wavelength
of the refracted wave will be different from that of
refracted waves will be less than that of the incident
the incident wave.
wave.
Furthermore, when a wave travelling in a rarer
The velocity of a wave depends upon the medium in
medium, it undergoes a phase change of or 180°.
wave will be the same as that of the incident wave.
But the velocity of the refracted wave will be dif- a rarer medium, it does not undergo any phase
ferent from that of the incident wave. The frequency change. The refracted wave, in both cases, does not
undergo any phase change. Thus the correct choice
same as that of the incident wave. From v = , we are (a) and (d).
have. 2. The correct choices are (a) and (b).
14.36 Comprehensive Physics—JEE Advanced

3. Statement (a) is correct. Let us write u


and 2 = 1 (ii)
y (x, t) = f (vt + x) = f (z) v
Differentiating with respect to t, we have Expression (i) may be written as
y f z f u 1
= =v 1 = 1
t z t z v
Differentiating again w.r.t time t we have Expanding binomially and retaining terms upto
2 2 order u2/v2, we have
y f
2
= v2 2 u u2
t z 1 = 1 (iii)
Similarly differentiating twice with respect to x v v2
we have 1 > 2.
2 2
y f 5. A stationary wave is characterized by a function
=
x 2
x 2 of the type y = f (t) g(x). Hence choices (a) and
Hence,
(b) represent a stationary wave. Choice (d) is a su-
2 2
y y perposition of two oppositely travelling waves of
= v2
t 2
x 2 the same amplitude and the same frequency which
gives rise to a stationary wave. Hence choice (d)
which is the standard equation (in differential form)
also represents a stationary wave.
of a travelling wave.
Statement (b) is also correct. Because the wave is 6. Comparing y = a sin 2 (bt – c x) with
2
y = a sin vt x
remains unchanged. The wavelength cannot change
we have
Statement (c) is incorrect. The ultrasonic wave 2 v 2
bends away from the normal because the speed of 2 b= and 2 c =
the wave (being a sound wave) is greater in water
1
than in air. which give v = b and = . Thus v = b/c.
Statement (d) is correct. The reason is that solids Particle velocity is c
have a much higher modulus of elasticity than gas- dy d
es at STP. V= = [a sin 2 (bt – cx)]
dt dt
4. Statement (a) is incorrect. A change in pressure = 2 ab cos 2 (bt – c x)
has no effect on the speed of sound. The decrease Vmax = 2 ab. Now Vmax = 2v, if
in the speed of sound at high altitudes is due to
2b
a fall in temperature. Statement (b) is correct. 2 ab =
Standing waves are produced due to superposi- c
1
which gives c = .
a
v b/c
- Frequency = = =b
1/ c
dent wave. Consequently, the resultant amplitude
Thus the correct choices are (b), (c) and (d).
at nodes is not exactly zero. Thus the nodes are not
7. Person A hears the sound of his own source whose
frequency is . He also hears the sound of the
Statement (c) is also correct. To observe beats the
source carried by person B, towards whom he is
difference between the two interfering frequencies
moving with a speed u. The apparent frequency of
must be less than about 10–16 Hz. Since visible
this sound is given by
light waves have very high frequencies, beats are
not observed due to persistence of vision. = 1
u
or – =
u
Statement (d) is correct. We know that v v
u 90v
1 = (i) Beat frequency b = – = =
1
u v 10v
v = 9 Hz ( u = v /10).
Waves and Doppler’s Effect 14.37

Person B hears the sound of his own source of fre- 3v


quency in . He also hears the sound of the source 2 =3 c = (4)
4 Lc
carried by person A, who is approaching with a
speed u. The apparent frequency of this sound is 330
Now 110 = Lc = 0.75 m.
given by 4Lc
v Given 1 – 2 = ±10. There are the following two
v = = possibilities.
u v±u
1 Case a : 1 – = 10. Thus
v 2

v u v 3v
or – = 1 = = 10
v u v u Lo 4 Lc
u Putting v = 330 ms–1 and Lc = 0.75 m, we get
Beat frequency = – =
b
v u 33
Lo = m
90 v /10 34
= = 10 Hz
v v /10 Case (b) : 1 – 2 = – 10. In this case, we get
Hence the correct choices are (a) and (d). 33
L = m
P T 32
8. 420 = (1)
2L m Hence the correct choices are (a) and (d).
10. Including end correction, we have for the funda-
p 1 T mental mode,
and 490 = (2)
2L m
Dividing (1) and (2) we have = L + 0.3 D or = 4 (L + 0.3 D)
4
490 p 1 where D is the diameter of the tube. Now v =
= giving p = 6.
220 p v
=
Substituting this value of p in Eq. (1) we get
v
p T 600 or 4 (L + 0.3 D) = (i)
420 = =
2L m L
Given L = 16 cm = 0.16 m, v = 336 ms–1 and
10 = 480 Hz. Using these values in (i) and solving
which gives L = m. Hence the correct choices
are (a) and (d). 7 we get D = 5 10–2 m = 5 cm. The correct choice
9. The fundamental frequencies of the open and closed is (b).
pipes respectively are 11. The correct choice is (a).
v
0= (1) dx
2 L0 12. Since v = , dx = vdt = k T dt. As the tem-
dt
v perature increases linearly, the rate of change of
and c= (2) temperature with distance is given by
4Lc
where v is the speed of sound. dT T T
= 2 1
In an open pipe, all harmonics are present. Hence dx L
the frequencies of the overtones are 2, 3, 4, ..... etc.
times the fundamental frequency. Hence the fre- where L is the distance between locations A and B.
Thus
2v v LdT
1 =2 = 0 = (3) dx =
2 L0 L0 T2 T1
In a closed pipe, only odd harmonics are present.
Hence, the frequencies of the overtones are 3, 5, 7, But dx = k T dt. Therefore
... etc. times the fundamental frequency. Hence the LdT
k T dt =
T2 T1
is
14.38 Comprehensive Physics—JEE Advanced

LdT If T1 is kept constant and T2 is increased such that


or dt = the new value 2 becomes greater than 1 by 6,
k (T2 T1 ) T
again 6 beats will be heard per second. Hence
Integrating from T = T1 to T = T2, we have choice (c) is correct.
T2 If T2 is kept constant and T1 is decreased such that
L –1/2
t= T the new value of 1 becomes less than 2 by 6, again
k T2 T1 T1 6 beats will be heard per second. Hence choice (b)
1/ 2 T2 is also correct.
t=
L T v 300
16. = = = 12 m
k (T2 T1 ) 1 / 2 25
2
T1
Phase difference = path difference
2L
= T2 T1
k (T2 T1 ) 2
= 6=
2L 12
=
k T2 T1 In a non-absorbing medium, the amplitude of the
Thus the correct choices are (a) and (c). wave remains constant as the wave propagates.
Thus the correct choices are (a), (b) and (d).
13. Linear velocity of source (whistle) is us = r = 17. The expression for u(x, y) must satisfy the follow-
1.5 20 = 30 ms–1. The observer will hear a range ing boundary conditions:
of frequencies lying between a minimum value vmin (i) u = 0 at x = 0 and at y = 0
and a maximum value max, which are given by (ii) u = 0 at x = L and at y = L.
v The choices (b) and (c) satisfy these conditions.
min =v ; source receding
v us 18. y = a sin ( t – kx)
Speed at a point on the string is
v
and vmax = v ; source approaching V=
dy
= a cos( t – kx)
v us dt
v
Substituting v = 440 Hz, v = 330 ms–1 and us = 30 Vmax = a = 2 af. Given Vmax = .
Hence 10
ms–1 and solving we get vmin = 403.3 Hz and vmax =
484 Hz. The correct choice is (d). v v 10
= 2 af f =
14. The standard equation of a wave travelling in the 10 20 a 20 10 3
negative x-direction is
103
y = A sin ( t + kx + 0) = Hz
2
2 v 2
where = ,k= and 0 is the phase at v 10
= = =2 10–2 m
x = 0 and t = 0. f 103 2
Comparing the given equation with this equation, So the correct choices are (a) and (c).
we have 19. The amplitude (and hence intensity) of a plane
2 wave remains constant as the wave propagates in
k = 10 = 10 = 0.2 m
a non-absorbing medium. For a spherical wave,
2 v the same energy crosses a spherical surface of area
and =15 = 15 4 r2 where r is the distance from the source. The
15 15 0.2
v== = 1.5 ms–1 unit area per second. Hence, for a spherical wave,
2 2 the intensity decreases as 1/r2. But the total inten-
Thus the correct choices are (b), (c) and (d). sity spread over the spherical surface is the same at
all times. Hence the correct choices are (a), (c) and
1 T
15. = . Since the wires are identical, (d).
2L 20. The displacement y(x, t) is maximum when the de-
T . Since T1 > T2; 1 > 2. Also 1 – 2 = 6. nominator is minimum, i.e.
Waves and Doppler’s Effect 14.39

[(4x + 5t)2 + 5] is minimum. Its minimum value is 22. Standing waves are produced due to a superposi-
5 when (4x + 5t)2 = 0 or 4x + 5t = 0
x 5 5
or = v= ms–1 clamped end of the string. In case (c) the incident
t 4 4 -
0.8 ing in the same direction cannot produce standing
ymax = = 0.16 m waves; they give rise to interference. Thus the cor-
5
rect choices are (a), (b) and (c).
Distance travelled by pulse in t = 2 s is 23.
5
vt = 2 = – 2.5 m tuning fork. At second resonance, the frequency
4
of the third harmonic equals the frequency of the
The negative sign shows that the pulse is travelling tuning fork. As the amplitude of oscillation of the
in negative x-direction. Since y is not a symmetric fundamental mode is the highest, the intensity of
function of x, the form of the pulses changes as it
travels. Hence the correct choices are (b) and (c). Hence choice (a) is correct. Choice (b) is wrong,
21. In an electromagnetic wave, y represents electric the prongs are kept in a vertical plane. Choice (c)
-
is also incorrect as the amplitude of vibration of
pendicular to each other as well as perpendicular
the ends of the prongs is typically around 1 mm.
to the direction of propagation of the wave. In a
Choice (d) is correct. Due to end-correction (= e),
mechanical wave, y represents displacement. In a
sound wave, y represents pressure. Thus all the four (L1 + e) =
choices are correct. choices are (a) and (d).

III

Multiple Choice Questions Based On Passage

Questions 1 to 3 are based on the following passage 1. What is the speed of sound in the medium?
Passage I (a) 332 ms–1 (b) 100 ms–1
–1
When two sound waves travel in the same direction in a (c) 92 ms (d) 200 ms–1
medium, the displacements of a particle located at x at 2. How many times per second does an observer hear
time t is given by the sound of maximum intensity?
(a) 4 (b) 8
y1 = 0.05 cos (0.50 x – 100 t) (c) 12 (d) 16
and y2 = 0.05 cos (0.46 x – 92 t) 3. At x = 0, how many times between t = 0 and t = 1 s
does the resultant displacement become zero?
where y1, y2 and x are in metre and t in second (a) 46 (b) 50
IIT, 2006 (c) 92 (d) 100

SOLUTION
1. The two displacements can be written as 2 =2 2 = 92 rad s–1. The speed of either wave
y1 = A cos (k1x – (1) is
1t)
and y2 = A cos (k2x – 2t) (2) 1 1
v1 = 1 1 =2 1 =
2 –1 2 k1
where A = 0.05 m, k1 = = 0.50 m ,
1
2 100
1=2 1 = 100 rad s1, k2 = = 0.46 ms–1 and = = 200 ms–1
2 0.50
14.40 Comprehensive Physics—JEE Advanced

92 = A cos (k1x – 1t) + A cos (k2x – 2t)


or v2 = 2 2 = 2
= = 200 ms–1.
k2 0.46 For x = 0, we have
Hence the correct choice is (d). y = A cos + A cos
1t 2t
2. Beat frequency = – 2. Now
1 1
= 2 A cos ( 1 2 )t
1 100 2
1= = = 50 Hz
2 2 1
cos ( 1 2 )t
2 92 2
and 2= = = 46 Hz
2 2 y = 0.10 cos (96 t) cos (4 t)
Beat frequency = 50 – 46 = 4 Hz. Hence the cor- Between t = 0 and t = 1 s, cos (96 t) becomes
rect choice is (a). zero 96 times and cos (4 t) becomes zero 4
3. The resultant displacement is given by times. Hence the resultant displacement y at x = 0
y = y1 + y2 becomes zero 100 times between t = 0 and t = 1 s.
The correct choice (d).

Questions 4 to 6 are based on the following passage 4. The frequency of the fundamental mode of the
Passage II closed pipe is
(a) 100 Hz (b) 200 Hz
A string 25 cm long and having a mass of 2.5 g is under (c) 300 Hz (d) 400 Hz
of tension. A pipe closed at one end is 40 cm long. When 5.
overtone is
the pipe in its fundamental frequency, 8 beats per second (a) 92 Hz (b) 108 Hz
heard. It is observed that decreasing the tension in the (c) 192 Hz (d) 208 Hz
string decreases the beat frequency. The speed of sound 6. The tension in the string is very nearly equal to
in air is 320 ms–1. (a) 25 N (b) 27 N
(c) 28 N (d) 30 N
IIT, 1982

SOLUTION
4. Frequency of fundamental mode of the closed pipe is It is given that the beat frequency decreases if the
v 320 tension in the string is decreased. As the frequency
np = = = 200 Hz
4L 4 0.40 decreases with decrease of tension, it is obvious
The correct choice is (b). that ns > np. Hence
5. Since the beat frequency is 8, the frequency of the ns = 208 Hz and not 192 Hz.
Thus the correct choice is (d).
ns = np ± 8 = 200 ± 8 = 192 Hz or 208 Hz
6. Substituting the values of l, m and ns in Eq. (1), we
1 T get T = 27.04 N. Hence the correct choice is (b).
ns = (1)
l m

Questions 7 to 11 are based on the following passage 7. The frequency of vibrations of the string is
Passage III (a) 48 Hz (b) 50 Hz
(c) 96 Hz (d) 100 Hz
ends are represented by the equation 8. The maximum displacement of a point at x = 10 cm
x is
y = 4 sin cos (96 t)
15 (a) 2 cm (b) 4 cm
where x and y are in cm and t in second. (c) 2 3 cm (d) 4 3 cm
IIT, 1985
Waves and Doppler’s Effect 14.41

9. How many nodes are formed on the string? 11. The velocity of the particle at x = 7.5 cm at t = 0.25 s
(a) 2 (b) 3 is
(c) 4 (d) 5 (a) zero (b) 320 ms–1
10. In which harmonic mode is the string vibrating? (c) 60 ms –1
(d) 96 ms–1
(a) Fundamental (b) third

SOLUTION
7. 2 = 96 = 48 Hz. Thus the correct choice 10. The correct choice is (c) because 5 nodes are formed
is (a) on the string.
8. Displacement is maximum when cos (96 t) = 1. 11. The velocity of the string at a point x at time t is
10 2 obtained by differentiating
At x = 10 cm, ymax = 4 sin = 4 sin
15 3 x
y = 4 sin cos (96 t)
= 2 3 cm 15
The correct choice is (c).
which respect to t.
9. At nodes the displacement is always zero. Hence
nodes are located at values of x given by dy x
Velocity = – (4 96 ) sin sin (96 t)
x x dt 15
sin = 0 or =p
15 15 At x = 7.5 cm and t = 0.25 s, the velocity is zero
where p = 0, 1, 2, 3, ... etc. because at t = 0.25 s, sin (96 t) = sin (24 ) = 0.
Thus x = 15 p = 0, 15, 30, 45 and 60 cm. Hence the correct choice is (a).
Thus the correct choice is (d).

Questions 12 to 15 are based on the following passage 13.


Passage IV (a) y2 = 0.8 A cos (– ax + bt)
The displacement of the medium in a sound wave is (b) y2 = – A cos (– ax + bt)
given by
(c) y2 = – 0.64 A cos (– ax + bt)
y1 = A cos (ax + bt)
(d) y2 = – 0.8 A cos (– ax + bt)
where A, a and b are positive constants. The wave is
x = 0. The intensity of 14. In the standing wave formed due to the superposi-
-
mum values of the particle speed in the medium is
IIT, 1991
(a) Ab (b) 1.64 Ab
12. The wavelength and frequency of the incident wave
respectively are (c) 1.8 Ab (d) 2 Ab
15. The minimum value of the particle speed in the
2 b a 2 medium is
(a) , (b) ,
a 2 2 b
(a) zero (b) 0.2 Ab
2 b 1
(c) , (d) a, (c) 0.64 Ab (d) 0.8 Ab
a 2 b

SOLUTION
12. The incident wave is given by 2 2
b. Hence wavelength = = and frequency
y1 = A cos (ax + bt) (1) b k a
The standard wave equation is = = .
2 2
y = A cos (kx + t) (2) The correct choice is (a).
where k is the wave number and , the angular fre- 13.
quency. Comparing (1) and (2) we get k = a and = times that of the incident wave, the amplitude Ar of
14.42 Comprehensive Physics—JEE Advanced

Differentiating Eq. (3) with respect to t, the particle


0.64 = 0.8 times that
of the incident wave, i.e.
Ar = 0.8 A d y2
V2 =
dt
suffers a reversal of amplitude (which implies a
phase change of radian), i.e. Ar = – 0.8 A. Since d
[– 0.8 A cos (bt – ax)]
the incident wave is travelling in the negative x di- dt
= 0.8 Ab sin (bt – ax)
x
(V 2)max = 0.8 Ab
wave can be obtained from Eq. (1) by replacing A
by Ar = – 0.8 A and x by – x From the superposition principle, the maximum
wave is given by particle speed in the medium due to both the inci-
y2 = – 0.8 A cos (– ax + bt) (3)
Thus the correct choice is (d). sum of the individual maximum particle speeds, i.e.
14. Differentiating Eq. (1) will respect to time t, we get Vmax = (V1)max + (V2)max
the expression for the particle speed in the medium = Ab + 0.8 Ab = 1.8 Ab
due to the incident wave, which is
d y1 d So the correct choice is (c).
V1 = {A cos (a x + bt)} 15. Since A and b are positive constants, the minimum
dt dt
particle speed is
= – Ab (a x + bt) (4)
Vmin = (V1)max – (V2)max
Maximum particle speed due to the incident
= Ab – 0.8 Ab = 0.2 Ab
wave is
(V1)max = Ab The correct choice is (b).

Questions 16 to 19 are based on the following passage (a) P0 (b) 2 P0


Passage V P0
(c) 2 P0 (d)
The air column in a pipe closed at one end is made to 2
vibrate in its second overtone by a tuning fork of frequency 18. The maximum and minimum pressures at the open
400 Hz. The speed of sound in air is 320 ms–1. The end end of the pipe respectively are
correction may be neglected. Let P0 denote the mean (a) P0 + P0, P0 – P0
pressure at any point in the pipe and P0 the maximum (b) P0 + P0, P0
amplitude of pressure variation. (c) P0, P0 – P0
IIT, 1998 (d) P0, P0
16. The length L of the air column is 19. The maximum and minimum pressures at the closed
end of the pipe respectively are
(a) 20 cm (b) 60 cm
(a) P0 + P0, P0 – P0
(c) 1.0 m (d) 1.4 m (b) P0 + P0, P0
17. The amplitude of pressure variation at the middle (c) P0, P0 – P0
of the air column is (d) P0, P0

SOLUTION
16. Figure 14.13 shows the longitudinal displacement The fundamental frequency of a closed pipe is giv-
y as a function of x for x lying between x = 0 and en by
x = L, where L is the length of the pipe. v
=
4L
In a closed pipe, only odd harmonics are present,
i.e. the frequency v1
the fundamental frequency, that of the second over-
tone v2 is 5 times the fundamental frequency and so
on. Thus
Fig. 14.13
Waves and Doppler’s Effect 14.43

5v
2 =5 = . Therefore, the distance of C from N2 is x =
4L 2
5v 5 320 5
L= = = 1.0 m . Using this value of x in Eq. (1) we
4 2 4 400 8 2 8
The correct choice is (c). have
17. Since the distance between two consecutive 2
P at C = P0 cos
nodes is and that between a node and the next 8
2
P0
antinode is , it follows from Fig. 14.7 that = P0 cos =
4 4 2
Thus the correct choice is (d).
5
L= 18. At an antinode, the pressure variation is zero, i.e.
2 2 4 4
P0 = 0. Hence, at an antinode
We know that the pressure variation (excess pres-
sure) is maximum at a node and minimum (equal to Pmax = Pmin = P0
zero) at an antinode. Therefore, the pressure varia- So the correct choice is (d).
tion at a distance x from a node is given by 19. At a node, the pressure variation is maximum equal
2 x to P0. Hence, at a node
P = P0 cos (1)
Pmax = P0 + P0 and
L 5
The centre C of the tube is at x = or x = Pmin = P0 – P0
5 2 8
since L . The second node N2 is at x = Thus the correct choice is (a).
4

Questions 20 to 22 are based on the following passage 20. The tension T in the wire is
Passage VI (a) 0.98 N (b) 9.8 N
A wire of mass 9.8 10 –3
kg per metre passes over a (c) 98 N (d) 980 N
21. The value of mass M1 is
plane which makes an angle of 30° with the horizontal. (a) 2 kg (b) 5 kg
Two masses M1 and M2 are tied at the two ends of the (c) 10 kg (d) 20 kg
wire. Mass M1 rests on the inclined plane and mass M2 22. The value of mass M2 is
hangs freely vertically downwards. The whole system is
(a) 5 kg (b) 10 kg
in equilibrium. Now a transverse wave propagates along
(c) 15 kg (d) 20 kg
the wire with a speed of 100 ms–1.

SOLUTION
20. Refer to Fig. 14.14. Let T be the tension in the string balances with the normal reaction N and the other
when the system is in equilibrium. component M1 g sin will balance with tension T
in the string. Also weight M2 g of mass M2 will bal-
position, the component M1g cos of weight M1 g ance with tension T. Thus
M1 g sin = T (1)
and M2 g = T (2)
Now, the speed of a transverse wave in a wire of
mass m per unit length and stretched with a tension
T is given by
T
v=
m
or T = v2 m (3)
Fig. 14.14
14.44 Comprehensive Physics—JEE Advanced

Given m = 9.8 10–3 kg m–1 and v = 100 ms–1. Thus the correct choice is (d).
Using these values in Eq. (3), we have 22. From Eq. (2), we have
T = (100)2 9.8 10–3 = 98 N
21. From Eq. (1), we get T 98
M2 = = = 10 kg, which is choice (b).
T 98 g 9.8
M1 = = = 20 kg
g sin 9.8 sin 30

Questions 23 to 25 are based on the following passage 24. The beat frequency of the sound heard by an ob-
Passage VII server behind the source is
A source of sound of frequency 90 Hz is moving towards a 200
(a) Hz (b) 20 Hz
wall with a speed u = v/10, where v is the speed of sound 9
in air. 200
IIT, 1981 (c) Hz (d) zero
11
23. The beat frequency of the sound heard by an 25. The beat frequency of the sound heard by the
observer between the wall and the source is observer moving with the source is
(a) 20 Hz (b) 10 Hz (a) 11 Hz (b) 9.9 Hz
(c) 5 Hz (d) zero (c) 10 Hz (d) 20 Hz

SOLUTION

23. The observer hears two sounds–one coming v 900


directly from the approaching source and the other = 90 = Hz
v v /10 11
considered as coming from the mirror image of the 900
source). The apparent frequency of the approach- Beat frequency = – = 100 –
11
ing source is
200
v v = Hz.
= = 90 = 100 Hz 11
v us v v/10
Hence the correct choice is (c).
When the observer is between the wall and the
25. If the observer is moving with the source, the fre-
quency of the direct sound is v = 100 Hz. The ap-
from the wall is also . Therefore, frequency of
beats = – = 0. The observer will not hear any
beats. v us
So the correct choice is (d). =
v us
24. When the observer is behind the source, i.e. when
the source is between the wall and the observer, the v + v /10
= 90 = 110 Hz
apparent frequency of the sound coming directly v v /10
from the receding source is
v Beat frequency = – = 110 – 90 = 20 Hz
= So the correct choice is (d).
v us
Waves and Doppler’s Effect 14.45

IV

Matrix Match Type

43. Column I shows four systems, each of the same length L, for producing standing waves. The lowest possible
natural frequency of a system is called its fundamental frequency, whose wavelength is denoted as f. Match each
system with statements given in Column II describing the nature and wavelength of the standing waves.
Column I Column II
(a) Pipe closed at one end (p) Longitudinal waves

(b) Pipe open at both ends (q) Transverse waves

(c) Stretched wire clamped at both ends (r) f =L

(d) Stretched wire clampeed at both ends (s) f = 2L


and at mid-point

(t) f = 4L
IIT, 2011

SOLUTION
(a) The closed end of a pipe is a node and the open end is an antinode. The distance between a node and the
next antinode is /4. Hence f = 4L. In pipes, the standing waves are due to superposition of oppositely
travelling sound (longitudinal) waves.
(a) (p, t)
f
(b) Here L = f = 2L ( distance between consecutive antinodes = /2)
2
(b) (p, s)

/2, f = 2L. Also standing waves on a string are due to superposition of transverse waves.
(c) (q, s)
L
(d) Here the mid point is a node. Hence = f f =L
(d) (q, r) 2 2

ANSWERS
(a) (p, t) (b) (p, s) (c) (q, s) (d) (q, r)
14.46 Comprehensive Physics—JEE Advanced

Assertion-Reason Type Questions

In the following questions, Statement-1 (Assertion) is 6. Statement-1


followed by Statement-2 (Reason). Each question has the Our ears cannot distinguish two notes, one produced
following four choices out of which only one choice is by a violin and other by a sitar, if they have exactly
correct. the same intensity and the same frequency.
(a) Statement-1 is true, Statement-2 is true and State- Statement-2
ment-2 is the correct explanation for Statement-1. When a musical instrument is played, it produces a
(b) Statement-1 is true, Statement-2 is true but State- fundamental note which is accompanied by a num-
ment-2 is not the correct explanation of State- ber of overtones called harmonics.
ment-1.
7. Statement-1
(c) Statement-1 is true, Statement-2 is false.
(d) Statement-1 is false, Statement-2 is true. Doppler’s effect does not occur in case of a super-
sonic source.
1. Statement-1
Statement-2
Only longitudinal mechanical waves can propagate
A supersonic source produceds a shock wave.
in gases.
Statement-2 8. Statement-1
Gases have only bulk modulus. If a source of sound moves always from a stationary
2. Statement-1 observer, the apparent frequency of sound as heard by
the observer is greater than the actual frequency.
Two sound waves of equal intensity I produced
Statement-2
beats. The maximum intensity of sound produced
The cause of the apparent change in frequency is
in beats is 4I.
the change in the wavelength brought about by the
Statement-2 motion of the source.
If two waves of amplitudes a1 and a2 superpose, the 9. Statement-1
maximum amplitude of the resultant wave = a1 + a2. If an observer moves towards a stationary source of
3. Statement-1 sound, the frequency of the sound as heard by him
A medium must possess elasticity in order to sup- is greater than the actual frequency.
port wave motion. Statement-2
Statement-2 The apparent increase in frequency is due to the
Restoring force does not exist in a medium which fact that the observer intercepts more waves per
does not have elasticity. second when the moves towards the source.
4. Statement-1 10. Statement-1
Solids can support both longitudinal and transverse If a source of sound is in motion and the observer
mechanical waves but only longitudinal mechani- is stationary, the speed of sound relative to him
cal waves can propagate in gases. remains unchanged.
Statement-2 Statement-2
Gases do not have shear modulus. The apparent change in frequency is due to the
5. Statement-1 change in the wavelength brought about by the
In standing sound waves, a displacement node is a motion of the source.
pressure antinode and vice versa. 11. Statement-1
Statement-2 If the observer is in motion and the source of sound
In a standing wave, the restoring force is the maxi- is stationary, the speed of sound relative to him is
mum at a node and minimum at an antinode. changed.
Waves and Doppler’s Effect 14.47

Statement-2 tionary observer with a certain velocity and (ii)


The wavelength of sound received by the observer observer approaching a stationary source of sound
does not change due to his motion. with the same velocity.
12. Statement-1 Statement-2
The apparent frequency is not the same in the The cause of the apparent change in the frequency
following two cases– (i) source approaching a sta- is different in the two cases.

SOLUTION
1. The correct choice is (a). Gases cannot withstand which is accompanied by a number of overtones
a shearing stress or longitudinal stress. Hence they called harmonics. The number of harmonics is not
do not have shear modulus and Young’s modulus; the same for all instruments. It is the number of
they have only bulk modulus. harmonics which distinguishes the note produced
2. The correct choice is (a). When two waves of am- by a sitar and that produced by a violin.
plitudes a1 and a2 superpose to produce beats, the 7. The correct choice is (a). If the source of sound is
resultant amplitude of the maximum of intensity is moving at a speed greater than the speed of sound,
A = a1 + a2 then in a given time the source advances more than
Now, intensity (amplitude)2. Since the two the wave. The resultant wave motion is a conical
waves have the same intensity, their amplitudes are wave called a shock wave which produces a sudden
equal, i.e. a1 = a2 = a. Thus A = 2a. Therefore, A2 and violent sound.
= 4a2 or Imax = 4I. 8. The correct choice is (d).
3. The correct choice is (a). 9. The correct choice is (a).
4. The correct choice is (a). Gases cannot withstand a 10. The correct choice is (a).
shearing stress. Hence gases do not have any shear
11. The correct choice is (a).
modulus; they have only bulk modulus. Solids have
Young’s modulus, bulk modulus and shear modu- 12. The correct choice is (a). In case (i) the speed of
lus. Therefore, solids can support both transverse sound relative to the observer remains unchanged;
and longitudinal waves. the change in frequency is due to a change in wave-
5. The correct choice is (c). length brought about by the motion of the source.
6. The correct choice is (d). When a musical instru- In case (ii) the wavelength of sound remains un-
ment is played, it produced a fundamental note changed; the change in frequency is due to a change
in the speed of sound relative to the observer.

VI

Integer Answer Type

1. An ambulance sounding a horn of frequency 256


Hz is moving towards a vertical wall with a veloc- vibration in Hz. The Young’s modulus of steel
ity of 5 ms–1. If the speed of sound is 330 ms–1, how = 2 1011 Nm–2
many beats per second will be heard by an observer of steel = 1.2 10–5 per °C.
standing a few metres behind the ambulance? IIT, 1981
3.
IIT, 1981
at both the ends. The tension in the string is 0.5 N.
2. A steel wire of length 1 m, mass 0.1 kg and uniform
The string is set into vibrations using an external vi-
cross-sectional area 10–7 m2
brator of frequency 100 Hz. Find the separation (in
ends. The temperature of the wire is decreased by
cm) between the successive nodes on the string.
80/3°C. If transverse waves are set up in the wire,
IIT, 2009
14.48 Comprehensive Physics—JEE Advanced

SOLUTIONS
1. The observer will hear a sound of the source Frequency of fundamental mode is
moving away from him and another sound after 1 T
= ; = mass per unit length of wire
2L
of these sounds are
v 256 330 1 6.4
1 = = = 252 Hz = = 4 Hz
v u (330 5) 2 1 0.1
v 256 330 3. Mass per unit length of the string is
and 2 = = = 260 Hz 3
v u (330 5) 1.0 10
m= = 5 10–3 kg m–1
No. of beats per second (beat frequency) = 260 20 10 2
– 252 = 8 Speed of waves in the string is
2. Contraction L = L
T 0.5
TL v= = 3
= 10 m s –1
Young’s modulus Y = ; T = tension m 5 10
A L
v 10
YA L Now v= = = = 0.1 m
T= = YA 100
L = 10 cm
80
=2 1011 10–7 1.2 10–5 Separation between successive nodes
3
= 6.4 N = = 5 cm
2
Thermal Expansion 15.1

15
Chapter
Thermal Expansion

REVIEW OF BASIC CONCEPTS 15.2 APPLICATIONS


(i)
15.1 THERMAL EXPANSION
mass
Density = or =
If the temperature of a body is increased, its length, surface volume
area and volume all increase. Since mass remains constant, = constant.
(i) If the body is in Thus
the form of a rod, the increase in its length when = 0 0
the temperature is increased by T is proportional
to ( ) the original length and (b) increase in tem- 0 0 0 0 0
perature T, i.e. = = =
0 1 (1 T)
T = T
Thus density of a substance decreases with increase
Thus = in temperature.
(ii)
where If a rod is held between two rigid supports and its
material of the body. Thus temperature is increased or decreased, the rigid
= 0 (1 + T) supports prevent the rod from expanding or con-
(ii) tracting. As a result, a stress (called thermal stress)
is developed in the rod. The change in length of the
Similarly = rod is
= T
where
Strain = T
= 0 (1 + T)
(iii) stress
Now Young’s modulus (Y) =
strain
=
Thermal stress = =Y T
= 0 (1 + T)
The SI unit of , and is (°C)–1 or K–1. (iii)
(iv) , A linear metallic scale expands when heated and
contracts when cooled.
: : =1 : 2 : 3
A reading of 1 unit of a heated scale is equivalent to a
i.e. =2 = 3 . an actual length of 1 unit (1 + T) where T is the
15.2 Comprehensive Physics—JEE Advanced

rise in temperature. If the reading of the heated scale 15.1


is units, the actual length = (1 + T) units.
If the temperature is decreased by T, the actual At 20°C, a brass rod has a length 50.0 cm. It is
reading = (1 – T) units. joined to a steel rod of the same length and the same
(iv) Consider two metal rods of lengths 1 and 2. Let diameter at the same temperature. Find the change
in the length of the composite rod when it is heated
1 and 2 be their lengths when their temperature
is increased by T, then to 220°C. For brass = 2 10–5 K–1 and for steel
= 1 10–5 K–1.
1 = 1(1 + 1 T))
and 2 = 2(1 + 2 T) SOLUTION
where 1 and 2 For brass:
linear expansion. The difference of their lengths is ( )b = b b T
–5
2 – 1 = 2(1 + 2 T) – 1(1 + 1 T) = (2 10 ) 0.5 (220 – 20)
–3
Their difference in lengths will remain constant if =2 10 m
2 – 1 = 2 – 1 = 2 mm
For steel:
i.e. if 1 1 = 2 2
( ) = T
(v)
= (1 10–5) 0.5 (220 – 20)
The time period of a simple pendulum is given by = 1 mm
=( )b + ( ) = 3 mm
T= 2

If the room temperature rises by T, the length 15.2


of the pendulum increases. Hence the time period A metal wire of cross-sectional area 5 10–6 m2 is
increases which implies that a metallic pendulum held taut at 30°C between two rigid supports with
clock slows down. If is the length of heated negligible tension in it. Find the tension developed in
clock, then its time period becomes the wire if it is cooled to –20°C. Given of metal =
2 10–5 K–1 and Y = 1 1011 N m–2.
T = 2
SOLUTION
T (1 ) Thermal stress = Y T
= =
T F
= (1 + T)1/2 Also stress =

1 F= T
= 1+ T( T is small)
2 = (1 1011) (5 10–6)
T 1 T (2 10–5) [30 – (– 20)]
T =
T 2 = 500 N
T 1
= T
T 2 15.3
This gives the time lost per second.
1 5 10–4 K–1. If its temperature is increased by 30°C,
Time lost in one day = T (60 60 24)
2
1
T =
86400 s SOLUTION
2
0
If the room temperature falls by T, then =
(1 T)
1
Time gained in one day = T 86400 s (1 + T) = 0 – 0 =– 0 T
2
Thermal Expansion 15.3

or 0
=– T 15.6
0 A steel tape gives correct readings at a temperature
=–5 10–4 30 = – 1.5 10–2. of 15°C. On a day when the temperature is 40°C,
this tape measures the distance between two points as
The negative sign indicates that the density decreases
3152 cm. What is the actual distance between the two
with increase in temperature.
points? Given for steel = 1.2 10–5 K–1.
Percentage change in density = (1.5 10–2)
100 = 1.5% SOLUTION
T = 40 – 15 = 25°C
15.4
Actual distance = measured distance (1 + T)
A block of mass 248 g and volume 205 cm3 –5
liquid contained in a vessel. The density of the liquid = 3152 (1 + 1.2 10 25)
at 0°C is 1.248 g cm–3. It is found that the block just = 3152.9 cm 3153 cm
sinks in the liquid if the temperature is raised to 50°C.
- 15.7
A metal pendulum clock gives correct time at a
expansion of the block is negligible compared to that
temperature of 20°C. How much time does it lose in a
of the liquid.
day if the room temperature is 35°C. Given of metal
SOLUTION = 1.25 10–5 K–1.
The block will just sink in the liquid if its density SOLUTION
becomes equal to the density of the liquid at 50°C.
1
Density of block Time lost per second = T
2
248 g
= = 1.210 g cm–3 1
205 cm 3 Time lost in a day = T (24 60 60)s
2
0 = (1 + T) 1
= 1.2 10 5 15
1.248 = 1.210 (1 + 50) 2
= 6.28 10 –4
(°C)–1 or K–1 (24 60 60)
= 8.1 s
The quantity T
Hence the value of must be rounded off to two sig-
15.8

= 6.3 10–4 K–1


bottle and water are heated to 80°C if
15.5 (a) the expansion of bottle is neglected
(b) the expansion of bottle is not neglected
same material. At 25°C, the diameter of the shaft is
8.00 cm and the diameter of the hole is 7.99 cm. The of water = 6.0 10–4(°C)–1, of glass
–5 –1
shaft is cooled using ‘dry ice’. At what temperature = 0.6 10 (°C) .
of the shaft does the wheel slip into the shaft. Give
SOLUTION
for steel = 1 10–5 K–1.
Volume of bottle = volume of water = 1 litre = 1000 cm3
SOLUTION
Decrease in temperature
0.01cm (a) If the expansion of bottle is neglected, the
T= = 5
= – 125°C
1 10 8.00 cm
= 1000 (1 + T) – 1000
T2 – T1 = –125 T2 = –125 + T1 = –125 +
= 1000 (1 + 6.0 10–4 60) – 1000
25 = –100°C
= 1036 – 1000 = 36 cm3
15.4 Comprehensive Physics—JEE Advanced

(b) = 3 = 3 0.6 10–5 = 1.8 10–5(°C)–1 = 1000 ( – ) T


–4
If the expansion of bottle is not neglected, the = 1000 (6.0 10 – 1.8 10–5) 60
3
= 34.9 cm
= 1000 (1 + T) – 1000 (1 + T)

Multiple Choice Questions with Only One Choice Correct


1. When a solid metallic sphere is heated, the largest 1 T 1 T
2 2
percentage increase occurs in its (a) (b)
1 T 1 T
(a) diameter (b) surface area 1 1

(c) volume (d) density 1 2 T 2


(c) (d)
2. A steel scale measures the length of a copper rod 1 1 T 1
as cm when both are at 20°C, the calibration IIT, 1991
6. A thin copper wire of length increases its length
linear expansion for steel and copper are and
by 1% when heated from temperature T1 to T2.
respectively, what would be the scale reading
What is the percentage change in area when a thin
(in cm) when both are at 21°C?
copper plate having dimensions 2 is heated
1 from T1 to T2?
(a) (b)
1 (a) 1% (b) 2%
(c) 3% (d) 4%
(c) (d) 7.
direction ( -axis) is 2.0 10–6 K–1 and that in the
3. Two uniform brass rods and B of lengths and other two perpendicular ( -and z-axes) directions
2 and radii 2 and respectively are heated to the is 1.6 10–6 K–1
same temperature. The ratio of the increase in the expansion of the crystal?
length of to that of B is (a) 1.6 10–6 K–1 (b) 1.8 10–6 K–1
(a) 1 : 1 (b) 1 : 2 (c) 2.0 10–6 K–1 (d) 5.2 10–6 K–1
(c) 1 : 4 (d) 2 : 1 8. A metal ball immersed in alcohol weighs W1 at
4. 0°C and W2
of thermal expansion 1 and 2 and Young’s modulii expansion of metal is less than that of alcohol. If
Y1 and Y2 the density of the metal is large compared to that
walls. The rods are heated to the same tempera- of alcohol, then
ture. If there is no bending of the rods, the thermal (a) W1 > W2 (b) W1 = W2
stresses developed in them are equal provided W1
Y Y (c) W1 < W2 (d) W2 =
(a) 1 = 1
(b) 1 = 2 2
Y2 2 Y2 1 IIT, 1980
Y1 Y1 9. A rod of length 20 cm made of a metal expands
1 2
(c) = (d) = by 0.075 cm when its temperature is raised from
Y2 2 Y2 1 0°C to 100°C. Another rod of a different metal B
IIT, 1989 having the same length expands by 0.045 cm for
5. the same change in temperature. A third rod of the
of volume expansion of the metal and mercury same length is composed of two parts, one of metal
are 1 and 2 respectively. If their temperature is and the other of metal B. This rod expands by
increased by T, the fraction of the volume of metal 0.060 cm for the same change in temperature. The
submerged in mercury changes by a factor portion made of metal has length
Thermal Expansion 15.5

(a) 20 cm (b) 10 cm and 2 -


(c) 15 cm (d) 18 cm ear expansion of the composite rod will be
10. A metallic circular disc having a circular hole at 1
its centre rotates about an axis passing through its (a) ( 1 + 2) (b) 1 2
2
centre and perpendicular to its plane. When the disc
is heated, its 1 1 2 2 1 2 1 2
(c) (d)
(a) moment of inertia increases but angular speed 1 2 1 2
decreases
16. -
(b) moment of inertia decreases but angular speed
neous rod changes linearly from 1 to 2 from one
increases
end to the other end of the rod. The effective coef-
(c) moment of inertia and angular speed both
increase
(d) moment of inertia and angular speed both 1
(a) ( 1 + 2) (b) ( 1 + 2)
decrease. 2
11. A uniform metal rod of length and mass M is (c) 1 2 (d) ( 1 2)
rotating with angular speed about an axis passing 17. Three rods of the same length are arranged to form
through one of the ends and perpendicular to the an equilateral triangle. Two rods are made of the
rod. If the temperature increases by ºC, then the
1
change in its angular speed is proportional to and the third rod which forms the base of the trian-
(a) (b) 2. The altitude of
1 the triangle will remain the same at all temperatures
2
(c) (d) if the 1/ 2 is nearly
1
12. A steel metre scale is to be ruled so that the mil- (a) 1 (b)
2
limetre intervals are accurate to about 5 × 10–5 m at
1
a certain temperature. The maximum temperature (c) (d) 4
4
of linear expansion of steel = 10 × 10–6 K–1) 18.
(a) 2 ºC (b) 5 ºC 80% of its volume immersed in a liquid at 0ºC. When
the temperature of the liquid is raised to 62.5ºC, the
(c) 7 ºC (d) 10 ºC
-
13. When the temperature of a rod increases from to cal expansion of liquid is
+ , the moment of inertia of the rod increases
(a) 1 × 10 3 K 1 (b) 2 × 10 3 K 1
from I to I + I -
I (c) 3 × 10 3 K 1 (d) 4 × 10 3 K 1
sion of the rod is , the ratio is 19.
I
fraction 1 of its volume is submerged, while at the
2
(a) (b) temperature 60°C, a fraction 2 is seen to be sub-

(c) (d) 2 iron is Fe and that of mercury is Hg, then the ratio
14. Two spheres made of the same material have the 1/ 2 can be expressed as
same diameter. One sphere is hollow and the other 1 60 1 60
Fe Fe
is solid. If they are heated through the same range (a) (b)
1 60 Hg 1 60 Hg
of temperature,
(a) the hollow sphere will expand more than the 1 60 Fe
1 60 Hg
solid sphere (c) (d)
1 60 Hg 1 60 Fe
(b) the solid sphere will expand more than the
hollow sphere IIT, 2001
(c) both spheres will expand equally 20. Two rods, one made of aluminium and the other
(d) the hollow sphere will not expand at all. made of steel, having initial lengths 1 and 2
15. Two rods of lengths 1 and 2 are welded together respectively are connected together to form a single
to make a composite rod of length ( 1 + 2). If the rod of length ( 1 + 2
expansion for aluminium and steel are 1 and 2
1
15.6 Comprehensive Physics—JEE Advanced

respectively. If the length of each rod increases by


the same amount when their temperature is raised expansion of the metal is . If the temperature of
by °C, then the ratio 1/( 1 + 2) the disc is increased by , the angular frequency of
(a) 1/ 2 (b) 2/ 1 rotation of the disc will
(c) 2/( 1 + 2) (d) 1/( 1 + 2) (a) remain unchanged
IIT, 2003 (b) increase by
21.
(c) increase by 2
18 10–5/°C. A thermometer has a bulb of volume
10–6 m3 and the cross-sectional area of the stem (d) decrease by 2
is 0.002 cm2 26. A vertical glass tube, closed at the bottom, contains
mercury when the temperature is 0°C. When the a mercury column of length 0 at 0°C. If is the
the
temperature rises to 100°C, the length of the mer-
cury column in the stem will be
of the mercury column when the temperature rises
(a) 9 cm (b) 18 cm to °C is (assuming that not more than 100°C)
(c) 9 mm (d) 18 mm
22. (a) = 0[1 + ( – 3 )]
when determined using two different vessels and (b) = 0[1 +( + 3 )]
B are 1 and 2 (c) = 0[1 + ( + 2 )]
linear expansion of vessel is
linear expansion of vessel B is (d) = 0[1 + ( – 2 ) ]
27. The brass scale of a barometer gives correct reading
1 2 1 2
(a) (b)
2
1 2 is 20 10–6 per °C. The barometer reads 75 cm at
40°C. The atmospheric pressure at 40°C is
1 2 1 2
(c) (d) + (a) 75 cm of Hg (b) 74.94 cm of Hg
3 3
(c) 75.06 cm of Hg (d) none of these
23. contains some mercury. It is
found that at different temperatures, the volume of 28.
and -
. When the tempera-
and mercury respectively, the volume of mercury ture is raised by T, the depth upto which the cube
is submerged in the liquid remains unchanged. If
the expansion of the beaker is ignored, the relation
(a) (b) between and is
(a) = (b) =
3 2
(c) 1 (d) 1 (c) =3 (d) =2
IIT, 2004
24. A steel rod and a copper rod have lengths and 29. A copper wire of length and cross-sectional area
respectively at a certain temperature. It is found is held at the ends by two rigid supports. At tem-
that the difference between their lengths remains perature T the wire is just taut with negligible ten-
constants at all temperatures. If and are their sion. If the temperature reduces to (T – T), the
speed of transverse waves in the wire is (here Y is
/ is given by Young’s modulus, density and
linear expansion of copper)
(a) 1 (b) 1
Y T T
(a) (b)
(c) (d)
T T
(c) (d)
25. A uniform metallic circular disc, mounted on fric-
tionless bearings, is rotating at an angular frequency
about an axis passing through its centre and IIT, 1979
Thermal Expansion 15.7

30. A metal ball immersed in alcohol weighs W1 at (a) W1 > W2 (b) W1 = W2


0°C and W2
W1
expansion of metal is less than that of alcohol. If (c) W1 < W2 (d) W2 =
the density of the metal is large compared to that 2
of alcohol, then IIT, 1980

ANSWERS

1. (c) 2. (c) 3. (b) 4. (d) 5. (a) 6. (b)


7. (d) 8. (b) 9. (b) 10. (a) 11. (b) 12. (b)
13. (d) 14. (c) 15. (c) 16. (b) 17. (c) 18. (d)
19. (a) 20. (c) 21. (a) 22. (d) 23. (a) 24. (d)
25. (d) 26. (d) 27. (c) 28. (b) 29. (a) 30. (b)

SOLUTIONS
1. On heating, the diameter, surface area and vol- Y1 2
ume of the sphere will all increase. Since the mass Y1 1 = Y2 2 or =
Y2 1
remains unchanged, the density decreases. The
percentage increase in the volume is the largest be- Hence the correct choice is (d).
5. The correct choice is (a). It follows from the fact
that the volumes of metal and mercury increase to
expansion. Hence the correct choice is (c). 0 (1 + 1 T) and 0 (1 + 2 T) respectively; 0
2. The length of 1 cm division of the steel scale at being the initial volume.
21°C is 6. Length of wire at temperature T2 is
(1 cm) [1 + (21 – 20)] = (1 + ) cm 1
Length of copper rod at 21°C will be = 1
100
( cm) [1 + (21 – 20)] = (1 + ) cm
2 1 2
2
1 2 =2 1
Scale reading at 21°C = . 100
1
Now 2 2 = area of the plate at temperature T2
Hence the correct choice is (a).
and 2 2 = area of the plate at temperature T1.
3. The increase in length due to heating is independent Therefore,
of the radius of the rod. The increase in the length
1 2
of rod is = 1
100
=
2 102
and of rod B is = =2 ( =2) = 1 =
100 100
1
= Thus the area increases by 2%, which is choice (b).
2
7.
Hence the correct choice is (b).
= + + z = +2
4. If a rod of length
( = z)
expansion is heated to a temperature , the in-
–6 –6
crease in the length is given by = 2.0 10 + 2 1.6 10
= = 5.2 10–6 K–1
Hence the correct choice is (d).
Strain = = 8. Let 0 be the volume of the metal at 0°C and its
volume at °C. At temperature the upthrust is
and stress = Y strain = Y
U =
Since the value of is the same for the two rods, where is the density of alcohol at temperature .
the stresses in them will be equal if Now
15.8 Comprehensive Physics—JEE Advanced

= 0 (1 + ) or – = ( – 1)
where i.e. ( – ) . Hence the corect choice is (b).
alcohol and 0 is the volume of alcohol displaced
at temperature = 0°C. Now the density of alcohol 12. The maximum temperature variation T allowed is
at temperature is given by = T, which gives
5
= 0 5 10
T= = 5ºC
1 1 10 10 6
where 0 is the density of alcohol at = 0°C. Hence the correct choice is (b).
Hence 13. The moment of inertia of a rod of mass M and
0 length is given by
U = 0 (1 + )
1 2
I= (1)
= 0 0 = U0
1
where U0 is the upthrust at 0°C. Since the upthrust where = if the axis of rotation is through the
12
is independent of temperature, W1 = W2. Hence the
correct choice is (b). 1
centre and = if the axis of rotation is through
9. Here 3
( )1 = the end of the rod. Partially differentiating (1), we
1
have
( )2 = 2
I= 2
( )3 = 1 +( – ) 2
where is the length of metal and ( – ) that Now = . Therefore,
of metal B in rod C. Now ( )1 = 0.075 cm, ( )2 I= 2 ×
= 0.045 cm and ( )3 = 0.060 cm. Notice that 2
=2( )× = 2I
1
( )3 = [( )1 + ( )2]. This is possible only if
2 or
I
=2 , which is choice (d).
20 cm I
= = 10 cm. Hence the correct choice 14. The correct choice is (c).
2 2
is (b). 15. Total length of the composite rod at 0°C is 0 = 1
+ 2. When the composite rod is heated to °C, its
10. Due to thermal expansion, the diameter of the disc
length at °C will be = 1 (1 + 1 ) + 2 (1 + 2 )
as well as that of the hole will increase. Therefore,
= ( 1 + 2) + 1 1 + 2 2 , or
the moment of inertia will increase resulting in a
decrease in the angular speed. Hence the correct = 0 +( 1 1 + 2 2)
choice is (a). Effective coefficient of expansion of the
11. At ºC, the length of the rod becomes = (1 + composite rod is
), where
0 1 1 2 2
From the law of conservation of angular momen- = ,
tum, we have 0 1 2

I =I which is choice (c).


1 2 1 2
16. Consider a small element of length at a dis-
or = tance from one end of the rod. Let be the length
3 3
2
1 of linear expansion by unit length of the rod is
or =
(1 )2 ( 2 – 1)/ . Therefore, the value of at the
Now, for a given value of , (1 + )–2 is element located at is
constant, say . 2 1
= 1 +
=
Increase in the length of the element = ,
where T is the rise in temperature. Therefore, the
or = –1
increase in the length of rod is
Thermal Expansion 15.9

18. Weight of cylinder = weight of the liquid displaced,


= T i.e.
0 b = (0.8 ) 0
5 b
2 1 or 0 =
b
(1)
= T 1 0 .8 4
0
Here 0 is the density of the liquid at 0°C. For the
2 block to sink in the liquid at °C, the density of the
2 1
= T 1 liquid must change from 0 to at °C. Now
2 0
0
2 1
= (2)
= T 1 1
2
For the block to sink = b. Using (1) and (2), we
= 1 2
T= T have
eff
2 5 b /4
b =
1 1
where eff = ( 1 + 2). Hence the correct
2 5
choice is (b). or 1+ =
4
17. Let be the length of each rod at 0°C and 0 be the
Given = 62.5°C. Using this value, we get
altitude at 0°C. Then (Fig. 15.1),
= 4 10–3 (°C)–1 or K–1, which is choice (d).
19. Let 0 be the total volume of the block of iron at
0°C and its total volume at 60°C. Then
= 0 (1 + 60 Fe) (1)
Let v0 be the volume of the block submerged in
mercury at 0°C and v the volume submerged at
60°C. Then
v = v0 (1 + 60 Hg) (2)
Fig. 15.1
Dividing (1) by (2), we have
2 1/ 2
2 /v 1 60 Fe
0 = (1) (3)
4 /
0 v0 1 60 Hg
When the rods are heated to a temperature , the
altitude becomes v0 v
Given = 1 and = 2. Using these in (3), we
2 1/ 2 0
2 2 2
= 1 1 1 2
4
20. The length of each rod increases by the same
Since 1 and 2 are much less than unity, we
amount if 1 1 = 2 2 or
can write (1 + 1 )2 1 + 2 1 and (1 + 2 )2
1 + 2 2 . Thus 2 1
1/ 2
=
2 1 2
2
= 1 2 1 1 2 2 (2)
4 2 1
or + 1= +1
Equating (1) and (2), we get 1 2
2 1/ 2 2 1/ 2
2 2 1 2 1 2
1 2 1 1 2 2
or =
4 4 1 2

1 1 1 2
, which is choice (c). or = , which is choice (c).
2 4 1 2 1 2
15.10 Comprehensive Physics—JEE Advanced

21. Increase in volume of mercury when the tempera- If the disc is heated, it expands. Hence R increases.
ture increases by 100°C is The resulting increase in I is obtained by partially
v= T = (18 10–5) (10–6) 100 differentiating (1).
= 18 10–9 m3 1
I= M 2R R ( M = constant)
v 2
Now v = or =
or I = MR R
9
(18 10 m3 ) But R=R . Therefore,
= 4 2
= 0.09 m = 9 cm
(0.002 10 m ) I = MR2 (2)
Hence the correct choice is (a).
Now, the angular momentum of the disc is given
22. - by
-
J=I (3)
i.e. Since no external torque acts, J remains constant.
= + v = +3 v ( v = 3 v) Partially differentiating (3), we have
For vessel : = 1 +3 1 = 1 +3 ( 1 = I + I=0
; given) I
or =– (4)
For vessel B: = 2 +3 2 I
Since the liquid is the same, is the same. Using (1) and (2) in (4) we get
Hence = –2
1 +3 = 2 +3 2 The negative sign indicates that the angular frequency
1 2 decreases due to increase in temperature. Hence the
which gives 2 = + , which is choice (d).
3 correct choice is (d).
23. Let 26. If 0 is the volume of mercury at 0°C and
0 the cross-sectional area of the tube at 0°C, then
the length of the mercury column at 0°C is
0
0 = (1)
= 0
The cross-sectional area at °C is given by
or = , which is choice (a).
= 0 (1 + )
24. When the rods are heated by °C, the increase in where
length of steel rod and copper rod are of glass. Since = 2 , we have
= = 0 (1 + 2 )
= If is the volume of mercury at °C, the length of
the mercury column at °C is
The difference between their lengths will remain
0 1
constant at all temperatures if = , i.e. if = =
0 1 2
= Using (1) we have
or = = 0 (1 + ) (1 + 2 )–1
Since is not too high (~ 100°C) and and are of
Hence the correct choice is (d). the order of 10–5, and will be very small com-
pared to unity. Hence we can expand (1 + 2 )–1
25. The moment of inertia of the disc about the given
binomially and retain terms upto order . Thus
axis of rotation is
1 (1 + 2 )–1 = (1 – 2 )
I= MR2 (1) = 0 (1 + ) (1 – 2 )
2
where M is the mass of the disc and R its radius. or = 0 [1 + ( –2 ) ]
Thermal Expansion 15.11

2
where we have neglected term of which is neg- Using (2) and (3) we have
ligibly small. Thus the correct choice is (d). 0
27. Due to rise in temperature, the brass scale expands. 0 (1 + 2 T) = 0 0
(1 T)
It will give lower readings because the graduations
which gives 2 = . Thus the correct choice is (b).
on the scale will be farther apart. If H is the baro-
29. = T. Since the wire is free to contract, the
metric height at 0°C, the error in the reading of the
tension F must increase to produce the same change
scale at 40°C is
in length. Now
H= H = 75 20 10–6 40
/
= 0.06 cm Y=
/
Atmospheric pressure at 27°C = H + H = 75 +
0.06 = 75.06 cm of Hg. F= = T
Thus the correct choice is (c).
28. Let the initial temperature be T and let M be the Speed of transverse waves in the wire is
mass of the cube. Let 0, 0 and 0 respectively be F
the base area of the cube, the density of the mate- v= ; = mass per unit length
= / =
rial of the cube and the depth upto which it is sub-
merged in the liquid, the upthrust = 0 0 0 . From F T Y T
= = =
= 0 0 0
30. Let 0 be the volume of the metal at 0°C and its
or M= 0 0 0 (1)
volume at °C. At temperature the upthrust is
When the temperature is raised to (T + T), let , U =
and be the base area, density and depth at this
where is the density of alcohol at temperature .
(area) expansion is = 2 . Now
Hence = 0 (1 + )
= 0(1 + T) where
= 0 (1 + 2 T) (2) alcohol and 0 is the volume of alcohol displaced
0
at temperature = 0°C. Now the density of alcohol
Also = (3) at temperature is
(1 T)
The upthrust at temperature (T + DT) = . From = 0
1
= where 0 is the density of alcohol at = 0°C. Hence
or M= (4) 0
U = 0 (1 + )
From (1) and (4), we get 1
= = 0 0 = U0
0 0 0
or = ( = where U0 is the upthrust at 0°C. Since the upthrust
0 0 0; given)
is independent of temperature, W1 = W2.

II

Multiple Choice Questions with One or More Choices Correct


1. Choose the wrong statements from the following. (a) its speed will decrease
A metallic circular disc having a circular hole at (b) its diameter will decrease
its centre rotates about an axis passing through its (c) its moment of inertia will increase
centre and perpendicular to its plane. When the disc (d) its speed will increase
is heated.
15.12 Comprehensive Physics—JEE Advanced

2. Choose the wrong statements from the following. (c) the angular frequency increases by 2 .
Two spheres made of the same material have the (d) the angular frequency decreases by 2 .
same diameter. One sphere is hollow and the other 5. A clock with a metallic pendulum gains 6 seconds
is solid. If they are heated through the same range each day when the temperature is 20°C and loses
of temperature, 12 seconds when the temperature is 40°C. Then
(a) the hollow sphere will expand more than the (a) the clock will keep correct time at tempera-
solid sphere 80
(b) the solid sphere will expand more than the ture °C
3
hollow sphere (b) the clock will keep correct time at tempera-
(c) both spheres will expand equally 100
(d) the hollow sphere will not expand at all. ture °C.
3
3. Two rods and B of different metals have lengths
and B at a certain temperature. It is observed
is 1.2 10–5 per °C.
that rod is 5 cm longer than rod B at all tempera-
tures. If = 1.0 10–5 per °C and B = 1.5 10–5
per °C, then is 2.1 10–5 per °C.
(a) = 10 cm (b) = 15 cm 6. A bimetallic strip is formed out of two identical
(c) B = 10 cm (d) B = 5 cm strips one of copper and the other of brass. The
4. A uniform metallic circular disc of mass M and ra-
are C and B. When the temperature of the strip is
dius R, mounted on frictionless bearings, is rotating
increased by T, it bends to form an arc of radius
an angular frequency about an axis passing thro-
of curvature R. Then R is
ugh its centre and perpendicular to its plane. The
(a) proportional to T
temperature of the disc is then increased by . If
(b) inversely proportional to T
(c) proportional to ( C – B)
(a) the moment of inertia increases by MR2 . (d) inversely proportional to ( C – B).
(b) the moment of inertia remains unchanged.
IIT, 1999

ANSWERS AND SOLUTIONS


1. Due to thermal expansion, the diameter of the disc 4. The moment of inertia of the disc about the given
as well as that of the hole will increase. Therefore, axis of rotation is
the moment of inertia will increase resulting in an 1
I = MR2 (1)
increase in the angular speed. Hence the correct 2
choices are (a) and (c). If the disc is heated, it expands. Hence R increases.
2. Statements (a), (b) and (d) are wrong. The resulting increase in I is obtained by partially
3. When the rods are heated by °C, the increase in differentiating (1).
the length of the rods is 1
I= M 2R R ( M = constant)
= 2
or I = MR R
and B = B B
( – B) will remain the same for all , if
But R=R . Therefore,
= B, i.e. I = MR2 , which is choice (a) (2)
= B B
Now, the angular momentum of the disc is given
1.5 10 by
= = 5
= 1.5
B B 1.0 10 =I (3)
= 1.5 B. Also ( – B) = 5 cm. Thus Since no external torque acts, remains constant.
1.5 B – B = 5 cm Partially differentiating (3), we have
B = 10 cm, and = 15 cm.
The correct choices are (b) and (c). I + I= 0
Thermal Expansion 15.13

I At 20°C, the gain in time is


or =– (4)
I 1
6 = ( – 20) 86400 (3)
Using (1) and (2) in (4) we get 2
= –2 At 40°C, the loss in time is
The negative sign indicates that the angular frequency 1
decreases due to increase in temperature. Thus the 12 = (40 – ) 86400 (4)
2
correct choices are (a) and (d).
Dividing (4) by (3), we have
5. Time taken for one oscillation of the pendulum is
12 40
=
T=2 6 20
80
2 2 which gives = °C. Using this value in Eq. (3),
or T =4 (1) 3
we have
Partially differentiating, we get 1 80
6= 20 86400
2T T = 4 2
(2) 2 3
which gives = 2.1 10–5 per °C.
Dividing (2) by (1), we get
Thus the correct choices are (a) and (d).
T 1
= = = ( = ) 6. C = 0 (1 + C T) and B = 0 (1 + B T)
T 2 2 2
where is the change in temperature. Now, one C – B = 0 ( C – B) T.
day = 24 hours = 86400 s. Therefore, gain or loss Initially the bimetallic strip is straight, i.e. R is
of time in one day is R
1 decreases. The amount of bending is proportional to
T= 86400 seconds ( C – B) and T. Greater the bending, the smaller
2
Let be the temperature at which the clock keeps is the value of R. Hence the correct choices are (b)
correct time. and (d).

III

Multiple Choice Questions Based On Passage


Question 1 to 3 are based on the following passage (a) 120°C (b) 130°C
Passage I (c) 140°C (d) 150°C
2. Work done by the block on the surroundings is
20,000 J of heat energy is supplied to a metal block of
mass 500 g at atmospheric pressure. The initial tempera- (a) 0.05 J (b) 0.1 J
(c) 1.0 J (d) 10 J
400 J kg–1 °C–1 3. The change in internal energy is
of volume expansion of metal = 8 10–5 °C–1 and atmo- (a) zero
spheric pressure = 105 Pa. (b) equal to 20,000 J
IIT, 2005 (c) slightly greater than zero
1. (d) slightly less than 20,000 J.
SOLUTION
1. Q= T. Therefore, Final temperature = 100 + 30 = 130°C, which is
choice (b).
Q 20, 000 J
T= = 1
= 100°C 2. Density of metal ( ) = 8000 kg m–3. Volume of the
(0.5 kg ) (400 J kg C 1) block is
15.14 Comprehensive Physics—JEE Advanced

0.5 kg 1 1
= = = 10–3 m3 = 10–6 m3
8000 kg m 3
16 2
1 6
Work done W = P = (105) 10
Increase in volume = = T 2
1 = 0.05 J, which is choice (a).
3
= (8 10–5 ) 10 100 3. Change in internal energy U = Q – W
16
= 20,000 – 0.05 = 19999.95 J
Thus the correct choice is (d).

Questions 4 to 6 are based on the following passage (a) 2 : 1 (b) 3 : 2


Passage II (c) 3 : 1 (d) 4 : 3
A rod of metal X of length 50.0 cm elongates by 0.10 cm 5. The length of the rod of metal X in the composite
when it is heated from 0°C to 100°C. Another rod of metal piece is
Y of length 80.0 cm elongates by 0.08 cm for the same (a) 10 cm (b) 20 cm
rise in temperature. A third rod of length 50 cm, made (c) 30 cm (d) 40 cm
by welding pieces of rods X and Y placed end to end,
elongates by 0.03 cm when its temperature is raised from 6. The length of the rod of metal Y in the composite
0°C to 50°C. piece is
4. X and (a) 10 cm (b) 20 cm
of metal Y are in the ratio of (c) 30 cm (d) 40 cm

SOLUTION
0.10 = (1 + 50) and = (1 + 50).
4. For rod X, = = Therefore,
50.0 100
= 2.0 10– 6 per °C. 50.03 = + +( + ) 50
For rod Y, we get = 1.0 10–6 per °C. So the = 50 + ( + ) 50
correct choice is (a).
Substituting the values of and and noting
5. In the composite rod, + = 50.0 cm. When this
that + = 50 cm, we get = 10 cm, which is
rod is heated by 50°C, let the new lengths be and
choice (a).
. Given + = 50.0 + 0.03 = 50.03 cm. Here
6. = 50 – = 50 – 10 = 40 cm, which is choice (d).

IV

Integer Answer Type


1. A composite rod is made by joining a copper length of the composite rod increases by 1.91 mm.
rod, end to end, with a second rod of a different
material but of the same cross-section. At 25°C, = 1.7 10–5 per °C and that of the second rod is
the composite rod is 1 m in length of which the = 10–5 per °C. Find the value of n.
length of the copper rod is 30 cm. At 125°C the IIT, 1979
SOLUTION
1. Length of the second rod at 25°C = 70 cm. Increase in the length of second rod = 70 100
Length of copper rod at 125°C = 7000 cm
= 30 (1 + 1.7 10–5 100) Total increase in length = 0.051 cm + 7000 cm
= 30.051 cm = 0.191 cm (given) which gives = 2 10–5 per °C.
Increase in the length of copper rod = 0.051 cm Thus the value of = 2.
16
Chapter
Measurement of Heat

REVIEW OF BASIC CONCEPTS Specific Heat Capacity

16.1 HEAT ENERGY If m = 1 unit and T = 1 unit, then s = Q


heat of a substance is the amount of heat required to raise
Heat is a form of energy. It is, therefore, measured in the temperature of a unit mass of the substance through a
energy units. The SI unit of heat is joule (J). Another unit unit degree. A commonly used unit of s is cal g–1 °C–1. In
commonly used is the calorie. A calorie is the amount the SI system s is expressed in J kg–1 K–1. The two units
of heat required to raise the temperature of 1 g of water are related as
through 1°C. Experiments have shown that 4.18 J of 1 cal g–1 °C–1 = 4.18 Jg–1 °C–1
mechanical work produce one calorie of heat. ( 1 cal = 4.18 J)
Thus
1 calorie = 4.18 joules or 1cal g–1 °C–1 = 4180 J kg–1 °C–1
Since the size of a degree on the celsius scale is equal
or 1 cal = 4.18 J
to that on the kelvin scale, a temperature difference of, say,
work done 1 °C is equal to a temperature difference of 1 K. Thus
The ratio J= = 4.18 J per cal
heat produced 1 cal g–1 °C–1 = 4180 J kg–1 K–1
is called the mechanical equivalent of heat.
capacity of water = 1 cal g–1 °C–1 or 4180 J kg–1 K–1.
16.2 CALORIMETRY Molar Specific Heat
If two substances having different temperatures are C of a substance is the amount of
heat energy required to raise the temperature of 1 mole of
the substance through 1 K. It is expressed in J mol–1 K–1.
energy will continue till the temperatures are equalized. s C) are related as
The common temperature at thermal equilibrium is called C
the equilibrium temperature. If no heat energy is allowed s =
m
to escape to the surroundings, the amount of heat energy
where m is the number of kilograms per mole in the
gained by the initially colder body is equal to the amount
substance.
of heat energy lost by the initially hotter body, i.e.
) is
Heat gained by one body = heat lost by the other body. v

This is the basic principle of calorimetry.


The heat energy Q needed to raise the temperature
through T of a mass m
capacity s is given by p

Q = ms T
16.2 Comprehensive Physics—JEE Advanced

SOLUTION
Cp – Cv = R
where R is the universal gas constant and its value is The amount of heat energy in copper block at 500°C
R = 8.315 J mol–1 K–1 is
Q = ms T
16.3 LATENT HEAT = (3.35 390 500) J
Now L = 335 J g–1 = 335 103 J kg–1.
The heat energy supplied to a substance to change from
solid to liquid state or from liquid to gaseous state is not The maximum mass of ice that can melt is
registered by a thermometer as the heat energy is used Q 3.35 390 500
m= =
up in bringing about a change of state. Hence it is called L 335 103
latent (or hidden) heat. A substance has two latent heats. = 1.95 kg
of a substance
is the heat energy required to convert a unit mass of 16.3
a substance from the solid to the liquid state, without
change of temperature. The latent heat of fusion of ice is 16 g of oxygen is heated at constant volume from
3.36 105 J kg–1 or 80 cal g–1. 25°C to 35°C. Find the amount of heat energy re-
quired. Given Cv = 20 J mol–1 K–1.
of a
substance is the heat energy required to convert a unit SOLUTION
mass of the substance from the liquid to the gaseous
state, without change of temperature. The latent heat of Mass of 1 mole of oxygen = 32 g
vaporisation of steam is 2.26 106 J kg–1 or 540 cal g–1. Number of moles in 16 g of oxygen is
16 1
16.1 n= =
32 2
A 12 kW drilling machine is used to drill a hole in a
Heat energy required is
metal block of mass 10 kg. Assuming that 25% power
is lost in the machine, calculate the rise in tempera- Q = n Cv T
1
of metal = 0.4 J g–1 K–1. = 20 (35 – 25) = 100 J
2
SOLUTION 16.4
3
Useful power available = 75% of 12 kW = 12 kW 290 J of heat energy is required to raise the tempera-
= 9 kW = 9000 W 4
Heat energy consumed in 2 minutes (= 120 s) is heat of nitrogen at constant pressure.
Q = (9000 120) J
SOLUTION
Now s = 0.4 J g–1 K–1 = 0.4 103 J kg–1 K–1
7 1
Q Number of moles in 7 g of oxygen is n = =
Rise in temperature is T = 28 4
Q = n Cp T
ms
9000 120 Q 290
= 3
= 270°C Cp = = = 29 J mol–1 K–1
10 (0.4 10 ) n T 1
40
4
16.2
16.5
A copper block of mass 3.35 kg is heated to
500°C and then placed on a large block of ice. 200 g of water at 25°C is added to 75 g of ice at 0°C
What is the maximum mass of ice that can melt?
–1
K–1 of the mixture?
and latent heat of fusion of water = 335 J g–1.
Measurement of Heat 16.3

T2 T2
SOLUTION
Q= msdT = T 3 dT
Heat energy of 200 g of water at 25°C is T1 T1
Q1 = ms T = 200 1 25
= 5000 cal = T24 T14
4
Heat energy required to melt 75 g of ice at 0°C is
0.2
Q2 = mL = 75 80 = 6000 cal = ( 4) 4 (1)4
4
Since Q1 < Q2, the whole of ice will not melt. Hence, = 12.75 joule

NOTES
16.7
If 1 kg of ice at 0°C is mixed with 1 kg of steam at 100°C,
the equilibrium temperature is 100°C, 0.665 kg of steam Three liquids 1, 2 and 3 of masses m1 = m, m2 = 2m
will be left and 1.335 kg of water will be formed. and m3 = 3m are at temperatures of 10°C, 18°C and
Water of mass m at °C is mixed with ice of mass mi at 30°C. When liquids 1 and 2 are mixed, the equilib-
0°C. rium temperature is 16°C. When liquids 2 and 3 are
L f mi mixed, the equilibrium temperature is 22°C. Find the
(a) If m = equilibrium temperature when liquids 1 and 3 are
temperature = 0°C. mixed. Assume that there is no loss of heat to the
L f mi surroundings.
(b) If m <
temperature = 0°C and mass of ice melted is SOLUTION

mi= . Amount of ice left = mi – m i. Let s1, s2 and s3 -


Lf uids 1, 2 and 3 respectively.
L f mi When liquids 1 and 2 are mixed,
(c) If m > heat gained by 1 = heat lost by 2
L mi
temperature = > 0°C m1s1(16 – 10) = m2s2(18 – 16)
m m ms1(6) = 2ms2(2)
16.6 2
s1 =
s (1)
3 2
very low temperatures (close to absolute zero), the When liquids 2 and 3 are mixed
s of a solid varies with absolute tem- heat gained by 2 = heat lost by (3)
perature T as 2ms2(22 – 18) = 3ms3(30 – 22)
s= 3 s
s3 = 2 (2)
where is a constant whose value depends upon the 3
material of the solid. Find the heat energy required
to raise the temperature of 200 g of the solid from Let T be the equilibrium temperature. When liquids
1 K to 4 K. 1 and 3 are mixed.
Heat gained by 1 = heat lost by 3
SOLUTION ms1(T – 10) = 3ms3(30 – T) (3)
The amount of heat energy required to raise the tem- Using (1) and (2) in (3), we have
perature of mass m of the solid by dT kelvin is given 2 s2
by s2(T – 10) = 3 (30 – T)
dQ = msdT 3 3
Heat energy required to raise the temperature of the T = 22°C
solid from T1 = 1 K to T2 = 4 K is
16.4 Comprehensive Physics—JEE Advanced

Multiple Choice Questions with Only One Choice Correct

1. 300 g of water at 25°C is added to 100 g of ice at 7. The temperature of a liquid does not increase dur-
ing boiling. The heat energy supplied during this
5 5 process,
(a) – °C (b) – °C (a) increases the kinetic energy of the molecules
3 2
of the liquid
(c) – 5 °C (d) 0 °C (b) increases the potential energy of the mol-
IIT, 1989 ecules
2. 100 g of ice at 0°C is mixed with 100 g of water (c) increases both the kinetic and potential
energy of the molecules
(a) 0°C (b) 20°C (d) is merely wasted since no increase occurs in
(c) 40°C (d) 60°C the total energy of the molecules.
3. Steam at 100°C is passed into 1.1 kg of water con- 8. A block of ice at – 10°C is slowly heated and con-
tained in a calorimeter of water equivalent 0.02 kg verted to steam at 100°C. Which of the following
at 15°C till the temperature of the calorimeter rises curves represents the phenomenon qualitatively?
to 80°C. The mass of steam condensed in kilogram
is
(a) 0.13 (b) 0.065
(c) 0.260 (d) 0.135
IIT, 1986
4.

minute, from 27°C to 77°C. If the heat of combus-


tion of LPG is 4.0 104 J g–1, how much fuel in
grams is consumed per minute?
(a) 15.25 (b) 15.5
(c) 15.75 (d) 16
5. A copper block of mass 2 kg is heated to a tempera-
ture of 500°C and then placed in a large block of Fig. 16.1
ice at 0°C. What is the maximum amount of ice that
–1
IIT, 2000
°C–1 and latent heat of fusion of water is 3.5 105 9. 2 kg of ice at –20°C is mixed with 5 kg of water
J kg–1. at 20°C in an insulating vessel having a negligible
IIT, 2005
4 6
(a) kg (b) kg and ice are 1 kcal/kg/°C and 0.5 kcal/kg/°C
3 5
re-spectively and the latent heat of fusion of ice is
8 10
(c) kg (d) kg 80 kcal/kg.
7 9
(a) 7 kg (b) 6 kg
6. How much heat energy is joules must be supplied to (c) 4 kg (d) 2 kg
14 grams of nitrogen at room temperature to raise
IIT, 2003
its temperature by 40°C at constant pressure. Molar
mass of nitrogen = 28 and R J K–1 mol–1 is the gas 10. A metal sphere of radius r S is
constant. rotated about an axis passing through its centre at
(a) 50 R (b) 60 R a speed of n rotations per second. It is suddenly
(c) 70 R (d) 80 R stopped and 50% of its energy is used in increasing
Measurement of Heat 16.5

its temperature. Then the rise in temperature of the 15. A 1 kW electric kettle contains 2 litre water at 27°C.
sphere is: The kettle is operated for 10 minutes. If heat is lost
2 to the surroundings at a constant rate of 160 J/sec,
2 2 2 2
n r n2
(a) (b) the temperature attained by water in 10 minutes
5S 10 r 2 S
(a) 57°C (b) 67°C
7 5 ( rn)2
(c) r2 n2 S (d) (c) 77°C (d) 87°C
8 14 S
11. If there are no heat losses, the heat released by the IIT, 2005
condensation of x grams of steam at 100°C into water 16. In an industrial process 10 kg of water per hour is
at 100°C converts grams of ice at 0°C into water to be heated from 20°C to 80°C. To do this, steam
at 100°C. The ratio /x is at 150°C is passed from a boiler into a copper coil
(a) 1 (b) 2 immersed in water. The steam condenses in the coil
and is returned to the boiler as water at 90°C. How
(c) 3 (d) 4
many kg of steam are required per hour?
12. 5 g of water at 30°C and 5 g of ice at –20°C are heat of steam = 1 kilo cal kg–1 °C–1. Latent heat of
steam = 540 kilo cal kg–1.
(a) 1 kg (b) 2 kg
ice = 0.5 cal g–1 (°C)–1 and latent heat of fusion of ice (c) 3 kg (d) 4 kg
= 80 cal g–1.
17. With what velocity should a lead bullet at an initial
(a) –5°C (b) 0°C
temperature of 30°C strike a target so that it just
(c) + 5°C (d) + 10°C melts? Assume that 84% of the heat produced is
13. A metal block of mass 10 kg is dragged on a hori-
zontal rough road with a constant speed of 5 ms–1. cal kg–1 (°C)–1, latent heat of lead = 6 kilo cal kg–1
and melting point of lead = 330°C.
the road is 0.5, the rate at which heat is generated in (1 kilo calorie = 4.2 103 joule)
(J s–1) is
(a) 100 5 ms–1 (b) 100 2 ms–1
(a) 100 (b) 245
(c) 9.8 (d) 10 (c) 100 10 ms–1 (d) 100 15 ms–1
14. Liquid oxygen at 50 K is heated to 300 K at constant 18. -
pressure of 1 atm. The rate of heating is constant. ids at extremely low temperature (close to abso-
Which of the following graphs represents the veri- s of a solid varies with
ation of temperature with time? (see Fig. 16.2). absolute temperature T as
IIT, 2004 s = cT 3
where c is a constant depending on the material
of the solid. The heat energy required to raise the
temperature of 0.1 kg of the solid from 0 K to 4 K
is.
(a) 4.2 c joule (b) 6.4 c joule
(c) 8.4 c joule (d) 12.6 c joule
19. 400 g of ice at 253 K is mixed with 0.05 kg of steam
at 100°C. Latent heat of vaporisation of steam =
540 cal/g. Latent heat of fusion of ice = 80 cal/g.

temperature of the mixture is


(a) 273 K (b) 300 K
(c) 330 K (d) 373 K
IIT, 2007
Fig. 16.2
16.6 Comprehensive Physics—JEE Advanced

20. Two litres of water (density = 1 g/ml) in an open- –1


K–1) from 20°C
lid insulated kettle is heated by an electric heater to 75°C is
of power 1 kW. The heat is lost from the lid at the (a) 340 s (b) 550 s
rate of 160 J/s. The time taken for heating water (c) 620 s (d) 760 s
IIT, 2005

ANSWERS

1. (d) 2. (a) 3. (a) 4. (c) 5. (c) 6. (c)


7. (b) 8. (a) 9. (b) 10. (a) 11. (c) 12. (b)
13. (b) 14. (c) 15. (d) 16. (a) 17. (d) 18. (b)
19. (a) 20. (b)

SOLUTION
1. Let the temperature of the mixture be C. Heat lost Heat energy supplied to the geyser per minute
by water in calories = 300 1 (25 – ) = 7500 – 300 = 3.0 4200 50
Heat in calories required to melt 100 g of ice
= 63 104 J min–1
= 100 80 = 8000
Now, heat of combustion of LPG
Now Heat lost = heat gained
5 = 4.0 104 J g–1 = 4.0 107 J kg–1
or 7500 – 300 = 8000 or = – °C
3 Amount of fuel consumed per minute
Since is negative, the water at 25°C cools to 0°C 63 104 J min 1
=
and melts a part of ice at 0°C. 4.0 107 J kg 1
Heat lost = 300 1 (25 – 0) = 7500 cal.
= 15.75 10–3 kg min–1
Hence only a part of the ice melts and resulting
temperature is 0°C. Hence the correct choice is = 15.75 g min–1
(d). 5. Heat energy in copper block = 2 400 500 =
2. The amount of heat required to convert 100 g of ice 4 105 J. The amount of ice that melts will be maxi-
at 0°C into water at 0°C = 100 80 = 8000 calories. mum if the entire heat energy of the copper block
This is precisely the amount of heat lost by 100 g of is used up in melting ice. Now, 3.5 105 J of heat
water at 80°C to bring its temperature down to 0°C. energy is needed to melt 1 kg of ice into water.
Therefore, the temperature of the mixture remains Therefore, the amount of ice melted by 4 105 J of
0°C. Hence the correct choice is (a). heat energy is
3. Let the mass of steam condensed be m kg. The 4 105 J 8
5 1
= kg
latent of vaporisation of water = 556 k cal/kg. 3.5 10 J kg 7
Therefore, heat lost by steam = 556 m + m (100
Hence the correct choice is (c).
– 80) = 576 m kcal. Heat gained by calorimeter and
water = (1.1 + 0.02) (80 – 15) = 72.8 kcal. Now, 6. Number of moles in 14 grams of nitrogen (n) =
heat lost = heat gained, i.e. 1
14/28 = . Since nitrogen is diatomic Cp = 7R/2.
576 m = 72.8 or m = 0.13 kg 2
4. Therefore, amount of heat energy supplied
1 7R
min–1 = 3000 cm3 min–1 = n Cp = 40 = 70R joules
Density of water = 1 g cm–3. 2 2
Hence the correct choice (c)
7. The correct choice is (b).
= 3000 g min–1 = 3.0 kg min–1
8.
Rise in temperature = 77 – 27 = 50°C ice increases from – 10°C to 0°C. At 0°C, ice starts
–1
°C –1 melting at a constant temperature. When the whole
Measurement of Heat 16.7

of ice has melted into water, the temperature of Heat required for step (ii) = 5 80 = 400 cal
water will increase from 0°C to 100°C. At 100°C, When water at 30°C is allowed to cool to 0°C, the
again the temperature becomes constant due to heat given out
the conversion of liquid water into water vapour = 5 1 (30 – 0) = 150 cal
(steam) at 100°C. Hence the correct graph is (a).
Thus it is clear that all the ice cannot melt and the
9. Let m kg be the mass of ice melted into water. Heat system will remain at 0°C. Since only (150 – 50) =
lost by 5 kg of water = 5 kg 1 kcal/kg/°C 20°C 100 cal are available for melting ice, the mass of ice
= 100 kcal. Heat gained = m kg 80 kcal/kg + 2 kg melted = 100/80 = 1.25 g.
0.5 kcal/kg/°C 20°C = 80 m kcal + 20 kcal.
Now, heat gained = heat lost. Therefore, 6.25 g of water at 0°C and (5 – 1.25) = 3.75 g of ice
80 m + 20 = 100
or m Hence the correct choice is (b).
+ 1 kg = 6 kg. Hence the correct choice is (b). 13. Heat energy generated per second = work done per
second against friction = force of friction distance
2
10. Moment of inertia of the sphere I = Mr2. Given moved in 1s
5 =
= n rotations per second = 2 n rad s–1. The kinetic
= 0.5 10 9.8 5 = 245 J s–1
energy is
Hence the correct choice is (b).
1 2
1 2 14. Liquid oxygen will undergo a change of phase
KE = I = Mr2 (2 n)2
2 2 5 when heated from 50 K to 300 K. During the phase
4 change the temperature will not change. When all
= M 2r 2n 2
5 the liquid oxygen has changed to gaseous state,
Since half of KE is converted into heat energy, we the temperature will increase because rate of heat-
have ing is constant (see Fig.16.3). Hence the correct
1 2 graph is (c).
dQ = KE = M 2 r2 n2
2 5

Now dQ = MSdT which gives


2
M 2 r 2 n2 2 2 r 2 n2
dQ 5
dT = = =
MS MS 5S

Hence the correct choice is (a).


11. The latent heat of vaporisation of water is very Fig. 16.3
nearly 540 calories per gram. Therefore heat re-
leased in the condensation of x gram of steam = 15. Power (P) = 1 kW = 1000 W, mass of water (m) =
540 x calories. The latent heat of fusion of ice is s) = 4200 J/kg °C. Time
very nearly 80 calories. Therefore, heat required to = 10 min = 600 s. Initial temperature T1 = 27°C.
convert gram of ice at 0°C to water at 100°C = Heat energy supplied by kettle in 600 s = = 1000
80 + 100 = 180 calories. Thus 600 = 600,000 J. Heat energy lost in 600 s = 160
600 = 96,000 J. Therefore, heat energy used up in
180 = 540 x heating the water is
or =3 H = 600,000 – 96,000 = 504,000 J
x
Now H = ms T = ms (T2 – T1) = 2 4200 (T2 –
Hence the correct choice is (c). 27), where T2
12.
temperature of ice from – 20°C to 0°C and (ii) melt 2 4200 (T2 – 27) = 504,000
the ice at 0°C into water at 0°C. which gives T2 = 87°C, which is choice (d).
Heat required for step (i) = 5 0.5 {0 – (–20)} = 16. Let the mass of steam required per hour be m kg.
50 cal Heat gained by water in boiler per hour is
16.8 Comprehensive Physics—JEE Advanced

=10 kg 1 kilo cal kg–1 °C–1 18.


(80 – 20)°C = 600 kilo cal (1) constant. The amount of heat energy required to
Heat lost by steam per hour is raise the temperature of the solid of mass m through
= heat needed to cool m kg of steam from 150°C to dT kelvin is
100°C + heat needed to convert m kg of steam at dQ = msdT
100°C into water at 100°C + heat needed to cool m Heat energy needed to raise the temperature of
kg of water from 100°C to 90°C the solid from 0 K to 4 K is
= m 1 (150 – 100) + m 540 4 4

+ m 1 (100 – 90) Q = msdT = m cT 3dT


0 0
= 50 m + 540 m + 10 m
4 4
= 600 m kilo cal (2) T4
= mc T 3dT = mc
Heat lost = heat gained. Equating (1) and (2) we 0
4 0
have
44
600 m = 600 = mc = 0.1 c (4)3
4
or m = 1 kg, which is choice (a).
17. Let the mass of the bullet be m kg and its velocity = 6.4 c joule.
be v ms–1. Before striking the target, the kinetic en- So the correct choice is (b).
1 19. Heat required to melt the whole of ice is
ergy of the bullet is mv2 joule which is converted
2 Q1 = ms T + mL
into heat energy when the bullet strikes the target. = 400 0.5 20 + 400 80
Thus heat energy produced is
1 ( 253 K = –20°C)
Q = mv2 joule
2 = 4000 + 32000 = 36000 cal
1 The maximum heat released by steam when the
m v 2 joule whole of it (= 50 g) is converted into water at 0°C is
= 2
Q2 = ms T + mL
4.2 103 joule/kilo cal
= 50 1 100 + 540 50
m v2 = 5000 + 27000 = 32000 cal
= kilo cal Since Q2 is less than Q1, the whole of ice will not
8.4 103
Heat energy absorbed by the bullet is will be 0°C or 273 K.
2 20. Mass of 2 litres of water = 2 kg. Heat energy needed
0.84 mv
Q = 84% of Q = 0.84 Q = kilo cal to raise the temperature of 2 kg of water from 20°C
8 4 103 to 75°C is
In order that heat energy Q melts the bullet, it Q = 2 (4.2 103) 55 = 4.62 105 J
If is the time taken, heat energy supplied by the
bullet from 30°C upto its melting point (330°C) and heater in time is
then to supply the latent for melting. Hence
Q1 = (power time) = 1000 joule
0.84 m v2 Heat energy lost in time is
=m 0.03 (330 – 30) m 6
8. 4 103 Q2 = 160 joule
= 9 m + 6 m = 15 m
Heat energy available for heating water is
which gives v = 100 15 ms–1 Q = Q1 – Q2 = 840 J
Thus the correct choice is (d). Equating Q = Q , we get 550 s.
Measurement of Heat 16.9

II

Multiple Choice Questions with One or More Choices Correct

1. A source of heat supplies heat at a constant rate to a (c) thermal capacity of liquid
solid cube. The variation of the temperature of the (d) thermal capacity of vapour.
cube with heat supplied is shown in Fig. 16.4. 4. In Fig. 16.4, it is observed that DE = 3 BC. This
(a) Portion BC of the graph represents conversion means that
of solid into liquid. (a) the thermal capacity of the vapour is 3 times
(b) Portion BC of the graph represents conversion that of the liquid
of solid into vapour.
(c) Portion DE of the graph represents conver- of the liquid
sion of vapour into liquid. (c) the latent heat of vaporisation of the liquid is
(d) Portion DE of the graph represents conver- 3 times the latent heat of fusion of the solid.
sion of liquid into vapour (d) the latent heat of fusion of the solid is 3 times
the latent heat of vaporisation of the liquid.
5.
800 J kg–1 K–1 is initially at a temperature 20°C. It
–1

and on returning to the starting point strikes a lump


of ice at 0°C and gets embedded in it. Assume that
all the energy of the bullet is used up in melting.
Neglect the friction of air. Latent heat of fusion of
ice = 3.36 105 Jkg–1.
(a) Energy of bullet used in melting is 1000 J.
(b) The mass of ice method = 5 g
Fig. 16.4
(c) The mass of ice melted is slightly greater
2. In Q.1 above, the slope of portion CD of the graph than 5 g.
shown in Fig. 16.4 gives (d) The mass of ice melted is less than 5 g.
(a) latent heat of fusion 6. -
(b) latent heat of vaporisation ity s falls from a height of 10 cm and bounces to
(c) thermal capacity of liquid a height of 4 m. If all dissipated energy is ab-
(d) thermal capacity of vapour sorbed by the ball as heat, its temperature rises by
3. In Q.1 above, the slope of the portion EF of graph 0.075 K. Take = 10 ms–2 .
shown in Fig. 16.4 gives (a) Q = 60 J (b) Q = 100 J
(a) (c) s = 800 J kg–1 K–1 (d) s = 1333 J kg–1 K–1

ANSWERS AND SOLUTIONS

1. The correct choices are (a) and (d). The heat sup- same. The portion DE represents the conversion
plied is the latent heat. of liquid into vapour at the same temperature. The
2. The slope of portion CD of the graph gives the heat supplied in the two cases is latent (hidden).
amount of heat supplied by unit rise in temperature of Hence the correct choice is (c).
5. If the friction offered by air is neglected, the speed
of the liquid. Hence the correct choice is (c). of the bullet on returning to the starting point
3. The correct choice is (d). will be equal to its initial speed v = 200 ms–1. The
4. The portion BC of the graph represents the conver- kinetic energy of the bullet is
sion of solid into liquid, temperature remaining the
16.10 Comprehensive Physics—JEE Advanced

1 x = 5.3 10–3 kg = 5.3 g. Hence the correct


K.E. = mv2 choices are (a) and (c).
2
6. Q = (h – h ) = 1 10 (10 – 4) = 60 J.
1 –3 2
= (50 10 ) (200) = 1000 J Also Q = ms T. Hence
2
Heat lost by bullet for its temperature to fall from Q 60
20°C to 0°C = (50 10–3) 800 20 = 800 J. If x s= =
m T 1 0.075
kg is the mass of ice melted, then
= 800 J kg–1 K–1.
x (3.36 105) = 1000 + 800
Thus the correct choices are (a) and (c).

III

Multiple Choice Questions Based on Passage


Questions 1 to 3 are based on the following passage 1. The time taken to raise the temperature of the con-
Passage I tainer and ice from – 20°C to 0°C is
The basic principle of calorimetry is heat gained by one (a) 12 s (b) 24 s
body = heat lost by the other body. This follows from the (c) 36 s (d) 48 s
principle of conservation of energy according to which
the total heat energy of the two substances must remain 2. The time taken to melt ice at 0°C into water at 0°C
constant. Hence heat lost by one body must be gained by is
the other, provided no part of heat energy is allowed to (a) 40 s (b) 80 s
escape.
(c) 120 s (d) 160 s
An aluminium container of mass 100 g contains
200 g of ice at – 20°C. Heat is added to the system at 3. The temperature of the system after 4 minutes is
(a) 15.45°C (b) 20.45°C
ice = 2100 J kg–1 K–1
aluminium = 840 J kg–1 K–1 and latent heat of fusion of ice (c) 25.45°C (d) 30.45°C
= 3.36 105 J kg–1.

SOLUTION
1. Heat energy needed to raise the temperature of the
6.72 104
container and ice from – 20°C to 0°C is Time needed is 2
= = 160 s, which is
420
Q1 = (100 10–3) 840 20 choice (d).
+ (200 10 ) –3
2100 20 3. Total time 1 + 2 = 24 + 160 = 184 s is less than
4 minutes (= 240 s). Hence heat energy supplied
= 1680 + 8400 = 10080 J
during (240 – 184) = 5 s will be used up in raising
10080 the temperature of the system (container + 200 g
Time needed is 1 = = 24 s
420 -
So the correct choice is (b). plied is 56 s = 420 56 = 23520 J. If
2. Heat energy required to melt ice at 0°C into water temperature, then
at 0°C is 23520 = (100 10–3) 840
Q2 = (200 10–3) (3.36 105) + (200 10–3) 4200
4
= 6.72 10 J which gives = 25.45°C. So the correct choice is (c).
Measurement of Heat 16.11

Questions 4 to 6 are based on the following passage 4. The rate at which heat is produced is
Passage II (a) 35.8 Js–1 (b) 36.8 Js–1
–1
(c) 37.8 Js (d) 38.8 Js–1
A steel drill making 180 revolutions per minute is used
5. The power of the drill is
in during a hole in a block of steel. The mass of the steel (a) 40 W (b) 42 W
block is 180 g. 90% of the entire mechanical energy is used (c) 48 W (d) 56 W
up in producing heat and the rate of rise of temperature of 6. The torque required to drive the drill is
(a) 1.2 Nm (b) 2.2 Nm
of steel = 420 J kg–1 K–1. (c) 3.2 Nm (d) 4.2 Nm
SOLUTION

Q 180
4. = mass sp. heat rise in temperature per 6. Number of revolutions per second =
60
second = (180 10–3) 420 0.5 = 37.8 Js–1. = 3 r.p.s.
Angular frequency of rotation ( ) = 2 3
So the correct choice is (c).
= 6 rad s–1
37.8
5. Power of drill = = 42 W, which is choice Now power = torque angular frequency or P =
0.9 P 42
(b). = = = 2.2 Nm
6 3.14
Thus the correct choice is (b).

IV

Integer Answer Type

1. 2 kg of ice at – 20°C is mixed with 5 kg of water at


20°C in an insulating vessel having negligible heat and ice are kcal/kg/°C and 0.5 kcal/kg/°C and latent
heat of fusion of ice is 80 kcal/kg.
IIT, 2003
SOLUTION

1. mi si [0 – (– 20)] + m L = m s (20 – 0) m = 1 kg.


2 0.5 20 + m 80 = 5 1 20 Mass of water in container = 5 + 1 = 6 kg
17
Thermodynamics
(Isothermal and Adiabatic
Chapter Processes)

REVIEW OF BASIC CONCEPTS 17.3 ZEROTH LAW OF THERMODYNAMICS


17.1 EQUATION OF STATE The Zeroth law of thermodynamics which states that

In the case of ideal gases, the equation of state is


PV = RT, for one mole
and PV = nRT, for n moles Thus if T = TC and TC = T , then T = T .
where P, V and T are respectively the pressure, volume
17.4 INTERNAL ENERGY
and temperature of the gas and R is the universal gas
constant.
amount of energy. This energy is called the
17.2 MOLAR SPECIFIC HEAT and is usually denoted by the symbol U. The
internal energy of solid, liquid or gas consists of two
C of a substance is the amount of
heat energy required to raise the temperature of 1 mole of rotational and vibrational) of the molecules, and
the substance through 1 K. It is expressed in J mol–1 K–1.
s C) are related as of the molecules.
C If the intermolecular forces are extremely weak or
s=
m absent, then the change in internal energy is given by
where m is the number of kilograms per mole in the U = mcv T
substance.
where m is the mass of the gas, cv T
v) is

heat Cv Mcv where M is the molecular mass), we have,


for n moles of an ideal gas
m
p U = nCv T = Cv T
M
17.5 FIRST LAW OF THERMODYNAMICS
Cp – Cv = R When heat energy is supplied to a system, a part of this
when R is the universal gas constant and its value is energy is used up in raising the temperature of the system
R = 8.315 J mol–1 K–1
the rest is used up in doing external work against the
17.2 Comprehensive Physics—JEE Advanced

surroundings. Thus, if Q is the heat energy supplied to Cp


a gas and if W is the work done by it, then from the law where = .
Cv
of conservation of energy, the increase U in the internal
Q – W) or For an adiabatic process, Q
Q= U+ W thermodynamics, Q = U + W, it follows that
U + W = 0 or W = – U
W is positive.
law of thermodynamics which may be stated in words hence U is negative, i.e. the temperature of the
as ‘ gas falls.
W is nega-
tive. Hence U is positive, i.e. the temperature of
the gas increases.
’ 3. Isochoric Process
Sign convention for Q, W and U
1. Q is positive if heat is supplied to the system and an isochoric process. The relation between P and T is
negative if heat is taken out of the system. P P1 P2
P T = constant
2. W is positive if work is done by the system and T T1 T2
negative if work is done on the system. The work done in a process is W = P V. For an
3. U is positive if the temperature of the system in- isochoric process, V = 0. Hence W
creases and negative if the temperature of the sys- law of thermodynamics it follows that
tem decreases. Q= U

17.6 THERMODYNAMIC PROCESSES Q is positive. Hence


U is positive, i.e. the temperautre of the gas rises.
Hence pressure also increases.
1. Isothermal Process
Q is negative. Hence
U is negative, i.e. the temperature of the gas falls.
. From PV = nRT, it follows Henc pressure also falls.
n = constant), the relation
between P and V is 4. Isoboric Process
PV = constant P 1V 1 = P 2V 2
. The relation between V and T is
T = constant)
V V1 V2
If T is kept constant T = 0. Hence U = 0, i.e. for an V T = constant
T T1 T2
isothermal process, the internal energy of the gas remains
from V T, that the temperature rises. Hence W
that
and U
Q= W thermodynamics, that Q is also positive. Hence Q,
i.e. all the heat energy supplied to the gas is used up in U and W are all positive in isobaric expansion.
doing work. Q,
U and W are all negative.
gas. Hence both W and Q are positive.
17.7 WORK DONE IN A PROCESS
on the gas. Hence both W and Q are negative.
2. Adiabatic Process V2
from volume V1 to volume V2 is given by W =
. For an V1

adiabatic process, the realtion between P and V is 1. Work Done in an Isothermal Process When an
PV = constant P 1V 1 = P 2V 2
17.3

at constant temperature) work is done by it. For 4. Work Done in an Isochoric Process In an iso-
an isothermal process, the equation of state for n choric process, volume V is constant, i.e. = 0.
moles of ideal gas is Hence, work done W = 0.
PV = nRT
where T is the constant absolute temperature and R 17.8 INDICATOR DIAGRAM
is the universal gas constant. The value of R for all P – V diagram) is a graph in which
gases is R = 8.31 J K–1 mol–1 P) is plotted on the V) on
n RT the -axis.
P= Figure 17.1 shows indicator diagrams for expansion,
The work done V
compression and for a closed cyclic process. The initial
V2
V2 P1, V1) is represented by point
W = nRT = nRT ln P2, V2) by point . The intermediate states are repre-
V V1
V1
sented by points between and on the curve .
e)
V2
or W = 2.303 nRT log
V1
where V1 is the initial volume and V2 -

P1 and P2 P 1V 1 =
P 2V 2)
P1
W = 2.303 nRT log
P2
2. Work Done in Adiabatic Process When a gas
undergoes an adiabatic change, the pressure-
volume changes obey the relation
PV = constant = C
where = Cp/Cv
the gas at constant pressure to that at constant
volume.

C
P= , where C is a constant
V
The work done is given by
1 Fig. 17.1
W= P 2V 2 – P 1V 1)
1 The work done in a process is given by
1 W = area enclosed by P – V curve and volume axis.
= P 1V 1 – P 2V 2) Thus, the work done is given by the area of the shaded
1
portion in Fig. 17.1.
nR
= T 1 – T 2)
1 17.9 EFFICIENCY OF AN IDEAL HEAT ENGINE
since P1V1 = nRT1 and P2V2 = nRT2, T1 and T2 ) of an ideal reversible heat engine is
being the absolute temperatures before and after the given by
adiabatic change. T2
3. Work Done in an Isobaric Process For an iso- =1 –
T1
baric process, pressure P is constant. Therefore, the
work done is given by where T1 = absolute temperature of the source
V2 which supplies heat
W =P = P V 2 – V 1) and T2 = absolute temperature of the sink which takes in
V1 the part of heat not converted into useful work,
17.4 Comprehensive Physics—JEE Advanced

T1 is always greater than T2. If T1 = T2 = 0 implying 17.2


that an engine working under isothermal conditions can
produce no useful work. Complete conversion of heat into = 1.4) at 127°C are
= 1) is possible only if T2 = 0, i.e. the sink is expanded adiabatically to twice the original volume.
at absolute zero, which is unattainable.
done in the process. Given R = 8.3 J K–1 mol–1 and
0.4
Cv, Cp AND = Cp /Cv FOR AN IDEAL = 0.76.
17.10 GAS
SOLUTION
1. For a monoatomic gas, T1 V1 – 1)
= T2 V2 – 1)

3R 5R 5 1
Cv = , Cp = and = = 1.67 V1
2 2 3 T2 = T1 0.4

2. For a diatomic gas, V2


5R 7R 7 = 400 0.76 = 304 K
Cv = , Cp = and = = 1.4
2 2 5 nR
W= T 1 – T 2)
3. For a triatomic or polyatomic gas 1
4
Cv = 3R, Cp = 4R and = 1.33= 2 8.3
3 =
17.11 RELATION BETWEEN Cp, Cv AND 1.4 1

Cp – Cv = R
17.3
CP
and =
Cv takes up 746 J of heat from the high temperature res-
or Cp = Cv

R R
Cv = and Cp =
1 1 SOLUTION
T1 = 100°C = 373 K, T2 = 0°C = 273 K, Q1 = 746 J
17.1
W T T1 T2
5 moles of an ideal gas are compressed to half the =1– 2 W = Q1
initial volume at a constant temperature of 27.0°C. Q1 T1 T1
Calculate the work done in the process. Given R = 373 273
= 746
8.3 J K–1 mol–1. Write your result up to appropriate 373
= 200 J
SOLUTION Q2 = Q1 – W = 746 – 200 = 546 J
Given T = 27.0°C = 300 K. Since T is constant, the W 200
process is isothermal = = = 0.268 = 26.8%
Q1 746
V2
W = nRT loge NOTE
V1
1 273
=5 8.3 300 loge =1–
T2
=1– = 0.268
2 T1 373
=5 8.3 300
= – 8628 J 17.4
Since the value of R s. .), Figure 17.2 shows the P-V diagram of a cyclic
the value of W must be rounded off to 2 s. . as process . Calculate the work done in process
W = – 8.6 103 J to to C
The negative sign indicates that work is done on the C to
gas.
17.5

8 17.6
= 1.4) are heated
6
at constant volume. If 280 J of heat energy is supplied
(Nm–2)
4
gas and the work done.
2
SOLUTION
Since V = 0, W -
2 4 6 8 10
modynamics,
(litre)
= = 280 J
Fig. 17.2
17.7
SOLUTION The pressure P of an ideal gas varies with volume
W V as P = where is a constant. The volume of n
moles of the gas is increased from V to mV. Find the
+ area of rectangle
work done and the change in internal energy.

1 SOLUTION
= +
2
mV
V2 2
mV mV
1
= 4 6 10 –3
+4 6 10 –3
W= V = = = m2 – 1)
2 2 2
1 litre = 10–3 m3)
V V V

= 36 10–3 J Q= U+W
W C = –24 10–3 J nCp T= U+W
Negative sign shows that the work is done on the Cp
gas. n C T= U+W
C
WC volume is constant)
U= U+W U=nC T)
W = 36 10–3 – 24 10–3 = 12 10–3 J
2
which is equal to area enclosed by the closed W m2 1
U= =
loop . 1 2 1

17.5 17.8
= 1.4) are heated Two moles of a monoatomic gas undergo a cyclic
at constant pressure. If 280 J of heat energy is sup- process as shown in Fig. 17.3.

2 x 105

SOLUTION
(Nm–2)
1 x 105
C nCp T
U=nC T=n Cp T =
Cp
Given Q = n Cp T = 280 J. Hence
0 1 2 3 4 5
2.80 ( 3)
U= = 200 J
1.4 Fig. 17.3
Process is isobaric, process C is adiabatic
W= Q– U = 280 – 200 = 80 J and process C is isothermal. Find
17.6 Comprehensive Physics—JEE Advanced

C Hence the sope of P-V graph for an adiabatic process


. is greater than for an isothermal process. Figure 17.4
shows two P-V curves, one for adiabatic expansion
SOLUTION and the other for isothermal expansion.
P = P = 2 105 Nm–2
V = 2 m3, V = 5 m3 and
C
P V = PC V C
For isothermal process C
P V = PC VC
P =P )
1
V 1
VC =
V
Substituting the values of V , V and , we get Fig. 17.4
3
VC
PV
PC = = 0.2 105 N m–2 17.10
VC
Two moles of a diatomic ideal gas, initially at pres-
W=W +W + WC C sure 5.0 104 Pa and temperature 300 K are expanded
1 isothermally until the volume of the gas is doubled
=P V –V)+ P V – P CV C)
1 and then adiabatically expanded until the volume is
V again doubled. Find
+ P V loge
VC
Substituting the values, we get end of the complete proces
W = 6 105 N m–2

17.9 process.
Show that the slope of P-V curve for an adiabatic P-V graph for the complete process.
process is greater than that for an isothermal Given R = 8.3 JK–1 mol–1.
process.
SOLUTION
SOLUTION
Let P1, V1, T1 be the initial pressure, volume and tem-
For an isothermal process, PV = constant. Differenti- perature of the gas
ating w.r.t. volume V we have
For isothermal process
P
+ =0 = P1V1 = P2V2 where V2 = 2V1
iso V
V1 1
For an adiabatic process, PV = constant, P2 = P1 = 5.0 104
Cp V2 2
where = . = 2.5 104 Pa
Cv
Differentiating w.r.t. volume V, we have In an isothermal process, the temperature remains
PV – 1 +V =0 constant. Hence T2 = T1 = 300 K. Therefore change
U)1 = 0.
P
= Work done is
adia V
V2
W1 = nRT1ln =2 8.3 300
Since > 1, . V1
adia iso
17.7

= 3.45 103 J 5
=2 8.3
For adiabatic process 2
P 2V 2 = P 3V 3 5R
Cv = for diatomic gas)
2
1.4
V2 1 = – 3.03 103 J
P3 = P2 = 2.5 104
V3 2
P3 103 Pa, T3 = 227 K
3
10 Pa W = W1 + W2 = 3.45 103 + 3.03 103 = 6.48 103 J
– 1) – 1)
T 2V 2 = T 3V 3 U U)1 U)2 = 0 – 3.03 103 =
1 – 3.03 103 J. The negative sign shows that
V2
T3 = T2 there is a decrease in internal energy.
V3
P-V graph for the complete process is shown in
0.4
1 Fig. 17.5.
= 300 = 227 K
2
Work done is Isothermal expansions
nR
W2 = T2 T3
1
2 8.3 Adiabatic expansions
=
1 4 1)
= 3.03 103 J
Change in internal energy is
5R 1 2 3
)2 = nCv T = n T
2
Fig. 17.5

Multiple Choice Questions with Only One Choice Correct


1. If 2 moles of an ideal monoatomic gas at temperature the work done on the gas is 50 J. The change in
T are mixed with 3 moles of another monoatomic internal energy of the gas in the process is
gas at temperature 2T, the temperature of the
mixture will be
8T 6T
5 5
4T 3T
3 2
2. If heat energy Q is supplied to an ideal diatomic gas,
the increase in internal energy is U and the work Fig. 17.6
done by the gas is W. The ratio Q : U : W is

3. Figure 17.6 shows a cyclic process. When a given 4. Figure 17.7 shows a cyclic process for an
mass of a gas is expanded from state to state ideal diatomic gas. The ratio of the heat energy
, it absorbs 30J of heat energy. When the gas is absorbed in the process to the work done on
adiabatically compressed from state to state , the gas in the process C is
17.8 Comprehensive Physics—JEE Advanced

10. For a thermodynamic process, the P-V graph for a


monoatomic gas is a straight line passing through
the origin and having a positive slope. The molar
heat capacity of the gas in this process is
R R
3
R R
2
11.
600 K has a work output of 800 J per cycle. How
much heat energy is supplied to the engine from the
Fig. 17.7
source in each cycle?
7 7
4 ln 2 2 ln 2
5 ln 2 5 ln 2 12.
4 2
5. The internal energy of 3 moles of hydrogen at should the temperature of the source be increased
temperature T is equal to the internal energy of n
moles of helium at temperature T/2. The value of
n
ideal gases)
13.
3 The pressures and volumes corresponding to
2 P = 3 104 Pa,
V = 2 10 m , P = 8 10 Pa, V = 5 10–3 m3.
–3 3 4
6. The temperature of n moles of an ideal gas is
increased from T to 3T in a process in which the In process , 600 J of heat and in process ,
temperature changes with volume as T = 2 where 200 J of heat is added to the system. The change in
is a constant. The work done by the gas in this the internal energy in process would be
process is
n RT n RT
3
n RT n RT
2
7. In a certain process, pressure P, volume V and
temperature T of a gas are related as PV = n
where and n are constants. The work done by the
gas when the pressure is kept constant, is propor-
tional to
T )1/n T)n
T n + 1) T n –1)
8. If the pressure of an ideal gas in a closed container is
increased by 2%, the temperature of the gas increases
Fig. 17.8
by 5°C. The initial temperature of the gas is
14. One mole of an ideal gas requires 207 J heat to
raise its temperature by 10 K when heated at con-
9. stant pressure. If the same gas is heated at con-
pressure P in the balloon is related to the volume V stant volume to raise the temperature by the same
as PV2/3 = , where is a constant. If T is the tem- R, the gas constant
perature of the mixture, volume V is proportional to = 8.3 JK–1 mol–1)
T T2
3
T T4
17.9

15.

during the cycle is


PV
PV PV

Fig. 17.10

21. In rising from the bottom of a lake to the top, the


temperature of an air bubble remains unchanged,
but its diameter is doubled. If h is the barometric

Fig. 17.9 density ) at the surface of the lake, the depth of the
16. The equation of state corresponding to 8 g of O2 is
2 to be an ideal gas)
h h
RT h h
PV = 8 RT PV =
4 22. Entropy of a thermodynamic system does not
RT change when the system is used for
PV = RT PV =
2
17. The equation of state of a gas is cold reservoir
2
aT
P Vc RT + )
V -
chorically
where a, , c and R are constants. The isotherms can
be represented by
P= m
– n 23. Heat energy absorbed by a system in going through
where and depend only on temperature and a cyclic process shown in Fig.17.11 is
7 4
m = – c, n m = c, n = 1 2 –3
m = – c, n m = c, n = – 1
18. When an ideal monoatomic gas is heated at con-
stant pressure, the fraction of heat energy supplied
which increases the internal energy of the gas is
2 3
5 5
3 3
7 4
19.
heats is 4150 J kg–1 K–1 and the ratio of the two Fig. 17.11
24. = 5/3) is mixed
gas at constant volume in units of J kg–1 K–1? = 7/5). What
will be value of for the mixture?

20. to
by three different paths 1, 2 and 3 as shown in Fig.
17.10. If W1, W2 and W3 respectively be the work 25. If the ratio Cp/Cv = , the change in internal energy
done by the gas along the three paths, then of the mass of a gas, when the volume changes from
V to 2V at constant pressure P is
W1 > W2 > W3 W1 < W2 < W3
R
W1 = W2 = W3 W1 < W2 W1 < W3 PV
1
17.10 Comprehensive Physics—JEE Advanced

PV PV
1 1
26. Two cylinders and
equal amounts of an ideal diatomic gas at 300 K.
The piston of is free to move, while that of is

gas in each cylinder. If the rise in temperature of the


gas in is 30 K, then the rise in temperature of the
gas in is Fig. 17.12

30. T 1,

piston. The gas is allowed to expand adiabatically


27. Two identical containers and - to a temperature T2 by releasing the pison sudden-
tionless pistons contain the same ideal gas at the ly. If L1 and L2 are the lengths of the gas column
same temperature and the same volume V. The before and after expansion respectively, then T1/T2
mass of the gas in is m and that in is m . is given by
The gas in each cylinder is now allowed to expand L1
2/3
L1
V. The
changes in pressure in and are found to be P L2 L2
2/3
and 1.5 P respectively. Then L2 L2
m m m =3m L1 L1
m =2m m =4m IIT, 2000
31. Starting with the same initial conditions, an ideal
28. Two monoatomic ideal gases 1 and 2 of molecular gas expands from volume V1 to V2 in three different
masses M1 and M2 respectively are enclosed in ways. The work done by the gas is W1 if the process
separate containers kept at the same temperature. is purely isothermal, W2 if purely isobaric and W3 if
The ratio of the speed of sound in gas 1 to that in purely adiabatic. Then
gas 2 is W2 > W1 > W3 W2 > W3 > W1
M1 M2 W1 > W2 > W 3 W1 > W3 > W2
M2 M1
IIT, 2000
M1 M2
32.
M2 M1 expansion. If its temperature falls by 2K, its inter-
IIT, 2000 nal energy will
29. T and
volume V. Its volume is increased by V due to
an increase in temperature T, pressure remaining
constant. The quantity = V V T) varies with 7
33. 5 moles of Hydrogen initially at S.T.P.
5
are compressed adiabatically so that its temperature
IIT, 2000
becomes 400ºC. The increase in the internal energy

R = 8.30 J mol–1 K–1)

34. During an adiabatic process, the pressure of a gas is


proportional to the cube of its absolute temperature.
The value of Cp/Cv for that gas is:
17.11

3 4 IIT, 2002
5 3
5 3
3 2
35. 2
32) at a temperature T. The pressure of the gas
is P
T has a
pressure of
P
P
8
P P
Fig. 17.15

36. In a given process on an ideal gas, dW = 0 and 40. Figure 17.16 shows the P-V
dQ < 0. Then for the gas mass of an ideal gas
undergoing cyclic pro-
cess. represents
isothermal process
and represents adi-
ill increase
abatic process. Which
IIT, 2001 of the graphs shown in
37. P-V plots for two gases Fig. 17.17 represents
during adiabatic processes the P-T diagram of the Fig. 17.16
are shown in Fig. 17.13. cyclic procee?
Plots 1 and 2 should corre-
IIT, 2003
spond respectively to
2

2 and He Fig. 17.13

2 and N2
IIT, 2001
38.
C , as shown in Fig. 17.14. If the net heat sup-
plied to the gas in the cycle is 5 J, the work done by
the gas in the process C is

IIT, 2002

Fig. 17.17

41. The pressure and density of a diatomic gas


p1, 1) to
p
p2, 2). If 2 = 32, then 2 is
1
p1
Fig. 17.14
1
39. Which of the graphs shown in Fig. 17.15 correctly
represents the variation of V/dp)/V with p 128
for an ideal gas at constant temperature?
17.12 Comprehensive Physics—JEE Advanced

42. = 1.4) is adiabatically Va V Va T


compressed so that its temperature rises from 27°C = = 2
Vc V V T1
to 35°C. The change in the internal energy of the
R = 8.3 J/mole/K) Va V Va T
= = 1
V Vc V T2

43. V, pressure
P and temperature T. The mass of each molecule
of the gas is m is the

P mP
T
44.
Fig. 17.18
a monoatomic ideal gas which expands at con-
stant pressure. What fraction of the heat energy is 49. in the
converted into work? V – T diagram. Which of the diagrams shown in
2 Fig. 17.20 shows the same process on a P – V
3 diagram.
2 5
5 7
45.
gas at 0°C and also contains an insulated piston
of negligible weight and negligible thickness at
the middle point. The gas on one side is heated to
100°C. If the piston moves through 5 cm, the length
of the hollow cylinder is

Fig. 17.19
46.
volume V1 to volume V2 and then compressed
adiabatically to original volume V1. The initial
pressure is P1 P3. If the net
work done is W, then
P 3 > P 1, W P 3 < P 1, W < 0
P 3 > P 1, W P 3 = P 1, W = 0
IIT, 2004
47.

resistance 50
for 10 minutes by connecting it to a d.c. source of
10 V. The change in the internal energy is
Fig. 17.20

50. = 1.4) expands from 5 10–3 m3 to


48. Two different adiabatic paths for the same gas inter- 25 10 m at a constant pressure of 1 105 Pa.
–3 3
sect two isothermals at T1 and T2 as shown in the The heat energy supplied to the gas in this process
P–V is
17.13

. The change in the


internal energy of the gas is
51. Three moles of an ideal gas are taken through a
cyclic process as shown on T–V diagram
in Fig. 17.21. The gas loses 2510 J of heat in the
complete cycle. If T = 100 K and T = 200 K, The IIT, 2005
work done by the gas during the process is 54. The P–V diagram for n moles of an ideal gas

The maximum temperature of the gas during the


process is
P0V0 3P0V0
4nR nR
2 P0V0 3P0V0
nR 2nR

Fig. 17.21

52. Liquid oxygen at 50 K is heated at 300 K at con-


stant pressure of 1 atmosphere. The rate of heat-
ing is constant. Which of the graphs shown in
Fig. 17.22 represent the variation of temperature
T )? Fig. 17.23
IIT, 2004 55.
lid insulated kettle is heated by an electric heater
of power 1 kW. The heat is lost from the lid at the
rate of 160 J/s. The time taken for heating water
–1 –1
K ) from 20°C
to 75°C is

IIT, 2005
2
56. PT = constant.

1 2
T T
Fig. 17.22
3 4
T T
53. One mole of a monoatomic ideal gas is contained
in a insulated and rigid container. It is heated by IIT, 2008

ANSWERS

1. 2. 3 4. 5. 6.
7. 8. 9. 10. 11. 12.
13. 14. 15. 16. 17. 18.
19. 20. 21. 22. 23. 24.
25. 26. 27. 28. 29. 30.
17.14 Comprehensive Physics—JEE Advanced

31. 32. 33. 34. 35. 36.


37. 38. 39. 40. 41. 42.
43. 44. 45. 46. 47. 48.
49. 50. 51. 52. 53. 54.
55. 56.

SOLUTIONS
1. Let T0 be the temperature of the mixture. Since the 1
total internal energy remains unchanged, we have = – 2 n RT0 ln
2
U of mixture = U1 + U2
= 2 nRT0
n 1 + n 2) C v T 0 = n 1 C v T 1 + n 2 C v T 2 Q 7
n 1 + n 2) T 0 = n 1 T 1 + n 2 T 2 =
T0 = 2T + 3 T ) = 8T W 4 ln 2
8T 5. The internal energy of n moles of an ideal gas at
Which gives T0 =
5 7R 5R temperature T is given by
2. For a diatomic gas Cp = and Cv =
2 2 U= n RT
7 2
Q = n Cp T = nR T
2 where = number of degrees of freedom.
For hydrogen, = 5. Therefore
5
U = n Cv T = nR T 5 15
2 U1 = 3 RT = RT
2 2
7 5 For helium, = 3. Therefore
W= Q– U= nR T – nR T 3 3
2 2 U2 = n R T/2) = n RT
= nR T 2 4
Q: U: W=7:5:2 Given U1 = U2, i.e.
3. In the process , work is done on the gas. 15 3
W) = –50 J. Since this process is RT = nRT
2 4
Q) which gives n = 10.
thermodynamics, the change in internal energy in 6. Given T = 2
. Therefore =2 or
this process is
U) Q) W) =
2
nR T
Since the process is cyclic, there is no net change PV = n RT P=
V
in internal energy. Hence 3T
U) U) = –50 J Work done W = V
4. In the process , V is proportional to T. Hence T
pressure P remains constant. Therefore, heat 3T
nRT
C p = 7 R/2 for =
V 2
a diatomic gas) T
7R 3T
= n Cp T = n T 0 – T 0) nR T
Q) =
2 2 2
T
7 3T
= n RT0 nR 2
2 = = T)
Process C is isothermal in which the gas is 2 T
compressed. Hence work done on the gas in this = n RT
process is
V0
W ) C = – n R T0) ln 7. Given V =
n
. Since P = constant,
2V0
P
17.15

= T n – 1) 300
T2 = = 750 K
P 0.4
n 1 T2 – T2
Work done W = V= T
13. Process is isochoric, i.e. the volume remains
= n
+c constant. Thus V = 0. Hence work done P V = 0.
where c = constant of integration. Hence the Process is isobaric, i.e. the pressure remains
constant and external work has to be done. The
8. Since the volume of the gas is constant, work done = P V – V ) = 8 104 10–3
P1
= 1
T – 2 10–3) = 240 J. Therefore, change in internal
P2 T2 energy is
Now P2 = P1 + 0.02 P1 = 1.02 P1 and T2 = T1 + 5. = – = 800 – 240 = 560 J

P1 T1 14. Heat energy required to raise the temperature of n


= T1 = 250 K
1.02 P1 T1 5 moles of a gas by T at constant pressure is
9. PV 2/3 = Qp = n Cp
Equation of state is PV = nRT P=
n RT
. Using Heat energy required to raise the temperature n
V moles of a gas by T at constant volume is
Qv C
n RT Qv = n Cv T, = v
V 2/3 = Qp Cp
V
Cv 3R / 2
or T V –1/3 = = constant or Qv = Qp = Qp
nR Cp 5R/2
Hence V T 3 3
10. Since the P – V graph is a straight line with a = 207 = 124.2 124 J
5
positive slope, P V
or PV –1 = constant
15. Work done = area enclosed by the indicator
For a process in which PV n = constant, the molar
diagram
heat capacity is given by 1
R R =
C= 2
1 1 n 1
= P – P) V – V)
5 2
Putting n = – 1 and =
3 = 3 PV
we have
R R 3R R
C= = = 2R 1
5 1 1 2 2 16. . Nown) =
1 4
3
the equation of state for n moles of an ideal gas is
W T 300 1
11. =1– 2 =1– = 0.5. Therefore Q = 2W PV = n R T = RT =
RT
Q T1 600 4 4
=2
12. T2 = 300 K. Now = 1 – T2/T1. When = 40% = 17. Expanding the equation of state we have
0.4, the value of T1 is given by PV c + aT2 V c–1 = RT +
T2
= 1 – 0.4 = 0.6 or P = – aT 2 V –1 + RT V–c + –c
–c –1
T1 or P= –
T 300 where = RT + and = aT2. We are
or T1 = 2 = = 500 K. When = 60%
0.6 0.6 given that
= 0.6, the value of T2 should be P= m
– n
17.16 Comprehensive Physics—JEE Advanced

Comparing the powers of V m Cp 3R


= – c and n = = = 3 = 1.5
Cv 2R 2
18. Now Qp = n Cp T and Qv = n Cv T Qv gives
the heat energy which increases the internal energy
of the gas. Thus the required fraction is 25. Let T be the increase in temperature when the
Qv C 1 1 3 volume of the gas is changed by V at constant
= v = = = pressure. The change in internal energy of n moles
Qp Cp 5/3 5
of a gas is given by
5
For monoatomic gas = U = n Cv T
3
We know that Cp – Cv = R
Cp R Cp
19. Given Cp – Cv = 4150 and Cp/Cv = 1.4 or Cp = or =1+ =
1.4 Cv. Therefore, Cv Cv Cv
1.4 Cv – Cv = 4150 R
Therefore, =1+ , which gives
4150 Cv
or Cv = = 10375 J kg–1 K–1 R
0.4 Cv =
1
20. P V P-V) curve, PV = n RT
which is the largest for curve 3 and the smallest for volume changes by V, the change in temperature
T is given by
21. Volume 3
. Since the diameter of the P V = nR T
bubble is doubled in rising from the bottom to the P V PV
or T= =
top of the lake, its volume becomes 8 times. Now nR nR
PV = constant. Therefore, the pressure at the bot- V = 2V – V = V
tom of lake = 8 times that at the top. Let H be the
depth of the lake.
h – h) m R PV PV
H U=n =
1 nR 1
or H = 7h m
=7h m

26. Heat is given to the gas in cylinder at constant


pressure while the same amount of heat is given to
22. When work is converted into heat at a constant tem- the gas in cylinder at constant volume. Heat given
perature, the entropy of the system remains con- to gas in is
Q = n Cp T
23. Heat energy absorbed = work done = area of the Heat given to gas in is Q = nCv T
loop
Since Q = Q , we have
= r2 = 2
/4 = 2
= 102 T = nCv T
4 nCp
Cp 7
or T = T = 30 K = 42 K
24. For a monoatomic gas, Cv = 3R/2 and for a diatomic Cv 5
gas, Cv = 5R/2. Since one mole of each gas is mixed for a diatomic gas, Cp/Cv = 7/5)
together, the Cv of the mixture will 27. The equation of state for an ideal gas of mass m and
1 3R 5R molecular mass M is
Cv = = 2R
2 2 2 m
PV = RT
Now Cp – Cv = R. Therefore, for the mixture, Cp = R M
+ Cv = R + 2R = 3R For an isothermal process, T = constant. Differen-
heats of the mixture is T, we get
17.17

P V+V P=0 1 V 1 1
V or = or =
or P= – P V T T T
V
Thus, the value of decreases as T is increased.
mRT
P=
MV – 1) – 1)
30. For adiabatic process, T1 V 1 = T2 V 2 . Thus
mRT 1
P= – V = 2V – V = V) T1 V2
MV =
T2 V1
m RT m RT
P =– and P =– For a monoatomic gas, V2/V1 = L2/L1.
MV MV Hence
5
m 1 2/3
P T1 L2 3 L2
Hence = = =
P m T2 L1 L1
1 m
Given P = 1.5 P . Therefore, = 31. Since the slope of the P–V graph for adiabatic
15 m expansion is times that for isothermal expansion,
or 3m =2m . curves and
28. The speed of sound in a gas of bulk modulus and represent isothermal and adiabatic expansions of
density is given by the gas from initial volume V1 V 2.

v=
V P
is given by =–
V
Now, for a perfect gas, PV = nRT. Differentiating
at constant T, we get
V P
P V + V P = 0 or = –P
V
P
Hence v=

If m is the mass of the gas and M its molecular Fig. 17.24


mass, then between volumes V1
m mRT
PV = RT or PM = = RT and V2 is greater than the area under curve be-
M V tween V1 and V2, it follows that W1 > W3.
or
P
=
RT
or v2 =
RT P–V graph for isobaric
M M -
ume V1 and pressure P1 V2 -
RT
or v= sure remaining unchanged at P1
M between
RT RT volumes V1 and V2 is greater than the area under
Hence v1 = and v2 = which curves and . Hence W2 is greater than W1 and
M1 M2
W3
v1 M2 32. = + . In an adiabatic process, = 0.
give =
v2 M1 Hence = – = – 4.5 J. Hence the correct
29. For an ideal gas, PV = nRT. Since pressure P is kept
constant, 33. Given T1 = 0ºC = 273 K, T2 = 400ºC = 673 K
P V = nR T nR 5 8.3 400
Work done W = T2 T1
or
V
=
nR
=
nRV
=
V nRT 1 7
P 1
T P nRT T V 5
17.18 Comprehensive Physics—JEE Advanced

= 41500 J = 41.5 kJ for a diatomic gas, -


atomic gas for which = 5/3. Therefore, the slope
to be negative, i.e. W of the P-V curve is less for a diatomic gas than for
law of thermodynamics = + . For an a monoatomic gas. Hence curve 1 corresponds to
adiabatic process, = 0. Hence = – = diatomic gas and curve 2 to monoatomic gas. Thus
implies
that the internal energy increases. Hence the correct 38. Process occurs at constant pressure.

34. For an adiabatic process of Q. 38)


TPn = W = = P V2 – V1) = 10
1 Cp Process C, occurs at constant volume. Hence
where n = , and is a constant. W = 0. Given Q = 5 J, i.e. total work done is
Cv
W = 5 J. Therefore, we have
Therefore,
1/ n
W =W +W +W
P= or 5 = 10 + 0 + W
T
or W
Since n = constant for a given gas,
39. For an ideal gas, pV = nRT. Differentiating, we
P T –1/n have since T is kept constant
1
Given P T 3. Hence = 3 or = 3, which + V = 0 or
V
n 1 p
p
3
gives =
2 1 1
Hence =
35. For a gas, PV = nRT. Hence V p
1 mole RT
P)O2 = of versus p.
V
40. Since is an isothermal process, the temperature
1 mole R 2T of the gas remains constant between . Hence
P)He =
V the P-T diagram must be perpendicular to the
T-axis between and . Hence the correct choice
P He
=2
P O2 1
41. pV = constant. For a given mass of gas, V .
P)He P)O2 Hence
p
36. = constant
= –
p1 p2
Given = 0 and < 0. Hence the change in =
internal energy < 0. Now, for an ideal gas, the 1 2

internal energy can decrease only by decrease in


p2 2 7/5 7
or = = 128
37. For an adiabatic process, PV = constant, Differen- p1 1
tiating, we have
–1 P Cp R
PV + V = 0 or 42. U = Cv T. Now Cp – Cv = R or –1=
V Cv Cv
Since at any instant PV = constant, , i.e. R Cp
or Cv = , where = . Hence
the slope of P-V curve is proportional to . Now, 1 Cv
17.19

R T 8.3 8 to V1, P3 are as shown in Fig. 17.25. Let W1 and


U= = = 166 J W2 be the work done in isothermal expansion and
1 1 4 1)
adiabatic compression respectively. Therefore, net
work done is
m W = W1 W 2) = W 1 – W 2
43. PV = RT. Therefore, the density of the gas is
M
m PM PmN mP
=
V RT RT

44. Heat energy supplied = Cp . Change in inter-


nal energy = Cv . Therefore, work done =
– Cp – Cv ) .

Cp Cv 1
= =1–
p T

1 2
=1– =
5/ 3 5
= 5/3 for a monoatomic gas) Fig. 17.25
Now, the area under the adiabatic curve is more
45. Let L than that under the isothermal curve. Hence
and r its radius. Since the mass of the gas remains W2 > W1. Therefore, W < 0. Hence the correct
unchanged and the pressures of the gas in both sides .
are equal, we have, from Charles’ law, 47. Heat produced is given by
2 2
V1 V 1010 60
= 2 = = 1200 J
T1 T2 R 50
Since the container is rigid, the change in volume
L L
Given V1 = 5 r 2, V 2 = 5 r 2, = 0. Hence work done = = 0. From the
2 2
T1 = 0°C = 273 K and T2 = 100°C = 373 K. Using energy is = – = = 1200 J. Hence the

L L 48. The two adiabatic paths and for the gas inter-
5 5 sect the two isothermals and at temperatures
2 = 2 T1 and T2 a and lie
273 373
on the same adiabatic path, we have
which gives L = 64.6 cm. T1Va – 1) =T2V – 1)
46. For an isothermal process: PV = constant and for an 1
Va T2
adiabatic process: PV = constant, where is the or =
Cp /Cv) of the gas. V T1
When a gas is compressed from a volume V to a Since points and c also lie on the same adiabatic
V – V), the increase in pressure is path,
– 1) – 1)
T1 V ) = T2 Vc
VP
P)adia = for an adiabatic compression 1
V V T2
or =
VP Vc T1
P)iso = for an isothermal compression.
V
Hence P3 will be greater than P1. Therefore, the Va
1
V
1
P–V diagrams of isothermal expansion from V1, =
P1 to V2, P2 and adiabatic compression for V2, P2 V Vc
17.20 Comprehensive Physics—JEE Advanced

Va V W = P V = nR T PV = nRT)
or = = nR T – T )
V Vc
= 3 8.3
49. Since the temperature T remains constant along the Process
path , P will be inversely proportional to V along work done in this process W = 0. Since the
this path. Hence, as P increases, V must decrease whole process is cyclic, the change in internal
in a nonlinear fashion. This is represented by the energy in the complete cycle is zero, i.e. U = 0.
curve in Fig. 17.26.
, the volume V is constant. Hence Q = – 2510 J)
the graph of P against V is a straight line perpen- Q= U+W= U+W +W +W
dicular to the V-axis. On a P – V diagram, the cor- W +0
responding path is as shown in Fig. 17.26. or W
For the path , V is directly proportional to T The negative sign shows that the work is done
pressure remaining constant. The corresponding
path is, therefore a straight line parallel to the 52.
V-axis. Thus the cyclic process on a P–V diagram oxygen is greater than 50 K. Therefore, between 50
K and 300 K, liquid oxygen undergoes a change of

53. The heat energy supplied is


Q = I2
2
100 60)
3
= 240 10 J = 240 kJ
Since V W
Fig. 17.26 of thermodynamics, U = Q = 240 kJ. Thus the
50. Work done on the gas is
54. The equation of straight line is
W = P V = P V – Vi)
P = mV + c
= 1 105 10 –3 where m is the slope and c is the intercept. For
= 2000 J points and , we have
PV 2 P0 = mV0 + c
The internal energy is given by U =
1 and P0 = m V 0) + c
PVi PV
Ui = ,U = . P0
1 1 These equations give m = – and c = 3 P0.
V0
Therefore, change in internal energy is nRT
P Now PV = nRT P=
U = U – Ui = V – Vi) V
1 we have
1
1 105 25 5 10 3 T= mV2 + cV
= = 5000 J nR
1.4 1 2
T will be maximum if = 0 and 2
< 0.
energy supplied to the gas is V and putting
Q = W + U = 2000 + 5000 = 0, we get
= 7000 J 2 mV + c = 0
c
which gives V=–
51. In process , the volume V increases linearly 2m
with temperature T. Hence process is isobaric c2 1 3P0 2
Tmax = – =– = P0V0
4nRm 4nR P0 /V0 4nR
process is
17.21

55. Mass of 2 litres of water = 2 kg. Heat energy needed nRT


to raise the temperature of 2 kg of water from 20°C P= . Hence
V
to 75°C is nRT 3 =
Q=2 103) 55 = 4.62 105 J
Differentiating we have
If is the time taken, heat energy supplied by the
V 3nRT 2
heater in time is 3nRT 2 T = V
Q1 time) = 1000 T
Heat energy lost in time is
Q2 = 160 V 3nRT 2
=
Heat energy available for heating water is V T
Q = Q1 – Q2 = 840 J nRT
Using V = and PT 2 =
Equating Q = Q , we get 550 s. Thus the P
3
we get = .
56. Given PT 2 = PV = nRT, we have T

II

Multiple Choice Questions with One or More Choices Correct


1. Figure 17.27 is the =
P-V diagram for a =–
Carnot cycle. In this
3. If Q represents the heat energy supplied to a
diagram,
system, U the increase in internal energy and W
repre- the work done by the system, then which of the
sents isothermal following are correct?
process and
Q = W for an isothermal process
adiabatic pro-
cess U=– W for an adiabatic process
repre- Fig. 17.27
U= Q for an isochoric process
sents adiabatic
U = – Q for an isobaric process.
process and isothermal process
represents isothermal process and 4. The initial state of n moles of an ideal gas is rep-
adiabatic process resented by P1, V1, T1 P 2, V 2,
represents adiabatic process and T2. Wi is the work done by the gas in an isothermal
isothermal process. T1 = T2 = T) and Wa in an adiabatic pro-
cess, then
2. Figure 17.28 shows the P-V diagram of a cyclic
V2
process. If is the heat energy supplied to the Wi = nRT loge
system, is the inter- V1
nal energy of the sys- P1
tem and is the work Wi = nRT loge
done by the system, P2
then which of the fol- 1
Wa = P 1V 1 – P 2V 2)
lowing relations is/are 1
correct
nR
= Wa = T 1 – T 2)
1
=0 Fig. 17.28
17.22 Comprehensive Physics—JEE Advanced

5. P, volume V
and temperature T is allowed to expand adiabati-
cally until its volume becomes 5.66 V while its
temperature falls to T/2. If is the number of
degrees of freedom of gas molecules and W is the
work done by the gas during the expansion, then
=5
5PV 3PV
W= W=
4 2
Fig. 17.29
6. -
namic process involving four steps. The amounts is 0.036 J.
of heat involved in these steps are Q1 C is – 0.024 J.
Q2 = – 5585 J, Q3 Q4 = 3645 J respec- C is zero.
tively. The corresponding amounts of work done is 0.06 J.
are W1 = 2200 J, W2 = – 825 J and W3 = – 1100 J 11. Figure 17.30 shows the P–V diagram for an ideal
and W4 gas. From the graph, we conclude that
. Then
W4 W4 = 275 J
16%
zero.
7. P, volume V and tempera-
ture T. The ratio CP/Cv = and U is the internal
energy. If R is the gas constant, then
R
Cv = U = nCv T
1
PV
U= U = nCpT
1
8.

Fig. 17.30

of the source should be increased by 28.6°C. 12. Figure 17.31 shows the P – V diagram for an ideal
gas. From the graph we conclude that
of the sink should be decreased by 28.6 °C.

800 J per cycle, the work output per cycle is


200 J.
9.
minutes. The room temperature is 27°C. The latent
heat of fusion of ice is 80 cal/g.
105 J.

to 10.
Fig. 17.31
104 J.
is adiabatic.
10. P–V diagram of a cyclic
process process.
17.23

to , the temperature T of

decreases.
in the process.
13. n moles of an ideal monoatomic gas is kept in a 16. One mole of oxygen at 27 °C is enclosed in a ves-
closed vessel of volume 0.0083 m3 at a tempera- sel which is thermally insulated. The vessel is
ture of 300 K and a pressure of 1.6 106 Pa. Heat moved with a constant speed v and is then suddenly
104 J is supplied to the gas. Given stopped. The process results in a rise of temperature
–1 –1
R = 8.3 J mol K . of the gas by 1 °C. Then, if M = molecular mass of
n=5 oxygen.
Cp = 20.75 J mol–1 K–1 5 7
Cp/Cv) = =
3 5
106 Pa
R 2R
v= v=
14. For an ideal gas M 1 M 1
-
sure when the temperature of n moles of the
17. Cv and Cp
gas changes by T is n Cv T.
of a gas at constant volume and constant pressure,
respectively. Then
adiabatic process is equal in magnitude to the
Cp – Cv is larger for a diatomic ideal gas than
work done by the gas.
for a monoatomic ideal gas
Cp + Cv is larger for a diatomic ideal gas than
isothermal process.
for a monoatomic ideal gas
Cp/Cv is larger for a diatomic ideal gas than
process.
for a monoatomic ideal gas
Cp Cv is larger for a diatomic ideal gas than
15. P, for a monoatomic ideal gas
volume V) to state P/2, Volume 2V)
along a straight line in the P-V diagram as shown
in Fig. 17.32. Then 18. Figure 17.33 shows the P-V plot of an ideal gas
taken through a cycle . The part is a
semi-circle and is half of an ellipse. Then,

Fig. 17.33

is isother-
Fig. 17.32
mal
to
exceeds the work that would be done by it C
if the system were taken from to along C is
the isotherm. zero
T-V diagram, the path becomes a
part of a parabola.
P-T diagram, the path becomes a
part of a hyperbola.
17.24 Comprehensive Physics—JEE Advanced

ANSWERS AND SOLUTIONS


1. For adiabatic process, PV = constant. Differentiating Since = 1.4, the gas is diatomic. For a diatomic
w.r.t V we get gas, the number of degrees of freedom of the mol-
–1
ecules = 5.
V +P V =0 We know that the work done by the gas during adia-
P batic expansion is given by
or =
V 1
W= PV – P V
For isothermal process, PV = constant. Hence 1
P where pressure P after expansion is obtained from
= the relation
V
Now, / P-V) graph. Thus, PV PV
=
P-V) graph for an adiabatic pro- T T
cess is times that for an isothermal process. or P =P
V T
Hence curves and both represent adiabatic V T
process and curves and both represent VT /2 P
isothermal process. Thus the correct choices are =P =
5.66V
T 11.32
P
2. In a cyclic process, the system returns to its initial Putting = 1.4, V = 5.66 V and P = in
state. Hence the change in internal energy = 0. 11.32
We have
of thermodynamics, 1 P
W= PV 5.66V
= + = = 0) 1 4 1) 11.32
1 1
= PV PV = 1.25 PV
3. W = P V and U = nCv T and = Cp/Cv 0.4 2
Q = U + W.
For an isothermal process, T = 0. Therefore U 6. Since W2 and W3 are negative, it means that the
= 0. Hence Q = W work is done on the gas. Hence Q2 and Q3 are nega-
adiabatic process Q = 0. So U = – W, which is tive which implies that heat is evolved in process-
W = 0, so Q es 2 and 3. Since Q1 and Q4 are positive, heat is
= U
Q= U+ W Q1 + Q4 Q2 + Q3), the gas absorbs
4. a net amount of heat energy in a complete cycle,
5. For an adiabatic change the relation between T which is given by
and V is Q = Q1 + Q2 + Q3 + Q4
Cp
TV – 1) =
Cv
1
– 1) – 1)V T The net work done by the gas is
TV =T V or =
V T W = W1 + W2 + W3 + W4
T = 2200 – 825 – 1100 + W4
Given V = 5.66 V and T = . Therefore, W4
2
– 1)
=2 Since the process is cyclic, the change in internal
energy U -
Taking logarithm of both sides, we have
namics,
we have
l 2) 0.3010 W= Q – U = Q
or =1+ =1+ or 275 + W4 = 1040 or W4 = 1040 – 275
l 5.66) 0.7528
= 765 J
= 1 + 0.4 = 1.4
17.25

Q = 800 0.25 = 200 J. So


net work done by the gas
h=
total heat absorbed by the gas 9. Q2 = mL = 103
80 = 80,000 J = 80,000 4.2 = 3.36 105 J. So
W 275 765
= =
Q1 Q4 3645
T2 273
1040 = =
= = 0.1083 = 10.83% T1 T2 300 273
is correct.
11%
Q 3.36 105
W= 2 = 3.36 104 J, which
10
7. The internal energy of n moles of an ideal gas at
absolute temperature T is given by
U = nCvT Q1 = Q2 + W = 3.36 105 + 3.36 104
37 104 J
where Cv -
ume. We know that Q 37 104 J
Power = 1 = 20 kW. So choice
180 s
Cp R
Cp – Cv = R or –1=
Cv Cv 1
10. W = +
R R 2
or – 1 = or Cv = 1
Cv 1 = 10–3 m3) 4 Nm–2 + 4 Nm–2 6 10–3m3
2
Now, the ideal gas equation for n nodes is = 0.012 + 0.024 = 0.036 J
PV W C = – 0.024 J. The negative
PV = nRT or n = sign shows that the work is done on the gas.
RT
WC = P V = 0 because V = 0.
PV R PV Work done in complete cycle = 0.036 – 0.024 + 0
U= T= = 0.012 J
RT 1 1

8. T2 = 0.25. 11. For point , PV = 5 10–3) = 10 10–3 J


For point =2 10–3) = 10 10–3 J
T2 300
=1– 0.25 = 1 – which gives Since PV = constant, the process is isothermal. For
T1 T1
an isothermal process, U = 0. Since the gas un-
T1 dergoes expansion W is positive and Q = W.
= 0.25 + 0.05 = 0.30. The
12. The P–V graph for an adiabatic process is not a
new temperature of the source should beT 1 so
P V
that
T 300 = n R T and P V = n R T . Therefore
0.30 = 1 – 2 = 1 –
T1 T1 PV T 6 1 T 3
= = ,
which gives T = 428.6 K. So, increase in tem- PV T 2 4 T 4
perature of the source = T1 – T1 = 428.6 – 400 i.e. T > T . Hence the internal energy increases.
Work done = area under upto the volume axis.
T2 of the sink should be Heat energy is absorbed in the process. Hence the
such that
T2 T PV
0.30 = 1 – =1– 2 13. PV = nRT n=
T1 400 RT
which gives T2 = 280 K = 7°C. Decrease in 1 6 106 ) 0.0083 16
temperature of the sink is 27°C – 7°C = 20°C, = =
8.3 300 3
17.26 Comprehensive Physics—JEE Advanced

5R 5 P0 and V0 be the intercepts on the P and


Cp = = 8.3 V axes. The equation of straight line is
2 2
= 20.75 J mol–1 K–1 P0
P= V – V 0)
3R V0
Q = n Cv T Cv =
2 P V
Q 2 Q 2 2. 104 P0 V0
T= = =
nCv 3nR 16 RT
3 8.3 Since P =
3 V
= 375 K RT V P V P0 V 2
=1 T= 0
Final temperature = 300 + 375 = 675 K V P0 V0 R RV0
= 402°C. which represents a parabola on the T-V graph.
P2 P P1 T2 1.6 106 675
= 1 P2 = =
T2 T1 T1 300 RT
6
V=
= 3.6 10 Pa P
P RT V0 P 2
=1 T = V0 P –
14. U = n Cv T P0 PV0 R P0
Q = U + W. For an adiabatic process,
which does not represent a hyperbola. So
Q = 0. Hence 0 = U + W or | U| = | W|.
T = 0, Hence
U = 0.
Q = 0. is correct.
16.
15. to Therefore, Cv = 5 R/2 and Cp = 7 R/2. So =
Cp/Cv = 7/5. The kinetic energy of oxygen mol-
W1 = area of trapezium
1
P 3 PV ecules with a velocity v0 = M v2, where M =
= P V= 2
2 2 molecular weight of oxygen.
3 RT Now heat energy = Cv = Cv 1 = Cv
=
2 Cp R
Cp – Cv = R or –1=
Cv Cv
R R
– 1) = or Cv =
Cv 1
1 R
Therefore, M v2 =
2 1

2R
or v=
M 1

7R 5R
17. Cp + Cv) for diatomic gas = =6R
2 2
Fig. 17.34 5R 3R
Cp + Cv) for monoatomic gas = =4R
If the process to were isothermal, the 2 2
work done would be 35 R 2
V2 Cp Cv for diatomic gas =
W2 = RT loge = RT loge RT 4
V1
15 R 2
Thus W1 > W2 Cp Cv for monoatomic gas =
4
17.27

Cp 2 PV V
= =1+ is smaller for diatomic gas than U= is negative because V < V .
Cv 1
for monoatomic gas. From Q = U + W Q is negative,
Cp – Cv = R. C .

18. C
process increases with V - the cyclic process is clockwise. Hence the correct
rect. For the process C , volume is decreas-
W

III

Multiple Choice Questions Based on Passage


Questions 1 to 4 are based on the following passage 2. Which of the graphs shown in Fig. 17.36 represents
Passage I the P – T diagram for the complete process?

7R
Three moles of an ideal gas Cp at pressure P
2
and temperature T are isothermally expanded to twice the
original volume. The gas is then compressed at constant
pressure to its original volume. Finally the gas is heated
at constant volume to its original pressure P .

1. Which of the graphs shown in Fig. 17.35 represents


the P – V diagram for the complete process?

Fig. 17.36

3. The net work done W by the gas during the


R is the gas constant)
RT RT
RT RT
4. The net heat energy supplied to the gas in the com-
plete process is
W
W W
Fig. 17.35

SOLUTION
1 2. During isothermal pro ), T = constant.
1. During isothermal process ,P . During
V C), P = constant and
C), P = constant and during C ), P T. Hence
C ), V = constant. Hence the the correct P – T diagram for the complete process
correct P – V
17.28 Comprehensive Physics—JEE Advanced

3. For process , P V = P V which gives P Work done in isobaric process C is


P
= because V = 2V W = P VC – V ) =
P
V – 2V )
2 2
V V
For process C, = C . Since V = 2 V and PV 3
T TC =– =– RT .
T 2 2
VC = V and T = T , we get TC = PC = P Work done is isochoric process C is W =0
2
=
P
. Total work done
2 3
P P P W = 3 RT loge RT
For process C , C = . Since PC = 2
TC T 2 = 3 RT RT
T = 2TC.
Thus th
Work done is isothermal process is
4. Since the process is cyclic, U = 0. From Q = U +
V
W = nRT loge = 3 RT loge W, we get Q = W
V

Questions 5 to 9 are based on the following passage 6. VC) of the gas in state C is
Passage II 8V
V
Two moles of an ideal gas at volume V, pressure 2 P and 3
temperature T undergo a cyclic process as shown
4V 2V
in Fig. 17.37.
3 3
7. V ) of the gas in state is
V V
V V
8. W) in the complete cycle is
RT RT loge
4 2
RT RT loge
Fig. 17.37 3 3
9. Q) in the heat energy in the com-
5. V ) of the gas in state is plete process is
V 2V W
3 3 W W.
4V
V
3
SOLUTION
5. For isobaric process , 7. For isobaric process C ,
V V V T V 4T / 3 4V VC V V T 8V / 3 T
= V = = = = V = = = 2V
T T T T 3 TC T TC 4T / 3

6. For isothermal process C, 4V 2 PV


8. W =P V –V )=2P V =
PV 2 P 4V / 3 8V 3 3
P V = PC VC VC = = = ,
PC P 3 VC
W = nRT loge
V
17.29

4T 8V / 3 8 Total work done is


=2 R log e = RT loge
3 4V / 3 3 W=W +W +W +W
8V 2 PV 2 PV 8 2 PV
W = P C V – V C) = P 2V =– = + RT loge – 2 RT loge
3 3 3 3 3

= nRT loge
V 2
W = RT loge
V 3
V 9. For a cyclic process, U = 0. Hence Q = W. So
=2 R T loge
2V
1
= 2 RT loge = – 2 RT loge
2

Questions 10 to 12 are based on the following passage 11. The adiabatic compressibility of the mixture at
Passage III pressure P is
V 3 5
5 5P 7P
consists of one gram mole of a gas with Cp/Cv) =
7 3
13 7
and another gas with = at a certain temperature T.
5 P P
The gram molecular weights of gases and are 4 and 12. If the temperature T of the mixture is raised from
32 respectively. The gases do not react with each other and 300 K to 301 K, the percentage change in the speed
are assumed to be ideal. The gaseous mixture follows the of sound in the gaseous mixture is
relation PV – = constant in an adiabatic process.
1 1
%
3 6
10. The number of moles of gas in the gaseous
mixture is 1
2

SOLUTION
10. Let n and n be the number of moles of gases 5 7
and C v) Cv) be their Given n = 1, = , = and m is given by
3 5
the adiabatic relation for the mixture,
Since the gases do not react with each other and are
assumed to be ideal, the internal molecular kinetic PV m
= constant
energy before and after the gases are mixed must
Given PV = constant. Hence = . Using
be the same. Hence m
13
n C v ) T + n C v) T n +n Cv )m T
Cv ) m - 1 n 1 n
+ =
ture at constant volume. Now 5 7
1 1 1
R R 3 5 13
Cv ) = Cv ) =
1 1
3 5n 13 1 n
R or + =
C v) m = 2 2 6
m 1 n = 13 + 13 n
Therefore, or 2 n = 4 or n = 2
n n n n Thus the mixture contains 2 moles of gas .
+ =
1 1 m 1
17.30 Comprehensive Physics—JEE Advanced

11. of the mixture is 1 13


= = ,
P P
1 V m
=– .
V P
12. We know that the speed of sound in a gas is propor-
We are given that
tional to the square root of absolute temperature, i.e.
PV m
= constant v = 1/2 = constant
1
where = . Partially differentiating, we have Taking logarithm, log v = log + log T
m
13 2
v 1 T
Differentiating, =0+
PV m
+ m PV m 1
V=0 v 2 T
V V m
V 1 1K 1
or =– =– = =
P PV m 1
m P 2 300 K 600
m
Percentage change
1 V 1 v 100 1
or – = 100 = = %
V P m P v 600 6

Questions 13 to 15 are based on the following passage


2
Passage IV 14. The change in the internal energy of the gas in this
Two moles of a monoatomic ideal gas occupy a volume process is
V at 27°C. The gas is expanded adiabatically to volume
2 2 V. Gas constant R = 8.3 JK–1 mol–1.
15. The work done by the gas during the process is
13.
150
2
SOLUTION
13. T1 = 300 K, V1 = V, V2 = 2 2 V. 14. U = nCv T2 – T1)
Let T2 T2 is 3R
For a monoatomic gas Cv = . Therefore,
obtained from the adiabatic relation 2
– 1) – 1) 3
T1 V1 = T2 V2
U=2 8.3
1 2
V1
or T2 = T1 The negative sign implies that the internal energy
V2
5 15. For a adiabatic process, Q = 0. Hence W = – U
For a monoatomic gas = . Therefore,
3
2/3
1
T2 = 300 = 150 K
2 2

Questions 16 to 21 are based on the following passage shown in Fig. 17.38. The molecular mass = 4 and R = 8.3
Passage V J K–1 mol–1.

16. The temperature of state is


is taken through the process and another sample of
2 kg of the same gas is taken through the process as
17.31

18. The temperature of state C is

19. The temperature of state is

20. The work done in process is


104 104 J
103 103 J
Fig. 17.38 21. The heat supplied in process is nearly equal
17. The temperature state is to
106 106 J
106 106 J

SOLUTION
16. Number of moles of helium is 19. For isobaric process
mass in gram 2000 T P 20
n= = = 500 = = =2
molecular mass 4 T P 10
From equation state at , which gives T = 2 T = 200 K. So the correct
PV
P V =nRT T = 20. Work done in process is
nR
4.15 104 10 W= W + W = 0 + P VC – V )
= = 100 K
500 8.3 = 8.3 104 103 J

17. For isochoric process 21. From U = n Cv T, heat energy in process


4 for a monoatomic gas Cv = 3 R/2)
T P 8.3 10
= = = 2.
T P 4.15 104 Q U) W)
Thus T = 2 T = n Cv TC – T ) + P V – V )
18. For isobaric process C, 3
= 500 83 400 100
TC V 20 2
= C = =2 + 4.15 104
T V 10
Thus TC = 2 T = 1.87 106
Questions 22 to 25 are based on the following passage 22. The work done by the gas is
Passage VI P 0V 0 P 0V 0
One mole of an ideal monoatomic gas is taken round the P 0V 0 P0V0
cyclic process 23.
C is
P0V0 3P0V0
2 2
5P0V0
P 0V 0
2
24. The heat energy absorbed by the gas in the process
is
3P0V0
P0 V0
2
Fig. 17.39
17.32 Comprehensive Physics—JEE Advanced

5P0V0 P0V0
P0 V0 P0 V0
2 2
25. 5P0V0
P0 V0
in the process C is 2

SOLUTION
22. Work done by the gas in the cyclic process is Using the ideal gas equation for points and , we
W = area enclosed in the P–V diagram have
1 P V = nRT or P0 V0 = nRT
= area of triangle =
2 and P V = nRT P0)V0 = nRT
1
= P 0 – P 0) V0 – V0) = P0V0, 2 P0 V0
2 which give T – T =
nR
23. In the isobaric process
by the gas is given by
Q = 2 Cv P0 V0
Q = nCp T = nCp T – TC R
Using ideal gas equation PV = nRT for points and 3R
C, we have Now, for a monoatomic gas, Cv = . Therefore
2
P V = nRT or P0 V0 = nRT Q = 3 P0 V0
and PC VC = nRTC or P0 V0) = nRTC
25. -
ergy absorbed by the gas is given by
P0V0 = nR TC – T ) Q= U+ W

or T – TC = –
P0 V0 where W = W = work done by the gas = P0V0.
nR Since the process is cyclic, there is no change in the
internal energy of the gas, i.e. U = 0. Hence
C p P0 V0 Q = W = P 0V 0
Q =–
R If Q is the heat absorbed in the process , then
5R Q =Q +Q +Q
Now, for a monoatomic gas Cp = . Hence
2
5 P0 V0
Q =– 5P0V0
2 P0V0 = 3P0V0 + Q –
2
24. The heat energy absorbed in the isochoric process
is given by or Q =
P0V0
Q = nCv T = nCv T – T 2

Questions 26 to 30 are based on the following passage 26. Which of the graphs shown in Fig. 17.40 represents
Passage VII the P–V diagram of the complete process?
Two moles of an ideal monoatomic gas, initially at 27. Pressure P2 is
pressure P1 = P and volume V1 = 2 2 V, undergo an 2P 2P
adiabatic compression until its volume is V2 = V and the 2 2P
pressure is P2. Then the gas is given heat energy Q at 28. The total work done by the gas is
constant volume V2.
PV 2 PV
2 PV PV
17.33

29. The change in internal energy due to adiabatic com-


pression is
PV 2 PV
2 PV PV
30. The temperature T2 of the gas after it is adiabati-
R is the gas constant).
PV 2PV
R R
2PV 2 2PV
R R

Fig. 17.40

SOLUTION
26. 2/3
3 P1 V1 V1
27. For an adiabatic change, P1V1 = P2 V2 . Therefore, =– 1
2 V2
V1
P2 = P1
V2 3P 2 2V 2/3
=– 2 2 1
For a monoatomic gas, = 5/3. Hence 2
5/3
2 2V 5/3 =–3 2
P2 = P = P 2 2 . So the correct The work done at contant volume = 0. Hence the
V
29. In adiabatic compression, Q = 0. Hence U = – W,
28. In an adiabatic process, heat Q = 0. Hence from the
30. Since U = nCv T = 2 Cv T2 – T1)
W1 = – U1 = – n Cv T U
we have T2 = T1 +
= – 2 Cv T2 – T1 n = 2) 2Cv
Now for 2 moles P1 V1 = 2 RT1 and P2V2 = 2 R T2.
P1 V1 3R
Thus Using T1 = and Cv = , we get
PV PV 2R 2
T1 = 1 1 and T2 = 2 2 2/3
2R 2R P1 V1 3 P1 V1 V1
T2 = + 1
Using these, we have 2R 2 2 3R / 2 V2
C
W = – v P 2 V 2 – P 1 V 1) P1 V1 V1
2/3
R =
C PV 2R V2
1 1
=– v V2 PV 1 1
R V2
P 2 2V 2 2PV
C 1 = =
=– v V1 R R
P 1V 1 1
R V2
17.34 Comprehensive Physics—JEE Advanced

Questions 31 to 35 are based on the following passage 31. Constant is given by


Passage VIII PT
1 1
P 1T 1
Two moles of an ideal mono-atomic gas is taken through 2
a cycle as shown in the P–T P 1T 1 2 P 1T 1
During the process , pressure and temperature of the 32. The work done in process R is the gas
gas vary such that PT = , where is a constant. constant)
IIT, 2000 RT1 RT1
RT1 RT1
33. The heat energy released in process is
RT1 RT1
RT1 RT1
34. The heat energy absorbed in process C is
RT1 RT1
RT1 RT1
35. The heat energy absorbed in process C is
Fig. 17.41 RT1 RT1
e e

e RT1 e RT1

SOLUTION
31. Using the ideal gas equation PV = nRT, the volumes V V 1/ 2
nR
of the gas in states , and C are W = =
V
n RT n R 2T1 2 nRT1 V V
V = = =
P P1 P1 V
= nR
n RT n R T1 1 n RT1 V
V = = = V
P 2 P1 2 P1
= 2 nR V V
nRTC n R 2T1 nRT1
and VC = = =
PC 2 P1 P1
It is given that in the process , the pressure 1 nRT1 2nRT1
W = 2 nR 2 PT
1 1
and temperature of the gas vary such that 2 P1 P1
PT = 1
where is a constant. Thus for point , we have = 2 2 nRT1 2 = – 2 nRT1
2
= P T = P1 T 1)
= – 4 RT1.
= 2 P 1T 1
The negative sign indicates that heat is absorbed in
32. For the process , we have
33. For a monoatomic gas Cv = 3R/2. The internal
PV = nRT energy of the gas in the process is
and PT = U) = n Cv T = n Cv T – T )
Eliminating T 3R
2
=2 T 1 – 2 T 1)
PV = nR or P V = 2
P = 3R T1) = –3RT1
1/ 2
nR
or P=
V Q U) +W
The work done in process is given by = – 3 RT1 – 4 RT1 = – 7 RT1 which
17.35

34. The process C takes place at a constant pressure 35. The process C takes place at a constant tem-
P = 2 P1. Therefore, the work done in this process perature T = 2 T1. Therefore, the work done in this
is P = nRT/V)
W =P V P1 VC – V ) V V
nRT
W = =
VC VC
V
nRT1 1 nRT1
W = 2P1 V
P1 2 P1 = nRT log e
VC
= nRT1 = 2 RT1 V
The change in the internal energy is process C is = 2. Therefore,
VC
U) = nCv T = nCv TC – T )
W =2 R 2T1 log e
= nCv T 1 – T 1)
= 4RT1loge T = 2 T 1)
3R
= nCv T1 = 2 T1 = 3RT1 U) = 0 as the temperature is constant.
2
Q U) +W Therefore
Q U) +W
= 3 RT1 + 2 RT1 = 5 RT1

Questions 36 to 38 are based on the following passage V V


Given = 2 and = 4. The temperature T = 27°C.
Passage IX V V
R is the gas constant.
cyclic process starting from as shown in Fig. 17.42. IIT, 2001
36. The temperature of the gas at point is

37. The total heat absorbed in the complete cycle is

38. The total work done by the gas in the complete


cycle is

Fig. 17.42

SOLUTION
36. For process , the plot of V versus T is linear. U = 0. Hence Q = W.
Hence VC
W C = nRT loge
V V V
=
T T VD
= nRT loge V D = V C)
V V
T = T =2 300 K = 600 K
V VD V
= nRT loge
37. Process occurs at constant pressure V T. V V
Hence 1
5R =2 R 600 loge 4
Q =2 T –T 2
2 = 1200 loge R
Process C occurs at constant temperature.
Q = U + W. Q C = 1200 loge R
17.36 Comprehensive Physics—JEE Advanced

Process C occurs at constant volume. Hence 1


QC D = nCv T – T ) =2 R 300 loge
4
3R = – 1200 loge R
=2
2 Total heat absorbed in the complete cycle is
R Q = 1500R + 1200 loge R R
Process occurs at constant temperature. – 1200 loge R = 600R
Hence 38. In a cyclic process U = 0. Hence W = Q =
V 600R
QD = WD = nRT loge
VD

Questions 39 to 41 are based on the following paragraph 40. When the gas bubble is at height from the bottom,
Passage X its temperature is
5 h
2/ 5
0
3 T0
0
is trapped inside a liquid of density l
2/ 5
0
with the liquid. The bubble contains n moles of gas. The T0
temperature of the gas when the bubble is at the bottom 0 H
is T0, the height of the liquid is H and the atmospheric 3/ 5
0 H
pressure is P0 T0
0
IIT, 2008
3/ 5
0
T0
0 H
41. The buoyancy force acting on the gas bubble is
R is the universal gas constant)
2/5
Fig. 17.43 0 H
l nR 0 7/5
39. - 0
ancy force the following forces are acting on it
0
2 /5 3/ 5
0 H P0
the pressure of the liquid 3 /5
0 H
0 8/ 5
pressure of the liquid and force due to viscos- 0
ity of the liquid
l 0
3 /5 2/5
to viscosity of the liquid 0 H P0

SOLUTION
41. F = weight of liquid displaced
39. = , V = volume of the bubble.
1
nRT
40. For an adiabatic process TP = constant. From PV = we have V = Therefore,
P
1 2 nR
For = 5/3, =– . Therefore , F=
5 P
2/5
2/5 2/5 nR 0
T0 0 H =T 0 = T0
0 0 h
2/5
0 nR
Which gives T T0 = 2/5
0 H
0 H P0 )]3 / 5
17.37

IV

Matching
1. P – V) graph of a cyclic process of an ideal gas is
shown in Fig. 17.44. Match the process listed in column I with the consequences
listed in Column II A
P
Column I Column II D
>0
C <0 B C
C >0
<0
V
IIT, 2006
Fig. 17.44
SOLUTION
In process , the volume remains constant. Hence = P T. Since the pressure decreases,
temperature T decreases. Hence the gas loses heat energy, i.e., < 0. In process C, the pressure remains
constant and volume increases. Now V T. Hence T increases which means that = is
positive. In process C , the volume remains constant but pressure increases. Hence > 0 and = 0. In
process , volume decreases and pressure increases. Hence < 0 and < 0.

2. Column I contains a list of processes involving expansion of an ideal gas. Match this with Column II describing
the thermodynamic change during this process.
Column I Column II

a valve. Chamber I contains an ideal gas and the

Fig. 17.45

1
its original volume such that its pressure P . or remains constant
where V is the volume of the gas. V2

1
its original volume such that its pressure P .
where V is its volume V 4/3

pressure P and volume V follow the behaviour shown


in the graph in Fig. 17.46.

Fig. 17.46 IIT, 2008


17.38 Comprehensive Physics—JEE Advanced

SOLUTION
= 0. Since the system is completely insulated, no heat can
enter or leave the sysytem, i.e. = – = 0, i.e.there is no

nRT 1
Given PV2 = constant. PV = n RT or P = n RT/V. Therefore, PV2 = V 2 = nRTV. Hence T . Thus if V
is increased, T will decrease, i.e. is negative. V V
nR.
For PV , the work done = . Hence
1
nR
= nCv +
1
nR
nCp = n Cv +
1
R
Cp = Cv +
1
3R
For monoatomic gas Cv = and for = 2
2
3R R 3R R
Cp = + = –R=
2 1 2 2 2
Now = nCp . Since is negative and Cp is positive, is negative, i.e. the gas loses heat. Hence the cor-

1
PV4/3 = constant, Then T 1/ 3
. Thus if V is increased, T will decrease, i.e. is negative.
V
Putting
R 3R 3R
Cp = Cv + = – 3R = – , which is negative. Now = nCp . Since Cp and are both negative,
1 4/3 2 2

-
ture of the gas increases. Therefore, is positive. Now
= + . Since both and are positive, will be positive, i.e. the gas gains heat. Hence the correct

3. One mole of a monatomic gas is taken through a cycle as shown in the P-V
II gives the characteristics involved in the cycle. Match them with each of the processes given in Column I.
IIT, 2011

Fig. 17.47

Column I Column II

C
17.39

SOLUTION
is isobaric. Hence V T. Therefore T > T . U = nCv T = nCv T – T ). Since T < T ,
U is negative, i.e. internal energy decreases.
Q = nCp T is also negative. Hence heat is lost.
W = 3P V – V ) = – 6PV. Which is negative. Hence work is done on the gas.

C is isochoric. Hence P T. Therefore T > TC. U = nCv T = nCv TC – T ) is negative, i.e.


internal energy decreases.
W=P V V = 0)
Q= U + W), Q= U. Since U is negative, heat is lost.

C is isobaric, i.e. V T. Hence TD > TC U = nCv TD – TC) is positive. Hence internal energy
increases.
Q = nCp TD – TC) is positive. Hence heat is gained by the gas.
W = P V = P V – V) = 8 PV which is positive. So work is done by the gas.

U = 0. Therefore Q = W. Since the


gas is compressed, work is done on the gas, i.e. W is negative. Hence Q is negative. Hence heat is lost
by the gas.

ANSWER

Assertion-Reason Type Questions

following four choices out of which only one choice is


correct.
-
ment-2 is the correct explanation for Statement-1.
-
ment-2 is the correct explanation for State-
ment-1.
Fig. 17.48
Statement-2

1. Statement-1 an ideal gas.


PV
Figure 17.48 shows versus P graph for a cer-
T 2. Statement-1
tain mass of oxygen gas at two temperatures T1 and If two bodies of equal mass and made of the same
T2. It follows from the graph that T1 > T2. material at different temperature T1 and T2 are
17.40 Comprehensive Physics—JEE Advanced

brought in thermal contact, the temperature of each Statement-2


T1 + T2)/2 when thermal equilibrium The latent heat of steam is high.
is attained. 7. Statement-1
Statement-2 V-T graphs of a certain
They have the same thermal capacity. mass of an ideal gas at two pressures P1 and P2. It
3. Statement-1 follows from the graphs that P1 is greater than P2.
Two vessels and of equal capacity are con-
-
tains a gas at 0°C and 1 atmosphere pressure and
vessel is evacuated. If the stopcock is suddenly
and will be 0.5
atmosphere.
Statement-2
If the temperature is kept constant, the pressure of
a gas is inversely proportional to its volume.
4. Statement-1 Fig. 17.49
Two vessels and are connected to each other
by a stopcock. Vessel contains a gas at 0°C and 1 Statement-2
atmosphere pressure and vessel is evaluated. The The slope of V-T graph for an ideal gas is directly
two vessels are thermally insulated from the sur- proportional to pressure.
roundings. If the stopcock is suddenly opened, there
will be no change in the internal energy of the gas. 8. Statement-1
Statement-2 The curves and in Fig. 17.50 show P-V graphs
No transfer of heat energy takes place between the for an isothermal and an adiabatic process for an
system and the surroundings. ideal gas. The isothermal process is represented by
5. Statement-1 curve .
Two vessels and are connected to each other by
a stopcock. Vessel contains a gas at 300 K and
1 atmosphere pressure and vessel is evacuated.
The two vessels are thermally insulated from the
surroundings. If the stopcock is suddenly opened,
the expanding gas does no work.
Statement-2
Since Q = 0 and U Fig. 17.50
law of thermodynamics that W = 0.
6. Statement-1 Statement-2
Heating system based on circulation of steam are The slope of the P-V graph is less for an isothermal
process than for an adiabatic process.
or circulation of hot water.

SOLUTION
mass and are made of the same material, they have
1. is parallel to the same thermal capacity.
the P-axis. This means that PV/T is a constant, in- 3.
dependent of pressure. Hence line corresponds of equal capacity, the volume occupied by the gas
to an ideal gas for which PV/T is doubled when the stopclock is opened. Hence,
temperatures, a real gas behaves more like an ideal pressure becomes half.
gas. Hence T1 is greater than T2. 4.
2. 5. -
if the two bodies have the same thermal capacity mally insulated from the surroundings, no heat
which is equal to mass of the body Q = 0. Since
heat capacity. Since the two bodies have the same U W = 0.
17.41

6.
7. From PV = RT, we have an adiabatic process PV = constant. Differenting,
PV we get
T
R) P V –1 + V =0
P
V R V 1 which gives =– = Cp/Cv
= , i.e. V
T P T P
Since > 1, the slope of the P-V curve for an adia-
Hence Statement-1 is true but Statement-2 is false. batic process is greater than that for an isothermal
8. For an isothermal process PV = constant. Differen- process. Thus both the statements are true and state-
P ment-2 is the correct explanation for statement-1.
tiating we get + =0 = – . For
V

VI

Integer Answer Type


1. Two soap bubbles and are kept in a closed n /n , where n and n are the number of moles of
chamber where the air is maintained at pressure air in bubbles and , respectively. [Neglect the
N/m2. The radii of bubbles and are 2 cm and 4 effect of gravity.]
cm, respectively. Surface tension of the soap-water
used to make bubbles is 0.04 N/m. Find the ratio
SOLUTION
1. If P0 is the external pressure, then Using P V = n RT and P V = n RT, we have
4 4 0.04 3
4
P = P0 + =8+ = 16 N m–2 n
=
P V P r
V= r 3)
2
r 2 10 n P V P r 3
4 4 0.04 3
P = P0 + =8+ = 12 N m–2 12 4
r 4 10 2 = =6
16 2
18
Chapter
Kinetic Theory of Gases

REVIEW OF BASIC CONCEPTS 18.3 MEAN TRANSLATIONAL KINETIC ENERGY


18.1 PRESSURE EXERTED BY AN IDEAL GAS The mean translational kinetic energy of a molecule of a
gas is given by
The pressure of a gas in a container is a result of the
1
continuous bombardment of the gas molecules against the E= mv2rms
walls of the container and is given by 2
2 2 In terms of E, the pressure of the gas is given by
1 m n vr ms 1 M vr ms 1 2 2U
P= vr ms 1 2 2 nE 2 U
3 V 3 V 3 3V P= v rms =
3 3 V 3V
where m = mass of each molecule, n = number of
molecules in the container, vrms = root mean square speed where U = nE is the total translational kinetic energy of
of molecules, V = volume of container, M = mass of gas all the n molecules of the gas. It is also called the internal
in the container, = density of the gas and U = internal energy of the gas.
R
energy of the gas. Boltzman’s constant k =
N0
18.2 ROOT MEAN SQUARE SPEED where N0 is Avagdro number.

1/ 2 18.4 EQUATION OF STATE OF AN IDEAL GAS


1 2
vrms = ( v1 v22 v32 vn2 )
n The relationship between pressure P, volume V and
absolute temperature T of an ideal gas is called the equation
where v1, v2, v3, vn are the speeds of the molecules 1, 2, of state. For n moles of a gas, this relation is
3, n respectively. In terms of P and , vrms is given by PV = nRT
3P 3k T 3 RT where R is the molar gas constant.
vrms = From Avogadro’s law, it follows that one mole of all
m M
gases, at the same temperature and pressure, occupies
where m is the mass of each molecule, T the absolute
temperature of the gas and k a constant called Boltzman’s gas occupies 22.4 litres at STP. Consequently, for on mole
constant. Its value is PV
the ratio is constant for all gases. This constant is the
k = 1.38 10–23 J K–1 per molecule T
molar gas constant and can be evaluated as follows:
R is the universal gas constant and its value is
At STP, V = 22.4 litre = 22.4 10–3 m3
R = 8.315 J K–1 mol–1
18.2 Comprehensive Physics—JEE Advanced

P = 0.76 m Hg = 1.013 105 Nm–2 f f


T = 273 K and Cp = Cv + R = R+R= 1 R
2 2
PV 1.013 105 22.4 10 3
f
R= 1 R
T 273 Cp 2 2
= 1
= 8.315 J mol–1 K–1 Cv f
R
f
2
18.5 VAN DER WAAL’S EQUATION OF STATE 2 5 2
Thus =1 + = for a monoatomic gas = 1 +
According to Van der Waal the true pressure exerted by a gas 3 3 5
is greater than P by an amount a/V 2 (where a is a constant) 7 2 4
= for a diatomic gas = 1 + = for a triatomic
due to attractive forces between molecules and the true 5 6 3
volume of the gas is less than V by an amount b (where b is or polyatomic gas
another constant) because molecules themselves occupy a
Relation between Cp, Cv and R
3R 5R
a For a monoatomic gas; Cv = and Cp =
P (V – b) = RT 2 2
V2
At high pressures, when the molecules are too many 5R 7R
For a diatomic gas; Cv = and Cp =
and too close together, the correction factors a and b both 2 2
become important. But at low pressures, when they are not
For a polyatomic gas; Cv = 3R and Cp = 4R
too many and not too close together, a gas behaves like an
ideal gas and obeys the equation PV = RT. Relation between Cp, Cv and
Cp – Cv = R
DEGREES OF FREEDOM AND
18.6 EQUIPARTITION OF ENERGY
Cp
= Cp = Cv
Cv
The total number of coordinates or independent quantities R
- Cv – Cv = R Cv =
1
tion of a dynamical system is called the degrees of free-
dom of the system. R R
The molecules of a monoatomic gas consist of single Cp = Cv + R = +R=
1 1
atoms. Therefore, the molecules of a monoatomic gas
have three degrees of freedom corresponding to trans-
lational motion. The molecules of a diatomic gas have 18.1
degrees of freedom-three corresponding to transla- Calculate the root mean square speed of the mole-
tional motion and two for rotational motion. A polyatomic cules of hydrogen gas at S.T.P. Density of hydrogen
molecule has six degrees of freedom including one of at S.T.P. is 9 10–2 kg m–3.
vibrational motion.
The law of equipartition of energy is stated as follows. SOLUTION
In any dynamical system with a uniform absolute
At S.T.P., pressure P = 1.01 105 Pa and density
temperature T, the total energy is distributed equally
= 9 10–2 kg m–3.
among all the degrees of freedom and the average energy
1 3P
per degree of freedom per molecule equals kT, where vrms =
–23
k = 1.38 10 J K . –1 2
If the molecules of a gas have f degrees of freedom,
1 3 1.01 105
then kinetic energy per molecule = f kT. Therefore, = 2
= 1840 ms–1
2 9 10
kinetic energy per mole is
1 f 18.2
U = Nf kT = RT
2 2 Calculate the temperature (in kelvin) at which the
Now Cv =
U
=
f
R root mean square speed of a gas molecule is half its
T 2 value at 0°C.
Kinetic Theory of Gases 18.3

SOLUTION SOLUTION
3kT (a) Average translational K.E. of a molecule of an
vrms = ideal gas is
m
3
v rms T T E = kT, where T = temperature
= 2 in kelvin
vrms T 273
1 T At T = 0°C = 273 K,
= T = 68.25 K 3
2 273 E= (1.38 10–23) 273
2
= 5.65 10–21 J
18.3
Find the mean translational kinetic energy of an oxy- At T = 100°C = 373 K,
gen molecule at 0°C. Given Avogadro number N = 3
6.03 1023 per mole and R = 8.3 JK–1 mol–1. E= (1.38 10–23) 373
2
= 7.72 10–21 J
SOLUTION
(b) Number of molecules in 1 mole of a gas is
1 3 3 RT N = 6.02 1023
E = mv2rms = kT =
2 2 2 N
K.E. of 1 mole at 273 K = (5.65 10–21)
3 8.3 273 (6.02 1023) = 3.40 J
E= = 5.64 10–21 J
2 6.03 1023 K.E. of 1 mole at 373 K = (7.72 10–21)
(6.02 1023) = 4.65 J

18.4 18.6
Calculate the mean translational kinetic energy of 1 The speed of sound in a gas at S.T.P. is 330 ms–1 and the
mole of hydrogen at S.T.P. Density of hydrogen at density of the gas is 1.3 kg m–3. Find the number of
S.T.P. is 0.09 kg m–3. degrees of freedom of a molecule of the gas.

SOLUTION SOLUTION
3P 3 1.01 105 P
vrms = = v =
0.09
= 1.84 103 ms–1
v2 (330) 2 1.3
Mass of 1 mole is = = 1.4
–3 3 –3 P 1.01 105
m = 22.4 10 m 0.09 kg m
If f is the number of degrees of freedom, then
= 2.016 10–3 kg
2 2
1 f = =5
K.E. = mv2rms 1 (1.4 1)
2
1
= (2.016 10–3) (1.84 103)2 18.7
2 The volume of 2 moles of a diatomic ideal gas at 300
= 3.41 103 J K is doubled keeping its pressure constant. Find the
change in the internal energy of the gas. Given R =
18.5 8.3 JK–1 mol–1.
Calculate (a) the average translational kinetic energy
SOLUTION
of the molecules of an ideal gas at 0°C and at 100°C
and (b) the energy per mole of the gas at 0°C and According to kinetic theory, there are no internal
100°C. Given Avogadro’s number N = 6.02 1023 forces of interaction between the molecules of an
and Boltzmann’s constant k = 1.38 10–23 JK–1. ideal gas. This implies that the potential energy is
18.4 Comprehensive Physics—JEE Advanced

zero. Hence, for an ideal gas, the internal energy is 5


only due to kinetic energy of the molecules. = 1.38 10–23 (273 + 27)
2
For n moles of an ideal gas at absolute temperature T, = 1.035 10–20 J
the internal energy is
5 5
f (c) Total K.E. = PV = 106 10–2 = 2.5 104 J
U= nRT 2 2
2
where f is the number of degrees of freedom. For a
diatomic gas f = 5. Therefore 18.9
5 n1 moles of a monoatomic gas are contained in a ves-
U = nRT sel A of volume V1 at pressure P1 and temperature T1.
2
Since pressure is kept constant, V T (Charles’ law), n2 moles of the same gas are contained in a vessel
i.e. B of volume V2 at pressure P2 and temperature T2.
The two vessels are now connected by a tube. Obtain
V1 V
= 2 the expression for (a) common temperature T and (b)
T1 T2 common pressure P in the Vessels.
V2
T2 = T1 = 2T1 = 600 K ( V2 = V1) SOLUTION
V1
If f = number of degrees of freedom,
5
U = U2 – U1 = nR(T2 – T1)
2
5 f f f
= 2 8.3 (600 – 300) n1RT1 + n2RT2 = (n1 + n2)RT
2 2 2 2
= 1.245 104 J Here f = 3 (monoatomic gas)
Equation (1) gives
Alternative Method
n T n2T2
U = nCv T T= 11
n1 n2
5R Final pressure is
For diatomic gas, Cv = . Therefore
2 (n n2 ) RT PV P2V2
P= 1 1 1
5R V1 V2 V1 V2
U= 2 (600 – 300)
2
18.7 IDEAL GAS LAWS
= 1.245 104 J
Boyle's Law
18.8
At any given temperature, the volume of a given mass of
Hydrogen gas is contained in a vessel of volume 10 a gas is inversely proportional to pressure, i.e.
litres at 27°C. The gas pressure is 106 Pa. Find
1
(a) total translational kinetic energy of hydrogen V (T = constant and n = constant)
P
molecules, PV = constant
(b) mean (average) kinetic energy of the molecules P 1V 1 = P 2V 2
and
(c) total kinetic energy of the molecules. Charle's Law
If the pressure of a gas is kept constant, the volume of
SOLUTION a given mass of the gas is directly proportional to its
Volume of gas = 10 litres = 10 10–3 m3 = 10–2 m3) absolute temperature, i.e.
3 3 V T (P = constant and n = constant)
(a) Total K.E. of molecules = PV = 106
2 2 V
10–2 = 1.5 104 J = constant
T
(b) Average K.E. of molecules
5 V1 V
= 2
= kT ( Hydrogen is diatomic)
2 T1 T2
Kinetic Theory of Gases 18.5

If the volume of a gas is kept constant, the pressure


18.11
of a given mass of the gas is directly proportional to its
absolute temperature, i.e. A cylinder of volume 30 litres contains oxygen at a
P gauge pressure of 15 atm at 27°C. When some oxygen
= constant (V = constant and n = constant) is ejected out from the cylinder, the gauge pressure
T
falls to 11 atm and temperature falls to 17°C. Find
P1
= 2
P the mass of oxygen ejected. R = 8.3 J K–1 mol–1 and
T1 T2 molecular mass of oxygen = 32.
Avogadro Law
SOLUTION
Equal volumes of all gasses at the same temperature
Let N1 be the number of molecules, P1 be the pres-
and pressure contain an equal number of molecules. The
sure and T1 be the temperature of oxygen before some
number of molecules in one mole of any gas is N0 = 6.02
oxygen is ejected and N2, P2 and T2 be their values
1023. N0 is called the Avogadro number. If volume V1 of
after the gas is ejected. Since the volume V remains
one gas contains N1 molecules and volume V2 of another
unchanged (= volume of cylinder),
gas contains N2 molecules at the same temperature and
pressure, then P1V = N1kT1 (1)

V1 V P2V = N2kT2 (2)


= 2
N1 N2 R 8.3
where k = = 1.38 10–23 JK–1 per
Equation of State in terms of Boltzmann constant (k) N0 6.02 1023
universal gas constant R molecules is Boltzmann constant. Given, V = 30 litres
k= = 30 10–3 m3, P1 = 15 atm = 15 1.01 105 Pa, T1
Avogadro number N0
= 273 + 27 = 300 k, P2 = 11 atm = 11 1.01 105 Pa
R
PV = nRT = nN0 T = nN0kT and T2 = 273 + 17 = 290 K. Substituting these values
N0 in Eqs. (1) and (2), we get
or PV = NkT
N1 = 10.87 1024 and N2 = 8.25 1024
where N = nN0 = number of molecules Number of molecules of oxygen ejected = N1 – N2
= 2.62 1024. The molecular mass of oxygen = 32,
18.10 i.e. one mole of oxygen (which contains 6.02 1023
An electric bulb of volume 250 cm3 was sealed dur- molecules) has a mass of 32 g = 0.032 kg. Hence 2.62
ing manufacture at a pressure of 10–3 mm of Hg at 1024 molecules will have a mass of
27°C. Find the number of air molecules in the bulb. 0.032 2.62 1024
= 0.14 kg
SOLUTION 6.02 1023
Let N be the number of air molecules in the bulb. It
is given that P = 10–3 mm of Hg = 10–4 cm of Hg, V 18.12
= 250 cm3 and T = 273 + 27 = 300 K. A narrow glass tube of length 100 cm is closed at
Now PV = NkT (1) both ends. It lies horizontally with 20 cm of mercury
column in the middle dividing the tube into two com-
At STP, one mole of a gas occupies a volume V0 = partments I and II of equal length. The air in each
22400 cm3 and contains N0 = 6.02 1023 molecules, compartment is at standard temperature and pressure.
P0 = 76 cm of Hg and T0 = 273 K. The tube is then turned to a vertical position. By what
P0V0 = N0kT0 (2) distance will the mercury column be displaced?
Dividing (1) by (2), we get
SOLUTION
N = N0
T0 P V Let A cm2 be the cross-sectional area of the tube.
T P0 V0 Then P0 = 76 cm of Hg and V0 = 40 A cm3 [Fig.
18.1(a)]. When the tube is turned to vertical posi-
273 10 4 250 tion [Fig. 18.1(b)], Let x cm be the displacement of
= (6.02 1023)
300 76 22400 mercury column and let P1, V1 be the air pressure in
= 8.04 1015 molecules compartment I and P2, V2 those in compartment II.
18.6 Comprehensive Physics—JEE Advanced

P0V0 P 40 A 40 P0
P1 = = 0
V1 (40 x) A (40 x)
P0V0 P 40 A 40 P0
and P2 = = 0
V2 (40 x) A (40 x)
The mercury column will be in equilibrium if P2 (in
cm of Hg) + 10 cm of Hg = P1(in cm of Hg), i.e. if
40 P0 40 P0
+ 10 =
40 x (40 x)
where P0 = 76 cm of Hg. Thus
40 76 40 76
Fig. 18.1
+ 10 =
(40 x) (40 x)
Using Boyle’s Law, we have x2 + 608x – 1600 = 0
P0V0 = P1V1 and P0V0 = P2V2 Solving for positive root, we get x = 2.6 cm

Multiple Choice Questions with Only One Choice Correct


1. In an adiabatic process, the root mean square speed 5. A vessel contains 4 moles of oxygen and 2 moles of
of the molecules of a monoatomic gas becomes argon at absolute temperature T. The total internal
twice its initial value. The ratio of the initial volume energy of the gas mixture is
(a) 6 RT (b) 9 RT
(a) 2 (b) 23/2 (c) 11 RT (d) 13 RT
(c) 4 (d) 8 6. The average translational kinetic energy of a mol-
2. The root mean square speed of hydrogen molecules ecule of a gas at absolute temperature T is propor-
at a certain temperature is v. If the temperature tional to
is doubled and the hydrogen gas dissociates into 1
(a) (b) T
atomic hydrogen, the rms speed will become T
v v (c) T (d) T 2
(a) (b) 7. The root mean square speed of the molecules of
4 2
an enclosed gas is v. What will be the root mean
(c) 2v (d) 4v square speed if the pressure is doubled, the tem-
3. A gas in a closed container has temperature T and perature remaining the same?
pressure P. If the molecules of the gas undergo (a) v/2 (b) v
inelastic collisions with the walls of the container, (c) 2 v (d) 4 v
then 8. The mass of an oxygen molecule is about 16 times
(a) both P and T will decrease that of a hydrogen molecule. At room temperature
(b) P decreases and T increases the rms speed of oxygen molecules is v. The rms
(c) P increases and T decreases speed of the hydrogen molecule at the same tem-
(d) both P and T remain the same perature will be
4. Two moles of hydrogen are mixed with n moles (a) v/16 (b) v/4
of helium. The root mean square speed of the gas (c) 4 v (d) 16 v
molecules in the mixture is 2 times the speed of 9. The average kinetic energy of hydrogen molecules
sound in the mixture. The value of n is at 300 K is E. At the same temperature, the average
(a) 1 (b) 2 kinetic energy of oxygen molecules will be
(c) 3 (d) 4
Kinetic Theory of Gases 18.7

(a) E/16 (b) E/4 the rotational kinetic energy per O2 molecule to that
(c) E (d) 4E per N2 molecule is
10. At room temperature (27°C) the rms speed of the (a) 1 : 1
molecules of a certain diatomic gas is found to be (b) 1 : 2
1920 ms–1. The gas is (c) 2 : 1
(a) H2 (b) F2 (d) depends on the moment of inertia of the two
(c) O2 (d) Cl2 molecules.
IIT, 1984 IIT, 1998
11. The temperature of an ideal gas is increased from 16.
120 K to 480 K. If at 120 K, the room mean square with air at temperature (T1, T2), volume (V1, V2) and
speed of the gas molecules is v, then at 480 K it will be pressure (P1, P2) respectively. If the valve joining
(a) 4 v (b) 2 v the two vessels is opened, the temperature inside
v v the vessel at equilibrium will be
(c) (d)
2 4 (a) T1 + T2
IIT, 1996 (b) (T1 + T2)/2
12. Three closed vessels A, B and C are at the same (c) T1T2 (P1V1 + P2V2)/(P1V1T2 + P2V2 T1)
temperature. Vessel A contains only O2, B only N2 (d) T1T2 (P1V1 + P2V2)/(P1V1T1 + P2V2T2)
and C a mixture of equal quantities of O2 and N2. 17. If the temperature of a gas is increased from
If the average speed of O2 molecules in vessel A is 27°C to 927°C, the root mean square speed of its
v1, that of N2 molecules in vessel B is v2, the molecules
average speed of O2 molecules is vessel C is (a) becomes half
1 (b) is doubled
(a) (v1 + v2) (b) v1
2 (c) becomes 4 times
3k T (d) remains unchanged
(c) v1 v2 (d)
M 18. At what temperature will oxygen molecules have
the same root mean square speed as hydrogen mol-
IIT, 1992
ecules at 200 K?
13. The average translational energy and the rms speed
of molecules of a sample of oxygen gas at 300 K (a) 527°C (b) 1327°C
are 6.21 10–21 J and 484 ms–1 respectively. The (c) 2127°C (d) 2927°C
corresponding values at 600 K are nearly (assum- 19. An enclosure of volume V contains a mixture of
ing ideal gas behaviour) 8 g of oxygen, 14 g of nitrogen and 22 g of car-
(a) 12.42 10–21 J, 968 ms–1 bon dioxide at absolute temperature T. The pres-
sure of the mixture of gases is (R is universal gas
(b) 8.78 10–21 J, 684 ms–1
constant)
(c) 6.21 10–21 J, 968 ms–1 3RT
RT
(d) 12.42 10–21 J, 684 ms–1 (a) (b)
V 2V
IIT, 1997 5RT 7 RT
(c) (d)
14. A vessel contains 1 mole of O2 gas (molar mass 32) 4V 5V
at a temperature T. The pressure of the gas is 20. At what absolute temperature T is the root mean
P. An identical vessel containing one mole of square speed of a hydrogen molecule equal to its
He gas (molar mass 4) at a temperature 2T has a escape velocity from the surface of the moon? The
pressure of radius of moon is R, g is the acceleration due to
(a)
P
(b) P gravity on moon’s surface, m is the mass of a hy-
8 drogen molecule and k is the Boltzmann constant
(c) 2P (d) 8P mgR 2mgR
(a) (b)
IIT, 1997 2k k
15. A vessel contains a mixture of 1 mole of oxygen 3mgR 2mgR
(c) (d)
and two moles of nitrogen at 300 K. The ratio of 2k 3k
18.8 Comprehensive Physics—JEE Advanced

21. Two perfect gases at absolute temperatures T1 and 23. 0.014 kg of nitrogen is enclosed in a vessel at a
T2 are mixed. There is no loss of energy in this temperature of 27 °C. The amount of heat energy
process. If n1 and n2 are the respective number of to be supplied to the gas to double the rms speed of
molecules of the gases, the temperature of the mix- its molecules is approximately equal to
ture will be (a) 6350 J (b) 7350 J
n1T1 n2T2 n2T1 n1T2 (c) 8350 J (d) 9350 J
(a) (b) 24. A vessel of volume V contains an ideal gas at
n1 n2 n1 n2
absolute temperature T and pressure P. The gas is
(c) T1 +
n2
T (d) T2 +
n1
T allowed to leak till its pressure falls to P . Assum-
n1 2 n2 1 ing that the temperature remains constant during
leakage, the number of moles of the gas that have
22. An insulated box containing a diatomic gas of mo- leaked is
lar mass M is moving with a velocity v. The box is
V V
suddenly stopped. The resulting change in tempera- (a) (P + P ) (b) (P + P )
ture is (R is the gas constant) RT 2 RT
M v2 M v2 (c)
V
(P – P ) (d)
V
(P – P )
(a) (b) RT 2 RT
2R 3R
M v2 2M v 2
(c) (d)
5R 5R
IIT, 2003

ANSWERS

1. (d) 2. (c) 3. (d) 4. (b) 5. (d) 6. (c)


7. (b) 8. (c) 9. (c) 10. (a) 11. (b) 12. (b)
13. (d) 14. (c) 15. (a) 16. (c) 17. (b) 18. (d)
19. (c) 20. (d) 21. (a) 22. (c) 23. (d) 24. (c)

SOLUTIONS
energy in a collision. Hence the molecules rebound
1. Let T1 be the initial temperature. Since vrms T,
with the same average speed whether the collision
T2 = 4 T1. For an adiabatic
is elastic or inelastic. Therefore, both P and T will
process,
not change.
T1 V1( – 1) = T2 V2( – 1)
3 RT
V1
1
T2 4. vrms = ; M = molecular mass of mixture
= =4 M
V2 T1 The speed of sound in the mixture is
5
1 RT Cp
V1 3
=4 v= M ; = of the mixture
V2 Cv
Given vrms = 2 v. Therefore
V1
= (4)3/2 = 8 3 RT RT
V2 = 2
M M
3 RT 3
2. vrms = ; M = molecular mass. Now T is =
M 2
doubled and M is halved. Hence the rms speed will
For the mixture,
become twice, which is choice (c).
3. Since the temperature of the molecules is the n1 Cv 1
n2 Cv 2
Cv =
same as that of the walls, there is no exchange of n1 n2
Kinetic Theory of Gases 18.9

n1 C p n2 C p 12. For a given M, vav T only. Therefore, the cor-


1 2
and Cp = rect choice is v1 as the temperatures of vessels A
n1 n2 and C are the same.
n1 C p n2 C p 13. E T and vrms T . Hence at 600 K,
Cp 1 2
= (1) 600
Cv n1 (C v )1 n2 (C v ) 2 E = 6.21 10–21
300
= 12.42 10 –21 J
3R 5R
For hydrogen, n1 = 2, (Cv)1 = and (Cp)1 = 600
2 2 and vrms = 484 684 ms–1
300
5R 7R which is choice (d).
For helium, n2 = n, (Cv)2 = and (Cp)2 =
2 2 14. For a gas, PV = nRT. Hence
Using these values and = 3/2 in Eq. (1) and solv- 1 mole RT
(P)O2 = and
ing, we get n = 2. V
5. The internal energy of n moles of a gas temperature 1 mole R 2T
T is given by (P)He =
V
f P He
U = (nRT)
2 = 2 or (P)He = 2 (P)O2
PO
2
where f = number of degrees of freedom.
For oxygen f = 5 and for argon f = 3. Hence 15. Since both the gases are diatomic, each has two
degrees of freedom associated with rotational
U = U1 + U2
motion. According to the law of equipartition of
5 3 energy, the rotational kinetic energy per degree of
= (4RT) + (2RT)
2 2 freedom is (1/2)kT. Since the temperatures of the
= 13RT two gases are equal, their rotational kinetic energies
will be equal. Hence the correct choice is (a).
3
6. E = kT. So the correct choice is (c). 16. According to the kinetic theory, the average kinetic
2
3
7. The rms speed = 3kT / m which is independent energy (KE) per molecule of a gas = kT. Let n1
of pressure. Hence the correct choice is (b). 2
and n2 be the number of moles of air in vessels 1
8. v = 3kT / m . Therefore, v0 = 3 kT / m0 and and 2 respectively.
vh = 3kT / mh . Before mixing, the total KE of molecules in the two
vessels is
v0 mh 1 1 3 3
Thus = = E1 = n1kT1 + n2 kT2
vh m0 16 4 2 2
or v h = 4 v 0. 3
= k(n1T1 + n2T2)
Hence the correct choice is (c). 2
9. The correct choice is (c) as explained above. After mixing, the total KE of molecules is

3 RT 3
10. We know that vrms = which gives E2 = (n1 + n2) kT
M 2
3 RT 3 8.3 300 where T is the temperature when equilibrium
M = v2 = is established. Since there is no loss of energy
r ms 1920 2 (because the vessels are insulated), E2 = E1 or
= 2 10 –3 kg = 2 g 3 3
Since M = 2, the gas is hydrogen. (n1 + n2)kT = k (n1T1 + n2T2)
2 2
3RT n1T1 n2T2
11. vrms = or vrms T . Hence the correct or T= (1)
M (n1 n2 )
choice is (b).
18.10 Comprehensive Physics—JEE Advanced

Now P1V1 = n1RT1 and P2V2 = n2RT2 which give 20. The root mean square speed is given by
PV
1 1 P2V2 3kT
n1 = and n2 = vr ms =
RT1 RT2 m
Using these in Eq. (1) and simplifying, we get
The escape velocity is given by ve = 2gR
T1T2 ( PV
1 1 P2V2 )
T= For vrms = ve, we require
( PV
1 1T2 P2V2T1 )
17. The root mean square speed is given by 3kT 2mg R
2 gR or T = , which is choice (d).
m 3k
3kT
vrms = 21. Average kinetic energy per molecule of a perfect
m
3
i.e. vrms T. Initial temperature T1 = 27 + 273 gas = kT.
= 300 K. Final temperature T2 = 927 + 273 = 1200 K. 2
Since temperature is increased by 4 times, the speed
3
is doubled. Hence the correct choice is (b). = n1 kT1
2
3 kT0 Average KE of molecules of the second gas
18. For oxygen : vrms = 3
m0 = n2 kT2
2
3kTh Total KE of the molecules of the two gases
For hydrogen : vrms =
mh before they are mixed is
The value of vrms will be the same if 3 3
K= n kT + n kT
T0 Th 2 1 1 2 2 2
=
m0 mh 3
= (n1 T1 + n2 T2)k (1)
m0 2
or T0 = Th = 16 200 = 3200 K = 2927°C If T is the temperature of the mixture, the kinetic
mh
energy of the molecules (n1 + n2) in the mixture is
Hence the correct choice is (d).
3
19. The pressure exerted by a gas is given by K = (n1 + n2)kT (2)
2
P=
nRT Since there is no loss of energy K = K . Equating (1)
V and (2) we get
mass RT n1T1 n2T2
= n1 T1 + n2 T2 = (n1 + n2) T or T =
molecular weight V n1 n2
8 RT 1 RT Hence the correct choice is (a).
Pressure exerted by oxygen P1 = = 22. Let n be the number of moles of the gas in the box.
32 V 4 V
1
14 RT 1 RT The kinetic energy of the gas = n M v 2 . When
Pressure exerted by oxygen P2 = = 2
28 V 2 V the box is suddenly stopped, this energy is used up
Pressure exerted by carbon dioxide in changing the internal energy, as a result of which
the temperature of the gas rises. The change in
22 RT 1 RT internal energy is given by
P3 = =
44 V 2 V 5
U = nCv T = n R T
From Dalton’s law of partial pressures, the total 2
pressure exerted by the mixture is given by 5
For a diatomic gas Cv = R.
P = P1 + P2 + P3 2
5R 1
1 RT 1 RT 1 RT Hence n T=n Mv2
= + + 2 2
4 V 2 V 2 V
5RT M v2
= , which is choice (c). or T= , which is choice (c)
V 5R
Kinetic Theory of Gases 18.11

23. The root mean square speed is related to absolute 5R


temperature T as U = 0.5 (1200 – 300)
2
3kT 0.5 5 8.314 900
crms = = = 9353 J.
m 2
For a given gas, m crms T. So the correct choice is (d)
Hence in order to double the root mean square PV
speed, the absolute temperature must be increased 24. Number of moles present initially is n = . Let
RT
to four times the initial value. Initial temperature n be the number of moles of the gas that leaked
T1 till the pressure falls to P . Since volume V of the
T2 = 4 T1 = 1200 K. vessel cannot change and temperature T remains
V = 0. constant during leakage, we have
Hence the heat energy supplied to the gas does no
PV
work on the gas; it only increases the internal en- n =
ergy of the molecules. RT
The increase in internal energy is Number of moles that leaked is
U = n Cv T
5R PV PV V
Since nitrogen is diatomic Cv = . The number n=n–n = – = (P – P )
2 RT RT RT
of moles of nitrogen in the vessel is So the correct choice is (c).
mass in kg 0.014 103
n= = = 0.5
molecular mass 28

II

Multiple Choice Questions with One or More Choices Correct


1. The root mean square speed of the molecules of a of the gas by 1 °C. Then, if M = molecular mass of
gas depends upon oxygen.
(a) the pressure of the gas 5 7
(b) the density of the gas (a) (= Cp /Cv) = (b) =
3 5
(c) the temperature of the gas
(d) the mass of a molecule of the gas R 2R
(c) v = (d) v =
2. The average translational kinetic energy of a mol- M( 1) M ( 1)
ecule of a gas at absolute temperature T is E and IIT, 1983, 1996
the root mean square speed of the molecules is v. 5. In a vessel a gas at temperature T has a pressure P.
Then The density of the gas is and vrms is the average
(a) E T (b) E T root-mean-square speed of a molecule. If N is the
(c) v T (d) v T number of molecules per unit volume and m the
3. The root mean square speeds of the molecules of mass of a molecule, then (k = Boltzmann constant)
hydrogen, oxygen and carbon dioxide at the same 2 2
1 2
temperature are vh, vo and vc respectively. Then (a) P = vrms (b) P = vrms
3 3
(a) vh = vo = vc (b) vh > vo
(c) vo > vc (d) vh < vo < vc 3kT
(c) vrms = (d) P = N kT
4. One mole of oxygen at 27 °C is enclosed in a ves- m
sel which is thermally insulated. The vessel is 6. N molecules of a gas, each of mass m, strike per
moved with a constant speed v and is then suddenly A at an
stopped. The process results in a rise of temperature angle to the vertical and rebound with a speed v.
18.12 Comprehensive Physics—JEE Advanced

The collisions are assumed to be elastic. of the piston from the base of the cylinder is h1 =
(a) The magnitude of change in momentum is 20 cm as shown in Fig. 18.2. When the tempera-
| p| = 2mv sin ture is raised to T2 = 177 °C, the new height of the
(b) | p| = 2mv cos piston above the base becomes h2. The system is
| p| then insulated from the surroundings and the pis-
(c) The pressure exerted on the wall is P =
NA ton is brought back to its original height. The new
N | p| equilibrium temperature becomes T3. Given (1.5)0.4
(d) P =
A = 1.18.
7. The pressure P of n moles of a gas varies with IIT, 2004
volume V as
P = a – bV2
where a and b are positive constants. The highest
absolute temperature to which the gas can be heated
is Tmax.
a3 / 2
(a) Tmax = ; R = gas constant
nR 3b
2a 3 / 2
(b) Tmax =
3nR 3b
1/ 2
a
(c) At Tmax, V =
b
1/ 2
a Fig. 18.2
(d) At Tmax, V =
3b (a) h2 = 30 cm (b) h2 = 24.5 cm
8. (c) T3 = 81°C (d) T3 = 258 °C
gas at temperature T1 = 27 °C. Initially the height

ANSWERS AND SOLUTIONS

3P 3kT 1 R
1. vr ms = . Hence all the four choices Therefore, M v2 =
m 2 1
are correct.
2R
3 3kT or v=
2. E = kT and vr ms = . Hence the correct M 1
2 m
choices are (a) and (d). So the correct choices are (b) and (d).
3. The rms speed is the maximum for the gas which is 5. The average pressure of a gas is given by
the lightest. Hence the correct choices are (b) and 1
(c). P= v 2rms
3
4. Oxygen is diatomic; it has 5 degrees of freedom. where density = mN; here m is the mass of a
Therefore, Cv = 5 R/2 and Cp = 7 R/2. So = molecule and N is the number of molecules per unit
Cp /Cv = 7/5. volume of the gas. Thus
The kinetic energy of oxygen molecules with a
1 2
1 P= mN vrms (1)
velocity v0 = M v2, where M = molecular weight 3
of oxygen. 2 The average kinetic energy of a molecule is given by
Now heat energy = Cv dT = Cv 1 = Cv 3
1
Cp R mv 2r ms = kT (2)
But Cp – Cv = R or –1= 2 2
Cv Cv Using (2) in (1), we get
R R P = NkT
or ( – 1) = or Cv =
Cv 1 Thus the correct choices are (a), (c) and (d).
Kinetic Theory of Gases 18.13

6. Refer to Fig. 18.3. Since the collision is elastic, the 1/ 2


a a
speed v of the molecule before collision = speed or a b = nR Tmax
after rebound. Therefore, the magnitude of the 3b 3b
change in momentum normal to the wall is 1/ 2
a a
| p| = mv cos – (– mv cos ) = 2 mv cos or a = nR Tmax
3b 3

2a 3 / 2
or Tmax = (4)
3 3b nR

d 2T
It is easy to check that is indeed negative at
dV 2
2
a = 3 bV . Hence Eq. (4) gives the maximum value
Fig. 18.3 of T. So the correct choices are (b) and (d).
Number of collisions per second = N. Therefore, 8. Let A be the cross-sectional area of the base of
time interval between successive collisions is the cylinder. Given T1 = 273 + 27 = 300 K and
T2 = 273 + 177 = 450 K.
1
t= At constant pressure, we have from Charles’
N law,
Force exerted on the wall by the molecules is
= rate of change of momentum V1 V2
=
T1 T2
| p|
F =
t Ah1 Ah2
or =
F T1 T2
Pressure P = . Hence the correct choices are (b)
and (c). A
T2 450 K
7. For n moles of an ideal gas, PV = nRT or h2 = h1 × 20 cm
T1 300 K
Using the given relation between P and V, we have = 30 cm
(a – bV2) V = nRT or aV – bV3 = nRT (1)
Since the system is completely insulated from the
Differentiating with respect to V, we get surroundings, it cannot take heat from or give heat
dT to the surroundings. Hence the second process is
a – 3 bV2 = nR adiabatic for which
dV
T 2V 2( –1)
= T 3V 3( –1)
dT 1
or = (a – 3 bV2) (2) where V2 = Ah2 and V3 = Ah1. Also, for a diatomic
dV nR
gas = 1.4. Therefore,
dT d 2T V2
( 1)
T will be maximum if = 0 and is negative. T3 = T2 ×
dV dV 2 V3
dT
Setting = 0 in Eq. (2), we get (1.4 1)
dV 30
a
1/ 2 = 450 ×
2
a = 3 bV or V = (3) 20
3b
Using (3) in (1) we get = 450 × (1.5)0.4.

a
1/ 2
a
3/ 2 = 450 1.18 = 531 K = 258 °C
a b = nR Tmax So the correct choices are (a) and (d).
3b 3b
18.14 Comprehensive Physics—JEE Advanced

III

Multiple Choice Questions Based on Passage

Questions 1 to 5 are based on the following passage


Passage I 3kT
2. From the relation vr ms = , it follows that the
Kinetic Theory of Gases m
constant k should be expressed in units of
The molecules of a gas move in all directions with various
(a) newton per metre per kelvin
speeds. The speeds of the molecules of a gas increase
(b) newton per kelvin
with rise in temperature. During its random motion, a fast
(c) joule per kelvin
molecule often strikes against the walls of the container of
the gas. The collisions are assumed to be perfectly elastic, (d) joule per kilogram per kelvin
i.e. the molecule bounces back with the same speed with 3. Choose the only correct statement from the following.
which it strikes the wall. Since the number of molecules (a) The pressure of a gas is equal to the to-
tal kinetic energy of its molecules per unit
is very large, billions of molecules strike against the walls
volume of the gas.
of the container every second. These molecules exert a
(b) The product of pressure and volume of a gas
sizeable force on the wall. The force exerted per unit area is always constant.
is the pressure exerted by the gas on the walls. According (c) The average kinetic energy of the mole-
to the kinetic theory, the pressure of a gas of density at cules of a gas is proportional to its absolute
absolute temperature T is given by temperature.
1 2 (d) The root mean square speed of a molecule
P= vr ms is proportional to the absolute temperature
3
of the gas.
where vr ms is the root mean square speed of the gas
4. If the temperature of a gas is increased from
molecule and is given by
27°C to 927°C, the root mean square speed of its
3kT molecules
vr ms =
m (a) becomes half (b) is doubled
where m is the mass of a molecule and k is Boltzmann (c) becomes four times (d) remains unchanged
constant. 5. The root mean square speed of oxygen gas mol-
1 2 ecule at T = 320 K is very nearly equal to (the
1. From the relation P = vr ms , molar mass of oxygen is M = 0.0320 kg per mole
3
dimensions of pressure are and gas constant R = 8.31 J mol–1 K–1)
(a) ML–1T–1 (b) ML–1T–2 (a) 300 ms–1 (b) 500 ms–1
–2 –1
(c) ML T (d) ML–2T–2. (c) 700 ms–1 (d) 900 ms–1

ANSWERS AND SOLUTIONS


1. Dimensions of P = dimensions of density dimen- 3. The correct choice is (c)
sions of (velocity)2
4. The correct choice is (b) since vr ms is proportional
= ML–3 (LT–1)2 = ML–1T–2 to the square root of absolute temperature
Hence the correct choice is (b).
3kT 3RT 3 8.31 320
2. Squaring we have 5. vrms = = =
m M 0.032
2
mvrms unit of energy = 499.3 ms–1
k= =
3T unit of temperature
Hence the correct choice is (b)
= joule per kelvin
Kinetic Theory of Gases 18.15

Questions 6 to 12 are based on the following passage At high pressures, when the molecules are too many and
too close together, the correction factors a and b become
Passage II
important.
Van der Waals Equation of State 6. In Van der Waals’ equation of state for real gases,
The equation of state PV = nRT holds for an ideal gas. The the product PV has the same dimensions as those of
behaviour of real gases shows departures from an ideal a
gas behaviour especially at high pressures. The model of (a) bP (b)
V
an ideal gas is based on a number of assumptions. Van der (c)
ab
(d) nRT
PV = nRT by taking V2
into account two of those assumptions which may not be 7. The dimensions of a are the same as those of
valid. He argued that (i) the volume of the molecules may
(a) PV (b) PV2
not be negligible compared to the volume V occupied by
the gas and (ii) the attractive forces between the molecules (c) P2V (d) P/V
may not be negligible. He said that pressure P in equation 8. The dimensions of b are the same as those of
PV = nRT is less than the true pressure by an amount p be- (a) P (b) V
cause of attractive forces between the molecules. Accord- (c) PV (d) nRT
ing to him, the pressure ‘defect’ p is inversely proportional a
9. The dimensions of are the same as those of
to the square of volume, i.e. b
1 (a) work (b) force
p
V2 (c) pressure (d) power
a 10. The dimensional formula for ab is
or p= 2 (a) ML2T–2 (b) ML4T–2
V 6 –2
where a is constant depending on the nature of the gas. (c) ML T (d) ML8T–2
a 11. The correction factors a and b depend upon
Thus the true pressure of the gas is P = P + p = P + 2 . (a) the pressure of the gas
V
He further argued that V is not the true volume of the (b) the volume of the gas
(c) the temperature of the gas
volume. According to him, the true volume of the gas is V (d) the nature of the gas
= (V – b) where b is a factor depending on the actual 12. The equation of state PV = nRT holds if the gas has
volume of the molecules themselves. Thus Van der Waals’ (a) low pressure and low density
equation for real gases is P V = nRT, i.e. (b) low pressure and high density
a (c) high pressure and low density
P V b = nRT
V2 (d) high pressure and high density.

ANSWERS AND SOLUTIONS


6. From Van der Waals’ equation, we have a dimensions of PV 2
9. Dimensions of =
a ab b dimensions of V
PV – bP + = nRT
V V2 = dimensions of PV
From the principle of homogeneity of dimensions, = dimensions of work
Hence the correct choice is (a).
a 10. Dimensions of ab = dimensions of PV2 dimen-
7. Dimensions of = dimensions of P sions of V
V2 = dimensions of (PV3)
Dimensions of a = dimensions of PV2.
= ML–1T–2 (L3)3 = ML8T–2
Hence the correct choice is (b).
Hence the correct choice is (d).
8. Dimensions of b = dimensions of V, which is 11. The correct choice is (d).
choice (b) 12. The correct choice is (a).
18.16 Comprehensive Physics—JEE Advanced

Questions 13 and 14 are based on the following passage 1 1


Passage III Tc = =
c d 2 vn
Mean Free Path
Hence the mean free path (i.e. the average distance the
During their random motion, the molecules of a gas often
molecule travels between two successive collisions) is
come close to each other. When the distance between
two molecules is comparable with the diameter of a 1
lc = v Tc =
molecule, the forces between them become very strong. d 2n
As a result, their individual momenta before and after the For air at S.T.P. the value of lc 3 10–7 m.
encounter are different. When this happens a ‘collision’
13. The mean free path of a molecule of a gas depends
is said to have occurred. The average distance a molecule
travels before it suffers a collision with another molecule (a) only on its diameter (d)
is called the mean free path (lc), which can be estimated (b) only on the number density (n) of the molecules
as follows. Suppose the average speed of a molecule of (c) on both d and n
diameter d is v . In one second, this molecule sweeps (d) neither on d nor on n.
out a volume d2 v
14. The average collision period in a gas
this volume, it will suffer collisions with them. If n is
(a) increases if the pressure is increased
the number density (i.e. number per unit volume) of the
(b) decreases if the pressure is increased
molecules, then the number of molecules in this volume
= d2 v n. The number of collisions per second = c (c) increases if the temperature of the gas is
increased.
= d2 v n. Therefore, the average time between two
collisions (called collision period Tc) is (d) decreases if the temperature of the gas is
increased.

ANSWERS AND SOLUTIONS


13. The mean free path is given by If the pressure is increased, the volume of the gas
1 decreases. Hence number density n increases.
lc = Therefore, Tc will decrease. If the temperature of
d 2n the gas is increased, the average speed v of the
Hence the correct choice is (c). molecules increases. Hence Tc will decrease. Thus
14. The collision period is given by the correct choices are (b) and (d).
1
Tc =
d 2 vn
19
Chapter
Transmission of Heat

REVIEW OF BASIC CONCEPTS

19.1 THERMAL CONDUCTIVITY


If a steady temperature difference (T1 – T2) is to be
maintained between the ends of a rod, heat must be
supplied at a steady rate at one end and the same must be
taken out at the other end. Suppose an amount of heat Q
t so that Q/t is the rate of
Fig. 19.1

(i) Q/t will be proportional to the area A of the cross-


section of the rod. temperatures T1 and T2 (T1 > T2
(ii) Q/t will be proportional to (T1 – T2).
(iii) Q/t will be inversely proportional to l, the distance Q1 k 1 A1 (T1 T2 )
between ends of the rod. = (1)
t L

Q A T1 T2 Q k A T1 T2 Q2 k 2 A 2 (T1 T2 )
H= or = and = (2)
t l t l t L
where k is a constant of proportionality called the thermal
conductivity of the substance. It is a measure of how (A1 + A2 L. If keq is the equivalent

stance. In the SI system, k is expressed in Js–1 m–1 °C–1 or the composite slab is
J s–1 m–1 K–1 or Wm–1 K–1. Q k eq ( A1 A2 ) (T1 T2 )
–3 –1 = (3)
[k K ] t L

Q Q1 Q 2
CONDUCTION THROUGH A COMPOSITE Since = +
19.2 SLAB t t t
keq (A1 + A2) = k1 A1 + k2 A2
Case 1. Two slabs placed one on top of the other k 1 A1 k 2 A2
keq = (4)
Suppose we have a composite slab made up of two different A1 A2
slabs of materials of thermal conductivities k1 and k2, and
cross-sectional areas A1 and A2 L 1
If A1 = A2, then keq = (k1 + k2).
2
19.2 Comprehensive Physics—JEE Advanced

Case 2. Two slabs placed in contact one after the ( k 1 T1 k 2 T2 )


other T0 =
(k 1 k 2)
L1 and L2
but of the same cross-sectional area A placed in contact as
19.3 THERMAL RESISTANCE

slab becomes much easier if we use the concept of thermal

V potential difference
R= ,
I e
Fig. 19.2

temperature difference T1 T2
temperatures T1 and T2 (T1 > T2). Let T0 be the temperature R=
rate of flow of heat Q/t
Q1 Q2
of the junction. In the steady state, = , i.e. (T1 T2 ) L L
t t R=
k A(T1 T2 ) kA
k 1 A(T1 T0 ) k 2 A(T0 T2 )
=
L1 L2
-
k1 k2 posite slab is
(T1 – T0) = (T0 – T2)
L1 L2 Req = R1 + R2
L1 L2
k 1 T1 k 2 T2 = L1 L2
keq A k1 A k2 A
L1 L2
T0 = (5)
k1 k2 k 1k 2 ( L1 L2 )
keq =
L1 L2 ( L1k2 L2 k1 )
(b) If the two slabs are joined in parallel as shown in
Q Q1 Q2
= or 1 1 1
t t t =
R R1 R2
Q k 1 A(T1 T0 )
= (6) keq ( A1 A2 ) k1 A1 k2 A 2
t L1 =
L L L
( k 1 A1 k 2 A2 )
Q k 1k 2 A(T1 T2 ) keq =
= (7) ( A1 A2 )
t k 1 L2 k 2 L1
L1 + L2) but its
cross-sectional area is A. If keq is the equivalent thermal 19.1
conductivity of the composite slab, -
Q k A (T1 T2 )
= eq (8) is maintained at 100°C and the other end is immersed
t ( L1 L2 )
minutes. Calculate the thermal conductivity of metal.
–1
k 1k 2 ( L1 L2 ) .
keq =
k 1 L2 k 2 L1
SOLUTION
2 k 1k 2 Given L = 0.3 m, A = r2 = (10–2)2 = 10–4 m2,
If L1 = L2, keq = and T1 = 100°C and T2 = 0°C. Now
(k 1 k 2)
19.3

–1
Q = mLf
= 31.4 80 cal
= 31.4 80 4.2 J ( 1 cal = 4.2 J)
Q k A (T1 T 2 )
Also = Fig. 19.3
t L
QL SOLUTION
k=
t A(T1 T2 ) Q1 Q2
In the steady state, = , i.e.
(31.4 80 4.2) 0.3 t t
=
(5 60) ( 10 4 ) (100 0) k1 A1 (T1 T0 ) k2 A 2 (T 0 T2 )
–1 –1
=
= 336 W m K L1 L2

50 0.02 (300 T0 ) 400 0.01 (T0 0)


19.2 =
0.1 0.2
A cylindrical metal boiler of radius 10 cm and
T0 = 100°C
on an electric heater. If the water boils at the rate
–1
- 19.4
A steel rod (L1 = 10 cm, A1 = 0.02 m2, k1 = 50 W m–1
102 W m–1 K–1 and latent heat of vaporisation K–1) and a brass (L2 = 10 cm, A2 = 0.02 m2, k2 = 110
= 2.26 103 –1. W m–1 K–1
ends of the composite rod are maintained at 403 K
SOLUTION
Q = mLv
Q –1
) (2.26 103 –1
)
t
= 50 2.26 103 Js–1
Base area of boiler A = r2 = (0.1)2 = 10–2 m2
L = 3.14 cm = 3.14 10–2 m
Q k A (T f T w ) Fig. 19.4
=
t L
3
SOLUTION
50 2.26 10 =
(1.13 102 ) ( 10 2 ) (T f Tw) 1 1
2 keq = (k1 + k2) = (50 + 110)
3.14 10 2 2
Tf – Tw = 1000°C = 80 W m–1 K–1

Tf = 1000 + Tw = 1000 + 100 Q k eq ( A1 A 2 ) (T1 T2 )


=
t 0.1
= 1100°C
80 0.04 (403 273)
=
19.3 0.1
= 4.16 103 Js–1
A steel rod (L1 = 10 cm, A1 = 0.02 m2 and k1 = 50 J
s–1 m–1 K–1) is welded to a silver rod (L2 = 20 cm, A2
= 0.01 m2, k2 = 400 J s–1 m–1 K–1 19.5
A metal cylinder of radius r and thermal conductivity
k1 = 2k is surrounded by a cylindrical metallic shell
- of inner radius r and outer radius 2r
conductivity k2 = k -
junction in the steady state. posite system are maintained at constant temperatures
19.4 Comprehensive Physics—JEE Advanced

T1 and T2 (T1 > T2). Find the equivalent thermal con-


ductivity of the system.

Fig. 19.6
Fig. 19.5 Consider a small element of thickness dr at a distance
r from the axis of the shell. Let dT be the temperature
SOLUTION between the inner and outer surfaces of the element.
If H
-
L and cross-sectional area k
2 rL dT
H=
A1 = r2 of conductivity k1 and a rod of the same dr
L and cross-sectional area A2 = [(2r)2 – r2] dr 2 kL dT
2
= 3 r and conductivity k2 placed one on top of the =
r H
other (in parallel). In the steady state
Q Q1 Q2
= + r2 T
t t t dr 2 kL 2
= dT
r H T
k1 A1 (T1 T2 ) k2 A 2 (T1 T2 ) r1 1
=
L L r2 2 kL
ln = (T2 – T1)
k1 ( r 2 ) (T1 T2 ) k2 (3 r 2 ) (T1 T2 ) r1 H
Q
=
t L L 4 cm 2 3.14 .3 0.5
ln = (100 – 0)
(i) 2 cm H
Cross-sectional area of the composite system is A = 2 3.14 .3 0.5 100
(2r)2 = 4 r2. If keq is the equivalent conductivity, H
then H = 3.14 10 Js–1
4

Q keq (4 r 2 ) (T1 T2 )
= (ii)
t L 19.4 CONVECTION
In convection heat is transferred by the physical movement
4 keq = k1 + 3 k2 of matter
k1 3k2 2k 3k 5k
keq = = = by molecules via collisions in their local regions.
4 4 4
19.5 RADIATION
19.6
All bodies emit heat from their surfaces at all tempera-
A cylindrical metallic shell has inner radius r1 = 2 cm
radiant heat
and outer radius r2 L = 50
or thermal radiation.
T1
= 0°C and T2
–1 –1
K
(1) Black Body A perfect black body is one which
from the outer to the inner surface.

SOLUTION
emitter of radiations. Consequently, a black body, when
19.5

black body T1 and T2, we have


radiation. T k
lo T T0 ) T2 = t,
(2) Emissive Power e) of a body is e 1
ms
area
of a SI unit of e is Js–1 m–2 or Wm–2. T1 T0
(3) Absorptive Power t =K e T2 T0
ms
where K=
by a k
completely absorbs all radiations, a = 1 for a black body.
(4) Kirchhoff’s Law T1 to T2 when placed in a medium of temperature T0.
An approximate formula is
emissive power to the absorptive power is the same for T1 T2
1 T1 T2
all substances, i.e. = T0
t K 2
e
= constant (7) Wien’s Displacement Law As the temperature of a
a black body increases, the maximum intensity of emission
(5) Stefan’s Law
by a unit area of a black body is proportional to the fourth other words,
power of its absolute temperature, i.e.
mT = b = constant
E T 4 or E = T 4 where m
where is a constant known as Stefan’s constant. Its takes place at absolute temperature T
value is constant b is
= 5.735 10–8 Wm–2 K–4
b 10–3 mK (metre kelvin)
When a black body at absolute temperature T is surround
by another black body at a lower absolute temperature, T0,
19.7
2

4
E= (T – T40)
If his skin temperature is 37°C, calculate the rate at
a perfect black body, then
0.8 and Stefan’s constant = 5.7 10–8 W m–2 K–1.
E= (T 4 – T 40)
where emissivity is always less than unity. SOLUTION
power to that of the black body. T = 273 + 37 = 310 K and T0 = 273 + 27 = 300 K
(6) Newton’s Law of Cooling Rate of loss of heat = A (T4 – T04)
by a body is directly proportional to the excess of its
= 0.8 2 (5.7 10–8) [(310)4 – (300)4]
temperature difference is small, i.e. = 1.03 102 W
dQ
(T – T0) 19.8
dt
where T is the temperature of the body and T0 that of the
m is the mass of the body, s
heat and dT dt, then
dQ = msdT constant b 10–3 mK.
dQ dT
= ms = – k (T – T0) SOLUTION
dt dt
where – k m T = b.
3
dT k b 2.8 0
dt T= = = 6080 K
(T T0 ) = ms m 475.3 10
19.6 Comprehensive Physics—JEE Advanced

If the temperature at the end of next 6 minutes is T,


19.9
then
A body cools from 80°C to 50°C in 6 minutes in a
50 20
room where the temperature is 20°C. What is the tem- e =6K (ii)
perature of the body at the end of next 6 minutes? T 20
From (i) and (ii)
SOLUTION
30
80 20 e e (2)
=6K T 20
e
50 20 30 = 2(T – 20)
e (2) = 6 K (i)
T = 35°C

Multiple Choice Questions with Only One Choice Correct


1. k1 1 s1 k1 1 s2
their ends kept at the same temperatures 1 and 2 (a) = (b) =
k2 2 s2 k2 2 s1
with 2 > 1. If A1 and A2 are their cross-sectional
areas and k1 and k2 their thermal conductivities, k1 1
(c) = (d) k1 = k2
k2 2
same if
4. A slab of stone of area 0.34 m2 and thickness
A1 k1 A1 k2 10 cm is exposed on the lower face to steam at
(a) = (b) =
A2 k2 A2 k1 100°C. A block of ice at 0°C rests on the up-
A1 k1 1 A1 k2 2
(c) = (d) = melted. Assume that the heat loss from the sides
A2 k2 2 A2 k1 1

2. A cylinder of radius R made of a material of 3.4 105 –1


. What is the thermal conductivity
thermal conductivity k, is surrounded by a cylin- of the stone in units of Js–1m–1°C–1?
drical shell of inner radius R and outer radius 2R (a) 1.0 (b) 1.5
made of a material of thermal conductivity k2
(c) 2.0 (d) 2.5
two ends of the combined system are maintained
5. -
heat across the cylindrical surface and the system is face area A P. If the emissivity
and is Stefan’s constant, the
of the system is
2
k1 k2 P P
(a) k1 + k2 (b) (a) T = (b) T =
k1 k2 A A
1/ 2 1/ 4
k1 3k2 3k1 k2 P P
(c) (d) (c) T = (d) T =
4 4 A A
6. What are the dimensions of Stefan’s constant?
3. (a) ML–2 –2K –4 (b) ML–1 –2K–4
–3 –4
areas of cross-section have their ends kept at the K (d) ML0 –3K–4
same temperatures 1 and 2. If k1 and k2 are their 7.
thermal conductivities, 1 and 2 their densities and upon
s1 and s2 (a) the nature of its surface
of heat in the two rods will be the same if (b) the area of its surface
19.7

(c) the temperature of its surface 14.


(d) all the above factors
8. are joined end to end. But when they are joined
depends upon
(a) the nature of its surface heat, in the same conditions, in
(b) the area of its surface (a) 24 s (b) 3 s
(c) the temperature of its surface (c) 1.5 s (d) 48 s
(d) all the above factors
9. A composite slab consists of two slabs A and B of 15.
different materials but of the same thickness placed

of A and B are k1 and k2 respectively. A steady


temperature difference of 12°C is maintained across (a) 2.5 s (b) 10 s
the composite slab. If k1 = k2/2, the temperature (c) 20 s (d) 5 s
difference across slab A will be
16.
(a) 4°C (b) 8°C made from the same metal form the sides of an
(c) 12°C (d) 16°C ABC B
10. l1 and l , radii points A and B are maintained at temperatures T
r1 and r2 have thermal conductivities k1 and k2 and 2 T respectively. In the steady state, the
temperature of point C is TC
at the same temperature difference. If l1 = 2l2 and heat conduction takes place, the ratio TC /T is
r1 = r2
1 3
same if k1/k2 is (a) (b)
(a) 1 (b) 2 2( 2 1) 2 1
(c) 4 (d) 8 1 1
(c) (d)
11. A solid sphere and a hollow sphere of the same 3( 2 1) 2 1
-
ature and allowed to cool in the same surround-
17. S1 and S2 are made of the same
T, then
S1 is three times that of S2. Both the spheres are heated
(a) the hollow sphere will cool at a faster rate
for all values of T
(b) the solid sphere will cool at a faster rate for
all values of T S1 to that of S2 is
(c) both spheres will cool at the same rate for 1 1
all values of T (a) (b)
(d) both spheres will cool at the same rate only for 3 3
1/ 3
small values of T. 3 1
12. If the temperature of a black body increases from (c) (d)
1 3

increases by a factor of
287 4 18. A spherical black body of radius 12 cm radiates 450
(a) (b) 16 W power at 500 K. If the radius were halved and
7
the temperature doubled, the power radiated in watt
(c) 4 (d) 2
would be
13. A body cools from 60°C to 50°C in 10 minutes. If (a) 225 (b) 450
-

of the body at the end of next 10 minutes will be


(a) 38.5°C (b) 40°C
19.
(c) 42.85°C (d) 45°C
19.8 Comprehensive Physics—JEE Advanced

U1, between (a) 1 (b) 1/2


U2 (c) 2/3 (d) 1/3
and 1500 nm is U3 b = 2.88
106
(a) U1 = 0 (b) U3 = 0
(c) U1 > U2 (d) U2 > U1

20.
black bodies at temperatures T1, T2 and T3 respec-

are such that


Fig. 19.8

24.

the same cross-section


have been joined as shown

Fig. 19.9

Fig. 19.7 the


junction of the three rods will be
(a) T1 > T2 > T3 (b) T1 > T3 > T2
(a) 45°C (b) 60°C
(c) T2 > T3 > T1 (d) T3 > T2 > T1
(c) 30°C (d) 20°C

21. When the temperature of a black body increases,


25. An ideal black-body at room temperature is thrown
into a furnace. It is observed that
m to
(a) initially it is the darkest body and at later
0.13
body at the respective temperature is:
(b) it is the darkest body at all times
16 4
(a) (b)
1 1 (d) initially it is the darkest body and at later

1 1
(c) (d)
4 16 26.
22. If the temperature of the sun were to increase from
T to 2T and its radius from R to 2R, then the ratio fall of temperature
(T ) of two bodies
was previously will be x and y
same surface area,
(a) 4 (b) 16
with time (t) due to
(c) 32 (d) 64 emission of radia-
23. tion. Find the cor-
- rect relation between Fig. 19.10
K and 2K emissive power (E)
and thickness x and 4x, respectively, are T2 and T1 and absorptive power (a) of the two bodies.
(T2 > T1 (a) Ex > Ey; ax < ay (b) Ex < Ey; ax > ay
A T2 T1 K
slab, in a steady state is f, with f (c) Ex > Ey; ax > ay (d) Ex < Ey; ax < ay
x
19.9

27. temperature of the sun. Given Wien’s constant


conductivity 420 W/m/K has one of its ends in b = 2.88 10–3 mK.
(a) 5000 K (b) 6000 K
area of cross-section is 10 cm2, the amount of ice (c) 8000 K (b) 106 K
that melts in 1 minute is 35.
the rate of 1400 Wm–2
the sun from the surface of the earth is 1.5 1011 m
and the radius of the sun is 7.0 108
28. A body cools from 75°C to 65°C in 5 minutes in a sun as a black body, it follows from the above data
- that the surface temperature of the sun is about
ture of the body at the end of next 5 minutes will be
(a) 55°C (b) 56°C (c) 6000 K (d) 6100 K
(c) 57°C (b) 58°C 36.
29. A liquid takes 6 minutes to cool from 80°C to 50°C. rods are joined one after the other and this combi-

(a) 6 min. (b) 8 min.


(c) 10 min. (d) 12 min. rods are placed one on top of the other and con-
30. A body initially at 80°C cools to 64°C in 5 minutes nected to the same vessels. If q1 and q2

q1
the two cases, then the ratio is
(a) 15°C (b) 16°C q2
(c) 20°C (d) 25°C 1 2
(a) (b)
31. In Q. 30 above, the temperature of the body at the 2 1
end of 15 minutes will be 1 1
(a) 41°C (b) 43°C (c) (d)
4 8
(c) 45°C (b) 47°C
32.
of thermal expansion 1, 2 -
lii Y1, Y2

1 : 2 = 2 : 3, the ther-
mal stresses developed in the two rods are equal
provided Y1 : Y2 is equal to:
(a) 2 : 3 (b) 1 : 1

33. A sphere, a cube and a thin circular plate have the


same mass and are made of the same material. All Fig. 19.11
37. A and B
respectively are coated with carbon black on their
(a) the maximum for the sphere and minimum -
for the plate. tensity of emission of radiation are 300 nm and 500
(b) the maximum for the sphere and minimum
for the cube. them are in the ratio of
(c) the maximum for the plate and minimum for
the sphere. 5 5
(a) (b)
(d) the same for all the three. 3 3
34. 5 2
5 4
(c) (d)
3 3
19.10 Comprehensive Physics—JEE Advanced

38.
transfer promarily due to radiation?
42. A layer of ice at 0 °C of thickness x1
pond of water. L, and k respectively are the latent
heat of fusion of water, density of ice and thermal
conductivity of ice. If the atmospheric temperature
is – T °C, the time taken for the thickness of the
layer of ice to increase from x1 to x2
39. A spherical body of emissivity , placed inside a (a)
L
(x1 + x2)2 (b)
L
(x2 – x1)2
perfectly black body (emissivity = 1), is maintained 2kT kT
at absolute temperature T L 2 2 L 2
a unit area of the body per second will be ( is Ste-
(c) x2 x1 (d) x2 x12
2kT kT
fan’s constant)
43. AB and CD
(a) T4 (b) T4 L, cross-sectional area A and thermal conductiv-
4
(c) (1 – ) T (d) (1 + ) T 4 ity k
A, C and D are maintained at temperatures T1 =
40. A and B made of the same 20°C, T2 = 30°C and T3
temperature at B is
(a) 32 °C (b) 33 °C
diameter of A is twice that of B, the ratio of rates of (c) 34 °C (d) 35 °C
A and B will be
(a) 1 : 1 (b) 2 : 1
(c) 4 : 1 (d) 1 : 4
41. A cubical ice box is made of thermocole of thick-

thermocole is 1.0 10–2 Wm–1 K–1 and latent heat


of fusion of water is 3.35 105 –1
, the mass of
ice left unmelted in 3 hours is very nearly equal to Fig. 19.12

ANSWERS
1. (b) 2. (c) 3. (d) 4. (a) 5. (d) 6. (d)
7. (d) 8. (c) 9. (b) 10. (d) 11. (c) 12. (b)
13. (c) 14. (d) 15. (b) 16. (b) 17. (d) 18. (d)
19. (d) 20. (b) 21. (d) 22. (d) 23. (c) 24. (b)
25. (a) 26. (c) 27. (c) 28. (c) 29. (d) 30. (b)
31. (b) 32. (c) 33. (c) 34. (b) 35. (a) 36. (d)
37. (c) 38. (d) 39. (b) 40. (c) 41. (a) 42. (c)
43. (a)

SOLUTION

Q kA 2 A1 k2
1. = 1
k1A1 = k2A2 or =
t l A2 k1
Since ( 2 – 1) and l are the same for the two rods,
Q/t will be the same if the product 2. l and let its ends
kA is the same for the two rods, i.e. if be maintained at temperatures 1 and 2. Area of
19.11

the cross-section of the inner cylinder = R2. Area E= T4


of cross-section of outer cylinder = (2R)2 – R2 where is the Stefan’s constant and T is the
= 3 R 2. absolute temperature.
E
2 = 4
Q1 = k1 R 1 2 (i) T
l Dimensions of =
dime per unit area per second
k (3 R 2 ) 1
Q2 = 2 2
(ii) dimension of T 4
l
Let the effective thermal conductivity of the ML2 2 2 1

compound cylinder be k = 4
= ML0 –3
K–4
K
across the compound cylinder is
7.
k (4 R 2 ) 1 2 8.
Q= (iii)
l 9. Let 1 and 2 be the temperatures at the two faces
Now Q = Q1 + Q2 (iv) of the composite slab and let be the temperature
at the common face of the slab. If l
k = k1 + 3k2
of each slab and A the area of their face, then, in
k1 3k2 A=
or k=
4 rate B, i.e.
k1 A ( 1 ) k2 A ( 2)
=
3. l l
is (d).
or k 1( 1 – ) = k 2( – 2)
4. Now k2 = 2k1
= mL, where m is the mass of ice melted and L its
latent heat. Now time (t) = 1 hour = 3600 s and ( 1 – ) = 2( – 2) (i)
thickness d Also, 1 – 2 = 12°C or 2 = 1 – 12 (ii)
heat is
mL
Q= ( 1 – ) = 2{ – ( 1 – 12)}
t
or 3( 1 – ) = 24 or 1 – = 8°C
Qd
k= ce the correct choice is (b).
A 2 1
10. A and B are
mLd
= Q2
tA Q1 k r2 k r2
2 1 = 1 1 and = 2 2
t l1 t l2
3.6 3.4 105 0.1
= = 1 Js–1 m–1 °C–1 Q1 = Q2, if
3600 0.34 100 0
k1r12 k r2 k l r22
= 2 2 or 1 = 1 =2 (2)2 = 8
5. - l1 l2 k2 l2 r12
perature T
= T4
11.
upon its material, its surface area and its tempera-
A is
1/ 4
P
P= A or T = at the same temperature, will cool at the same rate.
A

6. 12. From Stefan’s law, E = T 4


E1 = (273 + 7)4
19.12 Comprehensive Physics—JEE Advanced

E2 560 4
E2 = (273 + 287)4 =
E1 280
= (2)4 = 16

13.

Q
= k(T – T0)
t Fig. 19.13
where k is a constant, T kA(TB TC ) kA(TC T)
in time interval t and T0 is the temperature of the or ( AC = 2 BC )
m is the mass of the body and s its l 2l
is the temperature at the TC 3
end of the next 10 minutes) ( TB = T 2 )
T 2 1
ms 60 50 50 60
=k 25 and 17.
10 min 2
d
ms 50 50 ms = (4 r 2) T4
=k 25 dt
10 min 2 d r2
. Since
4 dt ms
m= r3 , r m1/3
3
14. Let Q be the heat transferred. If k is the thermal d m2 / 3
conductivity of each rod, their equivalent conduc- dt m
tivity, when they are joined in series (end to end) is 1
2k. If t1 is time of transfer of heat, then
m1 / 3
2k A t1
Q1 = d
l for S1 1/ 3 1/ 3
dt m of S2 1
= =
d m of S1 3
equivalent conductivity is k/2. for S2
dt
k
A t2 18.
2
Q2 = P = (4 r2) T4 or P r2 T 4
l
2 4 2 4
Now Q1 = Q2 P2 r2 T2 1 2
= =4
2k A t1 kA t2 P1 r1 T1 2 1
= or t2 = 4t1 = 4 12
l 2l = 48 s or P2 = 4P1 = 4 450 = 1800 W
19. From Wien’s displacement law m T = b, the maxi-
15.
-
b 2.88 106 nmK
tween the temperature of the body and that of the m = = 1000 nm
T 2880 K
difference = 50 – 30 = 20° and in the second case U2 is the maximum. Since a black body
the temperature difference = 40 – 30 = 10°. Since U1 0 and
the temperature difference in the second case is half U3
20. For black body radiations, T = constant. It is
m
2 > 3 > 1
16. TB > TA follows that T1 > T3 > T2, which is choice (b).
B to A and from B to C. In the steady state, rate 21. T =
B to C constant, i.e.
Q2 Q3 1 T2 0.26
from C to A, i.e. 1 T1 = 2 T2 or =2
t t 2 T1 0.13
19.13

or T2 = 2T1.
enters O O, i.e.
E = T4
QA + QB = QC
E1 = T 41 and E2 = T 42
4 kA t kA t kA t 0
E1 T14 T1 1 4
1 or
= = l l l
E2 T24 T2 2 16
t t) = t
( T2 = 2T1)
or 3t = 180 or t = 60°C.

22.

E= T4
where

R at absolute temperature T
Q = 4 R2 T 4
If R and T are both doubled, we have
Q = 4 (2R) 2 (2T ) 4 Fig. 19.14

= 64 4 R2 T4 25.
= 64 Q

23. Let A be the area of each slab. In the steady state,


26.
of body x falls more rapidly with time than that
Q T2 T1 A (T2 T1 ) of body y Ex > Ey
= (1)
t l1 l2 l1 l2 -
K1 A K2 A K1 K 2 ax > ay
correct choice is (c).
Given l1 = x, l2 = 4x, K1 = K and K2 = 2K
27. Given k = 420 W/m/K, A = 10 cm2 = 10 10–4 m2
= 10–3 m2, 1 = 100°C, 2 = 0°C, t = 1 minute
Q A (T2 T1 ) A (T2 T1 ) K 1
= = 60 s and l
t x 4x x 3
k A( 1 2)t
K 2K Q=
l
1 2520
f= Q = 2520 J = = 600 cal.
3 4.2
24. Let A and l be the area of cross-section and the
k Q 600 cal
conductivity and t°C the temperature of the junc- m= =
L 80 cal
tion O
O from rods A and B
28. We have
kA t
QA = T1 T0
l e = Kt
T2 T0
kA t
and QB = 75 25 50
l or =K 5 or = 5K
e e
C is 65 25 40
kA t 0 5
QC = e = 5K (1)
l 4
19.14 Comprehensive Physics—JEE Advanced

If the temperature at the end of next 5 minutes is T , 32. Let l


we have heated so that the increase in temperature is t°C.
65 25 lt
e = 5K
T 25 where
rod
40 Strain = l t/l = t
or e = 5K (2)
T 25
Stress = Y strain = Y t
Stresses are equal for the two rods if
40 5
e = e Y1 1 t = Y2 2 t
T 25 4
Y1 3
40 5 or = 2 =
or = Y2 1 2
T 25 4
T = 57°C. 33. Since the material is the same the density is the
29. Given same. Since the mass is the same and density is the
80 20 same, all three have the same volume. For the same
e = 6K e (2) = 6K (1)
50 20
If the body takes t minutes to cool from 60°C to
30°C, then heat by radiation is proportional to the surface area.
60 20
e = tK e (4) = tK (2) 34. m T=b
30 20
3
b 2.88 10
T= = 6000K
m 480 10
t lo e 4) lo 10 4) 0.602
= 2
6 lo e 2) lo 10 2) 0.301
35. Let R be the radius of the sun and let r be the radius
or t = 12 minutes.
of the earth’s orbit round the sun. If the surface
temperature of the sun is T (in kelvin), the ener-
30. If T0
we have 4 R2 T 4, where is Stefan’s constant whose
80 T0 value is 5.67 10–8 Wm–2 K–4. Now the area of
e = 5K (1) the spherical surface of radius r is 4 r2
64 T0
64 T0 earth’s surface is
e = 5K (2)
52 T0 4 R2 T 4 R2 T 4
= = 1400
4 r2 r2
80 T0 64 T0
= 1400 r 2
64 T0 52 T0 T4 =
R2
T0 = 16°C, which is choice (b).
2
1400 1.5 1011
31. If T is the temperature after 15 minutes, then =
8 2
52 T0 5.67 10 7 108
e = 5K
T T0 = 1.1338 1015
52 16
e = 5K (3) or T = 5803 K
T 16

36. If a steady temperature difference ( 1 – 2) is


80 16 52 16
=
64 16 T 16 L and cross-sectional area A
T = 43°C, which is choice (b).
19.15

k A( 4
1 2) b b4 A
q= Q= A =
L 4
m m
where k 2
For a sphere of radius r, A = 4 r
the two rods are connected in series. If two rods b4 4 r 2 r2
Q= = k (3)
L1 4 4
m m
and L2 and conductivities k1 and k2 are joined in
where k = 4 b4
series, the equivalent conductivity ks
r12
L1 L2 L L2 Q1 = k
= 1 (1) 4
ks k1 k2 m 1

For two identical rods, L1 = L2 = L and k1 = k2 = k, in r22


ks = k. Further, when two and Q2 = k 4
m 2

composite rod is (2L) but its cross-sectional area is 2 4


Q1 r1 m 2
A = 4
Q2 r2 m 1

kA
1 2 2 4 2
q1 = (2) 3 cm 500 nm 5
2L = =
5cm 300 nm 3
In the second case, the two rods are connected in
38.
k1
and k2 are joined in parallel, the equivalent condu-

kp = k1 + k2
For two identical rods, k1 = k2 = k kp =
(2k). Furthermore, the cross-sectional area of the
due to conduction or convection.
composite rod is (2A
39.
2k 2 A 1 2 body of emissivity is T 4, irrespective of the
q2 = (3)
L
q1 1 40.
= . Now, the rate proportional to the surface area.
q2 8
41.
0.30 m. Since a cube has 6 faces, the total surface
area of the cube exposed to air is A = 6 (0.3)2
37. = 0.54 m2.
black body at absolute temperature T
of thermacole (d) = 5.4 cm
Q = AT 4 (1)
where A is the surface area of the body and is = 5.4 10–2 m
- of exposure (t) = 3 hours = 3 60 60
ment law, = 1.08 104 s
m T = b conductivity (k) = 1.0 10 J s–1 m–1°C–1
–2

where m Ta) = 40°C


maximum emission of radiation and b is Wien’s
Ti)= 0°C
b t is
T= (2)
m kA(Ta Ti )t
Q=
d
19.16 Comprehensive Physics—JEE Advanced

2
1.0 10 0.54 (40 0) 1.08 104
= 2 kATdt L
5.4 10 = A Ldx or dt = · xdx
x kT
= 4.32 104 J
Now, heat of fusion of water, L = 3.35 102 –1
t x
L 2
= 3.35 105 –1 6
10 J of dt = xdx
0
kT x
1

x2
L x2 L
Q 4.32 104 or t= = (x22 – x21 ),
m= kT 2 2kT
L 3.35 105 x1
which is choice (c).
43. Let TB be the temperature at B
of heat from C towards B is
42. When the temperature of the air is less than 0°C,
Q1 k A (T2 TB )
the cold air near the surface of the pond takes heat =
t L/2
D towards B is
x Q2 k A (T3 TB )
be the thickness of the ice layer at a certain time. =
t L/2
If the thickness increases by dx in time dt, then the
dt B towards A is
Q3 k A (TB T1 )
=
kA[0 ( T )]dt kATdt t L
Q= = (1) In the steady state, the rate at which heat enters B =
x x
rate at which heat leaves B, i.e.
where A is the area of the layer of ice and – T °C is
dm is the Q1 Q2 Q
= 3
Q = dm L. But t t t
dm = A dx, where 2 k A (T2 TB ) 2 k A (T3 TB )
or
L L
k A (TB T1 )
=
L
or 2(T2 – TB) + 2(T3 – TB) = TB – T1
T1 2 T2 2 T3
TB =
5
20 2 30 2 40
=
Fig. 19.15 5
Q = A Ldx (2) = 32 °C, which is choice (a).

II

Multiple Choice Questions with One or More Choices Correct


1.
(a) All bodies emit thermal radiations at all tem- mirror
peratures
a velocity of 3 108 ms–1
19.17

2.
depends upon body increases.
(a) the nature of its surface
(b) the area of its surface
(c) the temperature of its surface
7. A composite block is made of slabs A, B, C, D and E
3.
does not depend upon a constant K L)
(a) the density of the body
(b) the nature of its surface Q
(c) the area of its surface
(d) the temperature of its surface A and E slabs are same.
4. E is maximum.
(c) temperature difference across slab E is small-
- est.
mum emission are T1 and T2 respectively. If Wien’s C B
constant is b 10–3 mK, then D.
(a) T1 = 3718 K (b) T1
(c) T2 = 2318 K (d) T2 = 4677 K
5. A and B of equal surface area are placed
one on top of the other to form a composite plate of
A and B are

the exposed surface of plate A is – 10 °C and that


of the exposed surface of plate B

the temperature of the contact surface is T1 if the Fig. 19.16


plates A and B are made of the same material and 8. Initially a black body at absolute temperature T is
T2 if their thermal conductivities are in the ratio 2 : kept inside a closed chamber at absolute temper-
3 then ature T0
(a) T1 = – 4°C (b) T1 = – 2°C allow sun rays to enter. It is observed that tem-
(c) T2 = – 3°C (d) T2 = 0°C peratures T and T0 remains constant. Which of the
6. Initially a black body at absolute temperature T is
kept inside a closed chamber at absolute temper-
ature T0 body remains the same.
allow sun rays to enter. It is observed that tem-
peratures T and T0 remains constant. Which of the body increases.

body increases.
body remains the same.

body increases.

ANSWERS AND SOLUTIONS


1. 3. From Wien’s law, m T = b, where b is a constant
and m
2. All the four choices are correct. m depends only on T, the
temperature of the body.
19.18 Comprehensive Physics—JEE Advanced

4. From Wien’s law, max T=b the thermal resistances are connected as shown
3
T1 = 7
= 3718 K
7.8 10
3
. 10
T2 = = 4677 K
6.2 10 7
So the correct choices are (a) and (d).
5. Let T be the temperature of the contact surface.

Q k A( 10 T ) k A(T 10)
= A = B
t 4.0 6.0
k A ( 10 T ) k (T 10)
or = B (1)
4 6 Fig. 19.17
(a) If kA = kB T1 = – 2°C. Reff
kA 2 1 1 1 1
(b) If = T2 = 0°C. =
kB 3 Reff RB RC RD

6. 3 2 5 4
= =
from a black body is proportional to (T – T0) . Since 4 4R R 4R R
T and T0 remain constant, the rate of emission of R
Reff =
4
-
-
QA = (Q)eff = Q .
rect choices are (a) and (d).
7. Let W
L, area A (= LW) T
Q= , where T is the temperature
and thermal conductivity K R
L L difference between the ends of the slab. Also the
R= = temperature difference between the ends of slabs
KA K ( LW ) B, C and D is the same = ( T)eff
A, B, C, D and Q R
E respectively are ( T)A = QARA = A
8
L R Q R
RA = = ( T)B = ( T)C = ( T)D = QAReff = A
(2 K )(4 LW ) 8 4
L 1 Q R
where R= = and ( T) = A ( Q = Q A)
K ( LW ) KW 24
4L 4R E
RB = = is the minimum.
3K ( LW ) 3
4L R ( T )B Q R/4 3
RC = = Now QB = = A = QA
4 K (2 LW ) 2 RB 4R /3 16
4L 4R ( T )C Q R/4 1
RD = = QC = = A = QA
5K ( LW ) 5 RC R/2 2
L R ( T )D Q R/4 5
R = = QD = = A = QA
6 K (4 LW ) 24 RD 4R /5 16
Since slabs B, C and D are in parallel and slabs A And Q = QA B is
and E are in series with this parallel combination,
19.19

3 5 1 T and T0 remain constant, the rate of emission of


Now QB + QD = QA + QA = QA which
16 16 2
-
is equal to QC
the correct choices are (a), (c) and (d). -
8. rect choices are (a) and (d).
4
from a black body is proportional to (T – T0) . Since

III

Multiple Choice Questions Based on Passage


Questions 1 to 6 are based on the following passage 1. -
Passage I fan’s constant are
Thermal Radiations (a) ML–2 –2K–4 (b) ML–1 –2K–4
–3 –4
K (d) ML0 –3K–4
2. What is the SI unit of Stefan’s constant?
(a) J s–1K–4 (b) W m–1K–4
–2 –4
(c) W m K (d) J m–2K–4
is transferred to us by a process which, unlike conduction 3.
or convection, does not require the help of a medium in the thermal radiations lie?

4.
thermal radiations?
(a) Constant volume air thermometer
(b) Platinum resistance thermometer

in terms of that emitted from a reference body (called 5. When a body A T1 is sur-
the black body) at the same temperature. A black body rounded by another body B at a lower temperature
T2, then the rate of loss of heat from body A will be
proportional to
E emitted by a unit area of a black body
(a) T 41 (b) (T1 – T2)4
(c) (T1 – T2) (d) (T 41 – T 42)
E = T4
6. -
where T is the absolute temperature of the body and is pends upon
a constant known as Stefan’s constant. If the body is not a (a) the surface area of the body
perfect black body, then (b) the temperature of the body
E= T4 (c) the nature of the surface of the body
where is the emissivity of the body. (d) the emissivity of the surface of the body
ANSWERS AND SOLUTIONS
1. E= T4 where E 2.
3.
E = dimensions of 4.
ML 2 2 5.
–3
= 2 6. All the four choices are correct.
area time
ML 3 –3
Dimensions of = K–4, which is
choice (d). K4
19.20 Comprehensive Physics—JEE Advanced

Questions 7 to 13 are based on the following passage 9.


Passage II revealed that the atmospheres of stars contain
Stellar Spectra
Like the solar spectrum, the spectra of stars show a (c) uranium (d) helium
continuous spectrum on which dark aborption lines are 10.
photosphere) of (a) surface temperature
(b) mass

the outer, relatively cooler, layer of the star, the radiations (d) all the above factors
11. Wien’s displacement law tells us that an extremely
hot star should look

in the outer layer of the star.


T of a star can be estimated by 12. In Wien’s displacement law the SI unit of Wien’s
constant b is
m at which the intensity of
(a) metre per kelvin
displacement law which states that (b) metre per kelvin squared
(c) metre kelvin
m T=b
(d) metre kelvin squared
where b is a constant called Wien’s constant and the above
relation is called Wien’s Displacement Law which states 13.
that as the temperature increases, the maximum intensity E of
of emission shifts (or is displaced) towards the shorter black body radiations where
b has been found
10–3 mK.
7.
(a) continuous emission spectrum
(b) emission line spectrum
(c) emission band spectrum
(d) absorption line spectrum
8.

the outer layers of the sun

(d) destructive interference between waves of


Fig. 19.18

SOLUTION
7. -
8.
9.
12. From m T = b, the SI unit of b = SI unit of m SI
10.
unit of T = metre kelvin, which is choice (c).
11. m T = constant, if T
13.
m

Questions 14 to 16 are based on the following passage


Passage III
A, B and C Fig. 19.19
equal diameters are joined in series as shown in the fol- In the steady state, the free ends of rods A and C are at
k, k and
0.5 k curved surfaces of rods.
19.21

14. A and 16. -


B is nation is
(a) 55.7°C (b) 65.7°C 7k 2k
(c) 75.7°C (d) 85.7°C (a) (b)
3 7
15. B and
C is 5k 3k
(c) (d)
(a) 57.1°C (b) 47.1°C 3 5
(c) 37.1°C (d) 27.1°C
SOLUTION
14. T1 = 85.7°C. So the cor-
same for all rods. If T1 and T2 are the temperatures rect choice is (d).
at the junction points between A and B and between
15. T1
and C respectively, then
T2 = 57.1°C, which is choice (a).
Q k A(100 T1 ) 16.
= A
t d diameters, the equivalent thermal conductivity of
k B A T1 T2 k A T2 0
= = C
d d 1 1 1 1 1 1 1
= =
Given kA = 2k, kB = k and kC = 0.5k ke kA kB kC 2k k 0.5k
2(100 – T1)= (T1 – T2) = 0.5(T2 – 0)
2k
T1 = T1 – T2 (1) ke = . So the correct choice is (b).
7
and T1 – T2 = 0.5 T2 (2)

Questions 17 to 19 are based on the following passage 17.


Passage IV (a) 26.5°C (b) 25.5°C
(c) 24.5°C (d) 23.5°C
18.
area 1 m2 and thickness 0.01 m separated by 0.05 m thick (a) 2.5°C (b) 2.0°C
(c) 1.5°C (d) 0.5°C
19.
is nearly equal to
–1 –1
K . (a) 1000 J s–1 (b) 2000 J s–1
(c) 3000 J s–1 (d) 4000 J s–1

SOLUTION
17. T2 and T3 be the temperature
dQ1 k g A(T1 T2 ) 0.8 1 (300 T2 )
= = (1)
dt dg 0.01
dQ2 k A(T2 T3 ) 0.08 1 (T2 T3 )
= a = (2)
dt da 0.05
dQ3 k g A(T3 T4 ) 0.8 1(T3 273)
= = (3)
dt dg 0.01
In the steady,
dQ1 dQ2 dQ3
= =
dt dt dt

Fig. 19.20 50 (300 – T2) = T2 – T3 (4)


19.22 Comprehensive Physics—JEE Advanced

19. T2 = 273.5 K and T3


(300 – T2) = T3 – 273 (5)
T3 26.5°C. dQ1
2000 J s–1, which is choice (b).
So the correct choice is (a). dt
18. T3
T2 = 273.5 K = 0.5°C, which is choice (d).

IV

Assertion-Reason Type Questions


Statement 2

convection.
correct. 4. Statement 1
(a) Statement-1 is true, Statement-2 is true and State- -
ment-2 is the correct explanation for Statement-1. uid.
(b) Statement-1 is true, Statement-2 is true but State- Statement 2
ment-2 is not the correct explanation for State-
ment-1.
(c) Statement-1 is true, Statement-2 is false. 5. Statement 1
(d) Statement-1is false, Statement-2 is true.
1. Statement 1 -
netic waves.
-
tion. Statement 2
Statement 2 -
e) and absorptive
power (a 6. Statement 1
temperature and for radiation of the same wave-

2. Statement 1 and 4 m and termperatures 4000 K and 1000 K

two spheres will be the same.


same (room) temperature. Statement 2
Statement 2
- is directly proportional to its absolute temperature
tivity than plastic. and inversely proportional to its surface area.
3. Statement 1
If the earth did not have an atmosphere, it would
become extremely cold.

SOLUTION
1. Conversely, if a body is a poor emitter of a radia-
law,
e
= constant
a 2.
e a temperature is lower than our body temperature.
- Since metal is a better conductor of heat than plas-
tic, when we touch the metal cap and the plastic
19.23

4.
the metal cap much more quickly than to the plastic 5.
body. 6.
3.
E = T 4A = T 4 4 R2; R = radius of sphere
is a poor conductor of heat, the atmosphere acts
as a blanket for the earth and keeps the earth warn

Integer Answer Type


1. A is 3 times that of B. In
rod, end to end, with a second rod of a different the steady state, the temperature difference across
material but of the same cross-section. At 25°C, the the wall is 36°C. Find the temperature difference
(in °C) across the layer A.

3. A and B have thermal emissivities of


= 1.7
10–5 per °C and that of the second rod is =n of the two bodies are the same. If they emit total ra-
10–5 per °C. Find the value of n.
ratio TA/TB.
2. A wall has two layers A and B each made of differ-
ent material. Both layers have the same thickness.

SOLUTIONS
1. Given T1 – T2 = 36 T2 = T1 – 36 (2)
T1 – T0
= 30 (1 + 1.7 10–5 100)
= 30.051 cm

100
= 7000 cm
cm =
=2 10–5 per °C.
n = 2.
2. Fig. 19.21
layers A and B is the same, i.e. (a = area of cross- 3. E1 = eA A TA4,
E2 = eB A TB4
Given E1 = E2
kA a (T1 T0 ) k a(T0 T2 ) 1/4 1/4
= B TA e 0.81
x x = B = =3
TB TA 0.01
kA = 3 kB
3(T1 – T0) = T0 – T2 (1)
Electrostatic Field and Potential 20.1

20
Chapter
Electrostatic Field
and Potential

REVIEW OF BASIC CONCEPTS Case (a): Unlike charges (q1 q2 < 0) [Fig. 20.1(a)]
Force exerted on q2 by q1 is
20.1 COULOMB’S LAW
q1 q2 n
F12 =
On the basis of his measurements, Coulomb arrived at 4 0 r2
a law, known after his name as Coulomb’s law, which
states that the magnitude of the electric force between where n is a unit vector directed from q1 to q2.
two charges is directly proportional to the product of the Force exerted by q1 on q2 is
magnitudes of charges and inversely proportional to the q1 q2 n
square of the distance between them, i.e. F21 = –
4 0 r2
q1 q2
F Case (b): Like charges (q1 q2 > 0) [Fig. 20.1(b)]
r2
q1 q2 n
q1 q2 F12 = –
or F= k 4 r2
r2 0

In the SI system, k is written as 1/4 0 where 0 is called q1 q2 n


the permittivity of vacuum and its value is F21 =
4 0 r2
0 = 8.854 10–12 C 2 N–1 m–2
1 RELATIVE PERMITTIVITY (OR DIELECTRIC
Then k= 9 109 Nm2 C–2 20.2 CONSTANT)
4 0

The force F is attractive for unlike charges (q1 q2 < 0) and


repulsive for like charges (q1 q2 > 0). the permittivity of the medium to permittivity of vacuum,
Coulomb’s Law in vector form i.e.

r =
0
r is also called the dielectric constant (K) of the medium.

Thus K = or =K 0 K for air = 1. If


0
charges q1 and q2 are situated in a medium other than air
or vacuum, the magnitude of force between them is given
by
q1 q2 q1 q2
F= 2
Fig. 20.1 4 r 4 0 Kr 2
20.2 Comprehensive Physics—JEE Advanced

20.3 PRINCIPLE OF SUPERPOSITION SOLUTION


If many charges are present, the total force on a given A system of charges is in equilibrium if no charge of
charge is equal to the vector sum of the individual forces the system experiences any net force.
exerted on it by all other charges taken one at a time.

20.1
Two point charges q1 = +9 C and q2 = –1 C are
held 10 cm apart. Where should at third charge +Q be
placed from q2 on the line joining them so that charge
Fig. 20.3
Q does not experience any net force?
Equilibrium of charge Q at O [See Fig. 20.3]
SOLUTION Since Q is at the same distance from equal charges
Charge Q will not experience any net force if the forc- q1 and q2, it will be equilibrium for any positive or
es exerted on it by charges q1 and q2 are equal and in negative value, because it will experience equal and
opposite directions. opposite forces.
Equilibrium of charge q1 at A
If Q is negative, it will repel q1. Also q2 is repel q1.
Hence q1 cannot be in equilibrium if Q is negative.
So Q must be positive.
Fig. 20.2 Force exerted on q1 by Q is
It follows from Fig. 20.2 that charge Q will not 4 10 6
Qi
experience forces in opposite direction if it lies at any F= 2
r
point between AB. Let x be the distance of Q from q2. 4 0
Then forces exerted on Q by q1 and q2 respectively 2
are Force exerted on q1 by q2 is
6 6 6
q1 Q i 9 10 Qi 4 10 4 10 i
F1 = 2
= 2 F =–
4 0 (0.1 x) 4 0 (0.1 x) 4 0r 2

6
Net force on q1 will be zero if F + F = 0, i.e. if
q2 Q i 1 10 Qi
and F2 = – 6 6
4 x 2
4 x2 4 10 6
Qi 4 10 4 10 i
0 0
2
– 2
=0
Net force on Q = F1 + F2 r 4 0 r
4 0
Net force on Q = 0 if F1 + F2 = 0 2
Q= 1 10–6 C = 1 C
6 6
9 10 Qi 1 10 Qi
2
– =0 It is easy to check that charge q2 will also be in equi-
4 0 (0.1 x) 4 0 x2 librium. Hence the system of three charges will be in
(0.1 x) 2 equilibrium if Q = +1 C.
9=
x2 20.3
0.1 x Four point charges, each equal to q = 4 C, are held
3=
x at the corners of a square ABCD of side a = 10 cm.
x = 0.05 m = 5 cm Find the magnitude and sign of a charge Q placed at
the centre of the square so that the system of charges
20.2 is in equilibrium.
Two charges, each equal to – 4 C, are held a certain
SOLUTION
distance apart. A charge Q is placed exactly mid-way
between them. Find the magnitude and sign of Q so AC (= r) = 2a . Let us consider the equilibrium of
that the system of three charges is in equilibrium. charge q at A (Fig. 20.4)
Electrostatic Field and Potential 20.3

kq q
i.e. if 2
q 2Q =0
a 2 2
q
Q = – (1 2 2 )
4
4 C
=– (1 2 2 ) = – (1 2 2 ) C
4

20.4 ELECTRIC FIELD

Fig. 20.4 positive point charge q0 at the point in space where the

Force exerted on charge at A by charge at B is force F E is then given by


2
kq 1 F
FAB = 2
i, where k = E=
a 4 o q0
Similarly If a charge q
kq 2 due to other charge or charges is E, then the charge q will
FAD = j experience a force F given by
a2
F = qE
kq 2 (1) Electric
FAC = (cos 45 i sin 45 j)
(a 2 )2 r from a source charge q is given by

kq 2 1 q
= (i j) E=
a2 2 2
4 0 r2
For a positive charge (+q), vector E is directed radially
kqQ
and FOA = (cos 45 i sin 45 j) outwards from it and for a negative charge (–q), E is
2
a E is
2
k qQ
= 2 ( i j) individual charges.
a2
(2) A pair of
Net force on charge q at A in the x-direction is equal and opposite point charges separated by a certain
kq 2 kq 2 2 k qQ distance is called an electric dipole.
Fx = 2 2
i
a a 2 2 a2
Let 2a be the separation between point charges –q and +q
kq q (Fig. 20.5).
Fx = q 2Q i i
a2 2 2
kq q
where = 2
q 2Q
a 2 2
Similarly net force on charge q at A in the y-direction is

Fy = j Fig. 20.5

Resultant force on charge q at A is P due to +q and –q respectively are


F= Fx2 Fy2 2 2
2
qi
E+ =
Charge q will be in equilibrium if F = 0 i.e. if 4 a)2
0 (r
=0
20.4 Comprehensive Physics—JEE Advanced

NOTE
qi
E– = –
4 0 (r a)2
line of a dipole is along the dipole moment.
P is -
Ea = E+ + E– torial line of a dipole is antiparallel to the dipole mo-
2q (2a)r ment.
= 2
4 0 (r a 2 )2
2p r A conducting rod AB of negligible thickness and
= 2 length L carries a charge a charge Q uniformly distributed
4 0 (r a 2 )2
P at a distance a
where p = q(2a) is the dipole moment and 2a is the vector from end B (Fig. 20.7), we consider a small element of
distance between charges –q and +q. Dipole moment p is length dx of the rod located at a distance x from P.
a vector quantity directed from –q to +q.
For a very short dipole (a << r)
2p
Ea = 3
4 0r

Fig. 20.7
bisector (equatorial plane) of a dipole
Q Q
Charge of element is dq = dx = dx where = is
L L
P due
to the element is
dq dx
dE = 2 2
4 0x 4 0x
(L a)
dx
Hence E= dE
4 0 a x2

1 (L a)
=
Fig. 20.6 4 0 xa
Q due to +q and –q are (see Fig. 1 1
20.6) E=
4 0 (L a) a
q q
E+ = 2
and E– = 2
4 0x 4 0x E=
L
Q is 4 0 a( L a)
Ee = E+ cos + E– cos If Q is positive, E is directed from left to right.
a
Using cos = and x = r 2 a 2 , we get
x Consider a ring of
q ( 2a ) radius R carrying a chare Q distributed uniformly on it.
Ee = directed from Q to R
4 0 (r 2 a 2 )3 /2 P on its axis at a distance x
In vector form from the centre O, consider an element of length dl (Fig.
p 20.8). The charge of the element is
Ee = – Q dl
4 0 (r 2 a 2 )3 /2 dq =
2 R
For a very short dipole (a << r) A is dE given by
p dq
Ee = – 3 dE =
4 0r 4 0r 2
Electrostatic Field and Potential 20.5

Fig. 20.9

Properties of Electric Field Lines

Fig. 20.8
(ii) Field lines originate from a positive charge and
P due to element a terminate on a negative charge.
diametrically opposite point B. The x components of
y (iv) Field lines are closer together in the region where
components cancel. Hence
E= dE cos
dq cos on a charge is proportional to the magnitude of
= 2 the charge.
4 0r
Q 1 x NOTE
= 2 2 3 /2
dl
2 R 4 0 (R x )
a closed loop. But if the electric is induced by a time-
1 Qx
E=
4 0 (R2 x 2 )3 /2
20.6 ELECTRIC FLUX
dl 2 R
The direction of E is from O to P if charge Q is positive.
through the surface.
20.5 ELECTRIC FIELD LINES For a plane surface of surface area S
E
= E S = ES cos
the tangent to which at a point gives the direction of the where S is called the area vector, its magnitude is S and its
direction is normal to the surface and away from it. Angle
is the angle between E and S.
charge distributions. For a curved surface,
= E dS (E n) dS

where n is a unit outward


normal to the surface. dS is the
surface area of an element of Fig. 20.10
the surface (Fig. 20.10). The SI
–1 2
m or Vm (volt metre).

20.7 GAUSS’S LAW IN ELECTROSTATICS

q
surface S E is equal to , where q is
0
the net charge enclosed in the surface and 0 is electrical
permittivity of vacuum.
20.6 Comprehensive Physics—JEE Advanced

q
E dS = (1) r from a spherical shell of
S 0
radius R carrying a surface charge density (= q/4 R2)
Gauss’s law is used to obtain the expression for the electric is given by
qn R2 n
which are uniform and symmetric so that a proper and E= 2
= 2
(for r > R)
convenient closed surface (called the Gaussian surface) 4 0r 0r
can be chosen to evaluate the surface integral in Eq. (1). n
= (for r = R)
Some Important Points about Gauss’s Law 0
(1) Gauss’s law holds for any closed surface of any = zero (for r < R)
shape or size.
(2) The surface that we choose to evaluate electric where n is a unit vector pointing radially outwards if q is
positive and inwards if q is negative.
(1)] is called Gaussian surface.
(3) If the Gaussian surface is so chosen that there are 20.9 ELECTRIC POTENTIAL
some charges outside and some inside the surface,
then charge q on the R.H.S. of Eq. (1) is the net
charge (taking into account the sign of charges) is the work per unit charge that is done to bring a small
E on
W
the charges both inside and outside the surface. V = lim
q0 0 q0
(4) The exact location of charges inside Gaussian
surface does not affect the value of the electric
Electric potential at a point P
(5) If Coulomb’s law did not hold, Gauss’s law also charge is given by
would not hold. 1 q
V=
4 0 r
20.8 APPLICATIONS OF GAUSS’S LAW
where r is the distance of the point P from the charge. This
potential is spherically symmetric around the point, i.e. it
- depends only on r for a given charge q. Since potential is
dicular distance r a scalar function, the spherical symmetry means that the
or wire carrying a uniform linear charge density is potential at a point does not depend upon the direction of
given by that point with respect to the point charge; it only depends
on the distance of the point from the charge.
n Notice that the potential due to a positive charge
E=
2 0r (q > 0) is positive, it is negative in the neighbourhood of
where = q /L is the charge per unit length of the rod an isolated negative charge (q < 0).
and n is a unit vector pointing away from the rod if q is
positive and towords it if q is negative.
Electric
r to each charge, assuming that all other charges are absent,
and then simply add these individual contributions. Since,
is given by
addition here is the ordinary sum, not a vector sum.
n The potential at any point due to two point charges q1
E= and q2 is, therefore, simply given by
2 0
where = q / A is charge per unit area and n is a unit 1 q1 q2
V=
vector pointing away from the sheet if q is positive and 4 0 r1 r2
towards it if q is negative. Notice that E is independent of where r1 and r2 are the distances of the point in the question
r, the distance from the sheet. from charges q1 and q2 respectively.
Electrostatic Field and Potential 20.7

The charge on an electron is 1.6 10–19 C,


potential at any point due to a system of N point charges 1 eV = 1.6 10–19 J
in given by
1 N
qn POTENTIAL ENERGY OF AN ELECTRIC
V = V1 + V2 + + VN = 20.13
4 0 n 1 rn DIPOLE IN AN EXTERNAL ELECTRIC FIELD

20.10 RELATION BETWEEN E AND V

means that the potential decreases along the direction of

dV
E =–
dr
20.11 ELECTRIC POTENTIAL ENERGY
The electric potential energy of a system of point charges

Fig. 20.12
We assume that the charges were at rest when
E, as
kinetic energy. shown in Fig. 20.12, it experiences a torque given by
The electric potential energy of two point charges q1 = p E sin
and q2 separated by a distance r12 as shown in Fig. 20.11
(a) is given by where is the angle between the line joining the two
1 q1 q2
U12 = =p E
4 0 r12
The torque tends to rotate the dipole to a position where
= 0, i.e, p is parallel to E.
The electric potential energy of a dipole is
U =– p E

20.14 ADDITIONAL USEFUL FORMULAE

Fig. 20.11
(i) Charge q at each vertex of an equilateral triangle
The electric potential energy of a system of three point of side a (Fig. 20.13).
charges as shown in Fig. 20.11 (b) is given by
U = U12 + U23 + U31
1 q1 q2 q2 q3 q1 q3
=
4 0 r12 r23 r13
This expression can be generalized for any number of
charges.

20.12 THE ELECTRON-VOLT Fig. 20.13

The SI unit of potential energy is the joule. In atomic 3q


At centroid O, E0 = 0 and V0 =
physics a more convenient unit called the electron-volt 4 0r
(written as eV) is used. An electron-volt is the potential a
energy gained or lost by an electron in moving through where r = .
Since the magnitude of 3
20.8 Comprehensive Physics—JEE Advanced

(ii) Charge q at each vertex of a square of side a q 1 1


(Fig. 20.14). = 2
1
4 0r 4 16
q 1 q
=
4 0r
2 1 3 0r
2
1
4
Potential at O is
q 1 1
V0 = 1
4 0r 2 4
Fig. 20.14
q 1 q
=
4q a 4 0r
1 2 0r
At center C, Ec = 0 and Vc = ; r 1
4 0r 2 2

NOTE (vi) A short electric dipole of dipole moment p (Fig.


20.17)
In the above two cases, if one of the charges is
O and C is
E = q/4 or2, directed towards the empty vertex.

(iii) For Fig. 20.15,

Fig. 20.17
Fig. 20.15
2 p cos p sin
At point A, Er = 3
and E = 3
2 2q 4 0r 4 0r
Vc = 0 and Ec = 2
4 0r
A is EA = Er2 E2
(iv) For Fig. 20.16,
p
= 3
3 cos 2 1 1 /2
4 0r

1
Also tan = tan . Angle between and EA is
2
( + ).
2p
At point B on axial line, EB = 3
( = 0°)
Fig. 20.16
4 0x
p
At point C on equatorial line, EC =
Vc = 0 4 3
0y
( = 90°)
Ec = 0
p cos
q, placed Electric potential at A is VA = 2
4 0r
on the x-axis at x = r, x = 2r, x = 4r… and so
O is p
At point B; VB =
q 1 1 1 4 0r
2
E0 = 2 2 2
4 0r 1 2 42 At point C; VC = 0
Electrostatic Field and Potential 20.9

Q 1
Q At point P, EP =
(Linear charge density ) = 4 0 r (r L)
L
(i) Charge Q distributed uniformly on a rod of length Q r L
L (Fig. 20.18) VP = log e
4 0L r
(ii) Charge Q distributed uniformly on a conducting
sphere or shell of radius R (Fig. 20.19) (Surface
Q
charge density = )
4 R2

Fig. 20.18

Fig. 20.19

At point C outside the sphere or shell (r > R), Q


At centre O, V0 =
EC =
Q 4 0 R 4 0
2
4 0r
At point B just outside the surface (r = R),
Q
EB =
4 0 R2
At point A inside the sphere or shell (r < R),
EA = 0
Potential at center O of sphere or shell,
Q
V0 =
4 0R Fig. 20.20

At points inside (r < R), VA =


Q
= VB (at (iv) Charge Q distributed uniformly on a ring of
surface) 4 0R radius R P
on the axis is
Q
At point C outside (r > R), VC =
4 0R
(iii) Charge Q distributed uniformly on a semicircu-
lar wire of radius R (Fig. 20.20) (Linear charge
Q
density = )
R

At center O, E0 = along negative y-direc- Fig. 20.21


tion 4 0R
Q r
=
Q EP = 3 /2
2 4 R2 r2
4 0 R2 0
20.10 Comprehensive Physics—JEE Advanced

R (ii) Charge Q kept at each vertex


E is maximum at r = and of a square of side a (Fig.
2
20.23)
1 2Q
Emax = 4 Q2 2Q 2
4 0 3 3R 2 U=
4 0a 4 0 (a 2)
Electric potential at point P is
Q 1 Q2
VP = = (4 2) Fig. 20.23
4 0 1 /2 4 0a
R2 r 2
(iii) An electric dipole of dipole moment p placed in
V is maximum at r = 0 (i.e. at centre O) and
E with angle between p and
Q
Vmax = E. Torque = p and potential energy U = – p.E.
4 0R The zero of potential energy is taken at = 90°.
(a) When = 0°, = 0, U = minimum = –pE
(stable equilibrium)
(i) Charge Q kept at each
(b) When = 90°, = maximum = pE, U = 0
vertex of an equilateral of
side a. Potential energy is (c) When = 180°, = 0, U = maximum = pE
(Fig. 20.22). (unstable equilibrium)
(d) Work done in turning a dipole from angle 1
3Q 2 to angle 2 is
U=
4 0 a W = pE (cos 1 – cos 2)
Fig. 20.22
If 1 = 0° and 2 = 180°, W = 2pE

Multiple Choice Questions with Only One Choice Correct


1. Three point charges Q, – 2Q and – 2Q are placed at Q Q
the vertices of an equilateral triangle of side r. The (a) (b)
4 0 R 4 0 (2 R)
work done to increase their separation to 2 r is
Q2 3Q
(a) zero (b) (c) (d) zero
4 r 4 0 (2 R)
0

2 Q2 2 Q2 4. A metal sphere of radius R carries a charge Q. The


(c) (d) E and the electric po-
4 0 r 4 0r
tential is V. If R is doubled keeping Q the same, the
2. A point charge Q is placed at point P at a distance new values of E and V will be
R from the centre O of a metallic spherical shell of
inner radius 2R and outer radius 2.5 R. The electric E V E V
(a) and (b) and
potential at the centre of the shell will be 4 2 2 4
Q 1 5Q (c) 4E and 2V (d) 2E and 4V
(a) (b)
4 0 R 4 0 6R 5. A metal sphere of radius R has surface charge den-
sity E and the
1 9Q electric potential is V. If R is halved, keeping the
(c) zero (d)
4 0 10 R same, the new values of E and V will be
3. Two concentric metallic shells of radii R and 2R (a) 4E and 2V (b) 2E and 4V
are given charges Q and 2Q respectively. If the two V E
shells are connected by a metallic wire, the change (c) E and (d) and V
in electric potential on the outer shell is 2 2
Electrostatic Field and Potential 20.11

6. brought in contact with sphere B and removed. What


at a point P on its axial line at a distance r from it is the new force of repulsion between A and B?
is EP and at a point Q on its equatorial line at a 3F
(a) F (b)
distance 2 r from it is EQ . Then 8
F F
(a) EP = – 16 EQ (b) EP = –8 EQ (c) (d)
2 4
(c) EP = 8 EQ (d) EP = – EQ 13. Two small identical balls P and Q, each of mass
7. A hollow metal sphere is charged such that the po- 3 /10 gram, carry identical charges and are sus-
tential at its centre is V. The potential on the surface pended by threads of equal lengths. At equilibrium,
of the sphere is they position themselves as shown in Fig. 20.24.
1
(a) zero (b) V What is the charge on each ball. Given =
4 0
(c) more than V (d) less than V 9 2 –2
9 10 Nm C and take g = 10 ms . –2

8. The work done in carrying a charge q once round a (a) 10 –3 C (b) 10–5 C
–7
circle with a charge Q at the centre is W1. The work (c) 10 C (d) 10–9 C
done is W2 if charge q is moved from one end of a
diameter to the other. Then
(a) W1 > W2 (b) W1 < W2
(c) W1 = W2 0 (d) W1 = W2 = 0
9. Three point charges 4q, Q and q are placed in a
straight line of length l at points distant 0, l/2 and
l respectively. If the net force on charge q is zero,
the magnitude of the force on charge 4q is
q2 2 q2
(a) (b)
0 l2 0 l2 Fig. 20.24
14. Two point charges q1 = 2 C and q2 = 1 C are
3 q2 4 q2 placed at distances b = 1 cm and a = 2 cm from the
(c) (d)
l2
0 0 l
2
origin on the y and x axes as shown in Fig. 20.25.
10. A charge q is placed at the centre of the line join- P (a, b) will sub-
ing two equal charges Q. The system of the three tend an angle with the x-axis given by
charges will be in equilibrium if q is equal to (a) tan = 1 (b) tan = 2
Q Q (c) tan = 3 (d) tan = 4
(a) – (b) –
2 4
Q Q
(c) + (d) +
2 4
IIT, 1987
11. One thousand spherical water droplets, each of
radius r and each carrying a charge q, coalesce to
form a single spherical drop. If v is the electrical
potential of each droplet and V that of the bigger
drop, then Fig. 20.25
V 1 V 1 15. An electric dipole placed with its axis in the direc-
(a) = (b) =
v 1000 v 100
V V (a) a force but no torque
(c) = 100 (d) = 1000
v v (b) a torque but no force
12. Two metallic identical spheres A and B carrying (c) a force as well as a torque
equal positive charge + q are a certain distance (d) neither a force nor a torque.
apart. The force of repulsion between them is F. A 16. An electric dipole placed with its axis inclined at
third uncharged sphere of the same size is brought
in contact with sphere A and removed. It is then experiences
20.12 Comprehensive Physics—JEE Advanced

(a) a force but no torque 23. In a hydrogen atom, the electron and the proton
(b) a torque but no force are bound together at a separation of about 0.53 Å.
-
(c) a force as well as a torque
nite separation of the electron from the proton, the
(d) neither a force nor a torque potential energy of the electron-proton system is
17. An electric dipole placed in a non-uniform electric (a) – 54.4 eV (b) – 27.2 eV
(c) – 13.6 eV (d) zero
(a) a force but no torque
24. A positive charge (+ q) is located at the centre of
(b) a torque but no force
a circle as shown in Fig. 20.26. W1 is the work
(c) a force as well as a torque
done in taking a unit positive
(d) neither a force nor a torque. charge from A to B and W2
18. A cube of side b has a charge q at each of its is the work done in taking
vertices. What is the electric potential at the centre the same charge from A to C.
of the cube? Then
4q 3q
(a) (b) (a) W1 > W2
3 0b 0b (b) W1 < W2
Fig. 20.26
2q (c) W1 = W2
(c) (d) zero
0b (d) W1 = W2 = 0
19. 25. Two concentric spheres of radii r1 and r2 carry
4q 3q charges q1 and q2 respectively. If the surface charge
(a) (b)
3 2 2 density ( ) is the same for both spheres, the electric
0b 0b
potential at the common centre will be
2q
(c) (d) zero r1 r2
2 (a) (b)
0b r2 r1
0 0
20. Two point charges – q and + q are located at points
(0, 0, – a) and (0, 0, a) respectively. What is the
(c) (r1 – r2) (d) (r1 + r2)
electric potential at point (0, 0, z)? 0 0
qa q 26.
(a) 2
(b)
4 0z 4 0a a sphere of radius r having a uniform surface charge
density is
2qa 2qa
(c) (d)
4 0 z2 a2 4 z2 a2 (a) (b)
0 2
0 0
21. In Q. 20, how much work is done in moving a
small test charge q0 from point (5, 0, 0) to a point (c) (d)
(– 7, 0, 0) along the x-axis? 0r 2 0r
5 q0 q 7 q0 q 27. Two equal negative charges – q
(a) (b)
7 4 0 a 5 4 0 a (0, a) and (0, – a). A positive charge + Q is released
from rest at the point (2a, 0) on the x-axis. The
2 q0 q charge Q will
(c) (d) zero
12 4 0 a (a) execute SHM about the origin
22. A neutral hydrogen molecule has two protons and (b) move to the origin and remain at rest there.
two electrons. If one of the electrons is removed
we get a hydrogen molecular ion (H +2). In the (d) execute oscillations but not SHM.
ground state of H +2 the two protons are separated IIT, 1985
by roughly 1.5 Å and the electron is roughly 1 Å
28. Four charges q, 2q, 3q and 4q are placed at corners
from each proton. What is the potential energy of
the system? A, B, C and D of a square as shown in Fig. 20.27.
P of the square has the direction
(a) – 38.4 eV (b) – 19.2 eV
along
(c) – 9.6 eV (d) zero
Electrostatic Field and Potential 20.13

(a) zero
q
(b) (Q1 Q2 ) ( 2 1)
4 0 2R

q 2
(c) (Q1 + Q2)
4 0R
2 1 q Q1 Q2
(d)
24 0 R
Fig. 20.27
IIT, 1992
(a) AB (b) CB 34. An electron of mass me, initially at rest, moves
(c) AC (d) DB
29. Particle A has a charge + q and particle B has a in time t1. A proton of mass mp, also initially at rest,
charge + 4q, each having the same mass m. When takes time t2 to move through an equal distance in
allowed to fall from rest through the same potential
difference, the ratio of their speeds vA/vB will be gravity, the ratio t2/t1 is nearly equal to
(a) 2 : 1 (b) 1 : 2
(a) 1 (b) (mp/me)1/2
(c) 1 : 4 (d) 4 : 1
30. Four equal charges Q are placed at the four corners (c) (me/mp)1/2 (d) 1836
of a square of side a. The work done in removing a IIT, 1997
charge – Q 35. A nonconducting ring of radius 0.5 m carries a total
is charge of 1.11 10–10 C distributed non-uniformly
2 Q2 E
(a) zero (b)
4 0a everywhere in space. The value of the line integral
l 0
– E dl (l = 0 being centre of the ring) in
2 Q2 Q2 l
(c) (d) volts is
0a 2 0 a
31. A point charge + q is placed at the mid point of a (a) + 2 (b) – 1
cube of side L (c) – 2 (d) zero
cube is IIT, 1997
q
(a) (b) zero 36. A metallic solid sphere is placed in a uniform
0

6 q L2 q lines of force follow?


(c) (d) 2 (a) 1 (b) 2
6L
0 0 (c) 3 (d) 4
32. A solid conducting sphere having a charge Q is IIT, 1996
surrounded by an uncharged concentric conducting
hollow spherical shell. The potential difference be-
tween the surface of the solid sphere and the outer
surface of the hollow shell is V. If the shell is now
given a charge of – 3Q, the new potential difference
between the same two surfaces is Fig. 20.28
(a) V (b) 2 V
(c) 4 V (d) – 2 V 37. A charge + q x = x 0,
x = 3 x0, x = 5x0 q is
IIT, 1989 x = 2x0, x = 4x0, x = 6x0
33. Two identical thin rings, each of radius R are x0 is a positive constant. The
coaxially placed a distance R apart. If Q1 and Q2 are potential at the origin to this system of charges is
respectively the charges uniformly spread on the
q
two rings, the work done in moving a charge q from (a) zero (b)
the centre of one ring to the centre of the other is 4 0 x0 ln 2
20.14 Comprehensive Physics—JEE Advanced

q ln 2 43. Two identical charges are placed at the two cor-


ners of an equilateral triangle. The potential energy
4 0 x0
of the system is U. The work done in bringing an
IIT, 1998
38. Three charges Q, + q and + q are placed at the (a) U (b) 2 U
vertices of a right-angled isosceles triangle as (c) 3 U (d) 4 U
shown in Fig. 20.29. The net electrostatic energy 44. The magnitude of electric intensity at a distance x
Q is equal to from a charge q is E. An identical charge is placed
q 2q at a distance 2x from it. Then the magnitude of the
(a) (b) force it experiences is
1 2 2 2
(a) qE (b) 2qE
(c) – 2q (d) + q
qE qE
IIT, 2000 (c) (d)
2 4
45. A particle carrying a charge q is shot with a speed
v Q. It
approaches Q up to a certain distance r and then
returns as shown in Fig. 20.30.

Fig. 20.29
Fig. 20.30
39. Eight dipoles of charges of magnitude e are placed
If charge q were moving with a speed 2v, the
distance of the closest approach would be
the cube will be
(a) r (b) 2r
8e 16 e
(a) (b) r r
0 0 (c) (d)
2 4
e
(c) (d) zero 46. Four charges, each equal to – Q, are placed at the
0 corners of a square and a charge + q is placed at its
40. A point Q lies on the perpendicular bisector of an centre. If the system is in equilibrium, the value of
electrical dipole of dipole moment p. If the distance q is
of Q from the dipole is r (much larger than the size
Q is propor- Q Q
(a) 1 2 2 (b) 1 2 2
tional to 4 4
(a) p–1 and r–1 (b) p and r–2
2 –3 Q Q
(c) p and r (d) p and r–3 (c) 1 2 2 (d) 1 2 2
2 2
41. A particle of mass m and charge q is released
E. The kinetic 47. A charge having magnitude Q is divided into two
energy attained by the particle after moving a parts q and (Q – q) which are held a certain dis-
distance x is tance r apart. The force of repulsion between the
(a) qEx2 (b) qE 2x two parts will be maximum if the ratio q/Q is
(c) qEx (d) q2Ex 1 1
3 (a) (b)
42. Vm–1 along 2 3
the y-axis. A body of mass 1 g and charge 10–6 C
1 1
–1
(c) (d)
positive x-axis with a velocity of 10 ms . Its speed (in 4 5
ms–1) after 10 second will be (neglect gravitation) 48. A charge Q is given to a hollow metallic sphere of
(a) 10 (b) 5 2 radius R. The electric potential at the surface of the
(c) 10 2 (d) 20 sphere is
Electrostatic Field and Potential 20.15

1 Q 55. Three positive charges of equal value q, are placed


(a) zero (b) at the vertices of an equilateral triangle. The result-
4 0 R
ing lines of force should be sketched as in (see Fig.
1 Q
(c) (d) 4 0 Q/R 20.31).
4 0 R2
49. In Q. 48, the potential at a distance r from the
centre of the sphere where r < R is
1 Q
(a) zero (b)
4 0 R r

1 Q 4 0Q
(c) (d)
4 0 R r R r
50. The electric potential V at any point (x, y, z) in
space is given by V = 4x2 volt where x, y and z are

2 m) in Vm–1 is
(a) 8 along negative x-axis
(b) 8 along positive x-axis
Fig. 20.31
(c) 16 along negative x-axis
IIT, 2001
(d) 16 along positive x-axis
IIT, 1992 V
56. A quantity X is given by 0L where 0 is the
51. Two spheres of radii r and R carry charges q and Q t
permittivity of free space, L is a length, V is a
respectively. When they are connected by a wire,
potential difference and t is a time interval. The
there will be no loss of energy of the system if
dimensional formula for X is the same as that of
(a) qr = QR (b) qR = Qr (a) resistance (b) charge
(c) qr2 = QR 2 (d) qR 2 = Qr 2 (c) voltage (d) current
52. Two equal point charges of 1 C each are located IIT, 2001
at points ( i j k ) m and (2 i 3 j k ) m. What 57. A uniform electric field pointing in positive
is the magnitude of electrostatic force between x-direction exists in a region. Let A be the origin,
them? B be the point on the x-axis at x = + 1 cm and C
(a) 10–3 N (b) 10–6 N be the point on the y-axis at y = + 1 cm. Then the
potentials at the points A, B and C satisfy:
(c) 10–9 N (d) 10–12 N
53. Three equal point charges q are placed at the cor- (a) VA < VB (b) VA > VB
ners of an equilateral triangle. Another charge Q is (c) VA < VC (d) VA > VC
placed at the centroid of the triangle. The system of IIT, 2001
charges will be in equilibrium if Q equals 58. Two equal poi x = – a and
q q x = + a on the x-axis. Another point charge Q is
(a) (b) – placed at the origin. The change in the electrical
3 3
potential energy of Q, when it is displaced by a
(c)
q
(d) –
q small distance x along the x-axis, is approximately
3 3 proportional to
54. A metallic sphere A of radius a carries a charge (a) x (b) x 2
Q. It is brought in contact with an uncharged sphere (c) x3 (d) 1/x
B of radius b. The charge on sphere A now will be
IIT, 2002
aQ bQ
(a) (b) 59. A metallic shell has a point charge q kept inside
b a its circular cavity. The charge is not exactly at the
bQ aQ centre of the cavity. Which of the diagrams in Fig.
(c) (d) 20.32 correctly represents the electric lines of force?
a b a b
20.16 Comprehensive Physics—JEE Advanced

(a) + q 3 alone (b) + q1 and + q3


q (c) + q1, + q3 and – q2 (d) + q1 and – q2
IIT, 2004
62.
charge densities
1 = – , 2 = + 2 and 3 = + 3
are placed parallel to the x–z plane at y = a, y = 3a
and y = 4a
at point P is

Fig. 20.32
IIT, 2003
60. Three negative point charges – q each, and three
positive point charges + q, + q and + Q are placed
at the vertices of a regular hexagon as shown in Fig.
20.33. For what value of Q
O A, B, D, E and F be
O due to charge Q Fig. 20.35
at C alone?
2
q (a) zero (b) j
(a) q (b) 0
2
3 3
(c) 2q (d) 5q (c) j (d) j
0 0
IIT, 2004
IIT, 2005
63. A metallic sphere of radius R is charged to a po-
tential V
distance r (> R) from the center of the sphere is
V Vr
(a) (b)
r R2
VR
(c) (d) zero
r2
64. Two point charges q1 = 1 C
and q2 = 2 C are placed at
points A and B 6 cm apart
Fig. 20.33 as shown in Fig. 20.36.
61. Figure 20.34 shows A third charge Q = 5 C
a spherical Gaussian is moved from C to D
surface and a charge along the arc of a circle of
distribution. When radius 8 cm as shown. The Fig. 20.36
change in the potential
energy of the system is
the Gaussian surface, (a) 3.0 J (b) 3.6 J
(c) 5.0 J (d) 7.2 J
be due to
Fig. 20.34
Electrostatic Field and Potential 20.17

65. A partical of mass m and charge + q is midway when another positive point charge is moved from
(– a, 0, 0) to (0, a, 0) is
a charge + q and at a distance 2L apart. The middle (a) positive
charge is displaced slightly along the line joining (b) negative
(c) zero
oscillation is proportional to. (d) depends on the path connecting the initial and
(a) L1/2 (b) L
IIT, 2007
(c) L3/2 (d) L2 70. A long, hollow conducting cylinder is kept coaxi-
ally inside another long, hollow conducting cylin-
66. Five point charges, each equal to + q, are placed
der of larger radius. Both the cylinders are initially
L. The
electrically neutral.
magnitude of the force on a point charge – q placed
(a) A potential difference appears between the
at the centre of the haxagon is
two cylinders when a charge density is given
5q2 3q 2 to the inner cylinder
(a) (b)
4 2
4 2 (b) A potential difference appears between the
0L 0L
two cylinders when a charge density is given
q2 to the outer cylinder
(c) (d) zero
4 2 (c) No potential difference appears between the
0L
two cylinders when a uniform line charge is
IIT, 1992 kept along the axis of the cylinders
67. Two isolated metal spheres of radii R and 2R are (d) No potential difference appears between the
charged such that both have the same surface two cylinders when same charge density is
charge density . The spheres are located far away given to both the cylinders
from each other. When they are connected by a thin IIT, 2007
conducting wire, the new surface charge density on 71. Consider a neutral conducting sphere. A positive
the bigger sphere will be point charge is placed outside the sphere. The net
2 3 charge on the sphere is then,
(a) (b)
3 5 (a) negative and distributed uniformly over the
5 surface of the sphere
(c) (d) (b) negative and appears only at the point on the
6 2
sphere closest to the point charge
IIT, 1996 (c) negative and distributed non-uniformly over
68. A spherical portion has the entire surface of the sphere
been removed from a (d) zero
solid sphere having a IIT, 2008
charge distributed uni- 72. Three concentric metallic spherical shells of radii
formly in its volume R, 2R, 3R, are given charges Q1, Q2, Q3, respec-
as shown in Fig. 20.37. tively. It is found that the surface charge densi-
ties on the outer surfaces of the shells are equal.
the emptied space is Then, the ratio of the charges given to the shells,
(a) zero everywhere Q1 : Q2 : Q3, is
Fig. 20.37
(b) non-zero and uni- (a) 1 : 2 : 3 (b) 1 : 3 : 5
form (c) 1 : 4 : 9 (d) 1 : 8 : 18
(c) non-uniform IIT, 2009
(d) zero only at its centre 73. A disk of radius a/4 having a uniformly distributed
IIT, 2007 charge 6C is placed in the x-y plane with its centre at
69. Positive and negative point charges of equal (– a/2, 0, 0). A rod of length a carrying a uniformly
a a distributed charge 8C is placed on the x-axis from
magnitude are kept at 0, 0, and 0, 0, x = a/4 to x = 5a/4. Two point charges – 7C and 3C
2 2
are placed at (a/4, – a/4, 0) and (– 3a/4,3a/4, 0),
respectively. Consider a cubical surface formed by
20.18 Comprehensive Physics—JEE Advanced

six surfaces x = ± a/2, y = ± a/2, z = ± a/2. The elec-


74. E = E0 x , where E0 is
area (as
20.38]
2C 2C
(a) (b)
0 0

10 C 12 C
(c) (d)
0 0

Fig. 20.39

(a) 2E0a2 (b) 2 E 0a 2


E0 a 2
(c) E0a2 (d)
Fig. 20.38 2
IIT, 2009

ANSWERS

1. (a) 2. (d) 3. (d) 4. (a) 5. (c) 6. (a)


7. (b) 8. (d) 9. (c) 10. (d) 11. (c) 12. (b)
13. (c) 14. (b) 15. (d) 16. (b) 17. (c) 18. (a)
19. (d) 20. (c) 21. (d) 22. (b) 23. (b) 24. (d)
25. (d) 26. (a) 27. (d) 28. (b) 29. (b) 30. (c)
31. (a) 32. (a) 33. (b) 34. (b) 35. (d) 36. (d)
37. (d) 38. (b) 39. (d) 40. (d) 41. (c) 42. (c)
43. (b) 44. (d) 45. (d) 46. (a) 47. (a) 48. (b)
49. (a) 50. (a) 51. (b) 52. (a) 53. (b) 54. (d)
55. (c) 56. (d) 57. (b) 58. (b) 59. (c) 60. (b)
61. (c) 62. (c) 63. (c) 64. (b) 65. (c) 66. (c)
67. (c) 68. (b) 69. (c) 70. (a) 71. (d) 72. (b)
73. (a) 74. (c)

SOLUTIONS
1.
or W = Uf – Ui Q
+
1 –Q
Ui = [(q) (–2q) + q(–2q) + (–2q) (–2q)] 2.5
4 0 r
=0 2

1
Uf = [(q) (–2q) + q(–2q) + (–2q) (–2q)]
4 0 (2 r )
=0
W = 0. So the correct choice is (a).
Fig. 20.40
2. Charge Q at point P will induce a charge – Q on the
inner surface and a charge + Q on the outer surface 1 Q Q Q
V=
of the shell (Fig. 20.40). The electric potential at O is 4 0 R 2R 2.5 R
Electrostatic Field and Potential 20.19

1 9Q 4q2 + 4qQ = 0 4q (q + Q) = 0
=
4 0 10 R Hence Q = – q. The force on charge 4q is
3. Before connection, charge Q on the inner shell in- 4 qQ 4q 2
duces a charge – Q on the inner surface of the outer F= 2
l 4 0 l2
shell and a charge + Q on its outer surface. There- 4 0
2
fore, the total charge on the outer surface of the
outer shell = Q + 2Q = 3Q. When the two shells are Putting Q = – q, we get
connected by a conducting wire, the entire charge 3q 2
Q on the inner shell is conducted to the outer shell. 16 q 2 4q 2
F =– =–
Therefore, the charge on the outer shell now is 4 l2 4 l2 0 l2
0 0
Q + 2Q = 3Q, the same as before. Hence there will 3q 2
be no change in its electric potential. | F |= , which is choice (c).
0 l2
Q Q
4. E = 2
and V = 10. Refer to Fig. 20.42. The three charges will be in
4 0R 4 0R
equilibrium if no net force acts on each charge.
Hence if R is doubled, E becomes E/4 and V be- Charge q is in equilibrium because the forces acting
comes V/2. So the correct choice is (a). on it by charge Q at A and charge Q at B are equal
5. =
Q
. In terms of , and opposite. Charge Q at A will be in equilibrium if
4 R2
qQ Q2 Q
R 2 2
=0 q=– .
E= and V = 4 0 l 4 0 (2 l ) 4
0 0
Similarly charge Q at B will be in equilibrium if
If R is halved, E remains the same but V becomes Q
V q=– . Hence the correct choice is (d).
. So the correct choice is (c). 4
2
6. If p is the electric dipole moment, then

EP = 2p
4 0 r3 Fig. 20.42
p 11. If R is the radius of the big drop, we have
EQ = –
4 0 ( 2 r )3
4 R3 4 r3
EP = – 16 EQ, which is choice (a). = 1000
3 3
7. The potential inside a spherical conductor is con-
which gives R = 10 r. The electrical potential of
stant including that on its surface. Hence the correct
each droplet is
choice is (b). q
8. The potential at every point on the circle due to v =
4 0r
charge Q is the same. Work done = q (potential
and that of the big drop is
difference). Hence the correct choice is (d).
9. Refer to Fig. 20.41. 1000 q
V =
4 0R

V 1000 r
= = 100 ( R = 10r)
v R
Fig. 20.41 Hence the correct choice is (c).
The net force on q will be zero if 12. When two identical metallic spheres are brought in
contact, the charges on them are equalized due to
q (4 q) qQ
2 2
=0
4 0 l l sphere C is brought in contact with sphere A having
4 0
2 a charge + q, and then removed, the total charge q
20.20 Comprehensive Physics—JEE Advanced

is equally shared between the two so that the charge 1 q1


left on A is +q/2 and that developed on C is +q /2. E1 =
4 0 a2
The sphere C carrying a charge +q/2 is now brought
in contact with sphere B which is already carrying and is directed along + x E2
a charge +q. The total charge is q / 2 + q = +3q/2 at (a, b) due to q2 has a magnitude
which must distribute equally on B and C. Thus 1 q2
E2 =
when C is removed, B will have a charge of +3q/4 4 0 b2
and C also has a charge of +3q/4. Hence when C is and is directed along + y-axis. The angel
removed from both A and B, E with the x-axis
New charge on A = +
q is given by
2
3q
New charge on B = +
4
Since force is proportional to the product of the
charges, it follows that the new force of repulsion
between A and B is 3/8 of the earlier force (F).
Hence, the new force of repulsion between A and B
is 3F/8. Fig. 20.44
13. Refer to Fig. 20.43. Let us consider forces on a ball,
say, Q. Three forces act on it: (i) tension T in the E2 q a2
tan = = 2 2
thread, (ii) force mg due to gravity and (iii) force F E1 q1 b
due to Coulomb repulsion along + x-direction. For
2
equilibrium, the sum of the x and y components of 1 2
= =2
these forces must be zero, i.e. 2 1
T cos 60° – F = 0 Hence the correct choice is (b).
and T sin 60° – mg = 0 15. E
exerts a force q E on charge + q and a force – qE
on charge – q of the dipole. Since these forces are
equal and opposite, they add upto zero.
16. The correct choice is (b). A torque acts on the

17. The correct choice is (c). In a non-uniform electric

translational motion and a torque which gives it a


rotational motion.
18. The distance of a vertex from the the centre of the
cube of side b is r = 3 b / 2. Now the potential
Fig. 20.43 due to charge q at the centre is q/4 0r. Hence the
3 potential due to the arrangement of eight charges
These equations give F = mg cot 60° = 10 –3
10 (each of magnitude q) at the centre is
1
10 = 10–3 N. Now 8q 4q
3 V = =
4 0r 3
1 q2 0b
F =
4 0 r2 19. -
Putting F = 10 –3
N, r = 0.3 m torially. From the symmetry of the eight charges
with respect to the centre of the cube, it is evident
1
and =9 10 9, we get q = 10–7 coulomb.
4 0 charges cancel in pairs (being equal and opposite).
14. E1 at (a, b)
due to q1 has a magnitude will be zero.
Electrostatic Field and Potential 20.21

20. Refer to Fig. 20.45. The distance of point P1 from Here q1 = q2 = q = + 1.6 10–19 C (proton), q3 =
charge + q is r1 = z – a and from charge – q is r2 = – q = – 1.6 10–19 C (electron), r12 = 1.5 Å = 1.5
z + a. 10–10 m, r13 = r23 = 1 Å = 1 10–10 m and
1/4 0 =9 109 Nm2C–2. Thus
4 q 2 1010
U= – joule
3 4 0
4 q 1010
=– eV
3 4 0
19
4 1.6 10 1010 9 109
=–
3
= – 19.2 eV
23. Charge on electron (– e) = – 1.6 10 –19 C, charge
Fig. 20.45 on proton (e) = 1.6 10–19 C, separation r =
–10
0.53 Å = 0.53 10 m. If the zero of potential
1 q q
Potential at P1 =
4 0 r1 r2 potential energy of the electron-proton system is
q r2 r1 e2
1
= U= – joule
4 0 r1 r2 4 0 r
2qa 1 e
= , =– eV
4 0 z 2 a2 4 0 r
which is choice (c). 9 109 1.6 10 19

21. Refer to Fig. 20.45 again. Any point on the perpen- =–


0.53 10 10
dicular bisector passing through the centre of the
= – 27.2 eV
dipole is at the same distance from the two charges.
Hence the correct choice is (b).
Hence the potentials at point P2(5, 0, 0) and at point
P3(–7, 0, 0) are zero. Since P2 and P3 are at the same 24. Points A, B and C are at the same distance from
potential (zero), the potential difference between charge + q; hence electrical potential is the same
them is zero. Hence no work will be done in moving at these points, i.e. there is no potential difference
a charge from P2 to P3. The answer will not change between A, B and C. Hence W1 = W2 = 0.
if the path of the charge is changed because the work 25. The electric potential at the common centre is
done is independent of the path taken. q1 q2
V = +
22. Refer to Fig. 20.46. The total potential energy of 4 0 r1 4 0 r2
the arrangement of charges is the sum of the ener- q1 q2
gies of each pair of charges. The potential energy of Now = =
the system comprising the three charges q1, q2 and 4 r12 4 r22
q3 is 1 q1 r1 q2 r2
U = W1 + W2 + W3 V =
0 4 r12 4 r22
1 q1 q2 q1 q3 q2 q3
=
4 0 r12 r13 r23 = (r1 + r2)
0
Hence the correct choice is (d).
26. If q
surface is
q
E =
4 0 r2
q
But = . Therefore q = 4 r 2 .
Fig. 20.46 4 r2
20.22 Comprehensive Physics—JEE Advanced

4 r2
Hence E = =
4 0 r2 0

Thus the correct choice is (a).


27. Refer to Fig. 20.47. Forces exerted by charges – q at
A and B on charge Q are F1 and F2 which are equal
and have a magnitude
qQ
F=
4 0 r2 Fig. 20.48

P due to charge 4 q at D is
1 4q
E2 = along PB
4 0 r2
1 2q
PB is E = E2 – E1 =
4 0 r2
P due to charges
q and 3q at A and C will be
1 2q
E = directed along PA.
4 0 r2
Fig. 20.47 Thus E = E , but they are mutually perpendicular
to each other. therefore, their resultant will be
The resultant of these equal forces equally inclined
along PQ (see Figure) which is parallel to CB.
with the x-axis is along the negative x-direction to-
Hence the correct choice is (b)
wards the origin. The magnitude of the resultant is
given by 29. E. The force on charge q is
F 2r = F 2 + F2 + 2 F2 cos FA = qE
qE
= 2 F2 (1 + cos ) Therefore, its acceleration is aA =
m
1 Similarly, the acceleration of charge 4q is
Since F ; Fr is also proportional to (1/r2).
r2 4q E
Hence charge Q will move towards the origin and aB = = 4 aA
m
because of its inertia it will overshoot the origin O. Let s be the distance travelled by A and B to
Thus, charge Q will oscillate about O but its motion acquire speeds vA and vB. Then (since u = 0)
is not simple harmonic because the force Fr is not
v2A = 2 aA s and vB2 = 2 aB s
proportional to its instantaneous distance from the
origin as Fr 1/r2. Hence the correct choice is (d). v 2A aA 1
=
28. r from a point charge vB2 aB 4
Q is given by or vA/vB = 1/2. Hence the correct choice is (b).
1 Q 30. Refer to Fig. 20.49.
E=
4 0 r2 AO = BO = CO = DO = r =
a
If Q 2
from Q. Refer to Fig. 20.48. Let PA = PB = PC = Since electric potential is a scalar and since the
PD = r P due to charge 2q charges at corners are equal, the magnitude of the
at B is electric potential at point O due to the four charges
= 4 times that due to a single charge. Thus
1 2q
E1 = along PD 1 Q
4 0 r2 V=4
4 0 r
Electrostatic Field and Potential 20.23

34. Force F = qE. Therefore, acceleration a = qE/m.


1 Q 2Q
= = Hence the distance travelled by the particle in time
0 a/ 2 0a t is
1 2 1 qE 2
s= at = t
2 2 m
1 qE
For electron, se = t 21
2 me
1 qE
For proton, sp = t 22
2 mp
Given se = sp. Therefore
1/ 2
Fig. 20.49 t12 t2 t2 mp
= 2 or
me mp t1 me
2Q 2
Work done = Q V = . Hence the correct
0
0a
choice is (c). 35. The integral –E gives the potential at the
31. E =
s centre of the ring, which is zero.
q
. Hence the correct choice is (a). 36.
0 surface of a conductor. On the surface of a metal-
32. When any additional negative charge is given to a
hollow spherical shell, the potential on its surface to the surface and directed towards the centre of the
falls, but the potential at each point within the shell sphere. Hence the correct choice is (d).
also falls by the same amount. Hence the poten- 1 q q q
tial difference between the given surfaces remains 37. V = upto infinity
4 0 x0 3 x0 5 x0
unchanged. Thus, the correct choice is (a).
33. Refer to Fig. 20.50. Potential at C1 is 1 q q q
+ upto infinity
4 0 2 x0 4 x0 6 x0
1 Q1 Q2
V1 = 1 q 1 1 1 1 1
4 0
R 2R = 1 upto infinity
4 0 x0 2 3 4 5 6
Potential at C2 is
q q log e 2
1 Q2 Q1 = loge (1 + 1) =
V2 = 4 4 0 x0
4 R 2R 0 x0
0
38. The net electrostatic energy is ( hypotenuse side
of triangle = 2 a)
Qq Qq qq
U=
a 2a a
For U = 0, we require

Qq Qq q2
Fig. 20.50 = 0 or Q + Q + q = 0
a 2a a 2
Work done 2q
2
W = q (V1 – V2) which gives Q = – q =
2 1 2 2
q Q1 Q2 Q2 Q1
= 39. A dipole consists of two equal and opposite charg-
4 R 2R R 2R
0 es separated by a certain distance. Hence the total
q charge enclosed in the cube is zero. Therefore, the
= (Q1 – Q2) ( 2 – 1)
4 0 2R
20.24 Comprehensive Physics—JEE Advanced

40. - 45. Charge q will momentarily come to rest at a dis-


dicular bisector of a dipole is given by (for r >> a), tance r from charge Q when all its KE is converted
here p is the dipole moment to PE, i.e.
p 1 1 qQ
E= mv2 =
4 0 r3 2 4 0 r
Hence the correct choice is (d). Therefore, the distance of closest approach is
41. Initial kinetic energy of the particle is zero. The given by
qQ 2
gain in kinetic energy in distance x = decrease in r=
4 0 mv 2
to move the particle through a distance x = force 1
Thus r . Hence if v is doubled, r becomes
distance = q Ex. Hence the correct choice is (c). v2
42. Given vx = 10 ms–1 one-fourth. Thus the correct choice is (d).
directed along the y-axis, the acceleration of the a
46. Let the side of the square be a. OA = OC = r =
body along the y-axis is 2
6
(see Fig. 20.51).
qE 10 103
ay = 3 = 1 ms–2
m 10
Therefore, the velocity of the body along the y-axis
at time t = 10 s is
vy = ayt = 1 × 10 = 10 ms–1

Resultant velocity v = vx2 v 2y

= (10) 2 (10)2 10 2 ms 1

Hence the correct choice is (c).


43. Let q be the magnitude of each charge and a
the length of each side of the triangle ABC. The
potential energy of the system of two equal charges
placed at vertices A and B is U (given). This means
that U is the work done in bringing a charge q from Fig. 20.51
B with the charge q at vertex
1. Stability of charge + q at the centre
A. Hence the work done in bringing an identical
charge q C = work Charges – Q at corners A and C will attract charge + q
done to overcome the force of repulsion of q placed with equal and opposite forces. Similarly charges – Q at
at A placed at a distance a from it + work done to corners B and D will attract charge + q with equal and
overcome the force of repulsion of q placed at B at opposite force. Hence no net force acts on charge – q.
the same distance a from it = U + U = 2U, which 2. Stability of charge –Q at any corner
is choice (b).
Q at corner A. This
q
44. Given E = . Hence the magnitude of the charge will experience four forces:
4 0 x2 (i) Force of repulsion F1 due to charge – Q at B
electric intensity at a distance 2x from charge q is (ii) Force of repulsion F2 due to charge – Q at D
q q 1 E (iii) Force of repulsion F3 due to charge – Q at C
E = 2 2
4 0 (2 x) 4 0x 4 4 (iv) Force of attraction F due to charge + q at O.
Therefore, the force experienced by a similar charge Q2
q at a distance 2x is Now F1 = F2 = 2
4 0a
qE
F = qE =
4 Q2 Q2
and F3 = 2 2
Hence the correct choice is (d). 4 0 ( 2r ) 4 0 ( 2a )
Electrostatic Field and Potential 20.25

1 q ( Q) 2qQ d
and F = 2 2
E=– 4 x2 i 8 xi 8i Vm 1 .
4 0 r 4 0a dx
The resultant of F1 and F2 is given by The negative sign shows that E is along the nega-
2 tive x-axis. Hence the correct choice is (a).
2Q
F= F12 F22 2 F1 = 51. There will be no loss of energy if the potential of
4 0a2 the spheres is the same i.e. if
The forces F and F3 act along AP. Hence the net force
q Q
acting on charge – Q at A due to charges – Q at B, C and V=
D is 4 0r 4 0R
F = F + F3
q Q
2 Q2 Q2 Q 2 (1 2 2 ) or . Hence the correct choice is (b).
= r R
4 0 a 2 4 0 ( 2a 2 ) 4 0 ( 2a
2
)
For equilibrium, F = F , i.e. 52. r = (2 i 3j k) (i j k) (i 2j 2 k ) m.

2qQ Q 2 (1 2 2 ) The magnitude of r is


2
= 2
4 0a 4 0 ( 2a ) r= 12 22 22 1 4 4 =3m
Q 1
or q= 1 2 2 F=
q1 q2
4 4 0 r2
Hence the correct choice is (a).
47. The force of repulsion between the two parts is 9 109 10 6
10 6
= = 10–3 N
given by 3 2
1 q Q q
F= Hence the correct choice is (a).
4 0 r2
53. The system will be in equilibrium if the net force
dF on charge q at one vertex due to charges q at the
For F to be maximum, = 0, i.e.
dq other two vertices is equal and opposite to the
force due to charge Q at the centroid, i.e. (here a is
d 1 q Q q
=0 the side of the triangle)
2
dq 4 0 r
3 q2 Qq
Since r 2 2
4 0a a
d 4
[q(Q – q)] = 0 0
3
dq
q 1 q
which gives which gives Q = – . Hence the correct choice
Q 2 3
is (b).
Hence the correct choice is (a).
54. A to B until their potentials
48. For points on the surface of the sphere or outside the become equal. If charge q A to B, then
sphere, a charged sphere behaves as if the charge is
concentrated at its centre. Therefore, the potential Q q q
at the surface of the sphere is given by 4 0a 4 0b

Q 1 or Q – q =
a
q which gives q =
bQ
. Hence
V= , which is choice (b).
4 0 R b a b
49. At points inside a charged metallic sphere, i.e. for r charge left on A = Q – q = Q –
bQ aQ
.
< R, the potential is zero. Hence the correct choice a b a b
is (a). Hence the correct choice is (d).
dV 55. Since all the three charges are of the same polarity,
50. E = – i where i is a unit vector along the
dx
positive x-axis. Hence E at a point whose x-coordi- around them cannot be closed. Hence choices (a),
nate is x = 1 m is
20.26 Comprehensive Physics—JEE Advanced

(b), the line in the middle is a closed circular loop. 59. Because the charge is not located at the centre of
Thus, the only correct choice is (c). the cavity, inside the cavity the lines of force are
56. The capacitance of a parallel plate capacitor is giv- skewed. Hence choice (a) and (b) are incorrect.
en by C = 0 A/d. Hence the dimensions of 0 L are Outside the shell, the lines of force are the same as
the same as those of capacitance. if the charge were located at the centre of the cav-
V ity. Also there can be no line of force in the metallic
Dimensions of 0L
t body of the shell. Hence choice (d) also incorrect.
dimension of C dimensions of V Thus the correct pattern is shown in (c).
= 60. O due to charges – q at
time
A and D are equal and opposite. Hence they cancel
dimension of Q
= ( Q = CV) each other. Similarly charges + q at B and E do not
time contribute to electric at O. Due to charge – q at
charge F O will be
= = current
time q
Hence the correct choice is (d). E1 = 2
directed from O to F
4 0r
57.
O
dV charges at A, B, D, E and F O
E=–
dx due to charge + Q at C is given by
Thus V decreases as dx increases in the direction E2 =
Q
directed from O to F
VA VB , which is 4 0 r2
choice (b). For E1 = 2E2, we require
58. Potential energy of the system when charge Q is at
q 2Q
O is 2
= 2
4 0r 4 0r
qQ qQ 2qQ
U0 = q
a a a which gives Q = . Hence the correct choice is (b).
2
When charge Q is shifted to position O , the poten-
61.
tial energy will be (see Fig. 20.52).
E.ds E is due to all the
charges, both inside and outside the Gaussian sur-
face. Hence the correct choice is (c).
62.
plane sheet carrying a uniform charge density is
Fig. 20.52 given by
qQ qQ qQ 2a
U= E=
a x a x a2 x2 2 0
1
It is independent of the distance of point P from the
2qQ x2 sheet and is, therefore, uniform. The direction of
= 1
a a2 -
dicular to it if is positive and is towards the sheet
2qQ x2 and perpendicular to it if is negative. Hence
1 ( x a)
a a2
E1 = j along –ve y–direction
2 2 0
2qQ x 2qQ
U = U – U0 = 1 2 2
a a a E2 = j along –ve y–direction
2 0
2qQ
= 3
(x 2) 3
a and E3 = j along – ve y–direction
2 0
Hence U x 2 which is choice (b).
Electrostatic Field and Potential 20.27

From the superposition principle, the net electric


q q q2
P is F= =
4 (OD)2 4 L2
E = E1 + E2 + E3 o o

2 3
= j j j
2 0 2 0 2 0
3
=– j , which is choice (c).
0
63. Let the charge on the sphere be Q. Then
Q
V=
4 0R
which gives Q = 4 0RV
r is
Fig. 20.53
Q 4 0 RV RV
E= = = 67. Initial charge on sphere of radius R is Q1 = 4 R2
4 0r 2 4 r2 0r
2
and on sphere of radius 2R is Q2 = 4 (2R)2
Thus the correct choice is (c).
= 16 R2
64. If charge Q is moved from C to D along the arc,
Total initial charge is Q = Q1 + Q2 = 20 R2
the potential energy between pairs (q1, Q) and (q1,
Initial potential of the sphere of radius R is
q2) will not change as the distance between them
remains unchanged ( AC = AD). The potential Q1
V1 =
energy of the pair of chages q2 and Q will change. 4 R
Now, distance BC = (8)2 (6) 2 = 10 cm and BD and of the sphere of radius 2R is
= 8 – 6 = 2 cm. Therefore, change in P.E. is Q2
V2 =
q2Q 1 1 4 (2 R)
U=
4 0 BD BC When the spheres are connected by a thin wire,
1 1
= (2 10–6) (5 10–6) (9 10–9) their potentials become equal. Let Q 1 and Q 2 be
0.02 0.1
= 3.6 J, which is choice (b). the new charges, then the potential of each sphere
will be
65. If the middle charge is displaced by a distance x, the
Q1 Q2
net force acting it, when it is released, is V= =
4 R 4 (2 R)
1 q2 1 q2
F=
4 0 (L x)2 4 0 (L x)2 which gives Q 1 =
Q2
2
4q 2 Lx
= 2
From conservation of charge, we have
4 0 (L x 2 )2 Q1 + Q2 = Q 1 + Q 2
q2 x 3Q 2
For x << L, F = – 3
= – kx 20 R2 =
Q2
+ Q2 =
0L 2 2
q2
where k = 3 40
0L Q2 = R2
3
m New surface charge density on the sphere of
Now T = 2
k radius 2R is
40
So, the correct choice is (c). R2
Q2 3 5
66. As shown in Fig. 20.53, all forces cancel in pairs, = 2 = =
2
except the forces F on charge – q due to charge + q 4 (2 R) 16 R 6
at vertex D (i.e. the vertex opposite to the empty 68. Let O be the centre of the sphere and Q be the cen-
vertex). The magnitude of the net force on – q is tre of the cavity. Let r be the separation between
20.28 Comprehensive Physics—JEE Advanced

them. Let OP = b and QP = a. Here P is a point outer surface, so that the total charge on the outer
inside the cavity. Let be the charge density. From surface of shell 2 is (Q1 + Q2). This charge induces
a charge – (Q1 + Q2) on the inner surface of shell
point P is given by [See Fig. 20.54] 3 and a charge + (Q1 + Q2) on its outer surface so
that the total charge on the outer surface of shell
E = b a = b a 3 is (Q1 + Q2 + Q3) as shown in Fig. 20.56. Given
3 0 3 0 3 0
1= 2= 3

=
r Q1 (Q1 Q2 ) (Q1 Q2 Q3 )
r a b =
3 4 2
4 2 2
0 0 R 0 (2 R) 4 0 (3R )

Fig. 20.54
-
Fig. 20.56

choice is (b). Solving we get Q1 = 3Q2 = 5Q3. Hence the correct


69. The two charges constitute an electric dipole. choice is (b).
The points (– a, 0, 0) and (0, a, 0) are at the same q
73. E x through the cubical surface = where
distance from the positive and negative charges of 0
equal magnitude. Hence the electric potential is q is the net charge enclosed in the surface. Since
zero at these points. Hence the work done is zero. half the disc lies inside the surface, one-fourth of
70. When a charge density is given to the inner cylinder, the rod lies inside the surface, point charge –7C lies
inside the surface and point charge 3C lies outside
outer cylinders. Hence a potential difference ap- the surface, the net charge enclosed in the surface is
pears between the two cylinders. If a charge density
6C 8C
is given to the outer cylinder, the same potential ap- q= – 7C + 0
pears on the inner and the outer cylinders. Hence, 2 4
in this case, there is no potential difference between = 3C + 2C – 7C = – 2C
them. Thus the correct choice is (a). 2C
. So the correct choice is (a).
71. When a positive charge + Q is placed outside a 0
neutral conducting sphere, it will induce a negative
74. Refer to Fig. 20.57.
charge – Q on the side of the sphere closer to it and
an equal positive charge + Q on the opposite side of
the sphere. Thus the net charge on the sphere will
be zero, which is choice (d). [see Fig. 20.55]

Fig. 20.57
Fig. 20.55
= E A
72. Charge Q1 on shell 1 induces a charge – Q1 on the
inner surface of shell 2 and a charge + Q1 on its Area vector A = a j (a i a k)
Electrostatic Field and Potential 20.29

= E0 i [a j (a i a k )] = E0 a 2 ( i k i i)
= E0a2 (0 + 1) = E0a2
= E0 i [ a 2 k a2 i ]

II

Multiple Choice Questions with One or More Choices Correct


1. Choose the correct statements from the following. 4. Which of the following statements are correct?

potential must also be zero at that point. nucleus in moving an electron around it in a
(b) If electric potential is constant in a given complete orbit is zero irrespective of whether
the orbit is circular or elliptical.
zero in that region. (b) The equipotential surfaces corresponding to
(c) Two different equipotential surfaces can never
intersect. are concentric spheres with the point charge
(d) Electrons move from a region of lower as the common centre.
potential to a region of higher potential. (c) If Coulomb’s law involved 1/r3 dependence
2. Figures 20.58 (a) and (b) show the lines of force instead of 1/r2, Gauss’s law would still hold
q) and a good.
negative charge (– q) respectively. (d) A single conductor cannot have any capaci-
Which of the following statements are correct? tance.
(a) Potential at P is greater than that at Q. 5. A pendulum bob of mass m carrying a charge q is
(b) Potential at A is greater than that at B. at rest with its string making an angle with the
E. The
(c) Potential energy at P is less than that at Q.
tension in the string is
(d) Potential energy at A is greater than that at B. mg mg
(a) (b)
sin cos
qE qE
(c) (d)
sin cos
6. Four point charges + q, + q, – q and – q are placed
respectively at corners A, B, C, and D of a square.
Then
(a) the potential at the centre O of the square is
Fig. 20.58
zero.
3. - O of the square
tive charge (+ q) and a negative charge (– q) are is zero.
shown in Figs. 20.36 (a) and (b) above. Then (c) If E is the mid-point of side BC, the work
(a) the work done in moving a small positive done in carrying an electron from O to E is
charge (+ q0) from Q to P will be positive zero.
(d) If F is the mid-point of side CD, the work
(b) the work done in moving a small negative done in carrying an electron from O to F is
charge (– q0) from B to A will be positive zero.
(c) in going from Q to P, the kinetic energy of 7. Six charges, each equal to + q, are placed at the
a small negative charge (– q0) increases corners of a regular hexagon of side a. The electric
(d) in going from B to A, the kinetic energy of potential at the point where the diagonals intersect
a small negative charge (– q0) decreases is V E. Then
20.30 Comprehensive Physics—JEE Advanced

6q Form the graph we conclude that


(a) V = 0 (b) V =
4 0a
the surface of the sphere.
6q (b) for r > R, the entire charge Q may be
(c) E = 0 (d) E =
4 0a
2 assumed to be concentrated at the centre of
the sphere.
8. Two parallel plane sheets 1
(c) the electric potential is maximum at centre
and 2 carry uniform charge of the sphere.
densities + and – as shown (d) the electric potential is zero at centre of the
in Fig. 20.59. The electric sphere.
11. A sphere of raduis R is made of a conducting
I, II and III are E1, E2 and E3 material and carries a charge Q. Choose the correct
respectively statements from the following.
(a) E1 = 0 (a) Charge Q resides on the surface of the sphere.
(b) E2 =
0 the surface of the sphere.
(c) E3 = 0
2 Fig. 20.59 (d) No work is needed to move a charge from one
(d) E1 = E3 = point to another on the surface of the sphere.
0
12. A proton and an electron are placed in a uniform
9. Electric potential V -
due to a spherically enced by proton and electron are Fp and Fe and ap
symmetric charge and ae are the respective magnitudes of their accel-
system varies with erations. Then
distance r as shown (a) Fe > Fp (b) Fe = Fp
in Fig. 20.60 (c) ae > ap (d) ae < ap
Given V =
Q 13. A metal ring of radius R carries a charge Q distrib-
for r < r 4 0 r0 uted uniformly on it. A point P lies on the axis of
0 Fig. 20.60 the ring at a distance r from its center. Then
Q (a) The potential at the centre of the ring is zero.
and V = for r > r0
4 0r
zero
Which of the following statements are true? (c) For r >> R, the potential varies as 1/r.
(d) For r >> R, r 2.
discontinuous at r = r0. 14.
(b) The net charge enclosed in a sphere of radius ged. A point P lies at a perpendicular distance r
r = 2r0 is from it Then
(c) No charge exists at any point in a spherical r.
region of radius r < r0. r 2.
(d) Electrostatic energy inside the sphere of
radius r = r0 is zero. istant straight lines prependicular to the wire.
IIT, 2006
10. A sphere of radius 15.
R is made of a non- P lises at a perpendicular distance r from it. Then
conducting material r.
and carries a positive r 2.
charge Q. Figure 20.61
shows the variation of distant straight lines prependicular to the
E with plane sheet
distance r from the Fig. 20.61
centre of the sphere. sheet.
Electrostatic Field and Potential 20.31

q 21. Two small balls, each having a charge + Q are sus-


16. In Gauss’s theorem E ds . The surface pended by two insulating strings each of length L
s 0
integral is evaluated by choosing a closed surface
taken in a satellite orbiting the earth. In the satellite
called the Gaussian surface. Here
the angle between the strings is and the tension in
(a) the closed surface can be of any shape or size.
each string is T. Then
(b) q is the net charge enclosed inside the Gauss-
(a) = zero (b) = 180°
ian surface; charges outside the surface are
not considered. Q2 Q2
(c) T = 2
(d) T = 2
(c) E 16 0L 4 0L
both inside and outside the surface.
(d) The exact location of the charges inside the IIT, 1986
surface does not affect the value of the integral. 22. A positively charged thin metal ring of radius R is
17. Two equal point charges q each are held at x = + a x-y plane with its centre at origin O. A
and x = – a. A third charge Q is placed at x = 0. The negatively charged particle P is released from rest
potential of the system will at the point (0, 0, z) where z > 0. Then the motion
(a) decrease if Q is displaced by a small distance of P is
along the x-axis (a) periodic for all values of z satisfying
(b) increase if Q is displaced by a small distance 0<z<
along the x-axis (b) simple harmonic for all values of z satisfying
(c) decrease if Q is displaced by a small distance 0<z
along the y-axis (c) approximately simple harmonic provided
(d) increase if Q is displaced by a small distance z << R.
along the y-axis (d) such that it crosses O and continues to move
18. The electric potential at a point P at a distance x along the negative z-axis towards z = – .
from a point charge is given by
IIT, 1998
k 23. A non-conducting solid sphere of radius R is
V=
r uniformly charged. The magnitude of the electric
where k is a constant. r from its centre
(a) k is dimensionsless (a) increases as r increases for r < R
(b) the dimensions of k are [ML–3T–3A–1] (b) decreases as r increases for 0 < r <
k (c) decreases as r increases for R < r <
P= 2.
r (d) is discontinuous at r = R.
P = kr2.
IIT, 1998
19. Two point charges q1= 4 C and q2 = –1 C are 24. An ellipsoidal cavity is made within a perfect con-
placed at x = 0 and x = 15 cm on the x-axis. ductor. A positive charge q is placed in the cavity
(a) Electric potential is zero at x = 3 cm. and A and B are two points on the surface of the
(b) Electric potential is zero at x = 12 cm. cavity as shown in Fig. 20.62. Then
x = 0
and x = 15 cm.
x = ± 30cm.
20. Choose the correct statement(s) from the following.
(a) The electric potential due to a point charge
at a distance r from it varies as 1/r
(b) Electric potential at a distance r from the cen-
tre of a charged sphere varies as 1/r provided
r is less than the radius of the sphere Fig. 20.62
r from a point A in the cavity = electric
charge varies as 1/r2 B in the cavity.
(b) Charge density at A = charge density at B
IIT, 1980 (c) Potential at A = potential at B
20.32 Comprehensive Physics—JEE Advanced

(d) Electrostatic energy inside the sphere of


q radius r = r0 is zero.
cavity is .
0 IIT, 2005
IIT, 1999 27.
25. A positive charge q +Q, a charge –q is moving around it in an elliptical
orbit. Find out the correct statement(s).
dipole with dipole moment p is placed along the
(a) The angular momentum of the charge – q is
x-axis far away from the origin with p pointing constant
along the positive x-axis. The kinetic energy when (b) The linear momentum of the charge – q is
it reaches a distance x from the origin is K and the constant
magnitude of the force experienced by charge q at (c) The angular velocity of the charge – q is
this moment is F. Then constant
(a) K varies as 1/x (b) K varies as 1/x2 (d) The linear speed of the charge – q is con-
2
(c) F varies as 1/x (d) F varies as 1/x3 stant
IIT, 2003 IIT, 2009
26. Electric potential V due to a spherically symmetric 28. A spherical metal shell A of radius RA and a solid
charge system varies with distance r as shown in metal sphere B of radius RB (< RA) are kept far apart
Fig. 20.63. and each is given charge ‘+Q’. Now they are con-
Q nected by a thin metal wire. Then
Given V= for r < r0
4 0 r0
(a) EAinside = 0
(b) QA > QB
Q
and V= for r > r0 A RB
4 0r (c)
B RA
Which of the following statements are true?
(d) EAon surface < EBon surface
IIT, 2011
29. Which of the following statement(s) is/are correct?

varies as r–2.5 instead of r–2, then the Gauss


law will still be valid.
(b) The Guass law can be used to calculate the

Fig. 20.63
is zero somewhere, then the sign of the two
charges is the same.
discontinuous at r = r0. (d) The work done by the external force in
(b) The net charge enclosed in a sphere of radius moving a unit positive charge from point A
r = 2r0 is at potential VA to point B at potential VB is
(VB – VA).
(c) No charge exists at any point in a spherical
region of radius r < r0. IIT, 2011

ANSWERS AND SOLUTIONS


1. is normal to the surface at that point. If two differ-
a point, electric potential is not necessarily zero at ent equipotential surfaces intersect, there would be
two directions for the normals to the two surfaces
point exactly mid-way between two equal charges of at the point of intersection. Hence there will be two
the same sign, but the potential at this point is twice
that due to a single charge. Statement (b) is correct not possible. Statement (d) is correct. Since electron
because E = – V. If V is constant V = 0. Hence E has a negative charge, it has less potential energy at
= 0 in that region. Statement (c) is also correct. The a point where the potential is higher and vice versa.
Electrostatic Field and Potential 20.33

3. The small positive charge (+ q0) will tend to move


the region of lower potential to the region of higher from P to Q due to the force of repulsion exerted
potential. on it by the charge (+ q
2. charge (+ q) does positive work on charge q0 to
charge + q is given by move it from P to Q. Hence the work done by the
q) in moving the charge (+ q0)
1 q
V= from Q to P will be negative. Hence statement (a)
4 0 r is incorrect.
where r is the distance of that point from the charge.
The work done by the external agency to move a
Referring to Fig. 20.35 (a), the potentials at points
negative charge (– q0) from B to A is positive, since
P and Q are
the external agency has to overcome the force of
1 q 1 q repulsion exerted by the negative charge (– q) on
VP = and VQ =
4 0 OP 4 0 O Q the small negative charge (– q0). Hence statement
(b) is correct.
since OP < OQ, VP > VQ. Hence statement (a) is
correct. In going from Q to P, a small negative charge is
speeded up due to the force of attraction exerted
on it by the positive charge (+ q). Hence statement
charge – q is given by
(c) is correct.
1 q
V=– In going from B to A, the small negative charge
4 0 r (– q0) is slowed down due to the force of repulsion
Referring to Fig. 20.35(b), the potentials at points exerted on it by the negative charge (– q). Hence
A and B are the kinetic energy of the small negative charge de-
1 q 1 q creases. Hence statement (d) is correct.
VA = – and VB = –
4 0 OA 4 0 OB 4. E in moving
a charge (– q) around a closed path of any shape
Since OA < OB, the potential at A is more negative than
(circular or elliptical) is given by
at B, i.e. VB > VA. Hence statement (b) is in-correct.
We know that the potential energy of a charge q2 W = q E d1
q1 at a distance r from it is Now, we know that the line integral of an electro-
given by
1 q1 q2 E d1 = 0
U=
4 0 r
Hence the work done W = 0 irrespective of
Referring to Fig. 20.21(a) the potential energy of a whether the path is circular or elliptical. Hence
small negative charge (– q0 - statement (a) is correct.
tive charge (+ q) at points P and Q is (setting q1 = We know that the potential at points equidistant
q and q2 = – q0) from a point charge is the same. Hence, the equi-
1 q q0 1 q q0
U p= – and UQ = – charge are concentric spheres with the point
4 0 OP 4 0 OQ
charge as the common centre. Hence statement (b)
Since OP < OQ, it is clear that UQ > UP. Hence is correct.
statement (c) is correct. The derivation of Gauss’s law assumes 1/r2 depen-
Referring to Fig. 20.21(b), the potential energy of a dence of distance between charges in Coulomb’s
small negative charge (– q0 - law. Gauss’s law will not hold if Coulomb’s law
tive charge (– q) at points A and B is (setting q1 = involved 1/r3 or any other power of the distance r.
– q and q2 = – q0) Hence statement (c) is incorrect.
1 q q0 1 q q0 A single conductor can have capacitance. It is a ca-
UA = and UB =
4 0 OA 4 0 OB
Since OA < OB, UA > UB. Hence statement (d) is a single spherical conductor of radius r has a capaci-
correct. tance C = 4 0r. Hence statement (d) is incorrect.
20.34 Comprehensive Physics—JEE Advanced

5. Refer to Fig. 20.64. Since the bob is in equilibrium


Now, AF = BF = 5 a/2 and CF = DF = a/2.
Putting these values, we get
For equilibrium, we have
mg = T cos q 1
VF = 1
and qE = T sin 0 a5
Thus T =
mg
=
qE
. Work done in carrying a charge –e from O to F is
cos sin W = – e (VF – V0) = – eVF
Hence the correct choices are (c) and (b). qe 1
= 1
0 a 5
Hence the correct choice is (a) and (c).
7. The distance of the point of intersection of
diagonals = side of the hexagon = a. The potential
1 q
at this point due to each charge = . There-
4 0 a
1 6q
fore, total potential =
4 0 a

point due to equal charges at opposite corners will


Fig. 20.64 cancel each other in pairs.
So the correct choices are (b) and (c).
6. Refer to Fig. 20.65. Potential at O is
8. Refer to Fig. 20.66. Since the charge on sheet 1 is
1 q q q q
V0 = =0
4 0 r r r r E = /2 0 which points away from it; to the left in
region I and to the right in regions II and III. Sheet
E = /2 0. Since
the charge of sheet 2 is negative, the direction E is
towards it; i.e. to the right in regions I and II and to
the left in region III.

Fig. 20.65
2
a
Refer to Fig. 20.65 again. (AE)2 = a2 +
2
5 a2 5a 5a
= , giving AE = . Similarly DE =
4 2 2
a a
and BE = , CE = . Potential at E is
2 2
1 q q q q Fig. 20.66
VE = =0
4 0 AE BE DE CE
Region I : E1 = E – E = – =0
Work done in carrying a charge – e from O to E is 2 0 2 0
W = – e (VE – VO)
Region II : E2 = E + E = =
= – e (0 – 0) = 0 2 0 2 0 0
Potential at F is directed to the right.
1 q q q q
VF = Region III : E3 = E – E = 0.
4 0 AF BF DF C F Thus the correct choices are (a); (b) and (c).
Electrostatic Field and Potential 20.35

9. r > r0 is given by 12. Force on proton is Fp = eE, in the direction of E and


dV d Q force of electron is Fe = eE, opposite to the direction
E= – =– of E. Also ap = Fp /mp and ae = Fe /me. Since mp >
dr dr 4 0 r
me; ap < ae. So the correct choices are (b) and (c).
Q 13. P are
= 2
4 0r 1 Q
V=
d Q 4 0 ( R r 2 )1 / 2
2
For r r 0, E = – =0
dr 4 0 r0 1 Qr
( r0 = constant) and E= 2
,
4 0 (R r 2 )3 / 2
r = r 0.
Therefore, statement (a) is true. which is zero for r = 0.
For r < r0, E = 0. Hence the charge resides only on Q Q
the spherical surface of radius r = r0. No charge For r >> R, V = and E = . So the
4 0r 4 0r 2
exists in the region for which r < r0. Therefore, correct choices are (b), (c) and (d).
statement (c) is also true. Electric energy density is
given by 14. P is given by
1 2 n
u= 0E E=
2 2 0r
Since for r < r0, E = 0; u = 0 for r < r0. Hence state-
ment (d) is true. Where is the linear charge density and n is a
Let Q be the net charge enclosed inside the spheri- unit vector perpendicular to the wire. So the correct
cal surface of radius r = 2r0. Then from Gauss’s choices are (a) and (d).
theorem, we have 15. P is
Q n
E = E=
0 2 0
Q where n is a unit vector perpendicular to the sheet.
or E 4 r2 = Since E is independent of r -
0 form. Hence the correct choices are (c) and (d).
Q Q 16. All the four choices are correct.
or 2
4 r2 = 17. Initial P.E. of the system [Fig. 20.67(a)] is
4 0r 0
or Q = Q, which is independent of r as long as r is 1 qQ qQ q2
U=
greater than r0. Hence statement (b) is also true. All 4 0 a a 2a
the four choices (a), (b), (c) and (d) are correct
q q
10. For r R, = 2Q (1)
4 0a 2
Qr
E = 3
, i.e. E r
4 0R
For r > R, the entire charge Q may be assumed to
be concentrated at the centre of the sphere. Hence
for r > R,
Q
E = , i.e. E 1/r2
4 0r 2
Since the sphere is made of a non-coducting mete-
rial, charge Q is uniformly distributed over the
entire volume of the sphere. For a unifrom distri-
bution of charge, the potential is maximum at the
-
dicular to the surface of a conductor. Hence the Fig. 20.67
correct choices are (b) and (c). If charge Q is given a displacemet x along the x-axis
11. For a conductor, all the four choices are correct. the P.E. of the system becomes [Fig. 20.67 (b)]
20.36 Comprehensive Physics—JEE Advanced

state of weightlessness. The distance between the


1 qQ qQ q2
U = balls = 2 L. Hence
4 0 (a x) (a x) 2a
Q2
2 T=
=
q
2Q
a q
(2) 4 (2 L) 2
2 2
4 0a (a x ) 2
So the correct choices are (b) and (c).
2 2 2
Since a > (a – x ); it follows from Eqs. (1) and (2) 22. If Q1 is the charge on the ring and – Q2 is the charge
that U > on particle P, the force due to the ring on particle P
If charge Q is given a displacement y along the when it is at (0, 0, z) is
y – axis, the P.E of the system will be [Fig.
1 Q1 Q2 z
20.67(c)] F =–
4 (R2 z 2 )3 / 2
2
1 qQ qQ q
U = When z > 0, F is in the – z direction. When z < 0,
4 0 r r 2a F is in the + z direction. So the motion of P will be
periodic for 0 < z < .
q 2aQ q
= (3) 1 Q1 Q2 z
4 0a r 2 When z << R, F = , i.e. F – z. Hence
4 R2
Since r > a, it follows from Eqs. (1) and (3) that
U < U. So the correct choices are (b) and (c). for z << R, the motion of P will be simple harmonic.
So the correct choices are (a) and (c).
work [ML2 T 2 ] 23. E
18. V = . Therefore [V] =
charge with distance r. [Fig. 20.68]
=[ML2T–3A–1]
[k] = [V] [r]
= [ML2T–3A–1] [L]
3 –3 –1
= [ML T A ]
dV d k k
Now E = – =– = 2
dr dr r r
Hence the correct choices are (b) and (c).
19. Let the electric potential be zero at a point P at a
distance x from charge q1. Then
1 q1 q2 Fig. 20.68
=0
4 0 x (r x)
So the correct choices are (a) and (c).
q1 q2 4 1 24. The surface of a perfect conductor is an equipo-
= tential surface. Charge density at B is greater than
x (r x) x (r x)
at A because curvature at B is greater than that at
4r 4 15 cm A
which gives x = = =12 cm
5 5 q
closed surface = -
E is zero at a value of x given by
0
q1 x 2 face of the cavity is zero. Hence the correct choices
= are (c) and (d).
q2 (r x)2
25. Potential energy at position x is
which gives x = ± 30 cm. It is easy to check that E
1 qp
0 between x = 0 and x = 15 cm. So the correct U =–
choices are (b), (c) and (d). 4 x2
20. All the four statements (a), (b), (c) and (d) are cor- When the dipole is far away (x ), U = 0. Hence
rect.
1 qp
21. The two charges will repel and will come to rest K = change in P.E. = 0 –
when = 180° because an orbiting satellite is in a 4 x2
Electrostatic Field and Potential 20.37

1 qp are wrong because, for elliptical orbit, the speed of


= 2 charge – q is the highest when it is closest to charge
4 x
Q (as in planetary motion). Hence the only correct
dU 2q p choice is (a).
F= =– 28. Let QA and QB are the charges on metal shell A and
dx 4 x3
metal sphere B after they are connected by a wire.
Hence the correct choices are (b) and (d). Since their electric potentials will be equal,
26. r > r0 is given by VA = VB
dV d Q
E =– =– QA QB QA RA
dr dr 4 0 r
4 0 RA 4 0 RB QB RB
Q
= 2
Since RB < RA, QA > QB. So choice (b) is correct.
4 0r -
d Q cal shell is zero. So choice (a) is correct.
For r r 0, E = – =0 QA QB
dr 4 0 r0 Now = and =
A B
( r0 = constant) 4 RA2 4 RB2
r = r 0. 2 2
Therefore, statement (a) is true. A
=
QA RB RA RB
For r < r0, E = 0. Hence the charge resides only on B QB RA RB RA
the spherical surface of radius r = r0. No charge
exists in the region for which r < r0. Therefore, RB
=
statement (c) is also true. Electric energy density is RA
given by Hence choice (c) is also correct.
1 2
u= 0E are
2
Since for r < r0, E = 0; u = 0 for r < r0. Hence state- EA = A
ment (d) is true. 0
Let Q be the net charge enclosed inside the spheri-
cal surface of radius r = 2r0. Then from Gauss’s B
And EB =
theorem, we have 0

Q EA A
E = = < 1, i.e. EA < EB.
0 EB B
Q So choice (d) is also correct. All the four choices
or E 4 r2 =
0
are correct.
29. Gauss’s law is valid only if Coulomb's law holds,
Q Q
or 2
4 r2 = i.e. if E r–2. Hence choice (a) is wrong. Gauss’s
4 0r 0

or Q = Q, which is independent of r as long as r is distribution around an electric dipole. So choice (b)


greater than r0. Hence statement (b) is also true. All is also wrong.
the four choices (a), (b), (c) and (d) are correct Choice (c) is correct becausse the directions of
27. The torque of Coulomb force (which is radial) on
dL similar charges.
charge – q is zero. Hence = 0 L = constant. Work done WA = q(VB – VA) = (VB – VA)
B
dt ( q = +1 C)
Hence the angular momentum of charge – q is con- Hence choice (d) is correct.
stant. So choice (a) is correct. All other choices So the correct choices are (c) and (d).
20.38 Comprehensive Physics—JEE Advanced

III

Multiple Choice Questions Based on Passage


Questions 1 to 4 are based on the following passage 3. If the consecutive charges have opposite sign, the
Passage I electric potential at x = 0 would be
q, are placed along q q
the x-axis at x = 1, x = 2, x = 4, x = 8 .... and so on. (a) (b)
3 0 4 0
1. The electric potential at the point x = 0 due to this
q q
set of charges is (c) (d)
q q 5 0 6 0
(a) (b) 4. If the consecutive charges have opposite sign, the
0 2 0
x = 0 would be
q q
(c) (d) q q
3 0 4 0 (a) (b)
3 0 4 0
2. x = 0 is
q q q q
(a) (b) (c) (d)
3 0 4 0 5 0 6 0
q q
(c) (d)
5 0 6 0

SOLUTION
1. The potential at x 1 q q q q
number of charges placed on the x-axis as shown E= 2 2 2
to
in Fig. 20.69, is 4 0 1 2 4 82
q 1 1 1
= 1 to
4 0 4 16 64
Fig. 20.69
1
q 1
1 q q q q 4
V= to =
4 1 2 4 8 4 0 1
0 1
4
q 1 1 1
= 1 to q (1 0) q
4 0 2 4 8 = = , which is choice (a).
4 0
3 3 0
1 4
1
=
q 2 3. If the consecutive charges have opposite sign, the
4 1 potential at x = 0 is given by
0 1
2
1 q q q q q q
V= to
(1 0)
q q 4 1 2 4 8 16 32
= = , 0
4 0 1 2 0
q 1 1
2 = 1 to
4 4 16
which is choice (b). 0

2. Since the charges are placed along the same straight 1 1 1


x = 0 will be directed along – to
2 8 32
the x-axis and its magnitude is given by
Electrostatic Field and Potential 20.39

q 1 1 1 q 1 1
= = 1 to
4 1 2 1 4 16 256
0 1 1 0
4 4
1 1 1
q 4 1 4 q – to
= = 4 64 1024
4 0 3 2 3 6 0
Hence the correct choice is (d). q 1 1 1
=
4 1 4 1
0 1 1
1 q q q q 16 16
4. E = 2 2 2
4 0 1 2 4 (8)2 q 16 1 16 q
=
q q 4 0 15 4 15 5 0
to
(16) 2
(32) 2 Hence the correct choice is (c).

Question 5 to 9 are based on the following passage q1q2


(c) mg + (d) zero
Passage II 4 0l 2
7. The normal reaction on bead P is
A rigid insulated wire frame
in the form of a right-angled (a) 2 mg (b) 2 mg
triangle ABC, is set in a (c) 3 mg (d) 3 mg
vertical plane as shown in
Fig. 20.70. Two beads of 8. The normal reaction on bead Q is
equal masses m each and (a) mg (b) 2 mg
carrying charges q1 and q2 Fig. 20.70
are conn-ected by a cord of length l and can slide without (c) 2 mg (d) 3 mg
friction on the wires. The beads are stationary. 9. If the cord is cut, the magnitude of the product
IIT, 1978 q1 q2 of the charges for which the beads continue
5. The value of angle is to remain stationary is
(a) 30° (b) 45°
(c) 60° (d) 75° mgl 2 3mgl 2
(a) (b)
6. The tension in the chord is 4 0 4 0
q1q2
(a) mg (b) (c) 3 (4 )mgl2 (d) (4 )mgl2
4 0l 2

SOLUTION

1 q1 q2
F =
4 0 l2
(iv) Normal reaction N1.
The net force along the string is (T – F). Bead P will be in
equilibrium, if the net force acting on it is zero. Resolving
forces mg and (T – F) parallel and perpendicular to plane
AB, we get, when the bead P is in equilibrium,
Fig. 20.71
mg cos 60° = (T – F) cos (1)
Let us consider forces acting on bead P as shown in Fig. and N1 = mg cos 30° + (T – F) sin (2)
20.71. These forces are:
For the bead at Q, we have
(i) Weight mg vertically downwards
(ii) Tension T in the string mg sin 60° = (T – F) sin (3)
(iii) Electric force between P and Q given by and N2 = mg cos 60° + (T – F) cos (4)
20.40 Comprehensive Physics—JEE Advanced

5. Dividing Eq. (3) by Eq. (1), we get N2 = mg cos 60° + mg cos 60° = mg
tan = tan 60° or = 60°, which is choice (c). Thus is the correct choice is (a).
6. Using = 60° in (3), we have 9. When the string is cut, T = 0. Putting T = 0 in Eq.
mg sin 60° = (T – F) sin 60° (5), we get
1 q1 q2
1 q1 q2 mg = –
or T = F + mg = + mg (5) 4 0 l2
4 0 l2
The right hand side of this equation should be
So the correct choice (c).
positive which is possible if q1 and q2 have opposite
7. From Eq. (2) we have (since T – F = mg)
signs. Thus, for equilibrium the beads must have
N1 = mg cos 30° + mg sin 60° unlike charges. The magnitude of the product of the
= 2 mg cos 30° = 3 mg; charges is
which is choice (c). |q1 q2| = (4 0) mgl2,
8. From Eq. (4) we have which is choice (d).

Questions 10 to 13 are based on the following passage 1/ 2 1/ 2


ma3 ma3
Passage III (a) T = 2 (b) T = 2
2kq 2 kq 2
Two charges, each equal to q, are kept at x = – a and x = a 2kq 2
1/ 2
kq 2
1/ 2

on the x-axis. A particle of mass m and charge q0 = q/2 is (c) T = 2 (d) T = 2


placed at the origin. ma3 ma3
12. If charge q0 is given a small displacement y (<< a)
10. The charge q0 is given a small displaceplacement
along the y-axis, the net force acting on the particle
x (<< a) along the x-axis and then released. The
is proportional to
restoring force acting on q0 is (k = 1/ (4 0))
(a) y (b) – y
kq 2 2kq 2 1 1
(a) – x (b) – x (c) (d) –
y y
a3 a3
13. In Q. 12, the particle,
kq 2 2kq 2 (a) will execute simple harmonic motion.
(c) x (d) x (b) will execute oscillatory but not simple
a2 a2
harmonic motion.
11. In Q.10 the time period of oscillation of the particle (c) will execute a non-periodic and non-oscilla-
is tory motion.
(d) will never come back to x = 0.

SOLUTION
10. Refer to Fig. 20.72. Suppose the charge q0 = q/2 is
given at a small displacement x from origin O. Then
the force of repulsion on charge q0 due to charge
q1 = q is
1 q1 q0 q q/2
F1 = 2
k 2
4 0 a x a x
along positive x-direction
The force of repulsion on charge q0 due to charge Fig. 20.72
q2 = q is Since F2 > F1, the net force on charge q0 is along
1 q1 q0 kq 2 the negative x-direction. Hence the restoring force
F2 = 2 on charge q0 is
4 a x 0 2( a x) 2
F = F1 – F2
along negative x-direction
Electrostatic Field and Potential 20.41

q2 1 1 F1 cos along positive y-direction. Similarly, force


= k 2 2 F2 can be resolved into two components (F2)x =
2 a x a x F2 sin along negative x-direction and (F2)y =
1 1 F2 cos along positive y-direction. Since F1 = F2,
since x << a, 2 = 2 components (F1)x and (F2)x are equal and opposite
a x x
a2 1 and hence they cancel each other. The net force on
a charge q0 is along the positive y-direction and is
x 2 given by
1
=
a F = (F1)y + (F2)y = F1 cos + F2 cos
a2 = (F1 + F2) cos = 2F1cos ( F 1 = F 2)
Expanding binomially and retaining only the terms
2
x q cos
of order , we have or F= k
a
2x a2 y2
1
1 a 1 2x OC y
2
= 2
= 2 Now cos = . Therefore,
a x a a a3 AC a2 y2
1/ 2

1 1 2x
Similarly, 2
= 2 q2 y
a x a a3 F= k 3/ 2
along positive y-direction
Hence a2 y2
q2 1 2x 1 2x
F= k 2 3 2
2 a a a a3
2 q2 x
=–k
a3
which is choice (b).
11. Since F (– x), the motion of the particle is simple
harmonic whose time period is given by choice (a).
12. Refer to Fig. 20.73. Suppose the charge q0 = q/2 is
given a small displacement OC = y along the y-axis,
the force of repulsion on charge q0 due to the charge
q1 = q is Fig. 20.73
q q0 q q /2
F1 = k = k
a2 y2 Since y << a, (a2 + y2)3/2 a3. Thus
AC 2
The force of repulsion on charge q0 due to charge q2 y
F= k
q2 = q is a3
q q0 q q /2
F2 = k 2
= k 2 So the correct choice is (a).
BC a y2 13. Since the net force on charge q0 is along the postive
The directions of F1 and F2 are shown in Fig. 20.73. y-direction, it will keep on moving in the positive
Force F1 can be resolved into two components (F1)x = y-direction away from the origin O and will never
F1 sin along positive x-direction and (F1)y = come back. So the correct choice is (d).

Questions 14 to 16 are based on the following passage


Passage IV
A point particle of mass M is attached to one end of a
massless rigid non-conducting rod of length L. Another
point particle of the same mass is attached to the other end
of the rod. The two particles carry charges + q and – q. This

E such that the rod makes a small angle (say of about 5°) Fig. 20.74
IIT, 1989
20.42 Comprehensive Physics—JEE Advanced

14. The magnitude of the torque acting on the rod is 16. The minimum time taken by the rod to align itself
(a) qEL sin (b) qEL cos
(c) qEL (d) zero 1/ 2 1/ 2
15. When the rod is released, it will rotate with an an- ML ML
(a) (d) 2
gular frequency equal to 2 2qE qE
1/ 2 1/ 2
qE 2qE 2 ML
1/ 2
ML
1/ 2
(a) (b) (c) 2 (d) 2
ML ML qE 2qE
1/ 2 1/ 2
qE 1 qE
(c) (d)
2ML 2 ML

SOLUTION
14. A non-conducting rigid rod having equal and I= M (AO)2 + M (BO)2
opposite charges at the ends is an electric dipole. 2 2
L L M L2
=M +M =
experiences a torque which tends to align it with 2 2 2
M L2
forces F = qE each acting at A and B constitute a Thus = (2)
2
couple whose torque is given by
= force perpendicular distance Using Eq. (2) in Eq. (1), we get
= F AC = F AB sin = qEL sin 2q E 2
So the correct choice is (a). =– =–
ML
1/ 2
2q E
where = , which is choice (b).
ML
16. The time period of oscillation is
1/ 2
2 ML
T= =2
2q E
Fig. 20.75
Rotating in the clockwise sense, the minimum time
15. Since is small, sin , where is expressed in taken by the rod to align itself parallel to the elec-
radian. Thus = qEL
Restoring torque = – qEL (1) of angular oscillation, i.e.
If is the angular acceleration of the rotatory 1/ 2
T ML
motion, tmin =
=I 4 2 2q E
where I is the moment of inertia of the two masses So the correct choice is (a).
at A and B about an axis passing through the centre
O and perpendicular to the rod. Since the rod is
massless,

Questions 17 to 20 are based on the following passage 17. At the closest approach, speed v1 of particle A is
Passage V (a) v1 = v (b) v1 = 2v
Two identical particles A and B of mass m carry a (c) v1 = v/2 (d) v1 = v/ 2
charge Q each. Initially particle A is at rest on a smooth 18. At the closest approach, speed v2 of particle B is
horizontal plane and the partical B is projected with a
speed v along the horizontal plane from a large distance (a) v2 = v/2 (b) v2 = v/ 2
directly towards the first particle. The distance of (c) v2 = v (d) v2 = 2v
closest approach is x. 19. At the distance of closest approach, the total energy
of the system is
Electrostatic Field and Potential 20.43

20. The distance of closest approach is


mv 2 mv 2 Q2
(a) (b)
2 2 4 Q2 Q2
0x (a) x = (b) x =
2 2
0 mv 2 0 mv
Q2 mv 2 Q2
(c) mv 2 (d)
4 4 4 Q2 2Q 2
0x 0x (c) x = (d) x =
2 2
4 0 mv 0 mv

SOLUTION
17. Initially particle A is at rest and particle B moves 18. The correct choice is (a).
towards it with a velocity v from a large distance. 19. If x
Therefore, the initial momentum of the system is energy of the system is
pi = mv + 0 = mv
1 1 1 Q2
Since the initial distance is large, the particle B exerts Ef = mv21 + mv22 +
2 2 4 0 x
2 2
1 v 1 v 1 Q2
move in the same direction as the second particle or = m + m +
under the action of the force of repulsion. The 2 2 2 2 4 0 x
distance between them, therefore, keeps decreasing
m v2 1 Q2
until it attains a certain minimum value. Let v1 and or Ef = +
v2 be the velocities of particles 1 and 2 and let t be 4 4 0 x
the time taken by them to acquire these velocities. which is choice (d).
Then the distances travelled by them will be x1 = v1t 20. The initial energy of the system is
and x2 = v2t. The separation between them at this
Ei particle + KE of second parti-
time t is
cle + PE due to charge on each particle.
x = x1 – x2 = (v1 – v2)t
1 1 Q2
This separation will be minimum if dx/dt = 0. =0 + mv 2 + ×
Now 2 4 0
dx
= v1 – v2 1
dt or mv2 Ei =
2
For closest approach, v1 – v2 = 0 or v1 = v2.
From the law of conservation of energy, Ei = Ef.
Therefore
closest approach is
pf = mv1 + mv2 = m (v1 + v2) 1 1 1 Q2
mv2 = mv2 +
From the law of conservation of momentum, 2 4 4 0 x
pi = pf or mv = m (v1 + v2) or v = v1 + v2 which gives
Since the particles are identical, it follows that 1 4 Q2 Q2
x=
v1 = v2 =
v
. 4 0 m v2 0 m v2
2
So the correct choice (c). so the correct choice is (a).

Questions 21 to 24 are based on the following passage


Q2 3Q 2
Passage VI (c) (d)
4 0R 10 0 R
Charge Q is uniformly distributed within a non-conducting
sphere of radius R. The volume charge density is . 22. r
(> R ) from the centre of the sphere is
IIT, 1992
21. The potential energy of the spherical distribution of 3 R3 3 R3
(a) (b)
charge is 0r
2
4 0r 2
3Q 2 2Q 2 R3 3 R3
(a) (b) (c) (d)
20 0 R 5 0R 3 0r 2 2 0r 2
20.44 Comprehensive Physics—JEE Advanced

23. If the same charge is given to a spherical conduct- 24. -


ing sphere of the same reduis R, the potential en- tance r (> R) from the centre of the sphere will be
ergy of the system will be Q QR
(a) (b)
Q2 4 0r 2 4 0r3
(a) zero (b)
4 0R 3Q 3QR
Q2 (c) (d)
(c) (d) the same as in Q.21. 4 2
4 3
8 0r 0r
0R

SOLUTION
Hence, the total potential energy of the spherical
Q Q 3Q
21. = (1) distribution of charge is
4 3
R3 4 R
V
2 2
R5
R
3 4 4
U= r4dr =
Refer to Fig. 20.76. 3 0 0
3 0 5
Using Eq. (1) in this we have
2
spherical shells. Consider one such spherical shell
of radius r and thickness dr. The charge within the 4 Q R5
U=
volume (4/3) r3 is 3 4 5
0 R3
4 3
q= r3
3 3Q2
=
The charge in the spherical shell of thickness dr is 20 0 R
dq = 4 r2dr
So the correct choice is (a).
Therefore, the potential energy due to charges q
22. For points outside the sphere, the charge Q may be
and dq is
assumed to be concentrated at the centre. Hence
1 qd q (For r >R),
dU =
4 0 r Q R3
E= 2
= 2
, which is choice (c).
4 0r 3 0r

23. Any charge given to a solid conductor, cannot stay


in its body; it is immediately transferred to the
surface of the conductor. The electrostatic energy
of a charge Q on the surface of a conducting sphere
of R is given by
Q2
U=
2C
Fig. 20.76
where C = 4 0 R is the capacitance of the sphere.
Thus
4 3 Q2
r 4 r2 d r U=
1 3
= 8 0 R
4 0 r
So the correct choice is (c).
2 24. For points outside the sphere, it behaves if the
4
= r4dr entire charge Q is concentrated at its centre. Hence
3 0
the correct choice is (a).
Electrostatic Field and Potential 20.45

Questions 25 to 26 are based on the following passage 25. Take the electronic charge as ‘e’ and the permit-
Passage VII tivity as ‘ 0’. Use dimensional analysis to deter-
mine the correct expression for p.
A dense collection of equal number of electrons and positive
ions is called neutral plasma. Certain solids containing Ne m 0
(a) (b)
m 0 Ne
treated as neutral plasma. Let ‘N’ be the number density
of free electrons, each of mass ‘m’. When the electrons are
Ne2 m 0
(c) (d)
m 0 Ne2
becomes zero, the electrons begin to oscillate about the 26. -
positive ions with a natural angular frequency ‘ p’ which tion will occur for a metal having the density of
is called the plasma frequency. To sustain the oscillations, electrons N 4 1027 m–3. Taking 0 = 10–11 and
m 10–30, where these quantities are in proper
an angular frequency , where a part of the energy is SI units.
approaches (a) 800 nm (b) 600 nm
p all the free electrons are set to resonance together and (c) 300 nm (d) 200 nm

IIT, 2011

SOLUTION
1/ 2
25. Ne2
26. p=
q1q2 e2 m 0
From F = 2
, dimensions of = [Fr2]
4 19 2 1/ 2
0r 0 4 1027 (1.6 10 )
=
= [ML3T–2] 10 30
10 11

[N] = [L–3]. Hence = 3.2 10 15


rad s–1
Ne2 =
p
. Since c = ,
= [L–3] [ML3T–2] [M–1] = [T–2] p
m 0 2
c 3 108 2
2 1/ 2 = =
Ne –1 15
= [T ] = dimension of p. p 3.2 10
m 0
6 10–7 m = 600 nm

IV

Assertion–Reason Type Questions

In the following questions, Statement-1 (Assertion) is (c) Statement-1 is true, Statement-2 is false.
followed by Statement-2 (Reason). Each question has the (d) Statement-1 is false, Statement-2 is true.
following four choices out of which only one choice is
1. Statement-1
correct.
Figure 20.77 shows the tracks of two charged
(a) Statement-1 is true, Statement-2 is true and State-
partices A and B
ment-2 is the correct explanation for Statement-1
two charged plates. The charge to mass ratio of
(b) Statement-1 is true, Statement-2 is true but State-
B is greater than that of Neglect the effect of
ment-2 is not the correct explanation for State-
gravity.
ment-1.
20.46 Comprehensive Physics—JEE Advanced

7. Statement-1
In Q. 6 above, the work done to move an electron
from P to Q and then back to P is zero.
Statement-2

8. Statement-1

Fig. 20.77 moving an electron around it in a complete orbit is


Statement-2 greater if the orbit is elliptical than if the orbit is
The vertical acceleration of particle B is greater circular.
than that of particle Statement-2
2. Statement-1
A positive charge + q is 9. Statement-1
located at the centre of Electrons move from a region of higher potential to
a circle as shown in Fig. a region of lower potential.
20.78. W1 is the work done Statement-2 An electron has less potential energy
in taking a small positive at a point where the potential is higher and vice
charge + q0 from A to B versa.
and W2 is the work done 10. Statement-1
Fig. 20.78
in taking the same charge The equipotential surfaces corresponding to a
from A to C. Then W2 > W1 x-direction are
Statement-2 equidistant planes parallel to the y-z plane.
Work done = Charge potential difference. Statement-2
3. Statement-1 Electric is normal to every point on an equipotential
A small test charge is initially at rest at a point in surface
11. Statement-1
released, it will move along the line of force pass-
ing though that point. is uniform.
Statement-2 Statement-2
The tangent at a point on a line of force gives the
point charge is a sphere with the charge at its center.
4. Statement-1 12. Statement-1
- If a metallic sphere A of radius r carrying a charge
tial must also be zero at that point. Q is brought in contact with an uncharged metal-
Statement-2 lic sphere B of radius 2r, the charge on sphere A
reduces to Q/3.
potential. Statement-2
A to B until their potentials are
5. Statement-1
equalised.
If electric potential is constant in a certain region of
13. Statement-1
Statement-2 A metal ring of radius R carries a charge + Q
distributed uniformly. A point charge – q is placed
potential. on the axis of the ring at a distance x = 2R from the
centre of the ring. If the charge is released from
6. Statement-1 rest, it will execute a simple harmonic motion along
If an electron is moved from the axis of the ring.
P to Q, its potential energy Statement-2
increases (see Fig. 20.79) In simple harmonic motion, the restoring force act-
Statement-2 ing on the oscillator is proportional to (– x), where
Potential at Q is less than Fig. 20.79 x is the displacement from the mean position.
that at P.
IIT, 1988
Electrostatic Field and Potential 20.47

14. Statement-1 charge Q uniformly distributed on the surface is


For practical purposes, the earth is used as a Q
reference at zero potential in electrical circuits. given by .
4 0R
Statement-2
The electrical potential of a sphere of radius R with IIT, 2008

SOLUTION
1. The correct choice is (a). Let E be the electric it from Q back to P an equal positive work is done

downwords. If q is the charge of the particle, it will So the correct choice is (a).
experience a force F = qE. Hence its acceleration 8. The correct choice is (d). The work done by the
(in the vertical direction) is
F qE path of any shape (circular or elliptical) is zero.
a=
m m 9. The correct choicce is (d). Since the electron has a
where m is the mass of the particle. If t is time spent negative charge, it has less energy at a point where
by the particle between the plates (i.e in the region the potenial is higher and vice versa. Hence in an

lower potential to a region of higher potential.


1 2 qEt 2 10. -
y= at = tential surface. Therefore, for a constant electric
2 2m
q x-direction, the equipotential surfaces
Thus y. Since particle B - are planes parallel to the y-z
m
constant, the equipotential surfaces are equidistant
tion, it has a higher charge to mass ratio.
from each other. The correct choice is (a).
2. The correct choice is (d). Points A, B and C are at
the same distance from charge + q. Hence electric 11.
potential difference between points A, B and C is region around a point charge varies with distance
zero. Hence W1 = W2 = 0. Q
E=
3. The correct choice is (d). The test charge will move 4 2
0r
along the line of force if the line of force is straight
(as in the case of a single charge). If the lines of 12. A
force is curved, the charge will not move along the to B until thier potentials become equal. If charge
line of force. The reason is that the line of force q A to B, then
does not give the direction of velocity, it gives the Q q q
=
direction of the force which is along the tangent to 4 0r 4 0 ( 2r )
the curve at that point.
4. The correct choice is (d). Since E = – dV/dr, if E = q 2Q
which gives Q – q = q= . Hence charge
0, V = constant not necessarily equal to zero. 2 3
5. The correct choice is (a). 2Q Q
6. The correct choice is (c). Since charge of an left on A = Q – = .
3 3
electron is negative, P.E at P and Q is
eq 13.
UP = – x from its centre is given by
4 0 (OP)
1 Qx
eq E=
UQ = – 4 (R 2
x 2 )3 / 2
4 0 (OQ )
Force on charge – q is
Since OQ > OP, UQ is less negative than UP, i.e.UQ
> UP. For the same reason, VQ > VP. 1 qQ x
F=–qE=– 2
7. Since charge + q will attract the electron, work is 4 (R x 2 )3 / 2
done to move the electron from P to Q is negative The motion will be simple harmonic if F – x
which is true only if x << R and not when x = 2R.
20.48 Comprehensive Physics—JEE Advanced

Hence Statement-1 is false, but Statement-2 is 14. Both the statements are true but Statement-2 is not
true. the correct explanation for Statement-1.

Integer Answer Type


1. Two identical charged spheres are suspended 3. A solid sphere of radius R has a charge Q distrib-
by strings of equal length. The strings make an uted in its volume with a charge density = kra,
angle of 30° with each other. When suspended in where k and a are constants and r is the distance
a liquid of density 800 kg m–3, the angle remains R 1
r = is
the same. What is the dielectric constant of the 2 8
liquid? The density of the material of the sphere is times that at r = R a.
1600 kg m–3. IIT, 2009
IIT, 1978 4. Four point charges, each of +q
2. A circular ring of radius R with uniform positive
charge density per unit length is located in the y-z of side ‘a
plane with its centre at the origin O. A particle of is
mass m and positive charge q is projected from the q2
1/ N

point P (R 3 , 0, 0) on the positive x-axis directly in equilibrium, and a = k , where ‘k’ is


towards O, with initial speed v. The smallest (non-
a constant. What is the value of N?
zero) value of speed v such that the particle does
not return to P is IIT, 2011
q
v= , where n is an integer. Find the value
n 0m
of n.
IIT, 1993

SOLUTIONS
1. It follows from Fig. 20.80 that If q is the charge on each sphere and r the separa-
W = T cos and F = T sin tion between them, force F is given by
1 q2
F
= tan (1) F=
W 4 0 r2
When the spheres are immersed in a liquid of
dielectric constant K, the force of repulsion
between them becomes
F
F =
K
Apparent weight is
W = W – (weight of liquid displaced)
or W = W – Vg
where is the density of the liquid. If is the
density of the material of the sphere, W = Vg.
Fig. 20.80
Thus
Electrostatic Field and Potential 20.49

Vg 1 1 Qq
W =W 1 = mv2 +
W 2 4 0 ( A P)
2
W =W 1 Now (AP)2 = (OA)2 + (OP)2 = R2+ 3R =4R2

AP = 2R.
F F 1
=
W W
K 1 1 1 Qq
Therefore, EP = m v2 +
2 4 0 2R
As the angle remains unchanged, it follow from The total energy of the particle when it reaches O
Eq. (1) that
is (since its velocity at O is zero)
F F
tan = = E O = PE at O due to charge Q on the ring
W W 1 Qq
Hence =
4 0 R
1 From the law of conservation of energy, EP = EO.
K 1 = 1 or K =
1 Thus
1 2 1 Qq 1 Qq
mv =
Given = 800 kg m–3 and = 1600 kg m–3. 2 4 0 2R 4 0 R
Therefore,
1/ 2 1/ 2
1 1 1 Qq q
K= = =2 which gives v = =
800 1 4 0 mR 2 0m
1 1
1600 2 ( Q=2 R )
2. The positive charge on the ring is Thus the value of n = 2.
Q =2 R 3. According to Gauss’s theorem
A particle with a positive charge q is projected dV 4 r 2 dr
2 q
towards the centre O of the ring with a certain E 4 r = = =
velocity from a point P (lying on the x-axis) whose 0 0 0
x-coordinate is 3 R. Due to repulsion of charge where q is the charge enclosed inside a Gaussian
Q, the particle will keep slowing down. Now, at sphere of radius r. If E1 r=
the centre O of a ring, carrying charge, the electric R/2, then [see Fig. 20.82]
2 R/2
will be zero when it reaches O. It will then stop R 1
E1 4 = 4 r 2 dr
there and will not return to P. Let v be the smallest 2 0 0
velocity of the particle when it is at P which is just
R/2
O where it comes to rest. [see 4
= k r a r 2 dr
Fig. 20.81] 0 0

R/2
4 k
= r (a 2)
dr
0 0

4 k 3) R / 2
= r (a 0
0 ( a 3)
a 3
4 k R
=
Fig. 20.81 0 ( a 3) 2
a 3
The total energy of the particle at point P is 4 k R
E1 =
EP = KE due to its motion + PE due to charge Q 0 (a 3) R 2 2
on the ring
20.50 Comprehensive Physics—JEE Advanced

q2
F1 = 2
4 0a

q2 q2
F2 = 2
=
4 2a 4 0 2a 2
0

Fig. 20.82
F = 2F1 + 2F2 cos 45°
If E2 r = R, then
2q 2 2q 2
4 k
R
(a 2)
= 2
+
E2 4 R2 = r dr 4 0a 4 02 2a 2
0 0
q2 1
4 k a+3
= 2
2
= (R) 4 0a 2
0 ( a 3)
Force on AB due to surface tension = 2 a. Hence
k
E2 = (R)a + 3 q2 1
0 (a 3) R 2 2
2 =2 a
4 0a 2
E1 1 1/3
Given = q2
E2 8 a= k
Hence
where k is a constant. Hence N = 3
a 3
4k R k a 3
= ( R)
0 (a 3) R 2 2 8 0 (a 3) R 2
2(a + 3) = 32 2(a + 3) = (2)5 a=2
4. Refer to Fig. 20.83.

Fig. 20.83
21
Chapter
Capacitance and Capacitors

REVIEW OF BASIC CONCEPTS CAPACITANCE OF A SINGLE SPHERICAL


21.3
CONDUCTOR
21.1 CAPACITANCE
Consider a spherical conductor of radius r having a charge
When a conductor is given a charge Q, it acquires a poten-
Q
tial V which is proportional to the charge given to it, i.e.
Q i.e. the charge Q
Q V or Q = CV or C =
V
where C is a constant of proportionality and is called the Q
V= .
4 r
Q
C=
V
coul C= 4 r
volt
- capacitance.

–6
21.4 CAPACITORS

21.2 ENERGY OF A CHARGED CONDUCTOR


Q
have a potential difference V
Q = CV
where C is the capacitance of the capacitor and its value

Q is given to a
conductor of capacitance C, the potential energy in its C
Cm
Q is a dielectric other than air, then the dielectric constant of
U = = QV = CV Q = CV
C
Cm
K=
C
21.2 Comprehensive Physics—JEE Advanced

21.5 EXPRESSIONS FOR CAPACITANCE


Parallel Plate Capacitor
- = + + +
C C C C3

K A 21.7 ENERGY STORED IN A CAPACITOR


C=
d As in the case of a charged conductor, the energy stored
where A is the area of each plate and d is the dis-
K is dielectric constant of the
Q
U= = QV = CV
K C
Spherical Capacitor where Q = charge on each plate of the capacitor, V =
A spherical capacitor consists of a solid charged C = capacitance
sphere of radius a -
low sphere of radius b
ab
C= 4 K LOSS OF ENERGY ON SHARING
b a 21.8 CHARGES
3. Cylindrical Capacitor
A cylindrical capacitor consists of two co-axial Q and Q and
having capacitances C and C are connected with each
Kl
C=
b
log e Q Q CV C V
a V= =
C C C C
where l is the length of each cylinder and a and b
where V and V are the initial potentials of the charged
are the radii of the inner and outer cylinders.

d C C
E= V –V
and d having dielectric constants K and K , then C C
FORCE BETWEEN PLATES OF A PARALLEL
A 21.9 PLATE CAPACITOR
C=
d d
K K

21.6 CAPACITORS IN PARALLEL AND SERIES


Q
F=
K A

capacitances. F Q
f= = =
Q = Q + Q + Q3 + A K A K
Q Q Q3 where is the charge per unit area.
and V= = = =
C C C3
21.1
C = C + C + C3 +

Q = C V = C V = C3 V3 =
V = V + V + V3 +
Capacitance and Capacitors 21.3

V
3
d
4

21.2
SOLUTION
–6 –6 Given C C C3
Q = CV C
–6 –3
U= CV = J

V 4
E= –3
=4
d .

NOTE Fig. 21.1

plates SOLUTION
-
- C and C3
- is C = C + C3 = 3

A C Fig. 21.2
C = –6 C
d C
and C

=
Q C C C C
V =
C 6
C= C C = =
C C +
–6 –3
U = CV = =4 J
Let Q , Q and Q3 C , C , and C3
Q C and C are in series, charge on
–3
Alternatively, U = 6
=4 J C = charge on C = Q . Let V and V
C
differences across C and C
V 4 V C
E = 3
=4 Q = C V = CV
d V C 3
But V +V

V + V V
- 3
V
A C Q =CV C
C =
d
–6 e C and C3 are in parallel, potential difference
–6
V across C = potential difference V3 across C3 = V
Q =C V
–4
C Charge on capacitor C = C V
U = C V C and Charge on capacitor C3 = C3V3
C
–6 –3
= J Q C, Q C and Q3 C
21.4 Comprehensive Physics—JEE Advanced

C 3 when
21.3
S S is closed.

SOLUTION
SOLUTION S
–6
C
V

–6 –3
U= CV = =6 J
Notice that
U = U + U + U3 = CV + CV + C 3V 3 .
Fig. 21.4

21.4 C and C3 are in series, charge on C = charge on C3 =


C and C have a total Q -
tion of C and C3
a total capacitance 4.8 Q = C V = C 3V 3 C V = C 3V 3
C and C .
V = 3V3. Also V + V3
SOLUTION
V3
C +C
S is closed, the circuit is as shown
CC
C C

C C
= 4.8 C C = 96

96
C =
C

C C Fig. 21.5
C
C
C =3+4=7
C and C are in series, the charge on C =
21.5 charge on C
C C or CV =C V 3V = 7V
C3 = 3 C4 = 4 7V
V =
3
7V
Also V +V +V
3
V = 3V
C3 and C4 are in parallel, potential difference
across C3 = potential difference across C4

21.6

Fig. 21.3 S S is closed.


Capacitance and Capacitors 21.5

SOLUTION

A and B is
Ca = C + C
C C C C
=
C C C C
Fig. 21.7
6
= = CC
C = 4
C C
Energy stored is
C3C4
Ua = Ca V . 4
J C = 8
C3 C4
CAB = C + C
A and B is
C C 3 7 NOTE
6
Cb = =
C C 3 7
Energy stored is -
4
Ub = CbV . J

21.7
found.
A and B Q -
C Q
C C3 C4 C

principle of charge conservation. Let Q ,Q


the charges on the capacitors in the network and V ,
V
Q = CV Q ,Q ,
... etc. and V , V
Q
capacitance Ceq = , where E is the voltage of the
E

Fig. 21.6
21.8
A and B in
SOLUTION
C
-
C , C , C3 and C4 are
such that the condition
C C3
C C4

potential at P = potential at Q Fig. 21.8


P and Q SOLUTION
connected across A and B C is inef-
fective as no charge collects on its plates.
21.6 Comprehensive Physics—JEE Advanced

C and upper plate of


C is

Fig. 21.10

4
Fig. 21.9 Q = CE = =4 C
3
E When switch S is closed, the potential difference
A and B across C
+Q Q. C
B is q, then
Q+q
q= – Q = – 4 C
VA – VB VA – VP VP – VB
Now the charge on the upper plate of C = C E
Q Q Q
E= =4
C C through A.
CE Q Q–Q Q +Q
21.10
Also VA – VB VA – VP VP – VR VR – VB
Q Q Q Q capacitors C , C , C3, and C4 each of capacitance
E=
C C C 4 E
CE = Q Q –Q Q = 4Q – Q A and B.
Q
7CE
Q=

Q 7C
Ceq = 7
E
[ C

Fig. 21.11
21.9
-
pacitors C C =4 SOLUTION
E Capacitors C3 and C4 are in series, their equivalent
points A and B when switch S is closed capacitance is
C3C4 4 4
SOLUTION C
C3 C4 4 4
When switch S is open, the equivalent capacitance
C is in parallel with C .
is
CC 4 is
C = = =
C C 3 C =C +C Fig. 21.12
Capacitance and Capacitors 21.7

But V +V =E
V is the p.d. across C and V across C , then 3V
+V V
C V =C V
P and Q
3V
4V = 6V V = C3 and C4 A and B

21.11 SOLUTION
C C C3 = 3
- C , plate 4 of C C3 are con-
A and B. nected to point A C , plate 3 of C and plate
6 of C3 are connected to point B C , C and C3

capacitance Ceq A and B


Ceq = C + C + C3
Fig. 21.13

Fig. 21.14

CAPACITANCE OF DIELECTRIC FILLED


21.10
CAPACITORS

Fig. 21.15
21.8 Comprehensive Physics—JEE Advanced

KK A K eq A
A C = =
C = Kt K t d
d
K eq KK
Where =
dielectric constant K d Kt K t

K A K eq KK
C = KC = = d=t +t
d t t Kt K t
KK t t
Keq =
Kt K t
Capacitance of upper dielectric is
K A/ K A
C = =
d d K A K A
C = ,C =
A K d d
Capacitance of lower dielectric is C =
d CC K eq A
C and C are connected C = =
C C d
together, C and C -
KK
Where Keq =
K K A K eq A K K
C=C +C = = d
d

where Keq = K +K Capacitors with K and K are in parallel and this


constant K3
K eq A
C= , Where Keq
K eq A d
C = C + C + C3 =
d
K eq = K K K3
K A K A K3 A3
Where Keq =
A A A3
CHARGING AND DISCHARGING OF A
21.11
CAPACITOR THROUGH A RESISTANCE
A C Consider a capacitor of capacitance C connected in series
C= t = t to a resistor R E -
d t
K d K
A
Where C = t = d, C = KC
d

K A
Capacitance of the left capacitor is C =
t
K A
Capacitance of the right capacitor is C = Fig. 21.16
t
C and C are in series, the capacitance of the Growth of Charge

when t
CC KK A
C= =
C C K K and the potential difference q/C q is the charge at any
tt A
t t t
Capacitance and Capacitors 21.9

of the capacitor rises till t t is given


E
q = CE dq
I= = I e –t/RC
zero. dt
where I = E/R

q= q e –t/RC I
exponentially to a steady Decay of Charge When the charge has attained a steady
q as sho value EC
K
capacitor starts discharging through the resistor, i.e. the

Fig. 21.17 q = q e –t/RC = q e-t/


Time Constant RC
C is in farad and R CR will
t= ,q=q e q t= ,
RC =
q =q e –t/
q -
t= , t. Whether the charge decays slowly or
q= q =q q quickly d RC.
e .
of a CR

q . Whether the charge grows quickly or slowly depends


R
and C CR Fig. 21.18

Multiple Choice Questions with Only One Choice Correct

1. A spherical capacitor consists of an inner sphere C


of radius r = r and the outer sphere of radius C
r r C when the inner
sphere is charged and the outer sphere is earthed
and C when the inner sphere is earthed and the
C /C is

2. C =3
C =9 C is Fig. 21.19
21.10 Comprehensive Physics—JEE Advanced

3. C are con-
q q
V. When key K is
- r 4 r
q q
CV CV
8 r r
CV CV
3 7. r is charged to a
voltage V
V q is the charge on the sphere and the

3V
V

V V
3
8. C and
C are connected in parallel and charged to a po-
tential difference V

Fig. 21.20 capacitor of capacitance C


K
4.
state is reached, the energy stored in C is E and
that stored in C is E E /E is V V
K K
3V 3V
K K 3

9. A parallel plate capacitor of plate area A has a charge


Q
Q Q
A A

Q
o
A
10. n drops, each of capacitance C
Fig. 21.21

5. n 3C nC
n C n C
C is given to the 11. n drops, each of capacitance C and charged to a
potential V

n n 4/3
6. r
n n
r. A charge q is given 12.
to the inner sphere. When the inner sphere is con-
Capacitance and Capacitors 21.11

17.
has a capacitance C, the total capacitance of the -

C C
9
C C
13.
18.

C
C
C
C the plate is
14.
capacitors connected
- 19.
Fig. 21.22
ply. What is the charge
points A and B

Fig. 21.24

Fig. 21.23 9
C
C. 20.
15. A capacitor of capacitance C -
6

J kg K
separation d A of the
C plates can C N
–6 –3
d A
d A
–4 –4
16. d A
–4
d A

plates and increase


21. A parallel plate capacitor of capacitance C is con-
of potential difference across the plates
difference V C
change in the charge on the plates V.

capacitors are connected in parallel to each other


no change in the charge on the plates
21.12 Comprehensive Physics—JEE Advanced

3 26.
CV
S closed, V V
9
CV CV S3 closed, V = V
6
S and S closed V = V
22. E in the annular S and S3 closed V
region of a charged cylindrical capacitor and V

the inner cylinder

where r
r
axis
Fig. 21.25
where r
r 27. A parallel plate capacitor of area A, plate separation
the axis. d and capacitance C -
K ,K
and K3
23. resistor and a
C in this capacitor, then its dielectric constant K is

24. d is inserted in a paral-


lel plate capacitor whose negative plate is at x
Fig. 21.26
and positive plate is at x = 3d -

charge. As x d, = + +
K K K K3

= +
K K K K3
continuously K K
K= K3
K K
decreases and again increases. K = K + K + K3

25. 28.
charges Q and Q Q capacitances C and C C > C -
-
ed in parallel is -
pacitor with capacitance C, the potential difference 6

Q Q Q Q C /C is
C C 3 4
Q Q Q Q 3
C C
3 6
Capacitance and Capacitors 21.13

29.
capacitor A has a charge q on it whereas B is

Fig. 21.29
Fig. 21.27

q 34. exists in the region


q q
-
30.
C V and the plates is
other to V of the capacitors are
connected together. When the positive ends are also
35. -

C V –V C V +V
4 4

C V –V C V +V
4 4

31. -
36. A capacitor of capacitance C -

and this capacitor is connected to a second un-


charged capacitor of capacitance C
32.
C = C = C3 = C4 = 4 C - of the total energy stored in the capacitors after

connection is

3
37.

plate is A
is d
Fig. 21.28
A A
33. A and B in
d d
3 A 4 A
d d
21.14 Comprehensive Physics—JEE Advanced

43.

Fig. 21.30
44.
38.

plate is A
d 3
A A
d d
3 A 4 A 3
45. A and
d d
B C
and C = 3

Fig. 21.31

39. A capacitor of capacitance C -


V -
pacitor of capacitance C
V
Fig. 21.32
46.
R
,R ,C C =4
40.
K and con-
ductivity
K
K
K
K
41. Fig. 21.33
plates of a parallel plate capacitor increases its
d is the separation of
the two plates of the capacitor, the thickness of the

d d 47. A capacitior of capacitance C = C is charged to


3 9
7d
d
9 capacitances C = C and C3 = C
42.
CV CV
3 3
CV
Capacitance and Capacitors 21.15

48. R is increased d
constant K initially.
3
radius R R /R is
3 speed V t is
4
3
49. A perallel plate rapacitor of plate area A and plate
separation d V.

d is
AV AV
d d Fig. 21.35
6 R
AV 3AV
d Vt
d d
50. One plate of a parallel plate capacitor of plate area d 9Vt R
A and plate separation d is connected to the positive d dVt 9V t
V 6 R
d Vt
d 9Vt R
plates is d dVt 9V t
AV
d 52.

Vd AV the switch S
A d

Fig. 21.34

51. A parallel plate capacitor C with plates of unit area Fig. 21.36
and separation d

ANSWERS

1. 2. 3. 4. 5. 6.
7. 8. 9. 10. 11. 12.
13. 14. 15. 16. 17. 18.
19. 20. 21. 22. 23. 24.
25. 26. 27. 28. 29. 30.
31. 32. 33. 34. 35. 36.
37. 38. 39. 40. 41. 42.
43. 44. 45. 46. 47. 48.
49. 50. 51. 52.
21.16 Comprehensive Physics—JEE Advanced

SOLUTIONS

rr P.D. across C
1. C = 4 C
r r
E = C V
r
C =4
r r
E = C V
C r r
= = = E C
C r r = = V =V
E C
2.
Q 5.
Equivalent capacitance is Q = CV C
CC 3 9 9 C and
C = = =
C C 3 9 4
C is given to the positive plate,
9 C. Let Q
Q = CV = C
4 -
3. tive plate loses a charge Q and the negative plate
C C gains a charge Q such that the total positive charge
K is open, the total on the positive plate = total negative charge on the
CC C C C negative plate, i.e.
capacitance = = = Q Q
C C C C 3
Charge on capacitors is Q

CV
Q = Q C
3 V = =
C

6.
C =4 r

C =4 r r
Before connection, the total energy is
q q
U = =
C 8 r
Fig. 21.37
After connection, the entire charge q of the inner
When key K is closed, capacitor C is short- -
C. ergy after connection is
C is q q
Q = CV U = =
C r
Q –Q U –U
CV CV
= CV – = q q q
3 3 = =
r 6 r r
4.
7. Charge on the inner sphere is
. Current in the circuit is q = CV = 4 rV
6
I =
V =V –V
Capacitance and Capacitors 21.17

q
= 11. E = CV , E = C V
4 r r
q V E C V
= . = = =n n =n
4 r E C V
Q = CV = C V

8. C 12.
C=C C = 3C Q = 3CV. When capacitors, each of capacitance C connected in par-
capacitor C -
KC 13.
C = KC C K C.
Q, the potential

Q 3CV 3V -
= =
C K C K
-
9.
method of
virtual displacement.
14.
W d in the
their total capacitance is 6
plate separation d to the resulting change U in the
capacitors in series each of capacitance 6
stored energy, i.e.
W= U
F -
plates, then the work W done to increase the plate
d
W= F d
Now we know that the energy U of a parallel-plate
15. Energy stored in the capacitor is
capacitor of plate area A and capacitance C is
4
Q Q d CV = C C
U= =
C A
where Q
mc
increase U in U due to an increase d in d is,

Q d 4 –4
C C
U=
A 16.
Q on the plates does
C
Q Q = CV, the potential difference V
F =
A
stored energy = QV, a decrease in V results in a
10. r its radius,
4
m= r3
3
n 17.
4
R R3 .
nm =
3 A
C =
R=n r. Now, the capacitance of a sphere d
When a dielectric of dielectric constant K is intro-
C =n C
21.18 Comprehensive Physics—JEE Advanced

K A C 21. Q = CV and Q C V CV
C = , =K capacitors are connected in parallel such that the
d C
plates of opposite polarity are connected together,
Now Q = C V and Q = CV
V C Q Q 4CV CV
= = V = = =V
V C K C C C C
But V/V = K = 8. Equivalent capacitance C = C C = 3C -
8
18. t and dielec- 3
U = C V = 3C V = CV
tric constant K
a parallel plate capacitor, the potential difference 22.
region of a charged cylindrical capacitor is given
V =E d t
K E=
V, the separa- 4 r
d where is the charge per unit length and r is the dis-
E .
t r
d =t or K =
K t d 23. -
R
= = RC is zero.
.6
19. 24.

25. Q
and Q are
Q Q
E = and E =
A A
Fig. 21.38

Q >Q
3
A and B E= E – E = Q –Q
A
K A d Q Q
20. C = V = Ed = Q –Q
d A C
d K 8.8 A
= = –3 C
A C . 6 d
26. When switch S3 is closed, the potential difference
V across C and C
E=
d of V and V -
V V
d = = = 6 A K AK
E x dielectric strength 27. We have C = = ,
–6 d d
= 6.67
A K AK
C = =
d d
Capacitance and Capacitors 21.19

AK AK U U Q Q
and C3 = = =
d d U Q
C and C are in parallel and their U U
equivalent capacitance is Given
A U
C =C +C = K +K get Q Q Q –Q Q.
d
C3 Given Q – Q Q Q =
capacitance is
d d 32.
= + = +
C C C3 A K K AK
d
=
A K K K3
AK
or C = where = +
d K K K K3

C C
28. Given C + C = Fig. 21.39
C C 6
C +C CC C C4
C C3
or 6C + 6C CC CC
C , it is thus inef-
or 6C + 6C CC fective. Capacitors C and C4 are in series, hence
Let C =xC
equivalent capacitance of capacitors C and C3
6C + 6x C xC
or 6x x points A and B
3
which gives x = or C >C ,x= is 33. -
3 3

29. B voltage is
A to B V=V –V
30. Ui = U + U = CV + CV As the capacitors C and C are in series, the
When they are connected, the potential across each
is V = V +V C C C 3 6
V V 6
Uf = CV + CV = CV = C or C=

Decrease in energy = Ui – Uf
Q = CV C
= C V +V C V +V
4 Potential difference across A and B = potential
difference across capacitor C
= C V –V Q C
4
C
Q Q 34.
31. U = and U =
C C
V=E d
21.20 Comprehensive Physics—JEE Advanced

A A is
C= or A C Q Q Q Q Q
d . Uf =
K and thickness C C C C C
t Q
A . C C But Ui =
C = C
d t . . Uf
K 4 =
Ui
Now Q = CV = C V
37. -
CV CV V
V =
C C/ parallel, each of capacitance
35. A
A C=
is C = d
d
Q is the charge on the plates, the potential capacitance is
difference is
Q Qd 3 A
V= C = 3C =
C A d
Let d
t and dielectric constant
K is introduced, the new capacitance is
A
C =
d t
K
Q
difference is Fig. 21.40
Q d t
Q K 38.
V =
C A
Given V = V

d=d –t or d – d = t
K K capacitance C = A/d
Given d = d t the equivalent capacitance is
A
C= C=
K d
which gives K

36. Q is the initial charge on capacitor C , the initial

Ui = Q C
When the two capacitors are connected together,

on each capacitor is Q -

Fig. 21.41
C = Q/V C = 39.
C =C Q =Q =Q –6 –3
Q =C V C
Capacitance and Capacitors 21.21

.
hold is
–6 –3
Q =C V C
44. Given and
C C C3 C C 3
-
–3 C3
pacitor cannot hold a charge of 8 C; it can
–3
45.
–3
6
capacitor is
3
C
V = 6

-
stand = V + V = 6 kilovolts + 3 kilovolts =
Fig. 21.42

40. Let A d the plate separation. 46. Let R and C -


tance of the circuit.
RA For Circuit 1 R = R + R
=
d
C=C +C
where R
conductivity is = RC = 3 s
d d R R
= or R= For Circuit 2 R = = =
RA A R R 3
C C 4
K A C= = =
C= C C 3
d
4 8
= RC = = s
capacitor is 3 3 9
d K A K
= RC = R R
A d For Circuit 3 R = = =
R R 3
A C=C +C
41. C=
d = RC = 6= 4 s
thickness t 3
A
C = . Given C C
d t 47. Charge on C is Q = C V = CV
-
42. U = CV , U = C V = CV C and C3 is
CC C
U U C = = C = C3 = C
C C
U
C and C is
Q CV CV
43. U = CV U = CV V V = = = = V
C C C C C C 3
dU CV V V C is
U CV V
CV
Q =C V =
3
21.22 Comprehensive Physics—JEE Advanced

d
wires is is d = x and the thickness of water col-
3
CV CV
Q =Q =Q = CV – = , which is d
3 3 d = x
3
capacitance is
48. C =4 R
A
4 RR Ceq =
C = d
R R d
k
Given C = 3C d
= RCeq Putting d = x ,
4 RR 3
=3 4 e R d
R R d = x ,K A x = Vt we
3
R 3
which gives = 6 6 R
R get Ceq = =
d Vt d Vt
Q
49. Energy stored initially is Ui = . if d 52. When switch S
C
C C d is
V
Q
Uf =
C U = CV V
Work needed is
=V J
Q Q Q When switch S
W = Uf – Ui = CV Q = CV
C C C
A potential is
Now C =
d q CV
V =
C C C C
A V
W= V V
d = volt
50. -
drawn as shown in
U = C C V
on the capacitor plates
is V V
= J
A V
Q = CV =
d Percentage loss of energy is
Fig. 21.43 V
V
U U
51. t x x = Vt. =
U V

II

Multiple Choice Questions with One or More Choices Correct


1. A parallel plate capacitor is charged and the
of insulating handles
Capacitance and Capacitors 21.23

QE QE

6. When a capacitor of capacitance C is charge to


a potential V , the energy stored in it is U . When
increases this charged capacitor is connected to an uncharged
capacitor of capacitance C
2. is V U.

point V C V C
V C C V C

U C U C
point U C C U C

7.
3. A parallel plate capacitor of plate area A and plate V -
separation d is charged to potential difference V and

constant K the potential difference


Q, E and
W the charge on the plates is increased

increased
the energy stored in the capacitor increases.
8. A parallel plate capacitor is charged to a voltage V
AV V
Q= Q= KA
d d
V AV
E= W=
Kd d K
decreases
4. A parallel plate ai -
creases
and energy associated with this capacitor are giv- the energy stored in the capacitor decreases.
Q , V , E and U respectively. A dielectric 9. A parallel plate capacitor is charged to a voltage V.

-
Q, V, E and U
are related to the
Q>Q V>V the charge on the plates is halved
E>E U>U
unchanged
the energy stored in the capacitor is dou-
5. A parallel plate capacitor of plate area A has a
charge Q
10. -
E
V
Q Q
halved,
A A
21.24 Comprehensive Physics—JEE Advanced

halved C C.
C is 8 C.
unchanged A and B is
the energy stored in the capacitor is dou-
14. C, are con-
V
11
S

Fig. 21.46

is C and the energy stored in the capacitors is U,


then
C
C C C =
Fig. 21.44
U= CV U= CV
A CV. 4
B 15. A and B of capacitances C and
V/3. C
CV . 3 respectively are connected in series. When this
3
CV . supply, the potential difference across capacitor A
12. exists in the region
A
A and B are
constant 4 is inserted parallel to the plates, the V and V
C
V C 3
thickness x
V. C 3
V V C
x x V V
13.
V V

16.

Fig. 21.45 Fig. 21.47


Capacitance and Capacitors 21.25

A and B
x and of the other
x
total capacitance

–4
J.
17.
attains a steady value, then 20. A are arranged
fc is zero. d apart.
V
capacitor is Q is the charge on plate
3 Q4 on plate 4 then

rgy stored in the capacitor is nearly


–8
J.

Fig. 21.50
Fig. 21.48
AV AV
Q = Q =
18. d d
attains a steady value, then AV 4 AV
Q4 = – Q4 =
d d

C.
–4 21. Capacitor C C
J.

separately allowed to discharge through equal


t

circuits is zero at t

at t

t
C
Fig. 21.49 sooner then capacitor C
initial charge.
19.

ANSWERS AND SOLUTIONS

1. voltage V across capacitor plates increases due to


its capacitance C decreases. Now Q = CV
charge Q
21.26 Comprehensive Physics—JEE Advanced

V increases and Q 5.
on each plate of the capacitor is
QV also
Q
F=
A
V
2. - E=
tween the plates it is d
Q Qd A
Q 8.8 Now V = = C
E= = C A d
A 8.8 Q
E=
A
3. F= QE.

AV
Q = CV = 6. Q =C V .
d
they are connected. Q = Q + Q . Using Q = CV,
V/d
we have C V = C V + C V, which gives
K
V C
V =
E= V C C
Kd

U = CV
Q A V d AV
U = = =
C d A d
U = CV + CV

Q KA
U = where C =
C d U C
=
U C C
AV
=
Kd 7. Due to the introduction of dielectric the capacitance
AV C
Work done W = U – U =
d K V, the voltage of
Q = CV, the charge on the plated
4. C increases and V
V/d
V and d re
V=V in the capacitor is U = CV . U
increases capacitance C Q CV
Q>Q d C increases and V
unchanged, and V
E = V/d E=E 8. Due to the introduction of dielectric, the capacitance
C
energy stored U = CV increases C
disconnected, the charge Q on the capacitor plates
U>U
are correct.
Capacitance and Capacitors 21.27

V = Q/C Q
V
C increases, V = 3C V + 3C = CV
E = V/d energy 3 3

stored in the capacitor is U = QV Q U –U= CV – CV = CV .


3 3
V decreases, U
12.
all the four choices are correct.
9. Charge Q V=E d
capacitance C = A/d d is
V = Q/C A A
the capacitor is U = Q C. U C C= = or A
d .
Q
K and thickness
t
A
C =
d t
10. K
. C C
= =
unchanged equal to V, the
. .
4
capacitance C Q = CV
that Q E = V/d, E Now Q = CV = CV
V d is halved. Energy CV CV V
V = = =
stored U = CV C C C/

and V =

11. When switch S closed, the potential differnce across x


capacitors A and B V
the charges on capacitors A and B are Q = Q = CV. x x -
When the dielectric is introduced, the capacitance pacitance is
A . C
C = KC = 3C. C = =
x x
After the switch is opened, the potential difference
across capacitor A V volt. Let V Now Q = C V = C V = Given V = V
potential difference across capaciteo B. When the C =C
S . C C
or = = or =
charge on capacitor B Q. x x
which givens x
CV CV V
Q = CV = C V or V = = = volt
C 3C 3 13.
introduced, is
effective voltage is
U= CV + CV = CV V=V –V
As the capacitors C and C are in series, the effec-

introduced, is
= = =
U = CV + CV C C C 3 6
21.28 Comprehensive Physics—JEE Advanced

6 17.
or C = tors cf
is
Q = CV C
Potential difference across A and B = potential dif-
ference across capacitor C
Q C
= =
C

14. Fig. 21.51


pacitance is C = C
circuit is
U= CV = CV V V
4 I= = = A
R R 3 3
15. Given V V V is the potential difference across the capacitor,
then applying Kirchhoff’s loop rule to loop abcfa
Q, then
Q=C V =C V we have
C V – V – IR + V + V
=
C V 3 +V
When the dielectric of capacitor A 3
V= volt
3
–6
C = C Charge on capacitor = CV
3
–6
B C.
C =C
Energy stored is U = CV
C C
=
C C 3 3
–6 –8
V and V are the new potential differences = J
3
across A and B, then
V C
= 18.
V C 3 BC and AD
Also V +V of the current is A B D C A through the
V
current in the circuit is
16.
I=
capacitors each of capacitance 6
Now, p.d across A and C
is, therefore C AB
6 BC

C. AD. Charge on each capacitor is Q = CV =


C.
Energy stored in the circuit = CV –6
Energy stored = CV + CV = CV
–6 –4
= J –4
J
Capacitance and Capacitors 21.29

A A A
19. C = ,C = Cb = = Cb
d d d
A
Ca = C + C =
d
C x and of 20.
C x, then
A A
C = and C = AV
d x d x Q = CV =
A d d
Ca = C + C =
d x
A C C.
=
x
d AV
d Q4 = – C V = –
Ca > Ca d
wrong. -
CC A
Cb = =
C C d
V
21. At t ; V = voltage
Cb = C C R
C C of e–t/ where = RC,
A
Now C C =
d x d x
Ad
And C +C =
d x d x

III

Multiple Choice Questions Based on Passage

Questions 1 to 4 are based on the following passage AK


Passage I C=
d t
A parallel plate capacitor
A
C=
t
each of area A, separated d
d. A dielectric K
A
C=
area A and thickness t and
dielectric constant K is d t
K
introduced with its faces
parallel to the plates as A
C=
d t
1. K
Fig. 21.52
21.30 Comprehensive Physics—JEE Advanced

2. K = 3, for what value of t/d will the capacitance


3

3 3 4
3 4 4.
4
3. K = 3 and t/d

capacitors is

SOLUTION
1.
of two capacitor one of thickness t C
dielectric of dielectric constant K and the other of =
Ca t
d–t -
d K
tances respectively are
KA Given K = 3 and C/Ca
C =
t
A t
and C =
d t d 3
C t 3
CC which gives =
= + or C = d 4
C C C C C 3. Putting K = 3 and t/d =

we get 3 A
C= .
A d
C= A Ca
d t Now Ca = Q
K d C
Q Q U
Ua = and U = = , which is
2. Ca C Ua 3
A
Ca = 4.
d

Questions 5 to 7 are based on the following passage


Passage II

C.
5. A and
B is

6. A and B is

Fig. 21.53
Capacitance and Capacitors 21.31

7. A and C is
8
3 3

SOLUTION
5. C of the parallel As the 3
parallel, the potential difference across each is the
C
C3 - –6
Charge on 3
–6
C3 =6 =6 C
–6
Now C and C Charge on 4
–6
capacitance C =8 C=8 C
C and C =
= C +6 C+8 C.
C C C 4 3
C
or C = 3 C of the Potential difference across C =
C3 and C4 4
AB
=
C C3 C4 6 3
or C
A and B 7. C3 and C4 is C
C and C = C + C C3 and
–6 –6
C4
6. C =
–6 C
C. A and C is = =
3 3
6
= 6

Questions 8 to 12 are based on the following passage 8.


Passage III through the circuit is
C
A and B with capacitances 3
C
9.
capacitor A
capacitor C throught a switch S
C
C
10.
capacitor B
C C
C
11.
capacitor C
C
C

Fig. 21.54
21.32 Comprehensive Physics—JEE Advanced

SOLUTION
8. Q Q Q Q
through the circuit. When the switch is pressed, the 6 6 6
voltages developed on capacitors A, B and C are
–6
Q Q which gives Q
VA = 6
volt, VB = 6
volt and

Q 9. A Qi A Vi Ci A
VC = 6
volt. 3 C
A is
Qf A Qi A –Q C,

10. B is
Qf B Qi B –Q C
C

11.
Qf A Qf C C
Fig. 21.55
Qf C Qf A
Applying Kirchhoff’s law to the loop, we have
VA + VB – VC C,

Questions 12 to 15 are based on the following passage 12. R is


Passage IV
E
r R =6 ,R 13. R is
E r C =4
C
14.

3
4
3
15. C is
Fig. 21.56 C C
C C
SOLUTION
12. Current through R is 14. V=V +V
E CC
I = = Effective capacitance C =
R r C C
Potential difference across R is V = I R 6 4
= =
3
13. Current throught R is
E
I = = =3A 4
R r . . 15. Charge Q = CV = –6 –6
C
3
Potential difference across R is V = I R
C
=3
Capacitance and Capacitors 21.33

Questions 16 to 18 are based on the following passage Qrb Qra


Passage V
ra rb
ra
and rb Q rb ra Q rb ra
+ Q and the inner sphere is earthed. rb ra
16. Out of Q, a part Q which will appear on the outer 18.
surface of the outer sphere is
Q rb ra Q rb ra 4 rb 4 ra
C= C=
rb ra ra rb
Qrb Qra 4 ra 4 rb
rb ra rb ra C= C=
rb ra rb ra
17. Q which appears on the inner surface
of the outer sphere is
SOLUTION
16. Out of charge Q, a part Q A is earthed, VA
surface and another part Q Q Q Q Q
surface of the outer shere such that or Q
ra rb rb rb ra rb
Q+Q =Q
Q =Q–Q
Q
ra rb Q
Q–Q
Electric potential at shere B is ra rb rb
Q rb ra
which given Q =
rb

Q rb ra Qra
17. Q = Q – Q = Q – =
rb rb

18. A and B is
Fig. 21.57 V = VB – VA = VB VB
Q Q Q
VB = Q Q rb ra
4 rb rb rb V= =
Q 4 rb 4 rb
=
4 rb
Electric potential at the inner sphere A is Q 4 rb
C= =
Q Q Q V rb ra
VA =
4 ra rb rb

IV

Assertion–Reason Type Questions


-

following four choices out of which only one choice is -


correct.
21.34 Comprehensive Physics—JEE Advanced

Statement-2

1. Statement-1
4. Statement-1
source supplying a constant voltage V
are kept connected to the source and the space

charge on the plates will increase. Statement-2


Statement-2

plates. on the shape and size of the conductor.


2. Statement-1 5. Statement-1

voltage V
plates of a parallel plate
the energy stored in the capacitor will decrease. capacitor charged to a
Statement-2

3. Statement-1
Statement-2
Fig. 21.58
-

in the capacitor will decrease.

SOLUTION
1. 3. Q
constant voltage V
V capacitance C
C increases due the plates is increased. Now, energy stored U =
Q = CV, Q C Q C is decreased,
the charge Q on the capacitor plates will increase. U will increase.
2. Q on the 4.
5. E

C increases due charge q is F = qE E is constant, force F is the


to the introduction of the dielectric. Now, energy
stored U = Q C Q
C increases, U will decrease.

Integer Answer Type


1.
-
Capacitance and Capacitors 21.35

2.
R R ,
C C =4

SOLUTION
1. Q is the charge on the plates, the potential differ- Given V = V
ence is
d =d –t or
Q Qd K
V =
C A
d –d =t
Let d K
t and dielectric constant Given d – d t
K is introduced, the new capacitance is
A K
C =
Which gives K = 3.
d t
K R R
Q 2. R =
R R 3
difference is
C =C +C
d t
Q K = RC = 6=4 s
V =
C A 3
Electric Current and D.C. Circuits 22.1

22
Chapter
Electric Current and
D.C. Circuits

REVIEW OF BASIC CONCEPTS positive charge Conductor negative charge

22.1 ELECTRIC CURRENT (+) (–)

Fig. 22.2
current is said to be steady or constant
DRIFT SPEED OF ELECTRONS IN A
q 22.2
I= CONDUCTOR
t
q
t.

vd

E
Fig. 22.1
eE
vd =
m

dq
I=
dt
Convention regarding direction of current Relations between drift speed and current
- I in a conductor as
I
vd =
ne A
n
e
A = cross-sectional area of conductor
22.2 Comprehensive Physics—JEE Advanced

NOTE
= +…
R R R R3
.

22.3 OHM’S LAW


Ohm’s Law states that
EMF, TERMINAL VOLTAGE AND
22.6
INTERNAL RESISTANCE OF A CELL
.

V I or V = R I
E
R

22.4 ELECTRICAL RESISTIVITY


V

r r
A
R or R=
A A E r
RA connected to an external resistance R.
or =

is
called the

unit of R unit of A
Unit of =
unit of tre

Hence Fig. 22.3

Total resistance of the circuit = R + r. The current in the


circuit is
E
I=
R r
22.5 RESISTORS IN SERIES AND PARALLEL Potential difference across r is v = Ir
Potential difference across R is V = IR. V is called the
- v
tance R internal resistance.
Thus E= V + v = IR + r
R = R + R + R3 + … V = E – Ir

22.7 GROUPING OF CELLS

E and E and internal


resistances r and r
Electric Current and D.C. Circuits 22.3

nE
I=
R nr
n
Fig. 22.4 E
nE
I=
nR r
E =E + E n
and r =r + r m
For n cells in series mnE
I=
E = E + E + … + En nR m r
and rn = r + r + … + rn
22.1

SOLUTION
N
e

Fig. 22.5 q Ne
q = Ne and I = . Thus
E E E t t
= It
r r r N= electrons
e .
=
r r r
For n 22.2
E E E En
= ... –7
r r r rn

free electrons
and = ... 3
r r r rn
E SOLUTION
n
I
nE and the total internal vd =
resistance is nr r is the internal resistance of en A
each cell. .
= 7
E –3

L .
t=
internal resistance. vd 3
3. Cells should be connected in series if the external
resistance R r.
R < r. 22.3
R -
r.
n
E
22.4 Comprehensive Physics—JEE Advanced

-
tance of the tube. t t

SOLUTION SOLUTION
dq
I= dq = Idt t + 3t dt
dt
t
to t
I N + Ne e
q= t t dt

3 3
= t t
V 3
R=
I . 3 3
=

22.4
=

3
7 22.6

SOLUTION

L
= SOLUTION
RA
L increases and
L L V A
R= and I =
A A R
AL = constant. Thus
V A
I= AL =AL
L AL A
But I = enAvd L L . Therefore A =
L
V A L
enAvd = R =
L A

V L
vd = R =
Lne A
7
=
R L A
–3 =
R L A
R R
22.5
The current I NOTE
t R is stretched to n
I t + 3t n R.
Electric Current and D.C. Circuits 22.5

22.7
R L A
=
R L A
AL = constant
- A L A L
A L A L
SOLUTION
R = = 3 R
=
L
3 R L
nR 3=

22.9
= -
R
R resistance and

22.8

SOLUTION
AL = constant A L =A L
.
L =L + L =L

L A
A =A
L Fig. 22.6

L L
R = and R = . Therefore SOLUTION
A A
R L A
= of R R3 and R is R
R L A
= R
R

R R
R=R –R R
R
R

Simple Method Fig. 22.7

L Current in the circuit is


L E
used. I=
R R r
L
R=
A Potential difference across R is V = IR
R L A
22.6 Comprehensive Physics—JEE Advanced

Potential difference across R is V = IR


R R3 and R since these resis- resistor. There-
R3
is
V
I3 =
R3

v = Ir
Fig. 22.9
V =E – v
Current I =
22.10 Potential difference across R is
Calculate the steady state current in the 3 resistor in V = IR
resistor. Hence the
.
resistor is
3

22.11

and a

resistance of the battery.

SOLUTION
Fig. 22.8 E =IR + r
E r
SOLUTION
and E r
and
3 is
r r r
R =
r E

22.12 SOLUTION
Let R

Fig. 22.10
Electric Current and D.C. Circuits 22.7

PQ
22.14
be R
A and B

S
S is closed.

Fig. 22.11

R
RPQ = Fig. 22.14
R
A and B is
R SOLUTION
RAB R
R S
R
=R
R
R R
R
E Fig. 22.15
Current I =
R r
RAB =
22.13 S
A and
B
has a resistance of 3 .

Fig. 22.16

Fig. 22.12 RAB

SOLUTION 22.15
I

Fig. 22.13

= RAB
RAB 3 3 3 Fig. 22.17
22.8 Comprehensive Physics—JEE Advanced

SOLUTION
IR

A and B is Sign Convention for emfs and Voltage drops

R = . =3 bcdeb and

C and D is
3 3
R=
3 3
V
Current I =
R .

Fig. 22.20

Fig. 22.18

22.8 KIRCHHOFF'S LAWS

First Law or Junction Rule that cell.

IR

junction A.

junctions at b and e
b or e
I + I = I3
and bcdeb

E – I 3R 3 – I R
Fig. 22.19
and – E + I R + I 3R 3
I + I – I3 – I – I E E R R and R3
I + I = I3 + I + I I I and I3

NOTE
Second Law or Loop Rule

I and I I3
Electric Current and D.C. Circuits 22.9

I3 I
BCDEB
- –I R + E – I3R3
– 3I I3

I +I I
22.16 7I I
and – 3I I +I
I. I I

I = I =

and I3 = I + I =

B and E
B E.

VB – V E = I 3R 3 =
Fig. 22.21

SOLUTION 22.18
P E E -
PQ nal resistances r and r
are connected to an external resistor R as
Q QR

A and B
R
I I A and C
R.
22.17

I I and I3
B and E
E E R R = 3 and
R3 Fig. 22.23

SOLUTION
We choose any direction for the current I in the cir-
I

I
Fig. 22.22

SOLUTION
B
I + I = I3
ABEFA
I 3R 3 – E + I R Fig. 22.24
22.10 Comprehensive Physics—JEE Advanced

ABCDEA B and D are


– E + Ir + E + Ir + IR
I I I
I
I I is
I
I

Fig. 22.25
Fig. 22.27
VA – VB = – E – Ir
A Condition for balanced Wheatstone’s Bridge
B.
VB –VC = E – Ir ABDA and BCDB
B C.
VR = IR IP+IG–IR
I –I Q I +I S–IG
22.19
G
I
I I
I R
IP–IR
I P
I S
and IQ–IS
I Q

P R P Q
= or =
Q S R S

Fig. 22.26

SOLUTION
VA – VD I – 3I I
I
But VA = VD A and D are earthed.
I I

22.9 WHEATSTONE’S BRIDGE


P Q R and S
Fig. 22.28

A and C AD =
B and D P Q R and S the resistor R DC
= S AC
Electric Current and D.C. Circuits 22.11

AD and DC
22.21
and . For a balanced
A and
B
P R
=
Q S

22.20
A and B

R.
Fig. 22.31

Fig. 22.29

SOLUTION SOLUTION

A and B
C and D because V across A and B -
C and D rent I V
V = IR.

A and B
R R.

Fig. 22.32

Fig. 22.30

R R ABFGA ACDFA
RAB = =R and DEBFD
R R
V I–I I–I +I
22.12 Comprehensive Physics—JEE Advanced

I +I I–I R R 3
=
I –I I–I +I I R R

3I – 3I I =V R 3
= R
I – 3I – I 3 R
I – 3I I and R = R =
I 3 3
I I = .
22.10 THE POTENTIOMETER
I I = I.
AB
I and I
E E is a
V 7
7I V RAB
I
22.22

resistances R and R AC

A resistance is
R
D such that R and R .
Fig. 22.34

R is the resistance
AB and L
R = L/A
A
V = IR or V = I L/A = KL K = I /A

V L
V
or = K
Fig. 22.33 L

SOLUTION difference v v = . The


v E of the
R cell E or E = v =
=
R 3
V E
R
Applications of potentiometer
R
R =
R
and
R 3 E and E
=
R E = and E =
R R 3 E
= =
R E
Electric Current and D.C. Circuits 22.13

SOLUTION
E
S AB = L r
E= R=3 and V AC =

Fig. 22.36

AB
V
Fig. 22.35 I=
R r
R is connected across the cell E AB = I r
S is
E
V of E
AB is K =
V=
E
= E=
V
E = V + v = IR + Ir I 22.24
E and r is its internal resistance. Thus
V v v
= AB
V
resistance is 3 .
v
=
V
Ir
=
IR

r= R

R and R r is

22.23
Fig. 22.37

resistance of 3 E SOLUTION
AB
.
I =
E. 3
22.14 Comprehensive Physics—JEE Advanced

Potential difference across AB due to E is Potential difference across AC


cell is
Potential difference across due to E r
VAC = IRAC
.
Vl = -
resistance due to E is tial difference across AC E is
.
I =
VAC
resistance
. r
r

due to E is R
V connected across E

V =V
I =
R
.

R
VR = I R =
R
22.25
VR
across AC
VR = resistance of AC current I

=
R
R
R

R
SOLUTION VR =
R
AB I
AC AC r be the resis- 22.11 AMMETER
tance of AB.

Fig. 22.38
r
AB = . There-
r
AC is RAC = Fig. 22.39
r
Electric Current and D.C. Circuits 22.15

I
G
I V
I
S = shunt resistance R
G S
I G = I sS I = I + Is
IG I–I S

I
S= G
I I

22.26
Fig. 22.40

V = VG + VR
=IG+IR

V V
=G+R R= –R
I I

SOLUTION
I I G . 22.27

I .
S= G
I I . .
.

GS . -
RA =
G S

SOLUTION
resistance of the circuit and hence the current G I V
V
R= –G=
current. I .

22.12 VOLTMETER
.

RV = R + G
-
tor of resistance R << RV
22.16 Comprehensive Physics—JEE Advanced

R and RV RV >> R

R
is con-
R
-
R. tion is

22.28 R=
are connected

resistance.
-
sure the current in the circuit. Find the error in

SOLUTION Fig. 22.42

Total resistance of the circuit is


R
Current in the circuit is
.
I =

Fig. 22.41 .
V =
.
I=

-
. The current
V =I
.
I =

Multiple Choice Questions with Only One Choice Correct


1.
and are connected in series.

2.
+ + . The
Electric Current and D.C. Circuits 22.17

3.
resistor is

Fig. 22.45
6.

Fig. 22.46
Fig. 22.43
7.
4. A and B is
P and
3R
Q

P to Q R
3

Q to P 7R
3

P to Q R
3
Fig. 22.47
Q to P 8.
3
A and B is
3R

R Fig. 22.48
9. is used
Fig. 22.44

5. I
E is -

across it is
22.18 Comprehensive Physics—JEE Advanced

10.
-
. 15.

16.
11. L n R is
E - n R nR
E R R
L - n n
- 17. is bent into a circle. The
L L circle is
3
L L
3
12. 18.

-
tion. What is the current I in the circuit is
resistance X

Fig. 22.49
13. each are connected as
D -

A and D
Fig. 22.51
19. is bent into the
r
3

3
3

20.

Fig. 22.50 A and B is


14.
Electric Current and D.C. Circuits 22.19

22.
R

Fig. 22.52
21. K is
t -
23. E and different internal
I in the resistor AB
resistances r and r are connected in series to an
I t
I external resistance R R
I t
t I

Fig. 22.54
r
R= R=r +r
Fig. 22.53 r
R=r –r R=r =r

Fig. 22.55

24.

26.

25.

the conductor is/are


22.20 Comprehensive Physics—JEE Advanced

Fig. 22.56

27.

28.
resistances is S
the total resistance is P S = nP Fig. 22.57
n is
32.

29. - other. One cell has an


-

and the other cell has


in the ratio of and
3 3
internal resistance
Fig. 22.58
difference across the
3
X and Y is

30. 33.
- -
tance X Y
X<Y

X Y

34.
31.
current I in the circuit is
A and B is
Electric Current and D.C. Circuits 22.21

R
R

R R
38.
is connected in series

39. n
Fig. 22.59 R
35. A resistance of x
y
R is R x and y
xy
R= R y–x
x y
A
60 W R= xy R x+y
40.
x
E1 4V R 1V E2
y

xy xy
Fig. 22.60
x y y x
36.
C and D y–x xy
41.

is

42.

Fig. 22.61 x is
37.
A and B is

Fig. 22.63
Fig. 22.62
22.22 Comprehensive Physics—JEE Advanced

Fig. 22.64

43.
R R R R R R
3
R R = R R3
R R3 = R R = R R

44.

Fig. 22.67
47. P and Q of

Rr R + r
R R+r R+r
r R
Fig. 22.65 R r
45.

V V
V V/3

Fig. 22.68

48.
AC
x
Fig. 22.66 AB is
46. I is
R . Then x
x
R R R = R3 R R x x
Electric Current and D.C. Circuits 22.23

Fig. 22.71

51. The internal resistances of the cells in the circuit


Fig. 22.69
the circuit is

49.

Fig. 22.72
A to D
B to C
A to B
B to A.
52.
ABCD
B and D
BD
A and C is

53.

Voltmeter

2W R

Fig. 22.70
50. 6V

Fig. 22.73
Q
S 54.
A B and C R R and R3
A and B B
and C A and C R :
R : R3
22.24 Comprehensive Physics—JEE Advanced

Fig. 22.76
Fig. 22.74
60. -

55. RC circuit consists of a resistance R


C F connected in series -

56. resistor 3 3
61. A and B
E and of internal resistances
rA and rB L is an ideal inductor and C
K is closed. When the

R
A

Fig. 22.75
57.

-
-
sure is
Fig. 22.77

R = rA – rB if rA > rB.
58. R= rA rB

R= rA + rB
R
59. A and B in
A

62. RC circuit consists of a resistance R


C F connected in series
Electric Current and D.C. Circuits 22.25

63. AB and BC L
AB r
Fig. 22.78
BC is r. The current I
-
65.
BC X
across AB.
BC
AB.
AB and BC X
AB and BC

64.
S
Y to X is
C Fig. 22.79
C

ANSWERS

1. 2. 3. 4. 5. 6.
7. 8. 9. 10. 11. 12.
13. 14. 15. 16. 17. 18.
19. 20. 21. 22. 23. 24.
25. 26. 27. 28. 29. 30.
31. 32. 33. 34. 35. 36.
37. 38. 39. 40. 41. 42.
43. 44. 45. 46. 47. 48.
49. 50. 51. 52. 53. 54.
55. 56. 57. 58. 59. 60.
61. 62. 63. 64. 65.

SOLUTIONS

1. L A = cross-sec-
R= R +R d d

L L L
= = + R=
L
A A A A
22.26 Comprehensive Physics—JEE Advanced

4.
= +
R=
2. E = V + Ir = I R + r r is the internal resis-
tance of the cell and the current in the circuit is
E I=
I= R
R r
V = IR and

E= E–V
E V IR
E E I R r
R r
=
R r R r
r
R r Fig. 22.81

=
I +I =I
3. V APQDA
E. Points A B and C
at A B and C D I I I I
D and E is V V
E and B is V = V E and C = I = I =
V. Therefore 3 3
V
I = I = I =
3 3 3
V P
I =
3 I = I3 + I
V
I3 = I3 = I – I = =
3 3 3
I3
P to Q

5. I =

I +E I
I I E
6.
Fig. 22.80

I = I + I3
V V V
= V=
3 3
/3
Hence I3 = =
Fig. 22.82
Electric Current and D.C. Circuits 22.27

A and B .
V–V =
Current I =

I 3 10.
= –3
-
3 3
7.
3
–3
I=
R

–3
R
R
11.
E /L. Hence
Fig. 22.83
L E E
E= =
R L
RAB =

8. =E L x
xE
E=
L
L
x= .

12.
7
=
Fig. 22.84 X
R X . Hence the correct choice is
RAB =
13.

9. Current I = =
A and D
resistor is = + =
R 3
V=
or R=3

R=

I = =

Fig. 22.85

V = 14. I
22.28 Comprehensive Physics—JEE Advanced

R
I+ I I= R =

R R
I+R I R
R =R R . Hence the correct = + + =
R R R R R
R
15. Let the three resistances be X X and Y. Then X or R = =
X

R = X. 20.
3
= =
3 R
+ +
R Y X Y R
X circuit is
X = 3. Hence Y V
. Hence the R
the
16.
-
n
= + + +n resistor in
R R R R
or R = R/n A and B
17.
21. t

AB is
. Hence the
I=
18. B E. Hence no current t as
–t/
I= I e
t
A and D is the I=I t >>

= + =
R 3 I =
or R=3 I
22. resistor
19. r
R r
R
r
r
r r
R R resistors connected in
R = r=
r
Electric Current and D.C. Circuits 22.29

23. 26. A and B of the


E AB
E . Current I
I=
R r r
A and B .
Hence current in AC
resistance r is E – I r E=Ir CD
CD
Ir
I=
R r r 27.
sectional area A -
R = r – r . Hence the correct choice ity

24. - R=
A V
- V=
R V
R -
AB R. The tional to . Thus if R
resistances R - .
bined resistance is a
R
= or R =
R R R
AB
Resistance R R and
R
R
R R R
R Re =
R of the R R 3

R
R
R
R R

The roots are R or


.

Fig. 22.87

28. Let x and y


Fig. 22.86 xy
S = x + y and P = S = nP or
x y
25. A
nxy
x+y=
of A x y
22.30 Comprehensive Physics—JEE Advanced

x n xy + y
y x are

x= n y [n y y

or x = n y + n

x n
n
n
29.

R A R A
= =

R A r r Fig. 22.88
or = = = 3
R A r r = and 3 = . R

= and
r
. Hence A and B
3 r 3 E E
R= =
R 3
=
R 3
E 3 =

I R =I R E
Therefore R= = =7
I R
or =
I R 3
32.
X
30. =
Y
I=
X
=
Y
X
=
Y X and Y
33. is
=
V = IR =
= or A
31. V I . .
E E= = =
A

ACDA CBDC and ADBGFA


3
34.
– 3 + 3 3
+ 3 E B
Electric Current and D.C. Circuits 22.31

CC
B. Ceff = F F
C C 3 3
R D and Q is the
E
3 Q = VAB Ceff
= 3
R
C
AC is
or R =
3 Q .
R A and C VAC =
C
C and D is
VCD = VAD – VAC

37. -
R CD is

R R + R and R + R
R R R R
by

=
Fig. 22.89 R R R R
3 7 R
= + = R =
R
V 38.
or R= I=
7 / P R
35. Current due to E is Q S
-
E
I= = -
R R
R = IR
current due to E and
IR = E . Thus
I R
R
= R . Hence the correct choice
R R
R . Hence the correct choice is 39. For series connection x = nR -
R R
tion y = . Therefore xy = nR = R . Hence
36. I= n n

Potential difference across AB is 40.


R R Rn Rn x
VAB
n
AD is
VAD
R R Rn y
22.32 Comprehensive Physics—JEE Advanced

Subtracting (2) from (1), we have R 7 10


current I2 is given byI2= I =
1 1 1 R2 5 70 / 4
= 4
Rn x y A, which is choice (b).
5
xy
which gives Rn = which is choice (b). 44. The emfs of cells
y x
connected in reverse
41. Let m cells be connected in series and n such groups
polarity cancel each other.
are connected in parallel. If the emf of each cell
Hence cells marked 2, 3
is E and internal resistance r, then the total emf
and 4 together cancel the
of m cells in series in mE and the total internal
effect of cells marked 5,
resistance is mr. When n such groups are in paral-
6 and 7 and the circuit
lel, the effective internal resistance is mr/n. Then
reduces to that shown
the current through an external resistance R is
in Fig. 22.91. Now cells Fig. 22.91
mE mnE 1 and 8 are in reverse
I= (1)
mr nR mr polarity. Hence the voltmeter reading = 5 – 5 = 0 V.
R
n Hence the correct choice is (d).
mnE 45.
= 2 containing the capacitor. Thus, the current, say I,
nR mr 2 mnRr R and 2R. Apply-
Now, I will be maximum if the denominator is the ing Kirchhoff’s second rule to the loop abcdefa, we
minimum, i.e. if nR = mr. Given R = 3 and r = have (see Fig. 22.92)
1 . Using these values, we have 3n = m. But mn V
2V – I(2R) – IR – V = 0 or I =
m m 3R
= 48 (given). Therefore = 48, which gives
3 Potential drop across capacitor
m = 12. Thus n = 4. Hence the correct choice is (b). V
42. The given circuit is a Wheatstone’s bridge. The cur- = 2V – V – I(2R) = V – 2R
3R
rent through the galvanometer will be zero if the 2V V
bridge is balanced, i.e. if =V– , which is choice (c).
3 3
P R
where P = 2 + 3 = 5 , Q = 10 and
Q S
R = 4 . The value of S is given by
5 4
10 S
or S = 8 . Thus the effective resistance of the
parallel combination of 12 and x ohm must be 8
. Therefore
1 1 1 Fig. 22.92
12 x 8 46. R 6, resistances R1,
which gives x = 24 . Hence the correct choice is (d). R2, R3 and R4 constitute the four arms of a balanced
43. Since the seven resis- Wheatestone’s bridge. Hence
tances are in parallel, R1 R3
the effective resistance or R1 R4 = R2 R3
R1 R2 R2 R4
is R = 70/7 = 10 . Thus the correct choice is (c).
Therefore, the cur- I1
47. Refer to Fig. 22.93.
rent in the circuit is I I I2 The branches ABPQ and PQCD are a balanced
= 14/10 = 7/5A. The Wheatstone’s bridge. Therefore, resistances (each
Fig. 22.90
given circuit can be equal to 2R) between E and F and between F and
redrawn as shown in G do not contribute and the circuit simplies to the
70 70 Re
Fig. 22.90 where R1 = and R2 = . The
3 4 between P and Q is given by
Electric Current and D.C. Circuits 22.33

Fig. 22.93

1 1 1 1 53. The correct choice is (b).


Re 4R 2r 4R 54. Let the value of each resistance be r. The network
can be redrawn as shown in the Fig. 22.95.
2Rr
which gives Re = . Hence the correct choice
is (a) R r
48.
is
R1 R
= AC
R2 RCB
where RAC and RCB are the resistances of the bridge
wire of length AC and CB respectively. If the radius
of the wire AB is doubled, the ratio RAC/RCB will
remain unchanged. Hence the balance length will
remain the same. Thus, the correct choice is (b).
Fig. 22.95
49. The voltmeter must be connected in parallel with the
(i) Net resistance R1 between points A and B
resistor and the ammeter must be connected in series r r
with the resistor. Hence the correct circuit is (a). The series combination of resistances and ,
3 2
50. The potential at Q with respect to R is 15 V and r r
R is at 25 V higher potential than S. Thus Q is which has an equivalent resistance r1 =
3 2
40 V higher than S. When Q is grounded, its
= 5 r , is in parallel with resistance r. Hence
potential becomes zero. thus, Vs = – 40 V. Hence the
correct choice is (d). 6
5r
51. Since the cells are in opposition, the effective emf = r r1
r
R1 = 6 = 5r
r r1 5r 11
the positive to the negative terminal of the battery, r
6
a current of
4 (ii) Net resistance R2 between points B and C
= 0.5 A r
5 3 The series combination of resistances r and ,
B to C. Hence the correct choice is (b). 3
52. Resistance of each r
which has an equivalent resistance r2 = r + =
side of the square = 3
10 0.1 = 1 . As 4r r
, is in parallel with resistance . Hence
shown in Fig. 22.94, 3 2
the square forms r r 4r
r2
a W h e a t s t o n e ’s R2 = 2 2 3 = 4r
bridge which satis- r r 4r 11
r
fies the balancing 2 2 2 3
condition. Thus, no Fig. 22.94 (iii) Net resistance R3 between points A and C
r
the diagonal BD. Hence the correct choice is (a). The series combination of resistances r and ,
2
22.34 Comprehensive Physics—JEE Advanced

r The resistance of wire of resistivity is


which has an equivalent resistance r3 = r + = 2
2 l dl
3r r R= = kl2 (1)
, is in parallel with resistance . Hence A m
2 3
where k = d/m is a constant of the wire. Taking
r r 3r logarithm of both sides of (1) we have
r3
R3 = 3 3 2 = 3r log R = log k + 2 log l
r r 3r 11
r3 Differentiating
3 3 2
R 2 l 2 l
5r 4r 3r =0+
Hence R1 : R2 : R3 = : : = 5 : 4 : 3. Thus R l l
11 11 11
the correct choice is (c). l R
Given = 0.1%. Therefore, = 2 0.1%
l R
55. At instant of time t, the charge on the capacitor is = 0.2%. Thus, the resistance of the increases by
given by 0.2%.
q = q0 (1 – e–t /RC)
which is choice (b).
and the potential drop across the capacitor is given 59. The resistance between points A and E is given by
by ( V = q/C)
VC = V0 (1 – e–t/RC) 1 1 1
=
where V0 is the voltage of the battery. The potential RAE 6 6
drop across the resistor is by giving RAE = 3 . The network reduces to that
–t/RC –t/RC shown in Fig. 22.96 (a). Similarly the resistance
VR = V0 – VC = V0 – V0 (1 – e ) = V0 e
t / RC
VC 1 e
= = e t /RC – 1
VR e t / RC
VC
Given = 8. Therefore,
VR
8 = et/RC – 1
or et/RC = 9 = (3)2
t
or = 2 loge (3)
RC
or t = RC 2 loge (3) Fig. 22.96
= (5 106) (1 10–6) 2 1.1
= 11 s between points A and D in Fig. 22.96 (a) is RAD = 3 .
Hence the correct choice is (b). The network reduces to that shown in Fig. 22.96
56. The two sub circuits are closed loops. They cannot
send any current through the 3 resistor. Hence the between points A and C is RAC = 3 . The network
potential difference across the 3 resistor is zero,
which is choice (a). effective resistance between points A and B is 2
57. Given I = 1 mA = 10–3 A, G = 20 and R = 4980 . So the correct choice is (b).
V 60. Let the internal resistance of each battery be r.
Now I= Let R be the unknown resistance and G be the
R G
resistance of the galvanometer. Let E be the emf
or V = I(R + G) = 10–3 (4980 + 20) of each battery. When the batteries are connected
= 5.0 V in series, the total emf = 2E = 2 1.5 = 3 V and
Hence the correct choice is (c). total internal resistance is 2r. The current in the
58. The mass of a wire of length l, cross sectional area circuit will be
A and density d is given by 3
m I=
m = Ald or A = R G 2r
ld
Given I = 1 A. Therefore
Electric Current and D.C. Circuits 22.35

3 VC = V0 (1 – e–t/RC)
1= or R + G = (3 – 2r) (1) where V0 is the voltage of the battery. The potential
R G 2r
drop across the resistor is
When the batteries are connected in parallel, the
total emf = E = 1.5 V and the total internal resistan- VR = V0 – VC = V0 – V0 (1 – e–t/RC) = V0 e–t/RC
ce is r/2. Hence the current in the circuit will be VC 1 e t / RC
= = e t /RC – 1
I =
15
. VR e t / RC
r VC
R G Given = 8. Therefore,
2
VR
Given I = 0.6 A. Therefore,
8 = et/RC – 1
15
. r
0.6 = or R + G = 2.5 (2) or et/RC = 9 = (3)2
r 2 t
R G or = 2 loge (3)
2 RC
From Eqs. (1) and (2), we have or t= RC 2 loge (3)
r = (5 106) (1 10–6) 2 1.1
3 – 2r = 2.5 – = 11 s
2
1 l
which gives r = ohm, which is choice (d) 63. R = . Since the two wires are made of the same
3 r2
61. When the key K is inserted, the current starts growing material, resistivity is the same for wires AB and
and after some time it acquires a steady value. At BC. Since the wires have equal lengths, it follows
that R 1/r2 . Hence
RAB 1
= , i.e RBC = 4RAB
RBC 4
through the inductor (because an ideal inductor Since the current, is the same in the two wires, it
offers zero resistance to a steady current). Now,
follows from Ohm’s law (V = IR) that VBC = 4 VAB.
the network of resistors is a balanced Wheatstone’s
Hence choice (a) is wrong. Now power dissipated is
bridge. Hence, no current flows through the
P = I2 R. Since I is the same, P R. Hence
resistance 2 R. Therefore, this resistance can be
ignored. The net resistance between points X and PBC RAB
= =4
Y = resistance of the parallel combination of PAB RBC
2R 2R Hence chioce (b) is correct. Choice (c) is wrong
2 R and 2 R = = R.
(2 R 2 R) because current density (i.e. current per unit area)
Hence the current in the circuit is is different in wires AB and BC because their cross-

E E 2E
I= = wire is E = V/l. Since the two wires have the same
R rA rB R rA rB length (l), E is proportional to potential difference
Now, the terminal voltage of cell A is (V). Since VBC = 4 VAB, EBC = 4EAB. Hence choice
2ErA (d) is also incorrect.
VA = E – IrA = E –
R rA rB 64. Refer to Fig. 22.97.
2ErA
VA = 0, if E – =0
R rA rB
which gives 2 rA = R + rA + rB or R = rA – rB
So the correct choice is (a).
62. At instant of time t, the charge on the capacitor is
given by
q = q0 (1 – e–t /RC)
and the potential drop across the capacitor is given
by ( V = q/C) Fig. 22.97
22.36 Comprehensive Physics—JEE Advanced

When the switch S is open, capacitors C1 and C2 are Charge on capacitor C1 in Q1 = C1 V1 = 3 F 3V


in series and their combined capacitance is =9 C
3 6 Charge on capacitor C2 is Q2 = C2 V2 = 6 F 6V
C1 C2
C= = =2 F = 36 C
C1 C2 3 6
Now, when the switch is open, the initial charge
Charge on each capacitor = 2 F 9V = 18 C Y to X = 18 – 18 = 0 C because the
When the switch S is closed, in the steady state, right plate of C1 has a charge – 18 C and left plate
of C2 has a charge + 18 C. When the switch S is
resistors R1 and R2 will be in series and their com- Y to X = – 9
bined resistance is R = R1 + R2 = 3 + 6 = 9 . + 36 = + 27
Therefore, the current in each resistor is from Y to X when the switch is closed = 27 C – 0
9V = 27 C.
I= =1A 65. Corrected length L1 (= AJ) = 52 + 1 = 53 cm
9
Potential difference across R1 is V1 = IR1 = 1 3 Corrected length L2 (BJ) = (100 – 52) + 2 = 50 cm
=3V For a balanced Wheatstone bridge,
Potential difference across R2 is V2 = IR2 = 1 6 X L 53
= 1
=6V 10 L2 50
Since capacitor C1 is connected across R1, the
X = 10.6
potential difference across C1 is V1. Similarly, the
potential difference across C2 is V2.

II

Multiple Choice Questions with One or More Choices Correct


1. A galvanometer has a resistance of 100 and full- (c) greater or less than its emf depending on the
scale range of 50 A. It can be used as a voltmeter direction of the current through the battery
or an ammeter, provided a resistance is connected (d) greater or less than its emf depending on the
to it. Pick the correct range and resistance combina- magnitude of its internal resistance
tion (s): 4. The internal resistance of the cell shown in Fig. 22.98
(a) 50 V range with 10 k resistance in series is negligible. On closing the key K, the ammeter
(b) 10 V range with 200 k resistance in series reading changes from 0.25 A to 5/12 A, then
(c) 5 mA range with 1 resistance in parallel
(d) 10 mA range with 2 resistance in parallel (a) R1 = 10
IIT, 1991 (b) R1 = 15
2. Choose the correct statements from the following. (c) the power drawn from the cell increases
(a) A low voltage supply of, say, 6 V must have (d) the current through R decreases by 40%
a very low internal resistance.
(b) A high voltage supply of, say, 6000 V must
have a very high internal resistance.
(c) A wire carrying current stays electrically
neutral.
(d) A high resistance voltmeter is used to mea-
sure the emf of a cell.
3. The terminal voltage of a battery is Fig. 22.98

(a) always equal to its emf 5. A current I flows in the circuit shown in
(b) always less than its emf Fig. 22.99. Then
Electric Current and D.C. Circuits 22.37

(a) If a resistance R 2 = R is connected in (a) The potential at point P is 6 V.


parallel with R1 = R, the current through R1 (b) The potential at point Q is – 0.5 V
will remain equal to I. (c) If a voltmeter is connected across the 6 V bat-
(b) If a resistance R2 = 2R is connected in tery, it will read 7 V.
parallel with R1 = R, the current through R1 (d) If a voltmeter is connected across the 6 V bat-
will remain equal to I. tery, it will read 5 V.
(c) If a resistance R2 = 2R is connected in 8. In the circuit shown in Fig. 22.102,
parallel with R1 = R, the current through R1 (a) the current through NP is 0.5 A
will become I /3. (b) the value of R1 = 40
(d) If a resistance R2 = 2R is connected in parallel (c) the value of R = 14
with R1 = R, the current through R2 will be (d) the potential difference across R = 49 V
I/2.

Fig. 22.99

6. Two equal resistances R1 = R2 = R are connected Fig. 22.102


with a 30 resistor and a battery of terminal volt-
9. Choose the correct statements from the following.
age E. The currents in the two branches are 2.25 A
(a) If n identical cells, each of emf E and internal
and 1.5 A as shown in Fig. 22.100. Then
resistance r are connected in series, the emf of
(a) R2 = 15 (b) R2 = 60
the combination is nE and the internal resistance
(c) E = 36 V (d) E = 180 V
of the combination is nr.
(b) If n identical cells, each of emf E and internal
resistance r are connected in parallel, the emf
of the combination is E/n and the internal
resistance of the combination is r/n.
(c) Cells should be connected in series if the external
resistance R is greater than internal resistance r.
(d) Cells should be connected in parallel of R is
Fig. 22.100 smaller than r.
7. Which of the following statements are correct 10. The resistance network shown in Fig. 22.103 is
about the circuit shown in Fig. 22.101 where 1 connected to a battery of emf 30 V and internal
and 0.5 are the internal resistances of the 6 V and resistance of 1 . Then
12 V batteries respectively?

Fig. 22.101 Fig. 22.103


22.38 Comprehensive Physics—JEE Advanced

(a) the voltage drop across the 2 resistor is 12 V. resistances r1 = 1 and r2 = 2 respectively, with
(b) the voltage drop across the 12 resitor is 12 V. polarities as shown in Fig. 22.105.
(c) the terminal voltage of the battery is 24 V. IIT, 1997
(d) the voltage drop across the internal resistance
5
of the battery is 6 V. (a) E = V (b) E = 1.2 V
6
11. 2
nonuniform cross-section. Which of the following (c) r = 1.5 (d) r =
3
quantities remain constant along the length of the
conductor?
(a) Current (b) drift speed

12. A cell of emf E and internal resistance r supplies a


current of 0.9 A through a 2 resistor and a current
of 0.3 A through a 7 resistor. Then Fig. 22.105

(a) r = 1.0 (b) r = 0.5 16. In the circuit shown in Fig. 22.106, cells E1 and
E2 have emfs 4 V and 8 V and internal resistances
(c) E = 2.0 V (d) E = 2.25 V
0.5 and 1 respectively.
13. A voltmeter graded as 6000 per volt reads 3 IIT, 1978
R
is connected in series with it the reading of the

resistance of the new voltmeter is R . Then


(a) R = 5.4 104
(b) R = 3.6 104 V
(c) R = 5.4 104
(d) R = 7.2 104 V
14.
with 1 and 2 resistances as shown in Fig. Fig. 22.106
22.104. A 6 V battery of negligible internal resis-
tance is connected between A and B. (a) The potential difference across E1 is 4.25 V
IIT, 1987 (b) The potential difference across E1 is 3.75 V
(c) The potential difference across E2 is 8.5 V
1 1 1 1
A (d) The potential difference across E2 is 7.5 V.
To
17. In the circuit shown in Fig. 22.107, if the galva-
6V 2 2 2 2 nometer resistance is 6 , then in the steady state
infinity
B
B
C1 = 10 m F C2 = 20 m F
Fig. 22.104

(a) The effective resistance between A and B is A C


G=6
3 .
(b) The effective resistance between A and B is
2 . R1 = 16 R1 = 8
(c) The current in the 1 resistance closest to
D
the battery is 3.0 A.
(d) The current in the 2 resistance closest to
the battery is 1.5 A.
12 V
15. A single battery of emf E and internal resistance r
is equivalent to a parallel combination of two bat- Fig. 22.107
teries of emfs E1 = 2 V and E2 = 1.5 V and internal
Electric Current and D.C. Circuits 22.39

(b) the current through R2 is 4 A.


(c) the charge on C1 is 80 C.
(d) the charge on C2 is 80 C.
18. In the cirucit shown Fig. 22.108, the current through
(a) the 3 resistor is 1.0 A
9
(b) the 3 resistor is A
15
(c) the 4 resistor is 0.50 A
Fig. 22.108
(d) the 4 resistor is 0.25 A
IIT, 1998

ANSWERS AND SOLUTIONS


1. For voltmeter, the resistance R to be connected in ter one end of the wire as leave it from the other end.
series with the galvanometer is given by Statement (d) is incorrect. A voltmeter does not
V measure the emf; it measures only the potential
R= –G difference because it draws some current from the
Ig cell. A potentiometer is used for emf measurement
50 because at balance point, no current is drawn from
For 50 V range, R = 6
– 100 the cell. The potential difference measured with a
50 10
voltameter is always less than the emf. Thus the
= 10 –6 – 100 106 1000 k . correct choices are (a), (b) and (c).
Hence choice (a) is incorrect. 3. Terminal voltage V = E – I r, where E is the emf and
r the internal resistance of the battery. If the current
10
For 10 V range, R = – 100
50 100 6 in the external circuit V < E. However, if the cur-
= 2 10 5 – 100 2 105
across the battery would show V > E. Hence the
200 k . Hence choice (b) is correct.
correct choice is (c).
For ammeter, the shunt resistance is given by 4. Before closing the key,
Ig E E
S= G I1 = 0.25 = E = 2.5 V
Is R 10
6 After closing the key,
50 10
For 5 mA range, S = 100 = 1 RR1 10 R1
5 10 3 Effective resistance is R2 = = .
( R R1 ) (10 R1 )
Hence choice (c) is correct.
6
E
50 10 I2 =
For 10 mA range, S = 100 = 0.5 R2
10 10 3
Hence choice (d) is incorrect. Thus the correct 5 2.5 (10 R1 )
or =
choices are (b) and (c). 12 10 R1
2. Statement (a) is correct. which gives R1 = 15 .
The current I drawn from a supply of voltage E and Before closing the key,
internal resistance r is given by I = E/r. So r must
be very small so that a high current can be drawn. Power P1 = I12 R2 = (0.25)2 10 = 0.625 W
Statement (b) is also correct. At such a high volt- After closing the key,
age, the current drawn from the supply will become
5 2 10 15
dangerously large if its internal resistance is small. Power P2 = I 22 R2 = = 1.04 W
Hence a high voltage supply must have a very high 12 (10 15)
internal resistance so that the current drawn from it Hence power drawn from the cell increases.
does not exceed the safe limit. Before closing the key, the current through R is I1
Statement (c) is correct. A wire carrying current is not = 0.25 A. After closing the key, the current through
charged. It stays neutral because as many electrons en- R is
22.40 Comprehensive Physics—JEE Advanced

5 3 1 69
I1 = = = 0.25 A,which is equal to I1. 3.5 = yields R = 14
12 5 4 R 40 / 7
Hence the correct choices are (b) and (c). Hence the correct choices are (b) and (c).
5. When R2 = R is connected in parallel with R1 = R, 9. The correct choices are (a), (c) and (d). In choice
the resistance of the combination is R/2. Therefore, (b) the emf of the combination E and not E/n.
the current in the circuit is V/R/2 = 2V/R = 2I, where 10. Total resistance of parallel combination is given
I = V/R was the current in the circuit when R2 was by
not connected. Current 2 I divides equally among two 1 1 1 1
equal parallel resistors. Hence the current through R1 =
R1 12 6 4
will still be I. Thus choice (a) is correct. When R 2
= 2R is connected in parallel with R1 = R, the total or R1 = 2 . Total resistance in circuit is R = R1 + 2
resistance in the circuit is 2 R/3 and the current in the = 4 . Therefore, current in the circuit is
circuit is E 30
I = = =6A
V 3V 3I R r 4 1
= =
2R/3 2R 2 Potential drop across 2 resistor = 2 6 =
3I 2 12 V.
Current through R1 = = I. Hence choice The voltage drop across the 2 resistor is 12 V.
2 3
(b) is also correct. The remaining current I Since the resistance of the parallel combination is
through R2. Hence the correct choices are (a), (b), also 2 , the voltage drop across this combination
and (d). is also 12 V. Therefore, the total voltage drop across
6. Using Kirchhoff’s Ist law current through R2 is 2.25 the network = 12 + 12 = 24 V. Thus the terminal
– 1.5 = 0.75 A. Also since R2 is in parallel with the voltage of the battery is 24 V.
Voltage drop across the battery = 30 – 24 = 6 V. All
30 resistance, R2 must be 60 since only half
the four choices are correct.
11. Current does not depend on the cross-section of
through 30 resistor. Total resistance in the circuit
the conductor. Drift speed is inversely proportional
becomes 60 + 20 = 80
Potential drop across the battery E = I R = 2.25 80 upon the drift speed of free electrons. Resistivity
= 180 V. depends only on the material of the conductor.
Hence the correct choices are (b) and (d). Hence the correct choices are (a) and (c).
7. Total resistance = 4 + 1 + 0.5 + 0.5 = 6 . Net volt- 12. If E is the emf of the cell and r its internal resis-
6 tance, then
age in the circuit is 6 V. Current I = = 1 A in the
6 E E
anticlockwise direction = 0.9 and = 0.3
2 r 7 r
VPR = 1 4=4V
Dividing the two equations, we get
Since R is connected to earth, VR = 0. Hence VP = 4 V
7 r 9
VS Q = 0.5 1 = 0.5 V. S is at a =
2 r 3
higher potential than Q or r = 0.5 .
VQ = – 0.5 V E
Using r = 0.5 in = 0.9, we get E = 2.25 V.
Current is being forced into the 6 V battery in the (2 r )
opposite direction. Hence V6 = E + I r = 6 + 1 1 Hence the correct choices are (b) and (d).
= 7 V. Hence the correct choices are (b) and (c). 13. A voltmeter is graded according to its resistance
8. Potential difference across MP = p.d. across N O =
p.d. across NP (see Fig. 22.104) a voltmeter has a resistance R ohms and it reads V
Current across NP, INP 10 = 20 1 or INP = 2 A
as R/V ohm per volt. It is given that V = 3.0 V and
Across MP, 0.5 R1 = 20 or R1 = 40 the voltmeter is graded as 6000 /V. Hence the
resistance of the voltmeter is
Total current = 2 + 0.5 + 1.0 = 3.5 A
R = 6000 /V 3.0 V = 1.8 10 4
Electric Current and D.C. Circuits 22.41

or (R + 1) (R – 2) = 0
voltmeter is which gives R = – 1 or 2 . Since negative value
V 3.0 of R is not admissible, R = 2 .
I= = = 1.67 10 –4 A
R 1.8 104
In order to convert this instrument into a voltmeter
-
tance R that must be connected in series with it is
given by (here V = 12 V)
V 12
R = –R= 4
– 1.8 10 4
I 1.67 10 Fig. 22.110

= 5.4 10 4 Applying Kirchhoff’s loop rule to the two meshes


in Fig. 22.110, we have ( R = 2 )
The resistance of the new voltmeter = R + R =
1 I + 2I = 6 (1)
1.8 104 + 5.6 104 = 7.2 104 . Hence the cor-
rect choice are (a) and (d). and 2(I – I ) – 2I = 0

14. Let circuit be broken as shown in Fig. 22.109(a). which gives I = 2I . Therefore, using Eq. (1), we
- have
6
tance remains unaffected by removing one mesh 2I + 2I = 6 or I = = 1.5 A.
- 4
nite network be R. The effective resistance of the So the correct choices are (b), (c) and (d).
remaining part of the circuit beyond CD is also R. 15. Refer to Fig. 22.111. Applying Kirchhoff’s loop
The circuit can be recombined as shown in Fig. rule to loop abcda, we have
22.109 (b). The resistance R and 2 are in parallel. – Ir2 + E2 + E1 – Ir 1 = 0
Their combined resistance is
E1 E2 2 1.5 7
2R which gives I = = = A
R = r1 r2 1 2 6
R 2
R is the series with remaining 1 resistance. The
total combined resistance is
2R
+1
R 2
which must be equal to the total resistance of the
Fig. 22.111

The emf across A and B is


E = – E2 + Ir2
7 5
= – 1.5 + 2= V
6 6
This is the emf of the single battery. The internal
resistance of the single battery is the resistance r of
the parallel combination of internal resistances r1
Fig. 22.109
and r2 which is
r1 r2 1 2 2
2R 3R 2 r= =
R= +1= r1 r2 1 2 3
R 2 R 2
So the correct choices are (a) and (d).
or R + 2 R = 3 R + 2 or R2 – R – 2 = 0
2

or R2 – 2 R + R – 2 = 0 16. Equivalent resistance of the parallel combination of


3 and 6 is R = 3 6/(3 + 6) = 2 . As the cells
or R (R – 2) + (R – 2) = 0
are in opposition, net emf E = E2 – E1 = 8 – 4 = 4 V.
22.42 Comprehensive Physics—JEE Advanced

Therefore, current is = 2. Also V1 + V2 = 12. Therefore, V1 = 8 V and


E 4 V2 = 4V. Thus Q = C1V1 = 10 F 8 V = 80 C =
I= = = 0.5 A
R 4.5 r1 r2 2 4.5 0.5 1 C2. Thus the correct choices are (a), (c)
Potential difference across E1 is V1 = E1 + I r1 = and (d).
4 + 0.5 0.5 = 4.25 V 18. A and B of the
Potential difference across E2 is V2 = E2 – Ir2 = AB is 4 . Therefore, total
8 – 0.5 1 = 7.5 V resistance = 3 + 4 + 2 = 9 . Current I = 9 V/9
Hence the correct choices are (a) and (d). = 1 A. The current is equally divided in the 8
A and B and the remainder 8 .
17. Hence current in AC = 0.5 A. This current is equally
branch ABC CD
ADC. So choice (a) is CD. Therefore, current in the 4 resistor = 0.25 A.
R1 R2 Hence the corrects choice are (a) and (d).
=I A and C =
V C2
12 V. Therefore Q = V1C1 = V2C2. Thus 1
V2 C1

III

Multiple Choice Questions Based on Passage


Questions 1 to 4 are based on the following passage R3 = 2R2 = 4 ohm
Passage I C= 5 F
IIT, 1988
1. The current in resistance R1 is
(a) 0.5 A (b) 1.0 A
(c) 1.5 A (d) zero
2. R3 is
(a) 1.5 A (b) 1.2 A
(c) 0.9 A (d) 0.6 A
3. R4 is
(a) 0.3 A (b) 0.25 A
(c) 0.2 A (d) zero
4.
Fig. 22.112
(a) 4.8 10–6 J (b) 9.6 10–6 J
E1 = 3E2 = 2E3 = 6 volt (c) 1.44 10–5 J (d) 1.92 10–5 J
R1 = 2R4 = 6 ohm

SOLUTION
1. E1
I2 R3 = E1 or I2 = = 1.5 A
R3
R1 is zero, The correct choice is (a).
3.
2. EBCDE FDHGF
Electric Current and D.C. Circuits 22.43

I 3 R 2 – I 2 R 3 + I 3 R 4= – E 2 – E 3 1
= 5 10–6 (2.4)2 = 14.4 10 –6 J
or I3 (R2 + R4) = I2 R3 – E2 – E3 2
or I3 (2 + 3) = 1.5 4–2–3 Hence the correct choice is (c).
or 5I3 = 1 or I3 = 0.2 A R1 = 6 E1 = 6V
B
A C
So the correct choice is (c).
I1 I1
4. F and E is C=5 F
I3 R2 = 2 R3 = 4
V= E2 + I3 R2 = 2 + 0.2 E F D
E I2
and A E2 = 2V
F and A
I3
1 G H
CV2 E3 = 3V R4 = 3
2
Fig. 22.113

Questions 5 to 8 are based on the following passage 5. Current I1 r1 is


Passage II (a) zero (b) 0.5 A
(c) 1.0 A (d) 1.5 A
6. Current I2 r2 is
E1 = 3 V, E2 = 2 V, E3 = 1 V and r1 = r2= r3= R = 1 . (a) zero (b) 1.0 A
(c) 2.0 A (d) 3.0 A
7. Current I3 r3 is
(a) 1.0 A (b) 1.5 A
(c) 2.0 A (d) 2.5 A
8. If r2 A is connected to
B R
Fig. 22.114 (a) 1.0 A (b) 2.0 A
(c) 3.0 A (d) zero
IIT, 1981

SOLUTION
5. abcda and 6 I2
abcdefa choice (a).
– I 1r 1 + E 1 – E 2 – I 2 r 2= 0 7. I3 = I1 – I2 = 1 – 0 = 1 A. So
and – I1r1 + E1 – E3 – I3 r3= 0 the correct choice is (a).
E1 – I1r1=E2 + I2r2 = E3 + I3r3 (i) 8. Since I2
a and d = emf E2 = 2 V and remains equal to 2 V
a
r2 -
have
I1 = I2 + I3 (ii) tial difference across a and d
the currents I1 and I3 E1 and E2 do not
I1 = I3 A is con-
E1 – (I2 + I3) r1= E3 + I3r3 or 2I3 + I2 = 2 (iii) B R
Also E2 + I2r2= E3 + I3r3 or I3 – I2 = 1 (iv)
I1 = 1 A. So the
correct choice is (c).

Questions 9 to 12 are based on the following passage The direction of one of the currents in the branches is
Passage III
IIT, 1991
9. In branch ab, the current i1 is
a to b
22.44 Comprehensive Physics—JEE Advanced

b to a b
I4
a to b I3
10 W
b to a 5W
10. In branch ad, the current i2 is
I1
a to d
c
d to a 5
a
I5
a to d I2

d to a 5W 10 W I
11. In branch bd, the current I3 is I
b to d
d
d to b
b to d
f e
d to b. 10 W 8.5 V
12.
Fig. 22.115
(a) 8.0 (b) 8.5
(c) 17 (d) 17.5

SOLUTION
a, b and I1 = 0.2 A, I2 = 0.3
d A and I3 = – 0.1 A. Since I1 and I2
I = I1 + I2, I4 = I1 – I3 and I5 = I2 + I3. I3
abda, bcdb and indicates that the direction I3 should be from d to b and
adcefa not from b to d.
10 I1 + 5 I3 – 5 I2 = 0 9. The correct choice is (c).
10. The correct choice is (c).
or I2 = 2 I1 + I3 (i) 11. The correct choice is (d).
5(I1 – I3) – 10(I2 + I3) – 5 I3 = 0 12. Total current is I = I1 + I2 = 0.2 + 0.3 = 0.5 A. Hence
or I1 – 2 I2 = 4 I3 (ii)
V 8.5
and 5 I2 + 10(I2 + I3) – 8.5 + 10(I1 + I2) R= = = 17
I 0.5
or 2 I1 + 5 I2 + 2 I3 = 1.7 (iii)
So the correct choice is (c).

Questions 13 to 16 are based on the following passage 13. resistor is


Passage IV 1 1
(a) A (b) A
E, F, G and H of emfs 7 9
2 V, 1 V, 3 V and 1 V and internal resistances 2 , 1 , 1 1
3 and 1 (c) A (d) A
11 13
A (+) E (–) B 14. B and D is
2 2
(a) V (b) V
7 9
(+) 2W (–)
F H
2 2
(–) (+)
(c) V (d) V
11 13
15.
cell G is
D (–) C
(+) (a) equal to 3 V
G
(b) more than 3 V
Fig. 22.116
IIT, 1984
Electric Current and D.C. Circuits 22.45

16.
cell H is
(a) equal to 1 V
(b) more than 2 V
SOLUTION
13. Let I1 and I2 be the currents in branches BAD and 2I1 + I1 + 2(I1 – I2) = 2 – 1 = 1
DCB I3 or 5I1 – 2I2 = 1 (i)
DB resistor. and 3I2 + I2 – 2(I1 – I2) = 3 – 1 = 2
E 2V or 6I2 – 2I1 = 2 (ii)
A I1 I1 B
I1 I2
2W
I3 5 6
1V I1 = A and I2 = A
1W 13 13
F 2W H 1
I3 = I1 – I2 = – A
1W 13
1V
The current the
I1 I3 I2
2 BD DB as
assumed. So the correct choice is (d).
D C
I2 I2 14. B and D = 2 (1/13)
3V G 3W

Fig. 22.117 15.


D G = 3 – (6/13) 3 = 21/13 1.6 V. So the correct
have choice is (d).
I1 = I2 + I3 or I3 = I1 – I2 16. H
BADB = 1 + 6/13 = 19/13 1.46. Thus the correct choice
and DCBD is (d).

Questions 17 to 19 are based on the following passage 17. The resistance of the voltmeter is
Passage V (a) 600 (b) 800
and 400 are connected to a 60
(c) 1000 (d) 1200
connected across the 400 resistor reads 30 V. 18. The current in the circuit is
IIT, 1980 3 3
(a) A (b) A
Voltmeter 32 16
3 3
(c) A (d) A
300 400 8 4
19. When the same voltmeter is connected across the
300
60 V (a) 40 V (b) 22.5 V
Fig. 22.118
(c) 37.5 V (d) 25 V

SOLUTION

17. Potential difference across the 400 resistance resistance, their com-
bined resistance is
the 300 resistance = 60 – 30 V = 30 V. Let R be 400 R
the resistance of the voltmeter. As the voltmeter is R=
400 R
22.46 Comprehensive Physics—JEE Advanced

Total resistance in the circuit = 400 + 240 =640


and 400 resistances, Current in the circuit is
R should be equal to 300 . Thus 60 V 3
I= = A
400 R 640 32
300 =
400 R So the correct choice is (a).
R = 1200 19.
18. When the voltmeter is connected across the 300 240 resistance
resistance, their combined resistance is 3
300 R 300 1200 = 240 = 22.5 V
R = = 240 32
300 R 300 1200 Thus the correct choice is (b).

Questions 20 to 23 are based on the following passage 23.


Passage VI (a) 2 10– 4 J (b) 4 10–4 J
(c) 6 10–4 J (d) 8 10–4 J
state. The currents, the values of resistances and emfs of 1A

C = 4 F. 3
IIT, 1986 4V 3 5
A i1 E
20. The value of current i1 is 2A
(a) 1 A (b) 2 A
4 F 1
(c) 3 A (d) 4 A
3V
21. The value of current i2 is 1
i3
(a) 1 A (b) 2 A 2A B 2 i2 D 4
(c) 3 A (d) 4 A 3
22. The value of current i3 is
(a) 1 A (b) 2 A 1A

(c) 3 A (d) 4 A Fig. 22.119

SOLUTION

20. A, V = 5 i1 + 1 i1 + 2 i2 = 15 + 3 + 2 = 20 V
i1
21. B, i2 + 1 = 2 or i2 C = 20 V
choice (a). 1
CV2
22. D, i1 = i2 + i3 i3 = i1 – i2 = 3 – 1 2
= 2 A. 1
= 4 10 – 6 (20)2 = 8 10– 4 J
So the correct choice is (b). 2
23. AEDB Thus the correct choice is (d).

Questions 24 to 26 are based on the following passage 24. The value of current i1 is
Passage VII 1 1
(a) A (b) A
10 20
1 1
(c) A (d) A
A voltmeter of resistance 400 is connected across 30 40
the 400 25. The value of current i2 is
resistance. 1 1
IIT, 1996 (a) A (b) A
30 15
Electric Current and D.C. Circuits 22.47

1 2
(c) A (d)
10 15
26.
10
(a) V (b) 5 V
3
20
(c) V (d) 4 V
3

SOLUTION Fig. 22.120

24. – 200 I3 + 200(I1 + I2 – I3) + 100(I2 – I3) = 0


Resistance R = 200 or 2 I1 + 3 I2 – 5 I3 = 0 (3)
Simultaneous solution of Eqs. (1), (2) and (3)
of the 400 resistor and the resistance of 400 of
yields
the voltmeter. 1
I1 = I2 = I3 = A
30
– 100 I2 – 100(I2 – I3) + 100 I1 = 0 So the correct choice is (c).
or I1 – 2 I2 + I3 = 0 (1) 25. The correct choice is (a).
26.
– 100 I1 – 200(I1 + I2 + I3) + 10 = 0 R(= 200 )
1 20
or 3 I1 + 2 I2 – 2 I3 – 0.1 = 0 (2) = I3 R = 200 = V.
30 3
Hence the correct choice is (c).

Fig. 22.121

IV

Assertion-Reason Type Questions

correct.
22.48 Comprehensive Physics—JEE Advanced

(a) Statement-1 is True, Statement-2 is True; 5. Statement-1


-
ment-1. E1 and E2 are the emfs of cells C1 and C2 -
(b) Statement-1 is True, Statement-2 is True; E1 > E2. Cell C1
R
Statement-1. is x AB is
(c) Statement-1 is True, Statement-2 is False. x
(d) Statement-1 is False, Statement-2 is True. R
1. Statement-1
C1
-
x D
A B
Statement-2

C2 G

2. Statement-1 Fig. 22.123


Statement-2
–1
– 19
electron is also very small (= 1.6 10 C), yet AD due to cell C1 = E2, the emf of cell C2.
6. Statement-1
Statement-2
- AB -
7
tor R x
3. Statement-1 decrease.
Statement-2
Statement-2
A and D due to cell C1 = emf E2 of cell C2.
7. Statement-1
Electrons in a metallic conductor have no motion if
direction.
4. Statement-1 Statement-2

AC
x. If the radius IIT, 1982
AB
8. Statement-1
becomes 4 x.

Statement-2
R1 R2

G
IIT, 1993
A C B 9. Statement-1
x -
Fig. 22.122

Statement-2

the square of its radius. of the standard resistance.


Electric Current and D.C. Circuits 22.49

Statement-2

IIT, 2008

SOLUTIONS

1. The correct choice is (c). The electrons suffer a A and B due to cell
C1
- x.
6. The correct choice is (a). If the value of R is
decelerated by collision. The net acceleration A and
B due to cell C1
x.
7. Electrons in a conductor have random thermal mo-
2. The correct choice is (b). The current in a metal de- tion. Statement-1 is false and Statement-2 is true.
8. Both the statements are true and Statement-2 is the

9. R is the standard resis-


1029 3
. tance and X
3. -
rent stays neutral because as many electrons enter

4. The correct choice is (d). The condition for no

R1 RAC Fig. 22.124


R2 RCB
RAC and RCB X (100 l )
=
AC and CB R l
AB is doubled, the ratio RAC/RCB
The value of X -
l the same, the
value of R must be increased. So Statement-1 is
5. AB
false, Statement-2 is true.

Integer Answer Type Questions

1.
emf of 5 V and an internal resistance of 0.2 . Find

IIT, 1997
2. L
internal resistances are connected in series. Due to Fig. 22.125
22.50 Comprehensive Physics—JEE Advanced

T in a time t. A number N
AJ = 60 cm. Find the value of X (in ohm).
L - IIT, 2002
T in 4.
the same time t. Find the value of N.
IIT, 2001 AB in volts is

Fig. 22.126
Fig. 22.127
3. AB IIT, 2011
resistance X and a resistance of 12 are connected

SOLUTIONS
1. All the cells are in series. Therefore, total emf 3. In the balanced condition
= 40 V and total internal resistance = 1.6 . There- X BJ 40
fore, current in the circuit is = = X=8
12 AJ 60
40
I= = 25 A 4.
1.6

E – Ir = 5 – 25 0.2
= 5 – 5 = 0 volt.
2. t is Q = V 2t/R,
V
Here Q = ms T m s
R its resistance.

Fig. 22.128
3V 2 t
ms T = (1) CDFEC
R
m R 6–1 I–2 I – 3= 0
second case, I= 1A
NV 2 t
(2m) s T = (2)
2R VA – 6 + 1 I – VB = 0
VA – VB = 6 – 1 1=5V
2
N /2
2= or N 2 = 36 or N = 6,
9
23
Chapter
Current

REVIEW OF BASIC CONCEPTS POWER-VOLTAGE RATING OF


23.2 ELECTRICAL APPLIANCES
23.1 HEATING EFFECT OF CURRENT
If a current I t (P – V
R P = VI
V H
P
2 I=
V t V
H = VIt = I 2 Rt =
R
H J
V V V2
R= = =
Electrical Power If E I P /V P
r 1. Power of Electrical Appliances Connected in Parallel
R Let R1 R2 R3
V 2
E R 2 V
P = VI = I2 R = = P1 P2 P3
R R r 2
V2 V2 V2
[ E = I(R + r R1 = R2 = R3 =
P1 P2 P3
Electrical Energy A
V2 V2 V2
P1 = P2 = P3 =
R1 R2 R3

(60 R
1 1 1 1
= + + + ...
R R1 R2 R3

V2 1 1 1
= 1000 P= = V2 ...
R R1 R2 R3
= 3.6 106
V2 V2 V2
= + + + ...
R1 R2 R3
P = P1 + P2 + P3 + ...
23.2 Comprehensive Physics—JEE Advanced

2. Power of Electrical Appliances Connected in Series


R R1

R R1
R = R1 + R2 + R3 + ...
V 300
P = I 2 R = I2 (R1 + R2 + R3 I= =
R R1 1000 R1
P = P1 + P2 + P3 + ...

23.1

Fig. 23.1
300 R1
SOLUTION R1 = IR1 =
1000 R1
P V
300 R1
= 100 R1 = 500
V 2
200 200 1000 R1
R= = = 40
P 1000 23.3
R
V A B
2
V 160 160
P = =
R 40
P P
100
P
1000 640
= 100 = 36%
1000

23.2
SOLUTION
V2
A RA =
PA

= 200 200 = 1000


40
SOLUTION V 2

P V B RB =
PB

V2 200 200 200 200


R= = = 1000 = = 666.7
R 40 60

I.
V 12 300 300
P1 = = A PA = I2 RA
R 1000 B PB = I2 RB
P1 >> P PA I2 R R
= 2 A = A
PB I RB RB
RA > RB; PA > PB A
B.
23.3

P2 = I 22 R
= (5 105 2 10 = 2.5 1012
V.
V2 P1 << P2
A PA =
RA
V2
B PB =
RB
PA R
= B 23.6
PB RA
E r
RB < RA; PA < PB B
R
A.

23.4 R=r

SOLUTION
R = 0.2 P 103 V
P 10 103
I= = = 50 A
V 200
I 2R 2
0.2
Fig. 23.2

SOLUTION

ve
E
I=
R r
10 R
=
10.5 2
E E2R
P = I 2R = R= 2
R r R r
23.5
dP d 2P
P
dR dR 2
10 R
dP 1 3
= E2 2
2R R r
dR R r
SOLUTION
2
P 106 8
R = 10 E
= 3
[(R + r R
V1 R r
P 108 dP
R+r R=0
I1 = = = 5000 A
V1 20 000 dR
P1 = I 21 R R = r.
2 d 2P
10 = 2.5 108 R=r
V2 dR 2
P 108
I2 = = =5 105 A
V2 200
23.4 Comprehensive Physics—JEE Advanced

VARIATION OF RESISTANCE AND SOLUTION


23.3 RESISTIVITY WITH TEMPERATURE
R2 = R1 [1 + (t2 – t1
R2 R1 117 100
t2 – t1 = = = 1000°C
R1 (1.7 10 4 100
t2 = 1000 + t1 = 1000 + 27 = 1027°C
R2 = R1[1 + (t2 – t1
23.8
R1 t1 R2
t2
at

2 = 1[1 + (t2 – t1 SOLUTION


R2 R1 3.75 3.00
= = = 0.0025 K–1
R1 t2 t1 3.00 (100 0
Rt R0
Rt = R0(1 + t = t
R0
Rt R0 3.15 3.00
23.7 t= = = 20°C
R0 0.0025 3.00
at 27°C.

= 1.7 10–4 K–1.

Multiple Choice Questions with Only One Choice Correct


1. R1 3 8
4 9
R2 (< R1

1
R1 – R2 (R1 – R2
2
R1 R2
R1 R2
R1 R2
2.

tance

3.

1 1
3 2 Fig. 23.3
23.5

4. 4 3

V 2

T
11.
P = k (T – T0 k
T0 R
T
Y

kY kV 2Y 12.
2
RV R
V 2Y kYR
kR V2
5. G
S 13.

G
S
S= 1 G S= 3 G 14.
3 4
3 2
S= G S= G
2 3
6.
n
15.
P
n

P P
7. P P
16.

8.
17.
V Q
t

9.
Q Q
4 2
Q Q
10.
18. E r
P1 r1 Q
P2 t
P2 /P1
23.6 Comprehensive Physics—JEE Advanced

r2
t r 8
r12 r22
9
r2 25.
r1
1 R1 R2
(r1 + r2 r1 r2
2
19. t1.
1 1
t2 (R1 + R2 (R1 – R2
2 2
1
1 (R1R2 1/2 R 1R 2 1/2
(t1 + t2 t1 + t2 2
2 26.
t1 t2
t1 t2
t1 t2
20. t 1.

t2.

1
(t1 + t2 t1 + t2
2
t1 t2 27.
t1 t2
t1 t2
21. 4
. –1 –1
at –1 –1

22.

23.
Fig. 23.5
28.

–1 –1

–1 –1
R

24.

R = 0.5

Fig. 23.4 Fig. 23.6


R
23.7

4 33. B1 B2
9 B3
8 W1 W2 W3
B1 B2 B3
3
W1 > W2 = W3 W1 > W2 > W3
29. W1 < W2 = W3 W1 < W2 < W3

30.

1
5
Fig. 23.8
34.

Fig. 23.7

31.

32. L

T t N

L
T Fig. 23.9
t N
23.8 Comprehensive Physics—JEE Advanced

35. A B
B A
42.

36.

43. AB BC L
37. AB r
BC r I
10–1 –1
K–1

BC
AB.
BC
AB.
38. AB BC
AB BC

Fig. 23.10

44.
2
39. A B

A B
A
B
1 1
4 2

40. A B
Fig. 23.11
45.
1
B
A

41.

Fig. 23.12
23.9

46. P1 > P2 > P3 P3 > P2 > P1


R1 R2 R3 P2 > P1 > P3 P1 = P2 = P3
R1 R2 R3 P1
P2 P3

Fig. 23.13

ANSWERS

1. 2. 3. 4. 5. 6.
7. 8. 9. 10. 11. 12.
13. 14. 15. 16. 17. 18.
19. 20. 21. 22. 23. 24.
25. 26. 27. 28. 29. 30.
31. 32. 33. 34. 35. 36.
37. 38. 39. 40. 41. 42.
43. 44. 45. 46.

SOLUTIONS

1. If E r
R1
E
I
R1 r
R1
2
2 E
Q1 = I R1 = R1
R1 r Fig. 23.14
2 4 6
E =2
Q2 = R2 5 5
R2 r
Q1 Q2 V
I =
r = R1 R2 . 2
4 V 4 2V
2. =
4 5 2 5 5
1
5 2V
6 V1 =
. 5
5
23.10 Comprehensive Physics—JEE Advanced

2V 3V L
V4 = V2 = =
5 5 L
3V S
V3 = . Y = S = stress
5
S
=
V12 2 Y
P1 = = 4V
R1 25
V2 9V 2 2 V2Y
P2 = 2 = = 4.5 V S =
kR
R2 25 2 25
5. G S
V2 9V 2 2
P3 = 3 = = 3V G
R3 25 3 25 S=V
t
V2 4V 2 2
P4 = 4 = = V V2t 1
R4 25 4 25 H = H
R R
P2
HG S
=
HS G
3.
2 S 2
= S= G
4I I 3 G 3
I1 = I4 = 6. W = V × I I = W/V = 24/12
5 5
3I 2I
I2 = I3 =
5 5 R = 12/2 = 6 . If n
2 2
16 I 16 I n 1.0
P 1 = I 12 R 1 = 1 = =2 n = 12
25 25 6
7.
9I2 18 I 2
P 2 = I 22 R 2 = 2 =
25 25 500
2 2 I = =5A
4I 12 I 100
P 3 = I 32 R 3 = 3 = 100
25 25 R= = 20
5
I2 4I2
P 4 = I 42 R 4 = 4 =
25 25 R
P1 8
=
P2 9 200
=5 R = 20
4. R 20
8.

V2
= k (T – T0 k T
R
L
L = L T T= V I = 100
L

V2 k L k
= = R
R L
I 2R = 900
23.11

900 3V 2
R = = 4.0 P =
15 15 R
9. V = IR = P = 9 P.
V–V = 16.

10. P 10 6 R
17. Let R

100 106 R t
I1 = = 5000 A 2
20 000 V t
Q =
2R
P1 = I 21 R = 2
R = 2.5 107 R
R =R
t
100 106 V 2t 2V 2t
I2 = =5 105 A Q = =
200 R R
Q = 4Q
P2 = I 22 R = 2.5 10 11
R 18.
E
P2 2.5 1011 I1 =
= = 10 4 r1 r
P1 2.5 107
2
E
Q1 = I 2 r1 t = r 1t
r1 r
11. P=I R 2
P = 2I IR
P 2 I 2
= E
P I Q2 = r2 t
r2 r
I P
I P
r1 r2
2 = 2
2 r1 r r2 r
V 2V V
12. P= P= r1 (r2 + r 2
= r2 (r1 + r 2
R R
P 2 V r1 (r 22 + 2r r2 + r 2 r2 (r21 + 2r r1 + r2
=
P V r 2 (r1 – r2 r1 r2 (r1 – r2
V P r = r1 r2
=–2 2=
V P

19. Let R1 R2

13. R1 + R2 V
Q
14.
15. Let R V V 2t V 2 t1 V 2 t2
Q= = =
R1 R2 R1 R2
Q R1 R2 Q R1 Q R2
V2 t= = +
P = V 2
V 2
V2
3R
= t1 + t2
23.12 Comprehensive Physics—JEE Advanced

20. 25.
I 21 R1t = I 22 R2 t
1 1 1 E
= + I1 = I2 = E .
R R1 R2 R1 r R2 r
1 V2 1 V2 E2 E2
= =
t1 Q R1 t2 Q R2 2 R1 = 2 R2
R1 r R2 r
2 2 2
1 = V 1 1
=
1
+
1 R1 + r R2 = (R2 + r R1
t Q R1 R2 t1 t2
r= R1 R2
t1 t2
t =
t1 t2 V2t l l
26. H = R= =
R A r2
21. R1 = R0 (1 + t1 R2 = R0(1 + t2 t1 V 2 t r2
t2 H=
l
R2 1 t2 H r l
=
R1 1 t1
R2 /R1 t1 27. Let I

t2 I
5
22. I 2
2
P1 = I2 P2 = 4 = I2
Q = V 210 210 –1 2
t R 20 P2
= 1 P2 = P1 = 10 –1
P1 5 5 5
1
28.
2205
80 4.2
23. Let R
R1 = R + R + R = 3R
R R2 = R + R + 4R = 6R
R1 R2 3R 6R
R = = = 2R
R1 R2 3R 6R
R = R
4

V 2 = 4V 2 4 = R = 2R
R R R=2
–1
=4 29.

30. R V2 100 100 =


24.
10 P 1000
100
I=
10 R
10
R = 0.05 10 R
I R
P = I 2R 2 I = I
10 R
23.13

100 R 156.3 2
= W2 =
10 R 10 R 1042
10
10 R
250 2
10 R 5R W3 =
= 1042
R 5 R
1 10 R W1 < W2 < W3
10 R
34. Let R
Q = I2 10
R
25R 2 R R R
2
5 R
R = 5
35. V = 22 P P = VI
V V
31. R= I=
I P
32. t Q = V 2t/R
V V2 220 2
R= = = 484
Q = ms T m s P 100
R V
2
ms T = 3V t 2
R V 110 2
P = =
m R R 484
36.
NV 2 t
(2m s T =
2R R = 20 P
P = V2/R V2 = RP V= RP
2
R Pma
2 = N /2 N2 N V = = 20 1000
9 = 100 2
V
33. B1 B2 B3
are 37.
V2 250 2
R1 = = 625
W1 100 Q = KA 2 1
2 t d
250
R2 = = 1042 = R3 A
60
B3 V3 A=6
VR1 =6 (0.5 m 2
B1 V1 =
R1 R2 1
Q = 1.68 10 1.5 100
250 625 t 1 10 3
625 1042 = 2.52 104 –1
B2 V2

V12 93.7 2
W1 =
R1 625
23.14 Comprehensive Physics—JEE Advanced

V2 V
H= = 2.52 104 IB = = 400 = 1 A
R RB 400

420 420 IA IA IB IB
R= =7
2.52 104

V2 V2 A
38. = 200 2
R l 42. R= = 80 .
500

l
A V

L 2L V2 50 2
39. RA = RB = P= =
r2 2r 2
R 80
RA
RA = 2RB
RB l
43. R =
1 r2
Q
R AB
40. A B are BC
R 1/r2
V2 200 2
RA = = = 1000 RAB
PA 40 = 1 RBC = 4RAB
RBC 4
V2 200 2
RB = = = 400
PB 100 V = IR VBC = 4 VAB.

P = I2 R I P R
PA 40 PBC R
IA = = = 0.2 A = AB = 4
V 200 PAB RBC
PB 100
IB = = = 0.5 A
V 200
AB BC
R = RA + RB = 1000 + 400 =
1400 E = V/l
V 400 l E
I= = = 0.286 A (V VBC = 4 VAB EBC = 4EAB
R 1400
I IA IB A
2
B 44. ven V V 2 = 30 V2 V
2 3
41.
V
A B are 45.
V 400 cur
IA = = = 0.4 A
RA 1000
23.15

2
V2
rr P= V
R
1
46. P P 2 > P 1 > P 3.
R1 R1 = 1 R2 R
R2 = 0.5 R3 = 2 .

II

Multiple Choice Questions with One or More Choices Correct

1.

m
2m I

5. l A

V
2.
V A
l
6.

tance 0.5
R=8

3.

7.
R

4.

R = 0.5 .
8. E r

R P
P R = r.
P R = 2r.
E2/2r.
E2/4r.
23.16 Comprehensive Physics—JEE Advanced

9. E& A B
r A
P B
I A
E E B
I= I=
r 2r 14.
2 2
E E
P = P =
r 4r
10.

11.
–1
.

Fig. 23.16

BD

15.
Fig. 23.15

–1
.
A D

12. E r sup A D

E E
r=1 r=2
13. A

A B Fig. 23.17

ANSWERS AND SOLUTIONS


1. l
A d
m = Ald I H
t
A d
H = I 2Rt H R
23.17

R 1/P
ence V H

2
H = V t H 1
R R
R1
R2
R 1/P R1 R2. If R
2. V 2 = 200 200 V
W 40
= 1000 V
I1 =
200 200 R R1
60
= 666.7 V
I2 =
200 200 R R2
100 R1 > R2 I2 > I1
= 400

= 1000 + 666.7 + 400 = 2066.7

I = 200 = 0.097 A
2066.7

V 2/R
2
5. V2 = V A
l = l R l
3. R =
A r2
If l r R 6.
(r 0.5 = 3
H = V 2/R ce E = V E E
l

I= E = 88 = 8 A
4. R r 8 3
V2 2
Rb = = 200 = 400 I2 (R + r 2
Pb 100

2
R h = 200 = 40
1000

7. V I r = 0.1
23.18 Comprehensive Physics—JEE Advanced

P = V I = 1.5 d 2P E2
=–
dR 2 at R r 8r 3
2 2
Pc = I r
P
R R=r R=r
PR = P – Pc
2
R P = E
4r
1.5 R
R 0.10 9.
R E I
R R
1.5 R 2.0
= P = (E – E I
R 0.10
E = Ir.
PR
P = EI – I2r
1.5 R 2.0
= 2.6 I
R 0.10
dP = E – 2 Ir
R = 0.65 .
dI
P dP
dI
E
8. I=
2r
E
I = d 2P
R r =–2r
dI 2
P
E R 2 I = E/2r.
P = I 2R = 2 I = E/2r
R r 2
E r P P = E
dP/dR d2P/dR2 4r
R E
r 10.

dP E2 2R
= 2
1
dR R r R r
11. I I1 I2
dP/dR
A
2R B C D
1– =0
R r I1 I2 5
R=r P
R=r P I1 = I 2 .
2
R=r d2P/dR2 at R = r
P Q = I22 5 2
20 = I2 5 I2
I1
4 I 12 2 –1
2 2
d P 2E 3R I1 4=1 4=4
= 3
2
2 R r
dR R r
23.19

14.
A B B D D C

I= 6 =6A
Fig. 23.18 1

E E2R BD
12. I = P = I 2R = 2 2
R r R r
3
E 2 R1 E 2 R2
P1 = 2 P2 = 2
R1 r R2 r
15. C
2
P1 R1 R2 r
=
P2 R2 R1 r CB C B
2 B
1.5 = 6 4 r 6 + 2 = 8 A.
r 1
1.96 4 6 r
E2 6
E
6 12

2 2
100 200
13. P1 = P2 = P 1 = P 2.
R1 R2
R1
ce = 1 Fig. 23.19
R2 4 2
current I 2

A D = (2 A 1
V1 IR R 3 5
= 1 = 1 = 1
V2 IR2 R2 4

P1 I 2 R1 R
= 2 = 1 = 1
P2 I R2 R2 4

III

Multiple Choice Questions Based on Passage

Questions 1 to 5 are based on the following passage B A


passage I
B
Joule’s Law I
q A B t
V A
23.20 Comprehensive Physics—JEE Advanced

I= q
t
A B
V
W = qV

3. V
R causes a current I

2
H = qV = I tV = I 2Rt = V t ( V = IR
R V R
P R V
2 R I
P = H = IV = I2R = V R V
t R
4.
1. A
B
5.
P
A B.
B A.

P P
4 2
2. P P

ANSWERS AND EXPLANATIONS


1. I2 (2R I2 R = 2P

2V 2 V2
=2 = 2P
2R R

2.
4.
5. Let R V
P = V2/R

2
2V V2
3. =4 = 4P. V2
R R R = P = 4P.
R/4 2
V2 P
=
2R 2
23.21

Questions 6 to 9 are based on the following passage 7.


Passage II

8.
1/8

9.
6.

SOLUTION
6. P P1 + P2
= 16 + 80 = 96
7.

2
P1
2 1 8.
8
8

9.
2
P2
2

Questions 10 to 12 are based on the following passage 10. I


Passage III

11.

12.

Fig. 23.20

SOLUTION
10. 3I 2
5 = 45 I = 4 A.
I2 = 15 I = 3I 4
20 4
11. I2 2 =
I1 = 5 I = I 2
20 4
12. I1 6
I 22 I 4
= 6=
4 4
23.22 Comprehensive Physics—JEE Advanced

Questions 13 to 16 are based on the following passage 13. R1


Passage IV
R1 = 4 R2 = 6
R3 = 12 R4 = 10 14. R2
C = 500
S1 S2 S1
S1
S2 15. R3

16. R4

Fig. 23.21

SOLUTION
S1 P = I 2R
P1 P2 P3 = I 12 R1 I 22 R2 I 32 R3
U = 1 CV2 = 1 500 10–6 2 2 2 2
12
2 2
S1 S2
R4 R4
U
P4 = 0.
R1 R2 R3
Let I1
t I2
R2 I3 R3 I1 = I2 + 13. P1 = 3.6 3
I3 R2 R3 6
I 2R 2 = I 3R 3 I2 = 12 I3 I2 = 2I3 14. P2 = 3.6 2
I1 = I2 + I3 6
I1 2 I1 15. P3 = 3 . 6 1
I1 = 2I3 + I3 I3 = I2 =
3 3 6

I1 I2 I3 = I1
2 I1 I1 2 1 16. R4 P4
3 3 3 3

IV

Assertion–Reason Type Questions

one
23.23

Statement-2

1. Statement 1
3. Statement 1
(I – V
T1 T2
T2 T 1.

Statement-2

Fig. 23.22 4. Statement 1


Statement-2

2. Statement 1

Statement-2

SOLUTIONS
1. V V
I–V I1 = I2 =
R R1 R R2

T2 R1 > R2 I2 > I1
T2
ture T1 T2 T 1.
2.

4.
R R
1/P P
V
3. R1 R
R2
R V V2/R
24
Chapter
and Magnetism

REVIEW OF BASIC CONCEPTS

24.1 BIOT-SAVART LAW


dB at a
point whose position vector with respect to a current
element dl is r is given by
I (dI r )
dB = 0 (1)
4 r3
where 0 =4 10–7 Hm–1

24.2 AMPERE'S CIRCUITAL LAW Fig. 24.1 Fig. 24.2

a closed curve is proportional to the current threading or 0I


B= directed into the page if I is clockwise
passing through the closed circuit i.e. 2r
and outside the page if I is anticlockwise.
B·dI = 0I
where 0 is the permeability of free space. For a coil of N turns.
NI
B= 0
2r
MAGNETIC FIELD DUE TO CURRENT (iii)
24.3 (Fig. 24.3).
CARRYING CONDUCTORS
0I
(i) B=
2r 2
I (Fig. 24.1)
directed into the page.
0I
B= directed into the page (away from the For a semi-circular element
2 r
( = )
reader) if the I is upwards and towards the reader
I
if I is downwards. At points Q or S, B = 0 B= 0
(ii) 4r
Fig. 24.3
radius r (Fig. 24.2)
24.2 Comprehensive Physics—JEE Advanced

(iv) straight conductor are parallel. Therefore, dl r


(Fig. 24.4) = 0. Hence B = 0 at point .
0I
(v)
B= (sin + sin ) (Fig. 24.7)
4 r
O due to straight portions and
directed into the page
ST is zero and due to semicircular part is
Special Cases
(a) If the conductor XY I
B = 0 directed into the page.
4r
and point lies
near the centre of
the conductor (as in
Fig. 24.1), = =
90° so Fig. 24.4

0I
B= (sin 90° + sin 90°)
4 r Fig. 24.7

0I If the current in anticlockwise, B is directed


=
2 r towards the reader.
(vi)
(b) If the conductor XY is of (Fig. 24.8)
2 0I a 2 b2
lies near the end X or B= directed into the page
ab
Y as shown in Fig. 24.5,
then = 90° and = 0°, For a square coil (b = a)
Fig. 24.5
then 2 2 0I
B=
I a
B = 0 (sin 90° + sin 0°)
4 r
0I
=
4 r
Note:

near one of its ends.


(c) If the conductor XY L and point
lies on the right bisector of the conductor, as
Fig. 24.8
shown in Fig. 24.6, then
= and (vii) llow metal pipe of
L/2 (Fig. 24.9)
sin = sin =
x 0I
At point ,B=
L/2 2 r
=
L 2
r2
2
L
=
2
4r L2
0I L
so B = Fig. 24.6
2 r 4r 2
L2
(d) If the point lies on the straight conductor or
on its axis, then dl and r for each element of the Fig. 24.9
Magnetic Effect of Current and Magnetism 24.3

(Fig. 24.9) For a toroid of radius , B = 0nI, where n =


Inside the pipe at a point at a distance r from the N N
axis, ; N = total no. of turns. Outside the
L 2
Ir
B= 0 2 solenoid, B = 0.
2
Outside the pipe at a point at a distance r from 24.1
the axis
I bent wires and STU lying in the x- plane and
B= 0
2 r each carrying a current I as shown. Find the magni-
(viii) O.
(Fig. 24.10) Given OQ = OT = a

At point ,B= 0 2M
2 2 3/2
4 ( )
2
where M = IA = I is the magnetic moment.
If current I is anticlockwise B is directed from O
to . For clockwise current B is form to O.

Fig. 24.10

(ix)
revolving in a circular orbit of radius r with speed
r with speed v (Fig. 24.11) Fig. 24.12
e ev
Current along orbit I = = en =
T 2 r SOLUTION
The direction of I is opposite to the direction of mo- As point O is along the line segments and ST, the
tion of the electron. O due to and ST is zero. The
O O due to wires and TU respec-
is tively are
I I (k ) I (k )
B= 0 B1 = 0 and B2 = 0
4r 4 (OQ) 4 (OT )
ev both directed along the positive z-axis. The resultant
where I = and
2 r t O is ( OQ = OT = a)
is directed into the Fig. 24.11
0I 0I
page. B = B1 + B2 = 2 (k ) k
e vr 4 a 2 a
Magnetic moment M = IA = I r2 =
2
24.2
(x)
solenoid I in
In the middle region B = the opposite direction are placed perpendicular to the
0nI;n = no. of turns
per unit length. x- plane. One wire is located at point (0, a, 0) and
the other wire at Q (0, –a, 0). Find the magnitude and
0 nI
At the ends of solenoid, B = direction of at point A(x, 0, 0).
2
24.4 Comprehensive Physics—JEE Advanced

SOLUTION SOLUTION
Refer to Fig. 24.13. Wire 1 carries a current I along (a) (i) Radius r of wire A when it is bent into a
the positive z-direction and wire 2 carries a current I circle is given by
along the negative z-direction. L
= OQ = a, OA = x, = QA = r. 2 r=L r=
2

loop is
0I 0I
B1 = (1)
2r L
O
due to wire is (OT = a)
0I
B = (sin 45° + sin 45°)
4 a
0I
=
Fig. 24.13 2 2 a
4 0I L
M A due to wire 1 is = a
2 L 8
0I 0I
B1 =
2 ( 2 a x2) 2

According to Biot-Savart law, B1 is perpendicular to


both and wire 1 and therefore in the x- plane.
A due to wire 2 is

0I
B2 =
2
2 a x2
The -components of B1 and B2 cancel each other
but the x-components add up. These components are
B1 cos and B2 cos both along the positive x-direc-
A is
Fig. 24.14
B = B1 + B2 = (B1 cos + B2 cos ) i
Since centre O of the square is at the same
0I a distance from each side of the square and
= (i ) each arm carries the same current, the mag-
2 2 2
(a x ) a x2
a a of the same and in the same direction. Hence
cos
r a 2
x 2 O is

0I a 16 0I 8 2 0I
= 2 2
(i) B2 = 4B = (2)
(a x ) 2 L L
(b) Dividing (2) by (1) we get
24.3
Two wires A and B have the same length L and carry B2 8 2 8 1.41
= 1.16
equal currents I. Wire A is bent into a circle and wire B1 2
(3.14) 2
B is bent into a square. (a) Obtain expression for the
i) the centre of the Hence B2 > B1 centre
circular loop and (ii) the centre of the square. Which due to the square loop will be greater than that
due to the circular loop.
Magnetic Effect of Current and Magnetism 24.5

24.4 24.5
Figure 24.15 shows a wire loop ABCDEA carrying a A wire ABCDE is bent as shown in Fig. 24.16. The
current I as shown. wire carries a current I and the radius of the bent coil
BCD is r. Find the magnitude and direction of the
Given AE = ED = a and AB = CD = a/2.
O.
Find the magnitude and direction of the magnetic
F where BF = CF = a/2. SOLUTION
The straight line segments AB and DE are collinear
with O AB and DE at
O is zero. Angle subtended at O by arc BCD = 2 – .
BCD at O is
0I 2
B =
2r 2

Fig. 24.15

SOLUTION
F is
B = BAB + BBC + BCD + BDE + BEA
Fig. 24.16
Since point F lies in line with current elements AB
and CD, BAB = BCD = 0 The current through BCD is clockwise. Therefore, the
0I 0I O is into the page
Also BDE = BEA = (sin 0° + sin 45°) =
4 a 4 2 a and away from the reader.
directed out of the page and towards the reader.
24.6
0I A wire ABCEF is bent as
BBC = (sin 45° + sin 45°)
4 BC/2 shown in Fig. 24.17 and
directed into the page and away from the reader. Now caries a current I. The
radius of the smaller arc
2 2
a a a ABC is r1 = r and that of
BC = BF 2 FC 2
2 2 2 the bigger arc is r2 = 2r.
Find the magnitude of the
0I 1 1 0I
BBC = Fig. 24.17
a 2 2 a O.
4
2 2
directed into the page. SOLUTION
0I ABC at O is
Now BDE + BEA = directed out of the page.
2 2 a 0I 2
B1 =
Since BBC > BDE + BEA B is directed into 2r1 2
the page and has a magnitude DEF at O is
0I 0I 0I 1
B = 1 0I
a 2 2 a a 2 2 B2 =
2r2 2
24.6 Comprehensive Physics—JEE Advanced

since B1 and B2 are both directed into the page, the 2


0 Ir
O is B 2 r= 0i = 2

0I 2 0I
B = B1 + B2 =
2r1 2 2r2 2 0I
B= 2
r
Putting r1 = r and r2 = 2r, we get 2

0I
B = 1 NOTE
2r 4
1
(1) For r < ; B r and for r > ; B . Figure 24.19
24.7 r
A long straight cylinder of radius carries a current shows the variation of B with r.
I which is uniformly distributed across its cross-sec-

r from the axis of the cylinder in cases (a) r > and


(b) r <

SOLUTION
Figure 24.18 shows the cross-sectional view of the
cylinder.

Fig. 24.19

walls. Therefore, in the case r < [Fig. 24.18 (b)], no


current threads the Amperian loop. Hence B = 0 for
points inside a hollow cylinder.

24.8
The current density J (current per unit area) in a solid
cylinder of radius varies with distance r from its
axis as J = kr where k is a constant. Find the magnetic
Fig. 24.18 where (a) r > and (b) r < .
Case (a) r > . For this case, the Amperian loop is
SOLUTION
a circle of radius r concentric with the cross-section
[Fig. 24.18 (a)]. For this loop, L = 2 r and the current
threading the loop is i = I. From Ampere’s circuital Current I = JdA kr (2 rdr )
law. Case (a) We take the Amperian loop of radius r > .
I Since the loop is outside the cylinder, the current
BL = 0i B 2 r = 0I B= 0
2 r through the loop is
Case (b) r < . For this case, L = 2 r [Fig. 24.18 (b)] 3
2
and the current threading the loop is I= kr (2 rdr ) 2 k r 2 dr
3
i = current per unit cross-sectional area of the cylinder 0 0
cross-sectional area of the Amperian loop 3 3
2 0
2 B 2 r = 0I = B
I Ir 3 r
= 2
r2 2
Case (b) For r < , the current through the Amperian
From Ampere’s law, loop is
Magnetic Effect of Current and Magnetism 24.7

r
2 kr 3 qB
I = kr (2 rdr ) (vi) Frequency of revolution is v = which is indepen-
3 2 m
0
dent of both v and r.
0 2 kr 3 0 kr
2
Case (b): If v is inclined to B at an angle , the particle
B 2 r = 0I = B
3 3 mv sin
moves in a helical path. The radius of helix is r = ,
qB
FORCE ON A MOVING CHARGE IN A
24.4 2 m
MAGNETIC FIELD time period T = and pitch of the helix = v cos T
qB
The force on a charge q moving with a velocity v in a
B is given by Applications
F = q(v B) (i) -
The direction of F is perpendicular to both v and B. The
magnitude F of vector F is given by region of magnetic
F = qvB sin
where is the angle between vectors v and B. describes a semi-circle
(1) F = 0 if v = 0, i.e. a charge at rest does not of radius.
experience any magnetic force. mv
(2) F = 0 if = 0 or 180°, i.e. the magnetic force r=
qB Fig. 24.20
vanishes if v is either parallel or antiparallel to
the direction of B.
(3) F is maximum = Fmax if = 90°, i.e. if v is T m
perpendicular to B, the magnetic force has a t=
2 qB
maximum value given by
(ii) If the particle enters the
Fmax = qvB
The direction of the force when v B is given as shown in Fig. 24.21,
by Fleming’s left hand rule. then
(4) If v is perpendicular to both E and B and E is
mv
E r=
perpendicular to B, then F = 0 if v = . qB
B
MOTION OF A CHARGED PARTICLE 2 m
24.5 and t=
IN A MAGNETIC FIELD qB
Fig. 24.21
Case (a): If v is perpendicular to B, the particle describes a where is in radian.
circle in the region of the magn F v. (iii) In Fig. 24.22, the particle wall not be able to hit
(i) The speed along the circular path is constant. the wall if d > r, i.e.
(ii) The kinetic energy is constant. mv mv
(iii) Velocity and momentum continually change. d> B
qB qd
mv 2
(iv) The radius r of the circular path is given by
qvB r

mv 2mK
r=
qB qB
where m = mass of particle and K = kinetic energy. If
the particle is accelerated through a potential differ-
ence V, then K = qV.
2 m
(v) Time period of revolution is T =
qB
Fig. 24.22 Fig. 24.23
24.8 Comprehensive Physics—JEE Advanced

(iv)
SOLUTION
Refer to Fig. 24.24.
d qBd
sin =
r mv
Linear defection x = r(1 – cos )

24.9

through a potential difference of 18 kV and enters a

certain initial velocity. What is the trajectory of the


Fig. 24.24
initial velocity and (b) makes an angle of 30° with the
10–31 kg. A is
7
SOLUTION 0I 4 10 10 5
B= 4 10 T
V = 18 103 V 2 r 2 0.05

1 2eV 1/2 directed inwards along the negative z-direction


mv2 = eV v= (a) = 90°. Therefore, force on proton is
2 m
F = qvB sin
1/2
2 (1.6 10 19
) (18 103 ) = (1.6 10–19) 107 (4 10–5) sin 90°
= 8 107 ms 1
9 10 31 = 6.4 10–17 N
According to Fleming's L.H. rule, the direc-
(a) Since v is to B, = 90°, the trajectory of tion of the force is parallel to the wire and
the electron is circular having a radius opposite to the direction of current I, i.e. F
31
is along the negative -direction
mv (9 10 ) (8 107 ) (b) = 90°, F = qvB = 6.4 10–17 N. The force
r = 19
eB (1.6 10 ) 0.1 is directed towards the wire, i.e. along nega-
–3 tive x-direction
= 4.5 10 m = 4.5 mm
(c) = 180°. F = qvB sin 180º = 0
(b) The trajectory of the electron is helical. The
radius of heix is 24.11
A proton and an -particle move perpendicular to a
mv mv
r= sin -particle is
eB eB four times that of a proton and its charge is twice that
= 4.5 mm sin 30° = 2.25 mm of a proton. Find the ratio of radii of the circular path
followed by them if both
EXAM 24.10 (a) have equal velocities,
A long straight wire lying along the -axis carries (b) have equal linear momenta,
a current of 10 A along the positive -direction. A (c) have equal kinetic energies and
proton moving with a velocity of 107 ms–1 is at a (d) are accelerated through the same potential
distance 5 cm from the wire at a certain instant. Find difference.
the magnitude and direction of the force acting on the
proton at that instant if its velocity is directed SOLUTION
(a) along the negative x-direction m q
(b) along the positive -direction and Given =4 and =2
mp qp
(c) along the positive z-direction
Magnetic Effect of Current and Magnetism 24.9

mv mpv m v
(a) r = rp and r
qB qp B q B
rp mp q 1 1
2
r m qp 4 2
mv
(b) r =
qB
1 2 2K
Kinetic energy K = mv v
2 m
m 2K 1 Fig. 24.27
r= 2 mK
qB m qB (iv) Force per unit length between two long straight
rp q mp 1 1 and I2 and
2 1
r qp m 4
mv p 0 I1 I 2
f =
(c) r = 2 r
qB qB
attractive if I1 and I2 are in the same direction and
rp q repulsive if I1 and I2 are in opposite directions.
2 Force on a segment of length l of either wire is
r qp
F = f l.
(d) K = qV. Therefore (v) 1 placed at a
1 1 2mV
r= 2mqV
qB B q current I2 as shown in Fig. 24.28.
II L
rp mp q 1 1 F = 0 1 2 loge 1 vertically upwards.
2 2 r x
r m qp 4 2

FORCE ON A CURRENT CARRYING


24.6
CONDUCTOR IN A MAGNETIC FIELD
(i) Force on a straight conductor placed perpendicu-

F = BIL upwards if current I is from left to right


and downwards if I is from right to left (given by
Fleming’s left hand rule)
Fig. 24.28

(vi) 1

2 as shown in Fig. 24.29.

Fig. 24.25 Fig. 24.26

(ii)
(Fig. 24.26)
F = BI(2 ) = 2 vertically upward for clockwise
current and downward for anticlockwise current
(iii)
Net force F = F1 – F2 = 0 Fig. 24.29
24.10 Comprehensive Physics—JEE Advanced

Force F1 and F2 being equal and opposite cancel 10 7 ) 2


and F3 and F4 are given by expression above. Net = (4 5 20 5 10
2 5 10 2
force on coil is
= 2 10–5 N (repulsive since I1 and I2 are in
0 I1 I 2 a 0 I1 I 2 a opposite directions)
F = F3 – F4 =
2 x 2 (x L)
Force exerted by on CD is
0 I1 I 2 aL
F= 0 I1 I 2
2 x( x L) F2 = CD
2 r2
directed towards the wire (attractive)
(4 10 7 ) 5 20 5 10 2

24.12 = 2
2 2 10
The battery of a car is connected to the motor by 50
cm long wires which are 1.0 cm apart. If the current = 5 10–5 N (attractive since I1 and I2 are in the
same direction)
Is the force attractive or repulsive.
to current in is directed outwards (towards the
SOLUTION reader) and perpendicular to the plane of the coil.
Force per unit length is Therefore, forces F3 and F4 on BC and AD are equal
and opposite and hence cancel each other. Therefore,
0 I1 I 2 (4 10 7 ) 200 200 1 the net force on coil ABCD is
f= 2
0.8 Nm
2 r 2 (1.0 10 ) F = F2 – F1 = 5 10–5 – 2 10–5 = 3 10–5 N
F = f l = 0.8 0.5 = 0.4 N (attractive). Hence coil is attracted towards .
Since the currents in the wires are in opposite direc-
tion, the force is repulsive. 24.14
A particle of charge q and mass m moving in region
24.13 I with a velocity v enters normally a region II of
A small rectangular loop ABCD of sides 5 cm and width d where a uniform
3 cm carries a current of 5 A. It is placed with its B (directed
longer side parallel to a long straight conductor inwards) exists as shown
of length 5 m at a distance of 2 cm from it as shown in Fig. 24.31. There is no
in Fig. 24.30. If the current in
force on the loop. Is the loop attracted towards or and III.
(a) What is the maxi-
mum speed (vmax)
of the particle so
that it returns back
Fig. 24.31
(b) What will happen if
v = 2 vmax

SOLUTION
(a) Refer to Fig. 24.32(a). The particle describes
a circular path of radius r = mv/qB in region
Fig. 24.30 II. It will return to region I if it describes a
semicircle in region II. This happens if
SOLUTION mv qBd
r<d < d or v <
Force exerted by on AB is qB m

0 I1 I 2 qBd
F1 = AB vmax =
2 r1 m
Magnetic Effect of Current and Magnetism 24.11

(a) Tension in the wires will be zero if


Fm = Fg
BIl = mg
mg (60 10 3 ) 10
B= = 0.24 T
Il 5 (50 10 2 )
(b) If current I is reversed, force Fm acts down-
wards. Hence
Tension T = BIl + mg
= 0.24 5 0.5 + 60 10–3 10 = 1.2 N
Fig. 24.32 TORQUE ON A CURRENT CARRYING
24.7 COIL IN A MAGNETIC FIELD
(b) Refer to Fig. 24.32 (b). If v > vmax, the particle
is cross over to region III after describing a
The torque on a coil of N turns, area A carrying a current
circular trajectory in rejoin II with O as the
I B is given by
centre. In region III, te particle is move along
the tangent at Q. The particle will suffer a =M B
deviation . Magnitude of torque is = MB sin = NIAB sin
In triangle where M = NIA is the magnetic moment and is the angle
d
sin = The magnitude of torque on a coil in radial magnetic
OQ r

qBd vmax =k
sin =
mv v where k is the restoring couple per unit twist and is the
1 = 90°.
If v = 2 vmax, then sin = = 45° Then
2
k
NIAB = k I= or I
NAB
24.15
Current sensitivity of the galvanometer is
A straight horizontal conducting rod of length 50 cm
NAB
and mass 60 g is suspended by two vertical wires at Cs =
its ends. A current of 5 A set up in the rod. I k
TORQUE ON A BAR MAGNET IN A
to the conductor in order that the tension in 24.8
MAGNETIC FIELD
(b) What will be the tension in the wires if the The magnetic dipole moment of a bar magnet of pole
direction of the current is reversed, keeping strength q and length (2a
M = q(2a)
Ignore the mass of the wires and take g = 10
It is a vector pointing from the south to the north pole
m/s–2.
of a magnet.
SOLUTION Force on north pole N of magnet = qB (in the direction
of B)
Refer to Fig. 24.33. Force on south pole S of the magnet = –qB (opposite to B)

opposite forces on the magnet. The two forces, therefore,


constitute a coupe which tends to rotate the magnet in the
clockwise direction. The arm of the couple is 2 a. The
torque is given by
= arm of the coupe force
= 2a qB = q(2a) B
Fig. 24.33 or =M B
24.12 Comprehensive Physics—JEE Advanced

where M = q(2a) is called the magnetic moment of the bar


magnet. The direction of is perpendicular to both M and
B. If M and B are both in the plane of the paper then the
torque will be perpendicular to the plane of the paper
and directed into it away from the reader. The magnitude
of the torque is
= MB sin
where is the angle between M and B.
The SI unit of M is Nm T–1 or JT–1 (joule per tesla).

POTENTIAL ENERGY OF A MAGNETIC


24.9 Fig. 24.34 Fig. 24.35
DIPOLE
The magnetic potential energy of a magnetic dipole in any l M
orientation B is M =m
2 2
the dipole from its zero energy position ( = 90°) to the NOTE
given position. .
Magnetic dipole moment M is a vector quantity directed
U = –MB cos from south pole to north pole.
In vector notation,
U = – (M·B) experiences no net force but experiences a torque
For stable equilibrium U is minimum. Hence = 0 = M B. The magnitude of is = MB sin
and = 0. For unstable equilibrium, U is maximum i.e. where is the angle between M and B.
= 180°. Hence = 0 (a) when = 90°, max = MB
(b) when = 0°, min = 0 (stable equilibrium)
24.10 SOME USEFUL TIPS (c) when = 180°, min = 0 (unstable equilib-
rium)
1. Magnetic dipole moment of a bar magnet is M = m l, Potential energy is U = –M.B = –MB cos .
where m is pole strength and l is the length of the When = 0°, P.E is minimum Umin = –MB.
magnet. The value of M depends on the volume of U is max = Umax = MB when = 180°
the magnet. (d) Work done in rotating the magnet from 1 to
(a) If a magnet is cut into two equal parts by cut- 2 is
ting it by a plane along its length, its volume W = MB (cos 1 – cos 2)
is halved, Hence the magnetic dipole moment
of a piece is halved = M/2. The pole strength experiences a force as well as a torque.
M
m = is also halved as length l remains 3. The time period of a bar magnet oscillating in a
l
I
the same. T=2 , where I
(b) If a magnet is cut into two equal parts by MB
cutting it by a plane transverse to its length, ml 2
is the moment of inertia of the bar magnet = ,
the volume and length are both halved. Hence 12
the magnetic moment becomes M/2 but pole m = mass of magnet and l = length of magnet.
strength m remains the same. (a) If a bar magnet is cut into two equal parts
(c) If a wire of magnetic dipole moment M and by cutting along its length, then each part has
length l is bent as shown in Fig. 24.34, the M = M/2 and I = I/2. Hence T = T.
1 (b) If a bar magnet is cut into two equal parts by
distance between the pole becomes and
2 cutting perpendicular to its length, then each
magnetic moment becomes part has M = M/2 and I = I/8. Hence T = T/2.
l M (c) If two bar magnets of magnetic moments M1
M =m
2 2 and M2 are placed one on top of the other as
If the wire is bent as shown in Fig. 24.35, shown n Fig. 24.36, then time period is given
the magnetic moment becomes by (since I = I1 + I2 and M = M1 + M2)
Magnetic Effect of Current and Magnetism 24.13

( I1 I 2 ) 24.16
T1 = 2
( M1 M 2 ) B A closely wound solenoid of 1000 turns and area of
cross-section 5 cm2 carries a current of 3 A. It is sus-
pended through its centre (a) what is the magnetic

10–2 T
is set up at an angle of 30° with the axis of the

Fig. 24.36 SOLUTION


If the magnets are placed as shown in Fig. (a) Magnetic moment M = NIA = 1000 3 (5
24.37, then I = I1 + I2 but M = M1 – M2 and 10–4) = 1.5 JT–1 or Am2
( I1 I 2 )
T2 = 2
( M1 M 2 ) B acting on the solenoid is zero
Torque = MB sin = 1.5 (8 10–2) sin 30°
M (T22 T12 ) = 6 10–2 J
T1 and T2 are related as 1
M2 (T22 T12 )
24.17
In a hydrogen atom, the electron moves in a circu-
lar orbit of radius 0.5 Å making 1016 revolutions per
second. Calculate the magnetic moment associated
with the orbital motion of electron.

SOLUTION
Fig. 24.37
M = e r2 = 3.14 (1.6 10–19) 1016 (0.5
e to a bar magnet 10–10)2 = 1.26 10–23 Am2
(a) At a point on axial line (Fig. 24.38)
2Mr 24.18
Ba = 0
parallel to M = m 2l. A bar magnet is suspended at a place where it is acted
4 (r 2 l 2 )2
0 2M
For a very short magnet (l << r), Ba =
4 r3 a magnitude 2 10–2 T. The magnet attains stable

SOLUTION
B1 exerts anticlockwise torque 1
Fig. 24.38
to orient M B2 ex-
(b) At a point Q on erts a stockwise torque 2 to orient M along itself
the equatorial line (Fig. 24.40). For equilibrium,
(Fig. 24.39)

0 M
Be = 2
4 (r l 2 )3/2
antiparallel to M

For l << r,
0M
Be = 3
4 r Fig. 24.40
Fig. 24.39
24.14 Comprehensive Physics—JEE Advanced

1 = 2 24.20
MB1 sin 1 = MB2 sin 2 A wire loop ABCD carrying a current I2 is placed on a
frictionless horizontal table as shown in Fig. 24.42. A
B1 sin 1 2 10 2 sin 30
B2 = =10–2 T long straight wire carrying a current I1 is placed at
sin 2 sin 45 a distance a from side AB = l. Find the work done by

24.19 to position Q
A uniform wire is bent into the shape of an equilateral
triangle of side a. It is suspended from a vertex at
B exists parallel
to its plane. Find the magnitude of the torque acting
on the coil when a current I is passed through it.

SOLUTION
Area of the coil is (AB = a, BD = a/2)

Fig. 24.42

SOLUTION

Fig. 24.41

A= 2 area of triangle ABD


Fig. 24.43
1
=2 AD BD
2 Divide the loop into a large number of elements
each of a very small width dx. Consider one such
1 3 a element at a distance x from [see Fig. 24.43].
=2 a
2 2 2

3 2 0 I1
= a B= directed inwards
4 2 x
Magnetic moment of the loop is Magnetic moment of the element is
3 2 dM = I2 area of element
M = IA = I a
4 = I2 ldx
Since the current is clockwise the direction of vector Since the current in the coil is anticlockwise, dM is
M is perpendicular to the plane of the coil directed directed outwards. Hence angle between vectors B
inwards as shown in Fig. 24.41. Hence = 90°. The and dM is = 180°.
magnitude of the torque acting on the coil is
Potential energy when the wire is a distance x from
3 2 3 2 the elements is
= MB sin = Ia B sin 90° = Ia B
4 4 dU = –dM·B = –dM B cos
Magnetic Effect of Current and Magnetism 24.15

By symmetry, the potential energy of the system when


0 I1
= –I2 ldx cos 180° the wire is shifted to position P Q is
2 x
UP Q = –UPQ
0 I1 I 2 l
dx
= Work done in shifting the wire from position PQ
2 x to P Q is
Potential energy of the system when the wire is at
position PQ is W = –(UP Q – UPQ)
b
0 I1 I 2 l dx 0 I1 I 2 l b 0 I1 I 2 l b
UPQ = ln = 2UPQ = ln
2 x 2 a a
a

Multiple Choice Questions with Only One Choice Correct

B 2 RC
1. The dimensions of (where B is magnetic
0
R is resistance, C is capacitance and 0 is
permeability of free space) are the same as those
of
(a) impulse
(b) angular momentum
(c) energy
(d) viscosity
2. A current carrying metal wire of diameter 2 mm Fig. 24.44

5. A straight wire PQ of length L carries at current I.


2 10–3 T. The current in the wire is
(a) 10 A (b) 20 A R is
[see Fig. 24.45]
(c) 40 A (d) 40 2 A
(a) zero
3.
with velocity vector making an angle of 30° with 0I
(b)
2 2 L
trajectory of pitch x. The radius of the helix is
0I
x x (c)
(a) (b) 4 2 L
2 2 2
0I
(d)
x 3x 2 L
(c) (d)
2 3 2 Fig. 24.45

4. A current carrying wire AB is placed near another 6. A metal rod of length L and mass M is slipping
CD as shown in Fig. 24.44. Wire CD down a smooth inclined plane of inclination 30°
wire AB is free to move. When a current is passed with a constant velocity v. The rod carries a cur-
through wire AB, it will have rent I directed inwards perpendicular to the plane
(a) only translational motion
(b) only rotational motion upwards perpendicular to the length of the rod ex-
(c) both translational as well as rotational
motions
Mg Mg
(d) neither translational nor rotational motion. (a) (b)
IL 2 IL
24.16 Comprehensive Physics—JEE Advanced

1/ 2
3 Mg Mg R2 R2
(c) (d) (a) (b)
2 IL 3 IL R1 R1
7. Equal current I 2
R1 R1
ACB and ADB of equal radius r as shown in (c) (d)
Fig. 24.46. If R2 R2
O is IIT, 1988
11. Two identical coils carry equal currents and have a
0I
(a) common centre, but their planes are at right angles
r to each other. What is the magnitude of the resultant
0I
(b) coil alone is B?
2r
(a) zero (b) B/ 2
0I
(c)
3r (c) 2 B (d) 2B
Fig. 24.46 12. When a charged particle moves perpendicular to a
0I
(d)
4r (a) energy and momentum both change
8. The wire loop PQRSP formed by joining two semi– (b) energy changes but momentum remains
circular wires to radii R1 and R2 carries a current I unchanged
as shown in Fig. 24.47. The magnitude of the mag- (c) momentum changes but energy remains
netic induction at centre C is unchanged
I 1 1 I 1 1 (d) energy and momentum both do not change.
0 0
(a) (b) 13. A proton with kinetic energy K describes a circle of
4 R1 R2 4 R1 R2
radius r –particle
I 1 1 I 1 1 with kinetic energy K moving in the same magnetic
0 0
(c) (d)
2 R1 R2 2 R1 R2
r
(a) (b) r
IIT, 1983 2
(c) 2 r (d) 4 r

14. An –particle moving with a velocity v in a

at frequency called the cyclotron frequency. The


cyclotron frequency of a proton moving with a
speed 2v

Fig. 24.47
(a) (b)
4 2
9. (c) (d) 2
diagonals of a square loop of side L carrying a 15. -
current I is ated through a potential difference of 2.88 kV and
I 2 0I
(a) 0 (b)
L L
of an electron = 9 10–31 kg. The trajectory of the
2 0 I 2 2 0 I electron is a
(c) (d) (a) circle of radius 1.6 mm
L L
(b) circle of radius 0.9 mm
10. Two particles X and Y having equal charges, (c) helix of radius 0.9 mm
after being accelerated through the same potential (d) helix of radius 1.6 mm
16. In the region around a charge at rest, there is
and describe circular paths of radii R1 and R2 resp-
ectively. The ratio of mass of X to the mass of Y is
Magnetic Effect of Current and Magnetism 24.17

l2 I l2 I
(c) (d)
17. In the region around a moving charge, there is 2 4
23. Two circular current carrying coils of radii 3 cm and
6 cm are each equivalent to a magnetic dipole hav-
ing equal magnetic moments. The currents through
the coils are in the ratio of
18. An electron is accelerated to a high speed down the (a) 2 :1 (b) 2 : 1
axis of a cathode ray tube by the application of a
potential difference of V volts between the cathode (c) 2 2 : 1 (d) 4 : 1
and the anode. The particle then passes through 24. An electron of charge e moves in a circular orbit
of radius r
experiences a force F. If the potential difference due to orbital motion of the electron at the site of
between the anode and the cathode is increased to 2 the nucleus is B. The angular velocity of the
V, the electron will now experience a force electron is
2 0 eB eB
(a) F/ 2 (b) F/2 (a) = (b) = 0
4 r r
(c) 2 F (d) 2 F
19. A magnetic needle is kept in a non–uniform 4 rB 2 rB
(c) = (d) =
0 e 0e
(a) a force as well as a torque 25. Three long, straight and parallel wires C, D and
(b) a force but no torque
G carrying currents are arranged as shown in
(c) a torque but no force
Fig. 24.49. The force experienced by a 25 cm length
(d) neither a force nor a torque.
of wire C is
20. A conducting circular loop of radius r carries a con-
(a) 0.4 N (b) 0.04 N
stant i B
(c) 4 10–3 N (d) 4 10–4 N
such that B is perpendicular to the plane of the loop.
The magnetic force acting on the loop is
(a) i rB (b) 2 i r B
(c) zero (d) irB
IIT, 1983
21. A rectangular loop carrying a current i is situated
near a long straight wire such that the wire is paral-
lel to one of the sides
of the loop. If a
steady current I is es-
tablished in the wire,
as shown in Fig.
Fig. 24.49
24.48, the loop will
(a) rotate about an 26. A potential difference of 600 V is applied across the
axis parallel to plates of a parallel plate capacitor. The separation
the wire between the plates is 3 mm. An electron projected
(b) move away vertically, parallel to the plates, with a velocity of
from the wire Fig. 24.48 2 106 ms–1
(c) move towards
the wire between the capacitor plates?
(d) remain stationary. (a) 0.1 T (b) 0.2 T
22. A wire of length l metres carrying a current I am- (c) 0.3 T (d) 0.4 T
peres is bent in the form of a circle. The magnitude
of the magnetic moment is 27. A current I
long, straight, thin walled pipe. Then
l I2 l I2
(a) (b) is the same but not zero.
2 4
24.18 Comprehensive Physics—JEE Advanced

+ y direction enters a region of uniform electric


is zero.
B and E are directed
the pipe. respectively along
(a) – y axis and – z axis
points inside the pipe. (b) + z axis and + x axis
IIT, 1993 (c) + x axis and + z axis
28. Two straight and long conductors AOB and COD (d) – x axis and – y axis
are perpendicular to each other and carry currents 34. A proton of mass 1.67 10–27 kg and charge
of I1 and I2 1.6 10–19 C is projected with a speed of 2 106
a point P at a distance a from point O in a direction ms–1 at an angle of 60° to the x-axis. If a uniform
perpendicular to the plane ABCD is
y-axis, the path of the proton is
0 0 (a) a circle of radius 0.1 m and time period
(a) (I1 + I2) (b) (I1 – I2)
2 a 2 a 2 10–7 s
(b) a circle of radius 0.2 m and time period
2 I1 I 2 10–7 s
(c) 0
(I 1 + I22)1/2 (d) 0
2 a 2 a I1 I 2 (c) a helix of radius 0.1 m and time period
2 10–7 s
29. If a magnetic dipole of dipole moment M is rotated
(d) a helix of radius 0.2 m and time period
through an angle with respect to the direction of
4 10–7 s
H, then the work done is
IIT, 1995
(a) MH sin (b) MH (1 – sin )
35. A proton, a deuteron and an alpha particle having
(c) MH cos (d) MH (1 – cos )
the same kinetic energy are moving in circular tra-
30. A 2 MeV proton is moving perpendicular to a
rp, rd and r
denote respectively the radii of trajectories of these
proton is
(a) 2.5 10–10 N (b) 8 10–11 N particles, then
(c) 2.5 10 N –11
(d) 8 10–12 N (a) r = rp < rd (b) r > rd > rp
(c) r = rd > rp (d) rp = rd = r
31. A straight section PQ of a circuit lies along the
a a IIT, 1997
x-axis from x = – to x = and carries a current 36. Two particles each of mass m and charge q, are
2 2
attached to the two ends of a light rigid rod of
I PQ at point
length 2l. The rod is rotated at a constant angular
x = + a will be
speed about a perpendicular axis passing through
(a) proportional to a (b) proportional to a2
its centre. The ratio of the magnitudes of the
1 magnetic moment of the system and its angular
(c) proportional to (d) equal to zero.
a momentum about the centre of the rod is
32. Two charged particles M and N enter a region of q q
(a) (b)
- 2m m
2q q
Fig. 24.50. The possible reason is (c) (d)
m m
(a) The charge of M is greater than that of N
IIT, 1998
(b) The momentum of M is greater than that of N
(c) The charge to 37. Two very long straight parallel wires carry steady
mass ratio of M currents I and – I. The distance between the wires
is greater than is d. At a certain instant of time, a point charge q
that of N is at a point equidistant from the two wires, in the
(d) The speed of M plane of the wires. Its instantaneous velocity v is
is greater than perpendicular to this plane. The magnitude of the
that of N.
33. The monoener- at this instant is
getic beam of elec- I qv
0 0 I qv
trons moving along (a) (b)
Fig. 24.50 2 d d
Magnetic Effect of Current and Magnetism 24.19

2 I qv 42. An electron revolves in a circle of radius 0.4 Å with


0
(c) (d) zero a speed of 106 m/s in Hydrogen atom. The magnetic
d
IIT, 1998 motion of the electron in tesla is [ 0 = 4 10–7
38. A charged particle is released from rest in a region H/m; Charge on the electron = 1.6 10 C] –19

(a) 0.1 (b) 1.0


which are parallel to each other. The particle will
(c) 10 (d) 100
move in a
(a) straight line (b) circle 43. A proton of velocity (3 i 2 j) ms–1 of
(c) helix (d) cycloid
magnetic induction (2 j 3k) tesla. The accelera-
IIT, 1999
tion produced in the proton is (charge to mass ratio
39. An ionized gas contains both positive and negative
of proton = 0.96 108 C kg–1)
ions. If it is subjected simultaneously to an electric
x (a) 2.8 108 (2 i 3 j)
along the + z direction, then
y direction (b) 2.88 108 (2 i 3 j 2k)
and negative ions towards – y direction
(c) 2.8 108 (2 i 3k)
y direction
y direction (d) 2.88 108 ( i 3 j 2k)
y direction and
negative ions towards + y direction 44. A long wire carries a steady current. It is bent into
IIT, 2000 at the centre of the loop is B. The wire is then bent
40. Two long parallel wires are at a distance 2d apart. into a circular coil of n turns and the same current
plane of the paper, as shown in Fig. 24.51. The of coil will be
B along the line XX
(a) nB (b) n2 B
is given by
IIT, 2000 (c) 2nB (d) 2n2 B
45.
circular loop of radius 3 cm at a point on its axis at a
distance of 4 cm from the centre is 54 T. The mag-
T) at the centre of the loop will be
(a) 250 (b) 150
(c) 125 (d) 72
46. A wire ABCDEF (with each side of length L) bent
as shown in Fig. 24.52 and carrying a current I is
B parallel to
the positive y–direction. What is the magnitude and
direction of the force experienced by the wire?

Fig. 24.51

41. An electron moves with a speed of 2 105 ms–1


along the positive x B
= (i 4 j 3k ) tesla. The magnitude of the force
(in newton) experienced by the electron is (the
charge on electron = 1.6 10–19 C)
(a) 1.18 10–13 (b) 1.28 10–13 Fig. 24.52
(c) 1.6 10–13 (d) 1.72 10–13 (a) BIL along positive z-direction
24.20 Comprehensive Physics—JEE Advanced

(b) BI 2/L along positive z-direction (a) 1 MeV (b) 2 MeV


(c) BIL along negative z-direction (c) 4 MeV (d) 0.5 MeV
(d) BL/I along negative z-direction 51. A loosely wound helix made of stiff wire is
47. long bent wires mounted vertically with the lower end just touch-
are placed in the x-y plane as shown in Fig. 24.53. ing a dish of mercury. When a current from the
The wires carry a current I = 10 A each as shown. battery is started in the coil through the mercury
The segments RL and SM are along the x-axis. The (a) the wire oscillates
segments PR and QS are along the y-axis, such (b) the wire continues making contact
that OS = OR = 0.02 m. What is the magnitude and (c) the wire breaks contact just when the current
direction of the magnetic induction at the origin O? is passed
(a) 100 Wb m–2 vertically upward (d) the mercury will expand by heating due to
(b) 10–4 Wb m–2 vertically downward passage of current.
(c) 10–4 Wb m–2 vertically upward IIT, 1981
(d) 10–2 Wb m–2 vertically downward 52. A non-planar loop of conducting wire carrying a
current is placed as shown in Fig. 24.54. Each of the
IIT, 1989
straight sections of the loop is of length 2a. The

P(a, 0, a) points in the


direction
1
(a) j k
2
1
(b) j k i
3
1
(c) i j k
3
Fig. 24.53 1
(d) i k Fig. 24.54
2
48. A particle of charge q and mass m moves in a
circular orbit of radius r with angular speed . The IIT, 2001
ratio of the magnitude of its magnetic moment to 53. Two particles A and B of masses mA and mB respec-
that of its angular momentum depends on tively and having the same charge are moving in a
(a) and q (b) , q and m plane. A uniform magnetic
(c) q and m (d) and m
this plane. The speeds of
IIT, 2000 the particles are vA and vB
49. Two long parallel wires P and Q are held perpen- respectively and the trajec-
dicular to the plane of the paper at a separation of tories are as shown in Fig.
5 m. If P and Q carry currents of 2.5 A and 5 A re- 24.55. Then
spectively in the same direction, then the magnetic (a) vA < mB vB Fig. 23.55
P and Q is (b) mA vA > mB vB
(c) mA < mB and vA < vB
0 3 0 (d) mA = mB and vA = vB
(a) (b)
IIT, 2003
0 3 0 54. A coil having N turns is wound tightly in the form
(c) (d)
2 2 of a spiral with inner and outer radii a and b respec-
50. A proton of mass m and charge +e is moving in tively. When a current I passes through the coil, the

MeV. What should be the energy of an -particle


0 NI 2 0 NI
(mass 4 m and charge + 2e) so that it revolves in a (a) (b)
b a
circular orbit of the same radius in the same mag-
Magnetic Effect of Current and Magnetism 24.21

0 NI b 0 NI a
(c) ln (d) ln (b) E = a i ; B = c k bi
2 b a a 2 b a b
(c) E = 0; B = c j bk
IIT, 2001
55. A particle of mass m and charge q moves with a (d) E = a i ; B = c k bj
constant velocity v along the positive x direction. It
IIT, 2003
B directed along the negative z direction, extend- 58. A magnetized wire of magnetic moment M is bent
ing from x = a to x = b. The minimum value of v into an arc of a circle that subtends an angle of 60°
required so that the particle can just enter the region at the centre. The equivalent magnetic moment is
x > b is M 2M
(a) qbB/m (b) q(b – a)B/m (a) (b)
(c) qaB/m (d) q(b + a)B/2m
3M 4M
IIT, 2002 (c) (d)
56. Which pattern shown in Fig. 24.56 correctly repre-
59.
are held 0.1 m apart and carry a current of 5 A each
in the same direction. The magnitude of the mag-

is
(a) 10–5 T (b) 2 10–5 T
(c) 3 10–5 T (d) 2 10–5 T
60. A proton moving with a speed u along the positive
x-axis enters at y = 0 a region of uniform magnetic
B = B0 k which exists to the right of y-axis as
shown in Fig. 24.58. The proton leaves the region
after some time with a speed v at co-ordinate y.
Then
(a) v > u, y < 0 (b) v = u, y > 0
(c) v > u, y > 0 (d) v = u, y < 0

Fig. 24.56

57. An electron is moving in the x-y plane along the


positive x-axis. There is a sudden change in its
path due to the presence of electric and/or mag-
P as shown in Fig. 24.57. The
curved path lies in the x-y plane and is found to be
Fig. 24.58
non-circular. Which of the following combinations
is possible? IIT, 2004
61. A particle of charge q moves with a velocity v =
ai B = b j + c k where a, b
and c are constants. The magnitude of the force
experienced by the particle is
(a) zero (b) qa(b + c)
2 2 1/2
(c) qa(b – c ) (d) qa(b2 + c2)1/2
62. A current I -
lateral triangle of side a. The magnitude of the mag-
Fig. 24.57
3 0I 9 0I
(a) (b)
(a) E = 0; B = b i ck 2 a 2 a
24.22 Comprehensive Physics—JEE Advanced

3 3 0I
(c) (d) zero
2 a
63. A particle of mass m and charge q, accelerated by a
potential difference V enters a region of a uniform
B. If d is the thickness of
the region of B, the angle through which the par-
ticle deviates from the initial direction on leaving
the region is given by
1/2
Fig. 24.59
q
(a) sin = Bd 67.
2mV
1/2
q
(b) cos = Bd
2mV (a) (b)
1/2
q
(c) tan = Bd
2mV
q 1/2
(d) cot = Bd
2mV
(c) (d)
64. A metal wire of mass m slides without friction on
two rails spaced at a distance d apart. The track
B. A con-
stant current I Fig. 24.60
and back down the other rail. If the wire is initially
at rest, the time taken by it to move through a IIT, 2011
distance x along the track is 68. A long insulated copper wire is closely wound as
a spiral of ‘N’ turns. The spiral has inner radius ‘a’
BId 2xm
(a) t = (b) t = and outer radius ‘b’. The spiral lies in the XY plane
2 xm BId and a steady current ‘I
BIdm 2dm Z
(c) t = (d) t = the spiral is [see Fig. 24.61]
2x BIx
65. A particle of charge q and mass m is released from (a) 0 NI
ln
b
(b) 0 NI
ln
b a
the origin with a velocity v = a i in a uniform mag- 2 b a a 2 b a b a

B = b k . The particle will cross the y-axis (c) 0 NI


ln
b
(d) 0 NI
ln
b a
at a point whose y-coordinate is 2b a 2b b a
ma 2ma IIT, 2011
(a) y = (b) y =
qb qb
ma 2ma
(c) y = – (d) y = –
qb qb
66. A thin wire loop carrying a current I is placed in a
B pointing out of the plane
of the coil as shown in Fig. 24.59. The loop will
tend to
(a) move towards positive x-direction
(b) move towards negative y-direction
(c) contract
(d) expand
Fig. 24.61
Magnetic Effect of Current and Magnetism 24.23

ANSWERS

1. (d) 2. (a) 3. (c) 4. (c) 5. (c) 6. (d)


7. (c) 8. (a) 9. (d) 10. (c) 11. (c) 12. (c)
13. (b) 14. (d) 15. (c) 16. (a) 17. (d) 18. (c)
19. (a) 20. (c) 21. (c) 22. (d) 23. (d) 24. (c)
25. (d) 26. (a) 27. (b) 28. (c) 29. (d) 30. (d)
31. (d) 32. (c) 33. (c) 34. (c) 35. (a) 36. (a)
37. (d) 38. (a) 39. (c) 40. (b) 41. (c) 42. (c)
43. (b) 44. (b) 45. (a) 46. (a) 47. (c) 48. (c)
49. (c) 50. (a) 51. (a) 52. (d) 53. (b) 54. (c)
55. (b) 56. (d) 57. (b) 58. (c) 59. (c) 60. (d)
61. (d) 62. (b) 63. (a) 64. (b) 65. (d) 66. (c)
67. (c) 68. (a)

SOLUTIONS

B2
1. = magnetic energy density. Therefore
2 0
B2 energy [ML2 T 2 ]
= = 3
= [ML–1 T–2]
0 volume [L ]
RC = time constant = [T]

B 2 RC
= [ML–1 T–2] [T] = [ML–1 T–1], which
0
Fig. 24.62
are the dimensions of viscosity.
2. In Fig. (b), r = L, = 0° and = 45°. Using these
of the wire and is given by
I 6. Figure 24.63 shows the cross-sectional view of the
B= 0 (1) rod. From Fleming left-hand rule, the magnetic
2 R
R = 1 mm = 1 10–3 m, 0 = 4 10–7 Hm–1 and force F acting on the rod is directed to the right
–3
B = 2 10 T. Using these values in Eq. (1), we and is given by F = B I L. Since the rod moves with
get I = 10 A. a constant velocity, no net force acts on it. Hence
the components of Mg and F parallel to the inclined
2 mv cos 30 2 mv 3
3. Pitch x = = (1) plane must balance, i.e.
qB qB 2
mv sin 60° mv
Radius r = = (2)
qB 2qB
x
From Eqs. (1) and (2), we get r = , which is
choice (c). 2 3

4. CD is non-
uniform, wire AB will experience as force as well
as a torque. Hence it will have both translational as Fig. 24.63
well as rotational motions.
5. Refer to the Fig. 24.62. F cos = Mg sin
R in Fig. (a) is given by or B I L cos = Mg sin
I Mg tan
B = 0 (sin + sin ) (1) or B=
4 r IL
24.24 Comprehensive Physics—JEE Advanced

Mg I 0
Putting = 30°, we get B = , which is choice = (cos 45° – cos 90°)
3 IL 4 r
(d).
I 0 0I
7. = (cos 45° – 0) =
4 r 4 2 r
centre is
0I
B=
2r
45
ADB at centre O
is 90
0I L E
B1 = directed out of the page. r O
2r 2
ACB at centre O is
0I 2
B2 = directed into the page
2r 2
O is
Fig. 24.64
B = B2 – B1
O due to current
0I
= ( – ) directed into the page. element DE is the same as that due to AE. Hence,
2 r O due to one side AD is
0I 2 I 2 0I
Putting = 60° = , we get B = . BAD = 0
=
3 3r 4 2 r 4 r
8. The magnitude of magnetic induction due to a Since the centre of the square is equidistant from
circular loop of radius R carrying a current I is the ends A, B, C and D of each side of the square
I and each side produces at the centre O the same
given by B = 0
2R
that due to one side. Hence (because r = L/2)

I 2 0 I 2 2 0 I
B= 0 B = 4BAD = = .
4R r L

The direction of B is normal to the plane of the Hence the correct choice is (d).
loop. Since the current in the bigger loop is clock- 10. Let the masses of X and Y be m1 and m2 and let
wise and that in the smaller loop is anticlockwise their velocities after being accelerated be v 1 and v2
respectively. Since the particles have equal charges
centre C are in opposite directions. Therefore, the and have been accelerated through the same poten-
magnetic induction at the centre is given by tial difference, their kinetic energies are equal, i.e.
1 1
I 1 1 m1v 21 = m2v 22
B = B1 – B2 = 0 2 2
4 R1 R2
B, the radii of the
The wires PQ and S R circular paths are given by
the centre C. Hence the correct choice is (a). m1v12 mv
L = q B v 1 or q B = 1 1
9. Refer to Fig. 24.64. Here r = OE = . Referring to R1 R1
2
O due to m2 v22 mv
and = q B v 2 or q B = 2 2
the current element AE is given by R2 R2
45
0I I 0 45 m1v1 mv m2v2 m22 v22
BAE = – sin d = | cos |90 Therefore = 2 2 or 1 2 1
4 r 90
4 r R1 R2 R1 R22
Magnetic Effect of Current and Magnetism 24.25

But m1v 21 = m2v 22. Therefore, we have


2
m1 R1 helix whose radius is
=
m2 R2 m v sin 9 10 31
3.2 107 sin 30
r= 19
Hence the correct choice is (c). eB 1.6 10 0.1
–4
11. - =9 10 m = 0.9 mm
tre due to each coil is B. Since the planes of the
Hence the correct choice is (c).
coils are at right angles to each other, the directions
16. The correct choice is (a).
2 2 17. The correct choice is (d).
Br = B B
18. The velocity when the potential difference is V is
= 2 B
2 eV
Hence the correct choice is (c). v=
m
12. -
pendicular to the direction of motion of the parti- and force F = e v B
cle, the speed of the particle cannot change but its When the potential difference is doubled, i.e.
velocity changes. Hence the correct choice is (c). V = 2V, the velocity is
13. The radius of the circular orbit is given by 2 eV 2e 2V
v = = = 2 v
2m K m m
r=
qB
Force F = e v B = 2 evB = 2 F. Hence
The charge of an –particle is twice that of a pro- the correct choice is (c).
ton and its mass is four times the mass of a proton. 19. The correct choice is (a).
Therefore m /q is the same for both. Hence r will 20.
the same for both particles. Thus the correct choice produced by the current in the loop and the external
is (b).
14. The cyclotron frequency is given by 21. Referring to Fig. 24.65, the forces acting on arms
qB BC and AD are equal and opposite. The force on
= arm AB is given by
2 m
It is independent of the speed of the particle and Ii
F1 = 0
the radius of its circular path. Now q /m. The 2 a
charge of a proton is half that of an -particle and which is directed towards the wire. The force on
the mass of a proton is one-fourth. Therefore, will arm CD is given by
be doubled. Hence the correct choice is (d).
0I i
15. V = 2.88 10 3 V. The velocity of the electron is F2 =
2 a b
given by
which is directed away from the wire. Since
1
m v2 = e V F1 > F2, the loop will move towards the wire. Hence
2 the correct choice is (c).
1/2
2eV
or v=
m i

19 1/2
2 1.6 10 2.88 103
= I
31
9 10
19 1/2
2 1.6 10 2.88 103
= 31
9 10
= 3.2 10 7 ms–1
with the velocity v,
then v = v sin and v11 = v cos . The electron has
two motions: a linear motion parallel to magnetic Fig. 24.65
24.26 Comprehensive Physics—JEE Advanced

22. Magnetic moment m = AI = r 2I, where r is the V


radius of the circular loop. Now, the circumference 26. Electric intensity E =
d
of the circle = length of the wire, i.e.
where V is the potential difference between the
2 l2 plates and d, the separation between them.
2 r = l or r = 2
4 d = 3 mm = 3 10 –3 m
l2 I l2 I V 600
Therefore, m = r 2I = 2
= , E= = 3
=2 10 5 V m–1
4 4 d 3 10
which is choice (d).
23. Magnetic moment m = IA = I r 2.
m = I1 r 21 = I2 r 22
I1 r2 2 E 2 105
6 B e v = e E or B = = = 0.1 T
= 22 = =4 v 2 106
I2 r1 3
Hence the correct choice is (d). 27. From Ampere’s law, we have
24. An electron moving in a circular orbit is equivalent B dl = 0I
to a current carrying loop. As explained above, the
current is Since no current exists in the medium (air) inside
e the pipe I = 0. Hence B = 0. Hence the correct
I= e = choice is (b).
T
where T is the time period of the motion of the 28. P due to current I1 in
electron around the nucleus. If v is the speed of the conductor AOB is
electron, I
B1 = 0 1
2 r 2 a
T=
v P due to current I2
ev e in conductor COD is
I= ( v =r ) I
2 r 2 B2 = 0 2
2 a
I e 4 rB Since the two conductors are perpendicular to each
0
B= = 0 or = B1 and B2 will be perpendicular to each
2r 4 r 0e
P is
Hence the correct choice is (c).
25. D at wire C is B = (B21 + B22)1/2 = 0
(I21 + I22)1/2
2 a
2I 10 7 2 30
BD = 0
= =2 10–4 T Hence the correct choice is (c).
4 r 0.03
29. The work done is given by
which is directed into the page.
G at C is W= M H sin d = MH |– cos | 0
7
102 20 0
BG = = 0.4 10 –4 T = MH (1 – cos )
0.1
which is directed out of the page. Hence the correct choice is (d).
C is 30. The kinetic energy of proton is
B = BD – BG = 2 10 –4 – 0.4 10–4 K = 2 MeV = 2 106 eV
= 1.6 10 –4 T =2 106 1.6 10–19 J = 3.2 10–13 J
and is directed into the page. 1
mv 2 = 3.2 10–13
The force on 25 cm of wire C is 2
F = B I l sin 90° = 1.6 10–4 10 0.25 Now, mass of proton is m = 1.67 10–27 kg. There-
=4 10 –4 N fore,
Magnetic Effect of Current and Magnetism 24.27

13 m = IA
2 3.2 10
v2 = 27
= 3.83 1014 It is given that the charge q is moving in a cir-
1.67 10
cular path of radius 2l. Therefore, the time period
or v = 1.96 107 ms–1. = 2 (2l)/v. Hence
Now force on proton is qv
m = (2l)2 = qvl
F = evB 2 (2l )
= 1.6 10–19 1.96 107 2.5 The angular momentum L = mv(2l). Therefore,
= 7.84 10–12 N m qvl q
=
Hence the closest choice is (d). L mv(2 l ) 2m
31. The point x = + a lies along the line of the straight 37.
section PQ
at point x = a is zero. point equidistant from the two wires will be equal
32. The radius of the circular path of a particle of mass
m, charge e moving with a speed v perpendicular to is zero. Hence the force on a charge at this point is
B is given by zero.
mv 2 m v 38. E, the force on a particle of
= evB or r = charge q is F = qE in the direction of the electric
r e B
E is parallel to B, the velocity v of the
e particle is parallel to B. Hence B will not affect the
Thus, r is inversely proportional to , the
m motion of the particle since v B = 0. Thus the
charge to mass ratio. Hence the correct choice correct choice is (a).
is (c). 39. E on a particle of
33. The total Lorentz force on the electron is charge q is to impart to it a velocity v which is
F = – e (E + v B) proportional to qE, i.e.
v B. If E v qE qE i
is along + z-direction, the force – e E will be
B on a charge
along – z-direction. If B is along + x direction,
moving with a velocity v is to exert on it a force
force – e (v B) will be along + z direction. When
eE = evB, the electron moves along + y-direction F = q(v B) q (qE i ) Bk

for an electron moving along + y direction, the elec- Thus F q2EB i k q2EB ( j )
z direction and magnetic
x direction, then the electron will be j k j)
Since F q2, both positive and negative ions will
34. Refer to the solution of Q. 15. The correct choice be y direction.
is (c). 40.
35. The radius of the circular path is given by and 2 at a point between them act in opposite direc-
tions. But at a point to the left of wire 1 and to the
mv 2 2m K 1
r= = , where, K = mv 2.
qB qB 2 B
along the line XX is as shown in choice (b).
m
Thus r since K and B are the same for 41. Given v = (2 105 i ) ms–1. The force vector is
q
given by
the three particles. If mp is the mass of a proton
and qp its charge, then md = 2mp and qd = qp and F = q(v B)
m = 4 mp and q = 2qp. From these it follows that
r = rp < rd. = q {2 105 i (i 4 j 3k ) }
36. According to Ampere’s Law, the magnetic moment
=2 105 q ( 4k 3 j)
of a current I
cross-section A is given by Therefore, the y and z components of the force are
24.28 Comprehensive Physics—JEE Advanced

Fy = 6 105 q
0I
5 Now B =
and Fz = –8 10 q 2R
Magnitude of force = Fy2 Fz2 coil of n turns and radius r is
2
0 nI 0 nI 0 In
=q (6 105 ) 2 ( 8 105 ) 2 B = = n 2B
2r 2R / n 2R
= q 10 105 Hence the correct choice is (b).
= 1.6 10–19 10 105 45. Given x = 4 cm = 0.04 m and r = 3 cm = 0.03 m.
= 1.6 10–13 N, which is choice (c).
is given by
42. Given r = 0.4 Å = 0.4 10–10 m, v = 106 ms–1 r2
B= oI (1)
Speed of electron in the orbit is
2(r 2 x 2 )3 / 2
2 r
v= ; here t is time taken by the
t
electron to complete one revolution. Thus oI
B0 = (2)
10 2r
2 r 2 0.4 10
t= = =8 10–17 s Dividing (1) by (2), we get
v 106
B0 r3
19 =
e 1.6 10 2 B (r 2 x 2 )3/2
Current I = = 17
= 10–3 A
t 8 10
Substituting the values of r and x, we get
B0 125
2 or
(4 10 7 ) 10 3
B 27
I
B= 0 = 10
= 10 tesla 125 125
2r 2 0.4 10 B0 = B = 54 T = 250 T
27 27
Hence the correct choice is (c).
Hence the correct choice is (a).
43. Given v = (3i + 2j) ms–1 and B = (2j + 3 k ) tesla. 46. Wires AB and EF experience no forces since cur-
Force experienced by the proton is
F = q(v B) = q (3i + 2j) (2j + 3k) forces on BC and DE are equal in magnitude but are
directed in opposite directions. Hence their resul-
= q (6i j + 9i k + 4j j + 6j k) tant is zero. Only force acting is on CD. Hence the
correct choice is (a).
= q (6k – 9j + 0 + 6i ) 47. Since point O lies along the segments LR and SM,

= 3q (2i – 3j + 2k) newton at point O. As point O is close to R and S, the net


F 3q O due to segments PR and QS is
Acceleration = = (2 i 3 j 2 k )
m m 0I 0I 0I 1 1
B = BP + BQ =
4 RO 4 SO 4 d d
=3 (0.96 108) (2i – 3j + 2k)
2I 2 10
= 2.88 108 (2i – 3j + 2k) ms–2 = 0
= 10–7 = 10–4 Wb m–2
4 d 0.02
Hence the correct choice is (b).
44. Let L be the length of the wire and let R be the
radius of the circle when the wire is bent into one outside the plane of the paper. Hence the correct
circular turn and r be the radius of the coil of n choice is (c).
turns. Then 48. Magnetic moment M = current area
R charge area
L = 2 R = 2 nr or R = nr or r = =
n time period
Magnetic Effect of Current and Magnetism 24.29

q r2 1 2 mAv A rA
= q r2
T 2 T mB vB rB
Angular momentum L = m r2 rA > rB. Hence
1 mA v A > mB v B. Thus the correct choice is (b).
q r2 54. The correct choice is (c). For derivation of the ex-
M 2 q
2
. Hence the correct choice pression, refer to a Textbook of Physics.
L m r 2 m
55. The radius r of the cir-
is (c). cular path is given by
2 I1 2I 2 (see Fig. 24.66)
49. B= 0
· 0
·
4 r1 4 r2 mv 2
= qvB
Given I1 = 2.5 A, I2 = 5 A and r1 = r2 = 2.5 m. Using r
0 qB
these values, we get B = – . The magnitude of B or v= (r)
2 m
is 0/2 . Hence the correct choice is (c). qB
mv vmin= (rmin)
50. For proton: r = m
eB Fig. 24.66
qB
m v 4mv 2mv = (b – a),
For -particle r = m
eB 2eB eB
which is choice (b).
v
Given r = r . Hence v = . 56. The correct choice is (d) because the lines of force
2
are continuous inside the magnet.
1 2
Energy of proton E = mv . Energy of -particle 57. Since the path of the particle beyond P is non-cir-
is 2 cular, both E and B
1 1 v 2
1 P. Hence choices (a) and (c) are incorrect. Since the
2
E = mv = 4m = mv 2 = E curved path lies in the x-y
2 2 2 2
must be in the x-z plane. Hence choice (d) is also
Hence E = 1 MeV which is choice (a). incorrect. Thus, the correct choice is (b).
51. When a current is passed through the helix, the 58. Let r be the radius of the circle. The length of the
neighbouring coils of the helix attract each other 60 r
due to which it contracts. As a result the contact is arc = (2 r) = . Therefore, the length 2l
360 3
broken and the coils will recover their original state of the magnet is
r 6l
is made again and the process continues. Thus the 2l = or r =
wire oscillates. Hence the correct choice is (a). 3
52. If m is the pole strength of each pole of the magnet,
6l
the magnetic moment of the arc = m r = m
in the y-z plane, which is along the x-direction and 3 (2 ml ) 3M
= =
(ii) the other due to the planar loop in the x-y plane,
which is along the z-direction. The direction of the Hence the correct choice is (c).
59. Wires A and B carry current I = 5 A each coming out
( i k ) . Hence the correct choice is (d). of the plane of the page as shown in Fig. 24.66. The
53. The radius r of the circular path of a particle P due wire A
of mass m and charge q moving with velocity v is equal to that due to wire B, i.e.
B is given by 2I
2
BA = BB = 0
mv 4 a
= qvB 7
r 10 2 5
or mv = qrB. Hence mA v B = qrA B and mB v B = =
01
.
q rB B.
24.30 Comprehensive Physics—JEE Advanced

= 10–5 T Magnitude of F = [(q ab)2 + (q ac)2]1/2


BA is perpendicular to PA and = qa(b2 + c2)1/2
BB is perpendicular to PB. Therefore, The correct choice is (d).
= 60°. The 62. Refer to Fig. 24.68. Let AB = BC = AC = a. Let OD = r.
P is given by
BR2 = B 2A + B 2B + 2BA BB cos

which gives BR = 2BA cos


2
3
=2 10–5 = 3 10–5 T
2
Hence the correct choice is (c).

Fig. 24.68

O due to current I
AB of the triangle is given by

0I
BAB = (sin + sin )
4 r
It is clear that = = 60° and
AD a/ 2 a/2 a
OD = r = = = =
tan tan 60 3 2 3

0I 2 3
BAB= (sin 60° + sin 60°)
4 r a
Fig. 24.67 3 0I
=
60. When the proton enters the region of the magnetic 2 a
F given by
F = q (u B) sides BC and AC = that due to side AB. Hence, the
where q is the charge of the proton. The force F O due to the current in the three
is perpendicular to both u and B. Since the force sides of triangle ABC is
is perpendicular to the velocity of the particle, it B = BAB + BBC + BCA = 3BAB
does not do any work. Hence the magnitude of the Hence the correct choice is (b).
velocity of the particle will remain unchanged; only 63. Refer to Fig. 24.69. Let v be the velocity of the
the direction of the velocity changes. Hence v = u. particle. Its kinetic energy is
Since u is perpendicular to B, the proton moves in a
circular path. Since the charge of proton is positive, 1 2 2qV 1 / 2
mv = qV or v = (1)
u is along posotive x-axis and B is directed out of 2 m
the page, the proton will move in a circle in the x-y The particle follows a circular path from A to B of
plane in the clockwise direction. Hence its y coor- radius r which is given by
dinate will be negative, when it leaves the region.
mv 2 mv
Thus the correct choice is (d). = qvB or r = (2)
r qB
61. F = q(v B) = q{a i (bj + c k)}
Using (1) and (2), we have
= q(ab i j + ac i k)
1/2 1/2
m 2qV 1 2mV
= q(ab k – acj) = qa(b k – cj) r=
qB m B q
Magnetic Effect of Current and Magnetism 24.31

B = b in
the positive z-direction,
and the charge of the
particle is positive, the
path of the particle is a
circle as shown in the

circular path is
mv ma
r= =
qB qb
Thus y = – 2
2ma Fig. 24.71
Fig. 24.69 r=–
qb
BD d So the correct choice is (d).
In triangle BCD, sin = = . Therefore, 66. A circular metal loop carries a current. Hence charge,
BC r
say, q moves along the circle with a velocity, say v
q 1/ 2 which is tangential to the circle at every point (Fig.
sin = Bd , which is choice (a).
2mV 24.59 on page 24.22). The force experienced by the
64. Refer to Fig. 24.70. Wire PQ of length d, the spac- charge is F = q(v B). Since v is along the tangent
ing between rails carries a current I vertically and B is directed out of the x – y plane, the direction
of the force is towards the centre O of the loop.
the reader and perpendicular to the length PQ of Hence the force tends to contract the loop. Further,
the wire. Thus angle between I and B is 90°. since F is perpendicular to v, no work is done on
The force exerted on the wire of length d by the the loop. Hence it cannot have any translational
motion. Thus the correct choice is (c).
67.
F = BId sin 90° = BId
Using Fleming’s left hand rule, the direction of the
force is to the left. The acceleration of the wire is
force F BId
a= = =
mass m m Hence the correct choice is (c).
68.
1 2 2x
Now x = at t = . Hence the correct 0I
2 a radius r and carrying a current I = . The direc-
2r
choice is (b). z-direction if the current is
anticlockwise.
Consider a small element of width dr. The current
through the element is
total current in spiral
dI = width of element
total width of spiral
Idr
=
b a
b b
0 NdI 0 NI
dr
B= =
a
2r a
2 b a r
Fig. 24.70
b
0 NIdr 0 NI b
65. Refer to Fig. 24.71. Since the velocity of the particle = = ln
is v = a along the positive x-axis and the direction of 2 b a a r 2 b a a
24.32 Comprehensive Physics—JEE Advanced

II

Multiple Choice Questions with One or More Choice Correct

1. A straight wire carrying current is parallel to the (a) E = 0, B = 0 (b) E = 0, B 0


y–axis as shown in Fig. 24.72. The (c) E 0, B = 0 (d) E 0, B 0
P is parallel to the IIT, 1985
x– axis 6. The force F experienced by a particle of charge q
z–axis moving with a velocity v B is
(c) magnetic lines are concentric circles with the given by F = q (v B). Which pairs of vectors are
wire passing through their common centre at right angles to each other?
(a) F and v (b) F and B
wire are oppositely directed. (c) B and v (d) F and (v B)
7. Choose the correct statements from the following.

in magnitude from point to point but has a


constant direction, is set up in a region of
space. A charged particle enters that region

a constant speed. The initial velocity of the


Fig. 24.72 particle is either along the direction of the
2. -
dal solenoid
(a) is independent of the radius of the solenoid from point to point in magnitude and direc-
(b) depends on the number of turns and the cur- tion is set up in a certain region of space.
rent in the solenoid A charged particle enters the region with a
(c) is constant in magnitude inside the solenoid certain initial velocity. The direction of the
(d) is always radial inside the solenoid.
3. the initial velocity.
(a) is independent of the radius of the solenoid (c) A electron travelling in the positive x–di-
(b) depends on the number of turns and current rection enters a region of space having an
in the solenoid y– direction.
(c) is uniform throughout the solenoid
(d) is perpendicular to the axis of the solenoid. positive z–direction, the electron will travel
4. A charged particle of mass m and charge q enters a
B with a velocity v at an angle with (d) A charged particle moves in a uniform mag-
B.
(a) The kinetic energy of the particle will not of the particle will change during this time.
change if 0. 8. Which of the following statements are correct?
(b) The momentum of the particle will not change (a) A current carrying coil, free to rotate, when
if = 0.
(c) The particle moves in a circle of radius itself such that its plane becomes perpendicu-
mv/qB if = 90°.
(d) The frequency of circular motion is indepen- (b) The trajectory of a charged particle moving
dent of the speed of the particle if = 90°. -
5. A proton moving with a constant velocity passes
through a region of space without any change in
its velocity. If E and B represent the electric and
between two long parallel wires carrying
equal currents in the same direction is zero.
may have
Magnetic Effect of Current and Magnetism 24.33

(d) An electron is moving in the anticlockwise e vr e vr


sense in a horizontal circular orbit. The (a) M = (b) M =
4 2
of the orbit will be vertically upward. 0 ev 0 ev
(c) B = (d) B =
9. - 4 r 2
2 r 2

us r carries a current I. It is placed in a uniform 14.


B. The tension in the loop will be a circular coil of radius r, having n turns and carry-
doubled if ing a current I can be doubled by
(a) I is doubled (a) changing I to 2I, keeping n unchanged
(b) B is doubled (b) changing n to 2n, keeping I unchanged
(c) r is doubled (c) changing I to 2I and n to 2n
(d) Both B and I are doubled.
(d) changing n to n , keeping I unchaged.
10. A particle having a mass of 0.5 g carries a charge 2
of 2.5 10 –8 C. The particle is given an initial 15. The force experienced by a charged particle moving
horizontal velocity of 6 10 4 ms –1. To keep the B with a velocity v is zero if
particle moving in a horizontal direction (a) v = 0
(b) v is parallel to B
the direction of the velocity (c) v is perpendicular to B
- (d) v is antiparallel to B
tion of the velocity 16. -
pends upon
of 3.27 T (a) the number of turns in the solenoid
(b) the current in the solenoid
11. A wire is bent into a circular loop of radius R and (c) the radius of the toroid
carries a current I (d) the permeability of the core of the solenoid.
of the loop is B. The same wire is bent into a double 17. The plates of a parallel plate capacitor are in the
loop. If both loops carry the same current in the y – z plane. The separation between the plates is 3
mm and a potential difference of 600 V is applied
the double loop is B1. If they carry the same current across the plates. An electron is projected between
I the plates with a velocity of 2 106 ms– 1 along
centre is B2. Then the positive y-direction. The electron moves unde-
(a) B1 = 0 (b) B1 = 4B
B 105 V m–1.
(c) B2 = 0 (d) B2 =
4
12. A straight horizontal conducting rod of mass m and region between the plates is 0.1 T.
lenght l is suspended by two vertical wires (of neg-
ligible mass) at its ends. A current I is set up in the the positive z-direction.
B normal to the conductor
is required to keep the tension in the wires equal the negative z
to zero. If the direction of the current is reversed IIT, 1981
B, a tension T is 18. An alpha particle and a deuteron, after being accel-
developed in the wires. Then erated through the same potential difference, enter
mg 2mg -
(a) B = (b) B = dicular to their velocities. If r and rd are the radii
Il Il
of the circular paths of the alpha particle and the
(c) T = BIl + mg (d) T = BIl – mg deuteron respectively and and d their respective
13. In the hydrogen atom the electron (charge e) moves frequency of revolution, then
around the proton with a speed v in a circular orbit (a) r = 2 rd (b) r = rd
of radius r. The magnetic dipole moment of the
circulating electron is M (c) = d (d) = d
the site of the proton (i.e. at the centre of the orbit) 2
is B. Then IIT, 2004
24.34 Comprehensive Physics—JEE Advanced

19. - (b) the magnitude of the angular momentum


rent I in the same direction are in the x-y plane in a about the centre is doubled.
gravity free space (see Fig. 24.73). The central wire (c) the radius r becomes half
is at x = 0 while the other two wires are at x = d. (d) the frequency remains unchanged.
IIT, 1997 22. A long, thin and hollow cylindrical metal pipe of
radius R carries a current I along its length. For
such a pipe,

the pipe

and increases as we go towards the wall.


0I
(c) On the surface of the pipe, B = .
2 R
(d) at a point outside the pipe at distance r from
r 2.
23. A particle of charge q moving with a velocity
v = v0 i B = B0 ( i j k) .
(a) The particle describes a circle in the magnetic

(b) The magnitude of the force acting on the


particle is qv0B0
(c) The magnitude of the force on the particle is
Fig. 24.73
2 qv0B0.
(d) The force vector lies in the y-z plane.
(a) cannot be zero beyond B (i.e. x > d) 24. A thin rod AB of length l carries a current I1. It is
(b) cannot be zero beyond A (i.e. – x > – d) PQ car-
(c) will be zero between x = 0 and x = d. rying a current I2 as shown in Fig. 24.75.
(d) will be zero between x = 0 and x = – d.
20. Two charged particles 1 and 2 of masses m1 and
m2, charges q1 and q2 enter a uniform magnetic
v1 and v2
shown in Fig. 24.74.

Fig. 24.75

(a) The force experienced by the rod is F =

Fig. 24.74 0 I1 I 2 l
loge 1
4 2a
(a) If q1 = q2, m1v1 > m2v2
(b) If q1 = q2, m1v1 < m2v2 (b) The force experienced by the rod is F =
0 I1 I 2 l
m1 m2 loge 1
(c) If v1 = v2, > 4 a
q1 q2
(c) The rod experiences no torque
(d) If m1 = m2, v1q1 < v2q2. (d) The rod experiences a force as well as a
21. A charged particle is accelerated through a poten- torque.
tial difference V. It then enters a region of uniform 25. Choose the correct statements from the following.
r and
its frequency of revolution is . If V is doubled. is [ML0 T–2 A–1]
(a) the kinetic energy is doubled (b) 0 is dimensionless.
Magnetic Effect of Current and Magnetism 24.35

(d) The proton follows track D and electron fol-


(c) The dimensions of 0 0 are the same as lows track A.
those of speed. E
(d) The dimensions of are the same as those IIT, 1984
of speed. B 28. A particle of charge + q and mass m moving under
26. An annular wire loop ABCD carries a current I1 E i and
as shown in Fig. 24.76. O is the common cen- B k follows a trajectory
tre of the curved parts AB and CD of the loop. A from P to Q as shown in Fig. 24.78. The velocity at
straight wire passing through O and perpendicular
P is v i and at Q is – 2v j . Which of the following
to the plane of the loop carries a current I2 directed
statements is/are correct ?
towards the reader. Then

Fig. 24.76
Fig. 24.78
(a) the net force on the loop is zero. 2
(b) the net torque on the loop is zero. 3mv
(a) E =
(c) As seen from O the loop will rotate in clock- 4qa
wise sense about axis OP.
P
(d) As seen from O the loop will rotate in anti- 3
clockwise sense about axis OP. 3mv
is .
IIT, 2006 4a
27. A neutron, a proton, an electron and an alpha par- P
is zero.

inward normal to the plane of the paper. The tracks Q is zero.


of the particles are labeled as shown in Fig. 24.77. IIT, 1991
(a) The electron follows track D and alpha par- 29. A particle of mass m and charge q moving with
ticle follows track B velocity v enters Region II normal to the boundary
(b) The proton follows track A and alpha particle as shown in Fig. 24.79. Region II has a uniform
follows track B B perpendicular to the plane of the
paper. The length of the Region II is l. Choose the
correct choice(s)

Fig. 24.79
(a) The particle entres Region III only if its
qlB
velocity v >
m
Fig. 24.77 (b) The particle enters Region III only if its
qlB
(c) The electron follows track A and neutron velocity v <
follows tracks C m
24.36 Comprehensive Physics—JEE Advanced

(c) Path length of the particle in Region II is (b) O++


qlB (c) He+ and O++
maximum when velocity v =
m
(d) Time spent is Region II is the same for any IIT, 1994
velocity v as long as the particle returns to 31. A proton and an electron moving with the same
Region I velocity v
B which is perpendicular to their velocity. If rp and
IIT, 2008
re are the radii of their circular trajectories and tp
30. H+, He+, and O++ all having the same kinetic energy
and te the time after which each particle comes out
pass through a region in which there is a uniform

masses of H+, He+ and O++ are lu, 4u and 16u re- (a) rq < re (b) rp > re
spectively. Then (c) tp < te (d) tp > te
(a) H+ IIT, 2011

ANSWERS AND SOLUTIONS


1. 7. Statement (a) is correct. Since the direction of
to the plane containing the point P and the current the velocity of the particle remains unchanged,
element I, which is the z-direction. Hence choice (a) no magnetic force acts on the particle. The force
experienced by a particle of charge q moving with
the left and to the right of the current element are a velocity v B is
oppositely directed. Hence choice (d) is correct. F = q (v B)
Since F = 0, (v B) = 0, i.e. v and B are parallel to each
concentric circles with their common centre at the other. Thus, the initial velocity of the particle is either
wire. Hence choice (c) is also correct.
2. The correct choices are (a), (b) and (c). The mag- Statement (b) is also correct. Since the magnetic
force is always perpendicular to particle velocity,
3. it cannot change the magnitude of the velocity
uniform at the edges of the solenoid and is parallel (i.e. speed); it can only change the direction of
to the axis.
4. is equal to its initial speed. The direction of the
perpendicular to the velocity of the particle; the
speed of the particle cannot change; only the initial velocity.
direction of motion, i.e. velocity (and hence mo-
mentum) will change. The correct choices are (a), y-direction, the electron
(c) and (d). y-direction. It will
5. The force on a charge q moving with a velocity v is
given by y-direction. Since
F = q (E + v B) the magnetic force is perpendicular to the magnetic
There will be no change in velocity if F = 0. This
can happen in three cases. (i) Both E and B are zero
which is choice (a). (ii) E = 0 and v and B are paral- the negative z-direction (use Fleming’s left-hand
lel so that v B = 0 in which case B 0 which is rule).
choice (b). (iii) The electric force qE is equal and Statement (d) is also incorrect. No work is done
opposite to magnetic force q (v B) in which case
the net force is zero which is choice (d). Hence the Hence its kinetic energy remains constant.
correct choices are (a), (b) and (d). 8. Statement (a) is correct. The loop will rotate until
6. The pairs F and v and F and B are always at right equilibrium state is attained. It will then come to
angles to each other, because F is always perpen- rest because the torque acting on it becomes zero.
dicular to the plane containing B and v. Vectors B We know that the torque is given by
and v may have any angle between them. = BIA sin
Magnetic Effect of Current and Magnetism 24.37

where is the angle between the direction of mag- Since the currents in the two loops are in opposite
B and the normal to the plane of the B1 and B2 are equal and opposite.
loop. It is clear that = 0 when = 0. Thus, in the
equilibrium state, the orientation of the loop is such the double loop = B1 – B2 = 0. Hence the correct
choices are (b) and (c).
of the loop. 12. In order that the tension in the supporting wires is
Statement (b) is incorrect. If v is parallel to B, the zero the downward gravitational force mg on the
particle does not experience any force. Hence its rod must be balanced by an upward force BIl due
trajectory will be a straight line. Statement (c) is BIl = mg
correct. Since currents are equal and in the same mg
or B=
Il
point midway between the wires will be equal and If the current is reversed, the direction of the force
opposite and hence they will cancel each other. due to B becomes downwards, in the direction of the
Statement (d) is incorrect. The direction of the gravitational force. Hence the tension in the string is
T = BIl + mg
hand rule, will be vertically downward. Note that The correct choices are (a) and (c).
the charge of an electron is negative. 2 r e ev
9. The force acting on the loop is given by 13. Time period T = . Current I = = .
v T 2 r
F = m B sin
where m = r2I. Force will be doubled if I or B are Magnetic moment M = current area of orbit
ev e vr
doubled. Hence the correct choices are (a) and (b). =I r2 = r2
10. 2 r 2
experience gravitational force mg. As a result the
particle will not continue moving in the horizon- 0I 0 ev
tal direction but will describe a parabolic path. So B= = 2
2r 4 r
So the correct choices are (b) and (c).
must be perpendicular to the direction of the veloc-
14.
ity. The magnetic force experienced by the particle
the coil is given by
is given by
F = q (v B) 0 nI
B=
The magnitude of the force is F = q v B sin . If the 2r
particles is to move in the horizontal direction, this Hence the correct choices are (a) and (b).
force must balance the force of gravity, i.e. 15. F = q (v B). The magnitude of the force is
mg = q v b sin F = q v B sin
The minimum value of B corresponds to sin = 1 Thus F = 0 if v = 0 or = 0° or 180°. Hence the
or = 90°. Thus correct choices are (a), (b) and (d).
mg = v B 16. The correct statements are (a), (b) and (d). The
mg 0.5 10 3 9.8
or B = = 3.27 T B = μnI
qv 2.5 10 8 6 10 4
where is the permeability of the core of the sole-
Hence the correct choices are (a) and (c).
noid. B is independent of the radius of the toroid.
11. The radius of the double loop r = R/2. Now
17.
I
B= 0 V 600
2R plates is E = = 3
=2 105 V m–1.
r at the centre d 3 10
of the loop is
I I
B1 = 0 = 0 = 2 B ( r = 1/2)
2r 2R
Similarly for the second loop of the double loop,
B2 = 2 B
Since the currents in the two loops are in the same
B1
+ B2 = 4B. Fig. 24.80
24.38 Comprehensive Physics—JEE Advanced

The electron will experience a force qE in the nega- For Points on the x-axis beyond A: The directions of
tive x -
ed in the region between the plates, the magnetic III are along the positive z-axis. Hence the net mag-
force experienced by it must be along the positive B cannot be zero beyond A, (i.e. – x > – d).
x-direction and its magnitude must be equal to qE. For Points between O and B: The directions of the
Since magnetic force = q(v B), it follows that the
v (as well as E) and along the negative z-axis but the direction of the
directed along the negative z-direction. The magni-
tude of the magnetic force =qvB sin 90° = qvB. the positive z
Thus B will be zero for some value of x lying between
E 2 10 5 x = 0 and x = d.
qvB = qE or B = = = 0.1 T For Points between O and A: The directions of the
v 2 10 6
So the correct choices are (a), (b) and (d).
18. If a particle of mass m and charge q is accelerated along the positive z-axis but that due to current in
through a potential difference V, it acquires kinetic wire III is along the negative z-axis. Hence the net
energy = qV. Hence B will be zero for some value of x
1 lying between x = 0 and x = – d.
mv2 = qV So all the four choices are correct.
2
20. The radius r of the circular path of a particle of mass
2qV m and charge q moving with a velocity v perpen-
or v=
m B is given by
where v is the velocity acquired by the particle. The mv = qvB
2
radius of the circular path is given by mv = qrB (1)
r
mv m 2qV 2V q mv r
r= = = If q1 = q2, then 1 1 = 1
qB qB m B m m2 v2 r2
It follows from Fig. 24.66 that r1 > r2. Hence m1v1
q > m 2v 2.
For given V and B, r . Hence
m If v1 = v2, then from Eq. (1), we have
r q md
= (1) m1 qr
rd qd m = 11
m2 q2 r2
Now, alpha particle has 2 protons and 2 neutrons
and a deuteron has 1 proton and 1 neutron. There- r1 m m
Since > 1, m1q2 > m2q1 or 1 > 2 . If m1 = m2,
fore, m = 4 mp, md = 2 mp, q = 2 qd and qd = qp. r2 q1 q2
Using these in (1), we get v1q2 > v2q1. So the
correct choices are (a) and (c).
r 2q p 2m p
= =1 21. Kinetic energy = qV. Hence if V is doubled, K.E.
rd qp 4m p is also doubled. The magnitude of the angular
momentum about the centre is L = mvr. Now r =
The frequency of revolution is given by
mv/qB.
qB q
= Therefore,
2 m m
m2 v 2 2m 1 2 2m
L= = mv = (K.E.)
Hence =1 qB qB 2 qB
d Thus if V is doubled, K.E. is doubled and L is also
Thus the correct choices are (b) and (d). doubled. The radius of the circular path is given
19. By using the right hand rule, the following conclu- by
sions can be drawn: m2 v 2 2m
For Points on the x-axis beyond B: The directions r2 = 2 2 = 2 2 1 2 = 2m qV
mv
q B q B 2 q2 B 2
III are along the negative z-axis. Hence the net mag- 2mV
B cannot be zero beyond B (i.e. x > d). =
qB 2
Magnetic Effect of Current and Magnetism 24.39

24. Consider a small element of length dx of the rod at


Thus r V . The frequency of revolution is
a distance x from wire PQ as shown in Fig. 24.82.
qB PQ at the element
=
2 m is
0 I2
Hence the correct choices are (a), (b) and (d). B=
2 x
22. Consider a point P at a distance r from the axis of
Therefore, force exerted on the element is
the pipe. For all points inside the pipe, r < R
P, we choose an Amperean
loop to be a cylindrical loop of radius R as shown
in Fig. 24.81. Since the current enclosed in the Am-
perean surface is zero, it follows from Ampere’s

points inside the pipe. For points outside the pipe,


r > R, we have from Ampere’s law,

Fig. 24.82

0 I1 I 2 dx
dF = BI1 dx =
2 x
Total force experienced by the rod is

Fig. 24.81 F= dF
x (a l )
B .dl = 0I = 0 I1 I 2 dx
2 x a
x
B dl = 0I (B is || dl)
=
a l 0 I1 I 2
B 2 r= log e
0I 2 a
0I 1 B is not uniform; it
B= , i.e. B decreases as we go from and A to end B of the rod.
2 r r
Hence the rod will also experience a torque. Thus
At the surface of the pipe (r = R)
the correct choices are (b) and (d).
0I 25. From F = qvB, it is easy to see that the dimension-
B= al formula of B is [ML0 T–2 A–1], which is choice
2 R
(a). Choice (b) is wrong. From = 0/(2 ), the
Hence the correct choices are (a) and (c).
dimensions of 0 are
23. Since v is not perpendicular to B, the particle
will not describe a circle. The force acting on the B r
[ 0] =
particle is I
F = q(v B) ML0 T 2 A 1
L
= = [MLT–2 A–2]
A
= q[v0 i B0 i j k ]
1 q1q2
From F = , the dimensions of 0 are
= qv0B0 i i i j i k 4 0 r2
q1q2 A2T2
= qv0B0 k j [ 0] = 2
= 2 2
= M–1 L–3 T4 A2
F r MLT L
Thus the magnitude of the force is [ –2 –2
[M L T A ] = [L–2 T2]
–1 –3 4 2
0 0] = [MLT A ]
F = qv0B0 = qv0B0(12 + 12)1/2 = 2 qv0B0
0 0
= [L–1 T], which is reciprocal to speed.
and it lies in the y-z plane. Hence the correct choic-
es are (c) and (d). Hence choice (c) is incorrect.
24.40 Comprehensive Physics—JEE Advanced

E mv
From qE = qvB = v. Hence choice r=
(d) is correct. B qB
26. I2 is tangential Therefore, the particle can enter region III if r > l,
to the curved parts AB and CD of the loop. Hence qBl
i.e. if v > .
every current element dl of parts AB and CD is par- m
allel or antiparallel to B. The magnetic force on AB
In region II, the maximum path length is r = l,
or CD is zero since = 0° or 180° in the expression
dF = Bldl sin . The magnetic force on straight parts qBl
which gives v =
AD and BC is not zero. The magnetic force on AD m
is directed towards the reader which is equal and The time period of the circular motion is
opposite to the force on BC which is directed away 2 r 2 mv 2 m
from the reader. These equal and opposite forces T=
v v qB qB
cancel each other. Therefore, the net force on the
The particle will return to region I if the time spent
loop ABCD is zero. Since these equal and opposite
T m
forces do not act at the same point, they will exert by it in region II is , which is independent
a net torque on the loop which will rotate it in the 2 qB
clockwise sense when viewed from O. Hence the of the velocity. Hence the correct choices are (a),
correct choices are (a) and (c). (c) and (d).
27. The radius of the circular track is given by 1 1 1
30. mHv2H = mHev2He = mOv2O. Given mH = 1u,
mv 2 2 2
r=
qB 1
mHe = 4u and mO = 16u. Hence vHe = vH and
For a neutron q = 0. For an electron q = –e and for 2
a proton q = +e. For an alpha particle q = 2e and 1
m = mass of four protons. Hence the correct choices v O = v H.
4
are (a) and (b).
The radius of the circular path is given by
28. In going from P to Q, the change in the kinetic
1 1 3 mv
energy of the particle = m(2v)2 – mv2 = R= ; q = charge
2 2 2 qB
mv2 - Now charge of H+ = e, charge of He+ = e and charge
of O++ = 2e. It is easy to check that rHe = rO and
ing the particle from P to Q = F·d where d is the 1
displacement in the direction of E. So work done rH = rHe
2
W = qE i 2a i 2qaE i i 2qaE . Now work done Smaller the value of r
= change in K.E. Hence Thus the correct choices are (a) and (c).
31.
3 2 3mv 2 the electron and proton in the region of magnetic
2qaE = mv E
2 4qa
Thus choice (a) is correct. mv
Since r = , rp > re
P is qB
3mv 2
F vp qE i v i qE v q v
4qa

3mv3
4a
So choice (b) is also correct.
Q is zero Fig. 24.83
because i j 0 and i k 0 . Hence the correct Since v B, the charged particle describes a semi-
choices are (a), (b) and (d). circular trajectory. Hence the time after which
29. In region II, the particle follows a circular path of
radius period, i.e.
Magnetic Effect of Current and Magnetism 24.41

T m Since mp > me and q and B are the same, tp > te.


t= Hence the correct choices are (b) and (d).
2 qB

III

Multiple Choice Questions based on Passage


Questions 1 to 3 are based on the following passage An electron moving with a velocity of 4 105 m/s along
Passage I the positive x-direction experiences a force of magnitude
3.2 10–20 N at the point R.
Two long parallel wires carrying currents 2.5 amperes
and I ampere in the same direction (directed into the IIT, 1990
plane of the paper) are held at P and Q respectively such 1. R is
that they are perpendicular to the plane of the paper. The (a) 2.5 10–7 T (b) 5.0 10–7 T
points P and Q are located at a distance of 5 m and 2 m, (c) 5.0 10–6 T (d) 2.5 10–6 T
respectively, from a collinear point R (see Fig. 24.84). 2. R due to
current I = 2.5 A in wire P is
(a) 1 10–7 T (b) 2 10–7 T
–7
(b) 3 10 T (d) 4 10–7 T
3. The current I in wire Q is
Fig. 24.84
(a) 1 A (b) 2 A
(c) 3 A (d) 4 A

SOLUTION
1. The magnitude of the force experienced by a 3. R due to currect I in
particle of charge q moving with a velocity v in a wire Q is
7
0I 4 10 I
F = qvB sin B2 = = =I 10–7 T
2 r2 2 2
where is the angle between v and B. Given F = 3.2
10–20 N, v = 4 105 ms–1 and = 90°. For electron B1 and B2 will be in the downwar
q = 1.6 10–19 C. Using these value we get direction, parallel and colliner. Hence the resultant
B = 5 10–7 T, which is choice (b) R is
2. R due to currect I in B = B1 + B2 = (1 + I) 10–7 T
wire P is Now B = 5 10–7 T. Therefore
I 7
(1 + I) 10–7 = 5 10–7
B1 = 0 = 4 10 2.5 = 1 10–7 T
2 r1 2 5 or 1 + I = 5 or I = 4 A.
The correct choice is (a). So the correct choice is (d).

Questions 4 to 7 are based on the following passage 4. The force experienced by the charged particle in the

Passage II (a) along the positive y-direction


The region between x = 0 and x = L (b) along the negative y-direction
(c) in the x-y plane
B0 k . A particle of mass m, positive (d) in the y-z plane.
charge and velocity v0 i travels along x-axis and enters 5. If the particle emerges from the region of magnetic

initial velocity, the value of L is


IIT, 1999
24.42 Comprehensive Physics—JEE Advanced

2mv0 mv0
(a) (b) (c) v0 j (d) – v0 j
qB0 qB0 7. In Q. 6, the time spent by the particle in the mag-
mv0 3mv0
(c) (d)
2qB0 2qB0 2 m 2 m
(a) t = (b) t =
6. x = 2.1 L, qB0 qB0
3 m m
(c) t = (d) t =
2qB0 qB0
(a) v0 i (b) – v0 i

SOLUTION
4. The force experienced by the charged particle is mv0
given by r= (2)
qB0
F = q(v B) = (v0 i ) (B0 k ) Since the particle emerges from the region of the

= qv0B0( i k) angle of 30° with the initial vector, it follows from


triangle ABC that
= qv0B0(– j ) (1)
AB = AC sin 30°
The force is along the negative y-direction, which or L = r sin 30°
is choice (b).
m v0 sin 30 m v0
5. Refer to Fig. 24.85. = = (3)
qB0 2qB0
Thus the correct choice is (c).
6.

will continue to move in a circular path till it com-


pletes half the circular path and emerges out of the
v0 i
moving along the negative x-axis as shown in Fig.
24.39.
7. Distance travelled by the particle in the magnetic
. Therefore, time

r m
Fig. 24.85
t= = [Use Eq. (2)]
v0 qB0
The particle describes a circle of radius So the correct choice is (d).

Questions 8 to 11 are based on the following passage 9. B2 at O due to the circular arc
Passage III ACB is

A wire loop consists of a straight segment AB and a circular (a) 5 0 I


arc ACB of radius r. The segment AB subtends an angle 12r
of 60° at the centre O of the circular arc. The wire loop (b) 0I
carries a current I in the clockwise direction (Fig. 24.86). 2r
8. B1 at O due to the straight
segment AB is (c) 3 0I
8r
0I 0I
(a)
2 r
(b) (d) 7 0I
2 2 r
18r Fig. 24.86
0I 0I 10. B at O due to the whole wire
(c) (d)
2 3 r 4 r loop is
Magnetic Effect of Current and Magnetism 24.43

(a) B = B1 + B2 (b) B = B2 – B1 (b) perpendicular to the plane of the coil and


directed out of the page
(c) B = B12 B22(d) B = B22 B12 (c) perpendicular to the plane of the coil and
11. B is directed into the page.
(a) parallel to the plane of the coil (d) inclined at an angle of 60° with the plane of
the coil.
SOLUTION
8. O due 9.
to current I in AB is given by (use Biot-Savart law) (n = 1 turn) circular loop of radius r and carrying
a current I is
0 nI
B=
2r
Here loop ACB is a fraction of a circle i.e. n < 1.
Since ACB subtends an angle (360° – 60°) = 300°
at O, hence the fraction n is
Fig. 24.87
300 5
0I
n= =
BAB = (sin + sin ) 360 6
4 x
3r ACB is
Here = = 30°. Also x = r cos = r cos 30° = .
2 5
Therefore, I
6 0 5 0I
B2 = =
0I 2r 12r
B1 = (sin 30 sin 30 )
4 3r/ 2 As the current in ACB is clockwise, the direction of
0I 0I
= (0.5 + 0.5) = , the paper and directed into the page.
2 3 r 2 3 r The correct choice is (a).
which is choice (c). 10. Since B1 and B2 are in the same direction, the net
B = (B1 + B2), which is choice (a).
plane of the paper directed into the page. 11. The correct choice is (c).

Questions 12 to 15 are based on the following passage (b) N and A are increased and B and C are
Passage IV decreased
(c) N, B and C are increased and A is
A moving coil galvanometer consists of a coil of N turns decreased
and area suspended by a thin phosphor bronze strip in (d) N, A, B and C are all increased.
B. The moment of inertia of the 14. When a charge is passed almost instantly through
coil about the axis of rotation is I and C is the torsional the coil, the angular speed acquired by the coil
constant of the phosphor bronze strip. When a current is is
NAB BAQ
(in radian). (a) (b)
QI NI
IIT, 2004
12. Choose the correct statement from the following. NABQ NAQI
(c) (d)
The magnitude of the torque experienced by the I B
coil is independent of 15.
(a) N (b) B radian) of the coil is
(c) i (d) I I 1
13. The current sensitivity of the galvanometer is (a) max = (b) max = I
C C
increased if
(a) N, A and B are increased and C is de- (c) = I (d) = IC
max max
creased. C
24.44 Comprehensive Physics—JEE Advanced

SOLUTION
12. The magnitude of torque experienced by the coil is 14. If is the angular speed acquired by the coil when
given by a charge is passed through it for very short time t,
= iNAB sin then
where is the angle which the normal to the plane angular momentum
of the coil makes with the direction of the magnetic = = I
timeinterval t
coil is always parallel to the direction of the magnet- Q
or = t = = KQ i
= 90°. Hence = iNAB = Ki = where t
K = NAB NABQ
So the correct choice is (d). or I = NABQ or = , which is choice
I
13. Let (c).
current i is passed through the coil. Then, restor- 15. From the principle of conservation of energy, we
ing torque = C . When the coil is in equilibrium, have
1 2 1 2
iNAB = = max
2 2
NAB
Current sensitivity is = I
i C which gives max = , which is choice (a)
C
Hence the correct choice is (a).

Questions 16 to 18 are based on the following passage 18. The distance EF is


Passage V 2
(a) 2m (b) m
5
A particle of mass 1.6 10–27 kg and charge 1.6 10–19 C
2 1
E (c) m (d) m
along the direction shown in Fig. 24.88. The speed of the 10 2
particle is 107 ms–1
the inward normal to the plane of the paper. The particle
F.
IIT, 1984
16. The value of angle is
(a) 30° (b) 45°
(c) 60° (d) 75°
17. The radius of the circular path of the particle in the

(a) 0.1 m (b) 0.2 m Fig. 24.88


(c) 0.3 m (d) 0.4 m
SOLUTION

16. Let O be the centre of the circular path. It is ob-


F
such that BF is tangent to the circle. OE and OF
are normals, which meet at O (see Fig. 24.89).
Therefore, angle OFE = angle OEF. Hence = 45°.
mv
17. r = . Substituting the given values and solving
qB
we get r = 0.1 m
2
18. EF = 2 r cos 45° = 2 r = 2 0.1 m Fig. 24.89
10
Magnetic Effect of Current and Magnetism 24.45

IV

Matrix Matching Type


1. In Column I are listed some charged bodies and current carrying conductors. Match them with the effects they
produce listed in column II
Column I Column II

with a constant angular velocity


(c) A coil carrying a current I = I0 sin t (r) Magnetic moment

IIT, 2006

ANSWER
(a) (p) (b) (p), (q), (r)
(c) (p), (r), (s) (d) (q)
2. Two wires each carrying a steady current I Column I. Some of the resulting
effects are described in Column II. Match the statemens in Column I with those in Column II.
Column I Column II
(a) Point P B) at P due to the currents
in the wires are in the same direction.

(b) Point P B) at P due to the current


joining the centers of the circular wires which in the wires are in opposite directions.
have same radii.

(c) Point P P.
joining the centers of the circular wires, which
have same radii

(d) Point P is situated at the common centre (s) The wires repel each other.
of the wires.

IIT, 2007
24.46 Comprehensive Physics—JEE Advanced

SOLUTION
(a) At point P
the currents are equal but in opposite directions. Hence the correct choices are (q) and (r).
(b) The currents in the two coils are in the same sense (anticlockwise). Hence, at point, P
the current are the same direction, which is choice (p).
P due to the currents
are in the opposite directions. Since point P is the same distance from the centres of the coils, and their radii are

point P is zero. Hence the correct choices are (q) and (r).
P of the coils are not equal (because their radii are different). Further,

direction. If the currents are in opposite directions in coils (or wires), they repel each other. Hence the correct
choices are (q) and (s).
(a) (q), (r) (b) (p)
(c) (q), (r) (d) (q), (s)
3. Six point charges, each of the same magnitude q, are arranged in different manners as shown in
Column II. In each case, a point M and a line PQ passing through M are shown. Let E V
be the electric potential at M
Now, the whole system is set into rotation with a constant angular velocity about the line PQ. Let B the magnetic
M and μ be the magnetic moment of the system in this condition. Assume each rotating charge to be
equivalent to a steady current.
IIT, 2007
Column I Column II

(a) E = 0 (p) (p) Charges are at the corners of a regular hexa-


gon. M is at the centre of the hexagon. PQ is
perpendicular to the plane of the hexagon.

(b) V 0 (q) (q) Charges are on a line perpendicular to PQ at


equal intervals. M is the mid-point between the
two innermost charges.

(c) B = 0 (r) (r) Charges are placed on two coplanar insulating


rings at equal intervals. M is the common
centre of the rings. PQ is perpendicular to the
plane of the rings.

(d) μ 0 (s) (s) Charges are placed at the corners of a rectangle


of sides a and 2a and at the mid point of the
longer sides. M is at the centre of the rectangle.
PQ is parralled to the longer sides.

(t) (t) Charges are placed on two coplanar, identical


insulating rings at equal intervals. M is the
mid-point between the centres of the rings. PQ
is perpendicular to the line joining the centres
coplanar to
Magnetic Effect of Current and Magnetism 24.47

SOLUTION

2q
(p) M due to charge at A and B = 2
where r = AM = BM directed from M to B. The electric
4 0r
M due to other charges are
net V = 0. Since the total charge of the system is zero, on rotation net current = 0. Hence B= 0 and μ = 0

(q) The net charge on the right of M is 2q – q = q and on the left of M is –q M


and directed to the left of M. But net V = 0. Net charge = 0. On rotation, current = 0. Hence B = 0 and μ = 0.

M V 0. On rotating, net current is


not zero. Hence B 0 and μ 0.
E = 0. But net V 0. On rotating, net current 0. Hence B 0 and μ 0.
E 0 and is directed to the right but net V = 0. On rotation, net
current = 0. Hence B = 0 and μ = 0.

ANSWERS

(a) (p), (r), (s) (b) (r), (s)


(c) (p), (q), (t) (d) (r), (s)
24.48 Comprehensive Physics—JEE Advanced

Assertion-Reason Type Questions

In the following questions, Statement-1 (Assertion) is


followed by statement-2 (Reason). Each question has the
following four options out of which only one choice is Statement 2
correct. If a charged particle moves in a direction perpen-
(a) Statement-1 is True, Statement-2 is True
and State-ment-2 is a correct explanation for acting on it is given by Fleming’s left-hand rule.
Statement-1. 5. Statement 1
(b) Statement-1 is True, Statement-2 is True; but If a charged particle is released from rest in a region
Statement-2 is NOT a correct explanation
for Statement-1. each other, it will move in a straight line.
(c) Statement-1 is True, Statement-2 is False. Statement 2
(d) Statement-1 is False, Statement-2 is True.
1. Statement 1
- 6. Statement 1
nitude from point to point but has a constant direc- A proton and an alpha particle having the same
tion, is set up in a region of space. If a charged kinetic energy are moving in circular paths in a
particle enters the region in the direction of the
paths will be equal.
uniform rate in the region. Statement 2
Statement 2 Any two charged particles having equal kinetic
The force F experienced by a particle of charge energies and entering a region of uniform magnet-
moving with a velocity v B is ic B in a direction perpendicular to B , will
given by F = ( v B ). describe circular trajectories of equal radii.
2. Statement 1 7. Statement 1
A charged particle moves in a uniform magnetic Two particles having equal charges and masses
m1 and m2, after being accelerated by the same
energy of the particle cannot change but its momen- potential difference (V), enter a region of uniform
tum can change.
Statement 2 r1 and r2 respectively. Then
The magnetic force is always perpendicular to the
m1 r1
velocity of the particle. =
IIT, 1993 m2 r2
3. Statement 2 Statement 2
A current carrying loop is free to rotate. It is placed Gain in kinetic energy = work done to accelerate
the charged particle through potential difference .
8. Statement-1
Statement 2 No net force acts on a rectangular coil carrying a
The torque on the coil is zero when its plane is steady current when a suspended freely in a uni-

4. Statement 1 Statement-2
An electron moving in the positive -direction The magnitude of force experienced by a straight
enters a region where uniform electric and magnetic conductor of length L carrying a current I and placed
B is BIL.
IIT, 1981
Magnetic Effect of Current and Magnetism 24.49

9. Statement-I mv
There is no change in the energy of a charged parti- r=
qB
IIT, 1985
force is acting on it. 12. Statement-1
Statement-2 -
The magnetic force acting on a moving charged tude from point to point but has a constant direction,
particle is always perpendicular to its velocity. is set up in a region of space. If a charged particle
IIT, 1983 enters the region in the direction of the magnetic
10. Statement-I
A charged particle enters a region of uniform mag- the region
Statement-2
path of the particle is a circle. The force F experienced by a particle of charged
Statement-2 q moving with a velocity v B
- is given by F = q( v B )
allel and equidistant. IIT, 1989
IIT, 1983 13. Statement-I
11. Statement-I The sensitivity of a moving coil galvanometer is
An electron and a proton are moving with the same increased by placing a suitable magnetic material
kinetic energy along the same direction. When as a core inside the coil.
Statement-2
to their direction of motion, they describe circular Soft iron has a high magnetic permeability and can-
path of the same radius. not be easily magnetized or demagnetized.
Statement-2 IIT, 2008
The radius of the circular pathof a particle of charge
q, mass m and moving with velocity v perpendicu-
B is given by

SOLUTIONS
1. The correct choice is (d). If v is parallel to B , F = plane of the coil is perpendicular to the magnetic
0. Hence the particle does not experience any force
and is, therefore, not accelerated in the region. It 4. The correct choice is (a). Because electron has a

2. The correct choice is (a). Since the magnetic force


is always perpendicular to the velocity, no work

particle. Hence magnetic force cannot change the the magnetic force is perpendicular to the magnetic
magnitude of velocity (i.e. speed); it can only
change the direction of velocity. Hence kinetic en- -
1 ing’s left-hand rule) should be along the negative
ergy mv 2 remains unchanged but momentum -direction.
2
p = v will change. 5.
force is F = E in the direction of E . Since E
3. The correct choice is (a). The loop will rotate and is parallel to B , the particle velocity v (acquired
come to rest when the torque acting on it becomes due to force F ) is parallel to B . Hence B will not
zero. The magnitude to torque acting on a loop of exert any force since v B = 0 and the motion of
area A and carrying a current I the particle is not affected by B .
B is given by 6. The correct choice is (c). The radius of the circular
= BIA sin path is given by
where is the angle between the direction of the mv 2mK 1
= = ; where = v2
qB qB 2
coil. It is clear that = 0 when = 0, i.e. when the
24.50 Comprehensive Physics—JEE Advanced

These forces do not exert any net force on the coil.


m
Since and are the same for the two particles, r . The force on arms AD and BC are F2 = B I b sin .
q These forces merely compress the coil and are
Now, the charge of an alpha particle is twice that resisted by its rigidity. These forces also exert no
of a proton and its mass is four times the mass of a net force on the coil. Thus, both the statements are
proton, m / q will be the same for both particles. true but Statement-2 is not the correct explanation
Hence will be the same for both particles. for statement-1.
7. The correct choice is (d). Kinetic energy K = qV. 9. F = q(v B). Since F is perpendicular to v, power
2m1qV 2m2 qV p = F v = 0. Hence both the statements are true
Therefore r1 = and r2 = and statement-2 is the correct explanation for State-
qB qB
ment-1.
r1 m1 m1 r1 2 10. The path of the Particle is a helix; it is a circle if
Hence = = .
r2 m2 m2 r2
8. A coil ABCD B. is true.
Its planes makes an angle with B and it carries
mv 2m K 1
a current I as shown in the Fig. 24.90. The forces 11. r = , where K mv 2
F1 = B Il on AB and CD are equal and opposite but qB qB 2
constitute a couple which tends to rotate the coil. Since K and q are the same, r m . Hence the
electron will describe a circle of a circle of a smaller
radius. So. Statement-1 is false but Statement-2 is
true.
12. If v is parallel to B, F = 0. Hence the particle
does not experience any force and is, therefore, not

with a constant speed. Statement-1 is false and


Statement-2 is true.
13. Placing a soft iron core inside the coil makes the

in the coil. The sensitivity increases because, for

Fig. 24.90 when a core is inserted the coil. Statement-1 is true


and Statement-2 is false.

VI

Integer Answer Type

1. A potential difference of 600 V is applied across the 2. Two long straight parallel wires are 2 m apart, per-
plates of a parallel plate capacitor. The separation pendicular to the plane of the paper (Fig. 24.91)
between the plates is 3 mm. An electron projected Wire A carries a current of 9.6 A directed into
vertically, parallel to the plates with a velocity of the plane of the paper. Wire B carries a current
2 105 ms-1 P at a dis-
tance of 10/11 m from the wire is zero. Find the
in the region between the capacitor plates. Neglect magnitude of the current (in ampere) in wire B.
edge effects. 3. A steady current current I goes through a wire loop
IIT, 1981 PQR having shape of a right angle triangle with
Magnetic Effect of Current and Magnetism 24.51

PQ = 3x. PR = 4x and QR = 5x. If the magnitude of 4. A long circular tube of length 10 m and radius
0I 0.3 m carries a current I along its curved sur-
P due to this loop is k , face as shown. A wire-loop of resistance 0.005
k. 48 x
ohm and of radius 0.1 m is placed inside the tube
with its axis coinciding with the axis of the tube.
IIT, 1990
The current varies as I = I 0cos(300 t) where I0 is
constant. If the magnetic moment of the loop is
Nμ0I0sin (300 t), then ‘N’ is
IIT, 2011

Fig. 24.92

Fig. 24.91

SOLUTION
directed to the right in the plane of the paper. Its
V 600 magnitude is given by (see Fig. 24.94 on page
1. E = 3
=2 10 Vm 5 –1
d 3 10 24.52)
0 I1
24.93] B1 =
2 R1
E 2 105
e v B = eE B= = 1T P B2 due to
v 2 105 I2 is equal and opposite to B1. Therefore, the current
in wire B should be normal to it and directed out the
plane of the paper. Therefore
B1 = B2

0 I1 0 I2
or =
2 R1 2 R2
I1 R2
or I2 =
2 R1
10
Given I1 = 9.6 A, R2 = BP = m and
11
10 32
R1 = AB + BP = 2 + m.
11 11
Fig. 24.93 Thus
2. Let the currents in A and B be I1 and l2 respec-
9.6 10/11 9.6 10
tively. Let AP= R1, BP = R2. According to Ampere’s I2 = = 3.0 A
P due to I1 is 32/11 32
24.52 Comprehensive Physics—JEE Advanced

25 r 2 12 x
1 = 2
r=
144 x 5

3 4
sin = and cos =
5 5
3 0I 4 7 0I 7 0I
B(at P) =
4 r 5 5 20 r 48 x
k =7

4.
inside the tube is
0I
B= ; L = length of tube
L

0 I0 cos (300 t )
=
L
If r
Fig. 24.94
through it is
3. P is (Fig. 24.95) 0 I0 cos (300 t ) r 2
= BA =
0I L
B (at P) = [sin + sin 90° – ]
4 r Magnitude of induced emf is
0I r2
= (sin + cos ) d 0 I 0 (300)
4 r |e| = sin (300 t )
dt L
The induced current through the loop is
e 0 I 0 (300) r2
i= sin (300 t )
R LR
Magnetic moment of the loop is
2 4
300 r 0 I0
M=i r2 = sin (300t)
Fig. 24.95 RL
r Given M = Nμ0I0 sin (300t) (i)
Now sin =
4x Comparing (i) and (ii) we get (ii)
r
and cos = 300 2 r 4 300 (3.14) 2 (0.1)4
3x N =
r2 r2 RL 0.005 10
These equations give sin2 + cos2 =
16 x 2 9 x2 = 5.92 6
Electromagnetic Induction and A.C. Circuits 25.1

25
Chapter
Electromagnetic Induction
and A.C. Circuits

REVIEW OF BASIC CONCEPTS d


e
dt
25.1 MAGNETIC FLUX Lenz’s Law

Thus
A B is
d
e= – k
dt
=B A = B A cos
where k
where k = 1 and one
N turns, the can write
d
e= –
= N B A cos dt
d
e
dt
= B dA
N turns
FARADAY’S LAWS OF d
25.2 e N
ELECTROMAGNETIC INDUCTION dt
The -
induced em 1 d
i= =
total resistance circuit R dt
Faraday’s Law of Electromagnetic Induction The direct
law.
Flow of Induced Charge

q
circuit.
- 1 d 1 change in lu
q = idt = dt = d =
R dt R Resistance
25.2 Comprehensive Physics—JEE Advanced

Heat Dissipation

d
H = eidt = idt = i d
dt
= induced current
Fig. 25.3
Fleming’s Right Hand Rule
I1 and I2 -
I1 and I2 will decrease.
I1 and I2

25.3 EXPRESSION FOR INDUCED EMF

(B).
B increases with time, the induced current i is
Applications of Lenz’s Law
B
towards the magnet, the induced current i is

Fig. 25.1 Fig. 25.4

i d d dB
e BA R2
dt dt dt
B decreases with time, I
B
dR
increases at a rate , then
dt
d dR
e B R2 B 2 R .
dt dt

Fig. 25.2 PQRS


- B v, the
I
or
I increases with time.
The current I

or v
I decreases with time.
I1 and I2

Fig. 25.5
Electromagnetic Induction and A.C. Circuits 25.3

e Blv where l = PS = QR e = Blv


Induced current i R is the resistance

e Bl v v
i=
R R
v v
Force F v
v is
B 2l 2 v
F = Bil =
R
B 2l 2 v2
Power needed is P = F v = Fig. 25.7
R

e = Blv sin
R is
v B as
NOTE
ends P and Q e = Bv R
-
2BvR
v

XY slid-
ing on metallic rails PQRS to the right as shown in Fig.

Fig. 25.8

PQ l P is

v B

1
e= B l2
2

Fig. 25.6

Change in )

A N turns is rotated
B , the

e = e0 sin = e0 sin t Fig. 25.9


where e0 = NBA

25.4 MOTIONAL EMF


l
v B as shown in Fig.
P and Q

Fig. 25.10
25.4 Comprehensive Physics—JEE Advanced

R
25.2
B as
A solen
centre O P
1 -
e= B R2
2
25.5 ELECTRIC MOTOR
SOLUTION
I in a solenoid
V n
e B= 0nI
V R is

V e
i=
R
= 0°.
Vi and heat loss = i2R
A N the
= Vi – i2R = ei.
ei e -
= ing the coil is
Vi V
Some important points about a d.c. motor = NAB cos = NAB cos ° = NAB = N r2 0nI
e and hence current i -
V r
V = 0.1 m.
e= and
2 I1 = 0 to
e=0 I2 = 2 A is
and initial current = V/R = N r2 0n I2 – I1

i = 100 0.12 10
is minimum. 2
10
2
0 2
25.1 e= = 3
t 10
, 100 turns and side
EXAMPLE 25.3
- T
-
t
SOLUTION
t2 t
R=2 , N = 100, A = 0.1 0.1 = 10–2 m2,
Find the induced current in the coil at t = 10 s.
1
2 B = 0.1 T and t
SOLUTION
NBA 2 – cos 1
= 100 0.1 10 –2 d d
e| = = t2 t t
1 dt dt
= 0.1 0
2 At t = 10 s, e 10– 3
change in lu 0.0 Induced current at t = 10 s is
In e= =
time 0.0 3
e 10
e 1 I= = = 10–2A
Induced Current i = R
R 2
Electromagnetic Induction and A.C. Circuits 25.5

EX 25.4

10 T.

SOLUTION Fig. 25.12

100
=
3
l = 0.
=2
1 2
e= Bl
2
-
1 2
= 3 10
2 3
10 10 its diameter. Assume that the normal to the
= 0 with
t = 0.

SOLUTION

coil changes with time.

Fig. 25.11
the coil does not change with time, hence no

10 coil does not change with


time, hence the magnetic
25.5
-
–1
in the eastward direction at a Fig. 25.13
-
10 T. Find the
a diameter, as shown in

SOLUTION
–1
v = 300 ms–1
e = Blv 10
A = r2
Fig. 25.14
25.6
r N turns is
B then at time t, = t
this time is
= NBA cos = NBA cos t
25.6 Comprehensive Physics—JEE Advanced

Q acquires a nega-
d d
e= – =– NBA cos t Force on the element is
dt dt
0I
= NBA sin t dF = qvB = qv
2 x
= NB r2 sin t T
= Nr2B sin t dF qv 0 I Iv
dE = = = 0
q 2 xq 2 x
25.7
dV
A metal rod PQ v Now dE = –
dx
CD I as 0Iv
P and Q dV = – dE dx = – dx
2 x
at distances a and b
ere dV
PQ is
b
0 Iv dx 0 Iv b
V = ln
2 a
x 2 a

EXAMPLE 25.8
A metal rod PQ l v
AB and CD
straight wire XY I as shown in Fig.
R
PQ
Fig. 25.15

SOLUTION
-
dx. Consider one such element
at distance x CD

Fig. 25.17

resistance R.

on rod PQ v.

SOLUTION
Fig. 25.16 -
I in
wire CD is ACPQ with time.
0I
B=
2 x
The directi
v -
PQ

P to Q
P to Q. Hence end P Fig. 25.18
Electromagnetic Induction and A.C. Circuits 25.7

SOLUTION
dx at a distance x PQ is
ACPQ when the rod is at a
0 Ir
d = BdA = Brdx = dx distance x R is
2 x
= BA = Blx
where r is th PQ t.
ACPQ is
d dx
a l e= = Bl
= Blv
0 Ir dt dt
= d = dx
a
2 x R ACPQ = R + 2kx
a l induced in the circuit is
0 Ir dx Ir a l
= = 0 ln e Bl v
2 x 2 a i= =
a R 2kx R 2kx
d I a l dr i R 2kx
Induecd e= = 0 ln v=
dt 2 a dt Bl
0Iv a l dr
= ln v= 25.10
2 a dt
A metal rod PQ m -
ent induced in R is AB and CD
e Iv a l l
i= = 0 ln R
R 2 R a
-
width dx
ACPQ
0I u
dF = Bidx = idx
2 x distance x
F PQ is
a l
0 Ii dx
F= dF =
2 a
x
a l
0I 0I v a l dx
= ln
2 2 R a a
x
2
v 0I a l
= ln
R 2 a Fig. 25.20

25.9
A metal rod PQ l SOLUTION
AB and CD k Bvl
Induced currnet i =
R
B
R is connected dv
Force F = Bil = – ma = – m
dt
F
The ne
PQ so
that it is accelerated
dv Bvl
m =–B l
dt R
v B 2 vl 2
PQ when it is at a =–
R
distance x R. Fig. 25.19
25.8 Comprehensive Physics—JEE Advanced

dv B 2l 2 iR =
d
=– dt = –kdt dt
v mR
dq d
B 2l 2 R=
k= dt dt
mR
v t dqR = d
dv
Intergrating k dt d
v dq =
u 0 R
ln
v
– kt 2 NBA 2 NB r2
u Q= = =
R R R
v
= e–kt 25.12
u
Two circular coils A and B a and b -
v = ue–kt b>a
dx
= ue –kt x= 3b -
dt
sient current I B
The rod comes to rest when t = A is R
Integrating
x A
dx = u e kt
dt
0 0

u u u
x=– e kt
0 =–
k k k
umR
x=
B 2l 2

25.11
r has N turns and a resis-
tance R Fig. 25.21
B
Q SOLUTION
A due to current
I in coil B is
SOLUTION 2
0 Ib 0I
2 BAB = 2 2 3/2
= x = 3b
A r 2b x b

NBA cos 0° Sin A,


= NBA A, hence = 0°.
NBA A is
– NBA 0I a2
= BAB area o A cos 0° =
= NBA NBA NBA b
d
Induc e
d dt
e| =
dt d
IR =
e d 1 dt
Induced current is i =
R dt R IRdt = d
Electromagnetic Induction and A.C. Circuits 25.9

1 where V
Idt = d =
R R
0I a2 V= –
d
=–
d
BA
or Q= = dt dt
R bR
d
25.13 =– kt x2 kx2
dt
R and mass M

–E 2 x = – kx2
Q kx
E=
2
B = kt k is a
constant and t Force acting on the element is
Q kx
dF = dq E= 2 xdx
t R2 2
kQ
= 2
x2dx
R
t.
Torque acting on the disc is
R R
SOLUTION kQ kQR 2
= xdF = x3 dx =
R2 R2
Q 0 0

R2 = I , where I
is the angu-
dx at a distance
lar acceleration
x xdx
1 d
I= MR2 and = . Hence
2 dt
Q
dq = 2 xdx kQR 2 1 d
R2 = MR2
2 dt
kQ
d = dt
2M
t
kQ
d = dt
0
2M 0

kQt
=
2M
25.6 MUTUAL INDUCTANCE

Fig. 25.22 i
= Mi where M
i is changing with time, the
E. Since
dV di
E= – e= – M
dl dt
dV = – Edl = – E 2 x Expressions of M in some situations
l N1 wound
V= – E 2 x
N2 turns.
25.10 Comprehensive Physics—JEE Advanced

0 N1 N 2 A 25.7 SELF INDUCTANCE


M= ; A = common cross-sectional area
l
i = Li,
R and r
where L
R >> r di
e= L .
0 r2 dt
M=
2R N turns, cross-
sectional area A and length l
2
0N A
L=
l

Fig. 25.23

r
a and b with a, b >> r
Fig. 25.26
2 2 2
2 0 r a b 1 2
M= U= Li .
ab 2

L = L 1 + L2
M

Fig. 25.27
Fig. 25.24
L = L1 + L2 + 2M
a and b -
tance x L = L1 + L2 – 2M
0 a x b
M= log e
2 x
1 1 1
M
L L L2

M = k L1 L2 ; k

GROWTH AND DECAY OF CURRENT IN


25.8
Fig. 25.25 A D.C L—R CIRCUIT (FIG. 25.28)
S1 is closed at t = 0, with switch S2
NOTE
0 - t,
= 0 r, where L
e–t/
r
i = io = is the time constant.
R
Electromagnetic Induction and A.C. Circuits 25.11

SOLUTION

0
= LI L= = 3
I 1 10
–3

25.15
Fig. 25.28

t =
io = E/R

1 SOLUTION
At t = , i = io 1 i 0.
e dI
= = 20 As–1
dt 0.1
dI
e L
dt
–3
1 =L L H

25.16
Fig. 25.29

Decay of current: At time t = 0, let i0 = E/R


S2 S1

i = i0 e–t/
i0 SOLUTION
At t= , i = i 0.
e
0 AN1 N 2
Mutual inductance M =
l
2
0 0.
=
1.0

H
dI – –2
e M =
Fig. 25.30
dt 0.01

25.17
25.9 ENERGY STORED IN AN INDUCTOR
-
L is increased are connected in series to a
I
through a switch. The switch is closed at time t = 0
1
U= LI2 current.
2
25.14

- -
tor at t e–2
25.12 Comprehensive Physics—JEE Advanced

SOLUTION 25.19
i = i0 – e t/ i0 i
E
t , i = i0 = =
R 100
L
Time constant
R 100

i0 = i0 e–t/ SOLUTION
L = 100 mH = 0.1 H, R ,E
1
– e–t/ E 2
2 i0 = =
R
1 1
e–t/ = 1– = L 0.1
2 2 = = = 0.002 s
R
t i
e t/ = 2 = e–t/
i0
1 t
t = e–t/ e–t/ ln2 =
2
di d t = ln2 = 0.002 10–3 s
VL = – L =–L [i0 e–t/
dt dt
1 25.10 TRANSFORMER
= – Li0 e–t/

Li0
e–t/

.
e e–2

primary
EX 25.18 secondary
-

SOLUTION

E 12 3
i0 = = =
R
2
1 2 1 3 Ns Ns
U= Li 0 = 10 s
= or =
2 2 p Np
s
Np
p

3
P = i 0E = where Ns Np
2
3 es d s/dt
or P = i 02 R = ep d p/dt
Electromagnetic Induction and A.C. Circuits 25.13

Thus
SOLUTION
d Ns Ns d p
es = p Pi = epIp = = , ep =100 and es
d t Np Np d t
Pi
Ns Ip = =
or es = ep ep 100
Np
Ns > Np, then es > Po
ep = Po = Pi =
Pi
Po 3000
Is = =
step-up N s < N p, es 200
then es < ep step-down
25.22

I2R

and that

SOLUTION
Np Ns = 2000, Po es
ep Np Np
or es Is = ep Ip ep = es = 1000
Ip
es Ns Ns 2000
es Ns
=
ep Is Np = 200
where Ip and Is Po Po
=
Pi ep I p

25.20 Po 12000 200


Ip = = = A
ep . 200 3
Po 12000
Is = = = 12 A
es 1000
2
2 200
Pow Ip Rp =
SOLUTION 3
ep es Ip Is
2 2
Pi ep Ip = 220 Is Rs
Po es Is = 10
Po
cienc = = 25.23
Pi 1100

25.21

-
-
25.14 Comprehensive Physics—JEE Advanced

SOLUTION V0 2 t T
=– cos
es T T 0
Ns = Np = 1000 = 20,000
ep 220 V0
= =0
2
Po Pi .
Is = = = = 1.2 A Simila
es es
I = I0 t I =0
Is is called th
V = V0 t
25.11 ALTERNATING CURRENT
1
V = V0 sin t V2 = V02 t dt
T
resistance R, the current I in the circuit is
Vo2 1
T
V V
I= = 0 sin t = I0 sin t = t dt
R R T 02
where I0 = V0 /R, is the maximum or peak value
V02 1
T T
I 1
= dt 2 t dt
T 20 20
sinusoidally
varying current whose magnitude changes continuously V02 T 1 2 t T
=
with time and whose direction reverses periodically is T 2 2 2 0
called an alternating current
V02 T V02
= 0
T as T 2 2
2 Roo
= =2
T
–1
where T V0 1
T Vrms = V2 = = V
2 2
reads
2 t Simila -
I = I0 sin ing current I = I0 t
T
I0 1
Root Mean Square Voltage and Current Irms = I2 = = I
2 2
X t
T EXA 25.24
T V t
X t dt T
0 1
X = T
= X t dt V t
T0
dt
0

SOLUTION
2 t
V = V0 t V0 sin
T
V = V0 t
T
1 V0
V = V0 t dt
T0
V0 100
Vrms = =
V0
T
V0 2 2
= sin t dt = – co t T
0 and
T 0 T
or 2
Electromagnetic Induction and A.C. Circuits 25.15

25.25
Z=R -
A 100 mum.
Special Cases
(a) A.C. circuit containing only a pure resistor (Fig.
25.32)
SOLUTION

Vrms
V0 = 2 Vrms = 2
Vrms 200
Irms = = =2A
R 100
Fig. 25.32
V0 .
I0 = =
R 100 VR = V0 sin t
I = I0 sin t
25.12 SERIES LCR CIRCUIT
V0
V VL L where I0 =
R
VC C VR R
e across R
current in the circuit.
(b) A.C. circuit containing only an ideal inductor (Fig.
25.33)

Fig. 25.31

2
V = VR2 VL VC

2 Fig. 25.33
2 2 2 1
Z= R XL XC = R L
C VL = V0 sin t
The current in the circuit is
I = I0 t– I = I0 sin t
2
1
tan = L V0 V
C where I0 = = 0
XL L
1 1 XL = L is call
L> , i.e. > , then tan
C LC across the inductance leads the current in the circuit
/2.
1 1 (c) A.C. circuit containing only an ideal capacitor
L < , i.e. < (Fig. 25. 34)
C LC
VC = V0 sin t
1 1
L= , i.e. = , then =0 I = I0 sin t
C LC 2
25.16 Comprehensive Physics—JEE Advanced

V0 is cal edance.
where I0 = = CV0 I = I0 t+
XC
1
where = tan–1
R C

Fig. 25.34

XC = i -
C
Fig. 25.36
age a
/2.
25.13 POWER IN LCR CIRCUIT
(d) A.C. circuit containing an ideal inductor and a pure
resistor (Fig. 25.35) In a series LCR
V0 = I0 Z V = V0 sin t, the current in the circuit is
I = I0 t±
V0 = VR2 VL2
XL < XC or XL > XC.
VR = IR, VL = IXL
2
Vo 1
V
and Z = 0 = R 2
X L2 = R 2
L 2 is called I0 = ; Z = R2 L
I0 Z C
nce. 1
L
–1 C
and = tan
R
Insta
source is
Pt VI = V0 sin t I0 t
= V0 I0 sin t t+

T
1
Fig. 25.35 P P t dt
T0
I = I0 t–
T
1
where = tan –1 L
is t = V0 I0 sin t t cos cos t sin dt
R T 0
nd current in the circuit. T T
V0 I 0
(e) A.C. circuit containing an ideal capacitor and a = cos sin 2 t dt sin f sin t
pure resistor ( Fig. 25.36) T 0 0
V0 = I0 Z t dt
VI T
where V0 = VR2 VC2 = 0 0 cos 0
T 2
VR = IR, VC = IXC V0 I 0 V0 I0
= cos = cos
V 1 2 2 2 2
and Z = 0 = R2 X C2 = R2
I0 C or P = Vrms Irms cos
Electromagnetic Induction and A.C. Circuits 25.17

Power Factor of an A.C. Circuit V0


I0 = V0 I0 R
R
on Vrms and Irms is
In ter I0 , I0
1 I0 R
L I0 = ma
tan = C 2 1/2
1
R R2 L
C
1
and cos = 1/2
1 tan 2 Figure 2 I0 .

1
= 2
1
L
1 C
R

= R
2 1/2
1
R2 L
C
R
= Fig. 25.37
Z
I0 ma
resis tance and
1 2 when I0 = ,
P i.e. when 2
im e
Special Cases I0 R I0 R
ma ma
=
For an A.C. circuit containing only a resistor, 2 1/2 2
1
R R2 L
Z = R and cos = = 1 = 0 and C
P =V I R
rms rms 2
For an A.C. circuit containing only an inductor or 1
R2 + L = R2
a capacitor, C
P =0
1
LCR circuit. Hence P = L– =±R
C
1 2 1 R
Case 1: L – =+R – =
Wattless Current C LC L
2 R 2 1
– – r = 0, where r =
L LC
P = Vrms Irms Th
2 2 1/2
R rL
2= + 1 2
2L R
1
Bandwidth and Quality Factor of LCR Circuit Case 2: L – = –R
C
For an LCR
2 R 2
V = V0 sin t + – r =0
L
V0 V0
I0 = 2 2 1/2
Z 2 1/2
1 R rL
R 2
L 1 = – + 1 2
C 2L R
25.18 Comprehensive Physics—JEE Advanced

R
= 2 – 1 = V t=+1 t1 =
L 2
Qualit t1 =
Q LCR 2
resonant qu I t– t2 –
Q= = r
width = t2 = –
2 2
1 L 1 L
= = -
LC R R C
Q i
P Q. t = t1 – t2 = – =

L 100
Now tan = =
R 100
.
°= rad
.
t= = = 3.2 10–3 s
100

25.27
Fig. 25.38

Q. The resonance
Q, the

25.26
- SOLUTION
tance 100
C = 100 F = 100 10 F, R , Vrms =
–1
=2 =2
V = V0 sin t
I = I0 t+
V0 1
I0 = , tan =
SOLUTION 1/2 CR
1
=2 =2 rad s–1, R2 2
Vrms C2
L R = 100 .
V = V0 sin t 2 110
I0 = 1/2
2 1
I = I0 t– 2
120 10
V0
I0 = , t=+1 t1 =
2 2 2 1/2 V
R L 2
L t1 =
tan = 2
R I is t+
V0 = 2 Vrms = 2
t2 + = t2 = –
2 2
2
I0 = -
2 2 1/2
100 100 .
Electromagnetic Induction and A.C. Circuits 25.19

230
t = t1 – t2 = VR rms = Irms R=

1 1 1
Now tan = = VL,C rms = Irms rL
CR 120 rC
33. =
= rad
P = Vrms Irms cos
33. = Vrms
t= = 10–3 s Irms
120
2
Vrms 230 230
= =
25.28 R
A series LCR circuit with L C F and
R 25.29
a.c. source.
A

- B, again the
-
A and B.
L, C A and B.
and R at resonance.
SOLUTION
L and C at resonance. A is a resistor and B
- Vrms Irms

Vrms 220
SOLUTION A is R = =
I rms .
1 1 –1 220
r = = Reactance o B is XC =
Vrms
=
LC 0 10 I rms .
Im A and B are connected in series
= r
=
r
2 2 is
2 1/2 Z= R2 X C2 = 2 2
2 1
Z= R L
C Vrms 220
1 Irms = =
=R L= Z .3
C

V0 2 230
25.14 LC OSCILLATIONS
I0 = =
Z L and
VL = Irms XL = Irms C
rms rL
230
= 1
1 LC
VC rms = Irms XC = Irms
rC and tim T= 2 LC
230 1
= =
10 q = q0 t+ I = I0 t+
25.20 Comprehensive Physics—JEE Advanced

Multiple Choice Questions with Only One Choice Correct

1. g t = t1 t = t2
B less than g t1 and t2.
v g
to
v1/2 v 4. PQR a is at
v2 v3/2 B at t = 0 as

v and its edge R -


t = t0

induced current i with time t

Fig. 25.39

2.

Fig. 25.41 (a)

the other coil the current will decrease.


3.

t = t1
t = t2

Fig. 25.41 (b)


5. PQ and RS
CD
Fig. 25.40
g t CD v as
g t
Electromagnetic Induction and A.C. Circuits 25.21

9.
v
t as
- I = 3 sin t t

Fig. 25.42
3
A
2 2

A
6. a 2 2
- 10. A series LCR -
I. The centre O - tance L C -
tance x tance R L, C and R, the
v x >> a

x L
x2 v C
1 R
x R or L or C.
1 11.
2
x Fig. 25.43
7.

as shown. The ratio I1 / I2 t is


L1 L2
L2 L1
12. In a series LCR -
L1 L2
L2 L1

2
13.

Fig. 25.44
8. I2 = 3A in the V = 200 2 t+
where V t
resistor is the ammeter is

14.

Fig. 25.45
25.22 Comprehensive Physics—JEE Advanced

F
F
15. -

line is 1

16. - Fig. 25.48


I in the circuit shown
18.
t K
at t = 0

19. f is connected
to an LCR
I in
f

Fig. 25.46

Fig. 25.49
20.
Fig. 25.47

17.

V across the inductor L with time t K


t = 0, in the circuit shown in

21.
LC -
2
= 10.

22. L, C and R
Electromagnetic Induction and A.C. Circuits 25.23

28.
inductance and a resistance connected in series
RC LC
L/R C/L
23.
V = 200 2 t
is connected to 1 29. V = V0sin t
a circuit. As a result a current I = I0 t–

V 0I 0
V 0I 0 V 0I 0
24.
30. L and resistance
A and B is R v -
B,

Fig. 25.50

25. V and
current I
V t

and I = 100 sin 100 t mA


3
Fig. 25.51

BLv/R
BLv/R
26. BLv/R

31. A -
B. A

connected across the ends such that the total resis-


27. R
-

BR AB
A R
B2 A
ABR
R2

32. R is
-
B MNQ the
25.24 Comprehensive Physics—JEE Advanced

v P at a distance r
across the ring is region
1
1 r
Bv R2 and M 1
2 r .
RBv and Q r2
RBv and Q IIT, 2000

Fig. 25.53
Fig. 25.52
37. B
33. l r
L L >> l

R -
2
l l
L L r2 B r2 B 2

L L2 2R R
l l 2 2 2
r B r B
2R R
34. R I, lies 38.
in the x-y –1

x-y
I is 0.2 × 10
R
R

39. F is charged to a
35. is

36. Bt
a and is directed 40. r
B.
Electromagnetic Induction and A.C. Circuits 25.25

Br 2 Br 2
1 1
t 10 t
Br 2 Br 2
1 1
t t
41. l, mass m and resistance
R Fig. 25.55

height h IIT, 2000


45. ABCD

mgR m 2 gR 2 v
2 2
2 Bl 2B l
2
mgR m 2 gR 2
B 3l 3 2B l is induced
42. AD BC
r1 and r2 r1 > r2 BC AD
AD nor in Fig. 25.56
BC
0 r22 0 r12
AD and BC
2r1 2r2
IIT, 2001
2 2
0 r1 r2 0 r1 r2 46. P and Q -
2r1 2r2
43. - S
tance 1 v in Ip P E
B = 2T as shown in Fig. IQ1 Q
S IQ2 Q.
IQ1 and IQ2 E
are:
equal to 3

–1 –1
–1 –1

Fig. 25.57

Fig. 25.54

44. Two

IIT, 2002
25.26 Comprehensive Physics—JEE Advanced

47. 51. An LCR series circuit with R = 100 is connected to

2 3
A
3 2
IIT, 2002
48. E = E0 t 52.
v
E and the current I in the circuit current I
its ends at distances r1 and r2 r2 > r1
R-C or R-L or
L-C
0 Iv r2
log e
R , C = 10 F 2 r1
R , C=1 F 0 Iv r1 0 Iv r1
loge 1
R , L = 10 H 2 r2 r2
R , L=1H 53.
IIT, 2003 increasing according to the equation I t2

54.

s. The

Fig. 25.58

49. An air
ms–1 55. a and b
x as
is 2 10 B = kx where k
-
1 2 coil as shown in Fig.
10 10
3 1
10 kab2
50. L and resistance R is con-
1
kab2
1 2
Fig. 25.59
2 kab2
56.
L L
loge loge
R R

L L
loge loge
R R
Electromagnetic Induction and A.C. Circuits 25.27

57. 58.
along the x -
- x
resented as
x
x
x direction

59.

I1 and I2 are the


currents in the segments AB and CD. Then,
I1 > I2
I1 < I2
I1 is in the direction BA and I2 is in the direc-
tion CD
I1 is in the direction AB and I2 is in the direc-
tion DC

Fig 25.61
Fig. 25.60

ANSWERS

1. 2. 3. 4. 5. 6.
7. 8. 9. 10. 11. 12.
13. 14. 15. 16. 17. 18.
19. 20. 21. 22. 3. 24.
25. 26. 27. 28. 29. 30.
31. 32. 33. 34. 35. 36.
37. 38. 39. 40. 41. 42.
43. 44. 45. 46. 47. 48.
49. 50. 51. 52. 53. 54.
55. 6. 57. 58. 59.

SOLUTIONS

1. l Bl v
e Bl v R i=
R
25.28 Comprehensive Physics—JEE Advanced

F = Bil
B v2 a Bva
B2 l 2 v i= t
= Rb R
R
B 2 l 2 v2 i against t
Power needed is P = F v = . So the
R Bva B v2 a
.
R Rb
2. -
5. As the rod CD
in each coil will decrease. CD
C to D
3.
C to D and, as a result, end C -
D
t1 and t2 no current is in-
g
-
-
charge.
g.
6.
2
4. t = BA = B a A = a2
right a distance CD = v t RD = b. I
Now B = 0
2 x

0I a2
=
2 x
d I a 2 dx
e= = 0 2
dt 2 x dt

0 I a2 v dx
= 2
v
2 x dt
1
Hence e
Fig. 25.62 x2
7.
Now L1
RC
=
CT across L2 t. Hence.
RD QD
dI1 dI 2
L1 = L2
RD CD CT dt dt
=
RD QD L1 dI1 = L2 dI2
b vt CT Integrating, we get L1 I1 = L2 I2
= I1 L
a/2
b = 2.
I2 L1
a b vt
CT = 8.
2b
a
ST = 2CT = b – vt
b
a
e Bv ST = Bv a vt
b
R
e Bv a
i= = a vt
R R b Fig. 25.63
Electromagnetic Induction and A.C. Circuits 25.29

I2 resis- VL =
12 VC
tance = 3 I1 = = 2 A. There-
V= VR2 VL VC 2
= VR
I = I1 + I2
Current in the circuit is
V
I= =
9. Z 2R
2 2
I0 =
Hence VL = IXL = R XL = R
I0 2R
Irms = = A
2 2
2
1
10. XC > XL or > 13.
C
L V0 = 200 2 = 100 rad s–1.
1/ 2 Irms = Vrms C
2
R R
= = 2 200 2
Z 1 = 100 10
R2 L 2
C
14.
-
1/ 2
tor, i.e.
1 1 LI2
1 CV 2 = LI 2 or C =
= 2 2 V2
2
1 2
L 1 2
1 C = 2
10 F
R

15. Current in the transmission line is

I= = = 100 A
R, C and L are ltage
Power loss = I2R 2
11. -
nance, i.e. when Z = R, where R
10000
Vrms
Vrms 12
R= = =2
I rms
16. LR
–R t /L
I = I0 e
E
I= ;r
R r or I = I0 e –t /
where = L/R is the time constant. At
2= r = 1.0 t = 0, I = 0. At t >> t, I = I0.
2 r
12. VR = VL = VC R = XL = XC 17.
XC = 0 and the
dI d
V=–L = – LI0 e –Rt/ L
dt dt
Z= R2 X L2 = R2 R2 = 2 R
R –Rt /L
R = XL = – LI0 e
L
25.30 Comprehensive Physics—JEE Advanced

V V0 e –t / , where V0 = I0R is the initial


V = V0 at t -
t >> . Hence 22. RC charge /
L
18. = ltage charge charge
R = = time
R tan 100 tan current ltage charge/time
L= =
2 1000 RC
–3
10
23. V = 200 2 t
equation with V = V0 sin t,
19. The current in an LCR
V0 = 200 2 = 100 rad s –1
V
I=
2 1/ 2
2 1 Vrms 1
R L I= = Vrms C Zc
C Zc C
where =2 . Thus I increases with increase in
V0 200 2
= c = C= 100 1 10
2 2
1 1
cL = or c = = 20 10 –3 A = 20 mA
cC LC
24.
when I > c, I decreases
with increase in inductance L
20.
1 1 1 1 3 3
100 10 3 = + + = = =1
=
L
= =2 10 –3 s L L L L L 3
R or L
= 2 millisecond.
25. V0
Current at time t I0 = 100 mA = 100 10 –3
I = I0 e –t/ I and V is = -
3
where I0 I
I0 / 2 is V0 I 0
1 P= cos
e –t/ = or e t/ = 2 or t = 2
2 3
100 100 10
=
2
21. LC circuit is
1
= = 26.
2 2 LC
1 current is induced in the coil.
or C= 2 2
L 27.
where L 10 H.
3
due to which it contracts. As a result the contact is
For 10
1
C= 2
2
103 10 wire oscillates.
10 –12 28.
C the circuit is
Electromagnetic Induction and A.C. Circuits 25.31

V V R2 + V 2L 1/2

1/2 l l << L

29. V and current I is 2 2 2 0 I l2


12 = Bl =
L
Mutual inductance
30.
2 2 l2
12 0
M12 = = L
I
no current is induced.
l2
change f i
i.e. M12
31. Induced charge q = = . L
resistance R
34.
f -
i = BA q = BA/R.
32. v the decrease in x-y
area with time is
dA
Rv
dt

d d dA
e=– =– BA B = 2RBv.
dt dt dt

-
tion. Hence Q
33. -
rent I
B
0 I
+ cos
L/2
Fig. 25.65
2 0 I
= 35.
L
V
I= e –t/ = L/R.
2 2 0I R
= =
L L . mH
= =
R

12
1.0 = e –t
1 1
or e –t =1– =
2 2
t 1
or – = loge
. ms 2
or t

36. -
Fig. 25.64
25.32 Comprehensive Physics—JEE Advanced

distance r 39. Q = CV and Q = Q0 cos t. Also


radius a B Q0 = CV0.
Q V 1
cos t= or t= .
a2 d B Q0 V0 12 2 3
E=
2r d t Now
At r = a, E a dB/dt E 1
r > a, E =
decreases as 1/r LC
37. L 10–3 H and C = 2 10 F. Using
t 10
= rad s–1.
V = V0 sin t 2 3
dQ d
Now I= Q0 cos t
V0 = AB and dt dt
1 = – Q0 sin t
A= r2 -
2 I Q0 sin t = CV0 sin t
circular coil. Hence
10
1 10 12 sin
V0 = 2
r B 2 3 3
2

V2 V02 sin 2 t ma ld change in area


P= 40. e
R R time
B A
2
<P> = V0 < sin2 t> t
R r,
2 r r
1 V02 1 r2B
2 . There-
=
2 R 2R 2
2
r
2
r B
2 A = r2 – = r2 1
= 2
R
B r2
e= 1
38. L t
A= L2
BA = B L2
2 41. v= 2gh e = Blv = Bl 2gh .

Bl 2 gh
e= I=
tim n R

B L2 1 B 2l 2 2 gh
= B L2 Force F = BIl =
2 / 2 R
1 2
= equals mg
2
B 2l 2 2 gh
= mg
R
Electromagnetic Induction and A.C. Circuits 25.33

46. t = 0. The current


m 2 gR 2
h= Ip P t0
2B l
Ip
42. Q
IQ 1
0I -
B=
2r1
where I IQ 1 should
the inner coil is IP
S IP
0I
=B r 22 = r22
2r1
= MI IQ 2

0 r22
M= 47. -
2r1

43. PQRS i.e. V n


P and l l
R= 2
or R . Here
R r r2
. Total resistance R = 3 + 1 =
e = Blv = 2 0.1 v =
0.2 v. V2 n2 nr 2
Power P = or P
e 0.2v R l /r2 l
Induced current I = .
R 2 2
P2 n2 r2 l1
I=1 10–3 A =
P1 n1 r1 l2
0.2v
Hence 1 10–3 = l1 and radius r1 is stretched
–2 –1 –1 to a length l2 such that its radius reduced to r2, then
v=2 10 ms = 2 cm s , which is

m = r 21 l1 d = r 22 l2 d d
44. Th
2
l1 r2
or = we get
l2 r1
2
P2 n2 r2
=
P1 n1 r1
n2 r2 1
n1 r1 2
we get

P2 2 1
= 1, which is
Fig. 25.66 P1 2
45. AD and BC, 48. E = E0 t
E = E0 sin t = 100 rad s–1 -

is changing with time. Hence the correct choice is R-C


R-L circuit. Hence the circuit does not contain an
25.34 Comprehensive Physics—JEE Advanced

L
For R-C E and or t= loge
R
I
51.
1
tan =
RC
= = 100 rad s–1. Using these
L
tan = or L = R tan
R
1 1
tan = or RC =
100 RC 100
R and C

1
49. tan =
RC
BV 1
= R tan
Now C
BV = BH tan =2 10 LCR
–2 2
=2 3 10 1
Z= R2 L
C
e BV lv = 2 3 10 20
3 = R2 100 an 100 ta 2

10 = R = 100
50. LR
I = I0 I – e–Rt/L V 200
I= =
where I0 R 100
at time t is 52.
1 2
U= LI
2
dr r
- I in the wire at a
distance r
when

U0
U= where

1
U0 = LI 02
2

1 2 1 1 2 I0
i.e. LI = LI 0 or I =
2 2 2

I0 1
= I0 e–Rt/L = 1 – e–Rt/L
2 2
1 Rt 1
or e–Rt/L = or – = loge
2 L 2 Fig. 25.67
Electromagnetic Induction and A.C. Circuits 25.35

I b
B= 0 1
2 r = ka xd x = kab2
0
2
dr is
0 Iv dr
de = Bvdr =
2 r

0 Iv r2 dr = 0 Iv
e= log e r
r2
2 r1 r 2 r1

0 Iv r2
or e= loge
2 r1
1
53. W = L I 02 , where I0 I = 2 A. Thus
2 Fig. 25.68

1 2 1 2 1
W= 2.0 56. LI 0 = CV2
2 2 2
joule heating as R
I
54. e = – M C 2 10
t I0 = V 3
=1A
L 0
e I 000 10 10
or M=– =–
I
57. -
= 0.1 H e = – d /dt
-
55. x
58.
width dx and length a at a distance x O, as shown

59. I1 and
b
I2
= Bd A = kx adx
0

II

Multiple Choice Questions with One or More Choices Correct


1.

-
25.36 Comprehensive Physics—JEE Advanced

2. V t
ing.
-
2

6.
current.

is 2 2 A.

7. LCR circuit connect-


L
C F and R .

3.

Fig. 25.69

8. L, C and R

R 1
L RC
4. Two circuits A and B are connected to identical dc
R 1
A -
tance L1 = 10 H and circuit B LC LC
L2
9.
A and B is 1.
A and B

A and B
Fig. 25.70
A and B
At a certain instant the current I -
3
5. t As–1. At that instant, the
Electromagnetic Induction and A.C. Circuits 25.37

L
L
A and B
A and B 14.

10. I
t
I t
Fig. 25.71
15.
0.2
t=1s
I
t=2s P. Then
I I = 100 A
11. P P
L and resistance R 16. A1L and B1M -
V tance w R
and the B
I. The sole-
wire CD -
- v. Neglect the

is now
is I . Then
=2 =
I =2I I I

12. A reatangular coil 20 cm


10–3
-

Fig. 25.72
2 Bv 2
.
R
Bwv
.
R
CD tends to
R.

the wire CD Bwv 2/R.


IIT, 2001
17. An LR
100
L= mH and a resistance R = 10 . A sinusoidal
10
13. V = V0 t
Vrms

state is 20 A.
25.38 Comprehensive Physics—JEE Advanced

20. Two metallic rings


/2. A and B, identical in
t = 0, the current in the circuit is
– 10 2 A. -
T A and B,
t = ,
T
, ..., where T = 0.02 s. identical solenoids as Fig. 25.74

18. A metal rod PQ - I -


v - noids in identical manner, the rings A and B
to heights hA and hB hA > hB. The
B
their masses mA and mB
A > B and mA = mB
A < B and mA = mB
A > B and mA > mB
A < B and mA < mB
the rod
Fig. 25.73
21. A series R-C
-
C
P to Q. IR through the resistor and
VC
19.
I RA IR I RA IR
2
VCA VC VCA VC
IIT, 2011

ANSWERS AND SOLUTIONS


1. v
B. region.
2.
F=e v B
v
B and v B -

- when an iron core is inserted in the coil.


-
tance L
- is inserted in the coil the inductance L increases due
-
creases, causing a decrease in current in the circuit. As
-
3.

-
Electromagnetic Induction and A.C. Circuits 25.39

7.
1
C C is r =
LC
decreases, the reactance will increase and as a re-
1 –1
= /2
10
-

r = r
= =
2 2
4. E L1 = 10 H, L2 = 10 10–3 H and R .
I0 = E/R 2 1/ 2
2 1
Z= R L
C
E 12 = r = 1/ LC
I0 =
R L = 1/ C
Z=R
the current I0 is
1 2
E= LI 0
2 V0 V 2 Vrms
I0 = = 0 =
For circuit 1, Z R R
1 1
E1 = L1I 02 = 10 2
2 200
2 2 = 2 A.
10–1
For circuit 2, The rms current in the circuit is
1 1 Vrms 200
E2 = L2 I02 = 10 10–3 2
Irms = =
2 2 R
10 L is
E1 = Irms L
= 1000 r
E2
I0 is I20 R. Since I0 and
R C is
1
= Irms
P = I02R 2
rC

1
5. V t
10
with V = V0 sin t V0
= 100 rad s–1 or 2 = 100 or R is
V 200 = Irms R
Also Vrms = 0 = = 100 2
2 2
8.
6. Vrms 2 1
L and
C
100 2
= 2 A. resistance, where =2 .
9.
2 2
Irms = = 2 A.
2 dI
VA – I R + E – L = VB
dt
25.40 Comprehensive Physics—JEE Advanced

dI I 10 000
or VB – VA = – IR + E – L Ip = =
dt ep 200
dI
Since I is decreasing with t,
Hence dt
Is =
VB – VA 10–3 3
es

=
1000
dI
10. e L
dt 14.
–3 d –3
10 t 10
dt
Power P = eI 10–3 t
–3
Power at t 10
Power at t 10–3 15. -

L V
11. Time constant = I=
R R 220 210
I=
0.2
L
R

16. CD is made to slide on wires A1L and


L R B 1M
L = and R =

L L/ L
The new time constant = = = = . e
d
=
d
BA B
dA
R R/ R
dt dt dt
V V CD dx is time dt, then
I = = I.
R R A = wdx w = CD
d dx
e B wdx Bw = Bwv
dt dt
12. A = 20 cm 10 cm = 200 cm2 = 200 10 m2.
1200 The induced current is
N = 100, B 10–3 = e B wv
I= =
=2 rad s–1. The R R
CD.
e = e0 sin I
CD
where E0 = NA B and = angle which the normal
the wire CD CD
w
B wv B 2 w2 v
F = BIw = B w=
Ns 2000 R R
13. es = ep = 200
Np CD through a small
distance dx in time dt is
dW = Fdx
Electromagnetic Induction and A.C. Circuits 25.41

P
-
dW dx
P= =F = Fv
dt dt the direction Q to P. Thus the correct choices are

19. From = LI L
B 2 w2 v 2
P= e L
dI
L is
R dt

17. reactance XL = L= 2 L 1 2
From U = LI L is
100 3 2
=2 10 = 10 - 2

ance is
Z XL2 + R2 1/2 2
+ 102 1/2
= 10 2
20. Since the rings are identical and the solenoids are
B = 0nI and hence mag-
V0 Vrms 2 200 2 =B A A
I0 = = 20 A
Z Z 10 2
e hA > hB
A is greater than that on B. Hence,
current induced in A is greater than that in B, i.e.
XL 1 10 1
= tan–1 tan tan e e
R 10 IA > IB RB > RA
RA RB
= . A
Now R = . Hence A < B. Mass mA
l
In an LR than, equal to or greater than mB. Hence the correct

V = V0 t I0 = 20 A and =
current I t as 21. Case A: I RA =
V
=
V
ZA 2
1
I = I0 sin 2 t R2
C

2 t I RA V
or I = 20 sin VCA = =
T C R C 2
I
1 1 Case B: CB = KC C. Hence
where T = = = 0.02 s.
V
IR =
2
At t = 0, I = 20 sin = 10 2 A. 1
R2
C
T T
I = 0 at t = , , IR V
VC = =
C 2
R C
18.
Q to P. As a result a current
P to Q. Also end Q I RA < I R and VCA > VC
25.42 Comprehensive Physics—JEE Advanced

III

Multiple Choice Questions Based on Passage

The Alternating Current Genetator

E = E0 sin t
R is connected across the terminals, a
current I
E E
I= = 0 sin t = I0 sin t
R R
Fig. 25.75
where I0 = E0/R. Such a current is called a.c. or alternating
current. 4. In an a.c. generator
1. A N
B
through the coil is
is used.
NAB

5.
1
NAB
2 E = 100 sin 100 t
3
1 where E t in seconds.
NAB
2

2. t
t

E with time t
3.

SOLUTION
= NAB cos
1.
where
Electromagnetic Induction and A.C. Circuits 25.43

2. E = E0 t+
E = E0
E = 0. Since = t, = 100 or 2 = 100 or
E = 0 at t
t = 0, E = E0 sin , i.e. = =
3. 3 3
4.
magnet is used.
5. .
6.
2 x R
B vd B vd
d R 2 x
1
B vd B vd
R MN 2
m
7.
B vd B vd
induction B
F R 2 x
MN 2 B vd B vd
through R. R 2 x R 2 x
8.
B2d 2 2 x
1
2 m R

B2d 2 R
1
R 2 x

B2d 2 R
log e 1
2 m 2 x

Fig. 25.76 B2d 2 2 x


log e 1
2 m R

SOLUTION
6. R to MN x
B2d 2 v
MN and R is xd and the magnetic F = IBd =
Bxd R 2 x
dv 2 2
m = B d dx
e
d
Bxd Bd
dx
= Bvd
dt R 2 x dt
dt dt
B2d 2 dx
where v or dv =
MN. m R 2 x

7. R and MN
B2d 2
v x
dx
is R + 2 x dv =
0
m 0
R 2 x
e Bvd
I= = 2 2
R 2 x R 2 x B d R 2 x
or v= loge
2 m R
8. Force acting on the rod,
25.44 Comprehensive Physics—JEE Advanced

9. m
downward direction is
B 2 L2 v
g
mR
L. A conducting massless rod
R B 2 L2 v B 2 L2 v
g g
mR mR
m, tied to the other
10.
B g gR

mg R mg R
BL B 2 L2
11. m

g
g
2
g g
3

Fig. 25.77

SOLUTION

9. v x-direction. Since the rod is


x
B T x-direction,
y i.e. T = F = BIL.
induced in the rod is e = B Lv a m
induced current is downward direction, then
e BLv
I= = ma m = mg – T = mg – F
R R
I in magnetic F
L or a =g –
m
F = BIL
Bl L B BL2 v
a =g – =g–
m mR
B 2 L2 v
=g –
mR

10. v t when
a = 0. Putting a = 0 and v = v t
B 2 L2 vt mgR
0= g – or v t = 2 2
mR B L

11.
Fig. 25.78
Electromagnetic Induction and A.C. Circuits 25.45

v=
vt
=
mgR
, B 2 L2 mgR g g
=g– =g– =
2 2 B 2 L2 2mR 2 2
B L 2 2

B 2 L2 vt / 2
a=g–
mR

0M
0M
2
M
0M 0M
v in the x-direction.
a and a a
y-z 14. x=2a
centre at x x 3 0M v 3 0M v
2 2
a 32 a
12. x on
1 0M v 1 0M v
2 2
a a
0M 0M 15. x=2a
2 x 2 x2
3 0Mv 3 0Mv
32 R R
0M 0M
3
2 x 2 x 0Mv 3 0Mv
2R R
13. x=2a

SOLUTION

12. x=2a
distance x l 14.
M d dx d d
e= v
0 Mx dt dt dx dx
B= 2
2 x2 l2
0 Ma 2 v d 1 3 0Ma
2
v
Since x >> l =–
2 d x x3 2 x
0M
B= 3
2 x 3 0 Mv
Putting x = 2 a, we get e = , which is
32 a 2
15.
2
e 3 0Ma v
I=
R 2 x R

Fig. 25.79
M0 = I I a2

13. Due to B 3 0Ma v


=
2 2 x R
0M 0Ma
= BA = B a2 3
a2 3
2 x 2x Putting x = 2 a
25.46 Comprehensive Physics—JEE Advanced

0 IR
10 0 IR 10
R and r = R/100 are 3 0 IR 0 IR
10 10
x= 3 R I1 = 2
2I and I2 = I 18.
16. 2
due to current I1 = 2 I I I
0I 0I
R R 2I
19. M and m
0I 0I
M/m
R 3R is
17. 10
is 2
102
SOLUTION

16. =B
due to current I1
=B r2
2
R
=B
100

0I R2 0 IR
= = 10
R 10

Fig. 25.80
18.
2
0 I1 R
B= 2 2 3/2 M= =
2R x I2 I
Putting I1 = 2 I and x = 3 R 19.
magnetic moment = current
0I
B=
R M = I1 R2 = 2 I R2 I1 = 2I
17. Since r x, the magnetic
2
R IR 2
and m = I2 r2 = I =
B. 100 10

20. LCR circuit is


2
An LCR series circuit with 100 resistance is connected 2
21. The current in the circuit is
2
2 2
22.
Electromagnetic Induction and A.C. Circuits 25.47

SOLUTION

20. 2
1
Z= R2 L
C
2
L = R2 100 tan 100 tan
tan = or L = R tan
R = R = 100
- 21.
V 200
I= =2A
R 100
1
tan =
RC
1 22.
= R tan
C P = I 2R
LCR

23.
wheel is
QR 2
QR2
2
radius R Q QR 2
QR2
2
24.
. x
IIT, 2003 x
y
y
25.
1 1
QR2 B QRL B
2 2
QR2 B BRL B
26.
x
y
z
Fig. 25.81 z
SOLUTION

23. T
x
Q Q
I=
T 2 25. Torque = B = mi Bj
= mB k
Q 1
m=I R2 = R2 = QR2
2 2 1
= B Q R2 k
2
24.
26.
25.48 Comprehensive Physics—JEE Advanced

q 0.

An LCR t
q = q0 cos
LC 2
S1 and S2
t
q = q0 cos
LC 2

d 2q
q = – LC
dt 2

1 d 2q
q=–
LC dt 2
29. t
Fig. 25.82
V, switch S1
and S2 is closed. Then
27. At time t = 0 switch S1 is closed and S2 t

is q0 and RC circuit. t -
Then
t =
t
q = q0/2.
–2 in one direction.
t = 2 , q = q0 e
t = 2 , q = q0 e–1
C
circuit is V.
L
28. At time t
q, switch S1 2 is closed.

SOLUTION
27. In an RC 1 d 2q d 2q
at a time t q= – 2
= – LC
dt 2 dt 2
–t/
q = q0 e
where = RC is the time constant. At t = 2 , we 29. At t

t
q = q0 e–2t/ q0 e–2
q t as
28. 2 is closed and S1 1
oscillates in the LC q = q0 cos t, where =
LC
1
= dq d
LC I= q0 cos t q0sin t
dt dt
Now q 0 at t which is alternating and not unidirectional. The
wrong. The charge q t as
q = q0 t+ 1
I = q0 = CV q0 = CV
where is not equal to LC
t, we get C
d 2q = V
2 2
=– q0 t+ q L
dt 2
Electromagnetic Induction and A.C. Circuits 25.49

30.
to

31.

,
32.

train
-
,
s law.

SOLUTION
30. 32.
31.

IV

Assertion-Reason Type Questions

one
correct.
-
with time.
-
-
ment-1.

-
tor. changes at a constant rate.
25.50 Comprehensive Physics—JEE Advanced

9.

than g

10.
Three identical coils A, B and C
A and C

A B, with coils B and C


B

-
tance decreases.
Fig. 25.84

11.
-

A to B
12.
AB x-
B

Fig. 25.83

Fig. 25.85
Electromagnetic Induction and A.C. Circuits 25.51

z
A

y A to B.
rod goes through a conduct-
ing ring as shown in Fig.
13.

than g - Fig. 25.86


rent is induced in the ring

SOLUTIONS
1. v 8.
B . Since the

F =e v = B v 9.
to B

2.

changes.
3. -

10. Since coil C B, the cur-


- rent in C
B A B, then according
region. B -
4. -
ment B
11.

5. -

12. -
inserted in the coil.
13.
6.
in the coil, its inductance L increases. Hence its
reactance L increases, causing a decrease in the

14.
7.
C C is decreased, the
reactance will increase and as a result the current
in the circuit is decreased causing a decrease in the
Statement-1.
25.52 Comprehensive Physics—JEE Advanced

Integer Answer Type

1.
each other, and the ground, are connected to a mil-
–1
along

10

that when the


2.
line AD into two regions.
R1 and
R2
B
Find the terminal
L
–1
ACD is Fig. 25.88
a semicircular conduct-
r = 10
cm with centre at O, the 5.
Fig. 25.87 F
and the resulting LC circuit is set oscillating at its
- Q denote the instantaneous
–1
I the current in the
through O
R= charge Q is 200 I

3. An LCR series circuit with 100 resistance is 6. 10–3 m2

LCR circuit. through the coil.


IIT, 2000
4. 7. A series R-C AC

R1 and R2 as shown Fig. R-C circuit is R 1.2 , the


L

SOLUTIONS
–1 –1
1. v . The induced e = Blv = 0.2 10 1.0 10–3
Electromagnetic Induction and A.C. Circuits 25.53

1 P1 P2
2. e Br2 -
2
I1 .
2
e Br I2 1.2 30
I=
R 2R
. 0.1 2
= =2A
2 0.
3.

L
tan = or L = R tan
R

1
tan =
RC
1
= R tan
C Fig. 25.89
LCR
1 2
I = I1 + I2. Thus I= A
Z= R2 L
C
30I1 I1
2 2
= I1 + =
= R 100 tan 100 tan
= R = 100 I1 = A. Hence

30
V 200 I2 = I – I1 = = 2 A.
I= =2A
R 100
4. P1 .
e=
I1 /
BId = mg vt is
mg 0.2 . induced in it is
or I= A e .
Bd . 1 e = Bvt d or vt = = 1 ms–1
Bd . 1
Current I R1 and
R2 I1 and I2 R1 and R2, then 5. The charge Q
the current I
I = I1 + I2
e R1 1
and R2 =
LC
e = I 1R 1 = I 2R 2
1
R1 and R2 are = 1/2
P1 = eI1 2.0 10 . 10
–1
and P2 = eI2 = 10 rad s
Charge Q t at angular
P1 I1 . Since at t = 0, Q = Q0
P2 I2
25.54 Comprehensive Physics—JEE Advanced

Q = Q0 cos t
1
7. Z= R2 2
dQ d C
Current I = = Q0 t
dt dt
1
= – Q0 sin t Z2 = R 2 2
C2
I = Q0 = 10 10
2 1
1.2 R = R2 + 2
dB . –1 C2
6.
dt 0.2 2
= NAB R=
C
d dB 2 2
e= = NA Time constant = RC = = 10–3s
dt dt
=
= 100 10–3
e 2
Induced current I =
R .
26
Chapter
Ray Optics and Optical
Instruments

REVIEW OF BASIC CONCEPTS REFLECTION AT A SPHERICAL SURFACE


26.2 (CONCAVE MIRROR OR CONVEX
26.1 REFLECTION OF LIGHT
MIRROR)
(i)

-
-
(ii)

1 1 1 R
;f
(in v f
v f
R
360 360 f f
1 if
h v f
360 360 h0
1 if
f v
f

360 360 ge
if

-
26.2 Comprehensive Physics—JEE Advanced

26.3 SNELL’S LAW OF REFRACTION

26.4 REFRACTIVE INDEX AND SPEED OF LIGHT

Fig. 26.2
v1
1 v
OA h AI h h
REFRACTION OF LIGHT THROUGH A OI h h h h h
26.5
PLANE SLAB
26.7 TOTAL INTERNAL REFLECTION

)
1 (<

1 Fig. 26.3

Fig. 26.1
1

)
x
26.8 REFRACTION THROUGH A PRISM
26.6 REAL AND APPARENT DEPTH (OR HEIGHT) -

OA d AI d
d 1 + A
d d 1 1 -
OI d d d d 1 ; -
A
Fig. 26.4
26.3

A 1 +
1 1 v R
A Application: Image formed by a transparent sphere
n
O
1
A R
n
I O P

1 1 1
1 v R
A ( R
n I
Q I Q
A
n
1 1
v R
1 1 A v R
A ( R
O
26.9 REFRACTION AT A SPHERICAL SURFACE

v
v R
R Fig. 26.6
h1 v
1 1
h0
v R
1
R
1 1 ( R
v R O
1 1
1 v R
1 ( R

Fig. 26.5
26.4 Comprehensive Physics—JEE Advanced

1 1 1
f R R

v
Applications of lens maker’s formula
Fig. 26.7
R1 R R R
26.10 REFRACTION THROUGH A LENS 1 1 1
v f f R R

Fig. 26.9
1 1 1 R
f f
v
h v f v
h f R1 R R R
f 1
f R
f - f
R1 R) Fig. 26.10
R1 R1 R R
f -
1 1 1
1 ;f
f R R
1 1 1 R1 R)
f 1 R1 R R1 R1 R R
R1 1 1 1
;f
f R R
R

1
f 3 R1 R
Fig. 26.11

R1 R R
1 1 1 ( 1)
f R R
R1 R R
1 1 1 ( 1)
Fig. 26.8 f R R
1 f
P
f (in e)
26.5

1
P
f (in e)
P D)

P
Fig. 26.12 f (in e)

R1 R R f
1 1 1 ( 1) 26.12 CO-AXIAL COMBINATION OF LENSES
f R R
f1 f
R1 R R
1 1 1 ( 1) F
f R R
1 1 1
f
- F f f
P P1 + P
R1 R1 R R d
1 1 1 1 1 1 d
f R R F f f f f
Fig. 26.13 P P1 + P P1 P d
R > R1 f
- EFFECT OF SILVERING ONE OF THE
26.13
REFRACTING FACES OF A LENS
R1 R1 R R
1 1 1
f R R
Fig. 26.14
R > R1 f
f F
AB
f

Fig. 26.16

1 1
Fig. 26.15
F f
CD
f
f

26.11 POWER OF A LENS


1 1 1 1
F f f f f f
26.6 Comprehensive Physics—JEE Advanced

f f
26.1
R
R
1

1
F f f
SOLUTION
1 1 1
f R R
R
f

1 1 1
F R R R
R1 R R
1 ( 1)
F R R R

R
F
( 1) Fig. 26.19

(R1 R R ) 1 1 1
AI
1
1
F f f

( 1)
f
R Fig. 26.17
26.2
f
AB
f R
F
( )

R1 R R
1
F f f
1 ( 1)
f R
R
f
Fig. 26.18

1 ( ) Fig. 26.20
F R R R
SOLUTION
R
F AC CB
26.7

SOLUTION
1 1
n n
1

26.3 >

26.5

SOLUTION

a
a v a
SOLUTION
1 1 1 P1 P -
v f
P P1 + P
1 1 1
or a F
a a 30
F

26.6

v
d -
1 1 1
30 d

SOLUTION

26.4 1 1 1
+ (i)
v f
-

1 > > (b) 1 < < 1


or
1 < 1 > v f f f

or ( f f
1
or (ii)

v
v v v
1+ or 1 +
f f
or v f (1 + ) (iii)
Fig. 26.21
+v d
26.8 Comprehensive Physics—JEE Advanced

1
d + f (1 + )

f
1

26.7
f

f
Fig. 26.23
SOLUTION
1 1 1
26.9
v f
3
1 v A -
or A A
v f f
1 SOLUTION
or f
3 A
A A A
26.8 n n
nA
PQR - A A A
PR n n n
PQ
PR A A
A
A
n

A A 3
3

or A A

26.10

Fig. 26.22 3 -

SOLUTION
SOLUTION
AB BC A
n
PR PBC A
A
n
3
n 1
n 30
n
3
n 30
26.9

3 26.11
30 3

26.14 COMPOUND MICROSCOPE

SOLUTION
f

M
1 1 1
0 e v
f
1 1 1 1 1
v0 D v 10
|M 1
0 fe
v -
0
v0 v
fe u
D

v0 D
|M
0 fe

D
26.15 TELESCOPE M
x
x
x
D

M
f0 -
|M
fe
f0 fe

L f0 + fe
D
v D f

f0 fe 1 1 1
|M 1
fe D v
26.10 Comprehensive Physics—JEE Advanced

1 1 1 26.14
or
u 10 -
x -

D
v
/
SOLUTION
M fo
D D
M 1 1 fe
x f 10
D
M o e o 1 o 1
fe
26.12
o
- -
o
e
-
Position of object
AB
AB
AB
SOLUTION
fo fe v
o v

1 1 1
v o 1 1 1
fo v 0
1 1 1
1 1 1
v
v
AB
v D -
M 1 1 AB
fe 6
v
-

26.13
v

SOLUTION
M fe D
D
M o e o 1 o 1 v
fe
o Fig. 26.24
26.11

Positioning of eyepiece fo
M
fe
e A B
AB v D
D -
v
o v D fe
fo
M
fe D
1 1 1 1 1 1
or 26.16 DISPERSION OF LIGHT
v e 6

AB

OE OB + B E v+|
-

n
n
26.15
-

SOLUTION

fo + fe
fo + fe
fo
M fo fe (ii)
fe Angular Dispersion and Dispersion Power
fe fo
V R
A
26.16

V R
-
V V A R R A V
R

V R)A
1
A ( V + R
SOLUTION
fo fe
26.12 Comprehensive Physics—JEE Advanced

( V R )A V R
( 1) A 1

Dispersion without Deviation and Deviation without


Dispersion f1 f d f1 f

(ii) Chromatic aberration


V R)A +( V R)A
A+( A -

( A+( A fR > fY > fV

( V R)A +( V R)A

Fig. 26.25
Fig. 26.27
26.17 ABERRATIONS

(i) Spherical aberration

f
f
Fig. 26.28

d 1 (f + f

Fig. 26.26

Multiple Choice Questions with Only One Choice Correct


1. x
y x
+x
i j
(b)
+x
Ray Optics and Optical Instruments 26.13

(c) 4 ms–1 along the – x axis light parallel to the common principal axis is inci-
(d) 4 ms–1 along the + x axis dent on lens A as shown in Fig. 26.30. What should
2. A plane mirror is made of glass of thickness 3 cm be the value of d so that the ray emerges from lens
and refractive index 1.5 by silvering one of its fac- B
es. A point object is placed a distance of 6 cm in (a) 50 cm (b) 40 cm
front of the unsilvered face. The distance of the (c) 25 cm (d) 10 cm
image from the silvered face is
(a) 6 cm (b) 7 cm
(c) 12 cm (d) 15 cm
3. The image distance (v) is plotted against the object
distance (u) for a concave mirror of focal length f.
Which of the graphs shown in Fig. 26.29 represents
the variation of v versus u as u is varied from zero

v v Fig. 26.30

7. A convex lens has a diameter d and focal length f. A


point object is placed on its principal axis at a dis-
tance 3f from it. The eye of an observer is placed at
a distance of 3f on the other side of the lens and at a
distance h below the principal axis. The maximum
value of h so that the observer can see the image of
the object is
2d
(a) d (b)
v v 3
d d
(c) (d)
2 3
8. A glass prism ABC of refractive index 1.5 is
immersed in water of refractive index 4/3 as shown
in Fig. 26.31. A ray of light incident normally on
face AB AC if
(a) sin 8/9
(b) sin 2/3
Fig. 26.29
(c) sin = 3 / 2
4. A hollow thin convex lens made of glass is placed (d) 2 / 3 < sin < 8 / 9
in air. It will behave like a
IIT, 1981
(a) convex lens (b) concave lens
(c) prism (d) glass plate
5. Two thin equi-convex lenses each of focal length
3
f and made of glass are placed in con-
2
g

4
. The focal length of the combination is
3
f 2f
(a) (b)
2 3
3f 4f Fig. 26.31
(c) (d)
5 7 9. What is the relation between refractive indices ,
6. Two lenses A and B of focal lengths 30 cm and 20 1 and 2 if the behaviour of light rays is as shown
cm are placed co-axially a distance d apart. A ray of in Fig. 26.32.
26.14 Comprehensive Physics—JEE Advanced

(a) > 2 > 1 (b) < 2 < 1 13. The principal section of a glass prism is an isos-
(c) < celes triangle ABC with AB = AC. The face AC is
2; = 1 (d) 2 < 1; = 2
silvered. A ray incident normally on face AB, after
BC in a
direction perpendicular to it. What is the BAC of

(a) 30° (b) 36°


(c) 60° (d) 72°
14. An object is placed at a distance of 10 cm from
a co-axial combination of two lenses A and B in
contact. The combination forms a real image three
Fig. 26.32 times the size of the object. If lens B is concave
10. A lens forms a sharp image on a screen. On insert- with a focal length of 30 cm, what is the nature and
ing a parallel sided glass slab between the lens and focal length of lens A
the screen, it is found necessary to move the screen (a) Convex, 12 cm (b) Concave, 12 cm
a distance d away from the lens in order to focus (c) Convex, 6 cm (d) Convex, 18 cm
the image sharply. If the refractive index of glass 15. A plano-convex lens acts like a concave mirror
relative to air is , the thickness of the glass slab is of 28 cm focal length when its plane surface is
given by silvered and like a concave mirror of 10 cm focal
length when its curved surface is silvered. What is
(a) d/ (b) d
d 1
(c) (d) 1 d (a) 1.50 (b) 1.55
1 (c) 1.60 (d) 1.65
1
16. A pin is placed 10 cm in front of a convex lens of
11. A convex lens is placed between an object and a focal length 20 cm and refractive index 1.5. The
surface of the lens farther away from the pin is
- silvered and has a radius of curvature of 22 cm.
age obtained on the screen is m1. When the lens
is moved by a distance d (a) 10 cm (b) 11 cm
image obtained on the same screen is m2. The focal (c) 12 cm (d) 13 cm
length of the lens is (m1 > m2). 17. Two glasses have dispersive powers in the ratio of
d d 2 : 3. These glasses are used in the manufacture
(a) (b) of an achromatic objective of focal length 20 cm.
m1 m2 m1 m2
What are the focal lengths of the two lenses of the
m1 m2
(c) d (d) d
m2 m1 (a) 6.67 cm, – 10 cm (b) 7.5 cm, – 12.5 cm
(c) 9.67 cm, – 15 cm (d) 12.5 cm, – 20 cm
12. A ray of light, travelling in a medium of refrac- 18. A giant telescope in an observatory has an objec-
tive index , is incident at an angle i on a compos- tive of focal length 19 m and an eye-piece of focal
ite transparent plate consisting of three plates of length 1.0 cm. In normal adjustment, the telescope
refractive indices 1, 2 and 3. The ray emerges is used to view the moon. What is the diameter of
from the composite plate into a medium of refrac-
tive index , at angle x. Then The diameter of the moon is 3.5 106 m and the
(a) sin x = sin i radius of the lunar orbit round the earth is 3.8 108 m.
(a) 10 cm (b) 12.5 cm
(b) sin x = sin i
(c) 15 cm (d) 17.5 cm
19. Two convex lenses of focal lengths f1 and f2 are
(c) sin x = sin i separated co-axially by a distance d. The power of
the combination will be zero if
1 3
(d) sin x = sin i f1 f2 f1 f2
2 2
(a) d = (b) d =
2 2
Ray Optics and Optical Instruments 26.15

(c) d = f1 + f2 (d) d = f1 f 2 1
(c) cos A (d) tan A
2
20. A thin lens of focal length f has a aperture of
26. When a ray of light enters a glass slab from air,
diameter d. It forms an image of intensity I. Now,
(a) its wavelength decreases
the central part of the aperture upto diameter d/2
(b) its wavelength increases
is blocked by opaque paper. The focal length and
(c) its frequency increases
the image intensity will change to
(d) neither wavelength nor frequency changes
f I I
(a) , (b) f, IIT, 1980
2 2 4 27. A beam of light consisting or red, green and blue
3f I 3I colours is incident on a right-angled prism ABC as
(c) , (d) f ,
4 2 4 shown in Fig. 26.33. The refractive indices of the
21. The diameter of a plano-convex lens is 6 cm and material of the prism for red, green and blue wave-
the thickness at the centre is 3 mm. If the speed of lengths, respectively are 1.39, 1.44 and 1.47. The
light in the material of the lens is 2 108 ms–1, the prism will
focal length of the lens is
(a) 15 cm (b) 20 cm
(c) 30 cm (d) 10 cm
22. A thin prism P1 with angle 4° and made from glass
of refractive index 1.54 is combined with another
thin prism P2 made from glass of refractive index
1.72 to produce dispersion without deviation. The
angle of prism P2 is
(a) 5.33° (b) 4°
(c) 3° (d) 2.6°
IIT, 1990
23. A ray is incident at an angle of incidence i on one
face of a prism of a small angle A and emerges Fig. 26.33
normally from the opposite face. If the refractive (a) separate part of the red colour from green
index of the prism is , the angle of incidence i is and blue colours
nearly equal to (b) separate part of the blue colour from red and
A A green colours
(a) (b) (c) separate part of three colours from one
2
another
A (d) not separate even partially any colour from
(c) A (d)
2 the other two colours.
24. A converging lens is used to form an image on a IIT, 1989
screen. When the upper half of the lens is covered 28. Spherical aberration in a thin lens can be reduced by
by an opaque screen, (a) using a monochromasstic light
(a) half the image will disappear (b) using a doublet combination
(b) complete image will be formed (c) using a circular annular mask over the lens
(c) intensity of the image will increase (d) increasing the size of the lens.
(d) none of these IIT, 1994
IIT, 1986 29. A real image of a distant object is formed by a
25. The angle of a prism is A and one of its refracting plano-convex lens on its principal axis. The spheri-
surfaces is silvered. Light rays falling at an angle cal aberration
of incidence 2 A (a) is absent
(b) is smaller if the curved surfaces of the lens
the second (silvered) surface. The refractive of the faces the object
material of the prism is (c) is smaller if the plane surface of the lens
(a) 2 sin A (b) 2 cos A faces the object
26.16 Comprehensive Physics—JEE Advanced

(d) is the same whichever side of the lens faces of two liquids L1 or L2 having refractive indices
the object n1 and n2 respectively (n2 > n1 > 1). The lens will
IIT, 1998
30. A concave mirror is placed on a horizontal table, (a) air and placed in air
with its axis directed vertically upwards. Let O be (b) air and immersed in L1
the pole of the mirror and C its centre of curvature. (c) L1 and immersed in L2
A point object is placed at C. It has a real image, (d) L2 and immersed in L1.
also located at C IIT, 2000
water, the image will be 36. A diverging beam of light from a point source S
(a) real and will remain at C having divergence angle , falls symmetrically on
(b) real and located at a point between C and a glass slab as shown in Fig. 26.34. The angles of
incidence for the two extreme rays are equal. If the
(c) virtual and located at a point between C thickness of the slab is t and refractive index is n,
and O then the divergence angle of the emergent beam is
(d) real and located at a point between C and O. (a) zero (b)
IIT, 1998 –1 1 1
(c) sin (d) 2 sin–1
31. A spherical surface of radius of curvature R n n
separates air (refractive index 1.0) from glass IIT, 2000
(refractive index 1.5). The centre of curvature is in
the glass. A point object P placed in air is found to
have a real image Q in the glass. The line PQ cuts
the surface at point O and PO = OQ. The distance
PO is equal to
(a) 5R (b) 3R
(c) 2R (d) 1.5R
IIT, 1998
32. An eye specialist prescribes spectacles having a
combination of a convex lens of focal length 40
cm in contact with a concave lens of focal length
25 cm. The power of this lens combination is Fig. 26.34
(a) + 1.5 D (b) – 1.5 D
(c) + 6.67 D (d) – 6.67 D 37. A rectangular glass slab ABCD of refractive in-
IIT, 1997 dex n1 is immersed in water of refractive index n2
33. A concave lens of glass, refractive index 1.5, has (n1 > n2). A ray of light is incident at the face AB
both surfaces of the same radius of curvature R. On of the slab as shown in Fig. 26.35. The maximum
immersion in a medium of refractive index 1.75, it value of the angle of incidence max, such that
will behave as a the ray comes out only from the other side CD is
(a) convergent lens of focal length 3.5 R given by
(b) convergent lens of focal length 3.0 R n1 n2
1
(c) divergent lens of focal length 3.5 R (a) sin–1 cos sin
n2 n1
(d) divergent lens of focal length 3.0 R
IIT, 1999 1 1
34. In a compound microscope, the intermediate image is (b) sin–1 n1 cos sin
n2
n1
(c) sin–1
n2
(d) virtual, erect and reduced.
IIT, 2000 n2
(d) sin–1
35. A hollow double concave lens is made of a very thin n1
IIT, 2000
Ray Optics and Optical Instruments 26.17

41. A plano convex lens of refractive index 1.5 and


radius of curvature 30 cm is silvered at the curved
surface. Now this lens has been used to form the
image of an object. At what distance from this lens
an object be placed in order to have a real image of

(a) 20 cm (b) 30 cm
(c) 60 cm (d) 80 cm
42. The two lenses of an achromatic doublet should
Fig. 26.35 have
(a) equal powers
38. When a glass prism of refracting angle 60° is im- (b) equal dispersive powers
mersed in a liquid, its angle of minimum deviation (c) equal ratio of their power and dispersive
is 30°. The critical angle of glass with respect to the power
liquid medium is (d) equal product of their power and dispersive
(a) 42° (b) 45° power.
(c) 50° (d) 52° 43. Rays from the sun subtend an angle (in radians) at
39. In the visible region, the dispersive powers and the the pole of a concave mirror of focal length f. If the
diameter of the sun is D, the diameter of the image
prism are and and d and d respectively. When of the sun formed by the mirror is
the two prisms are combined, the condition of zero (a) D (b) 2 D
dispersion by the combination is (c) f (d) 2 f
(a) d d =0 44. A point source of light S is placed at a distance L in
front of the centre of a plane mirror PQ of width d
(b) d+ d =0 hung vertically on a wall as shown in Fig. 26.37. A
(c) d+ d =0 man walks in front of the mirror along a line paral-
2
(d) ( d) + ( d )2 = 0 lel to the mirror at a distance 2L from it as shown.
The greatest distance over which he can see the
40. A ray of light is incident normally on one face of a image of the light source in the mirror is
d
glass-air interface as shown in Fig. 26.36. If the (a) (b) d
2
of the prism is n, then (c) 2d (d) 3d
1 IIT, 2000
(a) n < (b) n > 2
2
1
(c) n > (d) n < 2
2

Fig. 26.37
45. A thin glass prism of refractive index 1.5 produces
45° a deviation of 4° of a ray incident at a small angle.
What will be the deviation of the same incident
ray by the same prism if it is immersed in water of

45° (a) 1° (b) 2°


(c) 8° (d) 16°
46. An air bubble in a glass slab ( = 1.5) is 5 cm deep
Fig. 26.36 when viewed through one face and 2 cm deep when
26.18 Comprehensive Physics—JEE Advanced

viewed through the opposite face. What is the thick-

(a) 7.0 cm (b) 7.5 cm


(c) 10.0 cm (d) 10.5 cm
47. The refracting angle of a prism is A and the re-
fractive index is cot (A/2). The angle of minimum
deviation is
(a) 180° – A (b) 180° – 2A
(c) 180° – 3A (d) 180° – 4A
48. A ray of light passes through four transparent
media with refractive indices 1, 2, 3 and 4 as
shown in Fig. 26.38. The surfaces of all media are
parallel. If the emergent ray CD is parallel to the
incident ray AB, we must have

Fig. 26.40

Fig. 26.38
51. Which one of the spherical lenses shown in

(a) 1 = 2 (b) 2 = 3 curvature of the surfaces of the lenses are as given


(c) = (d) = in the diagrams.
3 4 4 1
IIT, 2002
IIT, 2001
49. A given ray of light suffers minimum deviation in
an equilateral prism P. Additional prisms Q and R of
identical shape and of the same material as P are now
added as shown in Fig. 26.39. The ray will now suffer

Fig. 26.39
(a) greater deviation
(b) no deviation
(c) same deviation as before

50. An observer can see through a pin-hole the top Fig. 26.41
end of a thin rod of height h, placed as shown in 52. Two plane mirrors A and B are aligned parallel to
Fig. 26.40. The beaker height is 3h and its radius h. each other, as shown in Fig. 26.42. A light ray is
incident at an angle of 30° at a point just inside one
2h, he can see the lower end of the rod. Then the end of A. The plane of incidence coincides with the
refractive index of the liquid is
(a) 5/2 The maximum number of times the ray undergoes
(b) 5/ 2 -
es out is
(c) 3/ 2 (a) 28 (b) 30
(c) 32 (d) 34
(d) 3/2
IIT, 2002 IIT, 2002
Ray Optics and Optical Instruments 26.19

f u f
(a) b (b) b
u f f
2 2
f u f
(c) b (d) b
u f f
57. A point source of light S is placed at the bottom of
a vessal containing a liquid of refractive index 2 .
Fig. 26.42 A person is viewing the source from above the sur-
53. The size of the image of an object, which is at in- face. There is an opaque disc of radius r
on the surface. The centre of the disc is vertically
30 cm is 1.6 cm. If a concave lens of focal length above the source S. The liquid is gradually drained
20 cm is placed between the convex lens and the out from the vessel through a tap. The maximum
image at a distance at 26 cm from the convex lens, height of the liquid for which the source cannot be
seen from above is
(a) 0.8 cm (b) 1.2 cm (a) r (b) 2r
(c) 2.0 cm (d) 2.4 cm (c) 3r (d) 2r
IIT, 2003 58. Light is incident at an angle on one planar end
54. A ray of light is incident at the glass-water interface of a transparent cylindrical rod of refractive index
at an angle i - n. The least value of n so that the light entering the
face of water as shown in the Fig. 26.43. The value rod does not emerge from the curved suface of the
of g would be rod for any value of is
4 1 4
(a) sin i (b) (a) (b) 2
3 sin i 3
2 (c) 1.5 (d) 3
(c) (d) 1.5
3 sin i 59. A ray of light is incident at an angle of 60° on one
IIT, 2003 face of a prism of refracting angle 30°. The ray
emerges out of the prism making an angle of 30°
with the incident ray. The refractive index of the
material of the prism is
(a) 2 (b) 1.5
1
(c) 3 (d) 1 3
2
60. A square wire of side 3.0 m is placed 25 cm from
a concave mirror of focal length 10 cm. The area
Fig. 26.43
enclosed by the image of the wire is
55.
(a) 1 cm2 (b) 4 cm2
of focal length 20 cm. A second motor car 2.8 m
(c) 16 cm2 (d) 25 cm2
of 15 ms–1. The speed of the image of the second 61. A ray of light PQ is incident on an isosceles glass
prims placed on a horizontal table. If the prism
1 is in the minimum deviation position for the ray
(a) ms 1 (b) 1 ms 1 PQ
10 15
26.44).
(c) 10 ms–1 (d) 15 ms–1
IIT, 2004
56. A short linear object of length b lies on the axis of a
concave mirror of focal length f at a distance u from (a) = (b) >
the pole. The lenght of the image will be
(c) < (d) + = 90°
26.20 Comprehensive Physics—JEE Advanced

Fig. 26.44

62. White light is incident on face AB of a glass prism. Fig. 26.46


The path of the green component is shown in the 65. A convex lens and a concave lens are placed in
contact. The ratio of the magnitude of the power of
AC as shown in Fig. 26.45 the light the convex lens to that of the concave lens is 4 : 3.
emerging from face AC will contain If the focal length of the convex lens is 12 cm, the
(a) yellow, orange and red colours focal length of the combination will be
(b) violet, indigo and blue colours (a) 16 cm (b) 24 cm
(c) all colours (c) 32 cm (d) 48 cm
(d) all colours except green. IIT, 2005
IIT, 2004 66. A right-angled prism is to be made by selecting
a proper material and angles A and B (B A), as
shown in Fig. 26.47. It is desired that a ray of light
incident on face AB emerges parallel to the incident

be the minimum refractive index n for this to be

IIT, 1987

Fig. 26.45

63. A point object is placed at the centre of a glass


sphere of diameter 12 cm and refractive index 1.5.
What is the distance of the virtual image from the
Fig. 26.47
(a) 4 cm (b) 6 cm
(c) 9 cm (d) 12 cm 1
(a) nmin =
IIT, 2004 sin A
64. A beaker contains water (
1
(b) nmin =
surface of water as shown in Fig. 26.46. An object sin B
is placed at the bottom of the beaker and its image sin A
is formed 14 cm below the surface of water. The (c) nmin =
sin B
focal length of the mirror is
(a) 8 cm (b) 12 cm (d) nmin = sin A sin B
(c) 16 cm (d) 20 cm 67. A plano-convex lens has thickness 4 cm. When
IIT, 2005 placed on a horizontal table with the curved face in
Ray Optics and Optical Instruments 26.21

contact with it, the apparent depth of the bottom- Fig. 26.50. A ray
most point of the lens is found to be 3 cm. If the of light is incident
lens is inverted such that the plane face is in contact on the prism at a
with the table, the apparent depth of the centre of small height h. The
25 focal length of this
the plane face of the lens is found to be cm. The
focal length of the lens is 8 crude converging
lens is
IIT, 1984
h
(a) 25 cm (b) 50 cm (a) f =
Fig. 26.50
(c) 75 cm (d) 100 cm
68. Figure 26.48 shows a lens having radii of curvature h
R1 and R2 and 1 < 2 < 3. If the thickness of the (b) f =
( 1)
lens is negligible and R1 = R2 = R, the focal length
of the lens will be h h
(c) f = (d) f =
( 1)
IIT, 2005
71. A ray of light is incident at an angle of 45° on a
square slab of a transparent material. What should
be the refractive index of the material of the slab

face AB
Fig. 26.48

3R 2R
(a) f = (b) f =
( 3 1) ( 3 1)

1R ( 2 1)R
(c) f = (d) f =
( 3 2) ( 3 1)

IIT, 2003
69. A parallel sides slab ABCD of refractive index
2 is sandwiched between two slabs of refractive
indices 2 and 3 as shown in the Fig. 26.49.
The minimum value of angle such that the ray PQ
Fig. 26.51

AB and CD is (a) (b) 3


2
IIT, 2005 2
(a) 30° (b) 45°
(c) 60° (d) 75° (c) 1.5 5 (d)
3
72. Parallel rays from a distant object fall on a solid
transparent sphere of radius R and refractive index
. The distance of the image from the sphere is
R(2 ) R
(a) (b)
2( 1) ( 1)
R
(c) (d) R ( – 1)
( 1)
Fig. 26.49
73. An object O is placed at a distance of 20 cm from a
70. Two identical thin isosceles prisms of refracting an- thin plano-convex lens of focal length 15 cm. The
gle (in radian) and refractive index are placed plane surface of the lens is silvered as shown in Fig.
with their bases touching each other as shown in 26.52. The image is formed at a distance of
26.22 Comprehensive Physics—JEE Advanced

distance x from the pole P. The student looks at the


pin and its inverted image from a distance keeping
his/her eye in line with PA. When the student shifts
his/her eye towards left, the image appears to the
right of the object pin. Then,
(a) x < f (b) f < x < 2f
(c) x = 2f (d) x > 2f
IIT, 2007
Fig. 26.52 77. A ray of light travelling in water is incident on its
(a) 60 cm to the right of the lens surface open to air. The angle of incidence is ,
which is less than the critical angle. Then there will
(b) 30 cm to the left of the lens
be (Fig. 26.54)
(c) 24 cm to the right of the lens
(d) 12 cm to the left of the lens
IIT, 2006
between them would be less than 180° – 2
74. Figure 26.53 shows the graph between the image
distance v (in cm) and the object distance u (in cm)
between them would be greater than 180° – 2
for a thin convex lens. The focal length of the lens is
IIT, 2007

Fig. 26.54
Fig. 26.53 78. Two beams of red and violet colours are made to
(a) (5.00 ± 0.05) cm (b) (5.00 ± 0.10) cm pass separately through a prism (angle of the prism
(c) (10.0 ± 0.10) cm (d) (0.50 ± 0.05) cm is 60°). In the position of minimum deviation, the
IIT, 2006 angle of refraction will be
75. Image of the sun is formed by a biconvex lens of (a) 30° for both the colours
focal length f. The image is a circular patch of (b) greater for the violet colour
radius r and is formed on the focal plane of the lens. (c) greater for the red colour
Choose the correct statement from the following. (d) equal but not 30° for both the colours
(a) The area of the image is r 2 and it is directly IIT, 2008
proportional to f. 79. A light beam is traveling from Region I to Region
(b) The area of the image is r 2 and it is directly IV (Refer to Fig. 26.55). The refractive indices in
proportional to f 2. n n n
Regions I, II, III and IV are n0, 0 , 0 and 0 ,
(c) The intensity of the image will increase if f 2 6 8
is increased. respectively. The angle of incidence for which the
(d) If the lower half of the lens is covered with beam just misses entering region IV is
black paper, the area of the image will be-
come half. 3 1
(a) sin–1 (b) sin–1
IIT, 2006 4 8
76. In an experiment to determine the focal length (f) 1 1
of a concave mirror by the u – v method, a student (c) sin–1 (d) sin–1
4 3
places the object pin A on the principal axis at a
Ray Optics and Optical Instruments 26.23

(c)

Fig. 26.55
IIT, 2008
80. A light ray travelling in glass medium is incident on
glass-air interface at an angle of incidence . The
R) and transmitted (T) intensities, both as
function of , are plotted. The correct sketch is
(d)
[refer to Fig. 26.56]

(a)
Fig. 26.56

IIT, 2011

(b)

ANSWERS

1. (c) 2. (b) 3. (b) 4. (b) 5. (d) 6. (a)


7. (c) 8. (a) 9. (c) 10. (c) 11. (a) 12. (b)
13 (b) 14. (c) 15. (b) 16. (b) 17. (a) 18. (d)
19. (c) 20. (d) 21. (c) 22. (c) 23. (c) 24. (b)
25. (b) 26. (a) 27. (a) 28. (c) 29. (b) 30. (d)
31. (a) 32. (b) 33. (a) 34. (a) 35. (d) 36. (b)
37. (a) 38. (b) 39. (c) 40. (b) 41. (a) 42. (d)
43. (d) 44. (d) 45. (a) 46. (d) 47. (b) 48. (d)
49 (c) 50. (b) 51. (c) 52. (b) 53. (c) 54. (b)
55. (b) 56. (c) 57. (a) 58. (b) 59. (c) 60. (b)
61. (a) 62. (a) 63. (b) 64. (b) 65. (d) 66. (b)
67. (c) 68. (a) 69. (c) 70. (d) 71. (b) 72. (a)
73. (d) 74. (b) 75. (b) 76. (b) 77. (c) 78. (a)
79. (b) 80. (c)
26.24 Comprehensive Physics—JEE Advanced

SOLUTIONS
1. Refer to Fig. 26.57. 1
1 1
v 3. For a spherical mirror = . For a concave
v u f
v v mirror u = – u and f = – f. Hence
1 1 1 uf
= v=
v v u f f u
If u < f, the image is virtual. Hence v is positive for
u lying between zero and f. If u > f, the image is
–v real. Hence v is negative for u lying between f and
u ,v – f. When u f, v
± . Hence the correct graph is (b).
Fig. 26.57 4. Refer to Fig. 26.59.

Velocity of the object is vo = (2 i + 2 j ) ms–1


Speed of object is vo = 22 22 = 2 2 ms–1
= speed of the image (vi). The velocity vi of the im-
age will be as shown in Fig. (a). The relative veloc-
ity of the image with respect to the object is
v = vi – vo = vi + (– vo)
The magnitude of v is given by [see Fig. (b)]
v = [v02 + v2i – 2vo vi cos 90°]1/2
2 2 1/ 2
= 2 2 2 2

= 4 ms–1 along – x axis. Fig. 26.59

2. Refer to Fig. 26.58. The hollow lens can be considered to be a combina-


tion of two lens A and B. For a lens,
1 1 1
= ( – 1)
f R1 R2
For lens A, R1 = + R1 and R2 = + R2. Hence
1 1 1
= ( – 1)
fA R1 R2
For lens B, R1 = – R1 and R2 = – R2. Hence
1 1 1 1
Fig. 26.58 = ( – 1) =
fB R1 R2 fA
I1 is the image of O due to refraction at face I Thus fB = – fA. The focal length of the hollow lens
AI1 = (OA) = 1.5 6 = 9 cm is given by
I2 is the image of I1 1 1 1
I1 B = 9 + 3 = 12 cm, I2 B = 12 cm. = =0 ( f B = – f A)
fh fA fB
I3 is the image of I2 due to refraction at Face I
again. or fh = . Hence a hollow lens behaves like a glass
slab.
I2 A 15
AI3 = = = 10 cm 1 3 2 1
1.5 5. = 1 = ( R1 = – R2 = R)
f 2 R R
Distance of I3 from B = 10 – 3 = 7 cm.
Ray Optics and Optical Instruments 26.25

which gives f = R. When the space between the

lens of = 4/3 surrounded by a medium of g =


3/2. Therefore, for the water lens,
3 4
1 2 3 2
= ( R1 = – R and R2 = + R)
f 4/3 R
1 1
=
4R 4f Fig. 26.61

The focal length of the combination of the three 8. The ray falling normally on face AB is refracted un-
lenses is given by deviated into the prism and is incident on face AC
1 1 1 at an angle of incidence i =
1
= face AC
F f f f
4 8
2 1 7 sin i w
or sin or sin
= g 3 1.5 9
f 4f 4f
Hence the correct choice is (a).
4f 9. The ray does not suffer any deviation on entering
F= , which is choice (d).
7 the lens. Hence 1 = . The ray leaves the second
6. Refer to Fig. 26.60. surface of the lens bending towards the normal.
Hence 2 > . Thus the correct choice is (c).
10. When a glass plate of thickness t is introduced, the
1
image shifts by an amount t 1 . Hence

1 d
d=t 1 or t =
1
1

Thus the correct choice is (c).


v
11. m1 =
. For the
Fig. 26.60 u
conjugate (second) position, since u and v are
For no deviation, the ray must emerge from lens u
B parallel to the principal axis. For this to happen, interchanged, we have m2 = .
v
point F must be at the second focus of lens A and Therefore
B. Hence d = 30 + 20 = 50
v u v2 u 2
cm, which is choice (a). m1 – m2 = – =
7. u = – 3f. The distance v of the image I is given by u v uv

1 1 1 (v u) v u
= v = 1.5 f =
v 3f f uv
The distance of the eye from I is = 3f – 1.5f = 1.5f uv
But f = and v – u = d.
(see Fig. 26.61) u v
Triangles ABI and CDI are similar. Hence d
Hence m1 – m2 =
CD DI h 1.5 f f
=
AB BI d /2 1.5 f d
or f= .
d m1 m2
or h = . So the correct choice is (c).
2 Thus the correct choice is (a).
26.26 Comprehensive Physics—JEE Advanced

12. Refer to Fig. 26.62. If F is the focal length of the combination, we


have
1 1 1 1 15
= – = + or F = cm
F v u 30 10 2
Focal length of the concave lens B = f1 = – 30 cm.
If f2 is the focal length of lens A, we have
1 1 1 1 1 1
+ = or = –
f1 f2 F f2 F f1
2 1
= +
which gives f2 = 6 cm. 15 30
Since f2 is positive, the lens is convex. Hence the
correct choice is (c).
15. fm = )
1 2 1 2
Fig. 26.62 = + = (i)
28 f1 fm f1
Applying Snell’s law to successive refractions, we In the second case we have
have
1 2 1
sin i 1 sin r1 2 = + (ii)
= ; = 10 f1 fm
sin r1 sin r2 1
where fm is the focal length of the curved silvered
sin r2 sin r3
= 3; = surface. Hence fm =
R
where R is the radius of
sin r3 2 sin x 3 2
Multiplying these equations we get curvature of the curved surface.
Subtracting (i) from (ii) we get
sin i
= or sin x = sin i 1 1 1 140
sin x = – or fm = cm.
fm 10 28 9
13. The path of the ray is shown in Fig. 26.63.
280
Therefore R = 2 fm = cm
9
From (i) we have f1 = 28 2 = 56 cm. Now
1 1
= ( – 1)
f1 R
R 280
or –1= = = 0.55 or = 1.55
f1 9 56
16. The value of the effective focal length F is given
by
1 1 1 1 2 1
F = f1 + f m + f = f + f
1 1 m

Fig. 26.63 2 2
= +
20 22
It is clear that
110
+ = 180° and =2 or |F| = cm
21
Hence, = 36°, which is choice (b). Since the convex lens with a silvered surface
v behaves as a concave mirror of effective focal
14. Given m = =–3 ( the image is inverted) length F, we have
u
or v= – 3 u 110
F=– cm and u = – 10 cm
Now u = – 10 cm, therefore v = + 30 cm. 21
Ray Optics and Optical Instruments 26.27

Substituting these values in the mirror formula Hence the intensity of the image reduces by a
1 1 1 factor of 4. Thus the intensity becomes I – I/4 =
+ =
v u F 3I/4. Hence the correct choice is (d).
1 21 1 21. Refer to Fig. 26.64. Here AB = 6 cm. Therefore, a =
we have =– + or v = – 11 cm.
v 110 10 AE = 3 cm. Let C be the centre of curvature of the
The negative sign shows that the image is in front lens. The radius of curvature of the lens is
of the effective mirror and hence is real. R = AC = BC = CD
17. For an achromatic combination
f 1 1
=– =–
f f f
2
where = . Therefore,
3
1 2
=– (i)
f 3f
The focal length of the combination is
1 1 1
= +
F f f Fig. 26.64
where F = + 20 cm. Therefore
Now ED = 3 mm = t, say. In triangle AEC, we
1 1 1 have
= + (ii)
20 f f (AC)2 = (AE)2 + (CE)2
From Eqs (i) and (ii) we get
or R2 = a2 + (R – t)2 = a2 + R2 – 2Rt + t2
3f
f = 6.67 cm and f = – = – 10 cm t
2 or 2Rt 1 = a2
18. Since u >> f0, v = f0 = 19 m. Now u = – 3.8 108m. 2R
Since t << R, the term t/2 R can be neglected
ctive is compared to 1. Hence
v 19 2 Rt = a2
m0 = =– 8
= – 0.5 10 –7
u 3.8 10 a2 3 cm 3 cm
or R= = = 15 cm
Diameter of the image of the moon is 2t 2 0 3 cm
3.5 10 6 0.5 10 –7 = 0.175 m = 17.5 cm
c 3 108 ms 1
Hence the correct choice is (d). Now the refractive index =
v 2 108 ms 1
19. The focal length F of the combination is given
= 1.5. Therefore, the focal length of the plano-
1 1 1 d convex lens is given by
= + –
F f1 f2 f1 f 2 1 1 1 1
In terms of powers we have = ( – 1) = (1.5 – 1)
f R 15 30
P = P1 + P2 – d P1P2 for P = 0, or f = 30 cm. Hence the correct choice is (c).
P1 P2 1 1 22. For a prism with a very small refracting angle A, the
d= = + = f 2 + f 1. deviation is given by (Fig. 26.65)
P1 P2 P2 P1
Hence the correct choice is (c). = ( – 1) A
20. The focal length of a lens does not change if a part
of it is blcoked. If the central part of the aperture 1 = ( 1 – 1) A1
upto d/2 is blocked, the exposed area of the aperture and that produced by the second prism is
reduces by one-fourth the earlier area because
2 = ( 2 – 1) A2
d 2
The total deviation will be zero if 1 + 2 = 0. The
2 1
2
= emergent ray will then the parallel to the incident
d 4 ray (see Fig. 26.65). Thus
26.28 Comprehensive Physics—JEE Advanced

( 2 – 1) A2 = – ( 1 – 1) A1 v= or = v/ ,
The negative sign shows that the refracting angles wavelength decreases because speed v decreases.
of the two prisms are in opposite directions. Thus 27. As the beam falls normally on face AB, it goes
through undeviated. The critical angles for red,
green and blue colours respectively are
1 1
ir = sin–1 = 46°, ig = sin–1 = 44°
139
. 1.44
1
and ib = sin–1 = 43°
1.47
The angle of incidence at face AC is i = 45°. Sin-
ce i is greater than ig and ib but less than ir, the red
Fig. 26.65 colour will be refracted out from face AC but green
AC to-
1 1 A1 154
. 1 4 wards the base BC. Hence the correct choice is (a).
| A2 | = = = 3°
1 1.72 1 28. Using monochromatic light eliminates chromatic
2
aberration. Using a doublet combination mini-
23. Deviation produced by a prism having a small mizes chromatic aberration. Increasing the size of
refracting angle is given by the lens increases its resolving power. To reduce
= ( – 1) A spherical aberration, the aperture (i.e. exposed
Also A + = i + e = i ( e = 0, since the ray portion of the lens) must be decreased. Hence the
emerges normally from the opposite face. Thus correct choice is (c).
= i – A or ( – 1) A = i – A or i = A 29. Spherical aberration is reduced if the total deviation
is distributed over the two surfaces of the lens. If
24. A complete image will be formed but the intensity the plane surface of the lens faces the object, all the
of the image will decrease due to decrease in aper- deviation takes place at the curved surface. Hence
ture. Hence the correct choice is (b). spherical aberration is not reduced. Hence the cor-
25. Refer to Fig. 26.66. The refracted ray BC will re- rect choice is (b).
trace its path if it falls normally on the silvered face 30. Figure 26.67 shows the ray diagram for the image
PQ of the prism, i.e. PCB = 90°. Therefore, angle
in triangle PBC is = 90° – A. Hence r = A. Now with water, the image is real and located at C which
sin i sin 2 A is between O and C. Hence the correct choice is (d).
=
sin r sin A

2 sin A cos A
= = 2 cos A
sin A
Hence the correct choice is (b).

Fig. 26.67
31. Using the formula for a spherical surface
g g a
a
=
Fig. 26.66 u v R
26. Since the refractive index of glass is greater than 1.0 15
. 15
. 1.0
we have = (since v = u)
that of air, the speed of light is less in glass than u u R
in air. The frequency of light never changes due to which gives u = 5R.
Ray Optics and Optical Instruments 26.29

32. Power of the lens combination is n2


1 1 sin ic =
P = P1 + P2 = n1
f1 in m f 2 in m
1 1
= +
0.40 m 0.25 m
= – 1.5 m–1 = – 1.5 D
33. The focal length f of a lens of refractive index 2
surrounded by a medium of refractive index 1 is
Fig. 26.68
given by
1 1 1 Thus angle i must at least be equal to ic. Therefore
2 1
= ic = 90° – r or r = 90° – ic.
f 1 R1 R2
n n
Now, for a concave lens, f = – f. Given R1 = R and sin max = 1 sin (90° – i) = 1 cos ic
n2 n2
R2 = – R1 = – R.
Also 2 = 1.5 and 1 = 1.75. Hence, we have =
n1
cos sin 1 n2
n2 n1
1 15
. 1.75 1 1 0.25 2
– = = n2
f 1.75 R R 1.75 R sin ic
n1
which gives f = 3.5 R. Since the focal length is
positive, the lens acts like a convergent lens. Hence n1 1 n2
or max = sin–1 cos sin
the correct choice is (a). n2 n1
34. In a compound microscope, the object is placed just
38. The refractive index of the prism with respect to the
beyond the focus of the objective. Hence the image
liquid in which it is immersed is given by
formed by the object is real, inverted and highly
1
sin A m
35. Refer to the solution of Q. 33 above. Since the lens 2
=
is concave f = – f. Also R2 = – R1. Therefore, we have 1
sin ( A)
1 n2 n1 1 1 2
– =
f n1 R1 R2 60 30
sin
2 sin 45
where n2 = = 2
the lens and n1 that of of the liquid in which the lens 60 sin 30
sin
is immersed. 2
It will act as a divergent lens (i.e. f will remain The critical angle ic is given by
negative) if n2 > n1. Hence the correct choice is (d). 1
36. When a ray of light passes through a glass slab with sin ic = , which give ic = 45°. Hence the correct
parallel faces, it does not suffer any deviation; it 2
is only displaced parallel to itself. Therefore, the choice is (b).
direction of the beam remains unchanged after 39. Mean angular deviations produced by crown and
passing through the glass slab. However, the rays
are displaced slightly towards the outer side. Hence d = ( – 1) A and d = ( – 1) A
the divergence angle of the emergent beam will be Their dispersive powers are
the same as that of the incident beam.
37. The ray will emerge from side CD of the slab if = v r
and = v r

the ray refracted in the slab suffers repeated total 1 1


AD and BC of the slab Their angular dispersions respectively are
as shown in Fig. 26.68. From Snell’s law, we have D = ( v – r ) A and D = ( v – r) A
sin r n n When the prisms are combined, the dispersion by
= 2 or sin max = 1 sin r
sin max n1 n2 the combination will be zero if
The critical angle ic is given by D+D =0
26.30 Comprehensive Physics—JEE Advanced

or ( v – r) A+( v – r) A =0
( v r) ( v r)
or ( – 1) A + ( – 1)A = 0
( 1) ( 1)
or d+ d = 0. which is choice (c).
40.
given by
1
sin ic =
n
1
If n = 2 , sin ic = or ic = 45°. For total internal
2
greater

i = 45°. Hence the critical Fig. 26.69


- 45.
fore, n is greater than 2 . Thus the correct choice
is (b).
= ( g – 1)A (i)
41.
in water is
1 2 R 30 15
. 9
or F = g
F R 2 2 1.5 =
w 4/3 8
=( – 1)A (ii)
u = 2F = 2

42. 9
1
f1 1 1 8
=– = 4° = 1°
f2 2 g 1 . 1
15
Hence the correct choice is (a).
P2
=– 1 46. Real thickness = apparent thickness
P1 2
= 1.5
or P1 1 = – P2 2
43. angle in
A m
sin
2 A
= R = 2f ( R = 2f ) 47. = . Given = cot . Thus
A 2
sin
2
44. - A m
sin
tance is AB. Now, since PR = RD = L - A 2
cot =
angles PRT PBD we have 2 A
d sin
BD = 2RT = 2RS = 2 =d 2
2
d A A m
OD = . Therefore, OB = OD + BD or cos = sin
2 2 2
d 3d A A m
= d or sin 90 = sin
2 2 2 2
3d A A
AB = 2OB = 2 = 3d, which gives m = 90° –
2 2 2
or m = (180° – 2A) which is choice (b).
26.31

48.
1 sin i QG / QD h / 5h 2
1sin i = 2 sin r1 =
sin r sin 45 1/ 2 5
2 sin r1 = 3 sin r2
3 sin r2 = 4 sin e 5
or = , which is choice (b).
2
51.

correct choice is (c).


52.
AB x = BC which is given by
x = AC tan 30°
3 = 0.2/ 3

3
Fig. 26.70
2 3
n= = 30
0.2/
1 sin i = 4 sin e (1)
Ray CD will be parallel to ray AB, if e = i. Hence Hence the correct choice is (b).
1= 4
49.
P P is such that the re-
P
P, Q R
P suffers
P Q
Q R R will suf-
P. Fig. 26.72
Thus the correct choice is (c). 53. Refer to the Fig. 26.73. AB
50. h
Q QD travelling plane. It serves as the virtual object for the concave
DB lens. A B
Fig. 26.71 that D PB of f u v = ? Now
ABPR. Hence DE = PE = h. Also BDF =
45° since DPE

Fig. 26.73

1 1 1
v u f
or 1 1 1
v 4 20
which gives v
Fig. 26.71
26.32 Comprehensive Physics—JEE Advanced

AB v f
2
= | v| = b , which is choice (c).
AB u u f
AB 5c
or = 57. Referring to Fig. 26.74, the source S cannot be seen
1.6 c 4c
or A B x so that rays such as SA
54. For refraction at glass-water interface, we have SB suffer internal relection. The critical angle
ic is given by
sin i =
g sin r (i)
For refraction at water-air interface, we have
sin r = a sin 90° = a = 1 (ii)
Using (ii) in (i), we get
1
g sin i = 1 or g = ,
sin i
which is choice (b).
Fig. 26.74
1 1 1
55. Using f u = , 1 1
v u f sin ic = , which gives ic = 45°.
2
56 1 1 1
we get v = + = , In triangle OAS, we have
3 v u f
we have r r r
= tan ic or x = = = r.
x tan ic tan 45
1 dv 1 du dv v2 d u
– 0 or Hence the correct choice is (a).
v2 d t u2 d t dt u2 d t
58. Refer to Fig. 26.75.
v2
u2
correct choice is (b).
56.
1 1 1
= (1)
v u f
Fig. 26.75
f is
Ray OA at the planar face
be the angle of refrac-
2
v u v
0 or v=– u (2)
v2 u2 u sin sin
n= or sin = (1)
u, we get sin n
u u v f The ray AB B
1 or (3)
v f u u f ). This

2
n
f at point B
v=– u
u f B.
Given u = b. Therefore For this to happen (90° – ic, the critical angle
2
or
f
v=– b (4) sin (90° – ) sin ic or cos sin ic
u f 2 1/2
or (1 – sin ) sin ic
- 2 2
or 1 – sin sin ic (2)
The critical angle is given by
26.33

1 A
or sin ic = (3) or = 90° –
n 2
A
But = r1 = 90° –
sin 2
1 2
1– 2 2
or n2 – sin2 1 Hence = , which is choice (a).
n n
or n2 (1 + sin2 )
2
= + 1, it
follows that
n2 2 or n 2

AB to suffer total internal


B BC will be
CD suffering another total
D -

Fig. 26.77
59. Given i = 60°, A=
30°. Using = i + e – A, we get 62. -
e
AC of the 1
2

Now r1 = i – = 60° – 30° = 30°. Now the wavelength of violet light is the least
Hence
Fig. 26.76
sin i sin 60
= = = 3.
sin r1 sin 30
Hence the correct choice is (c).
60. u f AC,

1 1 1 1 1 3 1
or v=– sin ic =
v f u 10 25 50
G
2
Area of the object wire is 3.0 . The

area of age 2
area of object
2
v2 50 4
= 2
u 3 25 9

4 2 2
, which is choise (b).
9
61. PQ
RS
Fig. 26.78
the refracting faces AB AC
i1 i2 (see where G
Fig. 26.77). Therefore, r1 = r2 = r. Also ic is the critical angle for green
A
r1 + r2 = A, which gives r1 = r2 = .
2
In triangle ABC, A + + = 180°
or A + 2 = 180°
26.34 Comprehensive Physics—JEE Advanced

- Now u = OI
AC at angles greater than their respec-
v = OI
AC back into the
1 1 1
=
AC at angles less than their f v u
respective critical angles. Hence these colours will we have
AC 1 1 1 1
=
AC. Thus the f 20 30 12
correct choice is (a). or f
63. O fall
Hence the correct choice is (b).
| P1 | 4 |f | 4
65. Given f1 or 2
O. Hence the virtual | P2 | 3 | f1 | 3
O -
f2 4
f2 is negative, . Hence
the correct choice is (b). f1 3

4 4
f2 = f1 =
3 3
The focal length F
1 1 1 1 1 1
=
F f1 f2 12 16 48
which gives F

66. Referring to Fig. 26.81, the path of the ray is PQRS


Q R. It is clear
Fig. 26.79
-
64. er than the critical angle given by
26.80) 1
32 sin ic =
d= n
4/3
Thus I

Fig. 26.81

Also angle A = B= A B
n is given by
1 1 1
sin n = =
n sin sin B

67. Refer to Fig. 26.82.


Fig. 26.80
26.35

point O I1
2 1 2 1
at the plane surface as shown in Fig. 26.82 (a). =
v R1

2 2 1
or = (i)
v R1
=v–t v( t
is negligible),
v = + v1 R = + R2. We have
Fig. 26.82
3 2 3 2
= (ii)
v1 v R2
Hence
4 the ray is parallel to the principal
= v1 = f using
3
v1 = f (i) in (ii), we get
When the plane face of the lens is in contact with
O of the plane 3 2 1
= 3 2
I2 f R1 R2

refraction at this face, we have 1 2 1 1 3 2 1


or = (iii)
f 3 R1 3 R2
1 2 1 2
=
v u R
R1 = R2 we get [put R1 = R2 = R in (iii)]
where 1 = 1, 2 = , u = OP v = OI2=
25 1 3 1 1
– =
8 f 3 R
8 1
=
25 4 R 69. Refer to Fig. 26.84.
4
Putting = R
3

by
1 1 4 1 1
= ( – 1) 1
f R 3 25 75
or f
68. Refer to Fig. 26.83.
Fig. 26.84

AB, angle
i1
given by
2 2 1
sin i1 =
1 2 2
which gives i1 = 45º
CD, angle
i2
given by
3 3
Fig. 26.83 sin i2 =
1 2
u=– ,v=+v
R = + R1. We have which gives i2 = 60º.
26.36 Comprehensive Physics—JEE Advanced

1
surfaces AB CD sin ic = (1)
= 60º. Thus the correct choice is (c).
Angle r is given by
70. ABC DBC
sin 45 1
their bases BC touching each other as shown in = or sin r = (2)
Fig. 26.85, which shows the path of parallel sin r 2
F.
ic + r = 90°
Distance OF = f
or ic = 90° – r
1
h cos r = which gives
tan =
f sin r = (1 – cos2r)1/2
1/ 2
1 1 2
= 1 2
( – 1)1/2

1 1 3
( 2–1)1/2 = which gives = ,
2 2
which the correct choice is (b).
72. Refer to Fig. 26.87. For refraction at face I,
Fig. 26.85 1 1
=
u1 v1 R
h
PQ u1 = , we have
then 1
by where =
v1 R
h
= R
f or v1 = (1)
( 1)
h
is given by (here angle
= ( – 1)
h h
Thus = ( – 1) or f =
f 1

71.
at E
than the critical angle ic given by
Fig. 26.87

For refraction at face II, u2 = – (v1 – 2R) = 2R – v1.

R R( 2)
u2 = 2R – (2)
( 1) ( 1)
v2 is given by
1 1
= (3)
u2 v2 R

Fig. 26.86
26.37

R (2 )
v2 = .
2( 1)
The correct choice is (a).
73.
given by
1 2 1 2 1 2
= + = + = , which gives
F f f m 15 15
15 Fig. 26.88
F= -
2 76.
x<f
1 1 1 x = 2f
+ = , we have
v u F
15
u 20 an F
2
1 1 2
=
v 20 15 false. If x lies between f f
which gives v - to the eye than the object as shown in Fig. 26.89.

74. u Object pin


Image
v Eye
1 1 1 1 1 C F
= – = – , which gives f P
f
f v u 10 10 x
f f is 2x
given by
f u v
2 = Fig. 26.89
f u2
v 2

where u v are the least counts of u v 77.

u= v
u v
f= f2
u 2
v 2
ray is
0.1 0.1 2 = 180° – ( + )
= (5)
(10)2 (10)2
> , < 180° – 2 . Hence the correct
5 choice is (c).
=
100 78. -
Hence f r = A/2 = 60°/2 = 30°
for lights of all wavelengths. However the corre-
75. Refer to the Fig. 26.88.

= r/f
colours. The correct choice is (a).
tan , where
we have 79.
r
= or r=f
f
n0
r2 = 2
f 2. Thus area f 2, n0 sin = sin 1
which is choice (b). 2
26.38 Comprehensive Physics—JEE Advanced

I II III IV n0 n0
q3 = 90°
= sin 2 = sin 90°
6 8
q2
1
which gives sin = .
8
q2 80. If I R+T
q1
=I
q1
-
q tain critical angle c. For < c a part of the inci-

> c
Hence for > c, the value of R is 100 %. Hence
Fig. 26.90 the only correct option is (c).

II

Multiple Choice Questions with One or More Choices Correct

1. 4.

x y respectively. If
–1
v. Then
(a) x
(b) y
1
(c) v = –1
5.
21
v –1
(a) violet light than for yellow light
2.

-
6.

3.

7. The focal lenght of a lens is


(a) greater for violet light than for yellow light

(c) greater for green light than for blue light


26.39

8. (a) R R
D (c) f f
13.
-
jugate positions A B R
d (see Fig. 26.91). The respective linear
m1 m2 for posi-
tions A B of the lens. Then (a) in case (i) the lens will behave as a conver-
(a) d d gent lens of focal length 2.5R
7 3 7 5
(c) m1 = , m2 = m1 = , m2 = lens of focal length 3.0R
3 7 5 7
(c) in case (ii) the lens will behave as a conver-
gent lens of focal length 3.0R

lens of focal length 3.5R


14. A short linear object of length b
f
u a. If the
Vo
Vi. Then
2
f f
(a) a = b (b) a = b
u f u f
Fig. 26.91 2
f f
9. (c) Vi = Vo Vi = Vo
slab u f u f
IIT, 1987
15. AB of length f
f, such that its
AB
. m. Then
f f
(a) A B = (b) A B =
3 2
10. When light travelling in air enters a glass slab, (c) m m = 3/2
IIT, 1991
16. R

. point object P mR
11. shown in Fig. 26.92. The value of m for which a ray
P
Its focal length is f1
f2. Then
(a) f1 f1
(c) f2 f2
12.

R. When the plane surface


Fig. 26.92
f. Then
26.40 Comprehensive Physics—JEE Advanced

4 3 4 3
(a) either or (b) neither nor is . Then
3 2 3 2
(a) i0 = 45° (b) i0 = 60°
4 3 (c) = 60°
3 2 IIT, 2005
IIT, 1999
17. ABC DCE,
3
in Fig. 26.93. A light ray PQ
AB at an angle i DCE C
C

of i
ABC i0. The angle through which Fig. 26.93
DCE C so that

ANSWERS AND SOLUTIONS


1. Putting f D
2. All the four choices are correct. d
3. v D d
m=
4. u D d
5. 100 40 7
= =– .
100 40 3
-
7 3
Thus | m | = . For lens at L2, | m | =
3 7
6.
7. 9. -

8. Refer to Fig. 26.94. If O I


L1 L2 are the two v
conjugate positions of the lens, then =

.
Fig. 26.94
10.
D d D d
For lens at L1, u = – x = – v= .
2 2 v=
1 1 1
Using these in = – , we get
f v u
D2 d 2 1 1 = (1.5 – 1) 1
f= 11. = ( – 1) f1
4D f1 R 10
26.41

2
f
is given by a=b
u f
1 2( 1) 2 (1.5 1)
= = f2 (4) by t, we have
f2 R 10
2
Thus the only correct choice is (c). v f u u
=– . Given = V0
12. = 1.5, f1 = 2 R t u f t t
R = f1 - v f
2
Vi = =– V0
t u f
R 30
f= =
2( 1) 2(1.5 1)

13. f
2

14. Vi = V0
u f
1 1 1
= (1)
v u f
15. Refer to Fig. 26.95. A B
f is AB. B B
B B.
v 2 Location of B x B
u v
– =– = 0 or v=– u (2) or B O u = – x,
v2 u2 u v =–x f = – f. Using these in the spherical
u, we get
1 1 1
u u v f =
+ 1= or = (3) v u f
v f u u f
1 1 1
We have – =–
2 x x f
f
v= – u which gives x = 2f. Thus OB = OB = 2f
u f
Given u = b. Therefore f
Location of A : A A AB = ,
2 3
f f 5f
v= – b (4) AO = y = 2f – = .
u f 3 3
- 5f
Thus for point A, u = – y = – -
3
tance A O

Fig. 26.95
26.42 Comprehensive Physics—JEE Advanced

1 1 1 1 3 1 ( ray BC R = – R.
= or = Therefore, we have
v u f v 5f f
2 1 2 1
5f 5f – =
which gives v = – . Thus A O = as shown u v R
2 2
15. 15
. 1
or –0=
R 15
. m 1 R
5f f
AB = – 2f =
2 2 which gives (1.5 m + 1) 0.5 = 1.5
f 2 4
AB = or m= = .
3 15
. 3

f /2 The only correct choice is (c).


AB 3
= = . 17. ABC, the ray QR
AB f /3 2 is parallel to base BC. Hence
r1 = r2 r1 + r2 = 60º
16.
Q P
surface. Ray PA refracts along AB
Q P
-
ity, we have

2 1 2 1
– =
v u
Here = 1, = 1.5, u = – mR. Therefore, Fig. 26.97
1 2

15
. 1 which give r1 = r2 = 30º
– = 0 or v = – 1.5 mR
v mR
sin i = sin r1

= 3 sin 30º

3
=
2
which gives i = 60º
e
Fig. 26.97 that the ray RS DC
RS
Fig. 26.96
-
BC RS on face DC
Q serves DCE C
through an angle of 60º in the clockwise or anti-
u = – (v + R) = – (1.5 mR + R) =
– R (1.5m v = –m
26.43

III

Multiple Choice Questions Based on Passage


Questions 1 to 5 are based on the following passage
Passage I focal lengths.
The Compound Microscope
focal lengths.

eyepiece has a long focal length

eyepiece has a short focal length.


4.

1.

(c) the objective of a higher focal length is

2. 5.

(a) its resolving power will increase

3.
is high if

SOLUTION
1. The correct choice is (a). where 2 = angle of the cone of light rays entering
2. the objective, -
3.
given by
L D AB
M= (for D >> fo ) fo
fo fe
or fe
where L
eyepiece, D fo
fe = focal length
of the eyepiece.
4.
(see Fig. 26.98)
2 sin
R.P. =
Fig. 26.98
26.44 Comprehensive Physics—JEE Advanced

5.
value of increases. Therefore, the light gathering
capacity of the objective increases. As a result, the

Questions 6 to 10 are based on the following passage


Passage II
8.
The Astronomical Telescope
focal lengths

focal lengths

eyepiece has a long focal length


focal plane of the objective. The position of the eyepiece
eyepiece has a short focal length
9.

(c) the objective of a higher focal length is


6. -

10. If the aperture of the objective of a telescope is

(a) its resolving power will increase


7.

SOLUTION

6. The correct choice is (b). 9. The resolving power of a telescope is given by


7. R.P. = d
8. 1.22
where d = wave-
fo D fe
M=
fe D of fo or fe. Hence the correct choice is (a).
where D 10.

–3/2
Questions 11 to 13 are based on the following passage where k = . The refractive of air is
Passage III 1.0.
IIT, 1995
11. The relation between the slope of the trajectory of
t the ray at the point B(x, y
A i at that point is
n(y) given by dy dy
3/2 1/2
(a) = sin i (b) = cos i
n(y) = (ky + 1) dx dx
26.45

dy dy
(c) = tan i = cot i
dx dx
12.

4
x
(a) y = (x)4 (b) y =
2
4
x x 4
(c) y = y=
3 4
13. x1 of point P where the ray inter-

Fig. 26.99
(a) x1 x1
(c) x1 x1

SOLUTION
11. Refer to Fig. 26.100. The variation of refractive because na ia at
A = 90°.
n(y) = (ky3/2 + 1)1/2 (1) 1
or sin i =
n
1/ 2
1
1
cot i = n2 = (n2 –1)12 (3)
1
n

2
dy dy
= (n2 – 1)1/2 or = n2 – 1 (4)
dx dx

2
dy
= ky3/2 + 1 – 1 = ky3/2
Fig. 26.100 dx
dy dy
y = 0 to y = t. or = k1/2 y3/4 or 3/ 4 = k1/2 dx
dx y
Therefore, the ray SA entering the slab at A -
i Integrating, we get
dy
ABP. = k1/2 dx or 4y1/4 = k1/2 x
y3/ 4
B(x, y) is i, then
Given k = 1.0. Therefore, we have
the slope CD of the trajectory at B is
4
dy 1 x
= tan = tan (90° – i) = cot i (2) y1/4 = x or y = (5)
dx 4 4

12. If i 13. For point P, y y


of refract n get x P are
have x1 y1
n sin i = na sin ia = 1 sin 90° = 1 is (a).
26.46 Comprehensive Physics—JEE Advanced

Questions 14 to 16 are based on the following passage


Passage IV 15.
The x-z (a) 30° (b) 45°
z 2
z 3 . A ray of 16. Ar = a i + b j + c k
where a b are
A=6 3 i +8 3
(a) a = 6 3 , b = 8 3
j –10 k
IIT, 1999 (b) a = 3 3 , b = 4 3
14. The angle between vector A (c) a = 8 3 , b = 6 3
z
(a) 90° (b) 120° a = 4 3, b = 3 3

SOLUTION
14. Refer to Fig. 26.101. Ray PQ in x-z plane travel- which gives r = 45°. so the correct choice is (b).
z i on the
z QR in
z r.
be the angle between vector A -
tive z k is the unit vector along the
positive z
A.k
cos =
A
(6 3 i + 8 3 j 10k ) (k )
= 1/ 2
(6 3 ) 2 + (8 3)2 ( 10) 2

10 1 Fig. 26.101
= =–
20 2
16.
( i . k = j . k = 0) x-y plane), the
which gives = 120°. which is choice (b).
15. i = 180° – = 180° –
Ar = a i + b j + c k

sin i 2 with a = 6 3 b=8 3


=
sin r 1
A. Thus

1 2 1 Ar = 6 3 i + 8 3 j + c k
or sin r = sin i = sin 60° =
2 3 2 This the corrcet choice is (a).

Questions 17 to 19 are based on the following passage 18.


Passage V
–1 –1
–1 –1

–1 19.
IIT, 2004
17. The focal length of the lens is
26.47

SOLUTION

1 1 = (1.5 – 1) 1 gives f
17. = ( – 1) The 60 2
–1
f R 10 = ,
30

18. u f = + 20 19.
v
m= (2)
1 1 1 u
= (1)
v u f t, we have
1 1 1 dm v du 1 dv
or = which gives v =
v 30 20 dt u 2 dt u dt
t, we get 1 du dv
= 2
v u
1 dv 1 du u dt dt
=0
v 2 dt u 2 dt 1
= (–60 × 1 – 30 × 4)
dv v 2
du (30) 2
or =
dt u2 dt
dm
v2
dt
u2

IV

Assertion-Reason Type Questions


3. Statement-1
-
four choices out of which only one choice is correct.
R. The focal length of the lens
R.
Statement-2

4. Statement-1

at an angle i -
1. Statement-1 face of water as shown in Fig. 26.102. Then

focal length will increase.


Statement-2

2. Statement-1

length increases.
Statement-2

Fig. 26.102
26.48 Comprehensive Physics—JEE Advanced

4 Statement-2
g = sin i
3

SOLUTIONS
1. 3. The correct choice is (c). The focal length f is
given by
curvature. 1 1 1 2
( 1) = (1.45 – 1)
2. The correct choice is (a). The focal length of a lens f R1 R2 R
R
which gives f = , i.e. f is greater than R.
0.9
4.
f
2 R1 R2
g sin i = sin r (i)
1is
given by For refraction at water-air interface, we have
1 2 1 1 1 sin r = a sin 90° = a =1 (ii)
=
f 1 R1 R2 Using (ii) in (i), we get
1
It is easy to see that f 1 g sin i = 1 or g =
sin i
1 = 1 for air).

Integer Answer Type


1.

IIT, 1980
2.
-

Fig. 26.103
- 4.
100
tance of
n -
value of n.
IIT, 1981
3.
IIT, 1988
4
5. ) in a tank is 18
3
= a a. 7
lies on water
IIT, 1987 4
26.49

R S
R = 6 cm m = 1.0

x m = 7/4
x
IIT, 2011
m = 4/3

Fig. 26.104
SOLUTION
A
1 1 1
1. For the objective, . Putting u0 =
v0 u0 f0
200 d
– 200 f0 v0 =
3 i e

r1 r2
v0 200 / 3 1
m0 =
u0 200 3
B C

have Fig. 26.105


1 1 1
= Therefore, r1 = i – = 60° – 30° = 30°. Hence
ve ue fe
where ve fe sin i sin 60
= 3
25 sin r1 sin 30
values, we get ue =
eye-piece is 6 Hence the value of a = 3.
v 25 4.
me = e =6 26.106]
ue 25 / 6
2 1 2 1
=
v u R
1
m = m0 me = 6=–2 where 1 = 4/3, 1 u= R
3 Thus
| m |= 2
2. 1 4/3 1 4/3
=
v1 2
which gives v1 -
I1

-
v
For refraction at the concave surface
1 1
= , where R
v u R
1.5 1 1.5 1 100
= u=
15 u 50 9
Hence n = 9.
3. Given i = 60°, A = 30°. We have
=i+e–A (1)
e – 30° or e = 0.
Fig. 26.106
Here also i = 30°. [see Fig. 26.105]
26.50 Comprehensive Physics—JEE Advanced

I1
2 1 2 1
= (i)
v2 u R
For this refraction, we use
7/4 1 7/4 1
1 2
= 1 2 – =
v2 u R v ( 24) 6
where u R v
4/3 1 4/3 1
=
v2 10 ( 2) u
which gives v2 4/3 7/4
I2 – =0
v 21
v
5.
1
2, then

VI

Matrix Match Type


1. Column I
Column II. Match all the
Column II Column I.
Column I Column II

S
(a)

S
(b)

S
(c)

IIT, 2009

SOLUTION
-

1 1 1
=( – 1)
f R1 R2
26.51

where R1
R2 R1 R2 R2 > R1, f is positive, i.e.
the lens is a converging lens for which all the four choices are correct.

ANSWER
(a)
(c)
2. 1 3
2
1, 2 3

Column I Column II

(a) 1 < 2 (p)

m1
m3 m2

(b) 1 > 2

m1
m3 m2

(c) 2 = 3 (r)

m3 m1
m2

2 > 3 (s)

m1
m3 m2

(t)

m3
m1
m2

IIT, 2010

ANSWERS
(a) (p), (r) (b)
(c)
26.52 Comprehensive Physics—JEE Advanced

Explanation:

II I

(p) Ray AB C A
Hence > Ray BC B
2 1.
Hence 2= 3.
m2 m1
m3

II I

B
Ray AB C A
2 < 1. The ray BC
face II. Hence 3 < 2. m1
m3 m2

II I
C
(r) Ray AB B A
2 > 1. Ray BC
3 = 2. m3
m2 m1

II B
C A
I
(s) Ray AB
2 < 1. Ray BC
II. Hence 3 < 2. m1
m3 m2

II B
A
C I
(t) Ray AB
m3
2 < 1. Ray BC m2
m1
3 = 2.
27
Chapter
Wave Optics

REVIEW OF BASIC CONCEPTS where x1 and x2 are paths of the waves up to point P where

= 1 – 2
27.1 WAVE NATURE OF LIGHT
=( 1 – 2)t – (k1 x1 – k2 x2)
Light is an electromagnetic wave which does not require a
=( – 2)
material medium for propagation. The electric and magnetic 1
x1 x2
=2 ( 1 – 2)t –2
of an electromagnetic wave even in free space. 1 2

aries in space and time as 1. If the two waves have different frequencies, i.e., 1
2 then 1 2 and depends on time t.
E = A sin ( t – kx) 2. If 1 = 2, then 1 = 2. In this case
which represents a wave travelling along the + x direction. 2
–1 = (x2 – x1)
A = amplitude, = 2 (
2
and k= ; = wavelength. 2
or Phase difference = (path difference)
Also

v= = i.e., the phase difference is independent of time and


k x2 – x1). This holds
where v

Phase Intensity
proportional to the square of its amplitude at that point.
The phase of a wave at a point x and at time t is given
27.2 REFLECTION AND REFRACTION OF LIGHT
representing the wave, i.e.
= t – kx

Phase Difference undergo a change and the wave is said to suffer refraction.
Suppose two waves meeting at a point P are represented
v1
in the medium in which the incident wave propagates and
E1 = A1 sin ( 1t – k 1x 1) v2
refracted wave propagates, then 1 2
and E2 = A2 sin ( 2t – k 2x 2)
27.2 Comprehensive Physics—JEE Advanced

v1 I is mini-
1 1
2 = mum if cos = – 1, i.e.
v2 2
= (2 n – 1)
where 1 and 2 are the wavelengths of the same wave
1
and = n ; n = 1, 2, 3, .....etc.
remains the same as that of the incident wave. 2
Then
Imin = I1 + I2 – 2I1I2
suffers a phase change of 180° (or radians) in relation 2
= I1 I2
to that of the incident wave. No phase change occurs if a

minima constitute the bright and dark fringes.

27.3 INTERFERENCE OF LIGHT 27.4 COHERENT LIGHT SOURCES


When two or more light waves meet (or superpose) at a
depends on the phase difference ( ) between the two
principle of superposition interfering waves. This phase difference depends upon
which states that two factors—(1) the initial phase difference between

difference resulting from the path difference for that


point. The initial phase difference depends upon the time
E = E1 + E2 + –8
to 10–10

with time that, due to persistence of vision, we are unable


to see the interference pattern. Thus, non-coherent sources
more waves is known as cannot produce sustained interference effects. We con-
If two waves of intensities I1 and I2, differing in phase

1. The sources must be coherent.


I = I1 + I2 + 2 I1 I 2 cos 2. The wavelength of the interfering waves must be
the same. Thus,
I -
imum if cos = + 1, i.e.
27.5 YOUNG’S DOUBLE SLIT EXPERIMENT
= 2n ;
where n = 0, 1, 2, 3, etc. is an integer Monochromatic light from a source slit S illuminates
two slits S1 and S2
2 equidistant from S (Fig. 27.1).
or = 2n

or =n
where is the path difference between the interfering
waves. Then
I = I1 + I2 + 2 I1 I 2
2
= I1 I2

The interference is said to be constructive. If the


Fig. 27.1
two interfering waves have equal intensities I1 = I2 =
I0, then S1 and S2 interfere giving rise
I = 4 I0 to bright and dark fringes on the screen. There is bright
fringe at centre P0 of the screen.
Wave Optics 27.3

(i) The distance of the nth bright fringe from the centre 2
a
n D
yn = ; n = 0, 1, 2, ... etc. 2f
, where
a
where = wavelength of light used, = seperation f
between slits S1 and S2 and D = distance between the slit.
the screen and the plane of the two slits.
(ii) The distance of the n th dark fringe from the centre SOME IMPORTANT POINTS ABOUT
27.8
INTERFERENCE OF LIGHT
1 D
y*n = n ; n = 1, 2, 3, ... etc -
2
(iii) The separation between two consecutive bright will be white; all other fringes will be coloured.
or dark fringes is called fringe width ( ) which is White light consists of colours between violet and
rad (VIBGYOR). Wavelength is the shortest for
=
D violet light and longest for red light. At the central

(iv) Angular separation between nth bright fringe and


the central fringe is
the central fringe will be violet colour.
yn n
n = ; n is in radian.
D -
(v) Angular separation between nth dark fringe and the ness t , then
central fringe is (a) the path difference at the centre of the screen
1 – 1)t.
*
n = n
2
( 1)tD
amount y0 = .
27.6 DISPLACEMENT OF FRINGES
(c) at the centre of the screen there will be a
If a transparent plate of thickness t
bright fringe if ( – 1) t = n ; n = 1, 2, 3,
is introduced in the path of one of the interfering waves,
... etc.
(d) At the centre of the screen there will be a
tD dark fringe if
y0 = ( – 1)
1
( – 1)t = n n = 1, 2, 3, ... etc.
( 1)t 2
Number of fringes shifted =
(e) the fringe width will remain the same.

27.7 DIFFRACTION AT A SLIT


When a parallel beam of monochromatic light falls
(because the two interfering beam do not now

will become less distinct.


If is the wavelength of light and a is the width of the
slit, then is closed (or covered with black paper), the inter-
3 5
(i) For bright fringes : sin = 0, , , ... etc
2a 2a pattern which has a bright central fringe bordered
2 3
(ii) For dark fringes : sin = , , ... etc
a a a in still water rather than in air, the fringe width
27.4 Comprehensive Physics—JEE Advanced

is greater than that of the air, the speed of light in sources have intensities in the ratio n : 1, then in the
water (v) will be less than that in air (c). Since the interference pattern
v
= I ma I in 2 n
1 =
c
and a = which give = = . I ma I in (n 1)
c
a -
Now < a. Fringe width .
in water < in air.
I1
light has two wavelengths 1 and 2 n : 1, then = n and
I2
will coincide if n1 1 = n2 2, where n1 and n2 are 2
integers. I ma n 1
=
I min n 1
light sources have intensities in the ratio n : 1, i.e.
I1 x
=n is the width of the source slit S and X its distance
I2 from the plane of the slits, the interference fringes
and minima in the interference pattern is will not be seen (because the interference pattern
I ma I1 I2 2 I1 I 2 becomes indistinct) if the condition
=
I ma I1 I2 2 I1 I 2 x
<
X
2
I1 I2
= 2
I1 I2
27.9 RESOLVING POWER
2
I1
1 2
I2 n 1
= close together.
I1 n 1
1 (a)
I2
2 sin
- R.P. =
1.22
ent light sources, if the ratio of the intensities of
where 2
= refrac-
I ma
n : 1, i.e. = n, then ratio of the intensities of
I min = wavelength of light used to
the coherent sources is
2 Objective
I1 n 1
= because
I2 n 1
2 Object
I1
1 O
2q
I ma I2
=
Lmin I1
1
I2
Fig. 27.2
I1
1 2
n = I2 I1 n 1 (b)
I1 I2 n 1 D
1 R.P. =
I2 1.22
Wave Optics 27.5

where D = wave-
27.1
length of light

27.10 POLARIZATION OF LIGHT light at a point on the screen where the path differ-
ence is is K
a point where the path difference is /3; being the
diffraction are common on both transverse and longitudinal wavelength of light used?
waves, mechanical as well as electromagnetic. The
SOLUTION
be . Path difference = . Therefore, phase difference
2
all the possible orientations in a plane perpendicular to = = 2
the direction of propagation. When this light is passed
= or = 2 is
= I1 + I2 + 2 I1 I 2 cos

= I1 + I2 + 2 I1 I 2 cos 2

Polarization by Reflection: Brewster’s Angle = I1 + I2 + 2 I1 I 2


= I + I + 2I 4I = K units
( I1 = I2 = I)
particular angle ip on the surface of a transparent medium, i.e. I = K
difference is
2
= or =
3
2 2
= = is
3 3
2
I = I + I + 2I cos
3
K
= 2I – I = I = units
4
Fig. 27.3 ip is Brewster’s angle
27.2

angle ip at the surface of glass. Brewster discovered that


when i = ip
90° apart. The angle ip when this happens is called the
or the . If rp is the corresponding

SOLUTION
ip + rp = 90° or rp = 90° – ip
Given I1/I2 = 100, i.e. I1 = 100 units and
I2 = 1 unit.
sin i p sin i p
n= =
sin rp sin(90 ip ) I = I1 + I2 + 2 I1 I 2
sin i p = 100 + 1 + 2 100 1 = 121
= = tan ip
cos i p
n = tan ip I = I1 + I2 – 2 I1 I 2 = 81
This equation is called the and the special I 121
= = 1.49
. I 81
27.6 Comprehensive Physics—JEE Advanced

2 2
27.3 I1 I2 x 1
= =
- I1 I2 x 1
I1
where x= .
and the central bright fringe is measured to be 1.2 I2
-
49 x 1 2 7 x 1
periment?
9 x 1 3 x 1
SOLUTION 5
which gives x = . Therefore
The position of the nth bright fringe with respect to 2
I1 25
n D = x2 =
yn = I2 4

For the central bright fringe (n = 0), y0 = 0. For the 27.5


fourth bright fringe (n = 4), y4 = 4 D/ . Therefore
4 D the intensities at points P and Q on the screen where
y4 – y0 = or = (y4 – y0) (i) the path difference between the interfering waves is
4D
It is given that (y4 – y0) = 1.2 cm = 1.2 10–2 m, D = (a) , where is the wavelength of
4
1.4 m and = 0.28 mm = 0.28 m. Substituting light used.
these values in (i) and solving, we get
7 SOLUTION
=6 m(or 600 nm or 6000 Å).
I = I1 + I2 + 2 I1 I 2 cos
27.4
2
where = ; = path difference
in an interference pattern is 49 : 9. What is the ratio of
I1 = I2 = I0. Therefore,
I = I0 + I0 + 2I0 cos
= 2I0 (1 + cos )
SOLUTION (a) For = 0, °
I ma 49 I1 = 2I0 (1 + cos 0°) = 4I0
Given =
I min 9 2
(b) For = , =
I = I1 + I2 + 2 I1 I 2 4 4 2

and Imin = I1 + I2 – 2 I1 I 2 I2 = 2I0 1 cos = 2I0


2
I ma I I2 2 I1 I 2 I1
= 1 =2
I min I1 I2 2 I1 I 2 I2

Multiple Choice Questions with Only One Choice Correct


1. Monochromatic light of wavelength in air is re- (a) (b)
. The
wavelength of light in glass is (c) (d) 2
Wave Optics 27.7

2. (a) 1 : 1 (b) 1 : 2
air and then in a liquid. It is observed that the 10th (c) 2 : 1 (d) 2 :1
8.

3 4 than in air?
(a) (b)
2 3 (a) Fewer fringes will be visible
5 20 (b) Fringes will be broader
(c) (d)
3 17 (c) Fringes will be narrower
3. (d) No fringes will be observed.
y when a glass plate of re-
9. -
ment affected if one of the slits is covered with
another plate of the same thickness but of refractive black opaque paper?
(a) The bright fringes become fainter
(b) The fringe width decreases
3y 2y
(a) (b) (c) There will be uniform illumination all over
2 3 the screen
7y 6y (d) There will be a bright central fringe bor-
(c) (d) 7
6
4.
monochromatic light of wavelength . The separa- 10. What is the effect on the interference fringes in
tion between the slits is = 50 and the distance
between the screen and the slits is D = 1000 . If I0 moved closer to the double slit plane?
(a) The fringe width increases
(b) The fringe width decreases
the slits is (c) The fringes become more distinct
(a) 0
I
(b) I0 (d) The fringes become less distinct.
2 11. What is the effect on the interference fringes in
(c) 2I0 (d) 4I0
5. source slit is increased?
(a) The fringe width increases
(b) The fringe width decreases
light from the slit? (c) The fringes become more distinct
(a) The fringe width will decrease (d) The fringes become less distinct.
(b) The fringes will become more distinct 12. What is the effect on the interference fringes in
(c) The bright fringes will become less bright and
two slits are increased?
(d) No fringes will be observed (a) The fringe width increases
6. (b) The fringe width decreases
are in the ratio of 1 : 2, the ratio of the intensities at

(a) 1 : 2 (b) 1 : 3 (d) The bright fringes are no longer


(c) 1 : 4 (d) 1 : 9
7. 13.
as coherent sources of equal amplitude A and of 0.5 mm apart and interference is observed on a
wavelength screen placed at a distance of 100 cm from the slits.
set up, the two slits are sources of equal amplitude It is found that the 9th bright fringe is at a distance
A and wavelength but are incoherent. The ratio of 9.0 mm from the second dark fringe from the
centre of the fringe pattern. What is the wavelength
of light used.
27.8 Comprehensive Physics—JEE Advanced

(a) 2000 Å (b) 4000 Å 20.


(c) 6000 Å (d) 8000 Å I. If the width of each slit is doubled

14. A screen is placed at a certain distance from a


I
(a) (b) I
2
- (c) 2 I (d) 4 I
scope? 21. -
imum of wavelength 1 is at a distance y1 from

observed. wavelength 2 is at a distance y2 from its central


y1/ y2 will be
(c) One bright fringe is observed at the centre of
the screen. 2 1 2 2
(a) (b)
(d) A bright central fringe bordered on both 2 1

1 2
(c) (d)
2 2 2 1
15.
above is and if is the width of the slit, what is 22. White light is used to illuminate the two slits in
-
(a) sin –1 ( / ) (b) 2 sin –1 ( / ) tween the slits is and the distance between the
–1
(c) sin (2 / ) (d) sin –1 ( /2 ) screen and the slit is D (>> ). At a point on the
16. A single-slit diffraction pattern is obtained using a
beam of red light. What happens if the red light is wavelengths are missing. The missing wavelengths
are (here = 0, 1, 2, is an integer)
(a) There is no change in the diffraction pattern. 2
2 1 2
(b) Diffraction fringes become narrower and (a) = (b) =
2 1 D
crowded together.
(c) Diffraction fringes become broader and far- 2 2
1
ther apart. (c) = (d) =
1 D
(d) The diffraction pattern disappears.
33. -
17. A parallel beam of light of wavelength 6000 Å is
centre of the screen is I. What will be the inten-
diffraction pattern is observed on a screen which is
closed?
length 50 cm. If the lens is placed close to the slit,
the distance between the minima on both sides of I
(a) I (b)
2
(a) 1 mm (b) 2 mm
I
(c) 3 mm (d) 4 mm (c) (d) none of these
4
18. be-
tween the slits S1 and S2 is 1.0 mm. What should the 24.
and blue light of wavelength 4000 Å is incident
the two slit interference pattern within the central

(a) 0.1 mm (b) 0.2 mm (a) red (b) blue


(c) 0.3 mm (d) 0.4 mm (c) violet (d) green
19. Monochromatic light is refracted from air into glass 25. Light of wavelength is incident on a slit of width
. The ratio of the wavelengths . The resulting diffraction pattern is observed on
of the incident and refracted waves is a screen at a distance D. The linear width of the
(a) 1 : 1 (b) 1 :
(c) :1 (d) 2 : 1 if D equals
Wave Optics 27.9

2 X
(a) (b) pattern will reveal
2 2
2
(c) (d) (b) more number of fringes
2 (c) less number of fringes
26. Two waves of intensities I and 4I superpose, then (d) no diffraction pattern

(a) 5I, 3I (b) 9I, I IIT, 1999


(c) 9I, 3I (d) 5I, I 32.
wavelength of light used is 6000 Å. If the path
IIT, 1982 difference between waves reaching a point P on the
27. n, wave- screen is 1.5 microns, then at that point P:
length v in air, enters (a) Second bright band occurs
(b) Second dark band occurs
(c) Third dark band occurs
(d) Third bright band occurs
n v v
(a) , , (b) n, , 33. The difference in the number of wavelengths, when

v n propagates through air and vacuum columns of the


(c) n, , (d) , , v
is 1.0003, the thickness of the air column is
28. A beam of light of wavelength 600 nm from a (a) 1.8 mm (b) 2 mm
distant source falls on a single slit 1.0 mm wide
(c) 2 cm (d) 2.2 cm
and the resulting diffraction pattern is observed
34.
-
fringe is
(a) 1.2 cm (b) 1.2 mm
(c) 2.4 cm (d) 2.4 mm
IIT, 1994 35. = 3.0
29. A parallel beam of monochromatic light is incident
r = 4.0. Then
formed on a screen placed perpendicular to the
remains unchanged.
the diffraction pattern, the phase difference between
becomes half

2 unchanged
(c) (d) 2
IIT, 1998 unchanged
–2
30. A string of length 0.4 m and mass 10 36. Two beams of light having intensities I and 4I
clamped at its ends. The tension in the string is interfere to produce a fringe pattern on a screen.
1.6 N. Identical wave pulses are produced at one The phase difference between the beams is /2 at
end at equal intervals of time t. The minimum point A and at point B. Then the difference be-
value of t which allows constructive interference tween the resultant intensities at A and B is
between successive pulses is (a) 2 I (b) 4I
(a) 0.05 s (b) 0.10 s (c) 5I (d) 7I
(c) 0.20 s (d) 0.40 s
IIT, 2001
IIT, 1998
37. A double slit apparatus is immersed in a liquid
31. Yellow light is used in a single slit diffraction
27.10 Comprehensive Physics—JEE Advanced

1 mm and distance between the plane of slits and 2


(c) sec = (d) sec =
3 3
beam of light whose wavelength in air is 6300 Å.
What is the fringe width? IIT, 2003
0.63
(a) (1.33 0.63) mm (b) mm
133
.
0.63
(c) mm (d) 0.63 mm
. 2
133
IIT, 1996
38.
to be 0.4 mm. If the whole apparatus is immersed in

in mm is
(a) 0.25 (b) 0.30 Fig. 27.4
(c) 0.40 (d) 0.53 43.
39. is 2.0 mm, The separation between the 9th bright
observed to be formed in a certain region of the fringe and the second dark fringe from the centre
screen when light of wavelength 600 nm is used. If
the light of wavelength 400 nm is used, the number (a) 5.0 mm (b) 10 mm
of fringes observed in the same region of the screen (c) 15 mm (d) 20 mm
will be 44.
(a) 12 (b) 18 is covered with a transparent sheet of thickness
(c) 24 (d) 8 3.6 10–3 cm the central fringe shifts to a position
IIT, 2000
40. (a) 1.50 (b) 1.55
fringes are obtained on a screen placed at some (c) 1.60 (d) 1.65
45. A beam of light, consisting of two wavelengths
distance of 5 10–2 m towards the slits, the change
6500 Å and 5200 Å is used to obtain interference
in the fringe width is 3 10–5 m. If the separation
between the slits is 10–3 m, the wavelength of light separation between the slits is 2.6 mm and the
used is distance between the plane of the slits and the
(a) 5 10–7 m (b) 6 10–7 m screen is 1.0 m. The least distance from the central
–7
(c) 7 10 m (d) 6 10–6 m
41. wavelengths coincide is
t is (a) 1.0 mm (b) 1.5 mm
introduced in the path of one of the interfering (c) 2.0 mm (d) 2.5 mm
beams (wavelength 46. n are

remains unchanged. The minimum thickness of the


glass-plate is interference pattern is
(a) 2 (b) 2 /3 n 1 n 1 2

(c) /3 (d) (a) (b)


n 1 n 1
IIT, 2002 2
n 1 n 1
42. In Fig. 27.4, PQ represents a plane wavefront and (c) (d)
AO and BP n 1 n 1
monochromatic light of wavelength . The value 47.
of angle BP
OP
pattern is n. The ratio of the intensities of the two
(a) cos = (b) cos = coherent sources is
2 4
Wave Optics 27.11

2 2
n 1 n 1 thickness 1.5 10–6
(a) (b)
n 1 n 1
2
n 1 n 1 (a)
I0
(b)
I0
(c) (d)
n2 1 n 1 2 3
I0
48. (c) (d)
have widths in the ratio n:1. The ratio of the 4
54. In Q. 53 above, the lateral shift of the central
interference pattern is
n 1 n (a) 1.5 mm (b) 4 mm
(a) (b) (c) 3 mm (d) 2 mm
n n 1
2 2 55. -
n 1 n 1 mum is observed at a point on the screen when
(c) (d)
n 1 n light of wavelength 480 nm is used. If this light is
49.
of a fringe formed on a distant screen is 0.1°. If the
wavelength of light used is 628 nm, the spacing (a) 16 (b) 14
between the slits is (c) 12 (d) 10
(a) 0.9 10–4 m (b) 1.8 10–4 m 56. A beam of light consisting of two wavelengths
(c) 3.6 10–4 m (d) 7.2 10–4 m 4500 Å and 7500 Å is used to obtain interference
50. Interference pattern is obtained with two coherent distance between the slits is 1 mm and the distance
n. In the interference between the plane of the slits and the screen is 120 cm.
I ma I in What is the minimum distance between two succes-
pattern, the ratio will be
I ma I in sive regions of complete darkness on the screen?
n 2 n (a) 4.5 mm (b) 5.4 mm
(a) (b) (c) 2.7 mm (d) 1.2 mm
(n 1) (n 1)
IIT, 2004
n 2 n 57.
(c) (d)
2
n 1
2 a point P -
n 1
51. Interference pattern is obtained with two coherent of light used is and is the distance between the
light sources of intensities I and 4I slits, the angular separation between point P and
a point where the phase difference is /2 is the centre of the screen is
(a) I (b) 2I
(c) 3I (d) 5I (a) sin–1 (b) sin–1
2
52.
light of wavelength (c) sin–1 (d) sin–1
at this wave- 3 4
length. The smallest thickness of the sheet to bring IIT, 2005
58. A parallel beam of fast moving electrons is inci-

(a) (b) at a large distance from the slit. If the speed of the
2( 1) ( 1)
electrons is increased, which of the following state-
(c) (d) ments is correct?
2 (a) Diffraction pattern is not observed on the
53. Monochromatic light of wavelength 500 nm is in- screen in the case of electrons

of 5 10–4 m. The interference pattern is obtained of the diffraction pattern will increase
on a screen at a distance of 1.0 m from the slits.
I0. When will decrease
27.12 Comprehensive Physics—JEE Advanced

(a) 3
will remain the same. (b) 4
IIT, 2007 (c) 5
59. - (d) much greater than 5.
62.
certain distance on the screen when a transparent sheet composed of two wavelengths 1 and 2 close to
of thickness t is introduced in each other (with 2 greater than 1) is used. The
the path of one of the interfering waves. The sheet is order n up to which the fringes can be seen on the
then removed and the distance between the screen
and the slits is doubled. It is found that the distance
2 1
(a) n = (b) n =
2 1 2 1
same as the observed fringe shift when the sheet was
introduced. The wavelength of light used is 1
2
IIT, 1983 (c) n = (d) n =
2( 2 1)
2( 2 1)
1
(a) = ( – 1)t (b) = ( – 1)t 63. Monochromatic light of wavelength emerging
2 from slit S illuminates slits S1 and S2 which are
1 placed with respect to S as shown in Fig. 27.5. The
(c) = ( + 1)t (d) = ( + 1)t distances x and D are large compared to the separa-
2
60. tion between the slits. If x = D/2, the minimum
value of so that there is a dark fringe at the centre
The wavelength of light used is 6000 Å. What will P of the screen is
- D 2 D
mental arrangement is immersed in water? Refrac- (a) (b)
3 3
4
.
3 D
(c) D (d) 2
(a) 0.15° (b) 0.18° 3
(c) 0.2° (d) 0.27°
61. A coherent parallel beam of microwaves of wave-

slit apparatus. The separation between the slits is


1.0 mm and the screen is placed at a distance of
1.0 m from the slits. The number of minima in the
interference pattern observed on the screen is
IIT, 1998 Fig. 27.5

ANSWERS
1. (c) 2. (b) 3. (a) 4. (c) 5. (c) 6. (d)
7. (c) 8. (c) 9. (d) 10. (d) 11. (d) 12. (d)
13. (c) 14. (d) 15. (b) 16. (b) 17. (c) 18. (b)
19. (c) 20. (c) 21. (a) 22. (a) 23. (c) 24. (c)
25. (c) 26. (b) 27. (b) 28. (d) 29. (d) 30. (b)
31. (a) 32. (c) 33. (b) 34. (c) 35. (c) 36. (b)
37. (c) 38. (b) 39. (b) 40. (a) 41. (d) 42. (a)
43. (c) 44. (a) 45. (a) 46. (d) 47. (a) 48. (c)
49. (c) 50. (b) 51. (d) 52. (a) 53. (d) 54. (a)
55. (a) 56. (c) 57. (d) 58. (c) 59. (b) 60. (a)
61. (b) 62. (d) 63. (a)
Wave Optics 27.13

SOLUTIONS
1. of light remains the same, we 6. The amplitudes of the two coherent waves will be
have A1 = 2A and A2 = A. Therefore
c= and v= 2
I ma A1 A2 2A A
where c = speed of light in air and v = speed of light = = =9
in glass. Therefore I min A1 A2 2A A
c
= = .
v 7. If the two sources are coherent, the resultant
So the correct choice is (c). amplitude at the midpoint of the screen due to
interference = A + A = 2 A
10 D 1 D I1 (2 A) 2 or I1 = k 4 A 2 where k is a constant of
2. y10 = and y8 = 8
2
y10 10 4
= = ( = ) I2 (A 2 + A2) or I2 = k 2A2
y8 7.5 3
4 I1 4 k A2
Given y10 = y8 = , which is choice (b). = =2
3 I2 2 k A2

tD
3. Displacement of fringes = ( – 1) 8.
-
y = (1.5 – 1)
tD periment will be less in water than in air. Since the

tD it follows from the relation = v / that the wave-


y = (1.75 – 1) length in water is less than in air. Since fringe
width , the value of
y 0.75 3 3y the correct choice is (c).
= = y =
y 0.5 2 2 9. We then obtain a single slit diffraction pattern on

4. Path difference = 10. and 11. Let x be the width of the source slit and X
D
the distance between the source slit and the plane
For the point P
of the two slits. For interference fringes to be dis-
of the slits,
x /X < / should
50 x is too large (i.e. the source slit
y= = = 25
2 2 is too wide) or if X is too small (X is the distance
between the source slit and the two slits) the
25 50 5 requirement x / X < / -
For point P, = =
1000 4 es will no longer be distinct. The reason is that
2 2 5 5 the interference patterns due to various parts of
Phase difference = =
P is 4 2

I = 4I0 cos2
2 remains visible, a change in x or X has no effect on
the fringe width .
5
= 4I0 cos2 = 2I0 12. The single slit diffraction effects at the two slits
4 becomes important and as a result, the interference
5.
from the covered slit will decrease resulting in a
difference in the intensities of the two virtual sourc- 13. The distance of the th bright fringe from the
central fringe is
27.14 Comprehensive Physics—JEE Advanced

since is small, sin . Therefore, = 3 10 –3 rad.


y = = If the lens is placed close to the slit then
where = D / is the fringe width. x = f tan f ( is small, tan )
y9 = 9 (i) where x
The distance of the th dark fringe from the central
between two minima on both sides of the central
fringe is
1 D 1
y = = 2 x = 2f = 2 0.5 3 10–3
2 2
3 =3 10 –3 m = 3 mm.
y2 = (ii)
2 18. Let the width of each slit be a. The linear sepa-
3 ration between bright fringes in the double slit
From Eqs. (i) and (ii), we get y9 – y 2 = 9 –
2 y =
15
= Since y << D, the angular separation between
2
It is given that y9 – y 2 bright fringes will be

9.0 2 y
= = 1.2 mm = =
15 D
= 1.2 10 –3 m For 10 bright fringes we have
Now = / D. Substituting for , and D, we get 10
10 = (i)
= 6 10 –7 m = 6000 Å
14.
a bright central fringe bordered on both sides with in the diffraction pattern due to a slit of width a is
2
2 1 = (ii)
15. a
Equating Eqs. (i) and (ii), we get
sin = 10 2 1.0 mm
= or a = =
where is the angular separation between the a 5 5
= 0.2 mm.
of it so that 2 is the angular width of the central
19. n of the light does not change
as light travels from air into glass, we have
= sin –1 2 = 2 sin –1 va = n a and v = n
va
Therefore a
= =
<< , then v
sin , where is in radians. In that case, the
=2 / .
16. The wavelength of blue light is less than that of the 20. I = I = I1 + I2 + 2 I1 I 2

will decrease which means that the fringes become When the width of each slit is doubled, I1 becomes
narrower and crowded together. 2 I1 and I2 becomes 2I2. Therefore,
17. The angular separation of the minima on both sides I =I = 2 I1 + 2I2 + 2 2 I1 2 I2
where

6000 10 10 = 2 (I1 + I2 + 2 I1 I 2 ) = 2 I = 2I.


sin = = 3
=3 10 –3
0.2 10
Wave Optics 27.15

= 3867 Å
21. We know that y = . Therefore, for wave- which is close to violet light (
the correct choice is (c).
10 1 D
length 1, y1 = and for wavelength 2, 25. If D >> , the linear width of the central principal
2 D
5 2 D y1 2 1
distance D = ,
y2 = , =
y2 2 where is the width of the slit. The linear width of

for a value of D
22. The distance of the th minimum from the centre
2
2 D
= or D =
1 D 2
y=
2
26. Given I1 = I and I 2 = 4I. Now
of the slits, y =
I = I1 + I2 + 2 I1 I 2
1 D = I + 4I + 2 4 I 2 = 9I
=
2 2
and I min = I1 + I 2 – 2 I1 I 2
2
or = = 5I – 4I = I
2 1
27. -
23. Let I0
to = glass is
n v
I = I0 + I0 + 2 I 0 I 0 = 4 I0 v =n = =
I
or I0 =
4 28. The angular separation between the th dark fringe
24. When a wave of wavelength falls at an angle
of incidence i and
a sin =
thickness t, then the condition for constructive
= 1. Therefore
1 a sin 1 =
2 t cos r = ; = 0, 1, 2, 3,
2 or sin 1 = /a. Since << a, sin 1 ~ 1
where r 1 = /a. This is also the angular separation between
normal incidence i = 0, hence r = 0. Therefore
( = 1 for air)
1 3 5
2t = = , , etc. either side of the central bright fringe = 2 1 = 2 /a .
2 2 2 2 Therefore, their separation at distance = 2 m is
2t 4t 4t 9
or = = 4t, , , etc. 2 2 (600 10 ) 2
1 3 5 = 3
a 1.0 10
2
Now t = 0.00029 mm = 2.9 10 –5 cm = 2.4 10–3 m = 2.4 mm
= 2900 Å. Therefore, 29. If follows from Fig. 27.6 that the path difference at the
= 11600 Å, 3867 Å, 2320 Å, etc. A and B is
= BC = a sin 1
Wave of = 11600 Å is in the infrared region and
wave of = 2320 Å is in the ultraviolet. These But sin 1 = /a. Therefore, = a /a = . A path dif-
ference of corresponds to a phase difference of 2 .
27.16 Comprehensive Physics—JEE Advanced

1
or1.5 10 – 6 = n (6 10 – 7 )
2
1 5
which gives n – or n
2 2
difference of 1.5 10–6 m corresponds to the third
dark fringe. Thus the correct choice is (c).
33. Let a and
va and c be their
respective speeds in air and vacuum. Since the

have
va c
Fig. 27.6 = =
a
30. Mass per unit length of the string is c
–2 –2 –1 or = . But c =
= 10 kg/0.4 m = 2.5 10 kg m . va a va
a

T
Speed of each pulse is v = a = or a = (1)
a a

1.6 Now, if t is the thickness of each column, then the


= 2 = 8 ms –1 number of wavelengths in the two media are
2.5 10
t t
na = and n = .Given (na – n) =
To have a constructive interference, the pulses must a
arrive at the same end of the string in the same t t t
L is the l= = 1 (2)
length of the string, the minimum value of t is a a
Using (1) in (2), we have
2L 2 0.4
( t)min = = = 0.1 s 1=
t
( a – 1) (3)
v 8
31. Å Given a = 1.0003 and = 6000 Å = 6000 10–8
–10
10 m. Now, the angular width of the central cm. Using these values in (3), we have
t
2 D l= (1.0003 – 1)
= 6000 10 8
0
a 8
6000 10
where D is the distance of the screen from the slit. or t= = 0.2 cm = 2 mm
Since 0.0003

34. Referring to Fig. 27.7, the direction along which


we have the n
32. Given = 6000 Å = 6000 10–10 m = 6 10–7 sin = n
m and = 1.5 microns = 1.5 10–6 m. For bright sin
or n=
fringes: = n ; where n is an integer.
1.5 10 6
5 If = 2 , we have
n= = 7
= , which is not an integer. n = 2 sin
6 10 2
10–6 m does not is 90 , n =
2 sin 90 = 2. Thus, there are two interference
correspond to a bright fringe. For dark fringes, we
have (which corresponds to
1
= n
2 which is choice (c).
Wave Optics 27.17

10
D 6300 10 133
.
l = = 3
l 133
. 1 10
= 0.63 10–3 m = 0.63 mm

0.4 mm
38. l = a
= 0.3 mm, which is choice (b).
nl 4/3
width of region L
39. Number of fringes = or n = .
fringe width
Now, fringe width is proportional to wavelength.

Fig. 27.7 600


n =n = 12 = 18,
400
35.
n of a dielectric medium is given which is choice (b).
D D
40. = . Therefore, = . Given D =
n= r = 4 =2
5 10–2 m, = 3 10–5 m and = 10–3 m. Using
If c is the speed of the electromagnetic wave in these values, we have
vacuum and v in the dielectric medium, then 2
5 10
c c c 3 10–5 = 3
n= or 2 = or v = 10
v v 2
which gives =6 10–7 m which is choice (b).
When a wave passes from one medium into
0 and
41. When a transparent plate is introduced in the path
are the wavelengths of the wave in vacuum and of one of the interfering beams, the entire fringe
y = ( – 1)t, where
t is the thickness of the
v= and c = 0, which give
v c /2 1 -
0
or = or =
c 0 0 c 2 2 3
y= or ( – 1)t = or 1 t= or t = 2 .
2

(c). 42. Since P and Q are points on the same wavefront,


36. I = I1 + I2 + 2 I1 I 2 cos . -
Given I1 = I and I2 = 4I. ference at point P BP and the re-
OP is
IA = I + 4I + 0 = 5I
2 = QO + OP (i)
and IB = I + 4I + 2 I 4 I cos PR
Now, in triangle POR, OP = . Also
= 5I – 4I = I cos cos
IA – IB = 5I – I = 4I, which is choice (b). in triangle QOP, QO = OP sin (90° – 2 ) =
OP cos 2
37. The fringe width in liquid is
= OP cos 2 + OP
= a
l
nl = OP (cos 2 + 1) = 2OP cos2

where =
D
is the fringe width in air. Thus =2 cos2 = 2 cos
a cos
27.18 Comprehensive Physics—JEE Advanced

n D
Since there is a sudden path change of due to y=
2
re -
ence at P is 6500 5
or = n
3 n 5200 4
= , etc.
2 2 The least integral values of and n
3 the above condition are
or 2 cos = , , etc.
2 2 = 5 and n = 4
3
or cos = , , etc. i.e., the 5th bright fringe of wavelength 5200 Å
4 4 coincides with the 4th bright fringe of wavelength
6500 Å. The smallest value of y at which this
43. The distance of the th bright fringe from the happens is
central fringe is
D ymin =
y = =
Substituting the values of , , D and , we get
D
where = is the fringe width. ymin = 1.0 mm, which is choice (a).
I
y9 = 9 (1) 46. Given 1 = n. Therefore, the amplitude ratio is
I2
The distance of the th dark fringe from the A1
central fringe is = n
A2
1 D 1
y = Now I = (A1 + A2)2 and Imin = (A1 – A2)2
2 2
2
3 A1
y2= (2) ( A1 A2 ) 2 1
2 I ma A2
= =
From Eqs. (1) and (2) we get I min ( A1 A2 ) 2 A1
2
1
3 15 A2
y9 – y 2 = 9 – =
2 2 n 1
2

15 = 2
= 2.0 mm = 15 mm n 1
2
the correct choice is (d).
44. I
47. Given ma = n.
D I min
y30 = 30
( A1 A2 ) 2
=n
( A1 A2 ) 2
D
y0 = 30 ( A1 A2 )
or = n
( A1 A2 )
D
But y0 = ( – 1)t A1 n 1
which gives =
D D
A2 n 1
30 = ( – 1)t 2
I1 n 1
=
30 30 (6000 10 10 ) I2 n 1
or ( – 1) = = = 0.5
t (3.6 10 5 ) the correct choice is (a).
= 1.5 48.
45. Let the nth bright fringe of wavelength n and the proportional to its width. Since the amplitude is
th bright fringe of wavelength coincide at a
distance y we have
Wave Optics 27.19

A1 I1 n Ir = I + I + 2 I I cos 3
n
A2 I2 1 = 2I – 2I = 0 ( cos 3 = – 1)
As shown in solution of Q. 46, the correct choice
is (c). is a dark fringe at the centre.
49.

= or = (1) 54. The correct choice is (a). The lateral shift is given

Given = 628 nm = 628 10–9 m and = 0.1° tD


0.1 y = ( – 1)
= rad. Using these values in Eq. (1), we
180 55. The position of the n
= 3.6 10–4
n D
is (c). yn =
50. The correct choice is (b). Refer to the solution of
Q. 46. For a given point yn D and are
51. The correct choice is (d). Use n = constant, i.e. n1 1 = n2 2 n2
1 20 480
I = I1 + I2 + 2 I1 I 2 cos = n1 = = 16, which is choice (a).
2 600
where I1 = I, I2 = 4I and . = 56. Let the nth dark fringe of wavelength n and the
2 th dark fringe of wavelength coincide at a
52. If t is the thickness of the glass sheet, the fringes are distance y from the centre of the screen, then
1 nD 1 D
y = ( – 1)
tD y= n = (1)
2 2
- At this position there is complete darkness on the
screen. Eq. (1) gives
the central bright fringe), the fringes must be 1
n
2 7500Å 5
1 = =
y= =
D 1 n 4500Å 3
2 2 2
tD 1 D 6n 2 3n 1
( – 1) = which gives = = (2)
2 10 5
or t= , which is choice (a). Integral values of n and
2( 1) n = 3, = 2; n = 8, = 5; and so on. Let n1 = 3
53. When a transparent plate of thickness t and refrac- and n2 = 8, then from Eq. (1) the distances from
is introduced in one of the interfer-
ing waves, the path difference at the centre of the regions of darkness ar
screen is 1 nD
tD y1 = n1
= ( – 1) 2
2 1 nD
Phase difference = and y2 = n2
2
2 nD
= ( – 1)
tD
(1) y = y2 – y1 = (n2 – n1)

Given = 5000 Å = 5 10–7 m, = 1.5, 4500 10 10


1 2
t = 1.5 10 m, D = 1 m and = 5 10–4 m. Us-
–6 = (8 – 3) 3
ing these values in Eq. (1). we get = 3 . If I is 1 10
= 2.7 10 –3 m = 2.7 mm,
27.20 Comprehensive Physics—JEE Advanced

57. If is the phase difference between the interfering where


waves at point P P is
sin =
I= I cos2 a
2 where a is the width of the slit. Thus, if
I ma decreases, and hence 2 will decrease. Therefore,
Given I=
2 the correct statement is (c).
1 59. When a sheet of thickness t
cos2 = , which gives is introduced in one of the interfering waves,
2 2 2 4
the distance y0 through which the fringes shift is
or =
2 D
The angular separation between points P and y0 = ( – 1)t (1)
O = y/D. Since
tan ~ sin The fringe width , i.e., the distance between suc-
y
sin = (1)
D D
If is the fringe width, then =
y /2 1 When the distance D between the slits and the
= (2)
2 4 screen is doubled, the new fringe width becomes
This is so because the phase difference between =2
D
(2)
D
. Now = .
Using this in Eq. (2), we get It is given that y0 = . Equating Eqs. (1) and (2) we get
1 D D
= 2 = ( – 1) t
D 4
1
or
y
= (3) = ( – 1)t
D 4 2
Using Eq. (1) in Eq. (3), we have which is choice (b)
60.
sin = or = sin–1 , which is choice (d).
4 4 =

In air: a = a

= or = (1)
a a a

speed of light in air va


= a a
speed of light in water v
Fig. 27.8 (2)
58. De Broglie wavelength of electron is where
unchanged.
h h
= Using Eq (2) in Eq (1) we have
p v
0.2
If speed v of electron is increased, momentum = a
= 0.15°
p(= v will 4/3
decrease. Now, the angular width of the central So the correct choice is (a).
Wave Optics 27.21

61. Refer to Fig. 27.9. When the incident beam falls 1


S1 and S2, the path difference or n 1 = n 2 or 2n 1 = (2n – 1) 2
2
at the central point P0 which gives
P 0.
2
n= , which is choice (d).
2 2 1

63. Refer to Fig. 27.10. To reach point P, wave 1 has to


travel a path (SS2 + S2P) while wave 2 has to trav-
el a path (SS1 + S1P). Therefore, when the waves
arrive at P, the path difference is

Fig. 27.9
Let the minima appear along directions with
respect to the incident direction. Coherent waves
from S1 and S2 along this direction are brought
to a focus at P. It is clear that the path difference
between the waves from S1 and S2 on reaching P is
= sin
Fig. 27.10
The interference minima will appear on the
screen if = (SS2 + S2P) – (SS1 + S1P) (1)
1 Now, in triangle SS2S1, we have
=
2 2 1/ 2
1 SS2 = (x2 + 2 1/2
) =x 1
or sin = x2
2
2
where is an integer. Thus the directions of mini-
= 1 ( << x)
2 x2
1 2
sin =
2 S2P = (D + 2 2 1/2
) =D 1
2 D2
Given = 1.0 mm and = 0.5 mm. Therefore
( << D)
1 0.5 1 1 Also (SS1 + S1P) = x + D. Using these in Eq. (1), we
sin =
2 1.0 2 2 have
2 2
The allowed values of are those integers for =x 1 +D 1 – (x + D)
which sin is not more than + 1 or less than – 1. 2 x2 2 D2
These values are 2 2
minima will be observed. The correct choice is (b) =x+ +D+ –x–D
62. 2x 2D
wavelengths close to each other, two interference 2
1 1
patterns corresponding to the two wavelengths are or =
remains 2 x D
distinct upto a point on the screen where the nth In order to have a dark fringe at P, =
2
1 = 5890 Å 2
falls on the nth order minimum of the other 1 1
= (1)
wavelength 2 = 5895 Å. Thus, interference pattern 2 2 x D
can be seen upto a distance yn from the centre of 1/ 2
the screen if xD
or = (2)
n 1D x D
yn = ; (n
D
Putting x =
1 2D 2
= n ; (nth minimum) (2)
2 choice is (a).
27.22 Comprehensive Physics—JEE Advanced

II

Multiple Choice Questions with One or More Choices Correct

1. x
is no change in its (c) the 5th dark fringe is at a distance of from
2
the central fringe.
(c) wavelength (d) speed 9x
(d) the 5th dark fringe is at a distance of
2. When a light wave travels from air into glass, which 20
of the following will change? from the central fringe.
7.
(c) wavelength (d) speed fringe of wavelength 1 is at a distance x from the
3. central fringe. Then
with light of wavelength 1 is 1 and with light of
wavelength 2 is 2. Using light of wavelength 1, (a) the 5th bright fringe of wavelength 2 will be
the fringe width becomes 3 if the entire apperatus x
at a distance of 1 from the central fringe.
is immersed in a transparent liquid of refractive 2 2
. Then (b) the 5th bright fringe of wavelength 2 will
x
1 2 be at a distance of 2 from the central
(a) 2 = 1 (b) 2 = 1 2 1
2 1 fringe.
1 (c) the 5th dark fringe of wavelength 2 will be at
(c) 3 = (d) 3 = 1
9 2x
a distance of from the central fringe.
4. 20 1
of equal widths, one slit is made twice as wide as (d) the 5th dark fringe of wavelength 2 will be at
the other. Then in the interference pattern, the in-
9 1x
a distance of from the central fringe.
20 2
8.
(c) of minima will increase
is observed on a screen placed at a distance of
(d) of minima will decrease 100 cm from the slits. The wavelength of light used
IIT, 2000 is 600 nm.
5. (a) the distance of the 4th bright fringe from the
of the slit is made double its original width. Then central fringe is 8 mm.
(b) the distance of the 4th dark fringe from the
become central fringe is 7 mm.
(a) narrower (b) fainter (c) the distance between the 9th dark fringe and
the second bright fringe on the same side of
(c) broader (d) brighter
the central fringe is 15 mm.
6.
fringe is at a distance x from the central fringe. second bright fringe on the opposite sides of
Then the central fringe is 5 mm.
19 x
(a) the 10th dark fringe is at a distance of 9. A narrow slit of width 1.3 10–6 m is illuminated
20
from the central fringe.
21x
(b) the 10th dark fringe is at a distance of
20
from the central fringe. is 30°.
Wave Optics 27.23

2
is 60°. (a) (b)
3 3
(c) the angular separation between central 4 5
(c) (d)
is 45°. 3 3
IIT, 1997
(d) the angular separation between the central
13.
between the slits is 2 mm and the distance of the
3
is sin–1
. screen from the plane of the slits is 2.5 m. A light
4 of wavelengths in the range 200 nm to 800 nm is
10. A parallel beam of monochromatic light of wave- allowed to fall on the slits. The wavelengths in the
visible region that will be present on the screen at
width 0.3 mm. The diffraction pattern is observed
on a screen which is placed at the focal plane of (a) 400 nm (b) 500 nm
(c) 600 nm (d) 700 nm
14. A parallel beam of light containing two wave-
x1 and x2 lengths and
slit. Fraunhofer diffraction pattern is obtained on a
(a) x1 = 1 mm (b) x1 = 2 mm
screen placed at the focal plane of a lens of focal
(c) x2 = 3 mm (d) x2 = 6 mm length 0.5 m. It is observed that the second mini-
11. A beam of light consisting of two wavelengths 750 mum of and the third minimum of overlap
nm and 450 nm is used to obtain interference fringes at the same point on the screen 2.5 mm from the
centre of the screen. If = 6000 Å,
between the slits is 1 mm and the distance between (a) = 4000 Å
the plane of the slits and the screen is 100 cm. The
(b) = 3000 Å
bright fringes due to both the wavelengths coincide (c) Slit width = 0.20 mm
is ymin and y min is the corresponding distance (d) Slit width = 0.24 mm
where the dark fringes due to both the wavelengths 15
coincide. Then between the two slits is and the wavelength of the
IIT, 2004 light is
(a) ymin = 2.25 mm (b) ymin = 2.0 mm
Choose the correct choice(s).
(c) y min = 4.5 mm (d) y min = 0.1 mm (a) If =
12.
(b) If < < 2
covered with a thin glass sheet of thickness t and
-
1 while the lower slit is covered
served on the screen
with another glass plate of the same thickness t but
2 (> 1). Interference
increased so that it becomes equal to that of
pattern is observed using light of wavelength . It
slit 2, the intensities of the observed dark and
is observed that the point P on the screen where the
bright fringes will increase.
n = 0) fell before the plates were
increased so that it becomes equal to that of
phase difference between the interfering waves at
slit 1, the intesities of the observed dark and
point P now is
bright frings will increase.
SOLUTIONS
1. The correct choices are (b), (c) and (d). 2 2
= , which is choice (b).
2. The correct choices are (a), (c) and (d). 1 1

1D 2D 1
3. 1 = , 1 = , Therefore, Now l = a
= . Therefore,
27.24 Comprehensive Physics—JEE Advanced

where = wavelength and a = width of the slit.


1 l 1
3 = , which is choice (c). -
1
2, where 2
4. In the case when the slits are of equal width, the
3
sin 1=
I0. Then 2a
10. The correct choices are (b) and (d). The linear
I = I0 + I0 + 2 I 0 I 0 = 4I0 2f
and
and Imin = I0 + I0 – 2 I 0 I 0 = 0 3f
a
.
S1 is made twice as wide as the a
S1 is doubled, i.e. 11. Let nth bright fringe of wavelength n and the
I1 = 2I0 but I2 = I0 th bright fringe of wavelength coincide at a
I = I1 + I2 + 2 I 1 I 2 distance y from the centre of the screen. Then

= 2I0 + I0 + 2 2 I 0 I 0 n D
y =
= 3I0 + 2 2 I = 5.83 I0
0 or n n =
and Imin = I1 + I2 – 2 I 1 I 2
or n
=
= 2I0 + I0 – 2 2 I 0 I 0 n
750
= 3I0 – 2 2 I 0 = 0.17I0 or =
450 n
Thus I >I and I min > Imin,
5
so the correct choice are (a) and (c). or . The minimum integral values of and
n 3
5. The angular width of the /a n = 5 and n = 3.
where a is the width of the slit. If the value of a is Therefore, the minimum value of y is
9
n nD 3 750 10 1
decreases to half its earlier value. This implies that ymin = 3
- 10
thermore if a - = 2.25 × 10–3 m = 2.25 mm

wrong.
The correct choice are (a) and (d). 12. -
6. The correct choices are (a), (c) and (d). The dis- periment, slits S1 and S2 are equidistant from the
tance of the nth bright fringe from the central fringe source S
light emerging from S1 and S2 has the same inten-
n D I0. Before the plates are introduced, the
xn = path difference (and hence the phase difference )
The distance of the nth dark fringe from the central at the central point P between the interfering waves
P is
1 D I = I0 + I0 + 2 I 0 I 0 cos 0° = 4I0
x*n = n
2
7. The correct choices are (b) and (c).
8. All the four choices are correct.
9. The correct choices are (b) and (d). The angular
1 where 1 is

sin 1=
a Fig. 27.11
Wave Optics 27.25

When the two plates are introduced the path differ- 2


ence at the central point P between the interfering 2 = (2)
a
waves becomes
The third minimum of lies along a direction 3
= ( 2 – 1)t – ( 1 – 1)t = ( 2 – 1) t n = 3 in Eq. (1).
2 2 3
Phase difference = ( 2 – 1)t 3 = (3)
a
P becomes Given 2 = 3
have
I = I0 + I0 + 2 I 0 I 0 cos 2 3
=
= 2I0 + 2I0 cos a a
3I 2 2 6000 Å
Given that I = 4 I 0 = 3I0. Thus or = = 4000 Å
4 4 3 3
3I0 = 2I0 + 2I0 cos
If y is the distance of the second minimum of (or
1 third minimum of ), then, using Eq. (1), we have
or cos =
2
2f 2f
5 7 y= f = or a = (4)
or = , , , etc. 2
a y
3 3 3
Thus the correct choices are (a) and (d). Given f = 0.5 m, = 6000 Å = 4 10–7 m and
13. The distance of the n y = 2.5 mm = 2.5 10–3 m. Using these values in
Eq. (4) we get a = 2.4 10–4 m = 0.24 mm.
n D So the correct choices are (a) and (d).
yn = or =
n
nD
15.
Given =2 10–3 m, D = 2.5 m and yn = 10–3 m.
Thus Path difference = sin =n
3 3
10 2 10 n = 0, 1, 2, n = 0),
=
n 2.5 sin 0 = 0 which gives 0 -
8000 Å mum appears at point P0.
1
= 8 10–7 m = (n = 1), sin 1= . For = , we have sin 1 = 1 or
n n
1 -
n is an integer having values 1, 2, 3, etc.
For n= 1 ; 1 = 8000 Å (a) is correct.
1
For n= 2 ; 2 = 4000 Å If sin 1 lies between and 1, i.e. 1
8000 Å 2
For n= 3 ; 3 = 2667 Å lies -
3
For n= 4 ; 4 = 2000 Å
choice (b) is also correct.
Wavelength 8000 Å is in the infrared region,
wave-length 4000 Å is in the visible region and
wavelengths 2667 Å and 2000 Å are both in the
ultraviolet region of the electromagnetic spectrum.

14. The nth minimum lies along a direction n


a sin n = n ; n = 1, 2, 3, etc.
or a n = n ( n is small)
n
or n = (1)
a
The second minimum of lies along a direction 2
n = 2 in Eq. (1).
27.26 Comprehensive Physics—JEE Advanced

If I1 = I = I2, than I = 4I and Imin = 0. So the in-


Ir = I1 + I2 + 2 I1 I 2 cos

I = I1 + I2 + 2 I1 I 2
If I1 = 4I = I2, then I = 16 I and Imin
and Imin = I1 + I2 – 2 I1 I 2
For I1 = 4I and I2, = I, we have I = 9I and Imin = I. incorrect. Choice (a) and (b) are correct.

III

Multiple Choice Questions Based on Passage

Questions 1 to 3 are based on the following passage 1. The ratio of the powers received at aperture A to
Passage I that at aperture B is
(a) 1 : 2 (b) 1 : 4
romatic and parallel beam of light of wavelength 6000
10 (c) 1 : 8 (d) 1 : 16
Wm–2
2. The phase difference between the interfering waves
circular apertures A and B of radii 0.001 m and 0.002 at point F is

(a) (b)
is placed in front of aperture A (Fig. 27.12). 6 3
IIT, 1989 (c) (d)
4 2
3.
in the original direction, the resultant power at point
F will be
(a) 5 W (b) 6 W
(c) 7 W (d) 8 W
Fig. 27.12

SOLUTION
10 (15
. 1) (2000 10 8
) 2
1 I) = Wm–2 = rad
8
(6000 10 ) 3
Power received at aperture A = I cross-sectional
area of A The correct choice is (b).
10 3. Since 10% the power received at each aperture
= (0.001)2 = 10–5 W goes in the original direction, the power at point
F
10
Power received at aperture B = (0.002)2 PA = 10% of 10–5 W = 10–6 W
=4 10–5 W PB = 10% of 4 10–5 W = 4 10–6 W
So the correct choice is (b).
2. The phase difference at F is
the square of the amplitude. If A1 and A2 are the
2 amplitudes at F due to the two sources, we have
= ( – 1) t
PA = kA21 and PB = kA22, where k is the proportional-
Wave Optics 27.27

A1 =
PA
and A2 =
PB 7 10 6
A=
k k k
Resulting amplitude Resultant power at F = kA2
6
A= A12 A22 2 A1 A2 cos 7 10
=k = 7 10–6 W
k
Substituting the values of A1, A2 and , we get

Questions 4 to 6 are based on the following passage


Passage II
IIT, 1990
4.
3I 3I
(a) (b)
16 32
9I 9I
(c) (d)
32 64
5. The ratio of the intensities of beams 1 and 2 is
16 4
(a) (b)
9 3
Fig. 27.13
25 5
I (c) (d)
16 4
is incident on a glass plate A as shown in Fig. 27.13.
6.
Another identical glass plate B is kept close to A and
in the interference pattern is
(a) 16 : 1 (b) 25 : 1
incident on it and transmits the remaining. Interference
(c) 36 : 1 (d) 49 : 1

SOLUTION
4. I is incident on plate 3I 25 3I
=
A I 4 100 16
I/16 falls on plate A which
25 I
I1 = I
100 4 of beam 2 is
3I 75 9I
I2 =
16 100 64
So the correct choice is (d).
I1 I /4 16
5. = , which is choice (a).
I2 9 I / 64 9
a 16 4
6. The ratio of amplitudes is 1 . Thus .
a2 9 3
a1 = 4 units and a2 = 3 units.
Fig. 27.14 2 2
I ma a1 a2 4 3
I/4 falls on plate B which = = 49
I min a1 a2 4 3
B is Thus the correct choice is (d).
27.28 Comprehensive Physics—JEE Advanced

Questions 7 to 9 are based on the following passage where is in Å and is a positive constant. The value of
Passage III
Two parallel beams of light P and Q (separation ) each the face AC
containing radiations of wavelengths 4000 Å and 5000
IIT, 1991
7.
( = 4000 Å, = 5000 Å)
(a) greater for beam than for beam
= 1.20 + (b) less for beam than for beam
2
(c) equal for both the wavelengths

8. The value of is (given sin = 0.8)


(a) 2 105 (Å)2 (b) 4 105 (Å)2
(c) 8 105 (Å)2 (d) 1 104 (Å)2
9. -
length is
(a) 1.5 (b) 2
(c) greater than 1.25
Fig. 27.15 (d) less than 1.25.

SOLUTION
7. Given = 4000 Å and = 5000 Å. Also

= 1.20 + 2
(1)

It follows from this relation that the value of is


greater for than for . The angle of deviation is
greater for greater value of
choice is (a).
8. and
critical angle ic is Fig. 27.16

1 1 1 1
sin ic = = = 1.25
sin ic sin 0.8

Thus, the smaller the value of , the greater is the [ sin = 0.8 (given)]
angle ic. It follows from relation (1) that for is Substituting = 1.25 and = 4000 Å in relation (1),
greater than for ic for is less than we have
i c for . It follows from Fig. 27.16 that the angle
1.25 = 1.20 +
of incidence at face AC is the same (= ) for both (4000)2
beams. It is given that the condition of total inter-
AC which gives = 8 105 (Å)2, which is choice (c).
of the wavelengths. Since ic for (= 4000 Å) is 9. Using = 8 105 (Å)2 and = 5000 Å in Eq. (1),
less than i c for (= 5000 Å), it is obvious that the we get
radiation of wavelength 8 105
= 1.20 + = 1.232
(5000) 2
= 5000 Å is transmitted through the face AC (see
Fig. 27.16) So the correct choice is (d).
Wave Optics 27.29

Questions 10 to 13 are based on the following passage 11. If = 648 nm, the least value of t for which the
Passage IV
(a) 90 nm (b) 180 nm
3= 1.5 is coated with a (c) 108 nm (d) 216 nm
t 2 = 1.8. Light 12.
of wavelength
n n
(a) t = (b) t =
2 2 2 3
IIT, 2000 1 1
10. n n
2 2
(n is an integer) (c) t = (d) t =
2 2 2 3
n n
(a) t = (b) t = 13. If = 648 nm, the least value of t for which the
2 2 2( 2 3)

1 1 (a) 90 nm (b) 180 nm


n n
2 2 (c) 108 nm (d) 216 nm
(c) t = (d) t =
2 2 2 3

SOLUTION
10.
( 1 2= 1.8) of path for wave 2 from P to Q and from Q to P in the
thickness t P as wave
meeting 2 2(PQ)
the surface of the glass plate ( 3 = 2 2t = 2 2t
at point Q and travels along QP. Optical path difference between waves 1 and 2
at point p is
= 2 – 1 = 2 2t –
2
Now, for constructive interference, n ; n = 0,
1, 2,
1
or 2 2t – = n or 2 2t = n
2 2
1
n
2
or t=
2 2

So the correct choice is (c).


Fig. 27.17
11. The minimum value of t corresponds to n = 0.

Waves 1 and 2 meet at point P 648 nm


tmin = = = 90 nm.
We know that when a wave is travelling in a rarer 4 2 4 18.

denser medium, it undergoes a phase change of So the correct choice is (a)


or a path change of /2. Thus wave 1 has an optical 1
12. For destructive interference = n
path of 1 = /2. Wave 2 travelling from P to Q in the 2
at Q from
1
2 2t – = n
2 2
27.30 Comprehensive Physics—JEE Advanced

n 648nm
which gives t = , which is choice (a). tmin = = = 180 nm
2 2 2 2 2 1.8
13. The minimum value of t corresponds to n = 1. Thus the correct choice is (b).

Questions 14 to 16 are based on the following passage 14. Light travels as a


Passage V (a) parallel beam in each medium
(b) convergent beam in each medium
Figure 27.18 shows a surface XY separating two
transparent media, medium-1 and medium-2. The lines (c) divergent beam in each medium
and represent wavefronts of a light wave traveling (d) divergent beam in one medium and conver-
in medium-1 and incident on The lines of ef and gent beam in the other medium
represent wavefronts of the light wave in medium-2 after 15. The phases of the light wave at and f are
refraction. c, , e and f c, f.
(a) c cannot be equal to
(b) can be equal to e
(c) ( – f) is equal to ( c – e)
(d) ( – c) is not equal to ( f – e)
16. Speed of light is
(a) the same in medium-1 and medium-2
(b) larger in medium-1 than in medium-2
(c) larger in medium-2 than in medium-1
Fig. 27.18 (d) different at and
IIT, 2007
SOLUTION
14. - smaller in medium-2 than in medium-1. Thus the
front. Since the wavefronts in both the media are correct choice is (b).
plane and parallel, the corresponding beam of light
in each medium will be parallel. Therefore, the cor-
rect choice is (a).
15. All points on a wavefronts are in the same
phase of oscillation. Therefore c = and
f= e
( – f) = ( c – e)

16.
wavefront. It is clear from the diagram that the
angle
r is less than angle i) as the beam travels from
medium-1 into medium-2. Therefore medium-2 is
Fig. 27.19
Wave Optics 27.31

IV

Assertion-Reason Type Questions

In the following questions, Statement-1 (Assertion) is 3. Statement-1

correct. become sharper and brighter.


(a) Statement-1 is True, Statement-2 is True; State- Statement-2

(b) Statement-1 is True, Statement-2 is True;


4. Statement-1
Statement-1. When light travels from a rarer to a denser medium,
(c) Statement-1 is True, Statement-2 is False. its speed decreases.
(d) Statement-1 is False, Statement-2 is True. Statement-2

1. Statement-1 5. Statement-1
Red light travels faster in glass than green light. When a light wave travels from one medium to
Statement-2
Statement-2
than for green light. The speed of the wave undergoes a change.
2. Statement-1 6. Statement-1

of the source slit is increased, the fringe pattern undergoes a phase change of .
becomes indistinct. Statement-2
Statement-2 The direction of the propagation of light is changed

increases if the width of the source slit is increased.

SOLUTIONS
1. - -

speed of light in vacuum 3. The correct choice is (a).


= 4.
speed of light in the medium
not depend on the speed of the wave.
longer wavelength. The wavelength of red light is 5.
R < G which does not depend on its speed or wavelength; it de-
implies that the speed of red light is more than that -
of green light in glass. duces that wave.
2. The correct choice is (c). If the source slit is wide, the 6. The correct choice is (c). The phase change is due
interference pattern becomes indistinct because the -
interference patterns due to various parts of the source tion from the mirror.
27.32 Comprehensive Physics—JEE Advanced

Matrix Match Type


1. Column I
from the slits S1 and S2. In each of these cases S1P0 = S2P0, S1P1 – S2P1 = /4 and S1P2 – S2P2 = /3, where is
and thickness
t is pasted on slit S2. The thicknesses of the sheets are different in different cases. The phase difference between
the light waves reaching a point P (P I(P).
Match each situation given in Column I with the statement(s) in Column II valid for that situation.
Column I Column II
(a) (p) (P0) = 0

(b) ( – 1) t = /4 (q) (P1) = 0

(c) ( – 1) t = /2 (r) I (P1) = 0

(d) ( – 1) t – 3 /4 (s) I (P0) > I (P1)

(t) I (P2) > I (P1)


IIT, 2009
ANSWERS
(a) (p), (s) (b) (q)
(c) (t) (d) (r), (s), (t)

Explanation:

I (P) = 4I0 cos2 where I0 I (P1) = 4I0 cos2 = 2I0


2 4
each interfering beam and is the phase difference I (P0) > I (P1).
2 2
= (P) At P2, (P2) = = . Therefore
3 3
(a) At P0, (P0) = 0, therefore (P0) = 0 and I(P0) =
4I0. I (P2) = 4I0 cos2 = I0
3
At P1, (P1) = = . Therefore I (P2) < I (P1)
4 2
So the correct choices are (p) and (s).
Wave Optics 27.33

(b) At P0, (P0) = = . Therefore, I (P0) = 2I0 Therefore I (P2) = 4I0 cos2 = 3I0
4 2 6
At P1, (P1) = 0. Therefore, I (P1) = 4I0
3
2 (d) At P0, (P0) = I (P0) = 2 I0
At P2, (P2) = = 4
3 4 12 12 6
3
At P1, (P1) = I (P1) = 0
Therefore I (P2) = 4 I0 cos 2
3.7 I0 4 4 2
12
So I (P1) > I (P2 3 5
At P2, (P2) =
4 3 12
(c) At P0, (P0) = . Therefore I (P0) = 0 2 5 5
4 =
12 6
At P1, (P1) = . Therefore I (P1) = 2 I0 5
2 4 4 Therefore, I (P2) = 4I0 cos2 = 0.27 I0 0
12
At P2, (P2) = = . So the correct choices are (r), (s) and (t).
2 3 6 3

VI

Integer Answer Type


1. 2. A coherent parallel beam of microwaves of wave-

a parallel beam of light of wavelength 600 nm. The slit apparatus. The separation between the slits is
1.0 mm and the screen is placed at a distance of
10–3 m. Find the 1.0 m from the slits. Find the number of minima in
width of the slit (in mm). the interference pattern observed on the screen.
IIT, 1997 IIT, 1998

SOLUTION
1. For a slit of width a, the angular separation between 2f
the n The linear separation y =
a

n 2f 2 0.25 600 109


sin n = a= = 3
a y 6 10
–3
n 1= . Since =5 10 m = 5 mm
a
2.
<< a, sin = (in radian) and = . Therefore, S1 and S2, the path difference at the central point
1 1 1
a P0
P0. (Fig. 27.20)
Let the minima appear along directions with
2 respect to the incident direction. Coherent waves
2 1 = from S1 and S2 along this direction are brought
a
27.34 Comprehensive Physics—JEE Advanced

to a focus at P. It is clear that the path difference 1 0.5 1 1


between the waves from S1 and S2 on reaching P is sin =
2 1.0 2 2
= sin
The allowed values of are those integers for
The interference minima will appear on the screen if which sin is not more than + 1 or less than – 1.
1 These values are
=
2 minima will be observed.
1
or sin =
2
where is an integer. Thus the directions of mini-

1
sin =
2
Given = 1.0 mm and = 0.5 mm. Therefore Fig. 27.20
Atomic Physics 28.1

28
Chapter
Atomic Physics

REVIEW OF BASIC CONCEPTS The work function W0, i.e. the energy required to pull
an electron away from the surface of the metal, is large for
28.1 PHOTOELECTRIC EFFECT heavier elements like platinum whereas for other elements
like alkali metals, W0 is quite small. The minimum, or
When electromagnetic radiation of appropriate frequency
threshold, energy which a photon must have to free the
falls on a metal, electrons are emitted. This phenomenon
electron from the surface of the metal should be equal
is called photoelectric effect and the emitted electrons are
to its work function. If the threshold frequency is 0 the
called photoelectrons because they are liberated by means
threshold energy is h 0. Thus
of light.
W0 = h 0
Einstein’s Photoelectric Equation
Einstein’s photoelectric equation therefore becomes
The classical electromagnetic wave theory of light, which
1
successfully explained interference, diffraction, and m v 2max = h ( – 0) (2)
polarization of light, could not account for the observations 2
related to photoelectric effect. In 1900 Planck postulated It is evident that when < 0, no electron is emitted
that light waves consist of tiny bundles of energy called for any intensity of light. When > 0, the energy of the
photons or quanta. The energy of a light wave of frequency electron increases linearly with the frequency of light.
is given by E = h , where h is Planck’s constant. Photon Since intensity of light is a measure of the number of
is simply a light wave of energy E. photons and since each photon emits a photoelectron on
Following Planck’s idea, Einstein proposed a theory absorption, the intensity of photoelectrons is proportional
for photoelectric effect. According to him, when a photon to the intensity of light.
of light falls on a metal, it is absorbed, resulting in the Below a certain negative voltage V0, no photoelectrons
emission of a photoelectron. The maximum kinetic energy are emitted no matter what the intensity of light is. This
Kmax = 1/2 mv 2max of the emitted electron is given by voltage is called the cut-off or stopping potential. Since
1 there is no photoelectric emission at potentials less than V0,
mv 2max = h – W0 (1) the maximum velocity v max acquired by the photoelectrons
2
is given by
This is the famous Einstein’s photoelectric equation. The
term h represents the total energy of the photon incident on 1 2
Kmax = m v max = eV0
the metal surface. The photon penetrates a distance of about 2
10–8 m before it is completely absorbed. In disappearing, the where Kmax is the maximum kinetic energy. V0 is given
photon imparts all its energy to a single electron. Part of this by
energy is used up by the electron in freeing itself from the eV0 = h ( – 0 )
atoms of the metal. This energy designated by W0 in Eq. (1)
is called the work-function of the metal and is a characteristic h
or V0 = ( – 0) (3)
of it. The rest of the energy is used up in giving the electron e
kinetic energy.
28.2 Comprehensive Physics—JEE Advanced

Laws and graphs of photoelectric effect (5) Graph of photoelectric current (i) versus voltage
(1) For a given emitter illuminated by radiation of a (V) for radiations of different intensities (I1 > I2)
given frequency, the photoelectric current is pro- but of the same frequency (Fig. 28.4).
portional to the intensity of radiation (Fig. 28.1)
1

– 0

Fig. 28.4

(6) Graph of photoelectric current (i) versus voltage


Fig. 28.1 (V) for radiations of different frequencies ( 1 > 2)
but of the same intensity (Fig. 28.5)
(2) The maximum kinetic energy (Kmax) of photoelec-
trons is proportional to the frequency ( ) of the
incident radiation and is independent of intensity
of the radiation (Fig. 28.2).

Fig. 28.5

c
(7) Threshold wavelength is 0 = . For photoelec-
tric emission < 0. 0

Fig. 28.2
28.1
Kmax = h( – 0) A metal of work function 3.3 eV is illuminated by
Slope of graph = h(Plank’s constant). light of wavelength 300 nm. Find (a) the threshold
Kmax = 0 when (threshold frequency) frequency of photoelectric emission, (b) the maxi-
mum kinetic energy of photoelectrons and (c) the
( 0) below which no photoelectrons are emitter no stopping potential. Take h = 6.6 10–34 Js.
matter what the intensity of radiation is.
(4) Graph of stopping potential (V0) versus frequency SOLUTION
of incident radiation (Fig. 28.3). (a) W0 = 3.3 eV = 3.3 1.6 10–19 J
Threshold frequency is
19
W0 3.3 1.6 10
0 = 34
h 6.6 10
= 8 1014 Hz
(b) Frequency of incident radiation is
c 3 108
= 9
300 10
Fig. 28.3
= 10 1014 Hz
h
V0 = ( – 0) Kmax = h( – 0)
e
–34
h = (6.6 10 ) (10 1014 – 8 1014)
Slope of graph = , which is the same for all
metals. e = 1.32 10–19 J
Atomic Physics 28.3

19
K max 1.32 10
(c) Kmax = eV0 V0 = 9
e 1.6 10
= 0.825 V

28.2
Photoelectric emission from a metal begins at a fre-
quency of 6 1014 Hz. The emitted electrons are fully
stopped by a retarding potential of 3.3 V. Find the
wavelength (in nm) of the incident radiation. Take Fig. 28.6
h = 6.6 10–34 Js.

SOLUTION SOLUTION
eV0 = h( – eV0 = h( – 0)
0)

(1.6 10–19) 3.3 = (6.6 10–34) ( –6 1014) h


V0 = ( 0) (1)
e
= 1.4 1015 Hz
(a) It follows from Eq. (1) that = 0 if V0 = 0.
c 3 108
= 15
= 2.14 10–7 m = 214 nm Hence 0 = 4 1015 Hz
1.4 10 h
(b) The slope of V0 versus graph =
e
28.3 16.5 0
Now slope = 15
= 4.125 10–15
Light of wavelength 300 nm is incident on two met- (8 4) 10
als A and B whose work functions are respectively h = e slope
4 eV and 2 eV. Which of the two metals will emit
= (1.6 10–19) (4.125 10–15)
photoelectrons?
= 6.6 10–34 Js
SOLUTION (c) W0 = h 0 = (6.6 10–34) (4 1015)
Energy of incident radiation is = 26.4 10–19 J
hc (6.6 10 34
) (3 108 ) = 16.5 eV
E=h = = 9
300 10
28.5
=6 10–19 J
Calculate the number of photons emitted per second
19
6 10 by a transmitter of power 10 kW sending radiowaves
= = 3.75 eV
1.6 10 19 of frequency 6 105 Hz. Take h = 6.63 10–34 Js.
Photoelectrons will be emitted from the metal if E is
SOLUTION
greater than the work function of the metal. Hence
metal B will emit photoelectrons but A will not. Let N be the number of photons emitted in time t.
Energy of 1 photon = h . Therefore, energy of N pho-
28.4 tons = Nh . Therefore, the power is
For photoelectric affect in a metal, the graph of stop- Nh
ping potential V0 (in volt) versus frequency (in Hz) P= = nh ;
t
of the incident radiation is shown in Fig. 28.6. From
n = number of photons emitted per second.

(a) threshold frequency P 10 103


or n= =
(b) Planck’s constant and h (6.63 10 34 ) 6 105
(c) work function of the metal. = 2.5 1031 photons per second
28.4 Comprehensive Physics—JEE Advanced

28.6 SOLUTION
The stopping potential of a metal is 3 V when it is c 3 108
illuminated by light of wavelength 500 nm. What = 9
= 1.2 1015 Hz
250 10
will be the stopping potential of the metal when the
- eV0 = h( – 0) = h – W0
cy and work function of the metal. Take h = 6.6 W0 = h – eV0
10–34 Js.
= 6.6 10–34 1.2 1015 – 1.6 10–19 1.2
SOLUTION =6 10–19 J = 3.75 eV
eV1 = h( 1 – 0) (1)
19
W0 6 10
eV0 = h( 2 – 0) (2) Now 0 = 34
= 0.9 1015 Hz
h 6.6 10
Subtracting, we get
Frequency of yellow light is
e(V1 – V2) = h( 1 – 2)
c 3 108
h y = = 0.5 1014 Hz
V1 – V2 = ( 1 2) y 600 10 9
e
h c c Since y < 0, the photocell will not work with yellow
= light as no photoelectrons will be emitted.
e 1 2

hc BOHR’S THEORY OF HYDROGEN LIKE


= 2 1 28.2
e ATOM
1 2

6.6 10 3
3 108 (a) Bohr’s quantization condition: The magnitude of
= 19 angular momentum of the electron in a circular
1.6 10 orbit is
7 7
6 10 5 10 nh nh
L= mvn rn
14 2 2
6 5 10
= 0.4 V where m = mass of electron, rn = radius of nth cir-
V2 = V1 – 0.4 = 3 – 0.4 = 2.6 V cular orbit, vn = orbital speed of electron in the
nth orbit, h = Planck’s constant and n is an integer
From Eq. (1), called the principal quantum number.
eV1 (b) Speed of electron in nth orbit is
0 = 1 –
h e2
Z
c eV1 vn =
= 2 0h n
1 h
where Z = atomic number of atom. For hydrogen
3 10 8
1.6 10 19
3 1
= Z = 1. For a given atom vn . Substituting the
6 10 7
6.6 10 34 n
14
known values of e, 0 and h we get
= 4.27 10 Hz
Z
W0 = h = 6.6 10 –34
4.27 1014 vn = (2.2 106 ms–1)
0 n
= 2.82 10–19 J = 1.76 eV (c) Radius of nth orbit is
h2 n2 n2
28.7 rn = 0
2
= (0.53 10–10 m)
me Z Z
Ultraviolet light of wavelength 250 nm falls on the
metal emitter of a photocell. If the stopping potential
n2
= (0.53Å)
photocell work if yellow light of wavelength 600 nm Z
is used? Take h = 6.6 10–34 Js. For a given atom rn n 2.
Atomic Physics 28.5

(d) Total energy of electron in nth orbit Spectral lines in Lyman series lie in the
Ze 2
Ze 2 ultraviolet region.
K.E. = ; P.E. = (2) Balmer series: n1 = 2, n2 = 3, 4, 5,…
8 0 rn 4 0 rn
P.E. = –2K.E. 1 1 1
= RH 2
Total energy of electron in nth object is 2 n22
En = K.E. + P.E. = K.E. – 2K.E. = –K.E. min = 364.5 nm, max = 656.1 nm
Spectral lines in Balmer series lie in the vis-
Z e2 ible region.
En = –
8 0 rn (3) Paschen series: n1 = 3, n2 = 4, 5, 6,…
Putting the value of rn, we get
1 1 1
= RH
m e4 Z2 3 2
n22
En = 2 2 2
8 0 h n min = 820.1 nm, max = 1874.6 nm
Z 2 Spectral lines in Paschen series lie in the
= (–21.76 10–19 J) infrared region.
n2
2 (4) Brackett series: n1 = 4, n2 = 5, 6, 7,…
Z
= (–13.6eV)
n2 1
= RH 1 1
(e) Time period of revolution of electron in nth orbit 42 n22
is
min = 1458 nm, max = 4050 nm
2 rn 4 02 h3 n3 –16 n3
Tn = = = (1.51 10 s) These spectral lines also lie in the infrared
vn m e4 Z2 Z2 region.
(f) Frequency of revolution of the electron in nth orbit is (i) The energy of electron in hydrogen atom in the
ground state is –13.6 eV.
1 Z2 (j) The ionization potential of hydrogen atom in the
n = = (6.6 1015 Hz)
Tn n3 ground state is 13.6 V. Ionization potential of a
(g) Wavelength of emitted radiation: When an electron 13.6 Z 2
hydrogen like atom in nth state = volt.
jumps from a higher energy state n = n2 to a lower n2
state n = n1, a photon of energy h of radiation is
emitted.
28.8
h = En2 En1
The energy required to excite a hydrogen atom from
hc m e4 1 1 n = 1 to n = 2 energy state 10.2 eV. What is the wave-
= 2 2
Z2 length of the radiation emitted by the atom when it
8 0 h n22 n12
goes back to its ground state?
1 m e4 1 1
= 2 3
Z2 SOLUTION
8 0 h c n12 n22
Given E2 – E1 = 10.2 eV = 10.2 1.6 10–19 J.
1 1 1 Therefore, frequency the emitted radiation is
= R HZ 2
n12 n22 E2 E1
=
m e4 h
where RH = = 1.097 107 m–1 is called and wavelength is
8 02 h3c
c ch
Rydberg constant. =
(h) Main Series of Hydrogen Spectrum (Z = 1) E2 E1
(1) Lyman series: n1 = 1, n2 = 2, 3, 4,… 3 108 6.6 10 34
= 19
1 1 1 10.2 1.6 10
= RH
12 n22 = 1.22 10–7 m
min = 91.2 nm, max = 121.6 nm = 1220 Å = 122 nm
28.6 Comprehensive Physics—JEE Advanced

28.9 28.12
The ionization potential of the hydrogen atom is 13.6 V. The wavelength of the second line of Balmer series
Find the energy of the atom in n = 2 energy state.
of Lyman series?
SOLUTION
Energy of hydrogen atom in the ground state is SOLUTION
( ionization potential is 13.6 V) Wavelengths in Balmer series for hydrogen are given by
E1 = – 13.6 eV 1 1 1
= RH 2
1 2 n2
Since En , the energy in the n = 2 state is
n2 1 1
E1 13.6 = RH ; n 3, 4, 5 ...
E2 = = – 3.4 eV 4 n2
2
( 2) 4
The second line in Balmer series corresponds to n = 4.
Hence
28.10 1 1 1 3RH 16
= RH or 2
The innermost orbit of hydrogen atom has a diameter 4 16 16 3RH
2
of 1.06 Å. What is the diameter of the 10th orbit?
n = 2) in Lyman
SOLUTION series is
1 1 1
Given d1 = 1.06 Å. We know that dn n2d1. Hence = RH 1 2
RH 1
2 4
d10 = (10)2 1.06 Å 1
3RH
= 106 Å =
4
4
28.11 or 1 =
3RH
Find the ratio of longest and shortest wavelength in
the Lyman series of hydrogen atom. 1 4 3RH 1
=
2 3RH 16 4
SOLUTION
For Lyman series 2 486.4
or 1 =
4 4
1 1 1
= RH 2 = 121.6 nm
1 n2
where n = 2, 3, 4, ... . The shortest wavelength ( s)
28.3 X-RAYS
corresponds to n = and the longest wavelength ( l)
corresponds to n = 2. X-rays are produced when energetic electrons fall on a
1 1 suitable target. The apparatus used for the production
= RH 1 RH of X-rays is called the Coolidge tube in which electrons
s
are produced by thermionic emission. X-rays are elec-
1 tromagnetic waves of wavelength of the order of 1Å or
s =
RH 0.001 nm.
1 1 3RH 1. Duane-Hunt Law
and = RH 1
l 4 4 The shortest X-ray wavelength emitted when electrons
incident on the target are accelerated through a potential
4
l = V volts is given by
3RH
hc 1239.6
4 min = nm
l
= eV V
s 3 This is called the Duane-Hunt Law.
Atomic Physics 28.7

2. Diffraction of X-rays 5. The characteristic X-rays consist of K and L series.


A crystal is a natural grating for diffraction of X-rays, since The K X-rays are produced when electrons jump
the spacing d between the crystal planes is of the order of from n = 2 to n = 1 orbit. They are called L when
the wavelength of X-rays. The angle of diffraction of electrons jump from n = 3 to n = 2 orbit. The wave-
X-rays is given by the relation length of the K line of the X-ray spectrum is given by
n=¥
2d sin = n K series L series
limit limit
where n is an integer having values 1, 2, 3, etc. and n = 4 (N shell)
is the wavelength of X-rays incident on the crystal. This Kg Lb Ma
relation is called Bragg’s equation. n = 3 (M shell)

3. Absorption of X-rays Kb La
n = 2 (L shell)
X-rays are absorbed by materials following the exponential
relation Ka
I = I0 e– x
where I0 is the initial intensity of X-rays, I their intensity
after they have traversed a thickness x and is the n = 1 (K shell)

Fig. 28.7
4. X-ray Spectra
1 1 1
= RH(Z – 1)2 2
1 22
spectrum and characteristic spectrum. X-ray spectrum
consists of a series of discrete spectral lines superimposed where R is the Rydberg constant and Z is the atomic
on a continuous luminous background. The background number. The wavelength of L line is
spectrum consists of all sorts of wavelengths and is called 1 1 1
= RH(Z 7 4)2 2
the continuous spectrum which is the same for all target 2 32
materials. The discrete line spectrum is a characteristic of 6. The frequency of the characteristics X-rays is
the target metal and is, therefore, called the characteristic proportional to (Z b)2 where b is a constant.
spectrum. The spectral lines in this spectrum are due to the
7. X-rays are absorbed by materials according to the
transitions of electrons from the outer orbits to the inner
relation
K, L, M shells, etc. The emitted radiations are called K ,
K , K , radiations. These transitions are shown in Fig. I = I0 e x
28.7. where I0 = incident intensity, = constant for the
The frequency ( ) of the characteristic X-rays is given material and x = distance penetrated.
related to the atomic number (Z) of the target metal by the
relation 28.13
= a (Z – b) What is the maximum frequency of X-rays emitted
where a and b are constants. This relation is known as from an X-ray tube operating at 50 kV?
Moseley’s law.
Some facts about X-rays can be summerized as SOLUTION
follows: eV = h max. Hence
1. X-rays are electromagnetic waves of wavelength of 19
eV (1.6 10 ) (50 103 )
the order of 1 Å or 0 001 nm. max = 34
2. h (6.63 10 )
= 1.2 1019 Hz
3. X-rays travel in vacuum at the speed of light,
c = 3 108 ms 1. 28.14
4. If electrons are accelerated through a potential dif-
ference V and then are made to fall on a target, An X-ray tube produces a continuous spectrum of
X-rays of wavelength greater than hc/eV are radiation with its shortest wavelength end at 0.66 Å.
produced. The shortest wavelength emitted is What is the maximum energy of a photon of this
radiation? Take h = 6.6 10–34 Js
min = hc/eV.
28.8 Comprehensive Physics—JEE Advanced

SOLUTION h
=
hc 2 m qV
Emax = h max = ( c= )
min
For an electron,
34 1/ 2
(6.6 10 ) (3 108 ) 150
= 10
= Å
(0.66 10 ) V
=3 10–15 J 4. For a gas molecule of mass m at temperature T
kelvin, the de Broglie wavelength is given by
28.15 h
The potential difference applied to an X-ray tube is =
3m k T
5 kV and the current through it is 3.2 mA. Find the
where k is the Boltzmann constant.
number of electrons striking the target per second.
28.16
SOLUTION
Calculate the wavelength associated with a dust par-
I 3.2 10 3 ticle of mass 1 g moving with a velocity of 106 ms–1.
n= = =2 1016 electrons per second
e 1.6 10 19 Given h = 6.6 10–34 Js.

SOLUTION
28.4 WAVE NATURE OF MATTER m = 1 g = 10–6 g = 10–9 kg
In 1924, Louis de Broglie, a French theoretical physi- h 6.6 10 34
cist, derived an equation which predicted that all atomic = =
mv 10 9 106
wavelength. Under certain circumstances, a beam of = 6.6 10–31 m
electrons or atoms will behave like a group of waves. On
the basis of theoretical considerations, de Broglie pre- 28.17
dicted that the wavelength of these waves is given by Calculate de Broglie wavelength of an electron hav-
h h ing kinetic energy of 1 BeV. Given mass of electron
= =
p mv (m) = 9.1 10–31 kg, h = 6.6 10–34 Js and e = 1.6
where h is Planck’s constant and p is the momentum 10–19 C.
of the particles. This equation is known as de Broglie’s
wave equation. For an electron moving at a high speed, SOLUTION
the momentum is large and the wavelength is small. K = 1 BeV = 109 eV = 109 1.6 10–19 = 1.6 10–10 J
The faster the electron, the shorter is the wavelength. 34
Notice that the particle need not have a charge to have h 6.6 10
= =
2mK 31 10
an associated wave. This is why de Broglie waves are 2 9.1 10 1.6 10
sometimes referred to as matter waves.
= 3.87 10–14 m
1. If the rest mass of a particle is m0, its de Broglie
wavelength is given by 28.18
2 1/ 2
v Calculate de Broglie wavelength associated with an
h 1 electron accelerated through a potential difference
c2
= of 200 V. Given m = 9.1 10–31 kg and h = 6.6
m0 v
10–34 Js.
2. In terms of kinetic energy K, de Broglie wavelength
is given by SOLUTION
h
= K = 200 eV = 200 1.6 10–19 = 3.2 10–17 J
2m K 34
h 6.6 10
3. If a particle of charge q is accelerated through a = =
2mK 31 17
potential difference V, its de Broglie wavelength is 2 9.1 10 3.2 10
given by = 0.86 10–10 m = 0.86 Å
Atomic Physics 28.9

28.19 28.21
Calculate the wavelength of de Broglie waves asso- Ultraviolet light of wavelength 99 nm falls on a metal
ciated with a neutron at room temperature of 27°C. plate of work function 1.0 eV. Find the wavelength
Given mass of neutron (m) = 1.67 10–27 kg, Boltz- of the fastest photoelectron emitted. Mass of electron
man constant (k) = 1.38 10–23 JK–1 and h = 6.63 (m) = 9.1 10–31 kg and h = 6.6 10–34 Js.
10–34 Js.
SOLUTION
SOLUTION
hc
Kinetic energy of neutron due to thermal speed is Kmax = h – W0 = W0
3
K = kT, where T = 273 + 27 = 300 K. 34
2 (6.6 10 ) (3 108 )
h h h =
= 99 10 9
2mK 3 3mkT
2m kT – 1.0 1.6 10–19
2
Substituting the values of h, m, k and T, we get =2 10–18 – 1.6 10–19
= 1.45 10–10 m = 1.45 Å = 1.84 10–18 J
28.20 h
=
A photon of wavelength 19.8 nm collides with an 2mK max
electron at rest. After the collision, the wavelength of
34
the photon is found to be 30 nm. Is the collision elas- 6.6 10
=
tic or inelastic? Calculate the energy of the scattered 2 (9.1 10 31
) 1.84 10 18
electron. Given h = 6.6 10–34 Js.
= 3.6 10–10 m = 0.36 nm
SOLUTION
Energy of photon before collision is 28.22
hc (6.6 10 34
) 8
(3 10 ) A proton and an electron have equal kinetic energy.
Ei = h 1 = 9 Which of the two has a greater de Broglie wave-
1 19.8 10 length?
= 10 10–18 J
SOLUTION
Energy of photon after collision is
hc (6.6 10 34
) (3 108 ) h h
Ef = p = , e
30 10 9 2m p K 2me K
2

= 6.6 10–18 J mp
e
Since there is a loss of kinetic energy ( Ef < Ei), = >1 ( m p > m e)
p me
the collision is inelastic. The energy of the scattered
electron = Ei – Ef = 3.4 10–18 J. Hence the electron has the greater wavelength

Multiple Choice Questions with Only One Choice Correct


1. Figure 28.8 shows the variation of photoelectric If Ia and Ib are the intensities of the incident radia-
current (i) with voltage (V) between the electrodes tions and a and b are the respective frequencies,
in a photo-cell for two different radiations. then
28.10 Comprehensive Physics—JEE Advanced

(a) Ia > Ib, b < a (b) Ia < Ib, b > a cities. The ratio of the de Broglie wavelength of the
(c) Ia > Ib, = (d) Ia < Ib, < lighter particle to that of the heavier particle is
a b b a
1
(a) (b) 1
2
i 1
(c) 2 (d)
4
6. A potential difference V is applied across an X-ray
tube. If e/m is the charge to mass ratio of an elec-
b tron and c the speed of light in vacuum, the ratio of
a the de Broglie wavelength of the incident electrons
to the shortest wavelength of X-rays produced is
0 V
given by

Fig. 28.8
1 eV 1 eV
(a) (b)
c 2m c m
2. When a radiation of energy 5 eV falls on a surface,
the emitted photoelectrons have a maximum kinetic 1 2 eV 2 eV
(c) (d)
energy of 3 eV. The stopping potential is c m c m
(a) 2 V (b) 3 V 7. The de Broglie wavelength of the electron in the nth
(c) 5 V (d) 8 V energy state of a hydrogen atom is proportional to
3. When a radiation of wavelength 1 falls on a sur- 1 1
(a) (b) 2
face, the maximum kinetic energy of the emitted n n
photoelectrons is K1. For a radiation of wavelength (c) n (d) n2
2, the maximum kinetic energy is K2. If 1 = 2 /2, 8. The magnitude of the magnetic moment of the elec-
then tron in the nth energy state of a hydrogen atom is
(a) K1 = 2K2 (b) K2 = 2K1 proportional to
(c) K1 > 2K2 (d) K1 < 2K2 1
(a) (b) n
4. Figure 28.9 represents the observed intensity (I) of n
X-rays emitted by an X-ray tube as a function of (c) n (d) n2
wavelength ( ). 9. The shortest wavelength in the Brackett series of
(a) Peaks A and B represent K lines a hydrogen like atom of atomic number Z is equal
(b) Peaks A and B represents K lines to the shortest wavelength in the Balmer series of
(c) Peak A represents K line and peak B repre- hydrogen atom. The value of Z is
sents K line (a) 2 (b) 3
(d) Peak A represents K line and peak B repre- (c) 4 (d) 5
sents K line 10. The difference between (n + 1)th Bohr radius and
nth Bohr radius is equal to the (n – 1)th Bohr radius.
The value of n is
I
B (a) 1 (b) 2
A (c) 3 (d) 4
11.
atom due to the motion of an electron in the nth
orbit is inversely proportional to
(a) n2 (b) n3
4
(c) n (d) n5
l
12. The magnitude of the angular momentum of an
Fig. 28.9 electron revolving in a circular orbit of radius r in
a hydrogen atom is proportional to
5. A particle at rest disintegrates into two fragments
(a) r1/2 (b) r
of masses in the ratio of 1 : 2 having non-zero velo-
(c) r3/2 (d) r2
Atomic Physics 28.11

13. The wavelength in air associated with a photon of monochromatic light of wavelength 2 , the stop-
energy E is (c is the speed of light in air and h is the ping potential is V0/2. The threshold wavelength for
Planck’s constant) photoelectric emission is
hc h (a) 1.5 (b) 2.0
(a) (b) (c) 2.5 (d) 3
E cE
20. The threshold frequency for a certain photosensi-
c
(c) (d) hcE tive metal is 0. When it is illuminated by light of
hE frequency = 2 0, the stopping potential for pho-
14. An image of the sun is formed by a lens of toelectric current is V0. What will be the stopping
focal length 30 cm on the metal surface of a photo- potential when the same metal is illuminated by
electric cell and it produces a current I. The lens light of frequency = 3 0?
forming the image is then replaced by another lens (a) 1 5 V0 (b) 2 V0
of the same diameter but of focal length 15 cm. The (c) 2 5 V0 (d) 3 V0
photoelectric current in this case will be 21. The threshold frequency for a certain photosensi-
tive metal is 0. When it is illuminated by light of
(a) I / 2 (b) 2 I
frequency = 2 0, the maximum velocity of pho-
(c) I (d) 4 I toelectrons is v0. What will be the maximum veloc-
15. The momentum of a photon of wavelength is ity of the photoelectrons when the same metal is
h
(a) (b) h c illuminated by light of frequency = 5 0?
c (a) 2 v0 (b) 2 v0
h (c) 2 2 v0 (d) 4 v0
(c) h (d)
22. The threshold wavelength for a metal whose
16. Violet light can cause photoelectric emission from work function is W0 is 0. What is the threshold
a metal but blue light cannot. If sodium light is wavelength for a metal whose work function is
incident on the metal, then W0/2?
(a) the photoelectric current decreases
0 0
(b) the number of photoelectrons ejected per (a) (b)
4 2
second increases
(c) the velocity of photoelectrons increases (c) 2 0 (d) 4 0
(d) no photoelectric emission occurs. 23. The minimum wavelength of X-rays emitted from
17. 1020 photons of wavelength 660 nm are emitted an X-ray tube operating at a voltage of 104 volts is
per second from a lamp. What is the wattage of the roughly equal to
lamp. Planck’s constant = 6 6 10 34 Js (a) 1 Å (b) 1 5 Å
(a) 30 W (b) 60 W (c) 2 Å (d) 2 5 Å
(c) 100 W (d) 500 W 24. The energy in monochromatic X-rays of wave-
18. When a certain photosensitive surface is illumi- length 1 Å is roughly equal to
nated with monochromatic light of frequency , (a) 2 10 15 J (b) 2 10 16 J
the stopping potential for photoelectric current is (c) 2 10 17 J (d) 2 10 18 J
V0/ 2. When the same surface is illuminated by 25. When the accelerating voltage applied on the
monochromatic light of frequency /2, the stop- electrons, in an X-ray tube, is increased beyond a
ping potential is V0. The threshold frequency for critical value
photoelectric emission is (a) the spectrum of white radiation is unaf-
2 3 fected
(a) (b)
3 2 (b) only the intensities of various wavelengths
3 5 are increased
(c) (d) (c) only the wavelength of characteristic radia-
5 3
tion is affected
19. When a certain photosensitive surface is illumi-
(d) the intensities of characteristic lines relative
nated with monochromatic light of wavelength ,
to the white spectrum are increased but there
the stopping potential for photoelectric current is
is no change in their wavelength.
2V0. When the same surface is illuminated with
28.12 Comprehensive Physics—JEE Advanced

26. In the continuous part of the spectrum of X-rays the 32. X-rays are incident normally on a crystal of
limiting frequency is lattice constant 0
(a) inversely proportional to the potential through diffraction from the crystal occurs at an angle of
which electrons have been accelerated 30°. What is the wavelength of X-rays used?
(b) is directly proportional to the accelerating (a) 0 3 nm (b) 0 6 nm
potential (c) 1 2 nm (d) 2 4 nm
(c) not dependent upon the accelerating poten- 33. X-rays of wavelength are incident normally on a
tial -
(d) is dependent upon the nature of the target tion from the crystal is observed at an angle of 45°.
material. The lattice constant of the crystal is
27. The potential difference applied to an X-ray tube is
increased. As a result, in the emitted radiation, (a) (b) 2
(a) the intensity increases 2
(b) the minimum wavelength increases (c) (d) 2
(c) the intensity decreases 34. The frequency of K line of a source of atomic
(d) the minimum wavelength decreases. number Z is proportional to
IIT, 1988 (a) Z2 (b) (Z 1)2
28. In an X-ray tube, electrons accelerated through a (c) 1/Z (d) Z
very high potential difference strike a metal target. 35. The wavelength of K line from an element of
If the potential difference is increased, the speed of atomic number 41 is . Then the wavelength of K
the emitted X-rays line of an element of atomic number 21 is
(a) increases (a) 4 (b) /4
(b) decreases (c) 3 08 (d) 0 26
(c) remains unchanged IIT, 2005
(d) is always equal to 3 108 ms 1. 36. The continuous X-ray spectrum is produced due to
29. An X-ray tube is operated at 66 kV. Then, in the (a) acceleration of electrons towards the nuclei
continuous spectrum of the emitted X-rays of the target atoms
(a) wavelengths 0.01 nm and 0.02 nm will both (b) retardation of energetic electrons when they
be present approach the nuclei of the target atoms
(b) wavelengths 0.01 nm and 0.02 nm will both (c) fall of the electrons of the target atoms from
be absent higher energy level to lower energy levels
(c) wavelengths 0.01 nm will be present but
(d) knocking out of the electrons from the target
wavelength 0.02 nm will be absent
atoms by the fast moving incident electrons
(d) wavelength 0.01 nm will be absent but wave-
length 0.02 nm will be present. 37. An X-ray photon of wavelength and frequency
collides with an electron and bounces off. If and
30. The minimum wavelength of X-rays produced in are respectively the wavelength and frequency
an X-ray tube is when the operating voltage is of the scattered photon, then
V. What is the minimum wavelength of the X-rays
(a) = ; = (b) < ; >
when the operating voltage is V/2?
(c) > ; > (d) > ; <
(a) (b) 38. The binding energy of the innermost electron in
2
tungsten is 40 keV. To produce characteristic
(c) 2 (d) 4 X-rays using a tungsten target in an X-ray tube, the
31. The maximum frequency of X-rays produced in accelerating voltage should be greater than
an X-ray tube is when the operating voltage is (a) 4 kV (b) 40 kV
V. What is the maximum frequency of the X-rays (c) 400 kV (d) 4000 kV
when the operating voltage is V/2? 39. The shortest wavelength of X-rays emitted from an
X-ray tube depends upon
(a) (b)
2 (a) the current in the tube
(c) 2 (d) 4 (b) the voltage applied to the tube
Atomic Physics 28.13

(c) the nature of the gas in the tube 48. The shortest wavelength in the Lyman series
(d) the atomic number of the target material is 911 6 Å. Then the longest wavelength in the
IIT, 1982 Lyman series is
(a) 1215 Å (b)
40. Which one of the following parameters of the
(c) 2430 Å (d) 600 Å
emitted X-rays increases when the potential dif-
ference between the electrodes of an X-ray tube is 49. If electron orbits with principal quantum numbers
increased? n > 3 were not allowed, the number of possible
(a) intensity (b) frequency elements would be
(c) wavelength (d) speed (a) 28 (b) 90
(c) 32 (d) 64
41. A proton, when accelerated through a potential dif-
50. Pauli’s exclusion principle states that no two
ference of V volts, has a wavelength associated
electrons in an atom can have identical values for
with it. If an alpha particle is to have the same
(a) one of the four quantum numbers
wavelength , it must be accelerated through a
(b) two of the four quantum numbers
potential difference of
(c) three of the four quantum numbers
(a) V/ 8 volts (b) V/4 volts
(d) all four quantum numbers
(c) 4 V volts (d) 8 V volts
51. Which energy state of the triply ionized berylli-
42. Two particles of masses m and 2m have equal
um (Be+++) has the same electron orbital radius as
kinetic energies. Their de Broglie wavelengths are
that of the ground state of hydrogen? Given Z for
in the ratio of
beryllium = 4.
(a) 1 : 1 (b) 1 : 2 (a) n = 1 (b) n = 2
(c) 1 : 2 (d) 2 : 1 (c) n = 3 (d) n = 4
43. The de Broglie wavelength of a neutron at 927°C is 52. In Q. 51, what is the ratio of the energy state of
. What will be its wavelength at 27°C? beryllium and that of hydrogen?
(a) 1 (b) 2
(a) (b) (c) 3 (d) 4
2
(c) 2 (d) 4 53. Which energy state of doubly ionized lithium (Li++)
44. The de Broglie wavelength of a neutron when its has the same energy as that of the ground state of
kinetic energy is K is . What will be its wavelength hydrogen? Given Z for lithium = 3.
when its kinetic energy is 4K? (a) n = 1 (b) n = 2
(c) n = 3 (d) n = 4
(a) (b) 54. In Q. 53, what is the ratio of the electron orbital
4 2
(c) 2 (d) 4 radius of Li++ to that of hydrogen?
(a) 1 (b) 2
45. What is the de Broglie wavelength of an electron (c) 3 (d) 4
of energy 180 eV? Mass of electron = 9 10 31 kg
55.
and Planck’s constant = 6 6 10 34 Js.
state of hydrogen is 3 4 eV. What is the kinetic
(a) 0 5 Å (b) 0 9 Å energy of the electron in this state?
(c) 1 3 Å (d) 1 8 Å (a) + 1 7 eV (b) + 3 4 eV
46. Moving with the same velocity, which of the (c) + 6 8 eV (d) + 13 4 eV
following has the longest de Broglie wavelength? 56. In Q. 55, the potential energy of the electron is
(a) -particle (b) -particle (a) – 1 7 eV (b) – 3 4 eV
(c) proton (d) neutron (c) – 6 8 eV (d) – 13 4 eV
47. The de Broglie wavelength of an electron moving 57.
with a velocity 1 5 108 ms 1 is equal to that of a the hydrogen spectrum is . What is the wavelength
photon. The ratio of the kinetic energy of the elec- of the second line.
tron to that of the photon is 20 3
(a) 2 (b) 4 (a) (b)
27 16
1 1
(c) (d) 5 3
2 4 (c) (d)
IIT, 2004 36 4
28.14 Comprehensive Physics—JEE Advanced

58. 66. If a hydrogen atom at rest, emits a photon of wave-


the hydrogen spectrum is . What is the frequency length , the recoil speed of the atom of mass m is
of the corresponding line in the spectrum of doubly given by
ionized Lithium? h mh
(a) (b) 3 (a) (b)
m
(c) 9 (d) 27
59. The energy difference between the first two (c) mh (d) none of these
levels of hydrogen atom is 10 2 eV. What is the 67. If elements with principal quantum number n > 4
corresponding energy difference for a singly were not allowed in nature, the number of possible
ionized helium atom? elements would be
(a) 10 2 eV (b) 20 4 eV (a) 60 (b) 32
(c) 40 8 eV (d) 81 6 eV (c) 4 (d) 64
60. The ionization energy of hydrogen atom is 13.6 eV. IIT, 1983
What is the ionization energy of helium atom? 68. When a monochromatic source of light is at a
(a) 3.4 eV (b) 13.6 eV distance of 0.2 m from a photoelectric cell, the
(c) 54.4 eV (d) 108.8 eV cut-off voltage and the saturation current are
61. The ionization energy of hydrogen atom is 13.6 eV. respectively 0.6 V and 18 mA. If the same source
Hydrogen atoms in the ground state are excited by is placed 0.6 m away from the cell, then
electromagnetic radiation of energy 12.1 eV. How (a) the stopping potential will be 0.2 V
many spectral lines will be emitted by the hydrogen (b) the stopping potential will be 1.8 V
atom? (c) the saturation current will be 6.0 mA
(a) one (b) two (d) the saturation current will be 2.0 mA
(c) three (d) four IIT, 1992
62. If an orbital electron of the hydrogen atom jumps 69. The energy of a photon of frequency is E = h
from the ground state to a higher energy state, its and the momentum of a photon of wavelength
orbital speed reduces to half its initial value. If the is p = h / . From this statement one may con-
radius of the electron orbit in the ground state is r, clude that the wave velocity of light is equal to
then the radius of the new orbit would be E
(a) 2r (b) 4r (a) 3 10 8 ms–1 (b)
p
(c) 8r (d) 16r
2
63. The orbital speed of the electron in the ground state (c) E p (d)
E
of hydrogen is v. What will be its orbital speed p
when it is excited to the energy state 3.4 eV? 70. When a centimetre thick surface is illuminated with
v light of wavelength , the stopping potential is V.
(a) 2 v (b)
2 When the same surface is illuminated by light of
v v wavelength 2 , the stopping potential is V/3. The
(c) (d) threshold wavelength for the surface is
4 8
64. In the Bohr model of the hydrogen atom, the ratio 4
(a) (b) 4
of the kinetic energy to the total energy of the elec- 3
tron in a quantum state n is 8
(c) 6 (d)
(a) – 1 (b) + 1 3
1 1 71. Energy levels A, B and C of a certain atom corresp-
(c) (d) 2 ond to increasing values of energy, i.e. EA < EB < EC.
n n
If 1, 2 and 3 are the wavelengths of radiations
65. The ratio of the wavelengths of the longest wave-
corresponding to transitions C to B, B to A and C to
length lines in the Lyman and Balmer series of
A respectively, which of the following relations is
hydrogen spectrum is
correct?
3 5
(a) (b) (a) = + (b) = 1 2
23 27 3 1 2 3
1 2
7 9
(c) (d) (c) + + =0 (d) 2
= 2
+ 2
29 31 1 2 3 3 1 2
Atomic Physics 28.15

72. Figure 28.10 represents the observed intensity (I) (c) 6V (d) 10V
of X-rays emitted by an X-ray tube, as a function of IIT, 1997
wavelength ( ). The sharp peaks A and B denote 78. The electron in a hydrogen atom makes a transition
(a) band spectrum n1 n2 where n1 and n2 are the principal quantum
(b) continuous spectrum numbers of the two states. Assume the Bohr model
(c) characteristic radiations to be valid. The time period of the electron in the
(d) white radiations.
possible values of n1 and n2 are
(a) n1 = 4, n2 = 1 (b) n1 = 8, n2 = 2
(c) n1 = 8, n2 = 1 (d) n1 = 6, n2 = 3
IIT, 1998
79. X-rays are produced in an X-ray tube operating at a
given accelerating voltage. The wavelength of the
continuous X-rays has values from
(a) 0 to
(b) min to where min > 0
Fig. 28.10
(c) 0 to max where max <
(d) min to max where 0 < min < max < .
73. In a photo-emissive cell, with exciting wavelength IIT, 1998
, the fastest electron has a speed v. If the exciting
80. The work function of a substance is 4.0 eV. The lon-
wavelength is changed to 3 / 4, the speed of the
gest wavelength of light that can cause photoelec-
fastest emitted electron will be
tron emission from this substance is approximately
3 4 (a) 540 nm (b) 400 nm
(a) v (b) v
4 3 (c) 310 nm (d) 220 nm
4 4 IIT, 1998
(c) less than v (d) more than v
3 3 81. A particle of mass M at rest decays into two
74. An energy of 24.6 eV is required to remove one particles of masses m1 and m2, having non zero
of the electrons from the neutral helium atom. velocities. The ratio of the de Broglie wavelengths
The energy (in eV) required to remove both the of the particles, 1/ 2, is
electron from a neutral helium atom is m1 m2
(a) 38.2 (b) 49.2 (a) (b)
m2 m1
(c) 51.8 (d) 79.0
IIT, 1995 m2
(c) 1.0 (d)
75. As per Bohr model, the minimum energy (in eV) m1
required to remove an electron from the ground IIT, 1998
state of doubly ionized Li atom (Z = 3) is 82. Imagine an atom made up of a proton and a
(a) 1.51 (b) 13.6 hypothetical particle of double the mass of the
(c) 40.8 (d) 122.4 electron but having the same charge as the electron.
IIT, 1997 Apply the Bohr atom model and consider all possible
76. The K X-ray emission line of tungsten occurs at
= 0.021 nm. The energy difference between K and excited level. The longest wavelength photon that will
L levels in this atom is about be emitted has wavelength (given in terms of the
(a) 0.51 MeV (b) 1.2 MeV Rydberg constant R for the hydrogen atom) equal to
(c) 59 keV (d) 136 eV 9 36
(a) (b)
IIT, 1997 5R 5R
77. The maximum kinetic energy of photoelectrons 18 4
emitted from a surface when photons of energy (c) (d)
5R R
6 eV fall on it is 4 eV. The stopping potential is
(a) 2V (b) 4V IIT, 2000
28.16 Comprehensive Physics—JEE Advanced

83. Electrons with energy 80 keV are incident on the E


tungsten target of an X-ray tube. K shell electrons (c) Ec (d)
c2
of tungsten have – 72.5 keV energy. X-rays emitted
88. The kinetic energy of the most energetic photoelec-
by the tube contain
trons emitted from a metal surface is doubled when
(a) a continuous X-ray spectrum (Bremsstrahl-
the wavelength of the incident radiation is reduced
ung) with a minimum wavelength of about
from 1 to 2. The work function of the metal is
0.155Å
hc hc
(b) a continuous X-ray spectrum (Bremsstrahl- (a) (2 2 – 1) (b) (2 1 – 2)
ung) with all wavelengths 1 2 1 2

(c) the characteristic X-ray spectrum of tungsten hc hc


(c) ( 1 + 2) (d) ( 1 – 2)
(d) a continuous X-ray spectrum (Bremsstrah- 1 2 1 2
lung) with a minimum wavelength of about 89. The slope of the graph of the frequency of inci-
0.155 Å and the characteristics X-ray spec- dent light versus the stopping potential for a given
trum of tungsten. metallic surface is
IIT, 2000 h
(a) h (b)
84. Photoelectric emission is observed from a metallic e
e
surface for frequencies v1 and v2 of the incident (c) (d) eh
h
light (v1 > v2). If the maximum values of kinetic
energy of the photoelectrons emitted in the two 90. Lights of two different frequencies, whose photons
cases are in the ratio 1 : n, then the threshold have energies 2 eV and 10 eV respectively, succes-
frequency of the metallic surface is sively illuminate a metal of work function 1 eV.
The ratio of the maximum speeds of the emitted
v1 v2 nv1 v2 electrons will be
(a) (b)
n 1 n 1 (a) 1 : 5 (b) 3 : 11
nv2 v1 v1 v2 (c) 1 : 9 (d) 1 : 3
(c) (d) 91. The mass of a photon of wavelength is given by
n 1 n
h
85. If 0 is the de Broglie wavelength for a proton (a) h c (b)
c
accelerated through a potential difference of
hc h
100 V, the de Broglie wavelength for -particle (c) (d)
accelerated through the same potential difference c
is 92. The de-Broglie wavelength of an electron moving
(a) 2 2 (b) 0 in the nth Bohr orbit of radius r is given by
0
2 2 r
(a) (b) n r
0 0 n
(c) (d)
2 2 2 nr nr
(c) (d)
86. The de-Broglie wavelength of a particle moving 2
with a velocity 2.25 108 m/s is equal to the wave- 93. The momentum of a particle of mass m and charge
length of a photon. The ratio of kinetic energy of q is equal to that of a photon of wavelength . The
the particle to the energy of the photon is speed of the particle is given by
1 3 h h
(a) (b) (a) (b)
8 8 m qm
mh
5 7 (c) qh (d)
(c) (d)
8 8 94. The kinetic energies of photoelectrons emitted
87. A radiation of energy E falls normally on a perfectly from a metal are K1 and K2 when it is irradiated
with lights of wavelength 1 and 2 respectively.
the surface is The work function of the metal is
E 2E K1 1 K2 2 K1 1 K2 2
(a) (b) (a) (b)
c c 2 1 2 1
Atomic Physics 28.17

K1 K2 K1 K2 (a) a photon of energy 3.4 eV and an electron


2 1 2 1
(c) (d) of energy 1.0 eV are released.
2 1 2 1
(b) a photon of energy 10.2 eV and an electron
95. When the energy of the incident radiation is of energy 1.0 eV are released
increased by 20%, the kinetic energy of the pho- (c) two photons of energy 10.2 eV are released
toelectrons emitted from a metal increased from (d) two photons of energy 3.4 eV and 1.0 eV are
0.5 eV to 0.8 eV. The work function of the metal is
released.
(a) 0.65 eV (b) 1.0 eV IIT, 2005
(c) 1.3 eV (d) 1.5 eV
100. Ultraviolet light of wavelengths 1 and 2 (with
96. An electron of mass m is moving such that its mo-
mentum is equal to that of a photon of wavelength 2 > 1) when allowed to fall on hydrogen atoms

. The velocity of the electron is (h is the Planck’s in their ground state is found to liberate electrons
constant) with kinetic energies E1 and E2 respectively. The
value of the Planck’s constant can be found from
h mh
(a) (b) the relation
m IIT, 1983
2h 1
(c) (d) mh (a) h = ( 2 – 1)(E1 – E2)
m c

97. The radius of hydrogen atom in the ground state 1


(b) h = ( 2 + 1)(E1 + E 2)
is 0.53 Å. After collision with an electron, it is c
found to have a radius of 2.12 Å. What is the ( E1 E2 ) 1 2
principal quantum number n (c) h =
c( 2 1)
the atom?
(a) n = 1 (b) n = 2 ( E1 E2 ) 1 2
(d) h =
(c) n = 3 (d) n = 4 c( 2 1)
IIT, 2003 101. The wavelength of the characteristic X-ray K line
98. Figure 28.11 shows the variation of photoelectric emitted by a hydrogen like element is 0.32 Å. The
current (i) with anode potential (V) for a photo- wavelength of K line emitted by the same element
sensitive surface for two radiations of intensities will be
Ia and Ib and frequencies a and b for the curves a IIT, 1990
and b respectively. It follows from the graph that (a) 0.21 Å (b) 0.27 Å
(a) a = b, Ib < Ia (b) a = b, Ib > Ia (c) 0.34 Å (d) 0.40 Å
(c) a < b, Ib > Ia (d) a < b, Ib = Ia
102. The potential energy
( IIT, 2004 U of a moving parti-
cle of mass m varies U
with x as shown in E
i
b Fig. 28.12. The de-
Broglie wavelengths
a
of the particle in the
0 1 x
regions 0 x 1 and
x > 1 are 1 and 2 Fig. 28.12
respectively. If the
-VO O V
total energy of the particle is nE, the ratio 1/ 2 is
Fig. 28.11 IIT, 2005
99. A photon of energy 10.2 eV undergoes an inelas- n n
tic collision with a hydrogen atom in the ground (a) (b)
(n 1) (n 1)
state. After a few microseconds, another photon of
energy 14.6 eV collides inelastically with the same n2 n2
(c) (d)
hydrogen atom. In these processes (n 2 1) (n 2 1)
28.18 Comprehensive Physics—JEE Advanced

103. The wavelength of K line from an element of 107. Photoelectric effect experiments are performed
atomic number 41 is . Then the wavelength of K using three different metal plates p, q and r hav-
line of an element of atomic number 21 is ing work functions p = 2.0 eV. q = 2.5 eV and
(a) 4 (b) /4 r = 3.0 eV, respectively. A light beam containing
(c) 3.08 (d) 0.26 wavelengths of 550 nm, 450 nm and 350 nm with
IIT, 2005 equal intensities illuminates each of the plates. The
104. Electrons with de-Broglie wavelength fall on the correct I-V graph for the experiment is (see Fig.
target in an X- ray tube. The cut-off wavelength of 28.13) [Take hc = 1240 eV nm]
the emitted X-ray is
2 2h
(a) 0 = 2mc (b) 0 =
h mc
2m 2 c 2 2
(c) 0 = (d) 0 =
h2
IIT, 2007
105. The largest wavelength in the ultraviolet region
of the hydrogen spectrum is 122 nm. The smallest
wavelength in the infrared region of the hydrogen
spectrum (to the nearest integer) is
(a) 802 nm (b) 823 nm
(c) 1882 nm (d) 1648 nm
IIT, 2007
106. Which one of the following statements is Wrong
Fig. 28.13
in the context of X-rays generated from a X-ray
tube? IIT, 2009
(a) Wavelength of characteristic X-ray decreases 108.
when the atomic number of the target increases. Balmer series of hydrogen atom is 6561Å. The
(b) Cut-off wavelength of the continuous X-rays wavelength of the second spectral line in the Balm-
depends on the atomic number of the target er series of singly-ionized helium atom is
(c) Intensity of the characteristic X-rays depends (a) 1215 Å (b) 1640 Å
on the electric power given to the X-rays tube (c) 2430 Å (d) 4687 Å
(d) Cut-off wavelength of the continuous X-rays
depends on the energy of the electrons in the IIT, 2011
X-ray tube.
IIT, 2008

ANSWERS

1. (b) 2. (b) 3. (d) 4. (d) 5. (b) 6. (a)


7. (c) 8. (c) 9. (d) 10. (d) 11. (d) 12. (a)
13. (a) 14. (c) 15. (d) 16. (d) 17. (a) 18. (b)
19. (d) 20. (b) 21. (b) 22. (c) 23. (a) 24. (a)
25. (d) 26. (b) 27. (d) 28. (d) 29. (d) 30. (c)
31. (a) 32. (b) 33. (b) 34. (b) 35. (b) 36. (b)
37. (d) 38. (b) 39. (b) 40. (b) 41. (a) 42. (d)
43. (c) 44. (b) 45. (b) 46. (a) 47. (d) 48. (a)
49. (a) 50. (d) 51. (b) 52. (d) 53. (c) 54. (c)
55. (b) 56. (c) 57. (a) 58. (c) 59. (c) 60. (c)
61. (c) 62. (b) 63. (b) 64. (a) 65. (b) 66. (a)
67. (a) 68. (d) 69. (b) 70. (b) 71. (b) 72. (c)
Atomic Physics 28.19

73. (d) 74. (d) 75. (d) 76. (c) 77. (b) 78. (d)
79. (b) 80. (c) 81. (c) 82. (c) 83. (d) 84. (b)
85. (c) 86. (b) 87. (b) 88. (a) 89. (c) 90. (d)
91. (b) 92. (a) 93. (a) 94. (a) 95. (b) 96. (a)
97. (b) 98. (b) 99. (b) 100. (c) 101. (b) 102. (b)
103. (b) 104. (a) 105. (b) 106. (a) 107. (a) 108. (a)

SOLUTIONS
1. The saturation current is proportional to the inten- The shortest X-ray wavelength ( 2) corresponds to
sity of incident radiation. The slopping potential continuous X-ray spectrum and is given by
(the magnitude of V when i = 0) increases with hc
increase in frequency. Hence Ib > Ia and b > a. So E=h eV = , which gives
2
the correct choice is (b).
2. Stopping potential is the negative potential needed hc
2 = (2)
to stop the fastest moving electrons. Kmax = eV0. eV
Thus 3 eV = eV0 V0 = 3 V. From Eqs. (1) and (2), we get
hc 1 eV
3. K1 = –W (1) 1
=
1 2 c 2m
hc 7. Speed of the electron in the nth orbit is
K2 = –W (2)
2 e2
v=
Putting = 2 0 hn
1 2 /2
in Eq. (1) and using Eq. (2),
2 hc m e2
K1 = –W Momentum p = mv =
2
2 0 hn
= 2 (K2 + W) – W h 2 h2 0
de Broglie wavelength = = n. Hence
p m e2
K1 = 2K2 + W
n, which is choice (c).
Hence K1 > 2K2, which is choice (c).
8. Magnitude of magnetic moment of an electron mov-
4. K X-rays are produced when electrons jump from ing in an orbit of radius r with angular frequency
n = 2 to n = 1 state and K X-rays are produced in is
a transition n = 3 to n = 1. Hence the energy of K charge area
M=
hc time period
rays is greater than that of K rays. Since E = ;
e r2 1 2
the wavelength K rays is less than that of K rays. = = e r2 T
T 2
Hence peak A represents K line and peak B repre-
sents K line. Magnitude of angular momentum is
L = m r2
5. From the conservation of linear momentum, the two
fragments will have equal and opposite momenta. M e
=
Now de Broglie wavelength = h/p. Hence L 2m
1 p eL
= 2 M=
2 p1 2m
nh
Since p1 = p2, 1
= 1. From Bohr’s postulate L = . Hence
2
2
eh
6. de Broglie wavelength of electrons is M= n
4 m
h h
1 = = (1) So M n, which is choice (c).
p 2 meV
28.20 Comprehensive Physics—JEE Advanced

9. The wavelengths in Brackett series are given by c


1 1 13. Energy of a photon is E = h . Now = . Hence
1
= RH Z2 2 ; n = 5, 6, 7, …
4 n2 E=
hc
or =
hc
where RH is the Rydberg constant. The shortest E
wavelength corresponds to n = . Thus Hence the correct choice is (a).
1 1 14. Lenses of equal diameters collect the same amount
= RH Z2 of lighter so that the intensity remains the same;
16
hence the photoelectric current also remains the
The shortest wavelength in Balmer series of hydro- same.
gen (Z = 1) is given by 15. Energy of photon h = m c 2. Therefore,
1 1 h h
= RH m= 2 = ( c= )
4 c c
16 h
Given = . Therefore Z2 = =4 Z = 2. So Momentum of photon = mc = . Hence the
the correct choice is (a). 4 correct choice is (d).
10. rn n2 or rn = k n2 where k is a constant. Hence 16. The wavelength of blue light is longer than that of
rn + 1 = k (n + 1)2 violet light and the wavelength of sodium light (yel-
rn = k n2 low light) is longer than that of blue light. Since no
rn – 1 = k (n – 1)2 photoelectric emission occurs for blue light, it fol-
lows that the wavelength threshold ( min) is less than
Given the wavelength of blue light and hence less than that
rn + 1 – rn = rn – 1 of yellow light. Hence the correct choice is (d).
k (n + 1) – k n = k (n – 1)2
2 2
17. Energy of photons of frequency ( ) = h . If the lamp
2
(n + 1) – n = (n – 1) 2 2 emits n photons per second, then the power of the
lamp is P = nh . Now, = c / . Therefore
which gives n = 4.
nhc
11. Current due to circulating electron is P =
e e e vn
I= = =
Tn 2 rn /vn 2 rn 1020 6.6 10 34
3 108
= = 30 W
660 10 9
18. h = h 0 + eV, where V is the stopping potential.
0 I 0 e vn
B= = (1) For frequency , we have
2 rn 2 rn2 eV0
h =h 0+ (i)
2
0 h2 n2 e2
Now rn = and vn = and for frequency / 2, we have
m e2 2 0 hn
h
1 = h 0 + eV0 (ii)
B . So the 2
5
correct choice is (d). n
From (i) and (ii) on eliminating V0, we get
2 2
n h 0 = 3 / 2. Hence the correct choice is (b).
12. r = 0
m e2 hc hc
19. h = h 0 + eV. Since = c / , we have =
nh 0
L= nh = 2 L + eV, where 0 is the threshold wavelength. For
2 wavelength , we have
0 (2 L) 2 hc hc
r= = + 2 eV0 (i)
m e2 0
1/ 2 and for wavelength 2 , we have
m e2
L= r1/2 hc hc eV0
4 0 = + (ii)
2 2
Thus L r1/2, which is choice (a) 0
Atomic Physics 28.21

Eliminating V0 from (i) and (ii) we get 0 =3 . c ch 1


Hence the correct choice is (d). 27. min = = . Thus V . Thus if V is
max eV min
20. For light of frequency 2 0, we have
increased, min decreased. Hence the correct choice
2 h 0 = h 0 + eV0 or h 0 = eV0 (i) is (d).
For light of frequency 3 0, we have 28. X-rays, being electromagnetic waves, always travel
3 h 0 = h 0 + eV or 2 h 0 = eV (ii) with the speed of light i.e., 3 10 8 ms –1 in vacuum.
From (i) and (ii) we get V = 2V0. Hence the correct It is only the energy of the incident electrons (and,
choice is (b). therefore, the energy of the emitted X-rays) that de-
pends on the accelerating p.d. The speed of X -rays
1
21. mv 2max = h ( – 0). For light of frequency has nothing to do with this p.d., it is a characteris-
2 tic only of the medium of propagation. Unlike the
= 2 0 we have case of visible light, the speed of X-rays changes
1 but little with a change in medium (the refractive
mv 20 = h (2 0 – 0) = h 0 (i)
2 index of different media for X-rays is very close to
For light of frequency = 5 0, we have unity)
1 29. V = 66 kV = 66,000 V. Now
mv 2 = h (5 0 – 0) = 4 h 0 (ii)
2 hc
min =
Dividing (ii) by (i) we get v 2 = 4v20 or v = 2v0. eV
34
Hence the correct choice is (b). 6.6 10 3 108
=
hc 1.6 10 19 66, 000
22. W0 = h 0 = . For a metal of work function
0 = 1.87 10 –11 m = 0.0187 nm
W0 / 2, the threshold wavelength is given by Since wavelength 0.01 nm is less than min, it will
W0 hc be absent from the continuous spectrum. Hence the
=
2 correct choice is (d).
Thus = 0. Hence the correct choice is (c). hc 1
30. min = , i.e. min . Hence the correct choice
eV c eV V
23. Now max = . Since = , we have is (c).
h
eV
c ch 31. max = . Thus max V. Hence the correct
min = = h
max eV choice is (a).
3 108 6.6 10 34
32. We use Bragg’s equation 2 d sin = n . Here order
= 19
1.6 10 104 n = 1, = 30° and d = 0.6 nm. Therefore,
1 10 –10 m = 1 Å =2 0.6 sin 30° = 0.6 nm
Hence the correct choice is (a). 33. Here order n = 2 and = 45°. From Bragg’s equa-
34 tion, we have
24. E = h =
hc
= 6.6 10 3 108
2 d sin 45° = 2
10 10
or d = 2
= 1.98 10 –15 J 2 10 –15 J
Hence the correct choice is (a). 34. According to Moseley’s law = a (Z – b). Hence
25. The correct choice is (d) because the emission (Z – b)2. For K line, b = 1. Hence the correct
of characteristic X-rays starts only when the choice is (b).
incident electrons are accelerated beyond a certain 35. From Moseley’s law, we have
critical value. 2
41 41 1
26. The maximum frequency is given by = = 4. Hence
eV 21 21 1
max = , i.e. max V
h 41
21 = =
Hence the correct choice is (b). 4 4
28.22 Comprehensive Physics—JEE Advanced

36. The correct choice is (b).


1 m2 2m
37. Due to collision, the energy of the scattered photon = = = 2
2 m1 m
will be less than that of the incident photon as some
hc Hence the correct choice is (d).
energy is lost in the collision. Now E = h = . 1
43. We have seen above that . Hence
Thus if E decreases decreases and increases. T
Hence < and > . T1 927 273
38. To produce characteristic X-rays, the energy of the = = =2
T2 27 273
incident electrons must be greater than the binding
energy of the innermost electron. Thus the acceler- = 2 . Hence the correct choice is (c).
ating voltage must be greater than 40 kV, then the
1
accelerated electrons will have energy greater than 44. In terms of kinetic energy K = mv2, de Broglie
40 keV. Hence the correct choice is (b). wavelength is 2
hc h
=
39. min = . Hence the correct choice is (b). 2m K
eV
40. If V is increased, the energy of X-rays E = eV 1
Thus . If K is increased by a factor of 4,
also increases. But E = h . Hence frequency is K
increased. is decreased by a factor of 2. Hence the correct
choice is (b).
41. The wavelength associated with a particle of
charge q, mass m and accelerated through a 45. K = 180 eV = 180 1.6 10 –19 J = 2.88 10–17 J.
potential difference V is given by h
h Now =
= 2m K
2 m qV
34
6.6 10
h2 =
or V = 31 17 1/ 2
2mq 2 2 9 10 2.88 10

h2 6.6 10 34
for proton: V = 2 = = 0.9 10 –10 m
2 mp qp 7.2 10 24

h 2
46. -particle is an electron. Since electron has the
For -particle :V =
2m q 2 smallest mass, its de Broglie wavelength is the lon-
gest as the velocity of all particles is the same.
V mp qp 1 1 1
= = = 47. The speed of photon = c = 3 10 8 ms –1. Wavelength
V m q 4 2 8 of photon is, say, .
( m = 4 mp and q = 2qp) h
Wavelength of electron = = . Now
Thus V = V/8. Hence the correct choice is (a). mv
42. The de Broglie wavelength of a particle of mass m 1
m v2
moving with a speed v is given by, KE of electron 2 1 m v2
= =
h KE of photon h 2 hc
=
mv m v2 h h
= =
Kinetic energy K =
1
mv 2 or m v 2 = 2 K or m2v 2 2hc mv mv
2
v 1
2 K m . Therefore = = ( v = c / 2).
= 2 Km or mv = 2c 4
h 4
= 48. For Lyman series, we have l
= . Hence
2K m s 3
4
Since K is the same for both particles, we have l= 911.6 = 1215 Å.
3
Atomic Physics 28.23

49. The maximum number of electrons that can be


K 32
accommodated in orbits with n = 3 is r3 = = 3K
3
2 12 + 2 22 + 2 23 = 28.
r3 3K
50. The correct choice is (d). = = 3.
r1 K
51. For an atom of atomic number Z, the radius of the Hence the correct choice is (c).
nth orbit is given by [see Eq. (4)]
55. The kinetic and potential energies of an electron in
K n2 the nth excited state are given by
rn = (i)
Z
0h
2 1 e2
where K = is a constant. For the ground KE = (i)
m e2 8 0 rn
state of hydrogen (Z = 1), n = 1 so that 1 e2
and PE = – (ii)
r1 = K 4 0 rn
Let n be the energy state of Be+++ for which the Adding (i) and (ii) we get the total energy E which
orbital radius is r1. Putting Z = 4 and rn = r1 = K in is
Eq. (i) we get 1 e2
n2 E = KE + PE = – (iii)
K=K or n 2 = 4 or n = 2 8 0 rn
4
Hence the correct choice is (b). Notice from (i) and (iii) that E = – KE. Given
E = – 3.4 eV. Hence KE = – E = – (– 3.4) =
52. The energy in the nth state is given by [see Eq. + 3.4 eV. Thus the correct choice is (b).
(5)] PE
56. E= or PE = 2E
me 2 Z2 C Z2 2
En = – = = 2 – 3.4 = – 6.8 eV. Hence the correct choice is
8 0 h2 n2 n2
(c).
me2
where C=– is a constant. For the ground 57. n = 3. Therefore,
8 0 h2
1 1 1
state n = 1 of hydrogen and for n = 2 state of Be+++, = RH
we have 1 4 32
C 4 2 1 1 5 RH
E1 = C and E 2 = = 4C = RH
2 4 9 36
2
E2 4C 36 16
= = 4. or = . We have seen above that = .
E1 C 1 2
5 RH 3 RH
Hence the correct choice is (d). Hence
2
CZ 16 5 RH 20
53. En = 2
. For n = 1 state of hydrogen, we have 2
= =
n 1 3 RH 36 27
E1 = C and for the nth state of Li++, we have 20 20
or 2 = 1 = .
2 27 27
C 3 9C
En = = 2 . For E1 = En, we require
n2 n 58. Now Z 2. For doubly ionized lithium Z = 3.
9C Hence the correct choice is (c).
C = 2 which gives n = 3. Hence the correct
n 59. Energy difference E Z2. For a singly ionized
helium atom Z = 2. Hence E = 10.2 (2)2 =
choice is (c).
40.8 eV.
K n2
54. Now rn = . Therefore, for hydrogen (n = 1 60. Ionization energy E Z2. For helium Z = 2. Hence
Z E for helium = 13.6 (2)2 = 54.4 eV, which is
state), we have choice (c).
r1 = K ( Z = 1) 61. When an electron in the ground state receives
and for Li++ (n = 3 state) we have ( Z = 3) 12.1 eV of energy, it jumps to a level where its
28.24 Comprehensive Physics—JEE Advanced

energy = 13.6 – 12.1 = 1.5 eV. This corresponds to E h


the third excited state corresponding to n = 3. It can 69. We know that c = . Now = and = .
h p
have three transitions, namely from n = 3 to n = 2, Therefore,
from n = 2 to n = 1 and from n = 3 to n = 1. Hence E h E
three spectral lines will be emitted. c= = =
h p p
1
62. Orbital speed v0 . Therefore, n = 2. Now Hence the correct choice is (b).
n
rn n 2. Hence the radius of the new orbit 70. h = eV + W0 or eV = h – W0 = h – h 0. Now
2
= (2) r = 4r. = c / and 0 = c / 0. Thus, for wavelength ,
we have
63. Energy state – 3.4 eV corresponds to a level n 1 1
given by – 13.6 eV / n 2 = – 3.4 eV which gives eV = h c (i)
1 0
n = 2. Now, orbital speed v0 . Hence the and for wavelength = 2 , we have ( V = V/3)
n
orbital speed in the excited state is v/2. eV 1 1
= hc
64. The total energy of an electron bound to an atom 3 2 0
is negative and is the sum of its P.E. and K.E. The 1 1
magnitude of the P.E. is twice that of the K.E. (as or eV = 3 hc (ii)
per the Bohr model) but since the P.E. is – ve, we 2 0

have From (i) and (ii) we have


Total Energy = (– 2K) + (K ) = – K 1 1 1 1
(K = Kinetic energy). Hence the ratio of =3
0 2 0
total energy and kinetic energy = – 1 : 1 = – 1.
which gives = 4 . Hence the correct choice is
65. The longest wavelengths in the two series are given 0
(b).
by
1 1 1 3 71. When an electron falls from energy level E2 to en-
= RH = RH
12
2 2
4 ergy level E1, the frequency of the emitted radia-
L
tion is given by
1 1 1 5
and = RH 2 2
= RH h = E2 – E1
B 2 3 36 hc
or = E2 – E1
B 3 36 27
= or L : B = 5 : 27
L 4 5 5 1 1
h or =(E2 – E1) = k (E2 – E1)
66. Since the momentum of photon is , from the law hc
of conservation of momentum, the recoil speed v of where k = 1 / h c. For energy levels A, B and C, we
an atom of mass m is given by have
1
h h = k (EC – EB) (i)
mv = or v = 1
m
1
67. The maximum number of electrons allowed in = k (EB – EA) (ii)
an orbit is 2 n2. Hence the number of possible 2
elements is 1
and = k (EC – EA) (iii)
2 (12 + 22 + 32 + 42) = 60 3

68. The stopping potential depends on the frequency Adding (i), (ii) and (iii), we get
(or wavelength) of the incident electromagnetic 1 1 1
wave and is independent of the distance of = k {(EC – EB) + (EB – EA)
1 2 3
the source from photocell. Hence the stopping + (EC – EA)}
potential will still be 0.6 V. However, the saturation 2
= k (2EC – 2EA) =
current varies as 1/r 2, where r is the distance of the 3
source from the photocell. Since r is increased by a
1 1 1 1 2
factor of 3, the saturation current will decrease by or = or 3 =
a factor of (3)2 = 9, i.e. it will be 18 mA/ 9 = 2 mA 1 2 3 1 2
at r = 0.6 m. Hence the correct choice is (d). Hence the correct choice is (b).
Atomic Physics 28.25

72. Peaks A and B represent characteristic X-rays; 3


they are different for different emitters. Hence the T1 n1
For the two orbits = . Given T1 =
correct choice is (c). T2 n2
73. For wavelength we have 8T2. Hence n1/n2 = 2. Hence the correct choice is
1 (d).
m v2 = h – W0 79. For a given accelerating voltage, the wavelength
2
of the continuous X-rays will vary from a mini-
hc hc hc mum value min min is greater
0
= (i)
0 0 than zero. Hence the correct choice is (b).
For wavelength , we have hc 6.626 10 34 3 108
80. max = =
1 2 hc 0 W0 4.0 1.6 10 19
mv = (ii)
2 0 = 3.10 10–7 m = 310 nm
From (i) and (ii) we get 81. Let v1 and v2 be the respective speeds of the
two particles. The law of conservation of linear
v 2
0
= momentum gives
v2 0 m2 v2
m1v1 + m2v2 = 0 or = 1.0
Now = / 4. Hence m1 v1
v 2
4 3 /4 Since de Broglie wavelength = h/(m v ), we will
0
= have
v 2
3 0 m v
1
= 2 2 = 1.0
v 2
4 4 2 m1 v1
Therefore, is greater than . Hence v > v .
v 2
3 3 82. The energy levels of the hypothetical particle of
74. Energy required to remove one electron is E1 = double the mass of the electron but having the same
24.6 eV. The energy required to remove the second charge as the electron are given by
electron is 2R hc
En = – H2 ( Z = 2)
1 1 n
E2 = Z2 RHhc 2
1 ( )2 For the longest wavelength photon emission leav-
= (2)2 13.6 eV = 54.4 eV
transition from n = 3 state to n = 2 state. Hence
Total energy required = E1 + E2
1 1 5
= 24.6 + 54.4 = 79.0 eV. | E| = 2 RHhc 2 2
= 2 RHhc
2 3 36
1 1 5
75. E = Z RHhc
2
= (3) 2
13.6 eV = 122.4 eV = RHhc
12 ( )2 18
The wavelength corresponding to this energy is
34
hc 6.63 10 3 108 hc 18
76. E= = 9 = 59 103 eV = = , which is choice (c).
0.021 10 E 5RH
= 59 keV. 83. The energy of incident electrons is
77. The stopping potential in volts = kinetic energy E = 80 keV = 80 103 eV
of the emitted photoelectrons in eV. Hence the = 80 103 1.6 10–19 J
correct choice is (b). = 1.28 10–14 J
2 r The minimum wavelength of the continuous X-ray
78. T = and m v r = nh/2 . Therefore,
v spectrum is
2 r 4 2m 2 hc 6.6 10 34 3 108
T= = r min = =
n h / (2 m r ) nh E 1.28 10 14
= 0.155 10–10 m = 0.155 Å
h2 0 2 4h 3 2
0 3
Also r= 2
n , T= n Since the energy of the incident electrons is more
me me 4 than that of the K shell electrons in tungsten, the
28.26 Comprehensive Physics—JEE Advanced

characteristic X-ray spectrum of tungsten will ap- (because E = h and c = ). Since the photon is
pear as peaks on the background of the continuous
X-ray spectrum. Hence the correct choice is (d). (– p).
2E
84. E1 = h ( 1 – 0) and E2 = h( 2 – 0). Dividing them, Momentum transferred = p – (– p) = 2p = .
we get Hence the correct choice is (b). c
E2 2 0 hc
= 88. Given Kmax = – W0 (i)
E1 1 0 1
Given E2 = nE1. Hence, we have
hc
2 0
and 2Kmax = – W0 (ii)
n= 2
1 0 Dividing (ii) by (i), we get
n hc
which gives 0 = 1 2
, which is choice (b). W0
(n 1) 2
2=
hc
85. Let m0 and e0 be the mass and charge of a proton W0
and m1 and e1 those of -particle. Then 1

h hc
0 = and which gives W0 = (2 2 – 1) which is choice
2m0 e0 V (a). 1 2
e
=
h 89. eV0 = h . Therefore, = V0. Hence the slope of
1 h
2m1 e1 V
e
versus V0 graph is which is choice (c).
1 m0 e0 h
Dividing, we get = (i) 1
0 m1 e1 90. E = mv 2 = h 0 – W0. Now E1 = 2 – 1 = 1 eV and
2
Now, -particle has twice the charge and 4 times E2 = 10 – 1 = 9 eV. Therefore E1/E2 = 1/9, i.e.
the mass of a proton, i.e. m1 = 4 m0 and e1 = 2 e0.
1
Using these in Eq. (i), we get mv 2
2 1 = 1
0 1 9
1 = , which is choice (c). mv22
2 2 2
hc v1 1
86. = is the de Broglie wavelength. The energy or . Hence the correct choice is (d).
mv v2 3
of a photon of this wavelength is hc
hc hc 91. h = mc 2 or = mc2 ( c = ). Hence
E= h = = = mvc (i)
h / mv h
m= which is choice (b).
Kinetic energy of the particle is c
1 nh
E = mv2 (ii) 92. For nth Bohr orbit, mvr = . The de-Broglie
2 wavelength is 2
From (i) and (ii), we have h
1 =
mv 2 mv
E 2 v 2.25 108
= nh
E m vc 2c 2 3 108 But mv = . Therefore,
2 r
2.25 3 2 r 2 r
=h which is choice (a).
6 8 nh n
Hence the correct choice is (b). h
93. Given mv = . Hence the correct choice is (a).
87. The momentum of a photon of wavelength is
h h E hc
p= 94. Given – W0 = K1 (i)
c 1
Atomic Physics 28.27

hc 100. The energy of a photon of wavelength is given


and – W0 = K2 (ii) by
2 hc
Eliminate hc from (i) and (ii). The correct choice E=
is (a). The energies of radiations of wavelengths 1 and 2
95. h = E + W0 = 0.5 eV + W0 (i) are
When the energy of the incident photon is hc hc
increased by 20%, we have E1 = and E2 = respectively
1 2
6 h = E + W = 0.8 eV + W (ii)
0 0
5 1 1
E1 – E2 = hc
Subtracting (ii) from (i), we get h = 1.5 eV. Hence 1 2
W0 = h – 0.5 eV = 1.5 eV – 0.5 eV = 1.0 eV. Thus
The value of h is given by choice (c)
the correct choice is (b).
h 101. For a hydrogen like element, we have
96. Momentum of photon is p = . Momentum of an
h
electron moving with velocity v is mv. Given = 1 1 1
= Z 2R H
h n12 n22
mv or v = , which is choice (a).
m
97. We know that rn n2 or n rn . If r1 and r2 are 1 1 1 3 Z 2 RH
For K -line: = Z 2R H
12 22 4
then
1 1 1 8 Z 2 RH
r2 2 .12 For K -line: = Z2 RH =
n= = =2 12 32 9
r1 0 . 53
Dividing, we have
Hence the correct choice is (b).
3 9 27
98. = =
potential (V0) is the same for the two radiations. 4 8 32
We know that eV0 = Emax and Emax = h – W0. 27 27
Since V0 is the same, Emax and hence is the same = = 0.32 Å = 0.27 Å.
32 32
for radiations a and b. Hence a = b. Since the
The correct choice is (b).
saturation current is greater for radiation b than for
radiation a, the intensity Ib is greater than Ia. Hence 102. Total energy = KE + PE = K + U
the correct choice is (b). In the region 0 x 1; U1 = E. Therefore, kinetic
99. The energy required to excite a hydrogen atom energy is
from state (n = 1 state) to the n = 2 state is 10.2 K1 = total energy – U1 = nE – E = (n – 1) E
eV. So when a photon of energy 10.2 eV under- h h
goes an inelastic collision with a hydrogen atom in 1 = (1)
2mK1 2 m (n 1) E
the ground state, the electron of the hydrogen atom
jumps from n = 1 state to n = 2 state. The elec- In the region x > 1; U2 = 0. Therefore, kinetic
tron spends a time between 10–10 s to 10–8 s in the energy is
excited state before falling back to the ground K2 = nE – 0 = nE
state.In this process, a photon of energy 10.2 eV is
h h
released. After a few microseconds, when a another 2 = (2)
photon of energy 14.6 eV collides with the same 2mK 2 2 mnE
Dividing (1) by (2), we get
state. Now, the ionization energy of hydrogen atom
1 n
is 13.6 eV. The part 13.6 eV of the energy of the = . So the correct choice is (b).
incident photon is used up in ionizing the atom, 2 (n 1)
i.e. in knocking the electron from the atom and 103. From Moseley’s law, we have
remaining energy = 14.6 – 13.6 = 1.0 eV is used up 2
41 41 1
in imparting kinetic energy to the released electron. = = 4. Hence
Hence the correct choice is (b). 21 21 1
28.28 Comprehensive Physics—JEE Advanced

107. For photoelectric emission, the wavelength of the


41
21 = = incident radiation must be less than the cut-off
4 4
wavelength of the metal given by 0 = hc/ 0. For
104. de-Broglie wavelength is metals p, q and r, the cut-off wavelengths are
h hc 1240 eV nm
= (1)
2mE p = = = 620 nm
p 2.0 eV
where E is the kinetic energy of the electrons. The
out-off wavelength is 1240
q = = 496 nm
hc 2.5
0 =
E 1240
r = = 413.3 nm
h2 3.0
From Eq. (1) E = 2
. Hence Hence metal plate p emits photoelectrons for all the
2m
three given radiations, metal plate q emits photo-
2
2mc electrons for radiation of wavelengths 450 nm and
0 =
h 350 nm and metal plate r emits photoelectrons only
for wavelength 350 nm. Therefore, photoelectric
1 1 1
105. = RH current is maximum for metal p and minimum for
n22 n12 r, i.e. Ip > Iq > Ir. So the correct choice is (a).
The largest wavelength in the ultraviolet region of
the hydrogen spectrum corresponds to the transi- 1 1 1
108. = RH Z 2
tion n1 = 2 to n2 = 1 (Lyman series). Thus n12 n22
1 1 3RH For singly ionized helium atom, Z = 2. For hydro-
= RH 1 = gen atom Z = 1.
122 nm 4 4
For Balmer series n1 = 2.
4
which gives RH =
3 122 nm
The smallest wavelength in the infrared region of 1 1 1 5R
= RH (1)2 =
the hydrogen spectrum corresponds to n1 = and 6561 22 32 36
n2 = 3 (Paschen series). Therefore 36
1 1 1 R RH =
= RH = H 5 6561
32 9 For second spectral line of helium,
9 9 3 122nm 1 1 1 3RH
= = 823 nm = RH (2)2 2 2 =
RH 4 2 4 4
hc 3 36
106. min = , which is independent. Hence choice (a) =
eV 4 5 6561
is wrong. = 1215 Å

II

Multiple Choice Questions with One or More Choices Correct


1. The threshold frequency for photoelectric emission (a) 50 watt infrared lamp
from a material is 4.5 1014 Hz. Photoelectrons will (b) 100 watt red neon lamp
be emitted when this material is illuminated with (c) 60 watt sodium lamp
monochromatic light from a (d) 5 watt ultraviolet lamp
Atomic Physics 28.29

2. When monochromatic light from a bulb falls on a the cut-off voltage and the saturation current are
photosensitive surface, the number of photoelec- respectively 0 6 volt and 18 0 mA. If the same
trons emitted per second is n and their maximum source is placed 0 6 m away from the photoelectric
kinetic energy is Kmax. If the distance of the lamp cell, then
from the surface is halved, then (a) the stopping potential will be 0 2 volt
(a) n is doubled (b) the stopping potential will be 0 6 volt
(b) n becomes 4 times (c) the saturation current will be 6 0 mA
(c) Kmax is doubled (d) the saturation current will be 2 0 mA.
(d) Kmax remains unchanged 8. When a point light source, of power W emitting
3. The maximum kinetic energy of photoelectrons in monochromatic light of wavelength is kept at a
a photocell depends upon distance a from a photosensitive surface of work
(a) the frequency of the incident radiation function , and area S, we will have
(b) the work function of the photosensitive (a) number of photons striking the surface per
material used in the cell W S
unit time as
(c) the intensity of the incident radiation 4 h c a2
(d) all the above parameters. (b) the maximum energy of the emitted photo-
4. When ultraviolet light is incident on a photocell, 1
electrons as (hc )
its stopping potential is V0 and the maximum
kinetic energy of the photoelectrons is Kmax. When (c) the stopping potential needed to stop the
X-rays are incident on the same cell, then most energetic emitted photoelectrons as
(a) V0 will increase e
(hc ).
(b) K max will increase
(c) V0 will decrease (d) photoemission occurs only if lies in the
(d) K max will decrease range 0 h c/
5. The work function of metal A is greater than that 9. Which of the following statements are correct about
for metal B. The two metals are illuminated with photons?
appropriate radiation of frequency so as to cause (a) The rest mass of a photon is zero
photoelectric emission in both metals. If 0 is (b) The energy of a photon of frequency is
the threshold frequency and Kmax, the maximum h
kinetic energy of photoelectrons, then (c) The momentum of a photon of frequency
(a) 0 for metal A is greater than that for metal h
is
B. c
(b) 0 for metal A is less than that for metal B. (d) Photons do not exert any pressure on a
(c) Kmax for metal A is greater than that for metal surface on which they are incident.
B. 10. Figure 28.14 shows the stopping potential V0 versus
(d) Kmax for metal A is less than that for metal frequency for photoelectric emission from two
B. metals A and B. Choose the correct statement(s)
from the following.
6. X-rays are used to cause photoelectric emission
from sodium and copper. Then
(a) the stopping potential is more for copper than
for sodium.
(b) the stopping potential is less for copper than
for sodium.
(c) the threshold frequency is more for copper
than for sodium.
(d) the threshold frequency is less for copper than
for sodium. Fig. 28.14
7. When a monochromatic point source of light is (a) Work function of A is greater than that of B.
at a distance of 0 2 m from a photoelectric cell, (b) Work function of B is greater than that of A.
28.30 Comprehensive Physics—JEE Advanced

(c) Threshold frequency of A is greater than that (d) Its kinetic, potential and tatal energies
of B. decrease.
(d) Threshold frequency of B is greater than that IIT, 2000
of A 14. Figure 28.16 shows graphs between cut-off voltage
11. The intensity of X-rays from a Coolidge tube is 1
V0 and for three metals 1, 2 and 3, where is the
plotted against wavelength as shown in Fig. 28.15.
The minimum wavelength found is C and the wavelength of the incident radiation in nm.
wavelength of K line is k. If the accelerating If W1, W2 and W3 are the work functions of metals
voltage is increased 1, 2 and 3 respectively, then
(a) W1 : W2 : W3 = 1 : 2 : 4
(b) W1 : W2 : W3 = 4 : 2 : 1
(c) The graphs for metals 1, 2 and 3 are parallel
to each other and the slope of each graph is
hc/e, where h = Planck’s contant, c = speed
of light and e = charge of an electron.
(d) Ultraviolet light will eject photoelectrons from
metals 1 and 2 and not from metal 3.
Fig. 28.15
(a) C decreases
(b) K increases
(c) ( K – C ) increases
(d) C and K both decrease but ( K – C

Вам также может понравиться